You are on page 1of 1275

Cracker Book for Bank (IBPS | SBI | RRB PO | Clerk) Mains Exams

1 Adda247 Publications For any detail, mail us at


Publications@adda247.com
Cracker Book for Bank (IBPS | SBI | RRB PO | Clerk) Mains Exams

Chapter
Simplification, Approximation and
1 Number Series

BEST APPROACH TO SOLVE THE QUESTIONS

Simplification: Simplification simply means simplifying the complex


expressions. There are ways available always which are simpler, easier and
faster to calculate a particular complex expression. Questions related to
simplification can be solved in variety of ways. But in examination where
one needs to calculate as fast as possible, applying good simplification
techniques may serve a good purpose. The most interesting about numbers
is that they can be expressed in an innumerable number of ways and thus
comes the concept of breaking the expression in such a way that one
reaches the answer in less time.

Following are some examples citing the ways to simplifying an


equation or expression.

9 11 17 12 16
1. + + =? + + + 27× 33
2 3 6 5 10
27+22+17 24+16
⇒ =? + + (30-3)(30+3)
6 10
66
⇒ =? +4 + 900 − 9
6

⇒ ?=11+9-900-4= –884

2. ? × 65 ÷ 72 = 195 × 312 + 260 × 6


?×65
⇒ =65× 6(3× 52 + 4 × 1)
72

⇒ ? = 72× 6 × 160 = 432 × 160 = 69120

2 Adda247 Publications For any detail, mail us at


Publications@adda247.com
Cracker Book for Bank (IBPS | SBI | RRB PO | Clerk) Mains Exams

Approximation Questions based on approximation are usually easy to


solve but turn out to be a bit difficult when approximate values are not
clearly visible. In case where a value like 17.97 is given, it is easy to replace
120 101
it with 18. But cases where values like or are given, then
13 11
approximate values can be used by making appropriate adjustments to
other terms of the expression.

Let’s, for example, try to find out the approximate value of expression given
below:
66 50
+ + 41% of 97 =?
16 17

66 50
The value of is a bit more than 4 and that of is a bit less than 3. So we
16 17
66 50
may take value of and to be equal to 4 and 3 respectively.
16 17

We know 40% of 100 is equal to 40. As 41 near to and greater than 40 and
97 is near to and smaller than 100, we may take the approximate value of
41% 𝑜𝑓 97 to be equal to 40 itself.
This way, the approximate value of the above expression= 4+3+40=47

A few sample problems:

178 217
− + 33 × 41 =? −32% 𝑜𝑓 207
12 18
⇒ 15 + 12 + 33(40+1) = ? – 33% of 200
⇒ 27 + 1355 = ? – 66 ⇒ ? = 1450
19.95 68.12
+ − 12.5% 𝑜𝑓 127.9 =?
4 17
20 68 1
⇒ + − × 128 =?
4 17 8

⇒ 5 + 4 − 16 =? ⇒ ? = −7

3 Adda247 Publications For any detail, mail us at


Publications@adda247.com
Cracker Book for Bank (IBPS | SBI | RRB PO | Clerk) Mains Exams

Number Series When different numbers are arranged in any mathematical


and logical pattern, a number series is said to be formed. There may be
infinite such patterns. So it’s really difficult to find out one out of them. But
there is one element that gives us the idea of any pattern.

That element is the difference of the terms of a number series. By finding


the difference of consecutive terms of a series we can make out the pattern
on which the number series is based. Some patterns are clearly visible but
others can be found out by cracking the pattern of the differences.

A few examples follow:

1. 45, 44, 48, 39, 55, 30, 66

2. 3, 9, 23, 97, 479, 2881, 20159

3. 10, 12, 15, 22, 38, 70, 127

4 Adda247 Publications For any detail, mail us at


Publications@adda247.com
Cracker Book for Bank (IBPS | SBI | RRB PO | Clerk) Mains Exams

Practice Exercise Based on new Pattern

Directions (1-15): What will come in place of the question mark (?) in the
following questions ?

1. (4 × 4)3 ÷ (512 ÷ 8)4 × (32 × 8)4 = (2 × 2)?+4


(a) 8 (b) 12 (c) 6
(d) 14 (e) None of these

2
2. (2√392 − 21) + (√8 − 7) = (? )2
(a) 4 (b) –4 (c) 12
(d) 2 (e) 6

1 1 1 1
3. 1 + 1 − 1 =? +1
4 6 8 12
5 7 5
(a) (b) (c)
24 24 12
7
(d) (e) None of these
12

3
4. of 30% of 3420 = (? )2 × 2
19
(a) (81)2 (b) 7 (c) 9
(d) 81 (e) 49

5. 65% of √3136 × 5 =? +154


(a) 56 (b) 28 (c) 35
(d) 32 (e) None of these

1
6. 29% of 193 – 20% of 204.85 = (3375)3 − (? )2
(a) 1 (b) 4 (c) 16
(d) 64 (e) None of these

5 Adda247 Publications For any detail, mail us at


Publications@adda247.com
Cracker Book for Bank (IBPS | SBI | RRB PO | Clerk) Mains Exams

1 3 1 1 1
7. 3 ÷ 6 × 1 × 3 = (? )2
3 7 2 7
22 484 22
(a) (b) (c)
3 3 9
484
(d) (e) None of these
81

8. 60% of 68.05 × 15% of 8 × 1.02 = ?


(a) 49.97592 (b) 49.73482 (c) 54.77442
(d) 41.84822 (e) None of these

(112.6 ×114.4 )
9. = (? )2
117.5 ÷115 ×√11
(a)√11 (b) 11 (c) 121
(d) 1331 (e) 14641

1
10. √3481 ÷ 7 × (441)2 = 12.5% of 1200 + ?
(a) 3 (b) 9 (c) 27
(d) 81 (e) None of these

19 1
11. 3 % of 675 + 25 % of 184 = x
27 4
(a) 70.64 (b) 71.64 (c) 71.46
(d) 70.46 (e) None of these

2
12. [2.5 ÷ 0.1)3 ÷ (6.25)2 ] × [(1.25)2 ÷ (5)2 ] = (25)3+x
(a) 1 (b) 2/3 (c) 1/3
(d) 2 (e) None of these

161 1595 7 1
13. × ÷ = (x)3
377 253 13

(a) 343 (b) 216 (c) 64


(d) 125 (e) None of these
6 Adda247 Publications For any detail, mail us at
Publications@adda247.com
Cracker Book for Bank (IBPS | SBI | RRB PO | Clerk) Mains Exams

2 4 7 2145 169
14. of of of ÷√ =x
3 5 11 1288 529
(a) 1 (b) 0.5 (c) 2
(d) 4 (e) None of these

3
√8649 √15625
15. × 3 = x2
√961 √1728
(a) 0.5 (b) 1.5 (c) 2.5
(d) 3.5 (e) None of these

APPROXIMATION

Directions (1-20): What approximate value will come in place of (x or ?) in


the following questions ?

𝑥
1. 294.01 × − 19.99% 𝑜𝑓 119.99𝑥 = 254.9 ÷ 1.9
8.01
(a) 10 (b) 16 (c) 8
(d) 5 (e) 12

410
2. 55.2 × + 499.9 – 127.9 =? % of 5600
69
(a) 17.5 (b) 12.5 (c) 10
(d) 15 (e) 8

3. 630 × ? + 599.85 – 55% of 12000 = 37 ½ % of 9200


(a) 12 (b) 18 (c) 15
(d) 20 (e) 8

11999.87 1
4. + 54.9% of 1800 – 389.9 = 11 % of 9900
? 9

(a) 24 (b) 28 (c) 20


(d) 18 (e) 32

7 Adda247 Publications For any detail, mail us at


Publications@adda247.com
Cracker Book for Bank (IBPS | SBI | RRB PO | Clerk) Mains Exams
31 5 1
5. 8099.9 × + 2699.8 × – 1799.8 = 62 % of ?
27 3 2
(a) 22500 (b) 15200 (c) 16200
(d) 18200 (e) 19200

6. 119.89% of 2000.23 + (35.96)² – 59.49% of 2999.89 = (49.89)² – ?


(a) 559 (b) 569 (c) 589
(d) 549 (e) 539

7. (11.87)² + 12.493 × 15.89 – √13224.98 – (?)² = (14.96)²


(a) 9 (b) 2 (c) 8
(d) 5 (e) 10

359.93
8. = (8.89)3 – 14.5 × 39.89 + (1.95)2 – 34
?
(a) 8 (b) 2 (c) 3
(d) 9 (e) 5

9. √429.87 + 520.23 + √120.97 = (? )2 + √35.98


(a) 2 (b) 3 (c) 8
(d) 5 (e) 9

10. (23.89)² + √3598.97 – 171.93 + (?)² = (27.98)² + 4


(a) 18 (b) 24 (c) 28
(d) 22 (e) 26

11. (13.97)² – 39.87% of 239.97 + (29.87)² + 330.97 = (?)³


(a) 18 (b) 19 (c) 17
(d) 11 (e) 15

?+134.5
12. + 209.87 + (69.87)² – 1999.83 = (54.87)² + 99.85
24
(a) 235.5 (b) 245.5 (c) 225.5
(d) 215.5 (e) 205.5
8 Adda247 Publications For any detail, mail us at
Publications@adda247.com
Cracker Book for Bank (IBPS | SBI | RRB PO | Clerk) Mains Exams

13. ? × 12.97 + (24.97)² – (19.89)² + 363.83 = (27.93)²


(a) 10 (b) 20 (c) 18
(d) 12 (e) 15

593.89 1259.81
14. + 14.87 × 35.88 + = (25.89)2
? 17.93
(a) 12 (b) 15 (c) 13
(d) 9 (e) 11

15. 48.01% of 249.99 + 𝑥% 499.99 = (11.01)2 + (2.11)2


(a) 2 (b) 3 (c) 5
(d) 1 (e) 6

2
16. 49.11 × ÷ 14.09 + √𝑥 = 1025 ÷ 257
7
(a) 9 (b) 8 (c) 27
(d) 16 (e) 25

3
17. × (15.99) + 31.9% 𝑜𝑓 3199 + 294.9 = 𝑥 3
4
(a) 10 (b) 11 (c) 12
(d) 15 (e) 8

18. ? % of 6519.97 + (21.97)2 – 29.98% 𝑜𝑓 9139.93 = (25.91)2


(a) 25 (b) 15 (c) 30
(d) 45 (e) 40

577.93+?
19. + 61.87% of 1449.87= –√2024.87 + 13.89 % 𝑜𝑓 7099.97
(15.97)
(a) 222 (b) 282 (c) 262
(d) 200 (e) 308

20. 32.87% of ? – 17.87 × 44.86 – √399.81 = (24.98)2 + 29.89


(a) 4450 (b) 4500 (c) 4350
(d) 4600 (e) 4900

9 Adda247 Publications For any detail, mail us at


Publications@adda247.com
Cracker Book for Bank (IBPS | SBI | RRB PO | Clerk) Mains Exams

NUMBER SERIES

Wrong Series

Directions (1-15): Find the wrong number in the given series that does not
follow the pattern?

1. 480, 960, 320, 1280, 272 , 1536


(a) 960 (b) 272 (c) 1280
(d) 320 (e) 1536

2. 210, 197, 171, 135, 80, 15


(a) 197 (b) 15 (c) 80
(d) 171 (e) 135

3. 4, 3, 5, 14, 60, 528


(a) 60 (b) 4 (c) 5
(d) 14 (e) 528

4. 9, 63, 25, 216, 49, 512


(a) 25 (b) 216 (c) 63
(d) 512 (e) 49

5. 4498, 4888, 5336, 5846, 6422, 7070


(a) 5846 (b) 4498 (c) 5336
(d) 7070 (e) 6422

6. 122, 125.2, 131.6, 141.2, 152, 170


(a) 125.2 (b) 131.6 (c) 141.2
(d) 170 (e) 152
10 Adda247 Publications For any detail, mail us at
Publications@adda247.com
Cracker Book for Bank (IBPS | SBI | RRB PO | Clerk) Mains Exams

7. 3671, 2341, 4537, 1163, 6072, − 783


(a) 6072 (b) 2341 (c) − 783
(d) 3671 (e) 1163

8. 15, 91, 457, 1831, 5497, 10997


(a) 457 (b) 91 (c) 5497
(d) 15 (e) 10997

9. 10, 17, 45, 108, 220, 390


(a) 220 (b) 390 (c) 10
(d) 108 (e) 45

10. 10, 30, 90, 450, 3150, 34650


(a) 10 (b) 34650 (c) 3150
(d) 3150 (e) 30

11. 325, 546, 754, 936, 1078, 1170


(a) 936 (b) 546 (c) 325
(d) 1078 (e) 1170

12. 192, 202, 210, 216, 225, 243, 288


(a) 210 (b) 202 (c) 288
(d) 192 (e) 243

13. 3, 11, 49, 191, 569, 1135, 1134


(a) 1135 (b) 1134 (c) 3
(d) 49 (e) 11

14. 23, 30, 42, 63, 95, 140, 200


(a) 42 (b) 140 (c) 30
(d) 200 (e) 23

15. 8, 30, 60, 120, 180, 180, 90


(a) 30 (b) 120 (c) 90
(d) 8 (e) 60

11 Adda247 Publications For any detail, mail us at


Publications@adda247.com
Cracker Book for Bank (IBPS | SBI | RRB PO | Clerk) Mains Exams

MISSING SERIES

Directions (1-15): What should come in place of the question mark (?) in
the following series ?

1. 10, 15, 45, 270, 3240, ?


(a) 72250 (b) 77760 (c) 75260
(d) 85280 (e) 762560

2. 12, 26, 81, 328, 1645, ?


(a) 9981 (b) 8281 (c) 7321
(d) 9876 (e) 9978

3. 1024, 512, ?, 16, 1, 0.03125


(a) 64 (b) 128 (c) 256
(d) 216 (e) 324

4. 193, 97, 49, 25, ?, 7


(a) 15 (b) 12 (c) 17
(d) 13 (e) 11

5. 17, 24, 13, 26, 9, ?


(a) 30 (b) 32 (c) 29
(d) 28 (e) 26

6. 18, 6, 2, −2 , − 10, ?
(a) −36 (b) – 48 (c) − 40
(d) −32 (e) − 52

7. 56, 1144, 2104, 2944, 3672, ?


(a) 4556 (b) 4456 (c) 4256
(d) 4296 (e) 4552

12 Adda247 Publications For any detail, mail us at


Publications@adda247.com
Cracker Book for Bank (IBPS | SBI | RRB PO | Clerk) Mains Exams

8. 21, 1750, 2752, 3267, 3487, ?


(a) 3556 (b) 3456 (c) 3536
(d) 3544 (e) 3558

9. 9, 15, 23, 33, 45, ?


(a) 57 (b) 59 (c) 53
(d) 51 (e) 49

10. 141, 197, 269, 359, 469, ?


(a) 607 (b) 609 (c) 601
(d) 603 (e) 605

11. 25, 33, 46, 69, 112, ?


(a) 175 (b) 180 (c) 185
(d) 190 (e) 195

12. ? , 5, 12, 39, 160, 805


(a) 2 (b) 4 (c) 3
(d) 5 (e) 8

13. ?, 10, 20, 46, 96, 178


(a) 6 (b) 7 (c) 8
(d) 9 (e) 5

14. 2160, ? , 360, 180, 60, 30


(a) 1080 (b) 720 (c) 900
(d) 810 (e) 1440

15. 2, 35, 156, 477, ?, 959


(a) 858 (b) 564 (c) 958
(d) 727 (e) 676

13 Adda247 Publications For any detail, mail us at


Publications@adda247.com
Cracker Book for Bank (IBPS | SBI | RRB PO | Clerk) Mains Exams

Solutions

SIMPLIFICATION

1. (c); (4 × 4)3 ÷ (512 ÷ 8)4 × (32 × 8)4 = (2 × 2)?+4


3 4
(42 ) ×(44 )
or, (43 )4
= (4)?+4
4 ×416
6
or, = (4)?+4
412
or, 4 = 4?+4
10

or, ? = 6

2. (e); ?2 = 2√14 × 14 × 2 − 21 + 8 + 49 − 28√2


= 28√2 − 21 + 57 − 28√2 = 36 = 62
∴?=6

1 1 1 1
3. (a); 1 + + 1 + − 1 − =? +1
4 6 8 12
1 1 1 1
⇒? = 1 + + 1 + − 1 − − 1 −
4 6 8 12
1 1 1 1
= + − −
4 6 8 12
6+4−3−2 5
= =
24 24

30 3
4. (c); 3420 × × = (? )2 × 2
100 19
2
⇒ 162 = (? × 2 )
162
⇒ (? )2 = = 81
2
∴ ? = √81 = 9

65
5. (b); √3136 × × 5 = ? +154
100
65
⇒ 56 × × 5 =? +154
100
⇒ 182 = ? +154
⇒ ? = 182 − 154 = 28

14 Adda247 Publications For any detail, mail us at


Publications@adda247.com
Cracker Book for Bank (IBPS | SBI | RRB PO | Clerk) Mains Exams

6. (e); (55.97 – 40.97) = 15 – (?)2


(?)2 = 0
?=0

10 7 3 22 1
7. (d); × × × = (? )2
3 45 2 7
1 22
(? ) = 2
9
484
?=
81

8. (a); 40.83 × 1.02 ×1.2 = ?


? = 49.97592

117
9. (c); = (? )2
113
(? )2 = 14641
? = 121

10. (c); 59 × 3 = 150 + ?


? = 27

100 101
11. (c); % of 675 + of 184
27 100×4
675
+ 46.46
27
25 + 46.46 = 71.46

2
+𝑥
12. (c); [(25 )3 ÷ (6.25 )2 ] × ((1.25) ÷ 5 2 2)
= (25)
3

253 1252 1 2
+𝑥
[ × 1002 ] × [ × ] = (25) 3
254 1002 52
1 100×100 125×125 2
[ × × ] = (25)3+𝑥
25 100×100 5×5
2
+𝑥
25 = (25) 3
2 2
⇒ +𝑥 = 1 ⇒𝑥 = 1−
3 3
1
𝑥=
3

15 Adda247 Publications For any detail, mail us at


Publications@adda247.com
Cracker Book for Bank (IBPS | SBI | RRB PO | Clerk) Mains Exams

161 1595 13 1
13. (d); × × = (𝑥 )3
377 253 7
Explanation
161 = 23 × 7
377 = 29 × 13
1595 = 29 × 55
253 = 23 × 11
23×7 29×55 13 1
× × = (𝑥 )3
29×13 23×11 7
1
5 = (𝑥 ) ⇒ 𝑥 = 125
3

2 4 7 2145 529
14. (a); × × × ×√
3 5 11 1288 169
13 23
× =1
23 13

3
√8649 √15625
15. (c); × 3 = 𝑥2
√961 √1728
93 25
× = 6.25 = 𝑥 2
31 12
𝑥 = 2.5

APPROXIMATION

𝑥 1 255
1. (a); ≈ 294 × − × 120𝑥 =
8 5 2
≈ 36.75𝑥 − 24𝑥 = 127.5
≈ 12.75𝑥 = 127.5
≈ 𝑥 = 10

X
2. (b); × 5600 = 8 × 41 + 500 – 128
100
700
X=
56
X = 12.5

16 Adda247 Publications For any detail, mail us at


Publications@adda247.com
Cracker Book for Bank (IBPS | SBI | RRB PO | Clerk) Mains Exams
9200 55
3. (c); 630X + 600 = 37.5 × + × 12000
100 100
630X + 600 = 3450 + 6600
630X = 10050 – 600
9450
X=
360
X = 15

12000 55 100 1
4. (a); = + × 1800 = × × 9900 + 390
X 100 9 100
12000
= = 1490 – 990
X
12000
X=
500
X = 24

62.5X 31 5
5. (e); = 8100 × + 2700 × – 1800
100 27 3
5x
= 9300 + 4500 – 1800
8
12000 × 8
X=
5
X = 19200

120 59.5
6. (c); × 2000 + (36)2 – × 3000 = (50)2 – ?
100 100
2400 + 1296 – 1785 = 2500 – ?
2500 – 1911 = ?
? = 589

7. (b); (12)² + 12.5 × 16 – √13225 – (?)²


= (15)²
144 + 200 – 115 – (?)² = 225
(?)² = 229 – 225
?=2

17 Adda247 Publications For any detail, mail us at


Publications@adda247.com
Cracker Book for Bank (IBPS | SBI | RRB PO | Clerk) Mains Exams
360
8. (e); = (9)3 – 14.5 × 40 + (2)2 – 81
?
360
= 729 – 580 + 4 – 81
?
360
?=
72
?=5

9. (d); √430 + 520 + √121 = (? )2 + √36


√430 + 520 + 11 = (? )2 + 6
31 – 6 = (?)²
?=5

10. (a); (23.89)² + √3600 – 172 + (?)² = (28)² + 4


576 + 60 – 172 + (?)² = 784 + 4
464 + (?)² = 788
(?)² = 788 – 464
? = 18

11. (d); (14)² – 40% of 240 + (30)² + 331 = (?)³


40×240
196 – + 900 + 331 = (? )3
100

1000 + 331 = (?)³


? = 11

?+134.5
12. (c); + 210 + (70)2 – 2000 = (55)2 + 100
24
?+134.5
+ 3110 = 3125
24

? + 134.5 = 15 × 24
? = 360 – 134.5
? = 225.5

18 Adda247 Publications For any detail, mail us at


Publications@adda247.com
Cracker Book for Bank (IBPS | SBI | RRB PO | Clerk) Mains Exams

13. (e); ? × 13 + (25)² – (20)² + 364 = (28)²


? × 13 + 225 + 364 = 784
? × 13 = 784 – 589
195
?=
13

? = 15

594 1260
14. (d); + 15 × 36 + = (26)2
? 18
594
+ 540 + 70 = 676
?

594 = ? (676 – 610)


594
?=
66

?=9

48 𝑥
15. (d); × 250 + × 500 ≈ 112 + 22
100 100
120 + 5𝑥 ≈ 121 + 4
5𝑥 ≈ 5
𝑥≈1

2 1 1024
16. (a); 49 × × + √𝑥 ≈
7 14 256
1 + √𝑥 ≈ 4
√𝑥 ≈ 3
𝑥≈9

3 32
17. (b); × 16 + × 3200 + 295 ≈ 𝑥 3
4 100
12 + 1024 + 295 ≈ 𝑥 3
𝑥 3 ≈ 1331
𝑥 ≈ 11
19 Adda247 Publications For any detail, mail us at
Publications@adda247.com
Cracker Book for Bank (IBPS | SBI | RRB PO | Clerk) Mains Exams
? 30
18. (d); × 6520 + (22)2 – × 9140 = (26)2
100 100
?
× 6520 = 676 + 2741 – 484
100
2934×100
?=
6520
? = 45

578+? 62 14
19. (a); + × 1450 =– √2025 + × 7100
16 100 100
578+?
+ 899 =– 45 + 994
16
578+?
= 949 – 899
16
578 + ? = 50 × 16
? = 800 – 578
? = 222

33
20. (b); ×? = 625 + 30 + 810 + 20
100
1485×100
?=
33
? = 4500

WRONG SERIES

1. (b);

So wrong number is 272

2. (e);

So wrong number is 135

20 Adda247 Publications For any detail, mail us at


Publications@adda247.com
Cracker Book for Bank (IBPS | SBI | RRB PO | Clerk) Mains Exams

3. (a);

So wrong number is 60

4. (c);

So wrong number is 63

5. (d); Wrong number = 7070

5. (e); Wrong number = 152

6. (a); Wrong number = 6072


Pattern of series —

21 Adda247 Publications For any detail, mail us at


Publications@adda247.com
Cracker Book for Bank (IBPS | SBI | RRB PO | Clerk) Mains Exams

8. (e);

So wrong number is 10997

9. (b);

So wrong number is 390

10. (a);

So wrong number is 10

11. (d);

So wrong number is 1078

12. (b);

So wrong number is 202


22 Adda247 Publications For any detail, mail us at
Publications@adda247.com
Cracker Book for Bank (IBPS | SBI | RRB PO | Clerk) Mains Exams

13. (b);

So wrong number is 1134

14. (e);

So wrong number is 23

15. (a);

So wrong number is 30

MISSING SERIES

1. (b); Series is ×1.5, ×3, ×6, ×12, ×24


So, 3240 × 24 = 77760

2. (d); Series is ×2+2, ×3+3, ×4+4, ×5+5, ×6+6


So, 1645 × 6 + 6 = 9876

3. (b); Series is
÷2, ÷4, ÷8, ÷16, ÷32
So, ? = 512 ÷ 4
= 128

4. (d); −96, −48, − 24, − 12, −6


So, 25 – 12 = 13
23 Adda247 Publications For any detail, mail us at
Publications@adda247.com
Cracker Book for Bank (IBPS | SBI | RRB PO | Clerk) Mains Exams

5. (d); Series
+7, −11, +13, −17, +19
So, 9 + 19 = 28

6. (d); Pattern of series –


18 × 0.5 − 3 = 6
6×1−4=2
2 × 1.5 − 5 = −2
− 2 × 2 − 6 = − 10
? = − 10 × 2.5 −7 = − 32

7. (d);

8. (a); 21+ (123 + 1) = 1750


1750 + (103 + 2) = 2752
2752 + (83 + 3)= 3267
3267 + (63 + 4) = 3487
? = 3487 + (43 + 5) = 3556

9. (b);

24 Adda247 Publications For any detail, mail us at


Publications@adda247.com
Cracker Book for Bank (IBPS | SBI | RRB PO | Clerk) Mains Exams

10. (c); 141 + 7 × 8 = 197


197 + 8 × 9 = 269
269 + 9 × 10 = 359
359 + 10 × 11 = 469
? = 469 + 11 × 12 = 601

11. (e);

12. (b);

13. (c);

14. (a);

15. (c); (2 + 5) × 5 = 35
(35 + 4) × 4 = 156
(156 + 3) × 3 = 477
(477 + 2) × 2 = 958
(958 + 1) × 1 = 959

25 Adda247 Publications For any detail, mail us at


Publications@adda247.com
Cracker Book for Bank (IBPS | SBI | RRB PO | Clerk) Mains Exams

1 Adda247 Publications For any detail, mail us at


Publications@adda247.com
Cracker Book for Bank (IBPS | SBI | RRB PO | Clerk) Mains Exams

Chapter
Ratio Proportion and Mixture
2 & Alligation

BEST APPROACH TO SOLVE THE QUESTIONS

Alligation is a method used to solve problems concerned with weightage


average. Or, in other words, alligation is just a way of writing equations of
weightage average for the easy calculation. Problems involving mixtures or
any form of weightage average can be solved easily using the method of
alligation.

Questions related to replacement, addition or omission of components of


mixture may be easily solved using fractions. Be it in form of ratio or by
taking volume into consideration. A very useful formula for problems based
on mixtures is given below:
Final ratio= Initial ratio (fraction of component remained)
Final volume= initial volume (fraction of component remained)
A few sample problems explaining the concept of alligation are given below.

Example 1: ‘An alloy containing 50% gold is formed by melting two metals
2
A and B. Metal A contains 66 % gold while metal B contains 40% gold. Find
3

the ratio in which metal A and B are mixed to form the alloy.’

Solution: Let’s take the fractional equivalents of given percentages and


calculate using alligation.

2 Adda247 Publications For any detail, mail us at


Publications@adda247.com
Cracker Book for Bank (IBPS | SBI | RRB PO | Clerk) Mains Exams

Hence, required ratio is 3:5.

Example 2: ‘Average weight of boys in a class is 55kg while average age of


girls in the same class is 35. If the average of all the students in the class is
40 kg, find the ratio of boys and girls in the class.’

Solutions:

Required ratio is 1:3.

Example 3: ‘A can contains mixture of two liquids A and B in the ratio 7 : 5.


When 9 litres of mixture are drawn off and the can is filled with B, the ratio
of A and B becomes 7 : 9. How many litres of liquid A was contained by the
can initially?’

Solution: Since B is filled by replacing some amount of mixture, we should


take A into consideration
Initial ratio of A to the whole mixture = 7/12
7
Final ratio of A to the whole mixture =
16
7 7 12 3
Using formula, = ×𝑦 Or, 𝑦 = =
16 12 16 4

3 Adda247 Publications For any detail, mail us at


Publications@adda247.com
Cracker Book for Bank (IBPS | SBI | RRB PO | Clerk) Mains Exams
3 1th
This shows the mixture remained th of itself which means part was
4 4

drawn off.
1
ATQ, of total = 9
4

Or, total mixture = 9 × 4 = 36 litres


7
Initial amount of liquid A = × 36 = 21 litres
12

Practice Exercise Based on new Pattern

1. 150 kg of alloy containing copper and zinc in the ratio 3 : 2 mixed with
‘X’ kg of alloy containing copper and zinc in the ratio 2 : 3. If the
overall alloy should contain copper between 45% to 55%, what can be
minimum value of X?
(a) 450 kg (b) 100 kg (c) 50 kg
(d) Cannot be determined (e) None of these

2. Three different liquids which have 10% water, 20% water and x% of
water are mixed in the ratio of their quantity 2 : 3 : 4 respectively. If
12% of water is present in final mixture. Calculate value of x.
(a) 9% (b) 20% (c) 7%
(d) 15% (e) 17%

3. ‘x’ liters of a 30% alcohol solution is mixed with 40 liters of 60%


alcohol solution & a resultant of 50% alcohol solution is formed. Now
‘3x’ liters of y% alcohol solution is added to 30 liters of 50% alcohol
solution which resulted in 45% alcohol solution. The ratio of y : x is
(a) 17 : 6 (b) 16 : 15 (c) 7 : 15
(d) 14 : 5 (e) 17 : 8

4 Adda247 Publications For any detail, mail us at


Publications@adda247.com
Cracker Book for Bank (IBPS | SBI | RRB PO | Clerk) Mains Exams

4. There are three varities of sugar with their quantity in the ratio of 3 :
4 : 5. If 9 kg of first variety and 4 kg of second variety are added to
their respective quantity and x kg of 3rd type is removed from it, then
final ratio becomes 9 : 10 : 10. Find the sum of initial quantities of
these varieties.
(a) 120 kg (b) 96 kg (c) 84 kg
(d) 108 kg (e) None of these

5. A jeweler mixed gold and copper in 2 proportion. In type ‘A’ alloy, 6


gm gold is mixed with 5 gm copper and in type ‘B’ alloy, 5 gm gold in
mixed with 3 gm copper. If jeweler have 122 gm gold and 90 gm
copper, then find the weight of type ‘B’ alloy.
(a) 60 gm (b) 80 gm (c) 70 gm
(d) 100 gm (e) 90 gm

6. Two liquid containing (X and Y) mixture mixed, in the ratio 2 : 3. X


contain 40% sugar in water, while Y contain alcohol in water. After
adding 20 gram sugar in the final mixture ratio of Sugar, alcohol and
water becomes 8 : 5 : 16. Given that all ratio is calculate on weight.
Find the weight alcohol in mixture ‘Y’.
(a) 25 grams (b) 50 grams (c) 60 grams
(d) 80 grams (e) 75 grams

7. Two vessels A and B of equal capacities contain mixtures of milk and


water in the ratio 4:1 and 3:1, respectively. 25% of the mixture from A
is taken out and added to B. After mixing it thoroughly, an equal
amount is taken out from B and added back to A. The ratio of milk to
water in vessel A after the second operation is
(a) 79: 21 (b) 83: 17 (c) 77: 23
(d) 81: 19 (e) None of these

5 Adda247 Publications For any detail, mail us at


Publications@adda247.com
Cracker Book for Bank (IBPS | SBI | RRB PO | Clerk) Mains Exams

8. A 280 ml mix real juice pack contains mixture of Mango juice and
orange juice in the ratio of 9 : 5. If x ml juice taken out from pack and
20 ml of orange juice mixed in pack so the new ratio of Mango juice
and orange juice become 7 : 5, then find what quantity of mango juice
taken out from pack initially?
(a) 48 ml (b) 42 ml (c) 54 ml
(d) 60 ml (e) 64 ml

9. There are two vessels A and B which contains mixture of sulphuric


acid and nitrous oxide in the ratio of 7 : 2 and 3 : 4 respectively.
Mixture of both vessels are mixed to obtain a mixture of 390 ml, in
which quantity of nitrous oxide is 160 ml. Find ratio of quantity of
mixture in vessel A quantity of mixture in vessel B?
(a) 7 : 6 (b) 6 : 7 (c) 5 : 7
(d) 7 : 9 (e) 4 : 7

10. There is two vessels A and B. Vessel A & B contains the mixture of
milk, mango juice and water in the ratio of 8 : 5 : 3 & 6 : 5 : 2
respectively. If both vessels are mixed respectively in the ratio of 16 :
13 into another vessel C then the total quantities of mango juice in
vessel C will be 20 liters. Find the difference between the quantity of
milk and water in vessel C?
(a) 15 liters (b) 10 liters (c) 18 liters
(d) 25 liters (e) 20 liters

11. Two container P and Q have mixture. In P three liquid A, B and C are
mixed in a ratio of 2 : 1 : 1 and in Q two liquid B and C are mixed in
ratio of 3 : 2. Mixture of both container P and Q poured into third
container X which show the quantity ratio of A, B and C is 8 : 13 : 10.
Find the quantity in Q is what % less or more than the quantity in P.
(a) 3.25% (b) 12.5% (c) 6.25%
(d) 11% (e) 25%

6 Adda247 Publications For any detail, mail us at


Publications@adda247.com
Cracker Book for Bank (IBPS | SBI | RRB PO | Clerk) Mains Exams

12. In vessel A 80% is milk and rest is water. Vessel B also contain same
1
mixture in which milk is 62 % of total volume. 25ℓ of mixture taken
2
out from vessel A and poured in vessel B so that milk becomes 100%
more than water in vessel B. If new quantity of mixture in vessel B is
16% less than initial quantity of mixture in vessel A then find initial
quantity of milk in vessel A?
(a) 100l (b) 125l (c) 115l
(d) 105l (e) 112l

Directions (13-14): Three milkman P, Q and R have mixture of milk and


water in the quantity of (X + 24) liters, (X+54) liters and (X + 84) liters and
milk and water in the ratio of 2 : 3, 8 : 7 and 13 : 5 respectively.

13. If P sold 50 liters of his mixture and 8 liters of milk added in


remaining mixture, new ratio of milk and water becomes 6 : 7 . What
quantity of water should be added by Q and R, so new ratio of their
mixture becomes 5 : 7 & 5 : 4 respectively ?(1 marks)
(a) 56 liters& 40 liters (b) 54 liters& 42 liters
(c) 50 liters& 30 liters (d) 60 liters& 50 liters
(e) 64 liters& 30 liters

14. Q sold 40% of his mixture and R sold 50% of his mixture, remaining
mixture of both became equal. If all three charge Rs. 2 for one liter of
water and production cost of milk for P, Q and R in the ratio of 5 : 6 : 7
and total mixture, which all three milkman have cost Rs. 3644. Find
production cost of milk per liter for P, Q and R respectively? (1 marks)
(a) 5 Rs. 6 Rs & 7 Rs. (b) 25 Rs. 30 Rs & 36 Rs.
(c) 10 Rs. 12 Rs & 16 Rs. (d) 10 Rs. 12 Rs & 14 Rs.
(e)10 Rs. 14 Rs& 15s.

7 Adda247 Publications For any detail, mail us at


Publications@adda247.com
Cracker Book for Bank (IBPS | SBI | RRB PO | Clerk) Mains Exams

15. Vessel A & B contains mixture of orange juice and mango juice in the
ratio of 5 : 4 and 5 : 2 respectively. Some quantity of mixture taken out
from vessel A and B and mixed in another vessel C, if vessel C contains
mixture of orange juice and mango juice in the ratio of 5 : 3 and total
quantity of mixture in vessel C is 96 l. then find the difference between
mixture taken out from vessel A & B?
(a) 16 l (b) 12 l (c) 8 l
(d) 10 l (e) 18 l

16. Mixture of petrol and water in vessel A are in the ratio of 4:1 and in
vessel B are in the ratio of 5:3. If 20 l of mixture taken out from vessel
A and proud in vessel B so new ratio of petrol to water in vessel B
becomes 19:10, then find initially quantity of petrol in vessel A, if new
quantity of mixture in vessel B is 26 l more than initial quantity of
mixture in vessel A?
(a) 64l (b) 72l (c) 68l
(d) 56l (e) 48L

17. Two container P and Q have mixture of different liquids. In P three


liquid A, B and C is mixed in a ratio of 2 : 1 : 1 and in Q two liquid B
and C is mixed in ratio of 3 : 2. Mixture of both container P and Q
poured into third container X which show the quantity ratio of A, B
and C is 8 : 13 : 10. Find the quantity in Q is what % less or more than
the quantity in P.
(a) 3.25% (b) 12.5% (c) 6.25%
(d) 11% (e) 25%

18. Ratio of Vodka and Wine in vessel A is 5 : 3 and same mixture in vessel
B in the ratio of 3 : 2, 16ℓ of mixture from vessel A taken out and
poured in vessel B new ratio of Vodka to wine becomes 29 : 19. If new
quantity of mixture in vessel B is equal to initial quantity of mixture in
vessel A, then find quantity of Vodka after 16 ℓ of mixture has been
taken out from vessel A ?
(a) 50𝑙 (b) 48𝑙 (c) 54𝑙
(d) 80𝑙 (e) 84𝑙

8 Adda247 Publications For any detail, mail us at


Publications@adda247.com
Cracker Book for Bank (IBPS | SBI | RRB PO | Clerk) Mains Exams

19. Sameer and Veer entered into partnership business with the capital of
Rs. x and Rs. (x + 12000) respectively. After One year Divyaraj joined
them with capital of Rs. (x + 8000). At the end of 2 nd year Sameer and
Divyaraj withdraw their capital and Veer invested for one more year.
If Sameer, Divyaraj and Veer gets profit in the ratio of 8 : 6 : 21
respectively. Find sum of capital invested by all three ?
(a) 66000 Rs. (b) 54000 Rs. (c) 64000 Rs.
(d) 68000 Rs. (e) 70000 Rs.

20. Wheat of x Rs./kg is mixed with wheat of y Rs./kg in the ratio 2 : 3. On


2
selling the mixture at 49 Rs./kg there is a gain of 16 %. If the quantity
3
in which they were mixed is reversed and selling price remains same,
550
then gain percent becomes %. What is the volume of ‘x’.
19
(a) 25 Rs./kg (b) 30 Rs./kg (c) 28 Rs./kg
(d) 32 Rs./kg (e) 20 Rs./kg

21. Three vessels A, B and C of same capacity are all full. Vessel A and B
contain mixture of milk and water in ratios 4 : 1 and 7 : 3 while vessel
C contains only pure milk. The contents of all the vessels are poured
into a larger vessel D. After drawing 30 liters of the mixture from
vessel D, the final quantity of water in Vessel D is 40 liters. Find the
capacity of each vessel
(a) 80 (b) 85 (c) 95
(d) 70 (e) 90

22. Two container P and Q have mixture. In P three liquid A, B and C is


mixed in a ratio of 2 : 1 : 1 and in Q two liquid B and C is mixed in ratio
of 3 : 2. Mixture of both container P and Q poured into third container
X which show the quantity ratio of A, B and C is 8 : 13 : 10. Find the
quantity in Q is what % less or more than the quantity in P.
(a) 3.25% (b) 12.5% (c) 6.25%
(d) 11% (e) 25%

9 Adda247 Publications For any detail, mail us at


Publications@adda247.com
Cracker Book for Bank (IBPS | SBI | RRB PO | Clerk) Mains Exams

23. Vessel A contains (X + 24) 𝑙 of mixture of milk and water in the ratio of
7 : 8, while vessel B contains (X + 54) 𝑙 of same mixture of milk and
1
water in the ratio of 3 : 2. If 37 % of mixture from vessel A and 40%
2
of mixture from vessel B taken out and mixed in vessel C, then the
remaining mixture in vessel B is 15𝑙 more than that of in vessel A. find
quantity of milk in vessel C?
(a) 49 litre (b) 57 litre (c) 55 litre
(d) 53 litre (e) 51 litre

24. Three container (A, B and C) have capacity in the ratio of 6 : 8 : 7


respectively. These three container are pour with three type of liquid
X, Y and Z in the ratio 2 : 3 : 1 (in A), 1 : 2 : 5 (in B) and 3 : 1 : 3 (in C)
respectively. Equal quantity of mixture is taken out from these
containers and poured in container D. Quantity of liquid Y is 1/8 liter
more than quantity of liquid X in container D. Find out the quantity
taken out from each container.
(a) 24 (b) 35 (c) 21
(d) 20 (e) None of these

25. A shopkeeper has mixtures of nitrous oxide and water in two vessels
A and B. Vessel A and vessel B contains mixtures of nitrous oxide and
water in the ratio of 7 : 2 & 5 : 3 respectively. The shopkeeper has
taken out the mixtures from vessels A and B in the ratio of 9 : 8 and
mixed it in vessel C. If shopkeeper sold 68 gm of mixture from vessel C
on the cost price of nitrous oxide, which is Rs. 80 per gm, find profit of
shopkeeper?
2 2 2
(a) 39 % (b) 41 % (c) 45 %
3 3 3
2 2
(d) 37 % (e) 35 %
3 3

10 Adda247 Publications For any detail, mail us at


Publications@adda247.com
Cracker Book for Bank (IBPS | SBI | RRB PO | Clerk) Mains Exams

26. Amount invested in business respectively by Rana, Nikhil and Yash,


was in AP respectively, and Yash invested thrice amount as that of
Rana. Rana was manager and Nikhil was a programmer, and they both
get salary of 8% and 5% of total profit respectively. If Rana invested
for a month, Nikhil for 8 months and Yash invested for 4 months and
Nikhil got Rs.6000 more than both of them together, find their total
profit.
(a) Rs. 100000/- (b) Rs. 150000/- (c) Rs. 174000/-
(d) Rs. 180000/- (e) Rs. 20000/-

27. There are three vessels A, B & C having Milk to water ratio as 3 : 5, 1 :
3 and 7 : y. After adding mixture of vessel B to A, the milk to water
ratio becomes 1 : 2 in A . Now 60% of mixture is taken out from vessel
A and mixture of vessel C is mixed with remaining mixture of vessel A
which results in final Milk to Water ratio of 3 : 4. Find the value of y
given that volume of B is 20 ltrs and volume of C is (7 + y) ltrs.
(a) 1 (b) 4 (c) 5
(d) 3 (e) 2

28. There are three mixtures A, B and C. Mixture A contains 10% of water
and B contains 15% of water. If A and B are mixed together than
resultant mixture contains 12% water. while If B is mixed with C,
which contains 22% water so the resultant mixture contains of 18%
water. All these mixture are mixed in a jar and then 30 ml water is
also added. Find total volume of water preset in jar, if initially mixture
C contains 66 ml water.
(a) 144 ml (b) 216 ml (c) 184 ml
(d) 196 ml (e) 200 ml
11 Adda247 Publications For any detail, mail us at
Publications@adda247.com
Cracker Book for Bank (IBPS | SBI | RRB PO | Clerk) Mains Exams

29. There are two containers A and B filled with oil with different prices
and their volumes are 140 litres and 60 litres respectively. Equal
quantities are drawn from both A and B in such a manner that the oil
drawn from A is poured into B and the oil drawn from B is poured into
A. The price per litre becomes equal in both A and B. How much oil is
drawn from each of A and B:
(a) 40 litre (b) 80 litre (c) 30 litre
(d) 21 litre (e) 42 litre

30. A shopkeeper has two types Wheat i.e Type A & Type B and cost price
per kg of Type A wheat is 10 times of cost price per kg of type B
wheat. Shopkeeper cheated a costumer by saying that he will give him
200 kg of Type A wheat but shopkeeper gives 200 kg of mixture of
Type A & Type B wheat on 10% above the cost price per kg of Type A
7
wheat. If shopkeeper made a total profit of 71 % in this transaction,
8
then find ratio between quantity of Type B wheat to Type A wheat in
sold mixture?
(a) 2 : 5 (b) 2 : 3 (c) 1 : 3
(d) 2 : 7 (e) 1: 5

31. Two farmers A & B have mixture of urea & Potassium in the quantity
1
of 120 kg and 80 kg respectively. The mixture of farmer A has 37 %
2
potassium and the mixture of farmer B has 40% potassium. If farmer
A used 60% of his mixture & B used 50% of his mixture and both gives
their remaining mixture to farmer C, who have already an equal
quantity of urea & Potassium. If ratio of urea & Potassium in resulting
mixture of farmer C is 7 : 5, find total initial quantity of mixture farmer
C had?
(a) 24 kg (b) 36 kg (c) 32 kg
(d) 42 kg (e) 48 kg

12 Adda247 Publications For any detail, mail us at


Publications@adda247.com
Cracker Book for Bank (IBPS | SBI | RRB PO | Clerk) Mains Exams

32. Vessel A, B and C contains mixture of milk and water. Ratio of the
quantity of A, B and C is 25 : 20 : 18 respectively. In vessel A milk to
water ratio is 2 : 3 and in vessel B water to milk ratio is 2 : 3. If vessel
C contain milk to water ratio is 4 : 5 and all the mixture of 3 vessel
poured to another vessel D then in vessel D quantity of water exceeds
by 6 litres than the quantity of milk in that vessel. Find out the
quantity of mixture contained in vessel A, B and C.
(a) 50 Ltr., 40 Ltr., 36 Ltr. (b) 25 Ltr., 20 Ltr., 18 Ltr.
(c) 75 Ltr., 60 Ltr., 54 Ltr. (d) 100 Ltr., 80 Ltr., 72 Ltr.
(e) None of these

33. In a party a tank is filled with some quantities of juice. Three vessels
of different volume are used to serve the juice. Volume of the smallest
vessel is 25% less than volume of second largest vessel and the
1
volume of second largest vessel is 33 % less than volume of the
3
largest vessel. If 5 times the juice is served by largest vessel, 8 times
1
juice is served by second largest vessel and finally when 3 times
3
juice is served by the smallest vessel, the tank gets emptied
completely. Find the total quantity of juice served by second largest
vessel is what percent of volume of the tank?
4 4 4
(a) 42 % (b) 44 % (c) 48 %
9 9 9
4 4
(d) 40 % (e) 36 %
9 9

34. There are three fruit sellers Ramesh, Suresh and Vikash, all of them
have a mixture of two types of apple. Ramesh has (x+18) kg of apple,
Suresh has 12 kg of apple more than that of Ramesh while Vikash has
3 kg of apple more than that of Suresh. The two types of Apple are
mixed in the ratio of 5 : 3, 2 : 3 and 4 : 3 for them respectively. If
Ramesh sold 16 kg of mixed apple and added 12 kg of second type of
apple then the new ratio becomes 5 : 6 for him. Vikash added y kg of
second type of apple to his mixture and thus the new ratio of both
types for him is 1 : 1. Then find the value of y.
(a) 8 kg (b) 9 kg (c) 10 kg
(d) 6 kg (e) 7 kg
13 Adda247 Publications For any detail, mail us at
Publications@adda247.com
Cracker Book for Bank (IBPS | SBI | RRB PO | Clerk) Mains Exams

35. There are two vessels, vessel A and vessel B. Vessel A contains milk
and water in the ratio 5 : 4 and vessel B contains Y litre of wine. 18
litre of mixture from vessel A is taken out and poured in vessel B.
Now, 10 litre of mixture from vessel B is taken out and poured in
vessel A. If quantity of wine in vessel A is found to be 4 litre then, find
the initial quantity of wine in vessel B.
(a) 8 L (b) 9 L (c) 10 L
(d) 12 L (e) 6 L

Solutions

1. (c); If overall alloy contain copper as 45%


3 2x 45
(150 × + ) = (150 + x)
5 5 100
2x 9
(90 + )= (150 + x)
5 20
1800 + 8x = 1350 + 9x
x = 450 kg
If overall alloy contain copper as 55%
2x 55
(90 + )= (150 + x)
5 100
1800 + 8x = 1650 + 11x
3x = 150 ⇒ x = 50 kg
Minimum value of X is 50 kg

2. (c); Let the quantity of three liquids is 200a, 300a and 400a
10% of water in first type means 20a water
20% of water in second type means 60a water
x% of water in third type means 4xa water
∴ ATQ,
20a+60a+4xa 12
=
900a 100
28a
⇒ 4xa = 108a – 80a ⇒ x= =7
4a

14 Adda247 Publications For any detail, mail us at


Publications@adda247.com
Cracker Book for Bank (IBPS | SBI | RRB PO | Clerk) Mains Exams

3. (e); From statement I


30 60
x×100+40×100 1 (2400+30x) 1
(x+40)
= ⇒ =
2 100(x+40) 2

⇒ x = 20
Now from statement II
y 50
(3x×100+30×100) 45
=
3x+30 100

Here 3X = 3 × 20 = 60 litres
60y
( )+15 45 60y 81
100
⇒ = ⇒ = – 15
90 100 100 2

⇒ y = 42.5
∴ y ∶ x = 42.5 : 20 = 17 : 8

4. (d); Let the initial quantity is 3y, 4y & 5y of these varities


According to condition
3y + 9 ∶ 4y + 4 ∶ 5y – x = 9 ∶ 10 ∶ 10
From 1st 2 ratios
3y+9 9
⇒ =
4y+4 10
⇒ y = 9, Hence sum of initial quantities is (3 + 4 + 5) × 9
= 108 kg
NOTE: No need calculate Value of x

5. (b); Let gold and copper in Type A alloy be 6a and 5a


Let gold and copper in type B alloy 5b and 3b
⇒ 6a + 5b = 122 … (i)
5a + 3b = 90 … (ii)
Solving equation (i) and (ii) we get
a = 12 ⇒ b = 10
Weight of type ‘B’ alloy = (5 + 3) × 10
= 80 gm
15 Adda247 Publications For any detail, mail us at
Publications@adda247.com
Cracker Book for Bank (IBPS | SBI | RRB PO | Clerk) Mains Exams

6. (a); Let weight of alcohol in Y = a gram


Let weight of mixture X = 100x gram
So, sugar weight in X = 40x gram
Water weight in X = 60x gram
So, weight of Y = 150x gram
Weight of water in Y = (150x – a) gram
ATQ,
40x+20 : a : 150x–a+60x = 8 : 5 : 16… (i)
a 5
⇒ =
210x–a 16
⇒ a = 50x … (ii)
Put value of ‘a’ in eqn. (i)
40x+20 8 40x+20 8 1
= ⇒ = ⇒ x=
a 5 50x 5 2
1
weight of alcohol in Y = 50 × = 25 gram.
2

7. (a); Assume there is 20litres of the mixture in both the vessels.


In vessel A, milk = 16litres and water
= 4litres
If 25% mixture from A added to B,
milk in B = 15 + 4 = 19litres
water in B = 5 + 1 = 6litres
Ratio = 19: 6
Equal amount from vessel B to vessel A
19 79
milk in A = 12 + =
5 5
6 21
= water in A = 3 + =
5 5
Hence, the ratio is 79: 21

8. (c); Mango juice in mixture


9
= 280 × (9+5) = 180 ml
5
Orange juice in mixture = 280 × (9+5)
= 100 ml
ATQ—
9x
180–14 7
5x =
100–14+20 5

16 Adda247 Publications For any detail, mail us at


Publications@adda247.com
Cracker Book for Bank (IBPS | SBI | RRB PO | Clerk) Mains Exams

5(2520 – 9x) = 7(1400 – 5x + 280)


45x – 35x = 12600 – 11760
10x = 840 ⇒ x = 84 ml
Quantity of mango juice taken out from pack initially
9
= 84 × = 54ml
14

9. (b); Let total mixture of sulphuric acid and nitrous oxide in vessel A
and B be P mℓ and Q mℓ respectively
ATQ,
P + Q = 390 … (i)
2P 4Q
+ = 160
9 7
14P + 36Q = 10080 … (ii)
From 14 × (i) – (ii)
Q = 210 mℓ
P = 390 – 210 = 180 mℓ
180
Required ratio = =6:7
210

10 (c); Let quantity of mixture mixed in vessel C from vessel A & B be


16x and 13x respectively.
ATQ—
5 5
16x × + 13x × (6+5+2) = 20 liters
(8+5+3)
5x + 5x = 20 liters
x = 2 liter
Required difference =
8 6 3 2
[(16 × 2) × + (13 × 2) × ] – [(16 × 2) × + (13 × 2) × ]
16 13 16 13
= (16 + 12) – (6 + 4) = 18 liters

11. (c); Let in P, Quantity of A, B and C is 2x, x and x respectively


In Q → B and C is 3y and 2y
Now in final mixture → (X) A : B : C
2x : x+3y : x+2y = 8 : 13 : 10
Solving →
x = 4 unit
y = 3 unit
Quantity in P → (2 + 1 + 1) × 4 = 16 unit
Quantity in Q → (3 + 2) × 3 = 15unit
1
Required % = × 100 = 6.25%
16
17 Adda247 Publications For any detail, mail us at
Publications@adda247.com
Cracker Book for Bank (IBPS | SBI | RRB PO | Clerk) Mains Exams

12. (a); Let milk and water in vessel A


4a and a respectively
Let Milk and water in vessel B
5b and 3b respectively
ATQ—
4
5b+25×5 2
1 =
3b+25×5 1
5b+20 2
=
3b+5 1
6b – 5b = 20 – 10
b = 10
New quantity of mixture in vessel B
= (5 × 10 + 20) + (3 × 10 + 5) = 105ℓ
Initial quantity of mixture in vessel A
100
= 105 × = 125ℓ
84
Quantity of milk in vessel A
4
= 125 × = 100ℓ
5

13. (b); In 50 liters sold mixture of P


2
Milk = 50 × = 20liters
5
3
Water = 50 × = 30 liters
5
ATQ -
2(P+24)
5
−20+8 6 2P+48−60 6
3(P+24) = ⇒ =
−30 7 3P+72−150 7
5
7P – 42 = 9P = 234
2P = 234 – 42
192
P= ⇒ P = 96 liters
2
Mixture, which Q have
= (96 + 54) = 150 liter
Mixture which R have = (96 + 84)
= 180 liters
For Q –
8(96+54)
Milk = = 80 liters
15
(96+54)
Water = = 70 liters
15
18 Adda247 Publications For any detail, mail us at
Publications@adda247.com
Cracker Book for Bank (IBPS | SBI | RRB PO | Clerk) Mains Exams

Lets Q added x liters of water


80 5
=
70+x 7
5x = 560 – 350
5x = 210 ⇒ x = 42 liters
for R-
13(96+84)
Milk = = 130 liters
18
5(96+84)
Water = = 50 liters
18
Lets R added y liters of water
130 5
=
50+y 4
5y = 520 – 250
270
y= ⇒ y= 54 liters
5

14. (d); ATQ-


(100−40) (100−50)
(P + 54) × = (P + 84) ×
100 100
60P + 3240 = 50P + 4200
10P = 960 ⇒ P = 96 liters
Let production cost be Rs 5x, Rs 6x and Rs 7x
ATQ_
(48 × 5x + 72 × 2) + (80 × 6x + 70 × 2) + (130 × 7x + 50 × 2) =
3644
240x + 480x + 910x = 3644 – (144 + 140 + 100)
1630x = 3260
x = 3260 ⇒ x = 2 Rs
production cost for P = 5 ×2 = 10 Rs
production cost for Q = 6 × 2 = 12 Rs
production cost for R = 7 × 2 = 14 Rs
5
15. (b); Part of Orange juice in vessel A =
9
5
Part of Orange juice in vessel B =
7
5
Part of Orange juice in vessel C =
8
By allegation —

19 Adda247 Publications For any detail, mail us at


Publications@adda247.com
Cracker Book for Bank (IBPS | SBI | RRB PO | Clerk) Mains Exams

=9:7
(9–7)
Required difference = 96 × = 12ℓ
16

16. (b); Lets petrol and water in vessel A be 4x and x


Petrol and water in vessel B by 5y and 3y
ATQ—
4
5y+20×5 19 5y+16 19
1 = ⇒ =
3y+20× 10 3y+4 10
5
50y + 160 = 57y + 76
7y = 84 ⇒ y = 12
New mixture in vessel B
= (5 × 12 + 16) + (3 × 12 + 4)
= 76 + 40 = 116 ℓ
Initial quantity of vessel A
= (116 – 26)
= 90ℓ
4
Quantity of petrol in vessel A = 90 ×
5
= 72 ℓ

17. (c); Let in P, Quantity of A, B and C is 2x, x and x respectively


In Q → B and C is 3y and 2y
Now in final mixture → (X) A : B : C
2x : x+3y : x+2y = 8 : 13 : 10
Saving →
x = 4 unit ⇒ y = 3 unit
Quantity in P → (2 + 1 + 1) × 4 = 16 unit
Quantity in Q → (3 + 2) × 3 = 15unit
1
Required % = × 100 = 6.25%
16
20 Adda247 Publications For any detail, mail us at
Publications@adda247.com
Cracker Book for Bank (IBPS | SBI | RRB PO | Clerk) Mains Exams

18. (a); Let ratio of Vodka and wine in vessel A be 5x and 3x


and ratio of Vodka and wine in vessel B be 3y and 2y
ATQ—
5
3y+16× 29 3y+10 29
8
3 = ⇒ =
2y+16× 19 2y+6 19
8
58y – 57y = 190 – 174
y = 16
New quantity of mixture in vessel B
= (16 × 3 + 10) + (16 × 2 + 6)
= 96 ℓ
= initial quantity of mixture in vessel A
Quantity of Vodka remaining in vessel A
5 16×5
= 96 × – = 60– 10 = 50ℓ
8 8

19. (d); Investment Ratio of Sameer : Divyaraj : Veer


= x × 2 : Rs. (x + 8000) × 1 : Rs. (x + 12000) × 3
= 8 : 6 : 21
ATQ—
2x 8
(x+8000)
=
6
6x – 4x = 32000
x = 16000 Rs.
Required sum of capital (Sameer + Divyaraj + Veer)
= 16000 + (16000 + 8000) + (16000 + 12000)
= 68000 Rs.
6
20. (b); C.P. of mixture = 49 × = 42Rs/kg
7
2 y –42
=
3 42 –x
84 – 2x = 3y – 126
3y + 2x = 210 … (i)
New cost price of mixture after reverting the quantity in which
they are mixed.
11
(1 + )  new CP = 49
38
38
New CP = 49 × = 38 Rs./kg
49
21 Adda247 Publications For any detail, mail us at
Publications@adda247.com
Cracker Book for Bank (IBPS | SBI | RRB PO | Clerk) Mains Exams

So,

3 y –38
=
2 38 –x
114 – 3x = 2y – 76
190 = 2y + 3x ... (ii)
Solving (i) and (ii)
x = 30 Rs/kg

21. (e); Let the capacity of each vessel be 10x liters.


Then, vessel A contains 8x liters of milk and 2x liters of water.
Vessel B contains 7x liters of milk and 3x liters of water.
Vessel C contains 10x liters of milk.
Hence, Vessel D contains 25x liters of milk and 5x liters of water
i.e. in the ratio 5 : 1.
According to the question,
30 liters of the mixture is drawn from vessel D.
Final quantity of water in Vessel D = 40 liters
1
⟹ 5x − × 30 = 40
6
⟹ 5x = 45
⟹x=9
Capacity of each vessel = 10x = 90 liters

22. (c); Let in P, Quantity of A, B and C is 2x, x and x respectively


In Q → B and C is 3y and 2y
Now in final mixture → (X) A : B : C
2x : x+3y : x+2y = 8 : 13 : 10
Saving →
x = 4 unit
y = 3 unit
Quantity in P → (2 + 1 + 1) × 4 = 16 unit
Quantity in Q → (3 + 2) × 3 = 15unit
1
Required % = × 100 = 6.25%
16
22 Adda247 Publications For any detail, mail us at
Publications@adda247.com
Cracker Book for Bank (IBPS | SBI | RRB PO | Clerk) Mains Exams

23. (b); ATQ,


(100−37.5) (100−40)
(x + 24) × + 15 = (x + 54) ×
100 100
5 3
(x + 24) × + 15 = (x + 54) ×
8 5
5(5x + 120 + 120) = 8(3x + 162)
25x – 24x = 1296 – 1200
x = 96 liters
total mixture take out from vessel
3
A = (96 + 24) × = 45 litre
8
Total mixture taken out from vessel B
2
= (96 + 54) × = 96 liter
5
7 3
Milk in vessel C = 45 × + 60 ×
15 5
= (21 + 36) = 57 litre

24. (c); Let P ltr. of mixture are taken out from all 3 containers.
Quantity of X, Y and Z from A container
2P 3P P
→ , ,
6 6 6
From B container
P 2P 5P
→ , ,
8 8 8
From C container
3P P 3P
→ , ,
7 7 7
3P 2P P
Quantity of Y → + +
6 8 7
84P+42P+24P 150P
= =
168 168
2P P 3P
Quantity of X → + +
6 8 7
56P+21P+72P 149P
= =
168 168
150P 149P 1
Diff. = – = ⇒ P = 21 liter
168 168 8

25. (b); In vessel A


Nitrous oxide : Water = 7 : 2
In vessel B
Nitrous oxide : Water = 5 : 3
In 68 gm of mixture
23 Adda247 Publications For any detail, mail us at
Publications@adda247.com
Cracker Book for Bank (IBPS | SBI | RRB PO | Clerk) Mains Exams

Mixture taken from vessel A


9
= 68 × = 36 gm
17
8
Mixture taken from vessel B = 68 ×
17
= 32 gm
Total nitrous oxide in 68gm of mixture
7 5
= 36 × + 32 × = 28 + 20 = 48 gm
9 8
Total cost of 68 gm mixture = 68 × 80
= 5440 Rs
Cost of nitrous oxide = 48 × 80 = 3840
5440−3840 2
Required profit = × 100 = 41 %
3840 3

26. (a); Let Rana invested Y Rs.


∴ Yash invested 3Y Rs.
And these are in AP.
Y+3Y
∴ middle term is = 2Y.
2
2Y is amount invested by Nikhil.
Let total profit be 100x.
Then as a salary Rana got 8X.
And Nikhil got 5X.
Remaining 87X was divided according to share.
Y × 1 : 2Y × 8 : 3Y × 4
Y : 16Y : 12Y
1 : 16 : 12
Out of 87X Nikhil got.
16
× 87X = 48X.
29
Total money Nikhil got is 53X
Hence ATQ,
53X – 47X = 6000 ⇒ X = 1000
∴ 100X = 100000/- Which is total profit.

24 Adda247 Publications For any detail, mail us at


Publications@adda247.com
Cracker Book for Bank (IBPS | SBI | RRB PO | Clerk) Mains Exams

27. (b); ATQ, Volume of B is 20 liters


1
Milk (vessel B)= × 20 = 5 liters.
4
3
Water (vessel B) = × 20 = 15 liters.
4
When this is added to vessel A.
3x+5 1
= ⇒ 6x + 10 = 5x + 15
5x+15 2
x= 5 liters
Hence, milk in vessel A = 3x + 5 = 3 × 5 + 5
= 20 liters
Water in vessal A = 5x + 15 = 5 × 5 + 15
= 40 liters
60
60% of mixture of A = × (40 + 20)
100
= 36 liters.
Remaining mixture = 24 liters.
Milk remaining = 8 liters.
Water remaining = 16 liters.
After adding the content of Vessel C.
8+7 3 12
= ⇒ 60 = 48 + 3x ⇒ x = = 4
16+x 4 3

28. (b); ATQ –

Therefore,
Ratio of mixture, A to B is 3:2.
And ratio of mixture B to C is 4:3.
Hence, ratio of volume of mixtures A, B & C is
A:B:C=6:4:3
It is given that contains 22% water, which is 66 ml. Hence total
volume of mixture C is
100
66 × = 300 ml
22
B is 400 ml
A is 600 ml

25 Adda247 Publications For any detail, mail us at


Publications@adda247.com
Cracker Book for Bank (IBPS | SBI | RRB PO | Clerk) Mains Exams
15
Water in B = × 400 = 60 ml
100
10
Water in A = ×600=60 ml
100
Total water in jar after adding 30 ml water
= 60 + 66 + 60 + 30 = 216 ml.

29. (e); Let the price of oil per litre in container 'A' be 'x' and that in 'B'
be 'y' and the litres mixed be 'a' Initial price of A is 140x.
When 'a' litres are removed,it should be 140x-ax.
When 'a' litres from B are added,it is 140x-ax+ay , similarly for B
is 60y-ay+ax.
Now, the overall litres of both mixtures do not change as ‘a’
litres are taken away and are replaced, So price per litre after
mixing for :-
1. Solution A :- (140x-ax+ay)/140
2. Solution B :- (60y-ay+ax)/60.
Equating them,
8400(x − y) = 200a(x − y)
Or, a = 8400/200
Or, a = 42 litres

30. (b); Let cost price per kg of Type B wheat = x Rs.


And, Cost price per kg of type B wheat
= 10x Rs.
Selling price of mixture of Type A & Type B wheat = 11x Rs.
Cost price per kg of mixture of Type A & Type B wheat = 11x
32
× = 6.4x Rs.
55
ATQ –
Let N kg of type B wheat in mixture of 200 kg of Type A & Type B
of wheat
6.4x × 200 = N × x + 10x (200 − N)
1280 = N + 2000 – 10N
9N = 920 ⇒ N = 80 kg
80 2
Required ratio = =
120 3

26 Adda247 Publications For any detail, mail us at


Publications@adda247.com
Cracker Book for Bank (IBPS | SBI | RRB PO | Clerk) Mains Exams

31. (c); Farmer A has ratio of urea & Potassium in mixture = 5 : 3


Farmer B has ratio of urea & Potassium in mixture = 3 : 2
Let farmer C have x kg of urea & x kg of potassium
Farmer C has Urea in resulting mixture
40 5 50 3
= 120 × × + 80 × × +x
100 8 100 5
= (54 + x ) kg
Farmer C has Potassium in resulting mixture
40 3 50 2
= 120 × × + 80 × × +x
100 8 100 5
= (34 + x) kg
ATQ –
(54 + X ) 7
(34 + x)
=
5
270 + 5x = 238 + 7x
2x = 32 ⇒ x = 16 kg
initial quantity of mixture farmer C had = 16 × 2 = 32 kg

32. (a); Let quantity of vessels A, B and C is 25x, 20x and 18x
respectively
Now in vessel A → Milk : Water
2 : 3
10x milk and 15x water
In vessel B → Milk : Water
3 : 2
12x milk and 8x water
In vessel C → Milk : Water
4 : 5
8x milk and 10x water
ATQ→
33x– 30x = 6 ⇒ 3x = 6 ⇒ x = 2
Quantity in A, B and C
50 liter, 40 liter and 36 liter

27 Adda247 Publications For any detail, mail us at


Publications@adda247.com
Cracker Book for Bank (IBPS | SBI | RRB PO | Clerk) Mains Exams

33. (b); Let volume of largest vessel = 3x liter


2
Volume of second largest vessel = 3x ×
3
= 2x liter
75
Volume of smallest vessel = 2x ×
100
= 1.5x liter
ATQ—
10
Q = 5 × 3x + 8 × 2x + × 1.5x
3
Q = 15x + 16x + 5x
Q = 36x liter
16x 4
Required percentage = × 100 = 44 %
36x 9

34. (b); Ramesh has = (x + 18) kg


Suresh = (x + 18) + 12 = (x + 30) kg
Vikash = (x + 30) + 3 = (x + 33) kg
For Ramesh,
5
First type of Apple = (x + 18) × kg
8
3
2nd type of Apple = (x + 18) × kg
18
ATQ,
5 5 5
(x+18)× −16× 5 (x+18)× −10 5
8 8 8
3 3 = ⇒ 3 =
(x+18)× −16× +12 6 (x+18)× +6 6
8 8 8
30 15
⇒ (x + 18) × − 60 = (x + 18) × + 30
8 8
15
⇒ (x + 18) × = 90 ⇒ x = 30 kg
8
For Vikash,
Total quantity = 63 kg
4
First type = 63 × = 36 kg
7
3
Second type = 63 × = 27 kg
7
ATQ,
36 1
= ⇒ y = 9 kg
27+y 1

28 Adda247 Publications For any detail, mail us at


Publications@adda247.com
Cracker Book for Bank (IBPS | SBI | RRB PO | Clerk) Mains Exams

35. (d); Let vessel A has 5x L and 4x L of milk and water


After first process ratio of wine, milk and water in vessel B will
be
5 4
= Y ∶ 18 × ∶ 18 × = Y : 10 : 8
9 9

Now 10 litre of mixture from B is poured in A


y
Then, wine will be = × 10
y+10+8
y
× 10 = 4
y+18

10y = 4y + 72 ⇒ 6y = 72
y = 12 litre

29 Adda247 Publications For any detail, mail us at


Publications@adda247.com
Cracker Book for Bank (IBPS | SBI | RRB PO | Clerk) Mains Exams

1 Adda247 Publications For any detail, mail us at


Publications@adda247.com
Cracker Book for Bank (IBPS | SBI | RRB PO | Clerk) Mains Exams

Chapter

3 Partnership

BEST APPROACH TO SOLVE THE QUESTIONS

Any partnership in business is based on two components. One is the money


or capital that is invested and the other is the time for which the money is
invested.
Mathematically,

Profit ∝ investment
Profit ∝ time
Profit ∝ investment × time
Profit share of an individual is directly proportional to investment and time.
Hence, any profit share of a person out of total profit is decided by the net
effect of these two components.
If ratio of investment by two persons is X:Y and ratio of time for which they
invested is A:B respectively, then their profit share is in the ratio of 𝑋 × 𝐴 ∶
𝑌 × 𝐵= XA : YB

Example 1: A, B and C entered into a business by investing their intial sum


of Rs. 12600, Rs. 14400 and Rs. 13200 respectively. After 6 months C left
the partnership and B also left the partnership after 8 months. If after a year
total profit was Rs. 69600, then find profit share of C.’

Solution. Here, instead of taking absolute values of money invested, we may


use their simplest values i.e. ratio.
Ratio of investment of A, B and C is 21:24:22 respectively. And ratio of time
for which they invested is 12: 8 : 6 = 6 : 4 : 3
Ratio of their profit share = (21× 6): (24 × 4): (22 × 3)= 21:16:11
11
Now, profit share of C= × 69600 = 15950.
48
2 Adda247 Publications For any detail, mail us at
Publications@adda247.com
Cracker Book for Bank (IBPS | SBI | RRB PO | Clerk) Mains Exams

Example: ‘Aman started a business investing Rs. 70,000. Rakhi Joined him
after six months with an amount of Rs. 1,05,000 and Sagar Joined them with
Rs. 1.4 lakhs after another six months. The amount of profit earned should
be distributed in what ratio among Aman, Rakhi and Sagar respectively, 3
years after Aman started the business?’

Solution. Ratio of profit share is given by the ratio of (investment×time)


Or ratio of their investment is 70000 : 105000 : 140000 = 2 : 3 : 4
Ratio of the time for which they invested = 36 : 30 : 24 = 6 : 5 : 4.
Hence, ratio of profit share among them = (2× 6): (3 × 5) ∶ (4 × 4)
= 12:15:16

Practice Exercise Based on New Pattern

1. A and B started a business with a capital of Rs. 32,000 and Rs. 56,000.
C join the business on a condition that they all will share the profit
equally. For the loss of A and B, C gives Rs. 19800 to A and B to
compensate their loss. How much amount A get out of Rs. 19800?
(a) 1200 (b) 1600 (c) 1800
(d) Data inadequate (e) None of these

Direction (2-3): A, B and C start a business. Ratio between investment of A


and B is 5 : x while ratio between investment of B and C is 9 : 11. A, B and C
invested for 8 month, 6 month and 12 month respectively.

2. If ratio between A’s share in profit to B’s share in profit is 5 : 6 and


difference between C’s share in profit to B’s share in profit is 2340,
then find the total profit earned by all three together? (2 Marks)
(a) 7700 (b) 5390 (c) 6160
(d) 6930 (e) Cannot be determined

3. Find the value of ‘x’. If out of total profit of Rs 2460, A’s share in profit
is Rs 600. (1 Mark)
(a) 8 (b) 7 (c) 6
(d) 5 (e) 4
3 Adda247 Publications For any detail, mail us at
Publications@adda247.com
Cracker Book for Bank (IBPS | SBI | RRB PO | Clerk) Mains Exams

4. Rahul and Rohit start a business by investing Rs.4900 and Rs.3600


2 1
respectively. After 4 months, Rahul and Rohit withdraw th and of
7 3
their initial investment and after 6 months of starting of business they
invested 60% of what they withdraw. Find the share of Rohit in total
profit of Rs. 8490
(a) Rs.3555 (b) Rs.4935 (c) Rs.1855
(d) Rs.4550 (e) Rs.4850

5. ‘P’ and ‘Q’ entered into a partnership with 50,000 and 60,000 for 8
and 6 months respectively. ‘P’ buy a car from its whole profit and ‘Q’
invested his whole profit in a scheme which offer 10% interest on S.I.
1
P’s car value depreciated 11 % every year. If after 2-year difference
9
between their money (excluding initial investment) is 23,480 then
find P’s profit.
(a) Rs. 72000 (b) Rs. 35000 (c) Rs. 64000
(d) Rs. 54000 (e) Rs. 81000

6. A and B started a business in partnership by investing in the ratio of


7 : 9 . After 3 months A withdraw 2/3 of its investment and after 4
1
months from the beginning B withdraw 33 % of its investment. If a
3
total earned profit is Rs. 10201 at the end of 9 months, find the share
of each in profit.
(a) Rs. 3535, Rs.6666 (b) Rs. 3055, Rs. 5555
(c) Rs. 4503, Rs. 1345 (d) Rs. 3545, Rs. 3333
(e) None of these

7. Satish and Saurbh entered into a partnership business for eight


months. Satish invested Rs. 40x and Saurbh invested 40% more than
Satish, after four months from starting of business Rituraj joined the
both with the equal capital of Satish and Saurbh invested together
initially. If they got a total profit of Rs. 7200, then find profit share of
Rituraj?
(a) 1600 Rs. (b) 1800 Rs. (c) 2000 Rs.
(d) 2200 Rs. (e) 2400 Rs.
4 Adda247 Publications For any detail, mail us at
Publications@adda247.com
Cracker Book for Bank (IBPS | SBI | RRB PO | Clerk) Mains Exams

8. Veer and Sameer enter into a business by making the investment in


the ratio of 3 : 4. After six months Veer added Rs. 2000 more in his
initial investment, while Sameer withdraw Rs. 4000 from his initial
investment. If at the end of one year Veer and Sameer shared profit in
the ratio of 7 : 9, then find initial investment of both?
(a) 69000 Rs. 92000 Rs (b) 36000 Rs. 48000 Rs.
(c) 27000 Rs. 36000 Rs. (d) 45000 Rs. 60000 Rs.
(e) 24000 Rs. 32000 Rs.

9. P and R entered into partnership business with the capital of Rs. x and
Rs. (x + 12000), after One year Q joined them with capital of Rs. (x +
8000) at the end of 2nd year P and Q with draw their capital and R
invest for one more year, if P, Q and R gets profit in the ratio of 8 : 6 :
21 respectively. Find sum of capital invested by all three?
(a) 66000 Rs. (b) 54000 Rs. (c) 64000 Rs
(d) 68000 Rs. (e) 70000 Rs.

10. Three partners A, B and C invested their amounts in ratio of 2 : 5 : 8.


At the end of four months, A invests some amount such that, his total
investment will be equal to C’s initial investment and C withdraw
some amount that its total investment will be equal to B’s initial
investment. If 10% of B’s share in profit of one year is Rs 2550. Then
share of A and B in total annual profit will be?
(a) 81500 (b) 61200 (c) 56100
(d) 75000 (e) None of these

11. A and B enter into a partnership business and decided to distribute


60% of profit as per (investment× 𝑡𝑖𝑚𝑒) ratio and rest in the ratio of
7:10. If the amount invested by A is 25 % more than B and B withdraw
half of his total investment after 9 months. Find the profit distribution
ratio of A and B according to given condition at the end of a year.
(a) 43:41 (b) 44:41 (c) 48:43
(d) 49:47 (e) None of these

5 Adda247 Publications For any detail, mail us at


Publications@adda247.com
Cracker Book for Bank (IBPS | SBI | RRB PO | Clerk) Mains Exams

Directions (12-13): Three partners Abhishek, Neeraj and Aniket enter into
a partnership business with the capitals in the ratio of 3 : 5 : 8. After four
month Abhishek, Neeraj and Aniket added Rs. 6000, Rs. 9000 and Rs. 12000
respectively, after next four months Abhishek and Aniket withdraw Rs.
5000 and Rs. 8000 respectively and Neeraj added additional Rs. 6000.

12. Two persons Ankur and Saurabh enter a business. Ankur invested
capital 6000 more than what Abhishek invested for first four month
and Saurabh invested what Aniket invested for last four months. If
Ankur and Saurabh got profit share after one year in the ratio of
15 :28 then find theinvestment of Neeraj for last four months?(1
marks)
(a) 24000 Rs. (b) 18000 Rs. (c) 16000 Rs.
(d) 30000 Rs. (e) 36000 Rs.

13. Amir and Mayank started a business in partnership. Amir


investmentfor six months isRs. 6000 more thanwhat Neeraj invested
for first fourmonthsand Mayank invested for eight months, Rs. 2000
lessthan whatAniket invested for first four monthsIf Mayank got profit
share of Rs. 15400, out of total profit Rs. 22750 thenfind the
investment of Amir is how much less than investment of Mayank?
(2 marks)
(a) 218000 Rs (b) 216000 Rs. (c) 210000 Rs.
(d) 220000 Rs. (e) 232000 Rs.

14. Veer, Sameer and Gopal enter into a business by making investment in
the ratio of 5 : 6 : 7 respectively. After six months Veer and Sameer
1
withdraw th and 25% of his initial investment respectively. After
5
1
eight months from the starting of business Gopal added 𝑡ℎ of his
7
initial investment. If at the end of one year profit share of Gopal is Rs.
2000 more than Sameer, then find profit share of Veer?
(a) 4420 Rs. (b) 4520 Rs. (c) 4820 Rs.
(d) 4320 Rs. (e) 4720 Rs.
6 Adda247 Publications For any detail, mail us at
Publications@adda247.com
Cracker Book for Bank (IBPS | SBI | RRB PO | Clerk) Mains Exams

15. Manish and Rituraj invested Rs. 12000 and Rs. 16000 in a business.
After four months Manish and Rituraj both added Rs. 4000 in their
initial investment. At the end of one year the total profit was Rs.
172500, if Manish and Rituraj invested their profit share on
compound interest at the rate of 20% and 10% respectively then find
difference between interests got by both at the end of two years?
(a) Rs.10250 (b) Rs.11520 (c) Rs.12210
(d) Rs.13110 (e) Rs.12660

Direction (16 – 17): Data given below about investment of three partners
in a business. Read the data carefully and answer the questions:
Swati, Monika and Anshika three business partners enter into a business by
making investment Rs. 4000, Rs. 6000 and Rs. 8000 respectively. After first
quarter Swati, Monika and Anshika added some amount in the ratio of 2 : 3 :
2 1
5. After second quarter Swati, Monika and Anshika withdrew 𝑡ℎ , 𝑟𝑑 and
5 3
3
𝑡ℎ respectively of capital invested in second quarter. At the end of nine
7
month profit share divided among Swati, Monika and Anshika in the ratio of
24 : 37 : 49 respectively.

16. Satish and Veer enter into a business, Satish invested 200% more than
Swati capital for second quarter, while Veer invested 300% more than
Anshika capital for third quarter. And profit share of Satish at end of
two year is Rs. 7250 out of total profit of Rs. 15950. Then find for how
many months Veer invested his capital?
(a) 14 months (b) 12 months (c) 18 months
(d) 10 months (e) 8 months

17. Monika and Anshika enter into a business with another partner
Shikha. Monika invested 144% of what she invested in second
quarter, while Anshika invested 160% of what she invested in second
quarter. If all three invested their capital for a year and Shikha gets a
profit of Rs. 9600 out of total profit of Rs. 28000, then find investment
of Shikha?
(a) 12,800 Rs. (b) 11,600 Rs. (c) 14,400 Rs.
(d) 14,200 Rs. (e) 13,600 Rs.

7 Adda247 Publications For any detail, mail us at


Publications@adda247.com
Cracker Book for Bank (IBPS | SBI | RRB PO | Clerk) Mains Exams

18. Satish, Veer and Neeraj entered into a business by investing in the
ratio of 4 : 6 : 3. After two years, Satish and Veer withdrew 25% and
1
33 % of their respective initial investment, while Neeraj added 50%
3
of his initial investment more. If at the end of three years they
received total profit of Rs. 93750, out of which 22.5% are used for
maintaining business and the remaining profit was divided among
them according to their investment ratio. Find difference between
profit share of Satish and Neeraj?
(a) 978.75 Rs. (b) 976.75 Rs. (c) 974.25 Rs.
(d) 968.75 Rs. (e) 966.75 Rs.

19. A, B and C enter into a partnership and invested some amount. After
1
one year A double its investment, B increase its investment by 33 %
3
and C increase its investment by 20%. In the third year A and B
withdraw their investments and D joins the partnership with C. After
three year they got profit in the ratio of 12 : 14 : 17 : 8 (A : B : C : D). If
difference between initial investment of B and C is 1150. Then Find
out the total initial investment made by A and D together?
(a) 12100 (b) 14400 (c) 13800
(d) 15000 (e) None of these

5 4 6
20. Three partners A, B and C invested in the ratio of ∶ ∶ in a business.
4 5 5
After 3 months A increased his capital by 40%, after 3 months more A
again increased his capital by 20%. B increased his capital by 25%
after every 4 months. C increased his capital by 25% after 3 months
and after 4 months more C increased his capital by 20%. If the total
profit of Rs. 30,130 is earned at the end of the year, find what was the
A’s share of profit?
(a) Rs. 12,420 (b) Rs. 14,420 (c) Rs. 13,240
(d) Rs. 15,340 (e) Rs. 12,340

8 Adda247 Publications For any detail, mail us at


Publications@adda247.com
Cracker Book for Bank (IBPS | SBI | RRB PO | Clerk) Mains Exams

21. A, B and C entered into a partnership business, A invested Rs. x, B


invested 25% more than A and C invested 20% more than B for first 6
months. After that A left the business and B withdraw 50% of his
investment. B left the business after 3 more months whereas C
2
increased his investment by 16 % . If at the end of year difference
3
between profit share of C and (A + B) together is Rs. 12375, then find
profit share of C ?
(a) 58500 Rs. (b) 56500 Rs. (c) 55680 Rs
(d) 55580 Rs. (e) 52680 Rs.

22. Veer, Sameer and Satish entered into a business by investing equal
amount each. After four months Sameer added half of his initial
investment, while Veer and Satish withdrew half of their initial
investment. At end of one year the total profit is Rs. 8000 and Veer
and Sameer invested their profit share at the rate of 10%
compounded annually for some time in a scheme. If the difference of
the amount received by Veer and Sameer from the scheme at the end
is Rs. 2420, find for how much time Veer and Sameer invested in the
scheme?
1
(a) 4 yr (b) 3 yr (c) 3 yr
2
(d) 2 yr (e) 1yr

23. P, Q and R entered into a partnership with their initial capital in the
1
ratio of 3 : 5 : 8 respectively. After third month P withdrew rd of his
3
initial capital and Q withdraw half of his initial capital. After next six
months P increased his capital by 100% more, Q increased by 300%
more and R increased by 25% more. If at the end of an year, difference
between profit share of Q and R was Rs. 7000, then find profit share of
P?
(a) Rs. 6600 (b) Rs. 7700 (c) Rs. 5500
(d) Rs. 5750 (e) Rs. 5950

9 Adda247 Publications For any detail, mail us at


Publications@adda247.com
Cracker Book for Bank (IBPS | SBI | RRB PO | Clerk) Mains Exams

24. Veer, Sameer and Divyaraj enter into a business by making an


1 1 1
investment in the ratio of ∶ ∶ respectively. After six months
129 301 602
1
from starting of business Veer withdrew 33 % of his initial
3
investment, while Sameer added 50% of his initial investment and
Divyaraj added Rs. 1800 respectively. If after one year profit share of
Veer is Rs. 10500 out of total profit of Rs. 20850, then find difference
between initial investment of Veer and Sameer?
(a) 6400 Rs. (b) 7200 Rs. (c) 7800 Rs.
(d) 5600 Rs. (e) 8400 Rs.

25. A, B & C, three friends started the business in which A invested for 6
months, B initially didn’t invest & started as working partner, while C
invested for 4 months. They decided to donate 1% of total profit and
to give 22% of total profit to B as salary. Find the ratio of their profit
shares (A:B:C) if A and C invested in the ratio of 2 : 3 and B also
invested an amount which is 80% of total amount invested by A and C
together for a single month.
(a) 1 : 3 : 1 (b) 1 : 1 : 3 (c) 3 : 1 : 3
(d) 1 : 1 : 1 (e) None of these

10 Adda247 Publications For any detail, mail us at


Publications@adda247.com
Cracker Book for Bank (IBPS | SBI | RRB PO | Clerk) Mains Exams

Solutions

A B
1. (c); Ratio of share of profit = 32 : 56
= 4 : 7 = 11
Let, total profit = Rs. 11
Now, A, B and C will distribute Rs. 11 equally
i.e.
A : B : C
11 11 11
3
: 3
: 3
11 1
Loss of A = 4 – =
3 3
11 10
Loss of B = 7 – =
3 3
Ratio of Loss = 1 : 10
Amount that got to compensate the loss
1
= × 19800 = 1800
11

Solution (2-3): Ratio between A’s, B and C’s profit share = 45 × 8 : 9x × 6 :


11x × 12
= 60 : 9x : 22x

2. (d); Let profit sharing of A, B and C be 60z, 9xz and 22xz


respectively.
ATQ,
22xz – 9xz = 2340
⇒ 13xz =2340
⇒ xz = 180
B’s and C’s share in profit is 9 × 180 and 22 × 180 respectively.
9×180
A’s share in profit = × 5 = 1350
6
Total profit earned by all three together
= 1350 + 1620 + 3960 = 6930
11 Adda247 Publications For any detail, mail us at
Publications@adda247.com
Cracker Book for Bank (IBPS | SBI | RRB PO | Clerk) Mains Exams

3. (c); ATQ,
60 600
=
60+9𝑥+22𝑥 2460
⇒ 246 = 60 + 31𝑥
186
⇒𝑥 = =6
31

4. (a); Investment of Rahul


For first four month = 4900
For fifth and sixth month
5
= × 4900 = 3500
7
3 2
For Last 6 month = 3500 + × × 4900
5 7
= 4340 Rs.
Investment of Rohit
For first 4 month = 3600 Rs.
2
Next 2 month = × 3600 = 2400 Rs.
3
3 1
Last 6 month = 2400 + × × 5600 = 3120
5 3
Profit ratio
Rahul Rohit
4900 × 4 + 3500 × 2 + 4340 × 6 : 3600 × 4 + 2400 × 2 + 3120 × 6
329 : 237
Total profit = 8490 Rs.
Share of Rohit out of total profit
8490
= × 237 = 3555 Rs.
566

5. (e); Profit ratio of P and Q


50,000 × 8 : 60,000 × 6
10 : 9
Let P’s and Q’s profit → 10𝑥 and 9𝑥
Value of P’s money after 2 years
8 8 640𝑥
= 10𝑥 × × =
9 9 81
Value of Q’s money after 2 years
12 Adda247 Publications For any detail, mail us at
Publications@adda247.com
Cracker Book for Bank (IBPS | SBI | RRB PO | Clerk) Mains Exams
9𝑥×2×10 120 108
=9𝑥 + = 9𝑥 × = 𝑥
100 100 10
108𝑥 640𝑥
Difference = – = 23,480
10 81
𝑥 = 8100
P’s profit = Rs. 81,000

6. (a);
A : B
Capital → 7×3 : 9×3
New Ratio → A : B
21x : 27x
Total capital invested by A in 9 months
= 21x × 3 + 7x × 6 = 105x
Total capital of B invested in 9 months
= 27x × 4 + 18x × 5
= 108x + 90x = 198x
A : B
Capital → 105x : 198x
According to question,
(105x + 198x) = Rs. 10201
303x = 10201
10201
x = Rs.
303
Hence,
10201
Share of A = × 105 = Rs. 3535
303
10201
Share of B = 198 × = Rs. 6666
303

7. (e); Satish capital = 40x Rs.


40
Saurbh capital = 40x + 40x ×
100
= 56x Rs.
Rituraj capital = 40x + 56x = 96x Rs.

13 Adda247 Publications For any detail, mail us at


Publications@adda247.com
Cracker Book for Bank (IBPS | SBI | RRB PO | Clerk) Mains Exams

Ratio of profit (Satish : Saurbh : Rituraj)


= 8 × 40x : 8 × 56x : 96x × 4
= 320x : 448x : 384x
=5:7:6
6
Rituraj profit share = 7200 × (5+7+6)
= 2400 Rs.

8. (a); Lets Veer and Sameer invested Rs. 3x and Rs. 4x respectively.
Ratio of investment of Veer and Sameer
= [3x × 6 + (3x + 2000) × 6] : [4x × 6 + (4x – 4000) × 6]
= (36x + 12000) : (48x – 24000)
ATQ—
36𝑥+12000 7
=
48𝑥 –24000 9
36𝑥 × 9 + 12000 × 9 = 7 × 48𝑥 − 24000 × 7
12000(9 + 14) = 12(7 × 4𝑥 − 3 × 9𝑥)
1000 × 23 = 28𝑥 − 27𝑥
x = 23000 Rs.
Investment of Veer = 3 × 23000 = 69000 Rs.
Investment of Sameer = 4 × 23000
= 92000 Rs.

9. (d); P : Q : R
Rs. x × 2 : Rs. (x + 8000) × 1 : Rs. (x + 12000) × 2
= 8 : 6 : 21
ATQ—
2𝑥 8
(𝑥+8000)
=
6
6x – 4x = 32000
x = 16000
Required sum of capital (P + Q + R)
= 16000 + (16000 + 8000) + (16000 + 12000)
= 68000 Rs.
14 Adda247 Publications For any detail, mail us at
Publications@adda247.com
Cracker Book for Bank (IBPS | SBI | RRB PO | Clerk) Mains Exams

10. (c); Let amount invested by A, B and C are 2x, 5x and 8x respectively.
⇒ At the end of 4th month A’s investment 2x + 6x = 8x
= c’s initial invested
⇒ C’s investment = 8x – 3x = 5x = B’s initial investment
Ratio of investment
A : B : C
= (2x × 4 + 8x × 8) : (5x × 12) : (8x × 4 + 5x × 8)
6 : 5 : 6
Let B’s profit = 5y
Now ATQ
5𝑦×10
= = 2550
100

y = 5100
share of A and B = 5y + 6y = 11y
= 11 × 5100 = 56100

11. (b); Let the total investment of B = 100


Investment of A = 100×125/100
Ratio of 60% profit
= 125×12 : 100×9+50×3= 10:7
Let the total profit = 170
Profit for A for only 60 % of profit
= 170×0.6×(10/17)=60
Profit of B for only 60 % of profit
= 102-60 = 42
Total profit of A = 60+ 68×7/17= 88
Total Profit of B = 42+68×10/17 = 82
Required ratio = 88:82 = 44:41
15 Adda247 Publications For any detail, mail us at
Publications@adda247.com
Cracker Book for Bank (IBPS | SBI | RRB PO | Clerk) Mains Exams

Solutions (12-13); Let investment of Abhishek, Neeraj and Ankit is


Rs. 3x, Rs. 5x and Rs. 8x respectively.
ATQ—Investment ratio of Abhishek, Neeraj and Aniket
= [3x × 4 + (3x + 6000) × 4 + (3x + 6000 – 5000)
× 4] : [5x × 4 + (5x + 9000) × 4 + (5x + 9000 + 6000) × 4] : [8x × 4 + (8x +
12000) × 4 + (8x + 12000 – 8000)
× 4]
= (36x + 28000) : (60x + 96000) : (96x + 64000)

12. (d); Investment of Ankur = Investment of Abhishek for first four


months and next four months
= (3x + 6000) Rs.
Investment of Saurabh = Investment of Aniket for last four
months
= (8x + 12000 – 8000) Rs.
ATQ—
(3𝑥+6000)×12 15
(8𝑥+4000)×12
=
28

120x - 84x = 168000 – 60000


36x = 108000
x = 3000 Rs.
Neeraj invested for last four months
= (5x + 9000 + 6000) Rs.
= (5×3000 + 9000 + 6000)
= 30000 Rs

16 Adda247 Publications For any detail, mail us at


Publications@adda247.com
Cracker Book for Bank (IBPS | SBI | RRB PO | Clerk) Mains Exams

13. (e); Investment of Amir = Investment of Neeraj for first four months
+6000
= 5x + 6000
Investment of Mayank = Investment of Aniket for first four
months – 2000
= 8x -2000
(5x+6000)×6 22750 –15400
(8x−2000)×8
=
15400
(5𝑥+6000) 7
(8𝑥−2000)
=
11

56x – 55x = 80000


x = 80000
required difference = 8x-2000-5x-6000
3x – 8000
240000 – 8000= 232000

14. (d); Let Veer, Sameer and Gopal invested Rs. 5x, Rs. 6x and Rs. 7x
respectively
Ratio between profit share of Veer, Sameer and Gopal
4 3
= (5𝑥 × 6 + 5𝑥 × × 6) : (6𝑥 × 6 + 6𝑥 × × 6):
5 4
8
(7𝑥 × 8 + 7𝑥 × × 4)
7

= 54𝑥 ∶ 63𝑥 ∶ 88𝑥


= 54 : 63 : 88
2000
Profit share of Veer = 54 × (88
–63)

= 4320 Rs.

17 Adda247 Publications For any detail, mail us at


Publications@adda247.com
Cracker Book for Bank (IBPS | SBI | RRB PO | Clerk) Mains Exams

15. (d); Ratio of profit of Manish and Rituraj


= [(12000 × 4)+ (12000 + 4000) × 8] : [(16000 × 4) + (16000 +
4000)× 8]
= 17600 : 224000 = 11 : 14
11
Profit share of manish = 172500 ×
25

= Rs.75900
14
Profit share of Rituraj = 172500 ×
25

= 96600 Rs
Equivalent CI of two year at the rate of 20%
20×20
= 20 + 20 + = 44%
100

Equivalent CI of two year at 10%


10×10
= 10 + 10 + = 21%
100

Required difference between interest


44 21
= 75900 × − 96600 ×
100 100

= 13110

Solutions (16-17): Let total amount added by Sawti and Monika after first
3
4000 ×3+(4000+2𝑥)×3+(4000+2𝑥)×5×3 24
quarter be Rs. 2x and Rs. 3x 2 =
6000×3+(6000+3𝑥)×3+(6000+3𝑥)×3×3 37

1
12000+12000+6𝑥+(36000+18𝑥)×5 24
=
18000+18000+9𝑥+12000+6𝑥 37
156000+48𝑥 24
=
240000+75𝑥 37

240500 + 74x = 240000 + 75x


x = 500 Rs.
18 Adda247 Publications For any detail, mail us at
Publications@adda247.com
Cracker Book for Bank (IBPS | SBI | RRB PO | Clerk) Mains Exams

16. (c); Total amount invested by Sawti in second quarter = (4000 +


2× 500) = 5000 Rs.
Total amount invested by Anshika in third quarter = (8000 +
500 × 5 − 4500)
= 6000 Rs.
300
Satish investment = 5000 ×
100
= 15000 𝑅𝑠.
400
Veer investment = 6000 × = 24000 𝑅𝑠.
100

Let Veer invested for T months


ATQ –
15000 ×24 7250
=
24000×𝑇 (15950−7250)
120 145
=
8×𝑇 174

T = 18 months

17. (c); Total amount invested by Monika in second quarter = 6000 +


3 × 500 = 7500 𝑅𝑠.
144
Monika investment = 7500 ×
100
= 10800 𝑅𝑠.
Total amount invested by Anshika in second quarter = 8000
+ 5 × 500 = 10500 𝑅𝑠.
160
Anshika investment = 10500 ×
100
= 16800 𝑅𝑠.
Let Shikha invested = P Rs.
ATQ –
𝑃×12 9600
=
(10800+16800)×12 (28000−9600)
𝑃 9600
=
27600 18400
P = 14,400 Rs.

19 Adda247 Publications For any detail, mail us at


Publications@adda247.com
Cracker Book for Bank (IBPS | SBI | RRB PO | Clerk) Mains Exams

18. (d); Let the amount invested by Satish, Veer and Neeraj initially be
4x, 6x and 3x respectively.
Ratio of profit share of Satish, Veer and Neeraj
3 2
= 4x × 24 + 4x × × 12) ∶ (6x × 24 + 6x × × 12) : (3x × 24 +
4 3
3
3x × × 12)
2
= 22 : 32 : 21
Total remaining profit after excluding the amount spend on
maintaining business
(100–225)
= 93750 × = 72656.25 Rs.
100
(22–21)
Required difference = 72656.25 ×
75

= 968.75 Rs.

19. (c); Let investment of A, B, C and D is a, b, c and d respectively.


A B C 𝐷

Now in firt year → a × 12 : b × 12 : c × 12

4b 6c
In 2nd year → 2a × 12 :
× 12 : × 12
3 5
6c
In 3rd year × 12 : 𝑑 × 12
5
4
A : B : C : D ⇒ (a × 12 + 2a × 12) : (b × 12 + b × 12) : c × 12 +
3
6
2 c × 12 : d × 12
5
7𝑏 17
3a : : 𝑐 : d = 12 : 14 : 17 : 8
3 5
⇒a:b:c:d=4:6:5:8
Difference between B and C initial investment = 1150
Total Investment of A and D together
1150
= × 12 = 13800
1

20 Adda247 Publications For any detail, mail us at


Publications@adda247.com
Cracker Book for Bank (IBPS | SBI | RRB PO | Clerk) Mains Exams

20. (a); Ratio of investment of A, B and C


5 4 6
= ∶ ∶ = 25 ∶ 16 ∶ 24
4 5 5
Ratio of profit after one year
A : B : C
25  3  35  3  42  6 : 16  4  20  4  25  4 : 24  3  30  4  36  5
= 432 : 244 : 372
= 108 : 61 : 93
30130
A’s share in profit = × 108 = 12,420
262

21. (a); Let investment of A, B and C


= x : 1.25x : 1.5x
= 4x : 5x : 6x
ATQ—
Profit share of A : B : C = 4x × 6 : (5x × 6 + 2.5x × 3) : (6x × 6 +7x
× 6)
= 24x : 37.5x : 78x
Given→ C – (A + B) = 12375
78x – (24x + 37.5x) = 12375
x = 750
Profit share of C = 750 × 78 = 58500 Rs.

22. (d); Let Veer, Sameer and Satish invested Rs. x initially
Ratio of profit of Veer, Sameer and Satish
x 3x x
= (x × 4 + × 8) : (x × 4 + × 8) : (x × 4 + × 8)
2 2 2
= 8x ∶ 16x ∶ 8x
=1:2:1
1
Profit share of Veer = 8000 × = 2000 Rs.
4
Profit share of Sameer
2
= 8000 × = 4000 Rs.
4
Let both invested for n years
10 n 10 n
4000 (1 + ) – 2000 (1 + ) = 2420
100 100
10 n
2000 (1 + ) = 2420
100
10 n
(1 + ) = 1.21
100
n = 2 years
21 Adda247 Publications For any detail, mail us at
Publications@adda247.com
Cracker Book for Bank (IBPS | SBI | RRB PO | Clerk) Mains Exams

23. (c); Lets P, Q and Q investment be 3x, 5x and 8x profit share of


P:Q:R
= (3x × 3 + 2x × 6 + 4x × 3) : (5x × 3 + 2.5x × 6 + 10x × 3) :
(8x × 9 + 10x × 3)
= 33x : 60x : 102x
= 11 : 20 : 34
ATQ—
34 – 20 => 7000
14 => 7000
1 => 500
Profit share of P = 11 × 500
= Rs. 5500

24. (b); Initial investment of Veer, Sameer and Divyaraj


1806 1806 1806
= ∶ ∶
129 301 602
= 14 : 6 : 3
Let Veer, Sameer and Divyaraj initial investment be 14x Rs., 6x
Rs. & 3x Rs. respectivelyProfit ratio of Veer, Sameer and Divyaraj
2
= (14x × 6 + 14𝑥 × × 6 ) : (6x × 6 + 9𝑥 × 6) ∶ [3𝑥 × 6 + (3𝑥 +
3
1800) × 6]
= (84x + 56x) : (36x + 54x) : (18x + 18x + 10800)
ATQ –
140𝑥 10500
=
90𝑥+36𝑥+10800 20850−10500
𝑥 1
(21𝑥+1800)
=
23

2x = 1800
x = 900 Rs.
Required difference = 14× 900 − 6 × 900
= 7200 Rs.

22 Adda247 Publications For any detail, mail us at


Publications@adda247.com
Cracker Book for Bank (IBPS | SBI | RRB PO | Clerk) Mains Exams

25. (d); Let A invested 2x and C invested 3x.


80
Then B invested = × (3x + 2x)
100
80
= × 5x = 4x
100
Let 100Y be their total profit.
According to question,
1% is given as donation and B gets 22% as salary of total profits.
Now ratio of their shares is
2x × 6 : 4x × 1 : 3x × 4
12x : 4x : 12x
3 : 1 : 3
3
A gets × [100Y– 23Y]
7
3
= × 77Y = 33Y
7
3
C gets = × 77Y = 33Y
7
1
B gets = × 77Y + 22Y = 33Y
7
Hence, ratio of their profit shares is
33Y : 33Y : 33Y
=1:1:1

23 Adda247 Publications For any detail, mail us at


Publications@adda247.com
Cracker Book for Bank (IBPS | SBI | RRB PO | Clerk) Mains Exams

1 Adda247 Publications For any detail, mail us at


Publications@adda247.com
Cracker Book for Bank (IBPS | SBI | RRB PO | Clerk) Mains Exams

Chapter

4 Average and Ages

BEST APPROACH TO SOLVE THE QUESTIONS

INTRODUCTION:
We all know that definition of average is sum of magnitude (weight, age,
marks or any measurable quantity) of all quantities divided by total number
of all quantities. Always remember, if you are calculating average you
are making all the quantities equal.

For eg. three brothers have number of candies with them is 9, 11 and 16.
But their mom took all the candies and distribute 12 to each. Average in
some questions referred as ‘Arithmetic mean’.
Questions from this topic are extremally easy and doesn’t require as much
time or sharp concept, mostly questions are asked in the form of average of
ages.

Example: Marks obtained by seven students are 70, 85, 95, 86, 74, 83 and
67. What is average marks obtained by them.

Solution: Sum of magnitude of all the observations is 70 + 85 + 95 + 86 +


74 + 83 + 67 = 560.
560
Required average = = 80.
7
NOTE: Please avoid short cut tricks mentioned in many books for such
small calculations. Those tricks are more time consumable. There is other
very simple and only trick that you must apply. Look at next question.

Example: Average age of 4 members of a family is 28 years and average


increases by 2 years when one of their relative joins them. Calculate the age
of relative.
2 Adda247 Publications For any detail, mail us at
Publications@adda247.com
Cracker Book for Bank (IBPS | SBI | RRB PO | Clerk) Mains Exams

Solution: Easy one! Let age of relative is x years. Total age of family before
joining the relative = 4×28=112
112+x
After joining the relative = 28+2, which gives us x = 38 years.
5
Right approach? Or wrong?
Try this approach, Imagine, they all have 28 rupees/candies/marbles
before their relative joins them. When he comes, they all have Rs 30 each.
What does it mean? It means their relative gave Rs 2 to all 4 of them and
even then, he was left with Rs 30, as average means amount is distributed
equally. How much he had given to them? Rs. 8, Right? Therefore, he
initially had total of Rs 38 with him.
Why to waste paper and time if we can do it mentally. Try next question.

Example: Average marks scored by a student in 4 subjects is 75. But when


marks of English are added to it, overall average became 70. How much did
he score in English?

Solution: Let 4 men have 75 gold coins each and when 5th one joins them,
each of those 4 men/exams gave 5 coins/marks to 5th one. And after taking
5 coins from 4 persons, 5th have total 70 coins. It means he initially had 50
coins and that the answer.

Example: Average marks scored by a student in 5 subjects was 79. But


when average is calculated for 4 subjects (without Hindi) it was 4 lesser
than the average of 5 subjects. What was the marks scored by him in Hindi?

Solution: Again let 5 men have 79 candies each and when one of them
leave, he took all candies belonging to him. It means each of these 4 had
borrowed 4 candies from him. He took 79 candies that was with him, plus
these 4 persons returned 16 of his. Therefore, he had 95 candies or marks
whatever you assume.

3 Adda247 Publications For any detail, mail us at


Publications@adda247.com
Cracker Book for Bank (IBPS | SBI | RRB PO | Clerk) Mains Exams

Example: Average run scored by Sachin in 19 innings of year 2007 is 49


runs while total run scored run scored by Afridi in 19 matches were 760. In
20th match, average of Afridi decreased by 2 runs/ match and average
run/match increased by 1 for Sachin. What is the difference b/w the scored
runs in 20th match by them?

760
Solution: Average runs by Afridi in 19 innings = 40 runs.
19
His average decreased by 2, which means all the 19 innings/persons have
given 2 runs/coins to 20th inning/person. New inning/person borrowed
19 × 2 = 38 runs/coins to become equal with others. Therefore, he himself
have no money and we can conclude Afridi scored 0 in that match.
While for Sachin, his 19 previous innings got 1 run each and even then, his
20th inning remains with 50 runs. Therefore, total runs by Sachin in 20th
inning are 69 runs.
Required difference is 69 runs.

This may look longer when you read it, but once you get the concept
you can solve these questions within 30 seconds.

AGES
The questions on ages are of two types either the average age of a group of
people is asked or we are given the ratios of ages of friends and we are
asked to calculate their individual age or average age. See it through a
question.

Example: Ratio of present age of Rahul and his father is 5:8 and 14 years
earlier this ratio was 1:2. What is present age of Rahul

Solution: 1 way to solve this is assume present age of Rahul and his father
5x and 8x. According to question 14 years earlier ratio of their age was
5x−14 1
=
8x−14 2

4 Adda247 Publications For any detail, mail us at


Publications@adda247.com
Cracker Book for Bank (IBPS | SBI | RRB PO | Clerk) Mains Exams

 (5x − 14) × 2 = 8x − 14
 2x = 14
 X=7
 Present age of Rahul = 7× 5= 35 years.
Or
If you want to solve it mentally, or in a shorter time, observe carefully
Present age ratio 5 : 8
14 years earlier 1 : 2
Note that as difference b/w ages of two persons is always same, so what we
do? We will make difference b/w two ratio’s same.
Multiply first ratio by difference of second and second ratio bye difference
of first.
First ratio when multiplied by difference of other i.e. (2-1=1) it becomes
5:8
When second ratio is multiplied by difference of first i.e. (8 - 5 = 3) we will
get
1×3 : 2× 3
3 : 6

Now think mentally, that 14 years ago age of Rahul was 3 units and now it is
5 unit.
Therefore 2 units are equal to 14 years, which means 5 unit (age of Rahul)
is 35 years.
NOTE: Concept behind multiplying by difference is that, we must make both
differences (b/w the ratios) equal. To make two numbers equal what we
do? We take their LCM as common. If difference b/w two ratios are 4 and 6,
we will multiply first ratio by 3 and second ratio by 2 to make their
difference equal. Cross check it by your self for some quantities for better
confidence.

Example: Ratio of age of two friends is 7:9 while 10 years later it will be
19:23. Calculate their average age after 7 years from present.

5 Adda247 Publications For any detail, mail us at


Publications@adda247.com
Cracker Book for Bank (IBPS | SBI | RRB PO | Clerk) Mains Exams

Solution: Difference b/w the ratios are 2 and 4. So to make them equal
multiply first ratio by 2. After multiplying we will get
14 : 18.
And second ratio we have is 19 : 23.
Hence conclude mentally, that 14 units is present age of younger friend and
19 units is his 10 years later age. It means that
19 – 1 4 =5 units = 10 year
1 unit = 2 years.
And their present average age is (28 + 36)/2 = 32 years.
And 7 years later mean 7 is added to both of their age, which means average
age is 32 + 7 = 39 years.

Key Points to Remember for New Pattern Questions


➢ Calculating average means making each quantity equal.
➢ To make difference of two ratios equal, try to think through
‘LCM of difference’ approach.
➢ If the average age of a group of people is X years, Y years
later it will be X + Y years.

6 Adda247 Publications For any detail, mail us at


Publications@adda247.com
Cracker Book for Bank (IBPS | SBI | RRB PO | Clerk) Mains Exams

Practice Exercise Based on new Pattern

1. In a group of 4 friends, ratio of present age of A and D is 4:5 and that B


to C is 3:4. Calculate the present average age of A, B and C, if 4 years
ago, A was 20 years younger than C and at present C is twice of age of
A.
(a) 20 years (b) 50 years (c) 40 years
(d) 25 years (e) 30 years

2. Ratio of age of Vikas to age of Rohit 2: 3 and average age of Rohit,


Vikas and Rahul is 23. When Arjun joins the group average age of
these 4 persons is 25 years. Calculate sum of age Arjun and Rahul, if
Rahul is 13 years older than Vikas?
(a) 40 years (b) 47 years (c) 60 years
(d) can’t be determined (e) None of these

3. S₁ is a series of five consecutive multiple of three, whose sum is 180


and S₂ is the series of four consecutive multiple of four whose second
smallest number is 13 more than second highest number of S₁ series.
Find the average of smallest number of S₁ series and highest number
of S₂ series.
(a) 51 (b) 49 (c) 47
(d) 45 (e) 43

4. A student finds the average of five two digits numbers. If One number
is reversed and the average is taken again then the average increase
by 5.4. If all five digits are consecutive multiple of four, then find the
number which is reversed?
(a) 58 (b) 36 (c) 74
(d) 48 (e) None of these

7 Adda247 Publications For any detail, mail us at


Publications@adda247.com
Cracker Book for Bank (IBPS | SBI | RRB PO | Clerk) Mains Exams

5. Average weight of a group is 20 kg. When 2 people include in the


group average weight becomes 22 kg after that when 4 another
people removed from the group, having weight half of the weight of 2
person included previously, average weight become 25. Find the
strength of group initially
(a) 18 (b) 20 (c) 22
(d) 16 (e) 14

6. Four years ago, Ratio of Ram’s to Shyam’s age is 9 : 11 and Shyam’s


age four years ago is same as Ram’s present age. If Rahul’s present age
is average of present age of Ram and Shyam, then find Rahul’s age 2
year ago.
(a) 20 (b) 18 (c) 24
(d) 22 (e) 26

7. Ratio of Ayush Age to his first son’s age is 20 : 9 and Ayush wife’s age
to Ayush 2nd son’s age is 3 : 1. If elder son of Ayush is 6 year elder than
another son (ayush has only two son and no daughter) and average
age of family is 26.5 year then find the age of Ayush’s wife.
(a) 24 (b) 36 (c) 40
(d) 38 (e) 28

8. Ritu take four, two-digit numbers and take average of these number
which is 52.5. If she reversed all the digits of 4 numbers she found
that their average is now 22.5 less than the previous one and they
formed A.P having common difference 4. Find the largest number.
(a) 82 (b) 80 (c) 36
(d) cannot be determined (e) 64

8 Adda247 Publications For any detail, mail us at


Publications@adda247.com
Cracker Book for Bank (IBPS | SBI | RRB PO | Clerk) Mains Exams

9. The average age of a group of six children is 15 years. From the group,
two children, whose ages were 3 years more and 5 years more than
the average age, left. 4 new children, whose average age is 4 years
more than the given average age, join the group. Find the new average
age.
(a) 15 years (b) 16 years (c) 17 years
(d) 18 years (e) 12 years

10. If age of P and R are added to twice the age of Q, the total becomes 59. If
the ages of Q and R are added to thrice the age of P, the total become
68. And if the age of P is added to thrice the age of Q and thrice age of
R, the total becomes 108, what is the total of ages of P, Q and R?
(a) 45 years (b) 47 years (c) 49 years
(d) 42 years (e) 44 years

11. Sweta and Neha profess to tell their present ages as 25 and 20 years
respectively. (Not original age). Ratio of their original ages 5 year ago
400
is 5 : 4. Sum of ages of both 5 years hence is % more than the sum
9
of present ages of both professed by them. Find the ratio of their
present original age.
(a) 4:7 (b) 5:6 (c) 6:5
(d) 4:5 (e) 5:7

12. Average age of a man, woman and their son is 30 years. Man’s age is
1
two year more than his wife and age of son is th the sum of age of his
4
mother and father.When two other family members were added, new
average becomes 27 years. If difference between age of two new
member are one year then find difference between son and the new
member who is elder.
(a) 7 years (b) 8 years (c) 4 years
(d) 2 years (e) 5 years

9 Adda247 Publications For any detail, mail us at


Publications@adda247.com
Cracker Book for Bank (IBPS | SBI | RRB PO | Clerk) Mains Exams

13. Present average age of A, B, C and D is 25 years. Sum of age of A and B


is 150% of sum of age of C and D. Ratio of age of B to age of C is 3 : 5.
Calculate 10 years later age of A, if B and D are of same age.
(a) 45 years (b) 40 years (c) 35 years
(d) 55 years (e) 50 years

14. Ratio of present age of A and B is 4 : 5, while that of C and D is 6 : 7. If


10 years ago Ratio of age of A and C is 1 : 2 and that of B & D is 3 : 5,
Find average of present age of A, B and C together.
(a) 27.5 (b) 25 (c) 27
(d) 26.5 (e) 24.5

1 1 1
15. The ratio of ages of three friends Veer, Sandeep and Satish is : : .
3 2 4
The average age of Veer and his wife is 33 years whereas the average
3
age of Sandeep and his wife is 39 years. The age of Satish’s wife is th
8
of the sum of age of Veer’s wife and Sandeep’s wife and the ratio of
Veer’s age to that of age of Satish’s wife is 4 : 3 then find the sum of
ages of all the three friends.
(a) 104 yr (b) 112 yr (c) 117 yr
(d) 91 yr (e) 120 yr

16. There are some teachers in a college with average age of 45 years. Sum
of maximum age and minimum age is 108 years and difference of
maximum age and minimum age is 27 years. If fourteen teachers left
the college with the average age of 32.5 years and six new teachers
joined college. Out of six new teachers four ages are equal to
maximum age teacher and two teachers age equal to minimum age
4
teacher. If new average becomes 49 years, then find the number of
7
teachers initially?
(a) 54 (b) 60 (c) 64
(d) 68 (e) 72

10 Adda247 Publications For any detail, mail us at


Publications@adda247.com
Cracker Book for Bank (IBPS | SBI | RRB PO | Clerk) Mains Exams

17. The ratio between present age of Veer & Saurbh is 3: 4 and between
present age of Divyaraj & Gopal is 2: 3. If sum of present age of Veer,
Saurbh, Divyaraj & Gopal is 128 years and ratio between sum of age of
Veer & Saurbh to sum of age of Divyaraj & Gopal is 63: 65, then find
the sum of age of Veer & Gopal after eight years.
(a) 90 years (b) 78 years (c) 86 years
(d) 84 years (e) 82 years

18. 2n years ago, the age of Raju was four times that of his son and n years
ago, the age of Raju was thrice that of his son. If n years later, the sum
of the ages of Raju and his son will be 80 years, then the difference in
the ages of Raju and his son is
(a) 20 years (b) 40 years (c) 24 years
(d) 30 years (e) 34 years

19. ‘2n’ years ago ratio of Amit’s age to Inder’s age is 5 : 4. ‘n’ years ago
ratio of Inder’s age to Satish’s age is 9 : 7. Difference between present
age of Amit to Satish’s present age is 12 years. Find the sum of present
ages of all three if ratio of Amit’s age to Satish’s age after ‘n’ year will
be 13 : 9.
(a) 81 (b) 84 (c) 87
(d) 90 (e) Cannot be determined

20. P1 is the series of four consecutive numbers divisible by 4 with an


average of 34, while P2 is the series of five consecutive numbers
divisible by 9, and whose second lowest number is equal to second
highest number of P1 series. If P3 is the series of seven consecutive
1
numbers, whose lowest number is 𝑟𝑑 𝑜𝑓 lowest number of P2 series,
3
then find the sum of average of P2 & P3 series?
(a)53 (b)51 (c)57
(d)55 (e)49

11 Adda247 Publications For any detail, mail us at


Publications@adda247.com
Cracker Book for Bank (IBPS | SBI | RRB PO | Clerk) Mains Exams

21. Average of five two digit numbers, in which 2 numbers say A and B
are reverse of each other, is 25.4. If all three numbers except A and B
multiplied by (–2) then the average of five numbers decreased by
43.2. What could be the greatest value among A and B.
(a) 32 (b) 23 (c) 50
(d) 41 (e) 14

22. Average of any 200 consecutive natural numbers is 499.5. If next 1000
numbers more add in it then find the new average.
(a) 1035.5 (b) 1299.5 (c) 1199.5
(d) 1099.5 (e) 999.5

23. Average age of a group of people is four times of the number of people
in the group. Sakshi leaves the group and the average age is still four
times of the number of people in the group. After that Sheetal leaves
the group and the average age is still four times of the number of
people in the group. If ratio between Sakshi’s age to Sheetal’s age is 21
: 19, then find the average age of the group if Ritu leaves the group
whose age is 20 years
(a) 36 years (b) 37 years (c) 38 years
(d) 39 years (e) 40 years

Directions (24-25): Rakesh distributed Rs.50000 into (X, Y, Z) 3 groups


having 50 people in total. Number of people in group ‘X’ is equal to the
number of people in group ‘Z’ and total money got by group ‘Y’ is equal to
the total money got by group ‘Z’. Average money got by group X and Y
3500
together is Rs. and average money got by group Y and Z is Rs.1000
3

24. Find the number of people in group Y?


(a) 20 (b) 16 (c) 10
(d) 18 (e) 24

25. Find the amount got by each person in group Z.


(a) 800 Rs. (b) 600 Rs. (c) 1000 Rs.
(d) 750 Rs. (e) 500 Rs.
12 Adda247 Publications For any detail, mail us at
Publications@adda247.com
Cracker Book for Bank (IBPS | SBI | RRB PO | Clerk) Mains Exams

Solutions

11. (e); Let the present age of B is 3K and that of C is 4K.


Also assume present age of A is 4m and D is 5m.
Note that age difference between A and C will always be 20
years.
4K – 4m = 20
K – m = 5 …(i)
4K
Also =2
4m
K = 2m …(ii)
m = 5 years
K = 10 years
20+30+40
∴ Average of A, B and C = = 30 years
3

2. (c); Let age of Arjun, Vikas, Rohit and Rahul is a, b, c and d


respectively
Now,
b : c = 2 : 3 …(i)
b + c + d = 23 × 3 = 69 …(ii)
a + b + c + d = 25 × 4 = 100
∴ a = 31 years.
Also d – b = 13
d=b+13
Put this value in eq. (ii)
2b + c = 56
From (i) b = 16 years, d = 29 years
c = 24 years
∴ a + d = 31 + 29 = 60 years.

13 Adda247 Publications For any detail, mail us at


Publications@adda247.com
Cracker Book for Bank (IBPS | SBI | RRB PO | Clerk) Mains Exams

3. (d); Let S₁ is a series consists 3x – 6, 3x – 3, 3x, 3x + 3, 3x + 6


ATQ,
3x – 6 + 3x – 3 + 3x + 3x + 3 + 3x + 6 = 180
⇒ x = 12
S₁ series = 30, 33, 36, 39, 42
Second smallest no. of S₂ = 39 + 13 = 52
S₂ series = 48, 52, 56, 60
30+60 90
Required average = = = 45
2 2

4. (b); Let the number which is reversed is “10a+b”


If it is reversed then the number becomes “10b+a”
ATQ,
10𝑏 + 𝑎 − 10𝑎 − 𝑏 = 5 × 5.4
⇒ 9𝑏 − 9𝑎 = 27
⇒𝑏−𝑎 =3
Numbers can be 14, 25, 36, 47, 58 and 69 but all the five digits
are multiple of 4
⇒ The number should be 36

5. (a); Let initially strength = x


ATQ,
Total weight → 20x
When two persons include
Total weight = 22 (x + 2)
Weight of two persons included = 2x + 44
4 persons excluded weight
2x+44
= = x + 22
2

ATQ,
22 (x + 2) – (x + 22) = (x + 2 – 4) × 25
x = 18
14 Adda247 Publications For any detail, mail us at
Publications@adda247.com
Cracker Book for Bank (IBPS | SBI | RRB PO | Clerk) Mains Exams

6. (d); Let age of Ram and Shyam 4 years age 9x and 11x respectively
So ATQ,
11x = 9x + 4
x=2
their present age is 22 and 26 years
Rahul’s age 2 year ago
22+26
= – 2 = 22
2

7. (b); Let Ayush Age, his wife age, his first son’s age and his second
son’s age is 20x, 3y, 9x and y respectively.
ATQ,
9x – y = 6 …(i)
20x + 3y + 9x + y = 26.5 × 4 …(ii)
Solving (i) & (ii)
x=2
y = 12
his wife’s age = 12 × 3 = 36

8. (a); Let four number is w, x, y and z after reversing they formed A.P.
Sum of all number = (52.5 – 22.5) × 4 = 120
n=4
d=4
n
Sn = [2a + (n– 1)d]
2
4
120 = [2a + (4 – 1)4]
2
a = 24
Reversed numbers
24, 28, 32, 36
Original number
42, 82, 23, 63
Largest number → 82

15 Adda247 Publications For any detail, mail us at


Publications@adda247.com
Cracker Book for Bank (IBPS | SBI | RRB PO | Clerk) Mains Exams

9. (b); Sum of the ages of 6 children


= 15 × 6 = 90
When two children left, sum of the ages of 4 children = 90 −
(18 + 20) = 52
Sum of the ages of New children
= (15 + 4) × 4 = 76
76+52
∴ Required average =
8
128
= = 16 years
8

10. (e); P + 2Q + R = 59
3P + Q + R = 68
P + 3Q + 3R = 108
Solving the equation, P = 12 years, Q = 15 years, R = 17 years.
Sum of their ages = 44 years.

11. (c); Let age of Sweta 5 years ago = 5x


Let age of Neha 5 years ago = 4x
According to question
400
(5x + 10) + (4x + 10) = (100% + %) (25 + 20)
9
13
9x + 20 = × 45
9
x=5
Sum of their present age = (5+ 4)×5+10
= 45 year + 10 years = 55 years
Required ratio = 6 : 5

12. (e); Let age of man is x years


Age of woman = (x – 2) years
(x+x–2) (x–1)
Age of son = = years
4 2
ATQ—

16 Adda247 Publications For any detail, mail us at


Publications@adda247.com
Cracker Book for Bank (IBPS | SBI | RRB PO | Clerk) Mains Exams
(x–1)
x+(x –2)+ 2
= 30
3
2x+2x–4+x –1
= 90
2
5x = 185
x = 37 years
(37–1)
son age = = 18 years
2
let age of two new members be y years and (y – 1) years
ATQ—
90+(y+y–1)
= 27
5
2y – 1 = 135 – 90
46
y=
2
y = 23
Required difference = 23 – 18 = 5 years

13. (d); If average age of A, B, C and D is 25 years, then total age of A, B, C


and D is 100 years.
Also if C + D = 2x
Then A + B = 3x [A + B is 150% of C + D]
5x = 100
x = 20
A + B = 60 …(i)
C + D = 40 …(ii)
Now, in question ratio of B : C is given as 3 : 5.
This is also ratio of D : C, as B and D are of same age. From (ii)
We can calculate
C = 25 years
D = 15 years
& Hence B = 15 years
∴ A = 45 years
10 years later age of A is 45 + 10 = 55 years.

17 Adda247 Publications For any detail, mail us at


Publications@adda247.com
Cracker Book for Bank (IBPS | SBI | RRB PO | Clerk) Mains Exams

14. (b); Let present age of A, B, C and D is 4x, 5x, 6y and 7y respectively.
Then, ATQ,
4𝑥–10 1
= ⇒ 8x – 20 = 6y – 10
6𝑦–10 2
⇒ 8x – 6y = 10 …(i)
Similarly
5x–10 3
= ⇒ 25x – 50 = 21y – 30
7y–10 5
⇒ 25x – 21y = 20 …(ii)
Solving (i) and (ii)
25𝑥 – 21𝑦 = 20
16𝑥 – 12𝑦 = 20
– + –
9𝑥 – 9𝑦 = 0
𝑥=𝑦
Put this value in equation (i), we will get x=y=5.
15 ×5 75
Therefore, average age of A, B and C together is = =
3 3
25 𝑦𝑒𝑎𝑟𝑠.

15. (a); Ratio of ages of Veer, Sandeep and Satish be 4 : 6 : 3


Let their age be 4x years, 6x years and 3x years respectively
Age of veer’s wife = (66 – 4x) years
Age of Sandeep’s wife = (78 – 6x) years
Atq,
Age of Satish’s wife
3
= × (66 − 4𝑥 + 78 − 6𝑥)
8
3
= × (144 − 10𝑥)𝑦𝑒𝑎𝑟s
8
Now,
4𝑥 4
3 =
(144−10𝑥) 3
8
⇒ 8x = 144 – 10x
⇒ 18x = 144
⇒ x = 8 years
Required sum = 8 × (4 +6 + 3) = 104 years
18 Adda247 Publications For any detail, mail us at
Publications@adda247.com
Cracker Book for Bank (IBPS | SBI | RRB PO | Clerk) Mains Exams

16. (c); Let total teachers in a college = N


Let maximum age = a year
Minimum age = b years
ATQ –
a + b = 108 ------- (i)
a – b = 27 --------(ii)
From (i) & (ii)
2a = 135 year
a = 67.5 years
Ans, b = 40.5 years
347
N× 45 − 14 × 32.5 + 4 × 67.5 + 2 × 40.5 + = (N – 14 + 6)
7
7(45N – 455 + 270 + 81) = 347N – 2776
315N − 728 = 347N – 2776
32N = 2048
2048
N=
32
N = 64

17. (e); Let present age of Veer & Saurbh be 3x and 4x respectively and
present age of Divyaraj & Gopal be 2y and 3y respectively
3𝑥+4𝑥 63
=
2𝑦+3𝑦 65
x : y = 9: 13=9k:13k
ATQ –
3x+4x+2y+3y=128
3× 9𝑘+4× 9𝑘+2×13k+3×13k=128k=128
K=1.
Therefore, age of Veer is 27 years.
Similarly, age of Gopal is 39 years.
Sum of their present ages=66years
After 8 years, sum of their ages = 66+16 = 82years.

19 Adda247 Publications For any detail, mail us at


Publications@adda247.com
Cracker Book for Bank (IBPS | SBI | RRB PO | Clerk) Mains Exams

18. (d); Let, the present ages of Raju and his son be 𝑥 and 𝑦 respectively.
2n years ago,
𝑥 − 2𝑛 = 4(𝑦 − 2𝑛)
𝑥 = 4𝑦 − 6𝑛 ……….(i)
𝑛 years ago,
𝑥 − 𝑛 = 3(𝑦 − 𝑛)
⇒ 𝑥 = 3𝑦 − 2𝑛 …………..(ii)
Solving (i) and (ii),
𝑦 = 4𝑛
And, 𝑥 = 4 × 4𝑛 − 6𝑛 = 10n
N years later,
𝑥 + 𝑛 + 𝑦 + 𝑛 = 80
⇒ 4𝑛 + 𝑛 + 10𝑛 + 𝑛 = 80
⇒ 16𝑛 = 80
⇒𝑛=5
Difference in their ages = 10𝑛 − 4𝑛 = 50 − 20 = 30

19. (d); Let, Present age of Amit, Inder and Satish be x, y and z
respectively.
ATQ,
x –2n 5
=
y –2n 4
⇒ 4x – 8n = 5y – 10n
⇒ 5y – 4x = 2n …(i)
y –n 9
=
z –n 7
⇒ 7y – 7n = 9z – 9n
⇒ 2n = 9z – 7y …(ii)
Equating (i) & (ii)
5y – 4x = 9z – 7y
12y = 9z + 4x
And, x – z = 12
x+n 13
=
z+n 9

20 Adda247 Publications For any detail, mail us at


Publications@adda247.com
Cracker Book for Bank (IBPS | SBI | RRB PO | Clerk) Mains Exams

⇒ 9x + 9n = 13z + 13n
⇒ 4n = 9x – 13z
9x – 13z = 18z – 14y
9x + 14y = 31z
Now x = 12 + z
9 (12 + z) + 14y = 31z
⇒ 22z – 14y = 108
or 11z – 7y = 54 …(iii)
and 10y – 8x = 9x – 13z
17x = 10y + 13z
17 (12 + z) = 10y + 13z
204 + 17z = 10y + 13z
4z + 204 = 10y …(iv)
On solving (iii) and (iv)
y = 30, z = 24, x =36
Required sum = 30 + 24 + 36 = 90

20. (c); Let four consecutive numbers of P₁


Series be a, (a + 4), (a + 8) and (a + 12)
ATQ—
𝑎+(𝑎+4)+(𝑎+8)+(𝑎+12)
= 34
4
4a + 24 = 136
4a = 112
𝑎 = 28
P₁ series = 28, 32, 36, 40
Let P₂ series be b, (b + 9), (b + 18), (b + 27), (b + 36)
Given,
𝑏 + 9 = 36
𝑏 = 27
P₂ series = 27, 36, 45, 54, 63
Let P₃ series be, 𝑐, (𝑐 + 1), (𝑐 + 2), (𝑐 + 3), (𝑐 + 4), (𝑐 + 5) and
(𝑐 + 6)
21 Adda247 Publications For any detail, mail us at
Publications@adda247.com
Cracker Book for Bank (IBPS | SBI | RRB PO | Clerk) Mains Exams

Given,
27
𝑐=
3
𝑐=9
P₃ series
9, 10, 11, 12, 13, 14, 15
Required average
(27+36+45+54+63) (9+10+11+12+13+14+15)
= +
5 7
225 84
= + = 45 + 12 = 57
5 7

21. (d); Let A = 10a + b


and B = 10b + a
Let sum of other three number is = X
X + 10a + b + 10b + a = 5 × (25.4) = 127 …(i)
And
–2X+10a+b+10b+a= 127 – 5 (43.2) …(ii)
Solving (i) and (ii)
We get a + b = 5
If 𝑎 would be → 0, 1, 2, 3, 4, 5
Then b would be → 5, 4, 3, 2, 1, 0
as 10a + b can’t be 50 because its reverse is 05 which is not a
two digit number.
Greatest 10a + b = 41
10a + b = 41

22. (e); Let 200 numbers are → a₁, a₂, a₅……, a₂₀₀
ATQ,
a1 +a2 +a3 +...+a200
499.5 =
200
⇒ 99900 = a1 + a2 + a3 +. . . +a200
Now,
This form an A.P.
a = a1
22 Adda247 Publications For any detail, mail us at
Publications@adda247.com
Cracker Book for Bank (IBPS | SBI | RRB PO | Clerk) Mains Exams

d=1
n = 200
200
⇒ 99900 = [2a1 + (200– 1)1]
2
a1 = 400
Required sum for 1200 terms whose a1 = 400
1200
= [2 × 400 + (1200– 1)1]
2
⇒ 1199400
1199400
Required Average = = 999.5
1200

23. (c); Let, initial Number of people in the group be ‘n’.


Let 21𝑥 𝑎𝑛𝑑 19𝑥 be ages of Sakshi and Sheetal respectively,
A.T.Q.,
4𝑛2 − 21𝑥 = 4(𝑛 − 1)2 ……..(i)
And 4𝑛2 − 21𝑥 − 19𝑥 = 4(𝑛 − 2)2
4𝑛2 − 40𝑥 = 4(𝑛 − 2)2 ………….(ii)
Solving (i) and (ii),
𝑥 = 4, 𝑛 = 11
Average age of group after Ritu leaves the group
4×112 −21×4−19×4−20 304
= = = 38
11−3 8

Solution (24-25): Let number of people in X, Y and Z group is a 𝑎, 𝑏 and 𝑎


respectively.
Let total money got by group X, Y and Z is h, k and k respectively.
Average amount got by X and Y together
ℎ+𝑘 3500
= …(i)
𝑎+𝑏 3
Average amount got by Y and Z together
𝑘+𝑘
= 1000
𝑎+𝑏
k = 500 (𝑎 + 𝑏)…(ii)
Put value of (𝑎 + 𝑏)from eq (ii) in eq (i)

23 Adda247 Publications For any detail, mail us at


Publications@adda247.com
Cracker Book for Bank (IBPS | SBI | RRB PO | Clerk) Mains Exams
ℎ+𝑘 7
=
𝑘 3
ℎ 7
+1=
𝑘 3
ℎ 4
= …(iii)
𝑘 3
We know
ℎ + 𝑘 + 𝑘 = 50,000
so, h = 20,000
and k = 15000
now ratio (i) becomes
𝑎 + 𝑏 = 30
And we know
𝑎 + 𝑏 + 𝑎 = 50
So, 𝑎 = 20
𝑏 = 10

24. (c); Number of people in Y group = 10

25. (d); Amount got by each person in group Z


15000
= = Rs. 750
20

24 Adda247 Publications For any detail, mail us at


Publications@adda247.com
Cracker Book for Bank (IBPS | SBI | RRB PO | Clerk) Mains Exams

1 Adda247 Publications For any detail, mail us at


Publications@adda247.com
Cracker Book for Bank (IBPS | SBI | RRB PO | Clerk) Mains Exams

Chapter
Percentage and Profit & Loss
5
BEST APPROACH TO SOLVE THE QUESTIONS

1
A percentage is a number or ratio that represents a fraction of 100 i.e. .
100
For faster and easy calculation one should use fractional equivalent of
2 2
percentage. For example, it’s always easier to use instead of 66 %.
3 3

It becomes easy to understand a question when one talks in terms of


percentage, but while calculating, it is always the fractional equivalent that
comes in hand.

While solving the question one must try to stay away from unnecessary
calculation. This happens in many of the questions related to profit and loss.
Many times one starts calculating the absolute values of CP, SP etc. even
when it’s not required.

Let’s take a couple of sample questions.

Example 1 : 'The CP of an item is Rs. 270. After giving a discount of Rs. 27 a


shopkeeper earns a profit of 20%. What will be the profit percentage if he
does not give the discount?’

Sol. Here, instead of finding out SP or MP, one should look at it the other
way round. If discount of Rs. 27 is not given, then Rs. 27 will become further
profit for the shopkeeper.
27 is 10% percent of 270. Profit of 20% is already there, so new profit%
would become 20% + 10% = 30%.

2 Adda247 Publications For any detail, mail us at


Publications@adda247.com
Cracker Book for Bank (IBPS | SBI | RRB PO | Clerk) Mains Exams

Example 2: ‘A man while buying cloth from a wholesaler uses a scale that
measures 10% more than the actual length and while selling the same cloth
he uses a scale that measures 10% less than the actual length. Find his
overall profit or loss in this transaction.’

Sol. Here, one just needs to deal with the percentage the shopkeeper gains.
While buying he gets 110 metre cloth on the cost of 100 metre coth and
while selling he gains the cost of 100metre just by selling 90 metre cloth.
10
Multiplication factor while buying = 1+
100
11
=
10
100−90
Multiplication factor while selling = 1+
90
10
=
9
11 10
The final multiplying factor becomes = ×
10 9
11 2
= =1+
9 9
2
which clearly shows a profit of 22 %.
9

Practice Exercise Based on New Pattern

1. Nishant bought an article at 20% discount on MRP, and claims to sell


it at profit of 10% of MRP. When Nikhil offered him Rs. 500 banknote,
he cheated again by giving him Rs. 125 instead of Rs. 225. Find overall
profit% of Nishant.
(a) 87.5% (b) 37.5% (c) 100%
(d) 62.5% (e) None of these

2. A shopkeeper marked up the price of a mobile phone by 40% of its


cost price, if he increases the discount from 5% to 10%, the profit
would decrease by 1400 Rs. How much profit shopkeeper would earn
if he gives a discount of 20% on the marked price ? Assume that, he
calculates discount only on MRP.
(a) 1800 Rs. (b) 1200 Rs. (c) 2800 Rs.
(d) 2200 Rs. (e) 2400 Rs.
3 Adda247 Publications For any detail, mail us at
Publications@adda247.com
Cracker Book for Bank (IBPS | SBI | RRB PO | Clerk) Mains Exams

3. A manufacturer of cricket balls wants to earn 25% profit on


manufacturing cost after giving a discount of 23 ⅓% on MRP marked
by him. But due to some reasons he lost 25% of balls & he decided to
9
offer discount of 7 % on MRP of remaining balls. Find his overall
13
profit% or loss%.
(a) 8% Profit (b) 12% Profit (c) 12% Loss
(d) 8% Loss (e) 5% Profit

4. Rahul bought a cycle at a discount of 16⅔% on MRP. He earned half


the amount of his CP by renting it for 200 days. After that he resells it
at half of MRP. In this transaction he earned Rs. 200, find MRP of cycle
(in Rs).
(a) 1860 (b) 2490 (c) 2400
(d) 2280 (e) 2310

5. A man purchased two items A & B and invested Rs 50 & Rs 75 on their


repairing respectively. If he earns profit of 10% on A and 12% on B,
overall profit earned by him is is Rs 84. But if he earns 20% on A &
10% on B, overall profit earned by him is 14% of total price of items.
Find initial total purchasing price of both items.
(a) Rs 675 (b) Rs 725 (c) Rs 750
(d) Rs 625 (e) Rs 775

6. A shopkeeper sold two articles, if he marked up second article at 11


1
% above first article’s selling price and gives a discount of 20% on
9
1
that, then a loss of 20% occur on first one and 33 % profit on second
3
article. Find total selling price of both articles, if he made total loss of
Rs.75 on both articles.
(a) 4000 Rs (b) 4500 Rs (c) 5100Rs.
(d) 4800 Rs (e) 5800Rs

4 Adda247 Publications For any detail, mail us at


Publications@adda247.com
Cracker Book for Bank (IBPS | SBI | RRB PO | Clerk) Mains Exams

7. Satish buy two articles i.e. type A at Rs 500 and type B at Rs 1500. He
sold type A article at x% profit and mark up type B article 2x% above
the cost price and gave x% discount at the time of the sale. By this
Satish earn (x -6) % profit. Find the value of ‘x’.
(a) 15% (b) 18% (c) 25%
(d) 20% (e) 30%

Directions (8-10): A article is mark up above cost price such that markup
percent is double of the profit percent. If discount is 12.5%, then profit
1
percent increased by 33 %.
3

8. On selling 20 such article at 12.5% discount, profit is Rs.300. Find the


M.P. of each article.
(a) Rs.60 (b) Rs.160 (c) Rs.80
(d) Rs.240 (e) Rs.72

9. If shopkeeper cheat his customer by giving 20% less quantity and


reducing value of discount percentage by 20% then find the new
profit percent.
(a) 60% (b) 75% (c) 62.5%
(d) 80% (e) 70%

10. Initially shopkeeper have 20 articles. Out of 20, 7 articles damaged


and remains unsold. Marked Price should he labeled by how much
percent more than cost price so that his overall profit does not change
neither his discount percentage.
(a) 156% (b) 146% (c) 136%
(d) 120% (e) 125%

5 Adda247 Publications For any detail, mail us at


Publications@adda247.com
Cracker Book for Bank (IBPS | SBI | RRB PO | Clerk) Mains Exams

11. Thirty percent of total employees of Bankersadda are females, and


sixty percent of female employees earn more than Rs. 17000 as
monthly salary. If fifty five percent of total employees of Bankersadda
earn more than Rs. 17000 as monthly salary. What fraction of total
male employees of Bankersadda earn Rs. 17000 or less as monthly
salary ?
13 11 51
(a) (b) (c)
20 20 100
33 69
(d) (e)
100 100

12. A shopkeeper raised the marked price of an article by 60% and allow
1 2
three successive discounts of 12 %, 14 % and 20% on new MP and
2 7
make a profit of 20%. If shopkeeper would allow only two discounts
1
of 12 % and 20% on new MP, he made a profit of Rs 800. Find the
2
cost price of article?
(a) 1800 Rs. (b) 2000 Rs. (c) 1600 Rs.
(d) 2400 Rs. (e) 2800 Rs.

13. Bankersadda started its publication with the 1500 DI books. Printing
cost per book is 125 Rs, packing cost per book is 15 Rs and shipping
cost per book is 40 Rs. if 50% of books sold on half of total cost price,
then find on how much percent above should be remaining book sold
to gets 20% profit on total outlay?
(a) 50% (b)60% (c) 80%
(d) 90% (e)70%

14. Adda247 publication sold a puzzle book in Rs. 475 and made profit of
25% on CP and sold a DI book in Rs. 575 and made a profit of 15% on
CP. If publication sold puzzle book in Rs. 360, then find what price
should DI book will be sold to made a profit of 30% on both the books
?
(a) Rs.784 (b) Rs.874 (c) Rs.847
(d) Rs.748 (e) Rs.478

6 Adda247 Publications For any detail, mail us at


Publications@adda247.com
Cracker Book for Bank (IBPS | SBI | RRB PO | Clerk) Mains Exams

Directions (15-16): Abhishek bought some chairs and tables from a


shopkeeper. The marked price of a chair and a table were in the ratio 5 : 8.
The shopkeeper gave discounts of 20% and 25% on the chair & the table
respectively. The ratio of number of chairs and tables bought by Abhishek
is 6 : 5.

15. If Abhishek sells each chair and table bought by him at discounts of
25% and 20% respectively after marking up the prices of both by 50%
and gives one table free for every four chairs bought by a customer
2
and only rd of the total chairs are sold in bunch of four chairs, then
3
what is the net profit /loss % made by Abhishek after selling all of the
items which he bought from the shopkeeper?
2 1 1
(a) 6 % (b) 3 % (c) 2 %
3 3 2
1
(d) 4 % (e) 5 %
4

16. If the marked price of a table set by the shopkeeper was Rs.300 more
than that of a chair and the total expenditure made by Abhishek in
purchasing the chairs and table from the shopkeeper was Rs.108000,
then how many chairs were purchased by Abhishek?
(a) 150 (b) 60 (c) 120
(d) 90 (e) 80

Directions (17-18): A car ‘X’ is manufactured in a company ‘A’. Cost of raw


material on a car is 25% of total cost and cost of labor is 20% of rest. 30% of
total cost occur due to four engineers working there and remaining cost is
due to machines and taxes which is in the ratio 7 : 3. Company sold the car
‘X’ at 28% profit.

17. If cost of raw material is increased by 30% and all their 4 engineers
leave the company, so they hire 2 new engineers by giving 50% more
salary as they give to previous engineer each. Find the % change in
selling price so overall profit must remain same.
(a) 0% (b) 100% (c) 2%
(d) 50% (e) 3%

7 Adda247 Publications For any detail, mail us at


Publications@adda247.com
Cracker Book for Bank (IBPS | SBI | RRB PO | Clerk) Mains Exams

18. Company A, started making car ‘Y’ in which same raw material used,
labor cost is increased by 20%. If all other cost is same then find the
ratio of selling price of car ‘X’ to car ‘Y’, given that car ‘Y’ sold at 20%
profit.
(a) 331 : 309 (b) 320 : 314 (c) 313 : 309
(d) 320 : 309 (e) 309 : 320

19. Marked price of two articles A and B are in the ratio of 7 : 9, at the
time of selling shopkeeper gives discount of d% on article A and (d +
5)% on article B and made a profit of 25% on each article, if cost price
of article A and B are in ratio 112 : 135. Then find percent of discount
given by shopkeeper on both articles?
(a) 12.5%, 17.5% (b) 25%, 30% (c) 10%, 15%
(d) 15 %, 20% (e) 20%, 25%

20. Neeraj purchased two mobile from a shop. He sold first mobile at the
price of Rs. 18750 and second mobile at the price of Rs. 14250. If the
profit percent on first mobile is five times of the loss percent on
second mobile, then find the overall profit made by Neeraj after
selling both the mobile phones. (C.P of both mobile is same)?
(a) 15% (b) 12% (c) 14%
(d) 10% (e) 18%

21. There are three societies A, B and C. Ratio of total population of A, B


and C is 2 : 3 : 5. In society A, 35% of total population is female. In
2
society B, number of females is 66 % less than number of males of
3
society B. In society C, ratio of male to female is 27 : 23. Find total
number of females is what percent less than total number of males in
all societies together ?
1
(a) 33 % (b) 40% (c) 60%
3
2
(d) 66 % (e) 75%
3

8 Adda247 Publications For any detail, mail us at


Publications@adda247.com
Cracker Book for Bank (IBPS | SBI | RRB PO | Clerk) Mains Exams

22. The marked price of a bike in two stores A and B is same.In store A
bike is available at 30% discount and in store B same bike is available
at two successive discounts of d% and 8%. A man bought bike from
store A at Rs. 49000. If man would have paid Rs. 7672 more, he could
have bought the same bike from store B. Find the discount ‘d’ allowed
by store B on bike ?
(a) 8% (b) 10% (c) 12%
(d) 16% (e) 15%

23. A shopkeeper gives a discount of 19% on marked price of article A


1
and selling price of article B is 11 % more than selling price at article
9
A. If shopkeeper made 20% profit on article B and selling price of
article A was Rs. 840 more than cost price of article B, then find the
cost price of article A. Shopkeeper sold article A at profit of 25%?
(a) 9070 Rs. (b) 9060 Rs. (c) 9040 Rs.
(d) 9020 Rs. (e) 9072 Rs.

Directions (24-25): A sale conducted on Samsung mobiles store where


Cost price of three types Samsung mobiles A, B and C in the ratio of 5 : 7 : 9.
Store owner made mark price of mobile A and C, 30% above cost price and
mobile B, 40% above cost price. Discounts allowed by store on mobiles A, B
5 3 1
and C are 15 %, 21 % 𝑎𝑛𝑑 11 % respectively.
13 7 9

24. A man purchased fourteen type A mobile, twenty type B mobile and
fifteen type C mobile in sale. If store owner made a total profit of Rs.
17500 on all type A mobile, purchased by man than find total profit
made by store owner on type B and type C mobiles, which was
purchased by man?
(a) 82500 Rs. (b) 84500 Rs. (c) 78500 Rs.
(d) 87500 Rs. (e) 88500 Rs.

9 Adda247 Publications For any detail, mail us at


Publications@adda247.com
Cracker Book for Bank (IBPS | SBI | RRB PO | Clerk) Mains Exams

25. Store decided to give no discount on purchase of any type of mobile it


will be given two mobiles free on purchase of six type A mobiles, one
mobile free on purchase of five type B mobiles and three mobiles free
on purchase of ten type C mobiles. A retailer come to purchase mobile
and take away48 type A mobiles, 36 type B mobiles and 39 type C
mobiles. Find overall loss percentage of store owner in this
transaction?(2 marks)
28 28 76
(a) 10 % (b) 8 % (c)4 %
281 281 281
28 28
(d) 14 % (e) 9 %
281 281

26. Ratio between marked price of article A to article B is 4 : 5.


Shopkeeper allowed d% discount on article ‘A’ and (d + 18)%
discount on article ‘B’, so selling price of both articles become equal. If
shopkeeper made a profit of 20% on article A and 25% on article B
and profit made on article B is Rs. 384 more than that of article A, then
find the cost price of article ‘A’ and article ‘B’ respectively?
(a) 9000 Rs. 8400 Rs (b) 9600 Rs. 9216 Rs.
(c) 9800 Rs. 9012 Rs. (d) 9600 Rs. 8488 Rs.
(e) 9200 Rs. 9216 Rs.

27. Aman started a shoes manufacturing company, manufacturing cost of


each shoes is Rs. 500 , Raw material cost is Rs 300 and transport cost
of Rs 100 If company manufactured 150 Shoes in first order and sold
50% of shoes on half of its cost price. Then find at what price of
should the company sold the remaining shoes to gets 50% profit on its
total cost?
(a) 100% (b) 150% (c) 75%
(d) 125% (e) 120%

28. The ratio between marked price of a same watch in two shops P and Q
is 7 : 5 . In shop P watch available at two successive discounts of 10%
& 25% , while in shop Q watch available at two successive discounts of
d% and 10%. If selling price of watch at shop Q is less than that of
44
selling price of watch at shop P by 𝑡ℎ of the selling price of watch at
105
shop P, then find the value of ‘d’ ?
(a) 39% (b) 29% (c) 31%
(d) 35% (e) 37%
10 Adda247 Publications For any detail, mail us at
Publications@adda247.com
Cracker Book for Bank (IBPS | SBI | RRB PO | Clerk) Mains Exams

29. Cost price of a Samsung mobile is 20% more than cost price of a Mi
mobile. Shopkeeper marked up Samsung mobile at 25% above its cost
price and Mi mobile at 40% above cost price. If shopkeeper allowed
discount of 20% on Mi mobile and 25% on Samsung mobile and
difference between selling price of Samsung mobile and Mi mobile
was 160 Rs. then find sum of marked price of one Samsung mobile
and one Mi mobile?
(a) 92840 Rs (b) 92260 Rs (c) 98460 Rs
(d) 92800 Rs (e) 98530 Rs

30. In a city 60 percent are male and remaining are females. Out of total
males, 20% of male are government employees. Out of remaining
males, ratio between private employees to unemployed male is 5 :7.
Out of total females, 40% of female are in government jobs and out of
remaining females 40% of female work for private firms and
remaining are unemployed. Find the total number of male and female,
who are government employees, if difference between male and
female who are unemployed is 3400?
(a) 6000 (b) 5000 (c) 7000
(d) 4000 (e) 3000

31. A person ‘A’, invested 37½% of his savings in scheme XYZ, while B
invested 62½% of his savings in the same scheme. If B invested 50%
of his remaining savings in scheme PQR, and A thought to invest
double of the amount invested by B in PQR scheme then find what
percent of A’s remaining savings after investment in XYZ, A should
invest in PQR. Given that amount invested by B in XYZ is twice the
amount invested by A in same scheme.
(a) 80% (b) 68% (c) 64%
(d) 72% (e) 60%

11 Adda247 Publications For any detail, mail us at


Publications@adda247.com
Cracker Book for Bank (IBPS | SBI | RRB PO | Clerk) Mains Exams

32. A man bought few kg’s of sugar of 3 variety (X, Y, Z) in the ratio of
their quantity 2 : 3 : 5 respectively and started to sell them at profit of
20%, loss of 10% and profit of 32% respectively. He managed to sell
only 80% of total sugar and rest was return at the same price as he
bought. The price of these sugar is Rs. 30/kg, Rs. 40/kg and Rs. 50/kg
respectively. Calculate his total profit amount, if total sugar bought
was 50 kg and he sold all the sugar of X and Y types.
(a) 320 (b) 300 (c) 0
(d) 180 (e) None of these

33. In an election survey, 83 ⅓% of total voters took part in survey, 50%


then claims to votes for candidate A, 10% are uncertain & rest says to
vote for B. If all of them voted according to their commitments on the
day of election and those who were not part of survey, voted to A and
B in ratio of 2 : 1. All the people who are uncertain in survey, voted to
A : B in the ratio of 1 : 4 in election. If A won by 640 votes then find
total number of votes in election.
(a) 6000 (b) 5000 (c) 9000
(d) 7200 (e) 6750

2
34. In a village 60% are males in which 6 % males are above 60 years, in
3
2
remaining 14 % males are below 18 years, out of remaining
7
1
62 % are above 18 years but below 50 years and remaining males are
2
above 50 years but below 60 years. If difference between males above
60 years and males above 18 years but below 50 years are 7800, then
find total female population of village?
(a) 16000 (b) 18000 (c) 20000
(d) 12000 (e) 24000

12 Adda247 Publications For any detail, mail us at


Publications@adda247.com
Cracker Book for Bank (IBPS | SBI | RRB PO | Clerk) Mains Exams

35. Divyaraj purchased jeans and shirt from a seller. Marked price of
2
jeans and shirt are in the ratio of 9 : 7 and seller offered 14 %
7
1
discount on shirt and 11 % on jeans and number of jeans and shirt
9
purchased by Divyaraj are in the ratio of 5 : 8. If Divyaraj marked up
jeans and shirt 50% above their cost price and offered 25% discount
1
on jeans and 11 % on shirt, Find overall profit of Divyaraj in this
9
transaction.
19 19 19
(a) 22 % (b) 21 % (c) 19 %
22 22 22
19 19
(d) 23 % (e) 25 %
22 22

36. In 2016 a store sold some mobile phone at certain price. In 2017 store
increase selling of mobile phone by reducing the price of mobile
phone by 25%. The total revenue generated by selling mobile phone
in 2017 is increased by 5% compared to previous year. The total
revenue generated by store in 2017 is 94.5 lakh and number of mobile
phones sold by store in the year 2017 is 90 more than that in 2016.
Find initial cost of one mobile.
(a) 32000 Rs. (b) 40000 Rs. (c) 36000 Rs.
(d) 24000 Rs. (e) 20000 Rs.

37. Veer bought some Shirts & some Jeans from a store. The marked price
of a shirt and a jeans is in the ratio 5 : 7 and store allows discounts of
20% and 25% on the shirt & the jeans respectively. The ratio between
number of shirt and jeans bought by Veer is 9 : 8. Veer marking up the
prices of both by 50% on the price at which he bought and sells each
shirt and jeans bought by him at discounts of 25% and 20%
respectively. If Veer gives one 1 shirt free for every 4 jeans bought by
a customer, then what is the net profit made by Veer after selling all of
the items which he bought from the store?
(a) 2% (b) 7% (c) 6%
(d) 5% (e) 8%

13 Adda247 Publications For any detail, mail us at


Publications@adda247.com
Cracker Book for Bank (IBPS | SBI | RRB PO | Clerk) Mains Exams

38. A shopkeeper has two types Wheat i.e Type A & Type B and cost price
per kg of Type A wheat is 10 times of cost price per kg of type B
wheat. Shopkeeper cheated a costumer by saying that he will give him
200 kg of Type A wheat but shopkeeper gives 200 kg of mixture of
Type A & Type B wheat on 10% above the cost price per kg of Type A
7
wheat. If shopkeeper made a total profit of 71 % in this transaction,
8
then find ratio between quantity of Type B wheat to Type A wheat in
sold mixture?
(a) 2 : 5 (b) 2 : 3 (c) 1 : 3
(d) 2 : 7 (e) 1: 5
1
39. In a store marked price of a shirt is 33 % less than marked price of a
3
jeans. There are two discounts schemes A & B. In Scheme A, if
someone buy one jean and one shirt together, then store gives overall
discount of 35%, while in scheme B if someone buy one shirt and one
jeans separately, store give 20% discount on shirt and 25% on jeans.
If difference between selling price of one jeans and one shirt in
scheme A and B is Rs. 384 then find cost price of one jeans & one shirt,
1
given that store made of 33 % on shirt and 25% on jeans, when it
3
sold one jeans and one shirt in scheme B ?
(a) 720 Rs, 1150 Rs (b) 768 Rs. 1152 Rs.
(c) 786 Rs. 1168 Rs. (d) 796 Rs. 1144 Rs.
(e) 790 Rs. 1108 Rs.

40. The Hero company manufactured cycles, 40% of the total


manufactured cost is on raw material, 20% on labour charges, 20% on
fixed charges and the rest on transportation. The cycle is sold at a
profit of 25%. The price of the raw material increased by 15% and the
labour charges increased by 20% and the cost on the transportation
increased by 50% while the fixed costs remained unchanged. If the
7
manufacturer wants a 13 % profit, then by what percentage should
11
company reduced its expenditure on raw materials (at the increased
price), the selling price remaining the same ?
17 17 17
(a) 19 % (b) 21 % (c) 19 %
23 23 23
17 17
(d) 23 % (e) 27 %
23 23

14 Adda247 Publications For any detail, mail us at


Publications@adda247.com
Cracker Book for Bank (IBPS | SBI | RRB PO | Clerk) Mains Exams

Solutions

1. (a); Nikhil gave Rs. 500 note, & Nishant have to actually pay him Rs.
225. Hence Nishant sold it at Rs. 500 – 225 = 275 to Nikhil.
Now, Let MRP is 100x.
If he will get 10% profit, then SP will becomes 110x.
110x = 275
x = 5.5
∴ MRP is Rs. 250
80
And CP for Nishant is × 25 = Rs. 200.
100

He got Rs. (500 – 125)


= Rs. 375 from Nikhil.
375–200
Hence Profit % = ( ) × 100
200
7
= × 100 = 87.5%
8

2. (e);
CP of Mobile M.P. mobile S.P. of mobile
100% 140% 133%(1st discount) (133 – 126)% = 1400
126%(2nd discount)
7% = 1400
1400
C. P. = × 100
7
= 20000 Rs.
112
S. P. after 20% discount = 20000 ×
100
= 22400
Profit = 2400 Rs.

15 Adda247 Publications For any detail, mail us at


Publications@adda247.com
Cracker Book for Bank (IBPS | SBI | RRB PO | Clerk) Mains Exams

3. (b); Let he manufacture 𝑛 balls and manufacturing cost of each ball


is 𝑦
And 𝑥 is marked price.
Now, ATQ
7 5
𝑥 [1– ] = 𝑦 ×
30 4
23 5
⇒𝑥× =𝑦×
30 4
5 30 75
⇒𝑥 =𝑦× × =𝑦× …(i)
4 23 46
3 1 12
After accident he sells 𝑛 balls on a price of 𝑥 (1– ) = 𝑥 ×
4 13 13
3 3 12
Total SP of 𝑛 balls = 𝑛. 𝑥.
4 4 13
3 75 12
= ( × 𝑛) (𝑦 × )( )
4 46 13
675
= 𝑛𝑦 ×
598
675
𝑛𝑦×598−𝑛𝑦
Hence profit % = × 100
𝑛𝑦
77
= × 100 ≅ 12% profit
598

4. (c); Let the MRP of cycle is 6𝑥.


50
∴ discount is % of 6x = x
3
∴ Rahul bought this cycle for 5x
He earns 2.5x by renting it and resells it at 3x.
ATQ
3x + 2.5x – 5x = 200
0.5x = 200 ⇒ x = 400
∴ MRP = 6x = 6 × 400 = Rs. 2400
16 Adda247 Publications For any detail, mail us at
Publications@adda247.com
Cracker Book for Bank (IBPS | SBI | RRB PO | Clerk) Mains Exams

5. (d); Let the purchasing price of items A is B be Rs x – 50, Rs y – 75


respectively.
When he applied Rs 50 & Rs 75 on them, their actual CP
becomes x & y
ATQ
10 12
x+ y = 84 …(i)
100 100
Also,
20 10 14
x+ y= (x + y)
100 100 100
⇒ 20x + 10y = 14x + 14y
⇒ 6x = 4y
2
x= y
3
Put this value in equation (i)
10 (2y) 12
+ y = 84
100 3 100
20y+36y
= 84
300
84×300
⇒y= = Rs 450
56
2
x = 450 × = Rs 300
3
Initial purchasing price is
= (300 + 450) – (75 + 50)
= Rs 625

6. (c); Let marked price of second article be


10x Rs.
Selling price of first article be 9x Rs.
Selling price of second article
8
= 10x × = 8x Rs.
10

C.P. of fisrt article


9x
= × 5 = 11.25x Rs.
4

17 Adda247 Publications For any detail, mail us at


Publications@adda247.com
Cracker Book for Bank (IBPS | SBI | RRB PO | Clerk) Mains Exams
3
C.P. of second article = 8𝑥 × = 6x Rs.
4

Total C.P. of both articles = 11.25x + 6x


= 17.25x Rs.
Total S.P. of both articles
= 9x + 8x = 17x Rs.
ATQ—
17.25x – 17x = 75
75
x= = 300
0.25

selling price of both articles = 17 × 300


= 5100 Rs.

7. (d); ATQ
𝑥 2𝑥 𝑥 𝑥−6
500 [ 1 + ] + 1500 [1 + ] [1 − ] = 2000 [1 + ]
100 100 100 100
2𝑥 𝑥 2𝑥 2 𝑥−6
500 + 5𝑥 + 1500 [1 + − − ] = 2000 [1 + ]
100 100 100×100 100
3𝑥 2
500 + 5𝑥 + 1500 + 15𝑥 − = 2000 + 20(𝑥 − 6)
10
3𝑥 2
2000 + 20𝑥 − = 2000 + 20𝑥 − 120
10
3𝑥 2
= 120
10

x² = 400
⇒ x =20%

18 Adda247 Publications For any detail, mail us at


Publications@adda247.com
Cracker Book for Bank (IBPS | SBI | RRB PO | Clerk) Mains Exams

Solutions (8-10): Let cost price = 100


Profit percent = 𝑥%
So, selling price ⇒ 100 + 𝑥
M.P. ⇒ 100 + 2𝑥
Now if discount is 12.5%
1 4 4
Then profit percentage increases by 33 % ⇒Profit percent = 𝑥 × = 𝑥
3 3 3
ATQ,
4 7
100 + 𝑥 = (100 + 2𝑥)
3 8
𝑥 = 30
So, if cost price = 100
Selling price = 130
M.P. = 160
Profit = 30
30
Discount % = × 100 = 18.75%
160

8. (c); Profit on one articles = 30


Profit on 20 articles = 30 × 20 ⇒ 600
600 → 300
1 → 0.5
M.P. → 160 → 160 × 0.5 ⇒ Rs.80

9. (e); Let total quantity ⇒ 1000


He gives → 800 → for → 100
Cost price of → 800 → 80
30
Now initial discount = × 100 = 18.75%
160
18.75×4
Reduced discount = = 15%
5
160×85
S.P. = = 136
100
(136–80)
Profit = × 100 = 70%
80

19 Adda247 Publications For any detail, mail us at


Publications@adda247.com
Cracker Book for Bank (IBPS | SBI | RRB PO | Clerk) Mains Exams

10. (b); Let cost price of 20 articles → 20 × 100


= 2000
Actual profit on 20 articles → 20 × 30
= 600
S.P. → 2600
S.P. of each undamaged article
2600
⇒ = 200
20–7
30
Initial discount = × 100 = 18.75%
160

So M.P. of each article should be


200
= × 100 ≈ 246
81.25

Approximately markup% = 146%

11. (d); Let total employee in Bankersadda = 100


Females employee = 30
Male employee = 70
Employee who earned more than Rs. 17000 as monthly salary
= 55
Employee who earned less than Rs. 17000 as monthly salary
= 45
Total female employee earned less than
Rs. 17000 as monthly salary
40
= 30 × = 12
100

Total male employee earned Rs. 17000 or less as monthly salary


= 45-12 = 33
33
Required fraction =
100

20 Adda247 Publications For any detail, mail us at


Publications@adda247.com
Cracker Book for Bank (IBPS | SBI | RRB PO | Clerk) Mains Exams

12. (b); Let original marked price be Rs 100x.


Then, New marked price of article
= Rs 160x.
Selling price of article
7 6 4
= 160𝑥 × × ×
8 7 5

= 96x Rs.
5
C.P. of article = 96x ×
6

= 80x Rs.
2nd selling price
7 4
= 160𝑥 × × = 112x Rs.
8 5

Given
112x – 80x = 800
32x = 800
x = 25
cost price = 80 × 25 = 2000 Rs.

13. (d); Total CP of one DI book = 125 + 15 + 40


= 180 Rs.
C.P. of 1500 books = 1500 × 180
750×180
SP of 750 book =
2

Let required profit percent = x%


750×180 100+x
∴ +( ) × (750 × 180)
2 100
120
= × 180 × 1500
100

21 Adda247 Publications For any detail, mail us at


Publications@adda247.com
Cracker Book for Bank (IBPS | SBI | RRB PO | Clerk) Mains Exams

67500 + 135000 + 1350x = 324000


1350x = 324000 – 202500
121500
x= = 90%
1350
475
14. (a); CP of puzzle book = × 100
125

= 380 Rs.
575
CP of DI book = × 100
115

= 500 Rs.
For 30% profit on both
130
= (380 + 500) × = 1144
100

DI book should be sold


= 1144 – 360 = 784 Rs.

15. (b); Let the MP of a chair and a table be Rs.5x and Rs.8x respectively.
And, the number of chairs and tables bought be 6y and 5y
respectively.
CP of a chair for Abhishek = (100 – 20)% of 5x = Rs.4x
CP of a table for Abhishek = (100 – 25)% of 8x = Rs.6x
Total CP for Abhishek = 4x × 6y + 6x × 5y = 24xy + 30xy = 54xy
SP of a chair for Abhishek = (100 – 25)% of (100 + 50)% of 4x
= 4.5x
SP of a table for Abhishek = (100 – 20)% of (100 + 50)% of 6x
= 7.2x
Number of chairs sold in bunch of four by Abhishek

22 Adda247 Publications For any detail, mail us at


Publications@adda247.com
Cracker Book for Bank (IBPS | SBI | RRB PO | Clerk) Mains Exams
2
= rd of 6y = 4y
3
So, number of table sold for free by Abhishek
1
= th of 4y = y
4
Total SP for Abhishek = 4.5x × 6y + 7.2x × (5y – y) = 27xy +
28.8xy = 55.8xy
55.8xy – 54xy
Profit % = × 100
54xy
1.8xy 1
= × 100 = 3 %
54xy 3

16. (c); According to the question,


MP of a table = 300 + MP of a chair
⟹ 8x = 300 + 5x ⟹ x = 100
Total CP for Abhishek = 108000
⟹ 54xy = 108000
⟹ 54 × 100 × y = 108000 ⟹ y = 20
Number of chairs purchased by Abhishek
= 6y = 120

Solutions (17-18): Let total cost of car X = 100𝑥


Cost of raw material = 25𝑥
(100–25𝑥)×20
Cost of labor = = 15𝑥
100

Cost of engineers = 30𝑥


Cost of machine = 21𝑥
Cost of taxes = 9𝑥
Profit = 28𝑥
So, proposed selling price = 128𝑥
23 Adda247 Publications For any detail, mail us at
Publications@adda247.com
Cracker Book for Bank (IBPS | SBI | RRB PO | Clerk) Mains Exams
25𝑥×13
17. (a); New cost of raw material = = 32.5𝑥
100
30𝑥
Now cost of each engineer = = 7.5𝑥
4
150
Cost on new engineer = 2 × 7.5𝑥 ×
100

= 22.5
So new CP = 100𝑥 + 7.5𝑥 – 7.5𝑥 = 100𝑥
So, % change in S.P. = 0%

15𝑥×120
18. (d); Cost on labor for car Y = = 18𝑥
100

Cost price of car Y = 103𝑥


128𝑥 320
Required ratio = =
103𝑥×120 309

19. (e); Let cost price of article A Rs. 112x and cost price of article B Rs.
135x
ATQ,
112𝑥
Selling price of article A = ×5
4

= 140x Rs.
135𝑥
Selling price of article B = ×5
4

= 168.75x Rs.
Let mark price of article A is 7y
And article B is 9y
100−𝑑
140x = 7y ×( ) ________(I)
100
100–(𝑑+5)
168.75x = 9y ×( ) ________(II)
100

24 Adda247 Publications For any detail, mail us at


Publications@adda247.com
Cracker Book for Bank (IBPS | SBI | RRB PO | Clerk) Mains Exams

From (I) and (II) _________


100–𝑑
140𝑥 7𝑦×( 100 )
= 95–𝑑
168.75𝑥 9𝑦×( 100 )

140×9 100–𝑑
=
168.75×7 95–𝑑
16 100–𝑑
=
15 95–𝑑

16d – 1520 = 15d – 1500


First discount d = 20%
Second discount = (20 + 5) = 25%

20. (d); Let cost price of each mobile = 100x Rs.


Percent profit on first mobile
18750−100𝑥 18750−100𝑥
= [( )] × 100 =
100𝑥 𝑥

Loss percent on second mobile


100𝑥−14250 100𝑋−14250
= [( )]× 100 =
100𝑥 𝑥

ATQ –
18750−100𝑥 100𝑋−14250
=5×
𝑥 𝑥

18750 − 100𝑥 = 500𝑥 − 71250


600x = 90000 ⇒ x = 150 Rs.
Total cost price of both mobile = 2 × 15000 = 30000 Rs.
Total selling price of both mobile = 18750 + 14250 = 33000 Rs.
33000−30000
Profit percentage = ×100 = 10%
30000

25 Adda247 Publications For any detail, mail us at


Publications@adda247.com
Cracker Book for Bank (IBPS | SBI | RRB PO | Clerk) Mains Exams

21. (b); Let population of society A, B and C be 200x, 300x and 500x
respectively
35
Female in society A = × 200x = 70x
100

Male in society A = 200x – 70x = 130x


In society B Let ‘a’ = Number of males
ATQ,
1 4𝑎
a + a = 300x ⇒ = 300𝑥
3 3

⇒ a = 225x = Number of males


Number of females = 300x – 225x = 75x
In society C,
23
Number of females = × 500x = 230x
50
27
Number of males = × 500x = 270x
50

Total number of female = 70x + 75x + 230x


= 375x
Total number of male = 130x + 225x + 270x = 625x
625x –375x
Required% = × 100
625x
250
= × 100 = 40%
625

22. (c); Let marked price of bike = 100x Rs.


Selling price for store A
(100 –30)
= 100x × = 70x
100

Given, 70x = 49000 ⇒ x = 700


marked price of bike = 70000 Rs.
26 Adda247 Publications For any detail, mail us at
Publications@adda247.com
Cracker Book for Bank (IBPS | SBI | RRB PO | Clerk) Mains Exams

ATQ—
Selling price of bike in store B=
(100−𝑑) (100−8)
Marked price of bike× ×
100 10

= 𝑠𝑒𝑙𝑙𝑖𝑛𝑔 𝑝𝑟𝑖𝑐𝑒 𝑜𝑓 𝑏𝑖𝑘𝑒 𝑖𝑛 𝑠𝑡𝑜𝑟𝑒 𝐴+7672


(100 –d) (100 –8)
70000 × ×
100 100

= (49000 + 7672)
644d = 64400 – 56672
7728
d= ⇒ d = 12 %
644

23. (e); Lets MP of article A = 100x Rs.


Selling price of article A
(100−19)
= 100x × = 81x Rs.
100
10
Selling price of article B = 81𝑥 ×
9

= 90x Rs.
90𝑥
Cost price of article B = × 100 = 75x
120
81𝑥
Cost price of article A = × 100
125

= 64.8x Rs.
Given,
81x – 75x = 840
6x = 840 ⇒ x = 140 Rs.
Cost price of article A = 64.8 × 140
= 9072 Rs.

27 Adda247 Publications For any detail, mail us at


Publications@adda247.com
Cracker Book for Bank (IBPS | SBI | RRB PO | Clerk) Mains Exams

Solutions (24-25): Lets cost price of A, B and C type of mobile be Rs. 5x, Rs.
7x and Rs. 9x respectively
130
M.P. of type A mobile = 5𝑥 × = 𝑅𝑠. 6.5𝑥
100
140
M.P. of type B mobile = 7𝑥 × = 𝑅𝑠. 9.8𝑥
100
130
M.P. of type C mobile = 9𝑥 × = 𝑅𝑠. 11.7𝑥
100

S.P. of type A mobile


200 1 11
= 6.5𝑥 × (100– )× = 6.5𝑥 × = 𝑅𝑠. 5.5𝑥
13 100 13

S.P. of type B mobile


150 1
= 11.7𝑥 × (100– )× = 𝑅𝑠. 7.7𝑥
7 100
8
S.P. of type C mobile = 11.7 × = 𝑅𝑠. 10.4𝑥
9

24. (d); Total cost price of Fourteen type A mobile


= 14 × 5x = Rs. 70x
Total selling price of fourteen type A mobile = 14 × 5.5x = 77x
Given total profit = Rs. 17500
7x = 17500 ⇒ x = Rs. 2500
Total profit made by store owner on type B and type C mobile
= (7.7 × 20 – 7× 20) × 2500 + (10.4 × 15 – 9 × 15) × 2500
= 14 × 2500 + 21 × 2500
= 35000 + 52500 = Rs. 87500

28 Adda247 Publications For any detail, mail us at


Publications@adda247.com
Cracker Book for Bank (IBPS | SBI | RRB PO | Clerk) Mains Exams

25. (c); In the transaction —


Given,
In 48 type A mobile, 12 mobile are free.
In 36 type B mobile, 6 mobile are free.
In 39 type C mobile, 9 mobile are free.
Total profit of store owner on selling of 48 type A mobile
= 48 × 5x – 36 × 6.5x
= 240x – 234x = 6x
Total loss of store owner on selling of 36 type mobile
= 36 × 7x – 30 × 9.8x
= 252x – 294x = 42x
Total loss of store owner on selling of 39 type C mobile
= 9x × 39 – 11.7x × 30 = 351x – 351x = 0
Total loss of store owner in this transaction = 42x- 6x = 36x
Total cost price = 48 × 5x + 36 × 7x + 39 × 9x
= 240x + 252x + 351x = 843x
36𝑥 76
Required% = × 100 = 4 %
843𝑥 281

26. (b); Let marked price of article A and B be 400x and 500x
respectively
ATQ—
(100–d) (100–d−18)
400x × = 500x ×
100 100

400 – 4d = 410 – 5d ⇒ d = 10%


90
400x×100
Cost price of article A = × 100
120
29 Adda247 Publications For any detail, mail us at
Publications@adda247.com
Cracker Book for Bank (IBPS | SBI | RRB PO | Clerk) Mains Exams

= 300x Rs.
(100–28)
500x× 100
Cost price of article B = × 100
125

= 288x Rs.
ATQ—
72 90
(500x × – 288x) – (400x × – 300x) = 384
100 100

72x – 60x = 384 ⇒ x = 32


Cost price of article A = 32 × 300 = Rs.9600
Cost price of article B = 32 × 288 = Rs.9216

27. (b); Total CP of one shoes = (500 + 300 + 100)


= 900
CP of 150 shoes = 900 × 150
900×75
CP of 75 shoes =
2

Let’s required percentage =x


900×75 100+𝑥
∴ +( ) × (900 × 75)
2 100
150
= × 900 × 150
100

= 33750 + 67500 + 675x = 202500


= 675x = 202500 – 101250
675x = 101250
101250
𝑥= ⇒ x = 150%
675

30 Adda247 Publications For any detail, mail us at


Publications@adda247.com
Cracker Book for Bank (IBPS | SBI | RRB PO | Clerk) Mains Exams

28. (a); Let marked price of watch at shop P & Q be Rs. 140x and Rs.
100x respectively
Selling price of watch at shop P
90 75
= 140𝑥 × × = 94.5𝑥 𝑅𝑠.
100 100

Selling price of watch at shop Q


(100−𝑑) 90
= 100x × ×
100 100

ATQ –
(100−𝑑) 90 44
94.5x - 100x × × = 94.5𝑥 ×
100 100 105

9450 – 9000 + 90d = 3960


90d = 3510
3510
d= ⇒ d = 39%
90

29. (d); Let cost price of one Mi mobile be Rs. 100x and cost price of one
Samsung mobile be Rs.120x
Marked price of Samsung mobile
125
= 120x × = 150x Rs.
100

Marked price of Mi mobile


140
= 100x × = 140x Rs.
100

Selling price of Samsung mobile


75
= 150x × = 112.5 x Rs.
100
80
Selling price of Mi mobile = 140x ×
100

= 112x Rs.

31 Adda247 Publications For any detail, mail us at


Publications@adda247.com
Cracker Book for Bank (IBPS | SBI | RRB PO | Clerk) Mains Exams

ATQ–
112.5 x – 112x = 160 ⇒ x = 320 Rs.
Marked price of one Samsung & one Mi mobile
= (150 × 320) + ( 140 × 320)
= 48000 + 44800 = 92800 Rs.
30. (c);

Male 60% Female 40%

Government Employees Government employees


20 40
= 60 × = 12% = 40 × = 16%
100 100

Private employees Private employees


5 40
= 48 × = 20% = 24 × = 9.6%
12 100

Unemployed = 48 – 20 =28% Unemployed = 24 – 9.6


= 14.4%
Given —
28% – 14.4% = 3400 ⇒ 13.6% = 3400
Government (male + female) employees
= 12% + 16% = 28%
3400
Government (male + female) employees = × 28 = 7000
13.6

31. (d); Let the savings of A & B be Rs 8 x and Rs 8y respectively. Then


37.5
amount invested by A in XYZ scheme = × 8𝑥 = 3x
100
62.5
Amount invested by B in XYZ = 8y = 5y
100

ATQ,

32 Adda247 Publications For any detail, mail us at


Publications@adda247.com
Cracker Book for Bank (IBPS | SBI | RRB PO | Clerk) Mains Exams

⇒ 6x = 5y
5 6
x = y or y = x
6 5
1
Now B invested 50% of (8y – 5y) = × 3y
2

= 1.5y in scheme PQR.


A wants to invest 2× 1.5y = 3y
6 18 𝑥
=3× x=
5 5

His remaining savings = 8x – 3x = 5x


18x
5
% he could invest in PQR = × 100
5x
1800
= % = 72%
25

32. (e); He bought total of 50 kg sugar.


And ratio of quantity of both is given as 2:3:5. Therefore, he have
10 kg of X, 15 kg of Y and 25 Kg of Z.
And cost price of X, Y & Z is Rs. 30/kg, Rs. 40/kg and Rs. 50/kg
20
Profit earn on 1kg of X is × 30 = Rs. 6
100
10
Loss earn on 1 kg of Y is × 40 = Rs. 4
100

Profit earn on 1 kg of Z is 32/100] × 50


= Rs. 16
But he sold all of X & Y and 40 – (10 + 15) =15 kg of Z.
∴ Total amount of profit earned is
6 × 10 – 4 × 15 + 16 × 15
= 60 – 60 + 240 = Rs. 240

33 Adda247 Publications For any detail, mail us at


Publications@adda247.com
Cracker Book for Bank (IBPS | SBI | RRB PO | Clerk) Mains Exams

33. (d); Let total votes be 6𝑥


People who took part in survey
= 83⅓% × 6𝑥 = 5𝑥
2.5𝑥 claims to vote for A,
0.5𝑥 are uncertain.
People who didn’t took part in survey
= 6𝑥 – 5𝑥 = 𝑥
2 2
And vote for A= ×𝑥 = 𝑥
3 3

Vote for A, from people who are uncertain.


1
= × 0.5𝑥 = 0.1𝑥
5
2
Total votes for A = 2.5x + 0.1x + 𝑥
3
25 1 20
= x+ x+ x
10 10 30
75𝑥+3𝑥+20𝑥 98𝑥
= =
30 30
98𝑥 82𝑥
Votes for B = 6𝑥– =
30 30

ATQ,
98𝑥 82𝑥 16𝑥 16𝑥
– = ⇒ = 640
30 30 30 30

⇒ 𝑥 = 40 × 30 = 1200
Hence total votes are 6 × 1200 = 7200

34. (d); Let total population of village = 100x


Male population = 60x
1
Male above 60 year = 60x × = 4x
15

34 Adda247 Publications For any detail, mail us at


Publications@adda247.com
Cracker Book for Bank (IBPS | SBI | RRB PO | Clerk) Mains Exams
1
Male below 18 years = (60x – 4x) × = 8x
7

Male above 18 but below 50 years


5
= (56x – 8x) × = 30x
8

Male above 50 year but below 60 years


= (56x – 8x – 30x) = 18x
ATQ—
30x – 4x = 7800
26x = 7800 ⇒ x = 300
Female population in village
= 300 × 40 = 12000

35. (d); Let marked price of Jeans be Rs. 9x and shirt be Rs. 7x for
Divyaraj
8
CP of jeans for Divyaraj = 9x × = 8x Rs.
9
6
CP of shirt for Divyaraj = 7x × = 6x Rs.
7

Let total number of jeans purchased by Divyaraj be 5y and shirt


be 8y
Total cost price for Divyaraj
= 8x × 5y + 6x × 8y
= 88xy
SP of jeans on, which Divyaraj sold
3 3
= 8x × × = 9x
2 4

SP of shirt on, which Divyaraj sold

35 Adda247 Publications For any detail, mail us at


Publications@adda247.com
Cracker Book for Bank (IBPS | SBI | RRB PO | Clerk) Mains Exams
3 8
= 6x × × = 8x
2 9
Total S.P. on which Divyaraj sold all items
= 9x × 5y + 8x × 8y
= 45xy + 64xy = 109xy
109xy –88xy
Required% = × 100
88xy
19
= 23 %
22

36. (b); Let cost of each mobile be Rs. ‘a’ in 2016 and number of mobile
sold in 2016 be ‘b’
Total revenue generated by store in 2016
= ab Rs.
In 2017 —
Cost of each mobile = 0.75a Rs.
Total revenue generated by store in 2017
105
= × ab = 94.5 lakh
100

ab = 90 lakh …(i)
Total number of mobile sold by store in 2017
1.05ab
= = 1.4b
0.75a

Given,
Number of mobile sold in 2017 – number of mobile phone sold
in 2016 = 90
1.4b – b = 90
b = 225
Cost price of each mobile in 2016
90,00,000
= = 40000 Rs.
225

36 Adda247 Publications For any detail, mail us at


Publications@adda247.com
Cracker Book for Bank (IBPS | SBI | RRB PO | Clerk) Mains Exams

37. (d); Let the marked price of a Shirt = 500 Rs


And the marked price of a Jeans = 700 Rs
Also, no. of shirt bought by Veer = 9x
And no. of jeans bought by Veer = 8x
4
C.P. of shirt for Veer = × 500 = 400 Rs.
5
3
C.P. of jeans for Veer = × 700 = 525 𝑅𝑠.
4

Total C.P. of shirt and jeans for Veer


= 9𝑥 × 400 + 8𝑥 × 525 = 7800𝑥 Rs.
Total S.P. of shirt and jeans for Veer
= 7𝑥 × 400 × 1.5 × 0.75 + 8𝑥 × 525 × 1.5 × 0.8
= 3150𝑥 + 5040𝑥
= 8190𝑥 Rs.
8190𝑥−7800𝑥
Profit % = × 100
7800𝑥
390
= × 100 = 5%
7800

38. (b); Let cost price per kg of Type B wheat = x Rs.


And, Cost price per kg of type B wheat
= 10x Rs.
Selling price of mixture of Type A & Type B wheat = 11x Rs.
Cost price per kg of mixture of Type A & Type B wheat = 11x
32
× = 6.4𝑥 𝑅𝑠.
55

ATQ –
Let N kg of type B wheat in mixture of 200 kg of Type A & Type B
of wheat
37 Adda247 Publications For any detail, mail us at
Publications@adda247.com
Cracker Book for Bank (IBPS | SBI | RRB PO | Clerk) Mains Exams

6.4𝑥 × 200 = N × 𝑥 + 10x (200 − N)


1280 = N + 2000 – 10N
9N = 920
N = 80 kg
80
Required ratio =
120
2
=
3

39. (b); Let marked price of jeans is 300𝑥 Rs. while marked price of shirt
is 200𝑥.
Selling price of a jeans & a shirt in scheme A
65
= (300𝑥 + 200𝑥) ×
100
= 325𝑥
Selling price of one jeans & one shirt in scheme B
75 80
= 300𝑥 × + 200𝑥 ×
100 100

= 225𝑥 + 160𝑥
= 385𝑥 Rs.
Given, 385𝑥 – 325𝑥 = 384 Rs.
60𝑥 = 384 Rs.
𝑥 = 6.4 Rs.
3
Cost price of a shirt = 160𝑥 ×
4
3
= (160 × 6.4 × )
4

= 768 rs.
4
Cost price of jeans = 225𝑥 ×
5
4
=225 × 6.4 ×
5

= 1152 Rs.

38 Adda247 Publications For any detail, mail us at


Publications@adda247.com
Cracker Book for Bank (IBPS | SBI | RRB PO | Clerk) Mains Exams

40. (b); Let, total manufactured cost of a cycle


= 100x Rs.
Then S.P a cycle = 125x Rs.
Expenditure on raw materials
= 100x × 0.40 = 40𝑥 𝑅𝑠.
Expenditure on labour = 100𝑥 × .20
= 20𝑥 𝑅𝑠.
Fixed charge = 100𝑥 × .20 = 20𝑥 𝑅𝑠.
Expenditure on transportation
= 100x − (40𝑥 − 20𝑥 − 20𝑥 ) = 20𝑥 𝑅𝑠.
New expenditure on raw materials
= 1.15 × 40x = 46x Rs.
New expenditure on labour
= 1.2 × 20x = 24x Rs.
New expenditure on transportation
= 1.5 × 20x = 30x Rs.
7
To get a profit of 13 % at a selling price of Rs. 125x, the C.P. of
11
1100
an item must be equal to 125𝑥 ×
1250

= 110𝑥 Rs.
We need to reduce C.P. (or expenditure) by 10, and this 10 has
to be reduced from expenditure on raw materials.
10𝑥
Required % = × 100
46𝑥
17
= 21 %
23

39 Adda247 Publications For any detail, mail us at


Publications@adda247.com
Cracker Book for Bank (IBPS | SBI | RRB PO | Clerk) Mains Exams

1 Adda247 Publications For any detail, mail us at


Publications@adda247.com
Cracker Book for Bank (IBPS | SBI | RRB PO | Clerk) Mains Exams

Chapter
Simple Interest and Compound Interest
6
BEST APPROACH TO SOLVE THE QUESTIONS

The interest can be defined as amount paid by the borrower to the lender in
addition to the amount which he had borrowed. In competitive exams,
interest is classified into Simple interest (SI) and Compound Interest (CI).

Easiest example, when we borrow money from bank, it doesn’t say pay Rs.
100 as interest every year no matter whether you took a loan of Rs 1000 or
Rs 100000. They offer us a rate generally in % at which we have to pay
interest. This rate is known as Rate of interest (ROI).

For eg. If I borrowed Rs. 500 for 1 year at a rate of 8%/annum. Find the
interest that I have to pay.
Always remember, this 8% we have to calculate on the amount we
borrowed. 8% of Rs. 500 means Rs. 40, that I have to pay as an interest.

Note: Different forms of ROI may be given or we have to change, as 3% for 3


months or 24% for 18 months. For easier calculation we can normalize
these in 12 month/year rate. In above rates, former means 12%/annum
and later means 16%/annum.

Now, if I asked what would be amount of interest that I have to pay, If I


borrow Rs. 500 for 3 years at same rate?

Here arise difference b/w SI and CI.

2 Adda247 Publications For any detail, mail us at


Publications@adda247.com
Cracker Book for Bank (IBPS | SBI | RRB PO | Clerk) Mains Exams

SIMPLE INTEREST:

Simple interest simply means that we have to pay same amount of interest
every time. If I have to pay interest on Rs. 500 at ROI of 8%/annum for 3
years, it means I will pay
40 × 3 = RS 120.

Formula to calculate SI on an amount is


PRT
SI =
100
Here, P is the amount borrowed and generally it is referred as Principal.
R is ROI in %. (100 in equation is used to because of %)
And T is time in years.

NOTE: If it is asked in a question, that what is interest obtained at the ROI


of 10%(or any value) for 5 years on amount of Rs. Z.

In order to save time first calculate mentally 10 × 5 = 50%. Total interest


will be 50 % of Principal amount.

COMPOUND INTEREST:

The one liner difference b/w SI and CI is that interest paid for every year is
not same. Consider the above example.

Interest accrued in 1st year = Rs. 40


In CI principal amount changes, after 1year principal amount will be Rs. 540
not Rs. 500.
8
After second year interest obtained on Rs. 540 is × 540 = Rs. 43.2
100
And interest after 3 years will be calculated on Rs 540+Rs 43.2= Rs 583.2 i.e
8
× 583.2= Rs. 46.65
100
And total interest will be 40 + 43.2 + 46.65= Rs 129.85
The formula for amount obtained after the given time period in CI
𝐑 𝐧
𝐀 = 𝐏 (𝟏 + )
𝟏𝟎𝟎
Here A is amount obtained after interest, P is principal, n is number of years
R is ROI.
CI= A – P.
3 Adda247 Publications For any detail, mail us at
Publications@adda247.com
Cracker Book for Bank (IBPS | SBI | RRB PO | Clerk) Mains Exams

In competitive exams, this approach is rarely used because it will be too


calculative if time is 3 years and ROI is 17%.
17 3 117 3
A = P (1 + ) = P( ) Imagine if you started calculating cube of 117 .
100 100

Best approach to use in CI is resultant ROI, it makes CI as easy as SI. Eg.


If ROI on Rs.10000 in a scheme is Y%, calculate total interest after two
years. Then resultant interest is

Y2
Y + Y + (Y × Y)/100 = 2Y +
100
17×17
Resultant interest % on 17%= 2×17 + = 34+ 2.89= 36.89%.
100
It is more time saving than traditional method. Little use of mind will help
you to solve all kind of problems.
Note that if we can calculate resultant interest % for two years, we can also
calculate it for three years. Actually general formula for calculating
resultant % for two years, if ROI on first year is X% and second year is Y%,
then resultant interest % will be
X×Y
X+Y+
100

For calculating 3year resultant interest, first calculate resultant interest of


first two years and then again use resultant formulae for adding resultant of
first two year and ROI of 3rd year.

REMARK: Check 3 years total interest % for ROI 0f 10% whether it comes
33.1%.

DIRECT FORMULAE TO CALCULATE 3 YEARS RESULTANT INTEREST %


3r. 3 r² r³ __
Here, r is ROI

Note that 4 underscores (_ _ _ _) represent 4 digits after decimal. Further,


make it clear that _ _ (last two digits are for last two digits of value of r³]
And starting two (_ _) are for value of 3r².
Consider eg.
3r. 3 r² r³_
When r = 2
r³ = 08, r² = 4, 3r² = 12
4 Adda247 Publications For any detail, mail us at
Publications@adda247.com
Cracker Book for Bank (IBPS | SBI | RRB PO | Clerk) Mains Exams

Resultant interest is for 3 years


6.1208
Now consider r = 8
r³ = 512, r² = 64, 3r² = 192
use last two digits of r³ for last two digits.
i.e. 3r. _ _ 12, and carry on 5 to 3r².
3r² = 192 + 5 = 197.
Here also, use only its last 2 digit and carry on 1.
3r. 9 7 1 2
3r = 24 + 1 → carry on from 197.
Resultant interest % = 25.9712%
Check for r = 10%.
While solving a question, student should always keep in mind
resultant interest %.
Example: Amount of Rs 4000 becomes Rs 6600 in SI in 5 years. Calculate
ROI.
PRT
Solution: Though general formula of SI i.e. SI = is an easy approach,
100
But try mentally to solve RS 2600 is what % Rs 4000. You can mentally
calculate that
2600 13
= which means 65%.
4000 20
And 65% in 5 years in SI means 13%/year.

Example: Difference b/w interest obtained in SI and CI on an amount at the


ROI of 10% is Rs. 310. Find the principal amount.

Solution:
Calculate resultant % for SI and CI for 3 years at ROI of 10%.
Its 30% for SI and 33.1% for CI. But we know that we calculate interest on
principal. It means that 3.1% of principal is equal to Rs 310. Hence principal
can be calculated.

POINTS TO REMEMBER ALWAYS:


Difference b/w interest obtained on CI and SI for 2 years is PR2/100.
Resultant interest % in CI for 3 years at 10% ROI is 33.1% and
15.7625% at ROI of 5%.
5 Adda247 Publications For any detail, mail us at
Publications@adda247.com
Cracker Book for Bank (IBPS | SBI | RRB PO | Clerk) Mains Exams

Practice Exercise Based on new Pattern

1. Ankur invested X Rs. at the rate of 15% per annum on compound


interest for two years and gets total interest of 5805 Rs. if Ankur
invest (X + 7000) Rs. for another two year at additional rate of 5%,
then what will be compound interest on that investment ?
(a) 10000 Rs. (b) 11000 Rs. (c) 12000 Rs.
(d) 15000 Rs. (e) 18000 Rs.

2. Shikha invested 32000 Rs. at simple interest for 2 years at the rate of
R% and gets an interest of Rs.8000. If he invested total amount
(Principle + Interest) in a scheme, which offered compound interest at
the rate of (R%+2.5%) then find total compound interest obtained by
Shikha after 2 years ?
(a) 12600Rs. (b) 12800Rs. (c) 14400 Rs.
(d) 12000 Rs. (e) 12900Rs.

3. Ankit borrowed Rs. 12000 from Veer on C.I. at 10% per annum for
three years and added some extra amount and lent to Arun on C.I. at
20% per annum for two year. If Ankit got Rs. 3948 more interest than
interest got by Veer.Find how much extra amount was added by Ankit
?
(a) 6000 Rs. (b) 4000 Rs. (c) 5000 Rs.
(d) 8000 Rs. (e) 9000 Rs.
4. Abhi invested some amount on scheme ‘P’ which offer CI at the rate of
15% p.a.. After 2 years he got Rs. 1032 as interest. Abhi invest the
amount he got from scheme ‘P’ in scheme ‘Q’ which offer 15% p.a. at SI
for 4 years. Find the total interest he earned from scheme P and
scheme Q together?
(a) 3571.2 (b) 3715.2 (c) 4232
(d) 3148 (e) 3379.2

6 Adda247 Publications For any detail, mail us at


Publications@adda247.com
Cracker Book for Bank (IBPS | SBI | RRB PO | Clerk) Mains Exams

5. A man invests Rs. 1,200 at 10% p.a. At the end of the year he
withdraws 30% of total amount and pays Rs. 24 as transaction fee. At
the end of 2nd year he withdraws 30% of the amount and pays Rs. 93
as transaction fee. What is the balance at the end of the third year?
(a) Rs. 660 (b) Rs. 825 (c) Rs. 500
(d) Rs. 770 (e) None of these

6. A sum of Rs. 1000 after 3 years at compound interest of 10% p.a.


becomes a certain amount that is equal to the amount that is the
result of 3 year depreciation from Rs. 1728. Find the approximate
difference between the rate of C.I. and rate of depreciation.
(a) 3.33% (b) 0.66% (c) 4%
(d) 2% (e) 6%

7. A person invested a certain amount at simple interest at the rate of 6


per cent per annum earning Rs. 900 as an interest at the end of three
years. Had the interest been compounded every year, how much more
interest would he have earned on the same amount with the same
interest rate after three years?
(a) Rs. 38.13 (b) Rs. 25.33 (c)Rs. 55.08
(d) Rs. 35.30 (e) None of these

1
8. Mayank borrowed Rs. 48000 from a bank at 12 % per annum
2

compound interest at the end of 1st, 2nd and 3rd year, he paid 14000,
13000 and 16000 respectively. If he wanted to clear his loan at the
end of 4th year, what would he pay at the end of the fourth year to
clear his loan ?
(a) 21000 (b) 22500 (c) 16800
(d) 26000 (e) None of these

7 Adda247 Publications For any detail, mail us at


Publications@adda247.com
Cracker Book for Bank (IBPS | SBI | RRB PO | Clerk) Mains Exams

9. Rakesh lent Rs. P to Rahul on an agreement that for first two year,
interest will be calculated on SI at 15% per annum and for next two
years interest will be calculated on C.I at 5% additional rate. If Rahul
paid total amount of Rs. 17971.2 at the end of four year, Find the
amount borrowed by Rahul?
(a) 9000 Rs. (b) 9800 Rs. (c) 9900 Rs.
(d) 9600 Rs. (e) 9200 Rs.

10. Veer invested Rs 22500 for 2 year at the Rate of x% in scheme A at


compound interest annually and gets a total amount of Rs 32400 If he
added Rs 2600 in amount and invested total amount in scheme B at
S.I. for 3 year at same rate . Then find the total simple interest veer gets
from scheme B?
(a) Rs 22500 (b) Rs 22000 (c) Rs 17500
(d) Rs 20000 (e) Rs 21000

11. Veer Invested X Rs. in SBI at the rate of 18% for 2 year and obtained a
total simple interest of 6750 Rs. If he invested 2250 Rs. more for same
2 2
period time at the rate of 14 % for first year and at 16 % for 2nd
7 3
year on compound annually. Then find the total compound interest
obtained by Veer after 2 year ?
(a) 6000 Rs. (b) 5000 Rs. (c) 3000 Rs.
(d) 7000 Rs. (e) 5500 Rs.

12. Satish invested 16000 Rs. in simple interest for 2 years on certain rate
and gets an interest of 4800 Rs, if he invested total amount (Principle
+ Interest) in a scheme, which offered compound interest on 5% more
interest rate as earlier rate. Then find total interest gets by Satish after
2 years ?
(a) 9252 Rs. (b) 9225 Rs. (c) 9512 Rs.
(d) 925 Rs. (e) 9152 Rs.

8 Adda247 Publications For any detail, mail us at


Publications@adda247.com
Cracker Book for Bank (IBPS | SBI | RRB PO | Clerk) Mains Exams

13. Bhavya and Veer invested their principle in two different schemes,
Bhavya invested X Rs. on compound interest for two year at rate of
20% annually and Veer invested 4000 Rs. more than Bhavya on
simple interest for three year at 15% annually, if both gets total
interest of Rs. 9632, then Find the amount invested by Veer ?
(a) 12900 (b) 12400 (c) 8800
(d) 12800 (e) 12600

14. A man invested Rs. x, y and z in three difference schemes which offers
10% p.a., 20% p.a. and 30% p.a. rate respectively and after one year
he earns Rs. 2000 interest. If x, y and z are in arithmetic progression
and sum of x, y and z is Rs. 9,000. Find the difference between ‘x’ and
‘z’.
(a) 1500 (b) 2000 (c) 2500
(d) 2250 (e) 1750

15. P invested Rs. X in a scheme for 2 year which offered simple at the
rate of 15% per annum and Q invested Rs. (X + 2500) in another
scheme for same period of time, which offered compound interest at
the rate of 20% per annum. If from both scheme P and Q got total
interest of Rs. 32550, then find the value of X ?
(a) 41500 (b) 42500 (c) 40500
(d) 40000 (e) 38250

16. Rajat have some money in his hand. He invested 20% of the money in
scheme ‘A’ for 4 year at 6% p.a., 30% of the amount in scheme ‘B’ for 6
year at 12% p.a., remaining in scheme ‘C’ 2 year at 15% p.a. If total
amount Rajat received from scheme ‘A’, ‘B’ and ‘C’ is 11,355, then find
the difference of sum invested in scheme ‘B’ and scheme ‘A’.
(a) 1800 (b) 1500 (c) 1200
(d) 750 (e) 270
9 Adda247 Publications For any detail, mail us at
Publications@adda247.com
Cracker Book for Bank (IBPS | SBI | RRB PO | Clerk) Mains Exams
1
17. Satish borrowed Rs. 24000 from a SBI at the rate of 12 %
2
compounded annually. If at the end of 1st, 2nd and 3rd year, he paid Rs.
7000, Rs. 6500 and Rs. 8000 respectively. If Satish wanted to clear his
loan at the end of 4th year, what would he pay at the end of the fourth
year to clear his loan ?
(a) 11500 Rs. (b) 11250 Rs. (c) 10500 Rs.
(d) 9500 Rs. (e) 14500 Rs.

18. Divyaraj invested an amount into two parts in the ratio of 4 : 3 on


compound interest for two years at the rates of 20% & 15%
respectively. If he exchange rate of interests, then he will get Rs. 705
less interest than earlier interest, then find how much Simple interest
he will get, if he invest total amount at the rate of 17.5% for two
years?
(a) 14500 Rs. (b) 14700 Rs. (c) 14900 Rs.
(d) 14800 Rs. (e) 14900 Rs.

19. Rahul took a loan of Rs.80000/- at rate of of 5% p.a. at SI & lent it at


5% pa at CI. After 3 years, he cleared all his debts and invested the
profit earned in a scheme which offers C.I. After two years he got
Rs.128.1/- as interest from the scheme. Find the rate of interest
offered by scheme.
(a) 8% (b) 7% (c) 6%
(d) 10% (e) 12%

20. A man invested his total savings in two parts. He invested Rs. 30000/-
in a scheme which offers CI at ROI of 10% per annum and remaining
in SI at ROI of 5%per half year. After two years the amount obtained
by him in former scheme is half of later, then calculate amount
invested in later scheme.
(a) Rs. 242000 (b) Rs. 60500 (c) Rs. 12100
(d) Rs. 80525 (e) None of these
10 Adda247 Publications For any detail, mail us at
Publications@adda247.com
Cracker Book for Bank (IBPS | SBI | RRB PO | Clerk) Mains Exams

21. Satish saved 55% of his monthly income, which he further invested in
two different schemes A & B in the ratio of 7 : 5 on CI compounded
annually at the rate of 10% p.a. and 20% p.a. respectively. If after two
years, Satish gets total interest of Rs. 8074 from both the schemes,
then find the total annual saving of Satish?
(a)316,600 Rs. (b)316,400 Rs. (c)316,000 Rs.
(d)316,200 Rs. (e)316,800 Rs.

22. Aman invest Rs. (x – 2000) in scheme ‘P’ which offers 20% CI pa while
Rs. (x + 2000) in scheme ‘Q’ which offers 8% SI pa. After 2 year
difference between interest earn from scheme P and Q is 1600, then
amount invested by Aman in scheme ‘Q’ is what percent more/less
than amount invested by him in scheme ‘P’ ?
1
(a) 75% (b) 133 % (c) 150%
3
1
(d) 50% (e) 33 %
3

23. Ritu invested her total saving in three different FD schemes A, B and C
in the ratio of 5 : 4 : 6 on CI for two years at the rate of 10%, 15% and
20% respectively.If interest is calculated annually and interest from
scheme B is Rs. 744 more than interest from scheme A then, find
difference between interest received from scheme C and scheme B by
Ritu?
(a) Rs. 4185 (b) Rs. 4175 (c) Rs. 3840
(d) Rs. 4580 (e) Rs. 3250

24. Raghv invested Rs. 8400 and Rs. 9600 at the rate of R% and (R + 5)%
respectively on simple interest for two years and gets total interest of
Rs. 6360. If Raghv invested Rs.(8400 + P) and Rs.(9600 + P) at the rate
of R% and (R + 5)% respectively on compound interest for two year,
then he would get total interest of Rs. 8153. Find value of P?
(a) 1800 Rs. (b) 1600 Rs. (c) 2200 Rs.
(d) 2400 Rs. (e) 2600 Rs.
11 Adda247 Publications For any detail, mail us at
Publications@adda247.com
Cracker Book for Bank (IBPS | SBI | RRB PO | Clerk) Mains Exams

Directions (25-26): A person invested Rs. 20000 in a bank which is


offering 10% per annum simple interest. After two years he withdrew the
money from the bank and deposited the total amount in another bank
which gives an interest rate of r% p.a. compounded annually. After 2 years
he received an amount of Rs. 2460 more than what he had invested in that
bank.

25. What is the value of r?


(a) 10% (b) 15% (c) 5%
(d) 12% (e) None of these

26. If the person had invested Rs. 50,000 instead of 20000 in the bank
that offered simple interest, what would have been his net profit after
following the same procedure as given above?
(a) Rs. 16,800 (b) Rs. 16,150 (c) Rs. 16,350
(d) Rs. 16,000 (e) None of these

27. A bank offers 10% p.a. at CI in scheme ‘A’, 20% p.a. at CI in scheme ‘B’
and 40% p.a. at CI in scheme ‘C’. Veer have same amount in his hand.
He invested 32% amount in scheme ‘A’, 20% in scheme ‘B’ and
remaining in scheme ‘C’. After two year he received 6600 as interest.
Find the difference between amount invested by Veer on scheme ‘C’ to
that of in scheme ‘B’.
(a) 5,000 (b) 4,000 (c) 2,000
(d) 3,000 (e) 2,500

28. Bhavya borrowed Rs. 37500 on compound interest with agreement


that, he will not pay amount after 2 year with interest then the rate of
interest increase by 5% as earlier interest rate. If Bhavya paid Rs.
12093.75 as total compound interest after two year. Then find how
much amount Bhavya have to pay after two year for completing his
entire borrowing?
(a) Rs. 58000 (b) Rs. 52000 (c) Rs. 42000
(d) Rs. 45000 (e) Rs. 54000
12 Adda247 Publications For any detail, mail us at
Publications@adda247.com
Cracker Book for Bank (IBPS | SBI | RRB PO | Clerk) Mains Exams

29. A man invested ‘X’ amount in scheme A and ‘Y’ amount in scheme B.
Scheme A offers 8% p.a. at S.I. scheme B offer 20%. p.a. at C.I. Man
invested both schemes for 2 years. The sum of interest he earns after
2 years will be equal to the simple interest he can earn if he invested
‘Y’ amount in scheme A for 7 years. Find the total amount he has
initially if difference between interest earned by him after 2 years is
2304.
(a) 12,300 (b) 12,900 (c) 12,600
(d) 13,500 (e) 13,800

30. Shikha and Sameer have equal amount. Shikha invested on C.I. for two
2
years at the rate of 10% p.a. and Sameer invested 66 % of amount at
3
the rate of R% p.a. on simple interest and remaining amount at the
rate of 6.5% p.a. on simple interest. If interest received by both at the
end of two years are equal, then find the value of ‘R’?
(a) 10% (b) 12.5% (c) 15%
(d) 12% (e) 8%

31. Veer invested Rs. x in SBI for three years on S.I. at the rate of 15% per
annum and Sameer invested Rs. (x + 4000) in BOB for two years on
C.I. at the rate of 8% per annum. If Divyaraj invested equal to sum of
what Veer and Sameer invested in UBI for two years on C.I. at the rate
of 20% per annum. Find the sum invested by Divyaraj, if they all got
total interest of Rs. 20382.4?
(a) Rs. 20000 (b) Rs. 18000 (c) Rs. 28000
(d) Rs. 30000 (e) Rs. 26000

32. Three employee Ankit, Veer and Sameer got Rs. P, Rs. (P+2400) and
Rs. (P+4400) Diwali bonus. Ankit and Veer invested their half of
bonus on CI at the rate of 10% and 20% respectively for two years.
Sameer invested 60% of his bonus on simple interest at the rate of
15% p.a. for three years. If Sameer got Rs. 132 more as interest got by
Ankit and Veer together, then find the bonus got by Sameer?
(a) 12000 Rs. (b) 10000 Rs. (c) 14400 Rs.
(d) 14000 Rs. (e) 18000 Rs.
13 Adda247 Publications For any detail, mail us at
Publications@adda247.com
Cracker Book for Bank (IBPS | SBI | RRB PO | Clerk) Mains Exams

33. Manoj lend Rs. P for three years on S.I. at the rate of 15% per annum
and Rajesh lend Rs. (P + 8000) for two years on C.I. at the rate of 8%
per annum. Suresh borrowed sum equal to of what Manoj and Rajesh
lend, for two years on C.I. at the rate of 20% per annum. If Suresh paid
interest Rs. 5352 more than, what Manoj and Rajesh got total interest
on their sums together. Find total sum borrowed by Suresh?
(a) Rs. 34000 (b) Rs. 44000 (c) Rs. 32000
(d) Rs. 46000 (e) Rs. 30000

34. A person invested his savings of Rs. 32000 in two parts at equal rate
of interest in CI and SI. After 3 years, he withdrew amount invested in
CI, and put the cash in his almirah. After another 2 years, the amount
invested in SI got doubled and he also withdrew them. If the final ratio
of both the amount is 36:65, then find amount invested by him in SI
(in Rs.)
(a) 20000/- (b) 18050/- (c) 19500/-
(d) 21500/- (e) can’t be determined

35. A man invested an amount into three schemes P, Q and R in the ratio
of 5 : 6 : 9 at the rates of 20%, 15% and 10% respectively. Schemes P
and R offered compound interest annually, while scheme Q offered
simple interest annually. Total interest received by man from scheme
P and Q together is Rs. 1899 more than total interest received from
scheme R after two years. If man would invest total amount on C.I at
the rate of 15% p.a., then find the total interest received by the man?
(a) 5800 Rs. (b) 5805 Rs. (c) 5580 Rs.
(d) 5590 Rs. (e) 5900 Rs
14 Adda247 Publications For any detail, mail us at
Publications@adda247.com
Cracker Book for Bank (IBPS | SBI | RRB PO | Clerk) Mains Exams

Solutions

15 2
1. (b); ATQ, 5805 = X [(1 + ) – 1]
100
129X = 5805 × 400
X = 18000 Rs.
Now
Total Interest=(18000+7000)
(15+5) 2
×[(1 + ) – 1]
100
11
= 25000 × ( )
25
= 11000 Rs.

2. (e); Let shikha invested at the rate of R%


ATQ—
8000×100
R= R = 12.5%
32000×2
New Rate = 12.5 + 2.5 = 15%
Total amount = 32000 + 8000 = 40000 Rs.
Equivalent CI of two years at the rate of 15%
15×15
= 15 + 15 +
100
= 32.25%
32.25
Required compound interest = 40000 × = 12900 Rs.
100

10
3. (a); Interest got by Veer=12000[(1 + )3 − 1]
100
= 3972
20×20
Two year CI on 20% = 20 + 20 +
100
= 44%
Atq,
44
(12000 + 𝑥 ) − 3972 = 3948
100
5280+.44x – 3972 = 3948
.44x = (3948 + 3972) – 5280
.44x = 2640
x = 6000 Rs
15 Adda247 Publications For any detail, mail us at
Publications@adda247.com
Cracker Book for Bank (IBPS | SBI | RRB PO | Clerk) Mains Exams

4. (a); Let, sum Abhi have initially = 100x


ATQ,
115 115
100x × × – 100x = 1032
100 100
132.25x – 100x = 1032
1032
⇒x= = 32
32.25
Amount initially Abhi have = 3200
4232×15×4
Interest earned from scheme Q =
100
= 2539.2
Required amount = 2539.2 + 1032
= 3571.2

5. (a); The man invests Rs. 1,200 at 10% p.a.


At the end of 1st year the amount = Rs. 1,320
30
Withdrawal × 1320 + 24 = Rs. 420
100
Amount at the end of second year
= 900 × 1.1 = Rs. 990
30
Withdrawal = × 990 + 93 = Rs. 390
100
∴ Amount at the end of 3 years
= 600 × 1.1 = Rs. 660

6. (d); The amount @ 10% C.I. could become Rs. 1331. Also,
Rs. 1728 depreciated at R% has to become Rs. 1331.
Thus,
100−R 3
1728 × [ ] =1331(approximately).
100

The closest value of R = 8%


Thus, the difference is 2%.

16 Adda247 Publications For any detail, mail us at


Publications@adda247.com
Cracker Book for Bank (IBPS | SBI | RRB PO | Clerk) Mains Exams
900 ×100
7. (c); P =
6×3
P = 5000

Required value = 955.08 – 900 = 55.08

8. (b); 1st year CI


1
48000 × = 6000
8
Amount = 48000 + 6000 = 54000
2nd year principle = 54000 – 14000 = 40000
2nd year CI =
1
40000 × = 5000
8
Amount = 40000 + 5000 = 45000
3rd year principle
= 45000 – 13000 = 32000
3rd year CI
1
= 32000 × = 4000
8
Amount = 32000 + 4000 = 36000
4th year principle = 36000 – 16000 = 20000
At the end of 4th year amount
1
= 20000 × + 20,000
8
= 2500 + 20000
= 22500

9. (d); For first two years total S.I = 15 × 2


= 30%
For next two years total C.I on (15+5)%
20×20
= 20+20+
100
= 44%
Amount after two year

17 Adda247 Publications For any detail, mail us at


Publications@adda247.com
Cracker Book for Bank (IBPS | SBI | RRB PO | Clerk) Mains Exams
𝑃×30
=P+
100
13𝑃
= Rs.
10
Amount after 4 years
13𝑃 144
× = 17971.2
10 100
17971.2
P=
1.872
P = Rs. 9600

10. (e); ATQ,


𝑅 2
32400 = 22500 (1 + )
100
324 100 + 𝑅 2
=( )
225 100
18 100 + 𝑅
=
15 100
15R = 300
R = 20%
20×3
Simple interest = (32400 + 2600) ×
100
= 21000 Rs

X×18×2
11. (d); Principle = = 6750
100
6750×100
⇒X=
18×2
X = 18750 Rs.
According to question —
1
1st year CI = (18750 + 2250) ×
7
= 3000
1
2 year CI = (21000 + 3000) ×
6
1
= 24000 ×
6
= 4000
Total CI after 2 year = 3000 + 4000
= 7000 Rs.

18 Adda247 Publications For any detail, mail us at


Publications@adda247.com
Cracker Book for Bank (IBPS | SBI | RRB PO | Clerk) Mains Exams
4800×100
12. (e); Rate =
16000×2
R = 15%
New rate = 15 + 5 = 20%
2 year CI on 20%
20×20
= 20 + 20 +
100
= 44%
Interest gets Satish
44
= (16000 + 4800) ×
100
= 9152 Rs.

13. (d); ATQ,


(X+4000)45 44X
+ = 9632
100 100
0.45X + 1800 + 0.44x = 9632
0.89X = 9632 – 1800
7832
X=
0.89
X = 8800
Veer principle = 8800 + 4000
= 12800 Rs.

14. (b); x + y + z = 9,000


& 2y = x + z
⇒ y = 3,000
10 20 30
×x+ × 3,000 + × z = 2000
100 100 100
0.1x + 0.3z = 2000 – 600 = 1400
and x + z = 6,000
0.1x + 0.3z = 1400 …(i)
x + z = 6,000 …(ii)
on solving (i) & (ii), we get
x = 2,000
z = 4,000
Difference between x & z = 2,000

19 Adda247 Publications For any detail, mail us at


Publications@adda247.com
Cracker Book for Bank (IBPS | SBI | RRB PO | Clerk) Mains Exams

15. (b); ATQ


15×2x 20 2
+ (x + 2500) [(1 + ) – 1] = 32550
100 100
0.3x + 0.44x + 1100 = 32550
0.74x = 31450
31450
x=
0.74
x = 42500

16. (d); Let the amount Rajat have = 100


ATQ,
20×4×6 30×6×12 50×2×15
100 + + + = 11,355
100 100 100
151.4 = 11355
⇒ 100 = 7500
7500[30 –20]
Required difference =
100
= 750

17. (b); 1st year CI


1
24000 × = 3000 Rs.
8
Amount = 24000 + 3000 = 27000 Rs.
2nd year principle = 27000 – 7000 = 20000 Rs.
2nd year CI =
1
20000 × = 2500 Rs.
8
Amount = 20000 + 2500 = 22500 Rs.
3rd year principle
= 22500 – 6500 = 16000 Rs.
3rd year CI
1
= 16000 × = 2000
8
Amount = 16000 + 2000 = 18000 Rs
4th year principle = 18000 – 8000 = 10000 Rs.
At the end of 4th year amount, which Satish have to pay
1
= 10000 × + 10,000
8
= 1250 + 10000 = 11250 Rs.

20 Adda247 Publications For any detail, mail us at


Publications@adda247.com
Cracker Book for Bank (IBPS | SBI | RRB PO | Clerk) Mains Exams

18. (b); Let Divyaraj invested total amount = 7x Rs.


Equivalent CI of two year at the rate of 20%
20×20
= 20 + 20 + = 44%
100
Equivalent CI of two year at the rate of 15%
15×15
= 15 + 15 + = 32.25%
100
ATQ –
44 32.25 32.25 44
4x × + 3𝑥 × − 4𝑥 × − 3𝑥 × = 705
100 100 100 100
(1.76x + .9675x) – (1.29𝑥 + 1.32𝑥) = 705
2.7275x – 2.61𝑥 = 705
0.1175x = 705
x =6000 Rs.
Total amount invested by Divyaraj = 6000 × 7 = 42000 𝑅𝑠.
17.5×2
Required simple interest = 42000 × =14700 Rs.
100

19. (d); Profit earned in 3 years = Difference in interest obtained


D=P r2 (300+r)
(100)3
80000×25×305
=
1000000
= Rs610
Let scheme offers R% rate of interest
𝑅 2
610 (1 + ) − 610 = 128.1
100
R = 10%

20. (b); Let he invested Rs. Y in second scheme


5% per half year means 10% per annum in SI
Therefore, amount obtained by him after two years
Y(2)(10)
=Y+
100
6𝑌
=
5
If he invested Rs. 30000/- in a scheme, which offers ROI of 10%
per annum
Amount obtained by him after two years is
10 2
= 30000 (1 + )
100

21 Adda247 Publications For any detail, mail us at


Publications@adda247.com
Cracker Book for Bank (IBPS | SBI | RRB PO | Clerk) Mains Exams
11 11
= Rs. 30000 × ×
10 10
= Rs. 36300
Now According to Q
1 (6𝑌)
⇒ 36300 =
2 5
36300×5×2
⇒Y=
6
= Rs. 60500/-

21. (e); Let total monthly income of Satish = 100x Rs.


Amount invested in scheme A
55 7
= 100𝑥 × ×
100 12
385𝑥
= Rs.
12
Amount invested in scheme B
55 5
= 100𝑥 × ×
100 12
275𝑥
= Rs.
12
Equivalent CI of two years on 10%
10×10
= 10 + 10 +
100
= 21%
Equivalent CI of two years on 20%
20×20
= 20 + 20 +
100
= 44%
ATQ—
385𝑥 21 275 44
× + × = 8074
12 100 12 100
1617𝑥+2420𝑥
= 8074
240
4037x = 8074 × 240
𝑥 = 480 Rs.
Annually saving of Satish
55
= 48000 × × 12
100
= 316,800 Rs.

22 Adda247 Publications For any detail, mail us at


Publications@adda247.com
Cracker Book for Bank (IBPS | SBI | RRB PO | Clerk) Mains Exams

22. (d); Compound interest rate for 2 years


20+20
= 20 + 20 + = 44%
100
ATQ,
44 (x+2000)×8×2
(x– 2000) × – = 1600
100 100
44𝑥 16𝑥
⇒ – – 880 – 320 = 1600
100 100
28𝑥
⇒ = 1600 + 880 + 320 = 2800
100
⇒ x = 10,000
12000−8000 4
Required % = × 100 = × 100 = 50%
8000 8

23. (a); Let total saving of Ritu be x Rs.


5𝑥
Invested in Scheme A =
15
𝑥
= Rs.
3
4𝑥
Amount Invested in scheme B = Rs.
15
2𝑥
Invested in scheme C = 𝑅𝑠.
5
10×10
Two year C.I on 10% =10+10+
100
= 21%
15×15
Two year CI on 15% =15+15 +
100
= 32.25 %
20×20
Two year CI on 20% =20+20+
100
= 44%
ATQ,
4𝑥 32.25 𝑥 21
× − × = 744
15 100 3 100
129𝑥 21𝑥
− = 744
1500 300
24x = 744 × 1500
744×1500
x=
24
x = 46500 Rs.
Required difference
46500×6 44 46500×4 32.25
= × − ×
15 100 15 100
= 8184 – 3999
= 4185 Rs.
23 Adda247 Publications For any detail, mail us at
Publications@adda247.com
Cracker Book for Bank (IBPS | SBI | RRB PO | Clerk) Mains Exams

24. (b); ATQ—


8400×𝑅×2 9600×(𝑅+5)×2
+ = 6360
1000 100
168R + 192R + 960 = 6360
360R + 960 = 6360
360R = 5400
R = 15%
Rs. (8400 + P) invested at 15%
And Rs. (9600 + P) invested at 20%
Equivalent CI of two years at the rate of 15%
15×15
= 15 + 15 +
100
= 32.25%
Equivalent CI of two year at the rate of 20%
20×20
= 20 + 20 + = 44%
100
ATQ—
32.25 44
(8400 + 𝑃 ) × + (9600 + 𝑃 ) × = 8153
100 100
270900 + 32.25P + 422400 + 44P= 815300
76.25P = 122000
P = 1600 Rs.

25. (c); Amount withdrawn from bank which offered simple interest =
20000×10×2
20000 + = 24000
100
Compound interest accrued from another bank = 2460
𝑅 2
2460 = 24000 [(1 + ) − 1]
100
⇒r=5%

26. (b); Amount withdrawn after 2 years from bank offering S.I. =
5000×10×2
50000 +
100
= 60000
Amount withdrawn after another 2 years from another bank
5 2
offering C.I. = 60000 (1 + ) = 66150
100
New profit = 66150 − 50000 = 16,150

24 Adda247 Publications For any detail, mail us at


Publications@adda247.com
Cracker Book for Bank (IBPS | SBI | RRB PO | Clerk) Mains Exams

27. (d); Let, Total amount Veer have = ‘x’


ATQ,
11 11 12 12 14 14
0.32x [ × – 1] + 0.20x [ × – 1] + .48x [ × – 1]
10 10 10 10 10 10
= 6600
21 44 96
0.32x [ ] + 0.20x [ ] + 0.48x [ ] = 6600
100 100 100
6.72𝑥 8.8𝑥 46.08𝑥
+ + = 6600
100 100 100
6600×100
⇒x=
61.6
6600×100
Required difference = × [. 48 – .20] = 3,000
61.6

28. (e); ATQ,


r 2
12093.75 = 37500 [(1 + ) – 1]
100
12093.75 r
√ +1 = 1+
37500 100
49593.75 r
√ =1+
37500 100
r
1.15 – 1 =
100
r = 15%
New rate = 15 + 5 = 20%
Amount paid by Bhavya after next two more years
144
= 37500 × = 54000 Rs.
100

29. (c); ATQ


𝑥×8×2 20 2 𝑦×8×7
+ 𝑦 [(1 + ) − 1] =
100 100 100
16x + 44y = 56y
16x = 12y
𝑦 4 3𝑦
= ⇒𝑥=
𝑥 3 4
44y – 0.16x = 2304
3
44y – 0.16 × 𝑦= 2304
4
0.44y – 0.12y = 2304
y = 7200
x = 5400
Required amount = 12,600
25 Adda247 Publications For any detail, mail us at
Publications@adda247.com
Cracker Book for Bank (IBPS | SBI | RRB PO | Clerk) Mains Exams

30. (b); Let Shikha and Sameer have Rs. 100𝑥


Equivalent CI for two years at the rate of 10%
10×10
= 10 + 10 + = 21%
100
ATQ—
21 2 R×2 1 6.5×2
100x × = 100x × × + 100x × ×
100 3 100 3 100
4x×R 13x
21x = +
3 3
63x = 4x × R + 13x
4x × R = 50x
50x
R=
4x
R = 12.5%

31. (c); 2 year C.I. on 8% per annum


8×8
=8+8+ = 16.64%
100
2 year C.I. on 20% per annum
20×20
= 20 + 20 + = 44%
100
ATQ—
x×3×15 (x+4000)16.64 (2x+4000)44
+ + = 20382.4
100 100 100
0.45x + 0.1664x + 665.6 + 0.88x + 1760 = 20382.4
1.4964x = 20382.4 – 2425.6
17956.8
x= = 12000 Rs.
1.4964
Sum of Divyaraj invested = (2 × 12000 + 4000) = 28000 Rs.

32. (d); Bonus of Ankit = P Rs.


Bonus of Veer = (P + 2400) Rs.
Bonus of Sameer = (P + 4400) Rs.
Equivalent CI at 10% for two years
10×10
= 10 + 10 +
100
= 21%
Equivalent CI at 20% for two years
20×20
= 20 + 20 +
100
= 44%

26 Adda247 Publications For any detail, mail us at


Publications@adda247.com
Cracker Book for Bank (IBPS | SBI | RRB PO | Clerk) Mains Exams

ATQ—
60 15×3 P 21 (P+2400) 44
(P + 4400) × × – × − × = 132
100 100 2 100 2 100
27P+118800 65P+105600
– = 132
100 200
54P + 237600 – 65P - 105600 = 26400
11P = 105600
P = 9600
Sameer bonus = (9600 + 4400)
= 14000Rs.

33. (c); Three years SI on 15% = 15×3=45%


8×8
Equivalent two years CI on 8 %= 8 +8+ = 16.64%
100
20×20
Equivalent two years CI on 20% = 20+20+ = 44%
100
ATQ –
44(2𝑃+8000) 45𝑃 16.64(𝑃+8000)
−( + ) = 5352
100 100 100
.88P + 3520 - .45P – .1664P – 1331.2
= 5352
.2636P = 3163.2
3163,2
P= = 12000 𝑅𝑠.
.2636
Suresh borrowed = 12000×2+8000 = 32000 Rs

34. (c); ATQ,


The amount invested by him in SI, got doubled after 5 years.
Therefore ROI was 20% per annum.
Let amount invested by him in SI was x, then amount invested by
him in CI is (32000 – x)
Now,
120 3
(32000–x)( ) 36
100
=
2x 65
(32000–x)216 36
⇒ =
2x×125 65
(32000–x)3 1
⇒ =
x×25 13
⇒ 32000 × 39 = 64x
x = 500 × 39 = 19500/-

27 Adda247 Publications For any detail, mail us at


Publications@adda247.com
Cracker Book for Bank (IBPS | SBI | RRB PO | Clerk) Mains Exams

35. (b); Let man invested in scheme P, Q and R be Rs. 5x, Rs. 6x and Rs.
9x respectively
20×20
Equivalent CI of two years on 20% = 20 + 20 + = 44%
100
10×10
Equivalent CI of two years on 10% = 10 + 10 + = 21%
100
ATQ –
44 15 ×2 21
5𝑥 × + 6x × – 9x× = 1899
100 100 100
2.2x + 1.8x – 1.89x = 1899
2.11x = 1899
1899
x=
2.11
x = 900 Rs.
Total amount = 900 × (5 + 6 + 9)
= 18000 Rs.
If man invested total amount on C.I at the rate of 15% p.a.
15×15
Equivalent CI of two years on 15% = 15 + 15 + = 32.25
100
32.25
Required interest = 18000× = 5805 Rs.
100

28 Adda247 Publications For any detail, mail us at


Publications@adda247.com
Cracker Book for Bank (IBPS | SBI | RRB PO | Clerk) Mains Exams

1 Adda247 Publications For any detail, mail us at


Publications@adda247.com
Cracker Book for Bank (IBPS | SBI | RRB PO | Clerk) Mains Exams

Chapter
Time & Work and Pipe Cistern
7
BEST APPROACH TO SOLVE THE QUESTIONS

• Questions asked from this topic generally to test mental ability of a


student calculate the time in which a man/team complete the given
work while efficiency is given.
• While doing the questions from this topic, a most common approach
that student must use is that
‘if a man completes 1 unit of work in N days, then amount/part of
work done by him in 1 day is 1/𝑁,’
Assume that work of man is to eat 100 chocolates and he eats all of
them in 10 days, then amount of work done (chocolates eaten) in 1
100
day is = 10 𝑐ℎ𝑜𝑐𝑜𝑙𝑎𝑡𝑒𝑠 𝑒𝑎𝑡𝑒𝑛.
10
Or
If Rashmi can make 10 cakes in 5 hours, then amount of work done
10
(Cake made) by her in 1 hour is = 2 cakes.
5
Another easy way to solve questions from this topic is % approach i.e
‘if a man can complete a work in 20 days, then amount of work done
100
by him in 1 day is % = 5%.
20
Consider an example: Rahul can complete a work in 10 days and Arun
can complete the same work in 20 days. Find the time in which the
work will be completed if both of them work together.
There are three approaches to solve this:

(i) TRADITIONAL APPROACH


If Rahul can complete the work in 10 days, then amount of work
done by him in 1 day
1
= 𝑢𝑛𝑖𝑡𝑠.
10
Similarly amount of work done by Arun in 1 day
1
= 𝑢𝑛𝑖𝑡𝑠.
20
Hence when both of them work together, total work done is
2 Adda247 Publications For any detail, mail us at
Publications@adda247.com
Cracker Book for Bank (IBPS | SBI | RRB PO | Clerk) Mains Exams
1 1 3
+ = 𝑢𝑛𝑖𝑡𝑠.
10 20 20
3
Hence if 𝑢𝑛𝑖𝑡𝑠 𝑜𝑓 𝑤𝑜𝑟𝑘 𝑖𝑠 𝑑𝑜𝑛𝑒 𝑖𝑛 1 𝑑𝑎𝑦
20
Then 1 unit of work will be completed in 20/3 days.

(ii) LCM APPROACH


In this method, assume the LCM of days as a number of
chocolates. LCM of 10 and 20 is 20. Now assume that there were
20 chocolates, and If Rahul take 10 days to eat them all, it can be
concluded that he ate 2 chocolates per day. Similarly, Arun can eat
1 chocolate in a day.
Therefore,
They both will eat 3 chocolates in 1 day.
20
Time taken to complete whole work(to eat all of them)= 𝑑𝑎𝑦𝑠.
3
NOTE: Our answer will be same, we consider any number of
chocolates, but for easy calculation purpose we use a number that
is easily divisible by number of days given.

(iii) % APPROACH
This method is kind of same as (i). Consider the work unit as
100% work.
Now note that if Rahul take 10 days to complete 100% of work,
then work done by him in 1 day is 10%. Similarly work done by
Arun in 1 day is 5%. Therefore, both of them working together
will complete 15% of work in 1 day.
100 20
100% of work will be completed in = 𝑑𝑎𝑦𝑠.
15 3

In all these approaches, 1 common thing that occurs is to calculate


amount/part/percentage of work done in 1 day.
LCM approach is best used in the questions in which efficiency of pipe or
volume of tank is given. Consider eg.

Example 1: Pipe A and Pipe B when both opened together can fill a tank in
4 minutes. Pipe A alone can fill it in 16 minutes. Find the time in which pipe
B alone will fill the tank.

3 Adda247 Publications For any detail, mail us at


Publications@adda247.com
Cracker Book for Bank (IBPS | SBI | RRB PO | Clerk) Mains Exams

In traditional way:
1
Sol. Tank filled by Pipe A alone in 1 minutes = 𝑢𝑛𝑖𝑡𝑠.
16
Let pipe B alone can fill it in x minutes.
1
Then tank filled by pipe B alone 1 minute = 𝑢𝑛𝑖𝑡.
𝑥
1 1 1
Therefore, + =
𝑥 16 4
1 3 16
= , 𝑥=
𝑥 16 3

LCM APPROACH
Let total units of tank are 16 liters. It means A can fill 1 liter in a minute.
Assume that B fills x unit in a minute. Tank was filled in 4 minutes’ means
4(𝑥 + 1) = 16
𝑥+1 = 4
x = 3 units per minute
16
time taken is minutes.
3
All this could be done was mentally through LCM method. Give it a try!
MAN-DAYS-HOUR-WORK THEOREM
According to this theorem if more than 1 man of equal efficiency are
working together on a same work, then amount/part of work done by them
is directly proportional to the time given to work by number of man. In
more simple words work done depends on efficiency and total time given.
𝐦 𝐝 𝐡 𝐦 𝐝 𝐡
⇒ 𝟏 𝟏 𝟏= 𝟐 𝟐 𝟐
𝐰𝟏 𝐰𝟐
Here m₁, m₂ are number of men
d₁, d₂ are number of days
h₁, h₂ are working hours in given days
w₁, w₂ are part/amount of work done.
For eg. 10 men in 5 days can do same amount of work as 5 men in 10 days
or if we explore it more we can see 10 men in 6 days working 4 hours a day
can do same amount of work as 30 men in 4 days working 2 hours a day.
Please verify above two examples, by assuming a man can eat 1 chocolate in
1 hour and try to solve given problem.

Example 2: 12 Men working 4 hours a day can complete 48% of work in 8


days. Calculate how many men are required to complete 2 units of a work, if
they have to complete it in 10 days, 8 hours a day.
4 Adda247 Publications For any detail, mail us at
Publications@adda247.com
Cracker Book for Bank (IBPS | SBI | RRB PO | Clerk) Mains Exams

Sol. In this type of problems, by default we have to assume that all men
have equal efficiency and they do the work with uniform speed.
Applying Man-day-hour formulae
m1 d1 h1 m2 d2 h2
=
w1 w2
Here,
m₁ = 12 m₂ = ?
d₁ = 8 d₂ = 10
h₁ = 4 h₂ = 8
w₁ = 48 w₂ = 200

Note that, here w₂ = 200 is used because, 2 units of work means 200%
of work.
12×8×4×200
m2 =
48×8×10
m2 = 20

Another type of concept that is introduced here is when people with


unequal efficiencies are working, for eg. 4 women can do the same amount
of work in a given time as 3 men. For these type of questions, we use ratio
approach, we simply write
4W= 3M
𝑊 3
=> = .
𝑀 4

And then we consider efficiency of a men as 4 units and that of women as 3


units throughout the problem.

Few points to remember:


If the ratio of efficiencies of two persons is x:y then ratio of respective time
taken by them is y:x.
If X is N times efficient than Y, time taken by X to complete a work is
1
𝑡𝑖𝑚𝑒𝑠 𝑜𝑓 time taken by Y.
𝑁
Total work done by a man of efficiency X in N days is 𝑁𝑋 units.
5 Adda247 Publications For any detail, mail us at
Publications@adda247.com
Cracker Book for Bank (IBPS | SBI | RRB PO | Clerk) Mains Exams

Practice Exercise Based on New Pattern

1. Ratio between efficiency of Arun, Yash and Rana is 6 : 4 : 5. All three


starts to work together with same efficiency. But Rana, destroys his
60% of work in every evening, due to which they have to work 20
more days then estimated time. Find the estimated days by them to
complete the work.
(a) 80 (b) 76 (c) 84
(d) 72 (e) 90

2. Pipe P can fill a tank in 24 minutes, pipe Q can fill the same tank in 36
minutes and an outlet pipe can empty the fully filled tank in 48
minutes. Find in how many minutes tank will be filled, if for first
minute only pipe P & Q opened together and in second minute all
three pipes opened in tank alternatively?
4 4 4
(a) 15 𝑚𝑖𝑛𝑢𝑡𝑒𝑠 (b)13 𝑚𝑖𝑛𝑢𝑡𝑒𝑠 (c)18 𝑚𝑖𝑛𝑢𝑡𝑒𝑠
5 5 5
4 4
(d) 16 𝑚𝑖𝑛𝑢𝑡𝑒𝑠 (e)12 𝑚𝑖𝑛𝑢𝑡𝑒𝑠
5 5

Direction (3-4): One day earning of P, Q and R together is Rs.1026 to do a


work. ‘P’ earns more than R which is same as ‘R’ earns more than ‘Q’.
Efficiency of R and Q is 19 : 18.

3. Find the amount earned by R and P together in 5 days to do the same


work?
(a) Rs. 3620 (b) Rs. 3430 (c) Rs. 3510
(d) Rs. 3310 (e) Rs. 3710

4. S, who’s efficiency is average of efficiency of P and Q, can complete a


work in 36 days. If P, Q and R work together, and complete that work
then find the total wage of Q
(a) 3888 Rs. (b) 4104 Rs. (c) 4320 Rs.
(d) 3666 Rs. (e) 4520 Rs.

6 Adda247 Publications For any detail, mail us at


Publications@adda247.com
Cracker Book for Bank (IBPS | SBI | RRB PO | Clerk) Mains Exams

5. (X+4) men can complete a work in 2X days while (X+12) women can
complete same work in (X+8) days. If ratio of efficiency of men to
women is 5 : 4 then find in how many days 12 men and 15 women
together can complete the same work?
(a) 32 days (b) 16 days (c) 48 days
(d) 64 days (e) 80 days

6. A cistern can be filled completely by pipe A and B together in 12


hours. If pipe A works with twice speed while pipe B work with 50%
more speed than cistern can be filled completely in 7 hours. Find the
capacity of cistern if flow of water through pipe A is 2.5ℓ/ minute.
(a) 2800 liter (b) 3150 liter (c) 3300 liter
(d) 3650 liter (e) 4200 liter

7. Ratio of efficiency of A and B in completing a work is 3 : 4. Both


started to work together but A left after 2 days. Another person C
joins B and they together complete the remaining work in 6 days. If A
and B together can complete the work in 8 days then C alone can
complete the work.
27 56 41
(a) days (b) days (c) days
4 3 3
28 49
(d) days (e) days
3 3

8. Pipe A can fill a tank in 45 hr, pipe B is 50% more efficient than A and
pipe C can fill the same tank in 7.5 hr less than B. A and B opened
together for X hr and closed after that and pipe C fill remaining tank in
(X + 9) hr, if the ratio between tank filled by (A + B) together to tank
filled by pipe C is 1 : 2. Find the value of X ?
(a) 3 hr (b) 4 hr (c) 6 hr
(d) 8 hr (e) 7 hr

9. A and B can do a piece of work in 72 days and 64 days respectively. C


2
can do the same work in 2 more days as A & B take together to
17
complete. If first day A & B work together and second day B & C work
together alternatively, then in how many days work will be
completed?
13 13 13
(a) 22 𝑑𝑎𝑦𝑠 (b) 27 𝑑𝑎𝑦𝑠 (c) 32 𝑑𝑎𝑦𝑠
25 25 25
13 13
(d) 25 𝑑𝑎𝑦𝑠 (e) 29 𝑑𝑎𝑦𝑠
25 25
7 Adda247 Publications For any detail, mail us at
Publications@adda247.com
Cracker Book for Bank (IBPS | SBI | RRB PO | Clerk) Mains Exams

10. Four persons started to do a work together. ‘A’ works only in starting
two days after that B, C and D works alternately starting from B. Ratio
of time taken by A, B, C and D if they work alone is 4 : 3 : 2 : 5. If the
work is completed in 12 days then in how many days A and C can
complete the work if they work together ?
(a) 6 days (b) 12 days (c) 10 days
(d) 8 days (e) 4 days

11. A person C can complete 21% of work in 10 days while working with
1
233⅓% of his efficiency. B is 11 % more efficient than C. A, while
9
working with his half efficiency can complete the work in half time as
compared to time taken by B. Find the time taken by A & B together to
complete the 50% of whole work.
(a) 15 days (b) 10 days (c) 20 days
(d) 25 days (e) 22 days

12. Two pipes A and B can fill a cistern in 15 hours and 10 hours
respectively. A tap can empty the full cistern in 30 hours. All the three
taps were open for 2 hours, when it was remembered that the
emptying tap had been left open. It was then closed. How many hours
more would it take for the cistern to be filled ?
(a) 4hr 30 min. (b) 4hr 12 min. (c) 4 hr24 min.
(d) 4hr 35 min. (e) 4hr 54 min.

Directions (13-15): ‘B’ and ‘C’ together started to do a work ‘X’. After 6
days ‘B’ is replaced by ‘A’ who can do work ‘X’ alone in same time in which
‘B’ and ‘C’ together can do. After 4 days more ‘A’ left the work and
1
remaining work is completed by ‘C’ alone in 2 days. ‘B’ did 33 % of work ‘X’
3
in total. ‘C’ and ‘D’ can complete the same work ‘X’ in 20 days while ‘D’ is
60% less efficient than ‘B’.
Efficiency of A, B, C and D remain same for work ‘X’, ‘Y’ and ‘Z’

13. Find the ratio between work done by ‘A’ in 4 days to work done by ‘B’
in 9 days?
(a) 3 ∶ 4 (b) 1 ∶ 2 (c) 1 ∶ 3
(d) 2 ∶ 3 (e) 3 ∶ 1
8 Adda247 Publications For any detail, mail us at
Publications@adda247.com
Cracker Book for Bank (IBPS | SBI | RRB PO | Clerk) Mains Exams

14. Find in how many days ‘A’ can complete another work ‘Y’ if ‘C’ and ‘D’
together can complete work ‘Y’ in 26 days? (2 Marks)
(a) 14.4 days (b) 15.6 days (c) 18.2 days
(d) 16.6 days (e) 19.5 days

15. ‘E’ who is 60% more efficient than ‘B’ can complete another work ‘Z’
1
in 12 days. Find in how many days ‘A’ and ‘C’ together can complete
2
work ‘Z’? (2 Marks)
(a) 20 days (b) 15 days (c) 12 days
(d) 10 days (e) 8 days
1
16. Veer and Sameer alone can do 25% & 33 % of a task in 16 days
3
individually. If Sameer & Satish together can do same task in 16 days,
then find in how many days Veer, Sameer and Satish will complete the
whole task, if they do it on alternate days, starting with Satish and
followed by Sameer & Veer respectively?
(a)38 days (b)42 days (c)36 days
(d)32 days (e)30 days

17. Working alone, A can complete a task in ‘a’ days and B in ‘b’ days. They
take turns in doing the task with each working 2 days at a time. If A
starts they finish the task in exactly 10 days. If B starts, they take half
a day more. How long does it take to complete the task if they both
work together?
1 1 5
(a) 5 days (b) 5 days (c) 5 days
3 7 9
5
(d) 5 days (e) None of these
11

18. Monika can do 40% of a work in 24 days and Anshika can do 12.5% of
same work in 4.5 days. Shikha can complete the same work in equal
time as Monika and Anshika can do together. If all three starts work
alternatively, starting with Shikha and followed by Monika and
Anshika respectively, then find how many days will be required to
complete the work?
1 1 1
(a) 30 𝑑𝑎𝑦𝑠 (b) 24 𝑑𝑎𝑦𝑠 (c) 33 𝑑𝑎𝑦𝑠
3 3 2
1 1
(d) 28 𝑑𝑎𝑦𝑠 (e) 20 𝑑𝑎𝑦𝑠
3 3

9 Adda247 Publications For any detail, mail us at


Publications@adda247.com
Cracker Book for Bank (IBPS | SBI | RRB PO | Clerk) Mains Exams

19. One day efficiency of A is 20% more than B and that of C is 20% less
than B. D can do a piece of work in twice of time as A, B & C complete
same work in together. If first eight days D work alone after that he
left the work and A, B & C complete the remaining work in 12 days.
Find in what time work will be completed, if B and D work in rotation
on each day, starting with D?
1 1 1
(a) 41 𝑑𝑎𝑦𝑠 (b) 38 𝑑𝑎𝑦𝑠 (c) 36 𝑑𝑎𝑦𝑠
3 3 3
1 1
(d) 44 𝑑𝑎𝑦𝑠 (e) 46 𝑑𝑎𝑦𝑠
3 3

20. 5 inlet pipes (same capacity) can fill a tank in same time in which 3
outlet pipes (same capacity) can empty it. If 2 inlet and one outlet pipe
is opened for first minutes and 5 inlet and 2 outlet pipe open for 2 nd
minute and process continues till tank is completely filled in 30
minutes. Find the time in which 2 outlet pipe can empty the
completely filled tank.
(a) 9 minute (b) 12 minutes (c) 6 minutes
(d) 10 minutes (e) 15 minutes

Direction (21-22): P, Q, R and S four pipes can fill a cistern ‘N’ in 7 minutes
if all are opened together. P is 25% less efficient then Q which is 100%
more efferent than R. ‘S’ filled 25% of the cistern. P and Q together can fill
the cistern in ‘x’ minutes while P and S can fill half cistern in ‘y’ minutes.

21. Pipe A and B together can fill another cistern ‘M’ in (x+12) minutes
while pipe B and C together can fill cistern ‘M’ in (y+5) minutes. If pipe
6𝑥
A, B and C together can fill cistern ‘M’ in ( ) minutes, then find
𝑦
efficiency of ‘B’ is what % more/less than efficiency of ‘A’ ?
(a) 50% (b) 75% (c) 100%
(d) 150% (e) 200%

22. ‘D’ men can complete a work in (x+9) day while (y+7) men can
complete same work in ‘E’ days. If y men can complete same work in
(E+12) days then find in how many day (E–D) men can complete the
same work?
(a) 21 days (b) 42 days (c) 35 days
(d) 14 days (e) 84 days
10 Adda247 Publications For any detail, mail us at
Publications@adda247.com
Cracker Book for Bank (IBPS | SBI | RRB PO | Clerk) Mains Exams

Directions (23-24): These questions are based on the information given


below.
Each of A, B, C and D need a unique time to do a certain work. A can do the
work in 𝑥 days and B can do the work in 2𝑥 days. A started the work and do
2
it for 22 days then he is replaced by B and B completed remaining work in
9
same time as C and D together can complete the whole work.
The ratio of the efficiency of C and D is 4 : 5. If C and D work for alternative
1
days starting from C then they can do the total work in 44 days.
2

23. What is the value of 𝑥?


2 1 2
(a) 66 (b) 33 (c) 16
3 3 3
2
(d) 14 (e) none of these
7

24. A and B together can compete 225% of the work in how many days?
2
(a) 66 days (b) 60 days (c) 50 days
3
(d) 25 days (e) none of these

1
25. P can do 50% of a work in 32 days, Q can do 37 % of same work in
2
1
27 days, while R can do 62 %of same work in 30 days. Q and R
2

together started and worked for x days. After x days Q left the work
and P joined R and both completed remaining work in (x +4) days. If
ratio between work done by (Q + R) together to work done by (P + R)
is 5 : 7 then in 2x days R will complete what fraction of work?
1 1 2
(a) (b) (c)
3 4 3
1 1
(d) (e)
2 5

11 Adda247 Publications For any detail, mail us at


Publications@adda247.com
Cracker Book for Bank (IBPS | SBI | RRB PO | Clerk) Mains Exams

26. Veer and Sameer can do a work in 64 days and 72 days respectively.
Satish and Ankit together take 48 days to complete the same work.
Ankit is 40% more efficient than Satish. If Veer and Sameer work for
18 days together, after that both left the work, then find in how many
days remaining work will be completed by Satish and Ankit if they
work on alternated days and Ankit starts the work?
6 6 6
(a) 40 𝑑𝑎𝑦𝑠 (b) 42 days (c) 48 𝑑𝑎𝑦𝑠
7 7 7
6 4
(d) 44 𝑑𝑎𝑦𝑠 (e) 49 𝑑𝑎𝑦𝑠
7 5

27. Veer takes 50% more time to complete a task than that of Satish takes
2
to complete the same task, while Mahendra takes 2 days less than to
5
complete same task as Veer and Satish takes together. If Satish
1
complete 𝑟𝑑 of task in 8 day, then find in how many days work will
3
be completed, if Veer and Mahendra work in rotation, starting with
Mahendra?
(a) 16 days (b) 18 Days (c) 24 days
(d) 12 days (e) 22 days

28. There are 3 pipes A, B and C. A can fill a bucket in 8 minutes. C can fill
18 3
buckets in 18 minutes & pipe B can fill buckets in a minute. These
5 20
pipes are opened in a tank alternatively, 1 minute each starting from
A, then C, then B. If tank is filled after 2 hours, then find the volume of
tank if the capacity of bucket is 5 liters.
(a) 88 liters (b) 95 liters (c) 90 liters
(d) 105 liters (e) 92 liters

29. Efficiency of Ram is 25% more than Shyam who completes a task in
2
60 days. Ghanshyam takes 6 days less than the days taken by Ram
3
and Shyam together to complete the work. If Ram and Shyam work for
16 days, after that both left the task then find in how many days
Ghanshyam will complete remaining work?
(a) 6 days (b) 8 days (c) 4 days
(d) 5 days (e) 10 days

12 Adda247 Publications For any detail, mail us at


Publications@adda247.com
Cracker Book for Bank (IBPS | SBI | RRB PO | Clerk) Mains Exams
1
30. A child can do th of work in same time as his father can do the whole
5
work and efficiency of child’s mother is equal to half the sum of father
and child efficiency. If they complete 70% of work in 24 days, when
they work alternatively, starting with father and child together on
first day followed by father and mother together on second day. Find
in how many days mother can complete 85% of work alone?
(a) 48 days (b) 68 days (c) 60 days
(d) 56 days (e) 54 days

4
31. Ankit and Apoorv together can finish a work in 8 𝑑𝑎𝑦𝑠 while Ankit
7
can do it individually in 15 days. Both work on another task for (x + 4)
90
days and (x + 20) days respectively. If remaining % of the second
7
task is completed by Bhavya in 18 more days with the efficiency of 2
units work/day. Then, find what portion of the second task will be
completed in x days, if all three work together?
29 26 31
(a) (b) (c)
35 35 35
23 27
(d) (e)
35 35

32. One day efficiency of Raj is equal to one day efficiency of Divyaraj and
Veer together. Divyaraj is 25% more efficient than Veer and starts
with 20% more of his efficiency with Veer and works for x days, after
that Divyaraj left the work and remaining work will be completed by
Veer and Sameer together in (x + 8) days. If ratio of work done by
Divyaraj and Veer together to work done by Veer and Sameer is 5 : 7
and efficiency of Sameer is 25% less than Veer. Then find Raj and
Sameer complete work together in how many days?
(a) 14 days (b) 12 days (c) 16 days
(d) 18 days (e) 20 days

13 Adda247 Publications For any detail, mail us at


Publications@adda247.com
Cracker Book for Bank (IBPS | SBI | RRB PO | Clerk) Mains Exams

33. Two pipes P and Q can fill tank A in 28 minutes and 56 minutes
respectively and empty pipe M can empty same tank in 42 minutes. If
all three pipes opened in tank B for (x – 24) minutes together they
filled 90 liter of the tank which is 25% of the quantity tank B. Find in
‘x’ minutes what portion of tank B filled, if all pipe P and Q and M
opened alternatively starting with P, followed by Q and M
respectively?
7 7 5
(a) (b) (c)
12 36 36
3 7
(d) (e)
35 41

34. There are two pipes A & B, pipe A is for filling the swimming pool and
pipe B is to empty the swimming pool. Capacity of swimming pool is
5040 m3 and volume of pipe B is 8 m3/minute more than that of pipe
1
A. If pipe A takes 11 more minutes to fill same swimming pool, than
4
time taken by B to empty the same swimming pool. If pipe B can
empty second swimming pool in 112.5 minutes, then find the capacity
of second swimming pool?
(a) 7200 m3 (b) 6400 m3 (c) 5600 m3
(d) 7800 m3 (e) 8400 m3

35. Two pipes P and Q can fill tank A in 28 minutes and 56 minutes
respectively and empty pipe M can empty the tank in 42 minutes.
Tank A have the capacity of 168 liters. If all three pipes opened in tank
B for (x – 24) minutes together they filled 90 liter of the tank which is
25% of the quantity of tank B. Find in x minutes what portion of tank
B filled, if all pipe P and Q and M opened alternatively in each minute
starting with P, followed by Q and M respectively?
5 7 9
(a) (b) (c)
36 36 38
7 7
(d) (e)
38 39

14 Adda247 Publications For any detail, mail us at


Publications@adda247.com
Cracker Book for Bank (IBPS | SBI | RRB PO | Clerk) Mains Exams

Directions (36-38): Working efficiency of A is 20% more than that of B. B


can complete a work ‘X’ in 36 days.
B and C together started to complete the work ‘X’ and after 10 days they
both left the work and then remaining work is done by A alone in 15 days.
A and C together started to complete another work ‘Y’ and after working for
12 days they both left the work. Remaining work is done by B alone in 16
days. D first completed work ‘X’ and then completed work ‘Y’ in total 38
days.
It is given that efficiency of all, in completing work ‘X’ and work ‘Y’ is same.
1
36. A, B and C working together completed rd of work ‘X’, and then A and
3
C are replaced by D. Now remaining of work ‘X’ is completed by B and
D together. For how many days B worked?
(a) 12 days (b) 10 days (c) 15 days
(d) 4 days (e) 8 days

37. A, C and D working simultaneously completed work ‘X’ in ‘n’ days and
A, B, C and D working simultaneously completed work ‘Y’ in ‘m’ days.
Find the value of (𝑚 + 𝑛).
(a) 15 days (b) 10 days (c) 12 days
(d) 8 days (e) 17 days

38. A person E starts the work ‘X’ and leave after 12 days, then B and C
complete the remaining work in 8 days. What is the ratio of number of
days taken by A and E together to complete the work ‘X’ to the
number of days taken by D, B and C together to complete the both
work ‘X’ and ‘Y’ .
(a) 3 : 5 (b) 5 : 3 (c) 8 : 7
(d) 1 : 2 (e) 2 : 3

Directions (39-40): Ankur & Shubham can do a piece of work 64 days


together and Shubham &Kartik can do the same work in 72 days together.
First 22 days Ankur & Shubham work together, after that Ankur left the
work and Kartik joined Shumbham. Kartik and Shubham work for next six
days, after that Shubham left the work and remaining work complete by
Kartik & Anup in 30 more days. Efficiency of Anup is 75% more than
efficiency of Kartik.
15 Adda247 Publications For any detail, mail us at
Publications@adda247.com
Cracker Book for Bank (IBPS | SBI | RRB PO | Clerk) Mains Exams

39. Kartik and Anup work for x days, Ankur and Shubham work for next
2 1
(x+2 ) days, If remaining 12 % 𝑜𝑓 𝑡𝑜𝑡𝑎𝑙 work complete by Prabhat
3 2
in 6 more day. Prabhat will do, what portion of total work in (x+12)
days?
2 1 3
(a) (b) (c)
3 4 4
2 2
(d) (e)
3 5

40. Ankur and Shubham start work alternatively and work for y days.
𝑦
After that Kartik and Anup replace them and work for next days
2
75
alternatively. If % of total work still remain, which complete by
8
Abhimanyu in 9 more days. Find if Abhimanyu work for (y+16) days,
then in how many days remaining work will be completed by Anup
and kartik? (y is divisible by 4)
48 34
(a) days (b) days (c) 20 days
11 11

(d) 28 days (e) 32 days

16 Adda247 Publications For any detail, mail us at


Publications@adda247.com
Cracker Book for Bank (IBPS | SBI | RRB PO | Clerk) Mains Exams

Solutions

1. (a); Ratio of efficiency of Arun, Yash and Rana is 6 : 4 : 5


Total work done by them in 1 day
= (6 + 4 + 5) units = (15) units.
Let they estimate 𝑥 days to complete the work.
Then total work = 15𝑥
40
But Rana’s 1-day work is only 40%, i.e. × 5 = 2 unit
100
The work done by them in actual
= (6 + 4 + 2) (x + 20) = 12(x + 20)
∴ 15𝑥 = 12𝑥 + 240
3𝑥 = 240
𝑥 = 80
Hence, estimated days are 80.

2. (d);

First minutes (P + Q) = (6 + 4) = 10 unit


Second minutes (P + Q – R)
= (6 + 4 – 3) = 7 unit
17 unit in 2 minutes
So,
17 × 8 = 136 unit in 16 minutes
(144 –136)
Remaining by (P + Q) =
10
4
= minutes
5
4
Total time = 16 +
5
4
= 16 minutes
5

17 Adda247 Publications For any detail, mail us at


Publications@adda247.com
Cracker Book for Bank (IBPS | SBI | RRB PO | Clerk) Mains Exams

Solution (3-4): Let R earns = y


And P earns = y + x
So, Q earns = y – x
ATQ,
y – x + y + y + x = 1026
y = 342
Each person earns according to their efficiencies so
342
Q earns → × 18 = 324
19
342
An P earns = × 20 = 360
19
Ratio of efficiency of P, R and Q
= 20 : 19 : 18
Money per unit work = Rs.18

3. (c); Amount earned by R and P together in 5 days


Work done = (20 + 19) × 5 = 195
Money earned = 195 × 18 = Rs.3510
20+18
4. (a); Efficiency of S = = 19
2
Total work → 19 × 36
Days taken by P, Q and R to do that work
19×36
⇒ = 12 day
(20+19+18)
Q’s earning ⇒ 12 × 324 = Rs.3888

5. (b); (X+4) men can complete work in 2X days


⇒ 1 man can complete same work in 2X(X+4) days
(X+12) women can complete work in (X+8) days
⇒ 1 woman can complete same work in (X+8)(X+12) days
Now ratio of efficiency of men to women is 5 : 4
⇒ Ratio of days taken by men to women is
4:5
4 2X(X+4)
So, =
5 (X+8)(X+12)
2
⇒ 2(𝑋 + 20𝑋 + 96) = 5𝑋(𝑋 + 4)
⇒ 2𝑋 2 + 40𝑋 + 192 = 5𝑋 2 + 20𝑋
⇒ 3𝑋 2 − 20𝑋 − 192 = 0
⇒ 3𝑋 2 − 36𝑋 + 16𝑋 − 192 = 0
18 Adda247 Publications For any detail, mail us at
Publications@adda247.com
Cracker Book for Bank (IBPS | SBI | RRB PO | Clerk) Mains Exams

⇒ 3𝑋(𝑋 − 12) + 16(𝑋 − 12) = 0


⇒ (3𝑋 + 16)(𝑋 − 12) = 0
16
⇒ 𝑋 = 12, −
3
16×24
12 men can complete work in
12
= 32 𝑑𝑎𝑦𝑠
24×20
15 women can complete work in
15
= 32 𝑑𝑎𝑦𝑠
32×32
Required time = = 16 𝑑𝑎𝑦𝑠
32+32

6. (e); Let, pipe A and pipe B alone can fill the tank in x and y hours
respectively.
ATQ,
1 1 1
+ = … (i)
x y 12
2 1.5 1
and, + = … (ii)
x y 7
On solving (i) and (ii)
x = 28 hour ⇒ y = 21 hour
Capacity of cistern = 28 × 60 × 2.5
= 4200 liter

7. (b); Let A and B can do 3𝑥 and 4𝑥 unit of work in one day.


So,
Total work = (3𝑥 + 4𝑥 ) × 8 = 56𝑥
(A + B) two day work = 7𝑥 × 2 = 14𝑥
Remaining work = 42𝑥
In 6 days B will complete
= 6 × 4𝑥 = 24𝑥 units
So, remaining 18𝑥 units are completed by C in 6 day So,
56𝑥 unit will be completed in
56𝑥 56
= 18𝑥 = days
3
6

19 Adda247 Publications For any detail, mail us at


Publications@adda247.com
Cracker Book for Bank (IBPS | SBI | RRB PO | Clerk) Mains Exams

8. (c); A = 45 hr
A : B = 100 : 150 = 2 : 3
Total capacity of tank = 45 × 2 = 90 liter
90
C= – 7.5 = 22.5 hr
3
90 ℓ
C efficiency = =4
22.5 hr
5X 1
According to question ⇒ =
4(X+9) 2
⇒ 10X – 4X = 36
X = 6 hr

9. (b);

576 36 612
C takes = (8+9) + = = 36 days
17 7
576
Efficiency of C = = 16 units/days
367
When First day (A and B) and second day (B and C) work
alternatively
Two day work = (A and B) one day work and (B and C) one day
work
= (8 + 9) + (9 + 16) = 17 + 25
= 42 units
26
In 26 day = × 42 = 546 units
2
(A and B) on 27th day = 17 units
After 27 days remaining work
= (576 – 546 – 17) = 13 units
13
13 units work done by (B and C) on 28th day =
25
13 13
Total time = (27 + ) = 27 days
25 25

20 Adda247 Publications For any detail, mail us at


Publications@adda247.com
Cracker Book for Bank (IBPS | SBI | RRB PO | Clerk) Mains Exams

10. (d); A, B, C and D worked for 2 days together after that A leave and B,
C and D worked alternatively for 10 days starting from B
∴ B worked for 4 days, C for 3 days, and D for 3 days.
Total days A worked = 2
Total days B worked = 4 + 2 = 6
Total days C worked = 3 + 2 = 5
Total days D worked = 3 + 2 = 5
Let, their alone time to complete the work is 4x, 3x, 2x and 5x
respectively.
2 6 5 5
∴ + + + =1
4x 3x 2x 5x
30+120+150+60
⇒ =1
60𝑥
360
⇒𝑥= =6
60
‘A’ can complete the work in 4 × 6 = 24 days
‘C’ can complete the work in 2 × 6 = 12 days
12×24 12×24
Required time = = = 8 days
12+24 36

11. (b); Let the efficiency of C [is C units/days] total work is 100a Units
ATQ,
7
c × 10 = 21a
3
1 7
[233 % = ]
3 3
9𝑎
C= 𝑈𝑛𝑖𝑡𝑠 /𝑑𝑎𝑦
10
1
Also B is 11 % more efficient means
9
10
B= of C
9
10 9𝑎
B = × Units/day
9 10
= a Units / day
A which while working with half efficiency total half time.
∴ A : B = 4 : 1 [efficiency ratio]
Efficiency of A = 4a
Work done by then together is 4a + a
= 5a/day
Time take to complete 50 a Units
50𝑎
= = 10 days
5𝑎

21 Adda247 Publications For any detail, mail us at


Publications@adda247.com
Cracker Book for Bank (IBPS | SBI | RRB PO | Clerk) Mains Exams

12. (c);

In 2 hours, part of the tank filled


= 2(4) = 8 units
(30−8)
Now required time =
5
22
= = 4hour 24mins.
5

Solution (13-15): Let, efficiency of A, B, C and D be ‘a’, ‘b’, ‘c’ and ‘d’
respectively
Total work = 6b + 6c + 4(a +c) + 2c
And also, a = b + c
⇒ Total work = 10b + 16c
ATQ,
1
B did of work in 6 days
3
⇒ B can complete whole work in 18 days
And,
(10b + 16c) = 18b ⇒ 16c = 8b
𝑏 2
⇒ =
𝑐 1
⇒ ‘C’ can complete whole work ‘X’ in 36 days
18×36
A can complete whole work ‘X’ in = 12 day.
18+36
18
D can complete whole work ‘X’ in × 5 = 45 days.
2

Ratio of efficiency of A, B, C and D


1 1 1 1
= ∶ ∶ ∶
12 18 36 45

= 15 : 10 : 5 : 4

22 Adda247 Publications For any detail, mail us at


Publications@adda247.com
Cracker Book for Bank (IBPS | SBI | RRB PO | Clerk) Mains Exams
4×15 60 2
13. (d); Required ratio = = =
9×10 90 3

14. (b); Let efficiency of A, B, C and D be 15x, 10x, 5x and 4x respectively


Total work ‘Y’ = (5x + 4x) × 26
= 9x × 26 =234x
234𝑥
‘A’ can complete work ‘Y’ in = 15.6 day
15

15. (d); Efficiency of ‘E’ = 16x


25
Total work ‘Z’ = 16𝑥 × = 200x
2
‘A’ and ‘C’ together can complete work ‘Z’ in
200𝑥 200𝑥
= (15+5)𝑥 = = 10 days
20𝑥

16. (a); Veer can complete the whole task alone


= 16 × 4 = 64 𝑑𝑎𝑦𝑠
Sameer can complete the same task alone
= 16 × 3 = 48 𝑑𝑎𝑦𝑠
Total work = 192 𝑢𝑛𝑖𝑡𝑠(LCM of 64 and 48)
Efficiency of Satish
192 192
= − = 8 𝑢𝑛𝑖𝑡𝑠/𝑑𝑎𝑦
16 48
If all three work alternatively
First day by Satish = 8 units
Second day by Sameer = 4 units
Third day by Veer = 3 units
Total work in three days = 8 + 4 + 3
= 15 units
36
In total 36 days = × 15 = 180 𝑢𝑛𝑖𝑡𝑠
3
Satish on 37 days = 8 units
Remaining work after 37 days
= 192 – 180 – 8 = 4 units
On 38 days remaining work by Sameer
4
= = 1𝑑𝑎𝑦𝑠
4
Total time = 38 days

23 Adda247 Publications For any detail, mail us at


Publications@adda247.com
Cracker Book for Bank (IBPS | SBI | RRB PO | Clerk) Mains Exams

17. (b); If each works 2 days at a time alternately starting with A, the
work is completed in exactly 10 days.
∴ A works for 6 days and B worked for 4 days.
6 4
+ =1 ………….(i)
a b
If B starts, the work is completed in 10.5 days.
∴ B works for 6 days and A worked for 4.5 days.
6 4.5
+ =1 ………….(ii)
b a
By solving (i) and (ii)
a = 9 days
And, b = 12 days
Time taken by A and B working together to complete the work
1 1
=1 1 =1 1
+ +
a b 9 12
36 1
= = 5 days
7 7

18. (c); Monika can do whole work alone


24
= × 100 = 60 days
40
Anshika can do whole work alone
4.5
= × 100 = 36 days
12.5
Let Total work = 180 units (LCM of days taken by Anshika and
Monika)
180
Efficiency of Monika = = 3 units/day
60
180
Efficiency of Anshika = = 5 units/day
36
Shikha take to complete whole work alone
180
= (3+5) = 22.5 days
Efficiency of Shikha = 8 units/day
when Shikha, Monika & Anshika work alternatively—
Work done on First day by Shikha = 8 units
Work done on First day by Monika = 3 units
Work done on First day by Anshika
= 5 units
In three days = (8 + 3 + 5) = 16 units

24 Adda247 Publications For any detail, mail us at


Publications@adda247.com
Cracker Book for Bank (IBPS | SBI | RRB PO | Clerk) Mains Exams
33
In total 33 days = × 16 = 176 units
3
Remaining work complete by Shikha
(180 –176) 1
= = days
8 2
1
Total time = 33 +
2
1
= 33 days
2

19. (b); Let efficiency of B = 100


So, efficiency of A = 120
80
And efficiency of C = 100 × = 80
100
100+120+80
Efficiency of D = = 150
2
Ratio of efficiency of A, B, C and D
= 6 : 5 : 4 : 7.5
Let one day work of A , B , C & D be 6x units, 5x units, 4x units &
7.5x units respectively
ATQ –
Total work
= 7.5x × 8 + (5𝑥 + 6𝑥 + 4𝑥) × 12
= 240x units
When B & D work alternatively
First day by D = 7.5x units
Second day be B = 5x units
Two day work of = 7.5x + 5x = 12.5x units
In 38 days total work
38
= × 12.5𝑥 = 237.5𝑥 units
2
Remaining work = 240x – 237.5x = 2.5x
2.5𝑥
Remaining work by D on 39 days =
7.5𝑥
1
= 𝑑𝑎𝑦𝑠
3
1
Total time = 38 𝑑𝑎𝑦𝑠
3

25 Adda247 Publications For any detail, mail us at


Publications@adda247.com
Cracker Book for Bank (IBPS | SBI | RRB PO | Clerk) Mains Exams

20. (a); Let efficiency of one inlet pipe = 3x


Efficiency of 5 inlet pipes = 5 × 3x = 15x
15x
Efficiency of 1 outlet pipe = = –5x
3
ATQ,
15 (2 × 3x + 1 × (–5x)) + 15 (5 × 3x + 2 × (–5x)) = Tank capacity
Tank capacity = 90x
90x
Required time = = 9 minutes
10x

Solution (21-22): Let efficiency of R is 2 units/ minute


⇒ Efficiency of Q is 4 units/ minute
So Efficiency of P is 3 units/ minute
P + Q + R together completed 75% of work
⇒ 75% of work = 9 units
⇒ 100% of work = 12 units
Efficiency of S is 3 units/minute
Total work = 7 × (12) = 84 unit
Number of days taken by P, Q, R and S individually: -

28×21
P and Q together can fill the cistern =
28+21
28×21
= ⇒ x = 12 min.
49
28×28
P & S together can fill half cistern =
28+28
y = 7 min.
26 Adda247 Publications For any detail, mail us at
Publications@adda247.com
Cracker Book for Bank (IBPS | SBI | RRB PO | Clerk) Mains Exams

21. (c); A and B together can fill cistern ‘M’ in (x+12) minutes
= 12+12=24 minutes
B and C together can fill cistern ‘M’ in (y+5) minutes
= 7 + 5 = 12minutes
6𝑥
A, B and C together can fill cistern ‘M’ in ( ) minutes
𝑦
6×12 72
= = minutes
7 7

A’s efficiency = 49 – 42 = 7 units/minute


C’s efficiency = 49 – 21 = 28 units/ minute
B’s efficiency = 49 – 7 – 28
= 14 units/minute
14−7
Required % = × 100
7
7
= × 100 = 100%
7

22. (b); ATQ,


D × (12+9) = (7+7) × E
3
⇒D× =𝐸
2
And (7+7) × E = 7 × (E+12)
⇒ 2E = E + 12
⇒ E = 12
⇒D=8
Let (E – D) men can complete same work in ‘a’ days
ATQ,
(12−8) × a = 8 × (12+9)
a = 2 × 21 = 42 days

27 Adda247 Publications For any detail, mail us at


Publications@adda247.com
Cracker Book for Bank (IBPS | SBI | RRB PO | Clerk) Mains Exams

23. (b); Let the C do in one day = 4𝑦 work


Let the D do in one day = 5𝑦 work
2 day work of C + D = 9𝑦
In 44 day they will complete 9𝑦 × 22
= 198𝑦
1
In another days 2𝑦 work will be done
2
200y
C will take = days = 50 day
4y
D will take = 40 days.
C and D will complete work together in
50×40 200
= =
90 9
According to condition
200 200
+ =1
9𝑥 9 × 2𝑥
400+200
=1
18𝑥
1
⇒ 𝑥 = 33
3

1
24. (c); A – 33 𝑑𝑎𝑦𝑠
3
2
B − 66 days
3
200
(A+𝐵)− 𝑑𝑎𝑦𝑠
9
200 9
Required Time − × = 50 𝑑𝑎𝑦𝑠
9 4

25. (d); P can do whole work alone = 64 days


Q can do whole work alone
27
= × 8 = 72 days
3
R can do whole work alone
30
= × 8 = 48 days
5

28 Adda247 Publications For any detail, mail us at


Publications@adda247.com
Cracker Book for Bank (IBPS | SBI | RRB PO | Clerk) Mains Exams

ATQ—
(8+12)𝑥 5
(9+12)(𝑥+4)
=
7
140𝑥 = 105𝑥 + 420
420
𝑥=
35
𝑥 = 12 days
In 2𝑥 days work done by R = 2 × 12 × 12
= 288 units
288 1
Required portion = =
576 2

26. (d);

Let efficiency of Satish is 5x unit/day and that of Ankit is 7x


unit/day
ATQ—
5x + 7x = 12 unit
x = 1 unit/day
Efficiency of Satish = 5 unit/day
Efficiency of Ankit = 7 unit/day
Work done by Veer and Sameer in 18 days
= (9 + 8) × 18 = 306 unit
Remaining work = 576 – 306Ru = 270 unit
When Ankit and Satish work alternatively
Then two days’ work = (7 + 5) = 12 units
44
In 44 days total work done = × 12
2
= 264 unit
(270 –264) 6
On 45th days remaining work done by Ankit = = days
7 7
6
Total time = 44 days
7

29 Adda247 Publications For any detail, mail us at


Publications@adda247.com
Cracker Book for Bank (IBPS | SBI | RRB PO | Clerk) Mains Exams

27. (b); Let Veer take 3𝑥 day and Satish take 2𝑥 days to complete the
task
So, efficiency of Veer and Satish be 2𝑥 units/day & 3𝑥 units/day
respectively.
Whole work completed by Satish in 24 days
24
So, while task done by Veer alone = × 3𝑥 = 36 days
2𝑥
Total task = 72𝑥 units
72𝑥
Veer and Satish do it together =
5𝑥
2
= 14 days
5
Mahendra do same task alone
72 12
= – = 12 days
5 5
72𝑥
Efficiency of Mahendra = = 6𝑥 unit/day
12
When Veer and Mahendra do it alternatively —
First day task = 6𝑥 unit
Second day task = 2𝑥 units
In two day = 8𝑥 unit
72𝑥
Task completed = × 2 = 18 days.
8𝑥

28. (b); When these pipes are opened for two hours, means each of them
open for 40 min. individually.
Pipe A can fill 1 bucket in 8 minutes
& 5 buckets in 40 minutes …(i)
18×40
Similarly pipe C can fill
5×18
= 8 bucket in 40 min. …(ii)
3
And Pipe B can fill × 40 = 6 buckets in 40 minutes …(iii)
20
Hence volume of tank = 5 + 6 + 8
= 19 buckets
= 19 × 5
= 95 liters

30 Adda247 Publications For any detail, mail us at


Publications@adda247.com
Cracker Book for Bank (IBPS | SBI | RRB PO | Clerk) Mains Exams

29. (b); Efficiency of Ram :Shyam = 125 : 100


=5:4
60
Ram takes = × 4= 48 days
5
Let total work = 60×4= 240 unit
Ram and Shyam takes together
240 80
= = days
(5+4) 3
80 20
Ghanshyam takes = − = 20 days
3 3
240
Efficiency of Ghanshyam = = 12 unit/day
20
Rams and Shyam 16 days work
16 × (5 + 4)= 144 unit
240−144
Remaining work by Ghanshyam =
12
96
= = 8 days
12

30. (b); Let efficiency of child and father be x unit/day and 5x unit/day
x+5x
Efficiency of mother =
2
= 3x unit⁄day
ATQ,
They start work alternatively—
First day efficiency =(5x + x) = 6x
Second day efficiency= (5x + 3x) = 8x
6x×12+8x×12
Total work = × 100
70
168x
= × 100
70
= 240x
Mother complete 85% of work in
85
240x×100
=
3x
204x
=
3x
= 68 days

31 Adda247 Publications For any detail, mail us at


Publications@adda247.com
Cracker Book for Bank (IBPS | SBI | RRB PO | Clerk) Mains Exams
36×100×7
31. (e); Total work = = 280 units
90
ATQ -
4 (x + 4) + 3 (x + 20) + 18 × 2 = 280
4x + 16 + 3x + 60 = 280 – 36
7x = 168
x = 24 days
They all together in 24 days
= 24 (4 + 3 + 2)
= 216 units
216
Required portion =
280
27
=
35

32. (c); Ratio of efficiency of Divyaraj : Veer


= 125 : 100
=5:4
3
Efficiency of Sameer = 4 ×
4
= 3 units/day
ATQ,
120
(5× +4)x 5
100
=
(4+3)(x+8) 7
10x 5
=
7x+56 7
70x – 35x = 280
x = 8 days
Total work = 10 × 8 + 7 × 16
= 80 + 112
= 192
Raj efficiency = 9 units/day
Sameer and Raj efficiency = (3 + 9)
= 12 units/day
192
(Sameer + Raj) =
12
= 16 days
32 Adda247 Publications For any detail, mail us at
Publications@adda247.com
Cracker Book for Bank (IBPS | SBI | RRB PO | Clerk) Mains Exams

33. (b); ATQ


For tank A —

ATQ
For tank B —
6 (x – 24) + 3 (x – 24) – 4 (x – 24) = 90
6x – 144 + 3x – 72 – 4x + 96 = 90
5x = (90 + 144 + 72 – 96)
210
x= = 42 minutes
5
Total quantity of tank B = 90 × 4 = 360 liter
Alternatively (P + Q – M) for 42 minutes, means each work pipe
for 14 minutes—
All three in 14 minutes
(P + Q – M) = 14 × 6 + 14 × 3 – 14 × 4
= 70 liter
70 7
Filled portion = =
360 36

34. (a); Let capacity of pipe A = y m3


So, capacity of pipe B = y + 8 m3
Required time to filled the swimming pool
5040
= minutes
𝑦
5040
Required time to empty the swimming pool = minutes
(𝑦+8)
ATQ –
5040 5040 45
– (𝑦+8) =
𝑦 4
112 112 1
− (𝑦+8) =
𝑦 4
3584 = y + 8y
2

y2 + 8y −3584 = 0
y2 + 64y – 56𝑦 − 3584 = 0
y (y + 64) – 56(𝑦 − 64) = 0
(y + 64) (𝑦 − 56) = 0
Y = 56 m3
Capacity of second swimming pool
= (56 + 8) × 112.5 = 7200 m3
33 Adda247 Publications For any detail, mail us at
Publications@adda247.com
Cracker Book for Bank (IBPS | SBI | RRB PO | Clerk) Mains Exams

35. (b); ATQ


For tank A —

ATQ
For tank B —
6 (x – 24) + 3 (x – 24) – 4 (x – 24) = 90
6x – 144 + 3x – 72 – 4x + 96 = 90
5x = (90 + 144 + 72 – 96)
210
x= = 42 minutes
5
Total quantity of tank B = 90 × 4 = 360 liter
Alternatively (P + Q – M) for 42 minutes, means each pipe for 14
minutes—
All three in 14 minutes
(P + Q – M) = 14 × 6 + 14 × 3 – 14 × 4
= 70 liter
70 7
Filled portion = =
360 36

Solutions (36-38):

Lets efficiency of Kartik is x unit/day and Anup is 1.75x unit/day


ATQ—
22 days (Ankur + Shubham) + 6 days (Shubham + Kartik) + 30 day
(Kartik + Anup) = 576
22 × 9 + 6 × 8 + 30 (x + 1.75x) = 576
30 (2.75x) = 576 – 246
330
2.75x =
30
11
x= ⇒ x = 4 unit/day
2.75
efficiency of Anup = 4 × 1.75 = 7 unit/day

34 Adda247 Publications For any detail, mail us at


Publications@adda247.com
Cracker Book for Bank (IBPS | SBI | RRB PO | Clerk) Mains Exams

36. (a); Working efficiency of A = 120% of working efficiency of B


B can complete ‘X’ work = 36 days
A can complete ‘X’ work = 30 days
Let ‘X’ work = 180 unit
A working efficiency = 6 unit/day
B working efficiency = 5 unit/day
15 × 6 + 10 × 5 + 𝐶 × 10 = 180
so, C working efficiency = 4 unit/day
For work ‘Y
Total of work ‘Y’=(6 + 4) × 12 + 16 × 5
= 200 𝑢𝑛𝑖𝑡𝑠
Total units of both work ‘X’ and work ‘Y’
= 180+200= 380 units
380
so,D working efficiency =
38
= 10 unit/day
Now:
A, B and C work together in ‘x’ work
1
= × 180 unit = 60 unit
3
A + B + C = (6 + 4 + 5) unit per day
60
= = 4 days
15
Remaining work = 120 unit
(B + D) ⇒ (5 + 10) unit per day
120
= = 8 days.
15
B work for = 4 + 8 = 12 days.

37. (e); ‘X’ work = 180 unit


A + C + D ⇒ 6 + 4 + 10 = 20 unit/day
180
Days = = 9 days
20
Work = 200 unit
(A + B + C + D) ⇒ (6 + 5 + 4 + 10)
= 25 unit/day
200
= = 8 days
25
Total time = (9 + 8) = 17 days

35 Adda247 Publications For any detail, mail us at


Publications@adda247.com
Cracker Book for Bank (IBPS | SBI | RRB PO | Clerk) Mains Exams

38. (a); Let efficiency of E is Z unit/day


he work for 12 days
work complete = 12Z unit
B and C work for 8 days = (5 + 4) × 8 unit
= 72 unit
Remaining work = 180 - 72 = 108 unit
108
Efficiency of E = = 9 unit⁄day
12
Now,
A and E completed work ‘X’
180
= = 12 days
15
D, B and C completed both work ‘X’ and ‘Y’
200+180
= = 20 days
19
Ratio = 12 : 20 ⇒ 3 : 5

2
39. (c); (Kartik + Anup) × x days + (Ankur + Shubham) × (𝑥 + 2 ) day
3
87.5
= 576 ×
100
3x+8 7
(4 + 7) × x + 9 × ( ) = 576 ×
3 8
11x + 9x + 24 = 504
20x = 480
x = 24 days
1
576×
8
efficiency of Prabhat = = 12 unit/day
6

Prabhat will do in (x + 12) days


= (24 + 12) × 12 = 432 unit
432 3
Required portion = =
576 4

𝑦
40. (a); If Ankur and Shubham work alternatively, then both work for
2
days individually
𝑦
Same, if Kartik and Anup work for day alternatively both work
2
𝑦
for day individually
4

36 Adda247 Publications For any detail, mail us at


Publications@adda247.com
Cracker Book for Bank (IBPS | SBI | RRB PO | Clerk) Mains Exams

ATQ—
y y y y
Ankur × + Shubham × + Kartik × + Anup × =
2 2 4 4
75 1
576 × (100 – ) ×
8 100
or
y y
(Ankur + Shubham) × + (Kartik + Anup) × = 522
2 4
𝑦
9 × 𝑦 + (4 + 7) × = 522 ∗ 2
2
y = 72 days
Efficiency of Abhimanyu
75 1
576× 8 ×100
= = 6 unit/day
9
Abhimanyu work did in (y + 16) days
= (72 + 16) × 6 = 528 unit
576 –528 48
Remaining work done by Anup and Kartik in = = days
11 11

37 Adda247 Publications For any detail, mail us at


Publications@adda247.com
Cracker Book for Bank (IBPS | SBI | RRB PO | Clerk) Mains Exams

1 Adda247 Publications For any detail, mail us at


Publications@adda247.com
Cracker Book for Bank (IBPS | SBI | RRB PO | Clerk) Mains Exams

Chapter
Speed Time and Distance
8
BEST APPROACH TO SOLVE THE QUESTIONS

Speed is defined as the distance covered in per unit time. Mathematically,


𝐷𝑖𝑠𝑡𝑎𝑛𝑐𝑒
Speed= .
𝑇𝑖𝑚𝑒

The whole concept of TSD (Time, speed and distance) lies within this
formula. Three most logical points in TSD are:
1. When speed is constant,
Distance is directly proportional to time i.e. D ∝ 𝑇
2. When distance is constant,
1
Speed is inversely proportional to time i.e. S ∝
𝑇
3. When time is constant,
Speed is directly proportional to distance i.e. S ∝ 𝑇

Another important term that we must know while solving questions in this
chapter is RELATIVE SPEED.
Relative speed is defined as the net speed of two entities when they are in
motion.

If the two entities are moving in opposite direction to each other,


net/relative/resultant speed is sum of the speeds. And when they move in
the same direction, their net/relative speed is the difference of their speeds.

Example: ‘A and B stay next to each other and study in the same school. A
and B start walking towards their school at speeds of 2 km/hr and 3 km/hr
respectively. The faster of the two reaches the school first, turns around and
starts walking back. If A and B meet 200 m away from the home. What is the
distance between their residences and their school?’

2 Adda247 Publications For any detail, mail us at


Publications@adda247.com
Cracker Book for Bank (IBPS | SBI | RRB PO | Clerk) Mains Exams

Solution: Let the distance where they met be ‘x’ metres from the school.
Distance travelled by A = 200m
Distance travelled by B= 200+x+x=200+2x
As the ratio of their speed is 2:3, and time is same, hence the ratio of
distance travelled by the them is also 2:3.
200 2
=
200+2𝑥 3
Solving, x= 50
Distance between their residences to school= 200+ x= 250m

Example: A goods train leaves a station at a certain time. After 6 hours, an


express train leaves the same station and travels in the same direction at a
speed of 90 km/hr. If the express train catches up the goods train in 4
hours, what is the speed of the goods train?

Solution: Distance covered by express train after leaving the station =


90× 4 = 360𝑘𝑚
Goods train covers this distance in 6+4=10 hours.
360
Hence speed of goods train= = 36 𝑘𝑚/ℎ𝑟
10

Key Points to Remember for New Pattern Questions


➢ Average speed is calculated by dividing the total distance
travelled by total time taken.
➢ Always be careful that units of speed, time and distance for
two or more entities should be same while solving a
problem.
➢ To change speed of km/hr into m/second multiply the
5 18
former by and by 𝑖𝑛 𝑣𝑖𝑐𝑒 𝑣𝑒𝑟𝑠𝑎 𝑐𝑎𝑠𝑒.
18 5

3 Adda247 Publications For any detail, mail us at


Publications@adda247.com
Cracker Book for Bank (IBPS | SBI | RRB PO | Clerk) Mains Exams

Practice Exercise Based on new Pattern

1. In how much time a train can cover a distance of 600 km if speed of


train is 25% more than the speed of car and speed of car is 20 % more
than the speed of a truck which cover 640 km in 20 hours in which he
takes 40 minute break after every 100 km.
(a) 9 hours (b) 8 hours 20 mins
(c) 10 hours (d) 8 hours
(e) None of these

2. A train travelling at 144 km/hr crosses another train, having 30 meter


less length and travelling in opposite direction at 126 km/hr in 6
seconds. If longer train cross a railway platform in 20 second then find
smaller train will cross same platform in how many seconds?
(a) 22 seconds (b) 24 seconds (c) 28 seconds
(d) 32 seconds (e) 30 seconds

3. Train A running at the speed of 108 km/hr crosses a man, who


running in the opposite direction at the speed of 12 km/hr in 7.2 sec.
If speed of train A increased by 25% and it takes 48 seconds to cross
another train B, which running at the speed of 90 km/hr in same
direction. Find the length of train B?
(a) 280 meters (b) 360 meters (c) 180 meters
(d) 160 meters (e) 220 meters

4. Train P leaves Delhi at 7 PM and is travelling at a speed of 80 kmph, 3


hours later another train Q leaves Delhi and is travelling in same
direction as train P in how many time (in minutes) train Q will be 18
km ahead train P, if speed of train Q is 120 kmph?
(a) 383 minutes (b) 387 minutes (c) 380 minutes
(d) 377 minutes (e) 375 minutes

4 Adda247 Publications For any detail, mail us at


Publications@adda247.com
Cracker Book for Bank (IBPS | SBI | RRB PO | Clerk) Mains Exams

5. A bus and a car start from a place A toward B simultaneously at 10:00


am. Bus reach at B in 4 hours and return for C, which is exactly in the
middle of A and B, and meet the car after 1 hour after leaving B. Find
the time at which car reach C after returning from B.
(a) 6:00 pm (b) 8:00 pm (c) 10:00 pm
(d) 8:00 am (e) 12:00 pm

6. Two trains A and B running at the speed of 108km/hr and 162km/hr


crossed another train ‘P’, which is standing stationary of length 240
meters in 14 sec and 16 sec respectively. How much time both train A
and B will take to cross each other, if both running in opposite
direction?
(a) 8.4 𝑠𝑒𝑐 (b) 8.6 𝑠𝑒𝑐 (c) 8.8 𝑠𝑒𝑐
(d) 9 𝑠𝑒𝑐 (e) 8.2 𝑠𝑒𝑐

7. There was a race of 3000 meters between A & B on a circular track of


750 meters. First time they meet during the race is after 5 minutes of
starting of race. Find the time taken by B to complete the race, if he
runs at half the speed of A.
(a) 20 minutes (b) 18 minutes (c) 15 minutes
(d) 10 minutes (e) None of these

8. Two cities Rampur and Dhamnagar are 300 km apart, Bhavya starts
from Rampur at 8 : 24 am and an hour later Abhi starts from same
city. After travelling for one hour Abhi reaches at city which Bhavya
had passed 40 min earlier. The city is in the way of Rampur and
Dhamnagar. If they reaches city Dhamnagar at same time. Find their
speed.
(a) 50 km/hr, 75 km/hr (b) 75 km/hr, 75 km/hr
(c) 100 km/hr, 100 km/hr (d) 75 km/hr, 100 km/hr
(e) None of these

5 Adda247 Publications For any detail, mail us at


Publications@adda247.com
Cracker Book for Bank (IBPS | SBI | RRB PO | Clerk) Mains Exams

9. A 240 meters long train crosses a 210 meters long train running in
opposite direction in 6 sec. Ratio between speed of longer train and
smaller train is 7 : 8. If faster train crosses a platform in 9 sec, then
find time taken by slower train to cross a bridge, which is 60 meters
more long than platform?
142 136 90
(a) 𝑠𝑒𝑐 (b) 𝑠𝑒𝑐 (c) 𝑠𝑒𝑐
7 7 7
148 142
(d) 𝑠𝑒𝑐 (e) 𝑠𝑒𝑐
7 7

10. Train P leave place A by is travelling at a speed of 120 kmph. 4 hours


later another train Q leaves place by travelling in same direction as
train P. Train Q will be 36 km ahead of train P in T hours and speed of
train Q is 160 kmph, then find a bus travel what distance in (T + 3.1)
hr, if speed of bus is 75% of speed of train P?
(a) 1360 km (b) 1440 km (c) 1540 km
(d) 1280 km (e) 1720 km

11. Satish started from point A in a boat to reach point B. After 6.5 hours
he covered only 20% of the distance and reach at point M. Now, Satish
started from point M reached at mid point of A and B and came back
to M in 29.25 hours. In what time Satish can cover the distance
between B and A if he started from B?
(a) 58.5 hours (b) 32.5 hours (c) 65 hours
(d) Can’t be determined (e) 50 hours

12. Train X having length 130 m and train Y having length 145 m moving
in opposite direction. They enter into a tunnel which have length
equal to the sum of length of both trains. Trains meet after 10 second
of entering in the tunnel. What percent of train X part is leave out the
tunnel when it meet train Y if they have there speed in the ratio of 5
:6.
11 11 1
(a) 2 % (b) 3 % (c) 4 %
13 13 3
(d) 5% (e) 8%
6 Adda247 Publications For any detail, mail us at
Publications@adda247.com
Cracker Book for Bank (IBPS | SBI | RRB PO | Clerk) Mains Exams

13. A car and a bus starts from point ‘A’. After ‘T’ hours bus is 48 km
ahead of car, while after ‘8’ hours distance between bus and car is
same as distance covered by bus in one hour. If relative speed of car
and bus if they move towards each other is 240, then how much
1
distance can bus cover in (𝑇 – ) hour.
2
(a) 320 km (b) 280 km (c) 384 km
(d) 336 km (e) 256 km

14. Point A to Point B is a downstream journey of 300 km on a stream


which flows at a speed of 5 km/hr. Two boats P and Q starts from point
A and Point B respectively with speed of 25 km/hr and 15 km/hr in
still water. After reaching the opposite point they return to their
starting points, find after how much time will they meet second time?
(a) 7.5hour (b) 15hour (c) 20hour
(d) 10hour (e) None of these

15. Distance between Delhi and Jaipur is 300 km. Aman starts from Delhi
and Rajiv from Jaipur at same time. After two hours, Aman realized he
was travelling slow and therefore increased his speed by 25% and
meet Rajiv at a point 108 km from Delhi. Find the increased speed of
Aman, if Rajiv derived at a constant speed of 75 km/hr.
(a) 40 km/hr (b) 50 km/hr (c) 60 km/hr
(d) 55 km/hr (e) 65 km/hr

Directions (16-17): Time taken by train A to cover a distance is 1.5 times


the time taken by train B to cover the same distance. Train A and train C
starts simultaneously from a station in same direction. Train B, start after
30 min and overtake train C in 1.5 hour later than it cross train A.

16. If speed of train A is 80 km/hr, then what is the speed of train C. (2


Marks)
(a) 60 km/hr (b) 100 km/hr (c) 125 km/hr
(d) 75 km/hr (e) 80 km/hr
7 Adda247 Publications For any detail, mail us at
Publications@adda247.com
Cracker Book for Bank (IBPS | SBI | RRB PO | Clerk) Mains Exams

17. If train A double its speed, then speed of train A is what % more than
the speed of train C. (1 Mark)
(a) 60% (b) 100% (c) 80%
(d) 50% (e) None of these

Directions (18-19): A consignment should deliver on time to customer for


which Mahendra start his journey with uniform speed, after 2 hours
customer call Mahendra to deliver his order 1 hour before the decided time.
Mahendra increased his speed by 50% to deliver it 1 hour before the
decided time.

18. Find the total time taken by Mahendra to deliver the consignment?
(a) 4 hours (b) 5 hours (c) 6 hours
(d) 7 hours (e) 3 hours

19. If initially, customer alto start moving toward Mahendra at speed of


60 km/hr and take his consignment after 3 hours then find the total
distance between Mahendra and Customer initially.
(a) 256 km (b) 150 km (c) 500 km
(d) 450 km (e) 300 km

Directions (20-21): A man decided to run 15 rounds of a circular track of


400 m in certain time with certain speed. He starts running but after
completing some round around the track he reduced his speed by 40% due
to which he takes 4 min extra as scheduled. But if he reduced his speed by
completing 3 more rounds he would have reached 2 min earlier than the
time he actually reached.

20. Find the original speed of man?


(a) 240 m/min (b) 160 m/min (c) 200 m/min
(d) 400 m/min (e) 350 m/min

21. Find the number of rounds at which he decided to reduce his speed?
(a) 7 (b) 9 (c) 10
(d) 8 (e) 11
8 Adda247 Publications For any detail, mail us at
Publications@adda247.com
Cracker Book for Bank (IBPS | SBI | RRB PO | Clerk) Mains Exams

22. A cyclist left point A for point B and travelled at the constant speed of
25 km/h. When he covered the distance of 25/3 km, he was overtaken
by a car that left point A twelve minutes after the cyclist and travelled
at a constant speed too. When the cyclist travelled another 30 km, he
encountered the car returning from B. Assume that the car did not
stop at point B. Find the distance between A and B.
(a) 39.5833 km (b) 41.0833 km (c) 60.833 km
(d) 43.33 km (e) 50 km

23. Two person left simultaneously two places A and B. One of them left A
for B while the other left B for A. Both travel at uniform speed . The
first person on reaching B returns to A and then again travels back to
B and so on. Similarly the second person on reaching A returns to B
and then travels back to A and so on. What will be the distance
covered by the first person when they meet for the third time given
the ratio of the speed of the first person to that of the second person is
3 : 2 and the distance between A and B is 500 m?
(a) 1000 m (b) 1500 m (c) 2500 m
(d) 1200 m (e) 1800 m

24. A train M running at the speed of 108 km/hr crosses a man running at
12 km/hr in opposite direction of train in 12 second and crosses a
platform in 32 sec. If a train N is Standing on same platform and
length of platform is 140 meter more than length of train N. Find in
what time train N will cross train M running in same direction, if train
N passes a pole in 12 second?
(a) 168 sec (b) 164 sec (c) 154 sec
(d) 186 sec (e) 172 sec

Directions (25-26): Satish and Bhavya starts from point P and Q


respectively at same time to reach at point B. Satish overtakes Bhavya after
10 hours at point A. If Bhavya starts 4 hours earlier than Satish then they
will meet at point B after 16 hours when Satish starts. Ratio between
distance between Q and A to Q and B is 1 : 2. Point P, Q, A and B lies on a
straight road.

9 Adda247 Publications For any detail, mail us at


Publications@adda247.com
Cracker Book for Bank (IBPS | SBI | RRB PO | Clerk) Mains Exams

25. If distance between point A and point B is 180km then find time taken
by Bhavya to reach point ‘C’ which is 432km ahead of point ‘B’?
(a) 36 hours (b) 44 hours (c) 24 hours
(d) 32 hours (e) 40 hours

26. If Satish and Bhavya started to move towards each other from P and Q
at 8:00 a.m, then at what time they will meet?
(a) 9 : 00 a.m (b) 9 : 30 a.m (c) 10 : 00 a.m
(d) 10 : 30 a.m (e) 11 : 00 a.m

27. A bus (X) start its journey from P to Q simultaneously another bus (Y)
start from Q to P they reach their respective destination after 3 hour.
Bus X travel with a speed of 40 km/hr in first hour 50 km/hr in
second hour and 60 km/hr in 3rd hour. Bus Y travel with a speed of 60
km/hr in first hour, 40 km/hr in second and 50 km/hr in last hour.
Find out the distance between Q and the point where the buses cross
each other.
(a)87 (b)88 (c)89
2 7
(d)82 (e) 87
9 9

175 2
28. Speed of a car is 𝑚/𝑠 and speed of bus is 14 % less to speed of car,
9 7

bus and car covered different distance in different interval of time and
the sum of the time taken by them to cover their respective distances
is10 hours but distance covered by bus is 80 km more than distance
covered by car. If a train covered total distance which covered by car
2
and bus together in 6 ℎ𝑜𝑢𝑟𝑠, then find speed of train?
3

(a) 96km/hr (b) 84km/hr (c) 78km/hr


(d) 72km/hr (e)64km/hr

10 Adda247 Publications For any detail, mail us at


Publications@adda247.com
Cracker Book for Bank (IBPS | SBI | RRB PO | Clerk) Mains Exams
20
29. Two trains running in opposite directions cross each other in sec.
3
Longer train cross shorter train in 60 sec when both train running in
same direction. If length of shorter train increased by 50% than
longer train cross shorter train in 72 sec, when both running in same
direction. If difference between length of both trains is 60 meters,
then find the time in which longer train will cross a platform whose
7
length is 77 % more than length of longer train?
9
(a) 20 sec (b) 10 sec (c) 15 sec
(d) 14 sec (e) 18 sec
30. Two trains P and Q started from two points A and B towards each
other to cover 300 km distance. Speed of train Q is twice of speed of
train P. If train P runs at its normal speed and train Q reduces its
speed to 50% then it will take 2.5 hrs. more to meet each other. Find
the time (in sec) in which train P can cover a platform of 150m length
if the length of train P is given as 50m.
(a) 18 seconds (b) 36 seconds (c) 72 seconds
(d) 54 seconds (e) 24 seconds
27
31. A train P, 180 meter long train passed a pole in sec and also passed
4
two trains Q and R in 9 sec and 39 sec respectively, where train Q
running in opposite direction of train P and train R is running in same
direction of train P. If length of train Q and R is 240 meter and 210
meter respectively, then in what time train Q will pass train R, if both
runs in opposite direction ?
7 3
(a) 35 sec (b) 9 sec (c) 12 sec
11 11
(d) 15 sec (e) 55 sec
32. Total distance between Delhi to Lucknow is 480 km. A train starts
running with an average speed of 60 km/hr from Delhi to Lucknow
while another train starts Journey after 120 minutes of first train and
reaches Lucknow 30 minutes before first train. If first train stops for
5 minutes on each station and second train did not stop at any station
,then find the ratio between speed of first train to speed of second
train given that total number of station between Delhi and Lucknow
are nine.?
(a) 25 : 33 (b) 25 : 32 (c) 35 : 25
(d) 25 : 31 (e) 21 : 31
11 Adda247 Publications For any detail, mail us at
Publications@adda247.com
Cracker Book for Bank (IBPS | SBI | RRB PO | Clerk) Mains Exams

33. The ratio between length of two trains is 3 : 2 and speed of both trains
is 72 km/hr & 90 km/hr respectively. Faster train crosses slower
trains in 60 sec running in same direction. If slower train crosses a
goods train, which is running at the speed of 108 km/hr in 7.2 sec
running in opposite direction. Then find the time taken by faster train
to cross goods train running in same direction?
(a) 40 sec (b) 64 sec (c) 60 sec
(d) 56 sec (e) 50 sec

34. Two athletes ‘A’ and ‘B’ practice running on 800 meters’ track. If they
start running at their usual speeds, in same direction, at same time,
from same point, first time they meet is in 100 seconds. During warm-
2
up, ‘A’ runs at 18 % of his usual speed and completes 400 meters in
11
50 second. Find the time taken by athlete ‘B’ to complete 9 rounds of
5
400-meter track, if he runs at of his usual speed. Assume athlete ‘A’
4
is faster than ‘B’.
(a) 100 second (b) 450 second (c) 90 second
(d) 80 second (e) 85 second

35. The speed of car is 25% more than the speed of bus. The time
difference between them to cover a certain distance D, is 1 hour. On
particular day, driver noticed that if they are (D- 40) km apart from
each other then they can meet in 2 hours driving in opposite direction
at their normal speed. Find 150% of the speed of bus ?
(a) 150 km/hr (b) 90 km/hr (c) 120 km/hr
(d) 105 km/hr (e) None of these

36. Time taken by car A to cover distance from point X to point Y is 6½


hrs including ½ hour of stoppage. B also started from point X and
stopped 90 km earlier from point Y after travelling for 6½ hrs. Had
they started from same point and at same time, the difference
between their distance covered in 7.5 minutes will be 2.5 km. Find the
distance between two cars, if they start a race for the distance which
is twice the distance between X and Y , assuming that both cars stops
when one of the car finish the race
(a) 360 km (b) 240 km (c) 180 km
(d) 300 km (e) 280 km
12 Adda247 Publications For any detail, mail us at
Publications@adda247.com
Cracker Book for Bank (IBPS | SBI | RRB PO | Clerk) Mains Exams

37. Two trains A and B with their length difference 17 m cross each other
in 2.6 seconds while running in opposite direction. Train A crosses
train B in 13 seconds while they are running in same direction. If train
A takes 2 seconds to cross a pole, then find the sum of speed of both
the trains, assuming that train B is longer than train A.
(a) 34 ms¯¹ (b) 51 ms¯¹ (c) 68 ms¯¹
(d) 85 ms¯¹ (e) 102 ms¯¹

38. ‘Rajdhani express’, 240 m long train crosses a pole in 6 sec and it also
120
crosses two trains ‘Shatabdi express’ and ‘Duranto express’ in 𝑠𝑒𝑐
17
& 42 sec respectively, where ‘Shatabdi express’ is running in opposite
direction of ‘Rajdhani express’ and ‘Duranto express’ running in same
direction as that of ‘Rajdhani express’. If length of ‘Shatabdi express’ &
Duranto express’ is 360 m and 180 m respectively, then find in what
time ‘Shatabdi express’ will cross ‘Duranto express’ while running in
same direction?
(a) 24 sec (b) 28 sec (c) 32 sec
(d) 36 sec (e) 48 sec

Directions (39–40): There are two trains A and B, both train starts
travelling to each other from stations P and Q respectively. Train A reaches
the station Q in total 4 hours while train B reaches the station P in total 4
hours 48 minutes. Speed of train A is 120 km/hours.

39. Train A and B running on two different railroads parallelly in opposite


direction to two different trains C and D respectively. Train A passed
train C, which running at the speed of 60 km/hr. in 6 sec. and train B
passed train D, which running at the speed of 110 km/hr. in 9 sec. if
ratio between length of train A to train C is 3: 2, and that of between
train B to train D is 4 : 3. Find in what time train D passed train C, if
both running in same direction ?(2 marks)
(a) 22.64 sec (b) 24.84 sec (c) 24.24 sec
(d) 28.84 sec (e) 20.84 sec

13 Adda247 Publications For any detail, mail us at


Publications@adda247.com
Cracker Book for Bank (IBPS | SBI | RRB PO | Clerk) Mains Exams

40. Ratio between speed of train A to train E is 6: 5.Train E starts a


journey from Kolkata to Delhi, which is 600 km at 9.45pm and train A
starts same journey after 30 minutes of train E.Train A stop for x
minutes on each three stoppages and train E stop for (x+10) minutes
each on three stoppages between Kolkata to Delhi.Train A reaches
Delhione hour earlier than train E, which reaches at 4.30
am.Findinwhat time train B a total distance of 720 km cover between
Lucknow to Jaipur. If train B stop for (x+10) minutes on four
stoppages between Lucknow to Jaipur?(2 marks)
(a) 4 hours (b) 8 hours (c) 8.2 hours
(d) 10 hours (e) 12 hours

Solutions

1. (c); Stop of truck = 6 × 40 = 240 mint or 4 h.


640 640
Speed of truck = = = 40 𝑘𝑚/ℎ
20−4 ℎ. 16
40×120
Speed of car = = 48 𝑘𝑚/ℎ
100
48×125
Speed of train = = 60 𝑘𝑚/ℎ
100
600
Required time = = 10 ℎ𝑜𝑢𝑟𝑠
60

2. (a); Let length of both trains be L meters and (L – 30) meters


respectively
ATQ,
5 𝐿+ (L – 30)
(144 + 126)× =
18 6
450 = 2L – 30
L = 240
Smaller train length = 210 meters
Let length of platform be P meters
ATQ,
5 240+𝑃
144 × =
18 20
P = 800 – 240 = 560 meters
Let required time = T
ATQ,
5 210+560
126 × =
18 𝑇
770
T = ⇒ T = 22 sec
35
14 Adda247 Publications For any detail, mail us at
Publications@adda247.com
Cracker Book for Bank (IBPS | SBI | RRB PO | Clerk) Mains Exams

3. (b); Let length of train A be L meters


5 𝐿
(108 + 12) × =
18 7.2
L = 240 meters
125
New speed of train A = 108 ×
100
= 135 𝑘𝑚/ℎ𝑟
Let length of train B be S meters
5 240+𝑆
(135 – 90)× =
18 48
S = 360 meters

4. (b); Relative speed of both train = (120 – 80) = 40 kmph


Distance covered by train P from Delhi when train Q was not
moving = 80 × 3 = 240 km
Train Q should be covered (240 + 18) km from Delhi for ahead
18 km to train P from Delhi
258
Required time = km
40
= 6.45 = 6.45 × 60
= 387 minutes

5. (b);

Car travel → 3/4 part of one side distance in 5 hour.


20
1 part = hour
3
Car can travel 1.5 part in
20
× 1.5 = 10 hour
3
Time to reach C after returning form point B → 10:00 + 10
= 8:00 pm

15 Adda247 Publications For any detail, mail us at


Publications@adda247.com
Cracker Book for Bank (IBPS | SBI | RRB PO | Clerk) Mains Exams

6. (c); Let speed of train A and B be ‘x’ meters and ‘y’ meters
respectively
ATQ,
5 x+240
108 × = m⁄s
18 14
30 × 14 = x + 240
x = 180 meters
And,
5 𝑦+240
162 × =
18 16
45 × 16 = y + 240
y = 480meters
Let required time be T sec
5 480+180
(108 + 162) × =
18 T
75 𝑇 = 660
T = 8.8 sec

7. (a); Let speed of A is 2x & speed of B is x


According to question,
750
= 5 min
a−b
⇒ a – b = 150 meters/min
⇒ 2x – x = 150 meters/min
⇒ x = 150 meters/min
It is speed of B hence
3000 3000
Time taken = = = 20 minutes
x 150

8. (d); Let the city in way of Rampur and Dhamnagar is C.

Time Speed
Bhavya 80 3x
Abhi 60 4x
16 Adda247 Publications For any detail, mail us at
Publications@adda247.com
Cracker Book for Bank (IBPS | SBI | RRB PO | Clerk) Mains Exams

ATQ,
300 300
– =1
3x 4x
25
= 1 ⇒ x = 25
x
∴ Bhavya Speed = 3 × 25 = 75 km/hr
Abhi speed = 4 × 25 = 100 km/hr

9. (c); Let speed of longer train and smaller train be 7𝑥 and 8𝑥


respectively.
ATQ—
(240+210)
(7𝑥 + 8𝑥 ) =
6
90𝑥 = 450
𝑥=5
Speed of longer train = 5 × 7 = 35 m/s
Speed of smaller train = 5 × 8 = 40 m/s
Let length of platform be l meter
𝑙+210
40 =
9
40 × 9 = 𝑙 + 210
l=150
Then, length of bridge = 150 + 60 = 210 meters
Let time taken by slower train to cross bridge be T sec.
210+240
T=
35
35T = 660
90
T= sec.
7

10. (b); Relative speed of both trains = (160 – 120) = 40 kmph


Distance covered by train P from place A when train Q was not
moving = 120 × 4 = 480 km
Train Q should cover (480 + 36) km from place A in T hrs
516
Required time, T = = 12.9 hr
40
3
Bus speed = 120 × = 90𝑘𝑚𝑝ℎ
4
Bus covered in ( T+3.1) hours = ( 12.9 + 3.1) × 90 = 1440 km

17 Adda247 Publications For any detail, mail us at


Publications@adda247.com
Cracker Book for Bank (IBPS | SBI | RRB PO | Clerk) Mains Exams

11. (c); Let total distance from A to B = ‘D’


ATQ, Satish cover 20% distance in 6.5 hours So, he can cover
30% distance (M to mid-point of A and B) in
6.5
× 3 = 9.75 hr.
2
Time taken by Satish to come back from mid-point to M
= 29.25 – 9.75 = 19.5 hr
30% distance covered by Satish in 19.5 hr.
19.5
100% distance covered by Satish in × 10 = 65 hr
3

12. (b); Speed of train X and Y 5x and 6x respectively.


Length of tunnel → 130 + 145 = 275 m.
Speed of trains per second
275
= = 27.5 𝑚/s
10
5x + 6x = 27.5
x = 2.5
speed of train X → 12.5 m/s
Speed of train Y → 15 m/s
Distance cover by train X in tunnel = 12.5 × 10 = 125 m.
Length of train X leaves out = 130 – 125 = 5m
5
Required% = × 100
130
11
=3 %
13

13. (a); Let, speed of car = x km/hr


Speed of bus = y km/hr
ATQ,
Distance covered by bus in one hour
= Distance between bus & car in 8 hour
⇒ y = 8 (y – x) [ ∵ y > x]
y = 8y – 8x
8x = 7y …(i)
and x + y = 240 …(ii)
On solving (i) & (ii)
x = 112, y = 128
After T hour bus is 48 km ahead of car
⇒ (128 – 112) T = 48
48
⇒T= =3
16
Required distance = 2.5 × 128 = 320 km
18 Adda247 Publications For any detail, mail us at
Publications@adda247.com
Cracker Book for Bank (IBPS | SBI | RRB PO | Clerk) Mains Exams

14. (c); Time when P and Q meet first time


300 300
= = = 7.5 hr
25+15 40
Distance travelled by P = 7.5 × (25 + 5) = 225 km
Distance travelled by Q = 7.5 × (15 – 5) = 75 km
75
P reach at point B in i.e. 2.5 hour
30
In 2.5hr Q travelled 2.5 × 10 = 25 km
Now Boat P returns and relative distance b/w P and Q is 100 km
and relative speed is 10 km i.e. difference of speed of P (25 -5) =
20 km/hr (upstream) and speed of Q (15 – 5) = 10 km/hr
(upstream)
100
Time taken by P and Q to meet = = 10 hr
10
Total time when P and Q meet second time
= 7.5 +2.5 + 10 = 20 hr
15. (b); Let initial speed of Aman is x km/hr.
Distance travelled by him in 2 hours is 2x km.
While distance travelled by Rajiv in these 2 hours is 75 × 2 = 150
km
When both of them meet, Aman had travelled a distance of 108
km.
Distance travelled by Aman with 25% increase in his speed =
(108 – 2x) km
125 5x
And his increased speed = x( ) = km/hr
100 4
If Aman had travelled 108 km, Rajiv had travelled 192 km.
192 – 150 = 42 km after 2 hours
Time taken by Rajiv to travel 42 km
42 14
= = hours
75 25
This is equal to time taken by Aman to travel (108 – 2x) km
(108–2x) 14
5 =
x 25
4
14×5x
108– 2x =
25×4
7
108 = x + 2x
10
27
x = 108 ⇒ x = 40 km/hr.
10
Therefore, increased speed of Aman was
5
= × 40 = 50 km/hr
4

19 Adda247 Publications For any detail, mail us at


Publications@adda247.com
Cracker Book for Bank (IBPS | SBI | RRB PO | Clerk) Mains Exams

Solutions (16-17); Time ratio between train A and train B = 1.5 : 1


So speed ratio between train A and train B is =1 : 1.5 or 2 : 3
Let speed of train A = 2x km/hr
Speed of train B = 3x km/hr
Relative speed = 3x – 3x = x km/hr
Distance travel by train A in 30 m
2𝑥
⇒ = x km
2
𝑥
Train B cross train A ⇒ = 1 hour
𝑥
So,
Train B cross train C ⇒ 1 + 1.5 = 2.5 hour
Let speed of train C = y km/h
ATQ
2.5 × 3x = 3y
x:y⇒2:5
Ratio of speed of A, B and C
4:6:5

16. (b); Speed of train A = 80 km/hr


80
Speed of train C = × 5 = 100 km/hr
4

17. (a); Let speed of train A = 4x km/hr


So speed of train C = 5x km/hr
(2×4𝑥–5𝑥)
Required % = × 100 = 60%
5𝑥

18. (b); Let speed of Mahendra = 2𝑥 km/hr


So, Distance covered by him in 2 hours = 2𝑥 × 2 = 4𝑥
Let he decide to deliver his consignment in 𝑡 hour
So total distance ⇒ 2𝑥 × 𝑡 = 2𝑥𝑡
ATQ
⇒ 2𝑥𝑡 = 2𝑥 × 2 + (𝑡– 3)3𝑥
3𝑥 → increase speed
𝑡– 3 = time for which he travelled after speed is increased
On Solving equation →
5 ℎ𝑜𝑢𝑟𝑠 = 𝑡

20 Adda247 Publications For any detail, mail us at


Publications@adda247.com
Cracker Book for Bank (IBPS | SBI | RRB PO | Clerk) Mains Exams

19. (d); Distance covered by Mahendra in 3 hours → 2𝑥 × 3 = 6𝑥


Total distance ⇒ 5 × 2𝑥 = 10𝑥
Remaining distance ⇒ 10𝑥 – 6𝑥 = 4𝑥 = Distance covered by
customer
and 4𝑥 = 60 × 3
𝑥 = 45 km
Total distance = 45 × 10 = 450 km

Solutions (20-21): Total distance = 15 × 400 = 6000 m


Let his original speed = x meter/min
Decided time = t min
Let after ‘n' round he reduced his speed 1st decided condition
6000
⇒t= …(i)
x
3x
is the reduced speed after n rounds
5
Now 2nd condition
400×n 6000–400×n
+ 3x = t + 4 …(ii)
x
5
3rd → supposed condition
400(n+3) 6000–400×(n+3)
+ 3x = t + 2 …(iii)
x
5
Putting value of ‘t’ from (i) into (ii) and (iii)
⇒ 3x + 200n = 3000 …(iv)
⇒ 3x + 400n = 4800 …(v)
Solving (iv) and (v)
x = 400 m/min
n=9

20. (d); Original speed of man = 400 m/min

21. (b); Number of rounds after which he reduced his speed = 9

21 Adda247 Publications For any detail, mail us at


Publications@adda247.com
Cracker Book for Bank (IBPS | SBI | RRB PO | Clerk) Mains Exams
25 1
22. (c); Time taken by Cyclist to reach 25/3 km = = hr = 20 min
25 × 3 3
Car has taken to reach 25/3 km = 20 – 12 = 8 min
25
Speed of Car = 3  60 = 62.5 km/h
8
Now time taken by cyclist to go further 30 km
30 6
= = hr = 72 min
25 5
72
Car will go in 72 min =  62.5 = 75 km
60
Now, according to question,
distance between first meeting and second meeting is 30
So,
distance between first meeting and point B will be
75+30
= = 52.5 km
2
Required answer = 52.5 + 8.33 = 60.833 km

23. (b); When they meet for the third time they together cover 5 × 𝐴𝐵
= 5 × 500 = 2500𝑚
The first person will cover
3
× 2500 = 1500 𝑚
2+3

24. (b); Lets length of train M = L meter


ATQ,
(108+12)×5 𝐿
=
18 12
L = 400 m
Let length of platform = Lp
108×5 𝐿𝑝 +400
=
18 32
𝐿𝑝 = 960– 400
𝐿𝑝 = 560 meter
Length of train N = 560 – 140
= 420 meter
Speed of train N = 𝑆𝑛
420
𝑆𝑛 = = 35 m/s
12

22 Adda247 Publications For any detail, mail us at


Publications@adda247.com
Cracker Book for Bank (IBPS | SBI | RRB PO | Clerk) Mains Exams

Relative speed of train M and N, when both running in same


direction
5
= 35 −108 × = 5 𝑚/𝑠𝑒𝑐
18
Let required time is t
ATQ,
400+420
5=
𝑡
820
t=
5
t = 164 sec

Solution (25-26)

Let speed of Satish = ‘a’


And, speed of Bhavya = ‘b’
Let distance between P and Q = x
And, QA and QB be y and 2y respectively.
After 10 hrs Satish overtakes Bhavya at point A.
𝐷𝑖𝑠𝑡𝑎𝑛𝑐𝑒
⇒ 𝑆𝑝𝑒𝑒𝑑 =
𝑇𝑖𝑚𝑒
y= 10 × b
and,
(a - b) × 10 = x …(i)
If Bhavya starts 4 hour earlier than Satish, then Satish overtakes Bhavya at
point B after 16 hours
Distance travelled by Bhavya in 4 hours = 4b
So,
(x + 4b) = (a – b) × 16 …(ii)
On solving (i) & (ii)
20𝑏 2𝑦
𝑥= ⇒𝑥= …(iii)
3 3
Satish can travel and reach point B in 16 hours
⇒ 16a = 2y + x …(iv)
On solving (iii) & (iv)
y = 6a
3𝑥 𝑥 3𝑥
So, 𝑦 = , 𝑎 = , 𝑏 =
2 4 20

23 Adda247 Publications For any detail, mail us at


Publications@adda247.com
Cracker Book for Bank (IBPS | SBI | RRB PO | Clerk) Mains Exams

25. (b); ATQ,


3𝑥
= 180
2
⇒ x = 120 km/hr
3𝑥 3𝑥
+ +432
2 2
Required time = 3𝑥
20
3×120+432
=
18
792
= = 44 hours
18

𝑥
26. (d); Required time = 𝑥 3𝑥
+
4 20
𝑥
= 5𝑥+3𝑥
20
20
= = 2 ℎ𝑜𝑢𝑟 30 𝑚𝑖𝑛𝑢𝑡𝑒
8
Satish & Bhavya meet at 10 : 30 am

27. (d); Distance between P and Q is 150 km.


Now X bus cover 40 km in 1 hour
Y bus cover 60 km in 1st hour
Remaining distance = 50 km
𝑅𝑒𝑚𝑎𝑖𝑛𝑖𝑛𝑔 𝑡ime to cross each other
50 50 5
= = = ℎ𝑟
40+50 90 9
5
Distance which is covered by ′Y ′ in hr
9
5 200
= 40 × = km
9 9
Distance between Q and the point where buses crosses each
other
200 2
= 60 + = 82 km
9 9
24 Adda247 Publications For any detail, mail us at
Publications@adda247.com
Cracker Book for Bank (IBPS | SBI | RRB PO | Clerk) Mains Exams
175 6 25×6
28. (a); Speed of bus = × =
9 7 9
50 50 18
= m⁄s = × = 60 km⁄hr
3 3 5
Let distance travel by car and bus be D and (D + 80)
respectively.
D (D+80)
+ = 10
70 60
6D+7D+560
= 10
420
13D = 4200 – 560
13D = 3640
D = 280 km
[280+(280+80)]×3
Speed of train =
20
= 96 km⁄hr

29. (a); Let length of shorter train be L meters and length of longer train
be (L + 60) meters
And also speed of shorter and longer train be V m/s and U m/s
respectively.
When both trains travelling in opposite direction
(L+L+60)3
= U+V
20
3L+180
= U + V …(i)
20
When both train running in same directions—
L+L+60
= U– V
60
(2L+60)
= U– V …(ii)
60
When length of shorter train increased and both train running in
same direction —
(1.5L+L+60)
= U –V
72
2.5L+60
= U– V …(iii)
72
From (i) and (ii)
3L+180 2L+60
2U = +
20 60
11L+600
U= …(iv)
120
7L+480
U= …(v)
120
25 Adda247 Publications For any detail, mail us at
Publications@adda247.com
Cracker Book for Bank (IBPS | SBI | RRB PO | Clerk) Mains Exams

After solving (iii), (iv) and (v)


L = 120 m, U = 25 m/s and V = 20 m/s
Length of longer train = 120 + 60 = 180 meter
7
Length of platform = 180 + 180 × = 320 meters
9
Time taken by longer train to cross platform
320+180 500
= = = 20 sec
25 25

30. (b); Let, speed of train P = x


Speed of train Q = 2x
Let, Normal time taken by train to meet each other = ‘y’
ATQ,
(x + 2x) y = (x + x) (y + 2.5)
3y = 2(y + 2.5)
y = 5 hr.
300
Now, x + 2x = = 60
5
3x = 60
x = 20 km/hr
50
= m/sec
9
(150+50)
Required time = ×9
50
200
= × 9 = 36 sec
50

31. (c); Lets speed of train P, Q and R be S₁, S₂ and S₃ respectively


180 80 m
Speed of train P (S1 ) = 27 m⁄s =
3 s
4
Speed of train Q (S2 )
80 240+180
+ 𝑆2 =
3 9
420 80
S2 = –
9 3
S2 = 20 m⁄s
Speed of train R (𝑆3 )
80 210+180
– S3 =
3 39
80
S3 = – 10
3

26 Adda247 Publications For any detail, mail us at


Publications@adda247.com
Cracker Book for Bank (IBPS | SBI | RRB PO | Clerk) Mains Exams
50
S3 = m⁄s
3
Lets required time be T sec
50 240+210
Required time = 20 + =
3 T
110 450 450×3
= ⇒T =
3 T 110
3
T = 12 sec
11

32. (b); Speed of first train = 60 km/h


Total distance = 480 km
Time taken by first train to cover that distance without stoppage
480
= = 8ℎ
60
9 station × 5 minute = Total stoppage time
= 45 minute
Train took total time to reach Lucknow
35
= 8 hr 45 minute = ℎ𝑜𝑢𝑟
4
2nd Train reach 30 min. before it start 2 hour late from Delhi
So,
Time taken by 2nd train = 8 h 45 min – 2h
– 30 min.
= 6 hour 15 minute
1 25
= 6 = hour
4 4
480×4 384
Speed of 2nd Train= = km/h
25 5
60
Required Speed ratio of train = 384 =25:32
5

33. (c); Let length of two trains be 3L meter and 2L meter


ATQ—
5 3L+2L
(90– 72) × =
18 60
300 = 5L
L = 60 meter
Length of faster train = 120 meter
Length of slower train = 180 meter
Let length of goods train be G meter

27 Adda247 Publications For any detail, mail us at


Publications@adda247.com
Cracker Book for Bank (IBPS | SBI | RRB PO | Clerk) Mains Exams
5 180+𝐺
(108 + 72) × =
18 7.2
50 × 7.2 = 180 + G
G = 360 – 180
G = 180 meter
Let faster train cross good train in T sec —
5 120+180
(108 – 90) × =
18 T
5T = 300
T = 60 sec

34. (d); Let the speed of athletes ‘A’ and ‘B’ is 𝑎 and 𝑏 respectively
From 1st condition
800
= 100 ⇒ 𝑎– 𝑏 = 8 ms –1
𝑎–𝑏
From 2nd condition
2
When athlete ‘A’ will run at 18 % of his usual speed
11
200 2
= ×𝑎= 𝑎
1100 11
400
= 2 = 50 ⇒ 𝑎 = 44 ms –1
𝑎
11
∴ 𝑏 = 36 ms –1
5
If 𝑏 runs at of his speed, which means with 45 ms –1
4
4×400
Time taken by him = = 80 seconds.
45

35. (c); Let speed of car is 5𝑥 𝑘𝑚/ℎ𝑟 and speed of bus is 4𝑥 𝑘𝑚/ℎ𝑟.
According to first condition,
𝐷 𝐷 5𝐷–4𝐷
– =1⇒ =1
4𝑥 5𝑥 20𝑥
⇒ 𝐷 = 20𝑥 …(i)
Also if they are travelling in opposite direction,
Then
𝐷–40
= 2 ⇒ D – 40 = 18x …(ii)
4𝑥+5𝑥
From (i) & (ii)
18𝑥 + 40 = 20𝑥 ⇒ 𝑥 = 20
Hence speed of bus = 4 × 20 = 80 km/hr
150
150% of speed of bus = × 80 = 120 km/hr
100

28 Adda247 Publications For any detail, mail us at


Publications@adda247.com
Cracker Book for Bank (IBPS | SBI | RRB PO | Clerk) Mains Exams

36. (b); Let the Distance b/w X to Y be D km and speed of car A and B be
A km /hr & B km/hr respectively
13
6A – B = 90 – (i)
2
& according to second condition
7.5 7.5
A– 𝐵 = 2.5
60 60
A – B = 20 – (ii)
Solving (i) & (ii)
A = 80 km, B = 60 km
Distance b/w X & Y is = 6 × 80 = 480 km
960
Time take by A to cover 960 km is = 12 hours
80
Distance covered by B in 12 hours = 12 × 60 = 720 km
Distance b/w them = (960–720) km = 240 km

37. (d); Let the length of train A be L m and speed is a m/s.


& the length of train B will be L + 17 m and speed is b m/s.
ATQ,
2𝐿+17
= 2.6 …(i)
𝑎+𝑏
2𝐿+17
= 13 ...(ii)
𝑎–𝑏
For train A
𝐿
= 𝑎 ⇒ 𝐿 = 2𝑎
2
Put L=2a in (i) and (ii)
4𝑎+17
= 2.6 …(iii)
𝑎+𝑏
4𝑎+17
= 13 .....(iv)
𝑎–𝑏
On solving (iii) and (iv),
a= 51m/s
and, b=34m/s
Hence sum of speed =51+34= 85 𝑚𝑠 –1

38. (d); Let speed of ‘Rajdhani express’ be S m/s


ATQ –
240
S=
6
S = 40 m/s
Speed of ‘Shatabdi express’ be V m/s
29 Adda247 Publications For any detail, mail us at
Publications@adda247.com
Cracker Book for Bank (IBPS | SBI | RRB PO | Clerk) Mains Exams
17(240+360)
(40 +V) =
120
4800 + 120V = 10200
120V = 5400
V = 45 m/s
Speed of Duranto express’ be u m/s
240+180
(40 − U) =
42
40 − U = 10
U = 30 m/s
Let ‘Shatabdi express’ will cross ‘Duranto express’ running in
same direction in T sec
360+180
(45 − 30) =
𝑇
15T = 540
T = 36 sec

Solutions (39-40): Speed of train A = 120km/h


Distance between P and Q = 120 × 4 = 480
480 ×5
Speed of train B = = 100 km/hr
24

39. (b); Relative speed of train A and C, when both running in opposite
direction
5
= (120 + 60) ×
18
= 50 m/sec
Lets length of train A and C is 3L meter and 2L meter
respectively.
ATQ—
3L+2L
50 =
6
5L = 300
L = 60 meter
Length of train C = 60 × 2 = 120 meter
Relative speed of train B and D, when both running in opposite
5 175
direction = (100 + 110) × = m/sec
18 3
Lets length of train B and D is 4L meter and 3L meter
respectively
ATQ—
175 4L+3L
=
3 9
30 Adda247 Publications For any detail, mail us at
Publications@adda247.com
Cracker Book for Bank (IBPS | SBI | RRB PO | Clerk) Mains Exams

7L = 525
L = 75
Length of train D = 75 × 3 = 225 meter
Relative speed of train C and D when both train running in same
direction
5
= (110 – 60) ×
18
125
= meter/sec
9
(120+225)×9
Required time = = 24.84 sec
125

40. (c); Speed ratio of train A and Train E = 6 : 5


So, ratio of time taken by train A and train E (without stoppage)
=5:6
Now → Train E start = 9 : 45
Halt → 3 × (x+ 10) ⇒ 3x + 30 min
Train A start →9 : 45 + 30 → 10 : 15
Halt time ⇒ 3 × x = 3x min
If halt time add, before starting journey
Train E ⇒9 : 45 + 30 + 3x
⇒10 : 15 + 3x
Train A ⇒10 : 15 + 3x
So, time difference on reaching on Delhi is have same proportion
6 – 5 = 1 hour
Train A take → 5 hour
Train E take→ 6 hour
Train A speed = 120 km/h
Train E = 100 km/h
For x –
(3.30 am – 10 .15 pm) – 3x = 5hours
3x = 5. 15 – 5
X = 5minutes
Speed of train B = 100 km/hr
Train cover total distance of 720 between Lucknow to Jaipur
720
= + 4(𝑥 + 10)
100
= 7.2 hours + 4(5+10)
= 7.2 hour + 60 minutes
= 8.2 hours
31 Adda247 Publications For any detail, mail us at
Publications@adda247.com
Cracker Book for Bank (IBPS | SBI | RRB PO | Clerk) Mains Exams

1 Adda247 Publications For any detail, mail us at


Publications@adda247.com
Cracker Book for Bank (IBPS | SBI | RRB PO | Clerk) Mains Exams

Chapter
Boat and Stream
9
BEST APPROACH TO SOLVE THE QUESTIONS

The questions based on boats and streams are simply based on the concept
of relative speed. Generally, two cases arise when a boat moves. Either the
boat moves along the flow of the stream or the boat moves in the direction
opposite to that of stream.

1. If boat moves in the direction of stream. The net speed is the


addition of the speed of stream and boat.
2. If boat moves in the direction opposite to that of stream, the net
speed is the difference of their speeds (speed of boat being more
than that of stream).

In questions related to boat and streams, usually it happens that one may
need to solve complex calculations but using a bit of intelligent guess work
one may save a good amount of time.

A sample problem shows one way of approach:

Example: ‘A man can row 24 km in upstream and 36 km in downstream in


9 hours. The same man can row 8 km in upstream and 48 km in
downstream in 6 hours. Calculate the speed of the stream.’

Sol. The basic approach to this question involves formation of two


equations, which are If we assume speed of stream as r and speed of boat as
x.
24 36
+ =9
𝑥−𝑟 𝑥+𝑟
8 48
+ =6
𝑥−𝑟 𝑥+𝑟

2 Adda247 Publications For any detail, mail us at


Publications@adda247.com
Cracker Book for Bank (IBPS | SBI | RRB PO | Clerk) Mains Exams

It’s cumbersome to solve these set of equations. So we need to look at


distance travelled in downstream or in upstream. In majority of cases it’s
better to take HCF of distance travelled in downstream in both cases. HCF of
36 and 48 is 12. Now we have to break 12 in two parts such that, adding or
subtracting them, completely divides all the four distance. Try (10,2) (9,3)
(8,4) and check whether these fulfill the condition. Or check from option.

In short we can only check this by hit and trial method.

Here, speed of stream=4km/hr and that of boat in still water is 8 km/hr.

Practice Exercise Based on New Pattern

1. A boat goes 28 km downstream and while returning covered only


75% of distance that covered in downstream. If boat takes 3 hr more
to cover upstream than downstream then find the speed of boat in
5
still water (km/hr) if speed of stream is m/sec ?
9
(a) 8 km/hr (b) 2 km/hr (c) 5 km/hr
(d) 4 km/hr (e) 3 km/hr

2. The ratio of time taken by boat A and boat B to swim a certain


distance downstream in a river is 3 : 4 respectively. The time taken by
boat B to cover some other distance in upstream is 50% more than
the time taken by it to cover the same distance in downstream. What
is the ratio of speed of boat A to that of boat B in still water?
(a) 7 : 5 (b) 7 : 4 (c) 7 : 3
(d) 7 : 9 (e) 7 : 2

3. There are three points P, Q and R in a straight line in a river such that
point Q is equidistant from point P and Point R. Boat travelled from
point P to R downstream in 8 hours and from point Q to P upstream in
12 hours. Find the ratio between speed of boat in still water to speed
of current?
(a) 3:1 (b) 2:1 (c) 1:2
(d) 5:2 (e) 2:3
3 Adda247 Publications For any detail, mail us at
Publications@adda247.com
Cracker Book for Bank (IBPS | SBI | RRB PO | Clerk) Mains Exams

4. A boat cover 60 km upstream and 60 km downstream in 22.5 hr with


its usual speed. If boat double its speed then new upstream speed is
150% more than the usual upstream speed. Find the time taken by
boat to cover 80 km in downstream with usual speed.
(a) 12 hr (b) 20 hr (c) 5 hr
(d) 16 hr (e) 10 hr

5. A boat covers total distance of 360km downstream in three equal


parts with the speed of boat be 4x km/hr, 5𝑥 km/hr and 7𝑥 km/hr
respectively. If speed of stream is x km/hr and boat takes total 29.5
hours, then find boat will cover total given distance upstream in three
equal parts with three givens speeds?
(a) 36 hours (b) 40 hours (c) 45 hours
(d) 48 hours (e) 54 hours

6. The speed of two boats A and B in the still water is in the ratio of 3 : 4
and the speed of current is 4km/hr. A start from point P ,45 minutes
earlier than B in downstream direction. If B catch boat A in three
hours then, find boat B takes how much time to cover 120 km distance
each in downstream and in upstream.
(a) 12 hours (b) 16 hours (c) 18 hours
(d) 14 hours (e) 20 hours

7. A boat has to travel from point P to point Q in upstream and reach Q


by 5.00 PM. If boat starts form point P at 2.00 PM with the speed of 20
km/hr and after travelling one hour the speed of current is increased
by 25% of its initial speed. If distance from point P to Q is 36 km and
speed of current is 8 km/hr, then find by what percent boat has to
increase its speed to reach point Q in time?
(a) 5% (b) 15% (c) 12%
(d) 10% (e) 17.5%

Directions (8-9): A boat ‘A’ start from point X, toward point Y in upstream
simultaneously second boat ‘B’ start from point Y to X having speed more
than first boat ‘A’. Difference between speed of both boats in still water is
equal to the speed of stream and they cross each other first time after 2
hours. Given that both boats start their respective return journey by
reaching their destination points.
4 Adda247 Publications For any detail, mail us at
Publications@adda247.com
Cracker Book for Bank (IBPS | SBI | RRB PO | Clerk) Mains Exams

8. If distance between X and Y is 56 km and speed of stream is 4 km/hr


then find the distance from ‘Y’ when boats cross each other 2nd time.
(a) 2.8 km (b) 2.4 km (c) 5.6 km
(d) 3.2 km (e) 4.8 km

9. Find the time taken by boats to cross each other on 2 nd time from the
first time if speed of second boat ‘B’ is 1.5 times of the speed of first
boat ‘A’ in still water?
(a) 5 hours (b) 3 hours (c) 2 hours
(d) 6 hours (e) 4 hours

10. Point A to Point B is a downstream journey of 300 km on a stream


which flows at a speed of 5 km/hr. Two boats P and Q starts from
point A and Point B respectively with speed of 25 km/hr and 15
km/hr in still water. After reaching the opposite point they return to
their starting points, find after how much time will they meet second
time?
(a) 7.5hour (b) 15hour (c) 20hour
(d) 10hour (e) None of these

11. Speed of current is 10 km/hr and speed of a motor boat is 80% more
than speed of current. Motor boat travels 280 km downstream with its
usual speed, after that it’s increased speed by ‘s’ kmph and travelled
for another 280 km then it returns and covers 560 km in upstream. If
boat complete whole journey downstream to upstream in 45 hr, then
find the value of ‘s’?
(a) 10 km/hr (b) 8 km/hr (c) 6 km/hr
(d) 12 km/hr (e) 4 km/hr

12. Speed of current is 5 km/hr and speed of boat in still water is 80%
more than speed of current.A boat travels 50% of total distance of 280
km, in downstream. After that the speed increase by x kmph because
of a man who added an engine. If boat complete whole journey
downstream and upstream in 45 hr, then find the value of x?
(a) 6 km/hr (b) 8 km/hr (c) 9 km/hr
(d) 4 km/hr (e) 5 km/hr
5 Adda247 Publications For any detail, mail us at
Publications@adda247.com
Cracker Book for Bank (IBPS | SBI | RRB PO | Clerk) Mains Exams

13. A lake in which speed of water current is zero has a stretch of 144 km.
Two boats, whose sum of speed in still water is 28 km/hr, starts from
two ends of lake. A fish starts swimming simultaneously with one boat
toward another boat and reach the second boat in 4 hours and then
returns toward first boat and touch it in 48 minutes. Find the speed of
fish.
(a) 12 km/hr (b) 16 km/hr (c) 24 km/hr
(d) 40 km/hr (e) 8 km/hr
3th
14. A boat goes certain distance downstream and then return of the
4
3
distance upstream.It takes of the time in upstream than in
2
1
downstream. If boat increases its speed by 33 % and cover a distance
3
of 60 km in downstream and then return upstream in 16 hours, find
increased speed of boat?
(a) 6 km/hr (b) 14 km/hr (c) 16 km/hr
(d) 12 km/hr (e) 8 km/hr
15. Vikas can swim in still water with twice the speed as that of speed of
water. Time difference to cover a certain distance in upstream and in
downstream is 4 hours at his usual speed. But if he had doubled his
usual speed, then time difference would reduce by 80% of its usual
time difference to cover same distance. Calculate total time taken by
Vikas to cover 6 km upstream and 2 km downstream, at his usual
speed.
3 5
(a) 1 hour (b) hour (c) hour
4 4
4
(d) hour (e) Can’t be determined
3

16. Speeds of three motor boats A, B and C are equal and all cover 8 km of
distance upstream in 48 minutes. Ratio between speed of motor boats
and speed of stream is 6 : 1. On first day A starts in downstream from
point P to Q, which shifts 9 km away from point P each day. On second
day B starts from point P in downstream and reach at point Q in 4.5
hours, then find time take by C on third day to reach at point Q,
(consider speed of stream same on all three days)?
1 1 1
(a) 4 ℎ𝑜𝑢𝑟𝑠 (b) 5 ℎ𝑜𝑢𝑟𝑠 (c) 3 ℎ𝑜𝑢𝑟𝑠
7 7 7
1 1
(d) 2 ℎ𝑜𝑢𝑟𝑠 (e) 9 ℎ𝑜𝑢𝑟𝑠
7 7
6 Adda247 Publications For any detail, mail us at
Publications@adda247.com
Cracker Book for Bank (IBPS | SBI | RRB PO | Clerk) Mains Exams

17. Usual speed of a boat is S km/hr. Boat rowing in a lake (neglect the
current speed) and covered distance of 360 km in three equal parts
with three different speeds in total 7.5 hours. If boat covered first part
with usual speed and second part with 20% increased speed and third
part with further increase of 25% of latest speed. Find the speed of
boat at which it covered third part of journey?
(a) 56km/hr (b) 36km/hr (c) 40km/hr
(d) 60km/hr (e) 48 km/hr

18. Ratio between speed of two boats A & B in still water is 4 : 3 and
speed of current is 4 km/hr .Boat B starts from point P in
downstream, two hours earlier than boat A and it is found that boat A
is16 km ahead of boat B after 12 hours of its start. If boat A start from
point Q in upstream and boat B start from point P in downstream at
same time and distance between P and Q is 108 km, then find in what
time boat A & B meet each other?
6 6 6
(a) 4 ℎ𝑜𝑢𝑟𝑠 (b) 5 ℎ𝑜𝑢𝑟𝑠 (c) 9 ℎ𝑜𝑢𝑟𝑠
7 7 7
6 6
(d) 3 ℎ𝑜𝑢𝑟𝑠 (e) 11 ℎ𝑜𝑢𝑟𝑠
7 7

19. When two ships A & B are travelling in opposite direction crosses
each other in 2 second. Speed of ship B is 50% more than the speed of
ship A in still water. On a particular day, ship A was ahead of ship B by
9 meters and both of them were travelling in downstream. If Ship B
overtook Ship A in 11 seconds and length of ship B is 54 meters then
find the speed of ship A.
(a) 56m/s (b) 36m/s (c) 40m/s
(d) 60m/s (e) 18m/s

20. A boat goes a certain distance downstream and then returns and
covers 40% of distance covered in downstream. Ratio of time taken in
covering downstream and upstream distances is 3 : 2. If speed of boat
in still water is reduced by 50% then it covers 60 km downstream in
10 hours. Find the speed of boat in still water.
(a) 9 km/hr (b) 8 km/hr (c) 6 km/hr
(d) 10 km/hr (e) 12 km/hr

7 Adda247 Publications For any detail, mail us at


Publications@adda247.com
Cracker Book for Bank (IBPS | SBI | RRB PO | Clerk) Mains Exams

Solutions

5 18
1. (c); Speed of current = ×
9 5
= 2 km⁄hr
Let’s still water speed = x km/hr
ATQ,
3
28× 28
4

(x–2) (x+2)
=3
21x + 42 – 28x + 56 = 3x² – 12
–7x + 98 = 3x² – 12
3x² + 7x – 110 = 0
x = 5 km/hr

2. (a); Let total distance be D km.


And, speed of boat A in still water be
x km/hr
Speed of boat B in still water be y km/hr
Speed of stream be r km/hr
ATQ -
𝐷 𝐷 3
∶ =
𝑥+𝑟 𝑦+𝑟 4
𝑦+𝑟 3
Or, = ……..(i)
𝑥+𝑟 4
Also, for a distance D1,
𝐷1 3 𝐷1
= ( )
𝑦−𝑟 2 𝑦+𝑟
or, 2𝑦 + 2𝑟 = 3𝑦 − 3𝑟
or, 𝑦 = 5𝑟
Putting this in (i),
6𝑟 3
=
𝑥+𝑟 4
or, 3𝑥 + 3𝑟 = 24𝑟
or, 𝑥 = 7𝑟
Ratio of speed of boat A and B in still water= 7r : 5r
=7:5

8 Adda247 Publications For any detail, mail us at


Publications@adda247.com
Cracker Book for Bank (IBPS | SBI | RRB PO | Clerk) Mains Exams

3. (b); Let still water speed x km/hr and


Speed of current y km/hr —
Downstream speed = (x + y) km/hr
Upstream speed = (x – y) km/hr
Let distance between P to R is 2ℓ and Q to P is ℓ km
ATQ—
2ℓ
(x+y)
=8 … (i)
and

(x–y)
= 12 … (ii)
From (i) and (ii)
x 2
=
y 1

4. (e); Let usual speed of boat in still water = x


River speed = y
ATQ,
(𝑥−𝑦)250
= (2𝑥 − 𝑦)
100
5𝑥 − 5𝑦 = 4𝑥 − 2𝑦
x = 3y
Now,
60 60
+ = 22.5
x –y x+y
60 60
+ = 22.5
2y 4y
y = 2 km/hr
x = 6 km/hr
80
Required time =
6+2
= 10 hr

5. (c); ATQ—
120 120 120
(4𝑥+𝑥)
+ (5𝑥+𝑥)
+ (7𝑥+𝑥)
= 29.5
120 120 120
+ + = 29.5
5𝑥 6𝑥 8𝑥
5760+4800+3600
=𝑥
240×29.5
𝑥=2
120 120 120
Required time = + +
3𝑥 4𝑥 6𝑥
1 1 1 3
= 120 ( + + ) = 120 × = 45 ℎ𝑜𝑢𝑟𝑠
6 8 12 8
9 Adda247 Publications For any detail, mail us at
Publications@adda247.com
Cracker Book for Bank (IBPS | SBI | RRB PO | Clerk) Mains Exams

6. (b); Let speed of boat A and B in still water be 3𝑥 km/hr and 4𝑥


km/hr respectively
Downstream speed for boat A
= (3𝑥 + 4) km/hr
Downstream speed for boat B
= (4𝑥 + 4) km/hr
Relative speed of boat A and B
= (4𝑥 + 4) – (3𝑥 + 4) = 𝑥 km/hr
ATQ—
45 (3𝑥+4)
× =3
60 𝑥
9𝑥 + 12 = 12𝑥
𝑥 = 4 km/hr
Speed of boat B = 4 × 4 = 16 km/hr
120 120
Required times = (16+4) + (16–4)
= 6 + 10 = 16 hours

7. (d); Boat travels total distance in the hours


Upstream speed of boat = 20 – 8 = 12 km/hr
In one hour, boat will cover a distance of
= 12 × 1 = 12 𝑘𝑚
Remaining distance = 36 – 12 = 24 km
To reach Q at 5.00 PM boat has to cover remaining distance at
speed
24
= = 12 𝑘𝑚/ℎ𝑟
2
125
New speed of stream = 8 × = 10 𝑘𝑚/ℎ𝑟
100
New upstream speed of boat =20 – 10
= 10 km/hr
For 12 km/hr speed boat has to increase its speed by 2 km/hr
2
Required percentage = × 100 = 10%
20

10 Adda247 Publications For any detail, mail us at


Publications@adda247.com
Cracker Book for Bank (IBPS | SBI | RRB PO | Clerk) Mains Exams

Solutions (8-9): Let speed of first boat ‘A’


= a km/hr
Speed of stream = b km/hr
So,
Speed of 2nd boat ‘B’ = 𝑎 + 𝑏 km/hr

Distance XY ⇒ 2 × (a – b + a + b + b) = 4a + 2b

8. (d); Given XY = 56 = 4𝑎 + 2𝑏
2𝑎 + 𝑏 = 28
Now speed of stream = 𝑏 = 4 km/hr
2𝑎 + 4 = 28
𝑎 = 12
Speed of first boat ‘A’ in still water = 12
Speed of 2nd boat ‘B’ in still water = 16
First time crossing

Time taken by 2nd boat ‘B’ to reach at X after first crossing


16 4
= = hour
20 5
4
Distance travelled by first boat ‘A’ in hour
5
4 32
= × 8 = km
5 5
Now,
Distance between first boat ‘A’ and 2nd boat ‘B’
32
16 + ⇒ 22.4 km
5
On return journey of second boat ‘B’, speed of boat = 12 km/hr
Time taken by first boat to reach Y
56–22.4
= = 4.2 hours
8
Distance remaining between both boat after reaching of first
boat ‘A’ at Y
⇒ 56 – 4.2 × (12 – 8) = 5.6 km
Time taken by both boat to cover this distance
5.6
= = 0.2 hours
12+16
Distance from Y = 0.2 × 16 = 3.2 km
11 Adda247 Publications For any detail, mail us at
Publications@adda247.com
Cracker Book for Bank (IBPS | SBI | RRB PO | Clerk) Mains Exams

9. (b); If speed of first boat ‘A’ = 𝑎 km/hr


3
Speed of second boat ‘B’ ⇒ 𝑎 km/hr
2
3 𝑎
So speed of stream ⇒ 𝑎– 𝑎 = km/hr
2 2
Distance travelled by first boat till first crossing
𝑎
⇒ (𝑎– ) × 2
2
⇒ 𝑎 km
𝑎
Speed of first boat in upstream = km/hr
2
Speed of 2 boat in downstream
nd

= 2𝑎 km/hr

𝑎 1
2nd boat reached at X ⇒ = ℎ
2𝑎 2
1 1 𝑎 𝑎
In hour first boat cover ⇒ × = km
2 2 2 4
𝑎
Distance between both boats ⇒ 𝑎 +
4
5𝑎
⇒ km
4
Relative speed of boat ‘A’ and ‘B’ when they cross each other
3𝑎 𝑎 𝑎 𝑎
= ( – – )=
2 2 2 2
5𝑎
Time taken to cross
4
5𝑎×2
⇒ = 2.5 hour
4×𝑎
So total time for 2nd crossing
⇒ 2.5 + 0.5 = 3 hours

10. (c); Time when P and Q meet first time


300 300
= = = 7.5 hr
25+15 40
Distance travelled by P = 7.5 × (25 + 5)
= 225 km
Distance travelled by Q = 7.5 × (15 – 5)
= 75 km
75
P reach at point B in i.e. 2.5 hour
30
In 2.5hr Q travelled 2.5 × 10 = 25 km

12 Adda247 Publications For any detail, mail us at


Publications@adda247.com
Cracker Book for Bank (IBPS | SBI | RRB PO | Clerk) Mains Exams

Now Boat P returns and relative distance b/w P and Q is 100 km


and relative speed is 10 km i.e. difference of speed of P (25 -5) =
20 km/hr (upstream) and speed of Q
(15 – 5) = 10 km/hr (upstream)
Time taken by P and Q to meet
100
= = 10 hr
10
Total time when P and Q meet second time
= 7.5 +2.5 + 10 = 20 hr

11. (d); speed of boat in still water


80
= (10 + 10 × ) km⁄hr
100
= 18 km⁄hr
ATQ—
280 280 560
(18+10)
+ (18+10)+s + (18–10)+s = 45
280 560
+ = 35
28+s 8+s
8 16
+ =1
28+s 8+s
64 + 8s + 448 + 16s = 224 + 28s + 8s + s2
s² + 12x – 288 = 0
s = 12 km/hr

80
12. (a); Still water speed = (5 + 5 × ) km⁄hr
100
= 9 km⁄hr
ATQ—
140 140 280
(9+5)
+ (9+5)+x + (9–5)+x = 45
140 280
+ = 35
14+x 4+x
4 8
+ =1
14+x 4+x
12x + 128 = 56 + 14x + 4x + x²
x² + 6x – 72 = 0
x = 6 km/hr

13 Adda247 Publications For any detail, mail us at


Publications@adda247.com
Cracker Book for Bank (IBPS | SBI | RRB PO | Clerk) Mains Exams

80
12. (a); Still water speed = (5 + 5 × ) km⁄hr
100
= 9 km⁄hr
ATQ—
140 140 280
(9+5)
+ (9+5)+x + (9–5)+x = 45
140 280
+ = 35
14+x 4+x
4 8
+ =1
14+x 4+x
12x + 128 = 56 + 14x + 4x + x²
x² + 6x – 72 = 0
x = 6 km/hr

13. (c);

Let speed of boat 2 and speed of fish is x and y respectively.


Boat 2 and fish meet in 4 hour
So,
(x + y) × 4 = 144 …(i)
Now
ATQ,
Speed of boat 1 = 28 – x
In 4 hour boat 1 cover = 4 × (28 – x)
48
× (𝑦 + (28 – 𝑥 )) = 144 – [4𝑥 + 4(28 − 𝑥 )]
60
4
(𝑦 − 𝑥 ) + 22.4 = 144 − 112
5
𝑦 − 𝑥 = 12 … … . (𝑖𝑖)
Sovling (i) & (ii)
x = 12 km/hr
y = 24 km/hr

14 Adda247 Publications For any detail, mail us at


Publications@adda247.com
Cracker Book for Bank (IBPS | SBI | RRB PO | Clerk) Mains Exams

14. (e); Let distance travelled by boat in downstream be D and speed of


boat in still water be x km/hr and speed of current be y km/hr
ATQ,
3
×𝐷 3 𝐷
4
= ×
(𝑥−𝑦) 2 (𝑥+𝑦)
1 1
=
2(𝑥−𝑦) 𝑥+𝑦
x = 3y
1
New sped of boat = 3y + 3y ×
3
= 4 y km/hr
60 60
+ = 16
(4𝑦+𝑦) (4𝑦−𝑦)
12 20
+ = 16
𝑦 𝑦
32
y = = 2 𝑘𝑚/ℎ𝑟
16
Increased speed of boat = 4×2 = 8 km/hr

15. (e); Let the speed of water is x km/hr, then speed of vikas in still
water is 2x km/hr
From 1st condition,
𝐷 𝐷
– =4
2𝑥–𝑥 2𝑥+𝑥
𝐷 𝐷
⇒ – =4
𝑥 3𝑥
⇒ D = 6𝑥 …(i)
From 2nd condition, when speed of Vikas is 4𝑥 km/hr
𝐷 𝐷 20
– = ×4
4𝑥–𝑥 4𝑥+𝑥 100
𝐷 𝐷 4
⇒ – =
3𝑥 5𝑥 5
⇒ D = 6𝑥 …(ii)
Hence both equations are same so answer can’t be determined.

16. (b); Speed of motor boats in upstream


60
= 8 × = 10 km/hr
48
ATQ,
Let speed of motor boats be 6x km/hr and speed of stream be x
km/hr
6x – x = 10
15 Adda247 Publications For any detail, mail us at
Publications@adda247.com
Cracker Book for Bank (IBPS | SBI | RRB PO | Clerk) Mains Exams

x = 2 km/hr
Downstream speed of all boats
= (6 × 2 + 2)
= 14 km/hr
Let distance between point P to Q on first day = y km
Second day distance = (y + 9)
𝑦+9
14 =
4.5
y= 63 – 9
y = 63 – 9
y= 54 km
Distance travelled on third day = 54 + 9 × 2
= 72 km
Total time taken by boat C on third day to reach point Q
72 1
= = 5 hours
14 7

17. (d); Three equal parts of journey


360
= = 120 km
3
ATQ—
120 120 120
+ S + 6S 6S 1 = 7.5
S S+5 + ×
5 5 4
120 600 120×10
+ + = 7.5
S 6S 12S+3S
3600+3000+2400 15
=
30S 2
9000 15
=
30S 2
9000×2
S=
30×15
S = 40 km/hr
Speed of which boat which it covered third part of Journey
20 20 1
= (40 + 40 × ) + (40 + 40 × ) ×
100 100 4
= 60 km/hr

18. (d); Let speed of boat A & B in still water be 4x km/hr and 3x km/hr
respectively
Downstream speed of boat A
= (4x + 4) km/hr
16 Adda247 Publications For any detail, mail us at
Publications@adda247.com
Cracker Book for Bank (IBPS | SBI | RRB PO | Clerk) Mains Exams

Downstream speed of boat B


= (3x + 4) km/hr
AT Q –
12 (4x + 4) – 16 = 2 (3x + 4) + 12 (3x + 4)
48x + 48 − 16 = 6x + 8 + 36x + 48
6x = 24
x = 4 km/hr
Speed of boat A in still water = 16 km/hr
Speed of boat B in still water= 12 km/hr
Relative speed = (16 – 4) + (12 + 4)
= 28 km/hr
108
Required time =
28
6
= 3 ℎ𝑜𝑢𝑟𝑠
7

19. (e); Let speed of ship A in still water is 2x m/s , and its length is ℓ m
And speed of ship B is 3x m/s and its length is 54 and speed of
water is Y m/s.
ATQ, when both of them are travelling in opposite direction.
Downstream speed of ship A = (2x + Y) m/s [assume ship A is
travelling in downstream and ship B in upstream]
Upstream speed of ship B = (3x – Y) m/s
Their relative speed = 2x + Y + 3x – Y
= 5x m/s
ATQ,
54+ℓ
= 2 …(i)
5x
Similarly when both are travelling in downstream their relative
speed is
= 3x + Y – 2x –Y= x m/s
ATQ,
54+9+ℓ
= 11
𝑥
63 + ℓ = 11x …(ii)
From (i)
ℓ = 10x – 54
put this value in eqn. (ii)
= 63 + 10x – 54 = 11x
x = 9 m/s
Hence speed of ship A in still water = 2 × 9 = 18 m/s
17 Adda247 Publications For any detail, mail us at
Publications@adda247.com
Cracker Book for Bank (IBPS | SBI | RRB PO | Clerk) Mains Exams

20. (b); Let speed of boat in still water and speed of current be
respectively x km/hr and y km/hr
And Let time taken to cover given distance in downstream and
upstream be = 3m hours and 2m hours
40% of distance in upstream is covered in 2 m hour
100% of distance upstream is covered in 5m hours
According to question
x−y 3
=
x+y 5
5x - 5y = 3x + 3y
x 4
⇒ =
y 1
And
60
x = 10
(2+y)
x
+y=6
2
4𝑦
+𝑦=6
2
6y = 12
y = 2 km/hr
so, x = 8 km/hr

18 Adda247 Publications For any detail, mail us at


Publications@adda247.com
Cracker Book for Bank (IBPS | SBI | RRB PO | Clerk) Mains Exams

1 Adda247 Publications For any detail, mail us at


Publications@adda247.com
Cracker Book for Bank (IBPS | SBI | RRB PO | Clerk) Mains Exams

Chapter
Mensuration
10
BEST APPROACH TO SOLVE THE QUESTIONS

Questions related to mensuration usually don’t involve any short-cut


methods for solving a problem. The reason being how can one use short
tricks when all one has to do is to deal with formulae. To solve questions
faster and in a lucid manner, one must avoid doing unnecessary and
multiple calculations. Rather one must calculate only where it is needed. In
case, ratio or percentage is asked in the question, then it becomes more
easy to do away with hefty calculations and arrive at the answer by using
basic elimination methods.

‘The length, breadth and height of a room are in the ratio 3:2:1. If the
breadth and height are halved while the length is doubled, then the
percentage change in total area of the four walls of the room.’

Area of four walls of the room= 2(lh+bh), where l,b and h are length,
breadth and height of the room respectively.

As we have been asked the percentage change, let’s take length, breadth and
height of the room to be 6,4 and 2 respectively.

Then changed values of length, breadth and height would be 12, 2 and 1.
Initial area of four walls= 2 (6 × 2 + 4 × 2) = 40
Final area of four walls= 2 (12× 1 + 2 × 1)= 28
28 is 70% of 40, which means there has been a reduction of 30%.

‘A solid cylinder has height 10 meters and radius R meters. Some part
of it is melted and casted into two cones. One of the cones has radius
3cm and height 4cm. The other cone has height 8 cm and radius 6cm. If
the ratio of initial curved surface area of cylinder to the sum of curved
surface areas of two cones is 4:1, then find the radius of the cylinder.’

2 Adda247 Publications For any detail, mail us at


Publications@adda247.com
Cracker Book for Bank (IBPS | SBI | RRB PO | Clerk) Mains Exams

Slant height of cone having radius 3cm is 5cm (PLEASE SOLVE IT


PROPERLY)
Slant height of cone having radius 6 cm is 10 cm
Curved surface area of a cone is πrl and that of cylinder is 2πrh
2𝜋×𝑅×10 4
ATQ, =
𝜋(3×5+6×10) 1
➢ R=15cm

Practice Exercise Based on new Pattern

1. The ratio of the radius and height of a cone is


2
5 : 12. Its volume is 314 cm. Then its slant height is ?
7
(a) 9 cm (b) 10 cm (c) 11 cm
(d) 12 cm (e) 13 cm

2. If the perimeter of a rectangle and a square are equal and the ratio of
2 adjacent sides of a rectangle is 2:1. Then find the ratio of area of
rectangle and area of square.
(a) 1:1 (b) 1:2 (c) 2:3
(d) 8:9 (e) 9:4

3. The parameter of a square is equal to the parameter of a rectangle


having dimensions 24 cm × 16 cm. Find the circumference of a circle
whose diameter is equal to the side of square ?
360 410 470
(a) cm (b) cm (c) cm
7 7 7
440 460
(d) cm (e) cm
7 7

4. Total surface area of a cylinder mounted with a hemispherical bowl


on one end is 2552 cm². If height of cylinder is 8 cm then find the
volume of the solid body?
1 1 1
(a) 10443 (b) 10677 (c) 10547
3 3 3
1 1
(d) 10977 (e) 10787
3 3

3 Adda247 Publications For any detail, mail us at


Publications@adda247.com
Cracker Book for Bank (IBPS | SBI | RRB PO | Clerk) Mains Exams

5. Sum of Area of circle and a rectangle is equal to 622 sq cm. The


diameter of the circle is 14 cm, then what is the sum of circumference
of the circle and the perimeter of the rectangle if the length of
rectangle is 26 cm?
(a) 142cm (b)132cm (c)152cm
(d) 140cm (e)134cm

6. A copper sphere of diameter 42 cm is drawn into a wire of diameter


70 mm. Then, the length of the wire is?
(a) 243 cm (b) 343 cm (c) 443 cm
(d) 972 cm (e) None of these

7. A cylindrical roller rolls over a square field. It takes 500 rounds to


cover the complete field. If the radius of the cylindrical roller is 0.63
meter and length is 7.7 m then find out the side of square
(approximately).
(a) 135 m (b) 125 m (c) 100 m
(d) 150 m (e) 110 m

8. A cuboid is cut along its base such that, now it become a cube. Cube
again cut along its base, now ratio of height to length becomes 1 : 𝑎.
Find the value of ‘𝑎’.
(a) 𝑎 > 1 (b) 𝑎 = 1 (c) 𝑎 ≥ 1
(d) 𝑎 < 1 (e) 𝑎 ≤ 1

9. Volume of a cone and cylinder, having same base, are in ratio of 4 : 3.


If height and radius of cone is 4 : 3 then find the ratio of total surface
area of cone and that of cylinder.
(a) 1 : 1 (b) 3 : 1 (c) 5 : 3
(d) 2 : 1 (e) 4 : 1

10. If the volume and curved surface area of a cylinder 616 m3 and 352m2
respectively, what is the total surface area of the cylinder (in m2 )?
(a) 429 (b) 419 (c) 435
(d) 421 (e) 417
4 Adda247 Publications For any detail, mail us at
Publications@adda247.com
Cracker Book for Bank (IBPS | SBI | RRB PO | Clerk) Mains Exams

11. Area of a rectangle is 252 cm². If length of the rectangle is equal to


side of a square, whose perimeter 72 cm then find the volume of cone,
whose radius is equals to breadth of rectangle and height equals to
length of rectangle?
(a) 4752 cm2 (b) 3586 cm2 (c) 4026 cm2
(d) 4246 cm2 (e) 3696 cm2

12. Sum of length, breadth and height of cuboid is 12 cm and length of its
diagonal is 5√2. Then find the total surface area of cuboid.
(a) 94 cm² (b) 84 cm² (c) 72 cm²
(d) 64 cm² (e) 90 cm²

13. Circumference of a circular garden is 66 cm and area of circular


garden is 25% more than perimeter of a square hall. Find the area of
square hall ?
(a) 4802.49 sq cm (b) 4704.48 sq. cm
(c) 4820.49 sq.cm (d) 4822.49 sq.cm
(e) none of these

14. Internal radius of pipe is 3 cm and the external radius of pipe is 5 cm


and the length of pipe is 15 cm then find total surface area of pipe ?
(a) 278 π cm² (b) 275 π cm² (c) 268π cm²
(d) 272 π cm² (e) none of these

15. A solid cube is melted to form 3 spheres, with radius, 1 cm, 2 cm and 3
cm and additional irregular shape was formed. Calculate the volume
of irregular shape (upto two decimal place) if diagonal of cube is 6√3
cm.
(a) 65.14 𝑐𝑚3 (b) 70.35 𝑐𝑚3 (c) 54.75 𝑐𝑚3
(d) 50.00 𝑐𝑚3 (e) 78.90 𝑐𝑚3

16. 20% increase in both radius and height of the cylinder increases the
total surface area of cylinder by 677.6 cm², If the ratio of radius to
height is 1 : 4, then find the radius of cylinder
(a) 21 cm (b) 10.5 cm (c) 3.5 cm
(d) 14 cm (e) 7 cm
5 Adda247 Publications For any detail, mail us at
Publications@adda247.com
Cracker Book for Bank (IBPS | SBI | RRB PO | Clerk) Mains Exams

17. A Cone cylinder and hemisphere have equal radius and height. Find
the ratio of total surface area of cylinder, cone and hemisphere.
(a) 4 ∶ √2: √6 (b) 4 ∶ √2 ∶ 3
(c) √2 + 1 ∶ 4 ∶ 3 (d) 3 ∶ 4 ∶ √2 + 1
(e) 4 ∶ √2 + 1 ∶ 3

18. Sum of circumference of a circle and perimeter of a rectangle is 220


1
cm while area of circle is 1386 sq. cm. If length of rectangle is 33 %
3
more than radius of the given circle then find the area of rectangle?
(a) 408 𝑐𝑚2 (b) 418 𝑐𝑚2 (c) 428 𝑐𝑚2
(d) 448 𝑐𝑚2 (e) 438 𝑐𝑚2

19. Four identical coins are placed in a square. For each coin, area of each
coin is equal to circumference of each coin. Then, find the area of the
square that is not covered by the coins

(a) 16(π − 1) (b) 16(8 − π) (c) 16(4 − π)


π
(d) 16 (4 − ) (e) None of these
2

20. The surface area of a sphere is 423.5 cm2 less than total surface area
of a hemisphere. If ratio between radius of hemisphere and sphere is
3 : 2, then find the radius of hemisphere?
(a) 5.5 cm (b) 5 cm (c) 4 cm
(d) 7 cm (e) 10.5 cm

21. A cylindrical jar, whose base has a radius of 15 cm is filled with water
up to a height of 20 cm. A solid iron spherical ball of radius 10 cm is
dropped in the jar to submerge completely in water. Find the increase
in the level of water (in cm) is
17 5 8
(a) 5 (b) 5 (c) 5
27 7 9
25 7
(d) 5 (e) 5
27 27

6 Adda247 Publications For any detail, mail us at


Publications@adda247.com
Cracker Book for Bank (IBPS | SBI | RRB PO | Clerk) Mains Exams

22. The dimensions of a field are 20 m and 9 m. A pit 10 m long, 4.5 m


wide and 3 m deep is dug in one corner of the field and the earth
(mud) removed has been evenly spread over the remaining area of
the field. What will be the rise the height of field as a result of this
operation ?
(a) 1.5 m (b) 2 m (c) 3 m
(d) 4m (e) 1 m

23. Difference between perimeter of two rectangles is 8 cm. If breadth of


both rectangle is 16 cm and length are in the ratio of 7 : 6 then find the
total surface area of cylinder whose radius is half of the length of
larger rectangle and height equal to length of smaller rectangle?
(a) 3344 cm2 (b) 3668 cm2 (c) 3444 cm2
(d) 3468 cm2 (e) 3462 cm2

Directions (24-25); There are three toys which are in cylindrical, conical
and spherical shape. Height of all the toys is same. Ratio between the height
of cylinder to radius of cylinder is 4 : 3.

24. Find the ratio between total surface area of cylinder to that of sphere.
(a) 17 : 15 (b) 21 : 16 (c) 28 : 23
(d) 21 : 8 (e) 15 : 4

25. Volume of cone is what percent of the volume of sphere if radius of


cone is equal to that of cylinder
(a) 112.5% (b) 100% (c) 125%
8
(d) 75% (e) 88 %
9

26. A well whose shape is of equilateral triangle having side 8 cm has to


be dug 44√3 cm deep. The earth taken out of the well is used to fill a
cylindrical tank having diameter 8 cm. Find the height up to which
earth is filled in the tank.
(a) 10.5 cm (b) 21 cm (c) 42 cm
(d) 84 cm (e) 32 cm

7 Adda247 Publications For any detail, mail us at


Publications@adda247.com
Cracker Book for Bank (IBPS | SBI | RRB PO | Clerk) Mains Exams

27. A circle is inscribed in a square. If the difference between area of the


square and circle is 262.5 cm², then find the area of the rectangle
whose perimeter is same as that of circle while length of rectangle is
20% more than the breadth of rectangle (in cm²)
(a) 1500 (b) 1400 (c) 700
(d) 750 (e) 3000

28. Inside a square plot a circular garden is developed which exactly fits
in the square plot and the diameter of the garden is equal to the side
of the square plot which is 28 metre. What is the area of the space left
out in the square plot after developing the garden?
(a) 98 m2 (b) 146 m2 (c) 84 m2
(d) 168 m2 (e) 68 m2

29. The area of a rectangle gets reduced by 9m2 if its length is reduced by
5 m and breadth is increased by 3m. If we increase the length by 3 m
and breadth by 2 m, the area is increased by 67 m2. The length of the
rectangle is :
(a) 9 m (b) 15.6 m (c) 17 m
(d) 18.5 m (e) 19 m

30. A toymaker makes a toy in which a cone is mounted on the base of a


hemisphere. If the total surface area of the toy is 858 cm² then find the
volume of the toy, given that the diameter of the toy is 14 cm?
2 2
(a) 1950 cm3 (b) 1250 cm3 (c) 1400 cm3
3 3
3
(d) 1500 cm (e) 1200 cm3

31. If sum of circumference of a circle and perimeter of rectangle is 204


cm and perimeter of rectangle is 50% of magnitude of area of square
and side of square is 12 cm .If length of rectangle is 8 cm more than
that of breadth, then find sum of area of circle and rectangle ?
(a) 1694sq cm (b) 1450sq cm (c) 1550sq cm
(d) 1750sq cm (e) 1460sq cm
8 Adda247 Publications For any detail, mail us at
Publications@adda247.com
Cracker Book for Bank (IBPS | SBI | RRB PO | Clerk) Mains Exams

32. A sphere is melted into ‘n’ number of small spheres. Total surface area
of these small spheres is what percent more or less than total surface
area of large sphere, if the ratio of radius of small sphere to radius of
large sphere is 1 : 3.
(a) 50% (b) 100% (c) 150%
(d) 200% (e) 75%

33. Ratio of height of cylinder to that of diameter is 2: 3. This cylinder is


formed by melting a sphere having same radius as the radius of circle.
Ratio of magnitude of area of circle to magnitude of its circumference
is 21: 2. Find height of cylinder.
(a) 14 units (b) 21 units (c) 42 units
(d) 28 units (e) 35 units

34. If the curved surface area of first cone is thrice that of second cone
and slant height of the second cone is thrice that of the first, find the
ratio of the area of their base.
(a) 81 : 1 (b)9 : 1 (c)3 : 1
(d)27 : 1 (e) None of these

35. There are two cylindrical vessels with same capacity and same
dimensions. If radius of one vessel increased by 7 cm and height of
second vessel increases by 15 cm, then capacity of both vessel
increased by equality ‘Q’ cm3. If actual height of both vessels is 12 cm,
then find value of Q?
(a) 9840 cm3 (b) 9240 cm3 (c) 9460 cm3
(d) 9160 cm3 (e) 9780 cm3

36. There are four light poles in four corners of a park, four at midpoint of
each side and one pole in the middle of that square park. Area of park
is 19600 cm2. If side of park is 50 times of radius of each light pole,
then find the total base area covered by these poles if pole’s base is
circular in shape?
(a) 197.12 cm2 (b) 172.48 cm2 (c) 246.4 cm2
(d) 220.76 cm2 (e) 221.76 cm2

9 Adda247 Publications For any detail, mail us at


Publications@adda247.com
Cracker Book for Bank (IBPS | SBI | RRB PO | Clerk) Mains Exams

37. Area of a square is 306.25 cm2 and breadth of rectangle is 3.5 cm less
than side of square. Area of rectangle is 252 cm2 and height and radius
of a cylindrical vessel equal to length & breadth of rectangle
respectively. If vessel filled, contains mixture of milk and water in
which milk is 75%, then find the volume of water contained by
cylindrical vessel?
(a) 2572 cm3 (b) 2472 cm3 (c) 2772 cm3
(d) 2872 cm2 (e) 2878 cm2

Directions (38-39); A hemispherical bowl is filled with hot water to the


brim. The contents of the bowl are transferred into a cylindrical vessel
whose radius is 50% more than its height.

38. If diameter of the bowl is the same as that of the cylindrical vessel,
then the volume of the hot water in the cylindrical vessel is
(a) 60% of the cylindrical vessel
(b) 80% of the cylindrical vessel
(c) 100% of the cylindrical vessel
(d) 50% of the cylindrical vessel
(e) none of these

39. This hemispherical bowl is joined at one end of the cylindrical vessel
and the solid obtained by combining these figure is filled completely
with water. If a drain pipe is connected to it then pipe will empty 539
cm³ of water is one minute. If radius of the solid is 21 cm, then in what
time whole of the water will be emptied from the solid. Given that
diameter of cylinder and hemisphere are same.
(a) 72 min (b) 60 min (c) 90 min
(d) 75 min (e) None of these

40. A cylinder whose height is equal to its radius is full of milk. Its milk is
poured into a hemi spherical bowl of same radius as of cylinder and
remaining milk is poured into another hemi-spherical bowl of same
volume as of previous one. What percentage of the volume of second
hemispherical bowl remained empty?
(a) 40% (b) 66⅔% (c) 33⅓%
(d) 50% (e) 100%
10 Adda247 Publications For any detail, mail us at
Publications@adda247.com
Cracker Book for Bank (IBPS | SBI | RRB PO | Clerk) Mains Exams

41. In a house, there is a semicircular corridor, a circular hall and a


circular garden. Sum of perimeter of corridor and hall is 102 m.If
radius of hall is 50% more than radius of corridor then find area of
circular garden, given radius of garden is 300% more than sum of
radius of corridor and radius of hall together.
(a) 6300𝜋 sq.m (b) 5400𝜋 sq.m (c) 5700𝜋 sq.m
(d) 4200𝜋 sq.m (e) 4900𝜋 sq.m

42. Height of a cylindrical vessel is equal to side of a square, whose area is


256 cm². Radius of cylindrical vessel is equal to average of radius of a
sphere and hemisphere. If ratio between the radius of hemisphere and
that of the sphere is 2 : 1 and difference between their total surface
area is 4928 cm², then find volume of cylindrical vessel ?
(a) 21,288 cm³ (b) 20,176 cm³ (c) 22,176 cm³
(d) 26,176 cm³ (e) 24,288 cm³

43. Radius of a cylinder is half of the side of square, whose perimeter is 6


cm more than perimeter of rectangle. If ratio between length to
breadth of rectangle is 7 : 2 and between length of rectangle to side of
square is 3 : 2. Find volume of cylinder, given height of cylinder is
equal to breadth of rectangle?
(a) 24,938 cm3 (b) 24,948 cm3 (c) 24,848 cm3
(d) 24,996 cm3 (e) 24,918 cm3

44. A solid sphere of some radius is melted and re-casted into a number of
small cylinders and cones. The height of the cone is 33⅓% less than
the height of the cylinder which is 6 cm and the ratio of radius of cone
to the radius of cylinder is √3 : 1 and the ratio of number of cylinder
to the number of cone is 2 : 3. Find the total surface area of the sphere
if the number of cylinders are 12 and the radius of cylinder is 4 cm.
1 2 4
(a) 1810 cm² (b) 1810 cm² (c) 1810 cm²
7 7 7
11 9
(d) 905 cm² (e) 905 cm²
14 14

11 Adda247 Publications For any detail, mail us at


Publications@adda247.com
Cracker Book for Bank (IBPS | SBI | RRB PO | Clerk) Mains Exams

45. Radius of a cylindrical vessel is 1.5 cm less than breadth of rectangle,


while height of cylindrical vessel is equal to length of rectangle, whose
area is 168 cm2. If ratio between length & breadth of rectangle is 7 : 6,
then find the think ness of 1 disc, if these discs are kept one above
another in cylindrical vessel and 10 discs get completely fit in
cylindrical vessel.(Let radius of discs equal to radius of cylindrical
vessel)?
(a) 1.2 cm (b) 1.6 cm (c) 1.8 cm
(d) 1.4 cm (e) 1.0 cm

46. The ratio between radius of two hemispheres solid tin pieces is 2 : 3 and
difference between volume of both is 836/21 cm3. These two
𝜋
hemispheres are melted into a cylindrical vessel and used 74 cm3
3
extra tin material for polishing the vessel. If ratio between height &
radius of cylindrical vessel is 3 : 4, then find the total surface area of
cylindrical vessel?
(a) 154 cm2 (b) 132 cm2 (c) 176 cm2
(d) 208 cm2 (e) 198 cm2

Solutions

1. (e); Let, radius = 5X and height = 12X cm.


2200
Volume =
7
1 22 2200
= × × 12X × 5X × 5X =
3 7 7
X=1
r = 5 and h = 12
ℓ = √52 + 122
= √25 + 144
= √169
= 13 cm

2. (d); let side of a rectangle=a, 2a, side of a square=b


2
2(a+2a)=4b , a= b
3
8
Area of rectangle=a×2a=2a2 or ⌈ ⌉b2
9
Area of square=b 2

Ratio = 8:9
12 Adda247 Publications For any detail, mail us at
Publications@adda247.com
Cracker Book for Bank (IBPS | SBI | RRB PO | Clerk) Mains Exams

3. (d); Parameter of square = parameter of rectangle


= 2 (24 + 16)
4a = 80 cm
Side of square = 20 cm
20
Radius of circle = = 10 cm
2
Circumference of circle
22 440
2πr = × 20 = cm
7 7

4. (b); T.S.A = 2πr² + πr² + 2πrh = 2552


3πr² + 2πr × 8 = 2552
2552
3r² + 16r = ×7
22
3r² + 16r = 812
3r² + 16r – 812 = 0
3r² + 58r – 42r – 812 =0
r(3r² + 58) – 14(3r + 58) = 0
58
r = 14, −
3
2
Required volume = πr 3 + πr 2 h
3
2 22 22
= × × (14 )3 + × (14)2 × 8
3 7 7
1
= 5749 + 4928
3
1
= 10677
3

14
5. (b); Radius of circle = = 7
2
Area of circle
22
= × 7 × 7 = 154 sq. cm
7
Rectangle area = 622 – 154 = 468 sq. cm
Breadth of rectangle
468
= = 18 cm
26
Required sum
= 2πr + 2(ℓ + b)
22
= 2 × × 7 + 2(26 + 18)
7
= 44 + (44) × 2 = 132 cm.

13 Adda247 Publications For any detail, mail us at


Publications@adda247.com
Cracker Book for Bank (IBPS | SBI | RRB PO | Clerk) Mains Exams

6. (e); Let length of the wire = h


70
Radius = = 35 mm = 3.5 cm
2
4
Volume of the wire = πr 2 h = π𝑟 3
3
4
and, volume of sphere = π × (21)3
3
2 4
π𝑟 h = π × 21 × 21 × 21
3
h = 1008 cm

7. (b); Radius of cylinder = 0.63 m


Length = 7.7 m
Let side of square is ‘a’ meter
ATQ,
Curved surface area of cylinder × 500 = Area of field
22
2 × × 0.63 × 7.7 × 500 = a2
7
a = 123.47 m
approximately (a) ≈ 125 𝑚

8. (a); Ratio of length, breadth and height of cuboid →


𝑎∶𝑎∶1
As, a cuboid is cut into a cube this means after cutting ratio
become 1 : 1 : 1 or a : a : a
In starting height is more than length and breadth. We conclude
this because of the condition that base is same, and cuboid is cut
along its base, so height is reduced.
After cutting this cube again along its base a : a : a becomes a : a :
1⇒𝑎 >1

9. (a); Same base → Same radius


Let radius = r
ATQ—
1 2
3
πr H 4
=
πr2 h 3
H and h are the height of cone and cylinder respectively.
H 4
=
h 1
Ratio of height and radius of cone
H:r=4:3
14 Adda247 Publications For any detail, mail us at
Publications@adda247.com
Cracker Book for Bank (IBPS | SBI | RRB PO | Clerk) Mains Exams

h=x
h = 4x
r = 3x
slant height of cone = 5x
πrℓ+πr2 1
Ratio of T. S. A = = ⇒1∶1
2πrh+2πr2 1

𝜋𝑟 2 ℎ 616
10. (a); =
2𝜋𝑟ℎ 352
𝑟 = 3.5 m
𝜋𝑟 2 ℎ = 616
616
ℎ= = 16 m
11×3.5
Total S.A. = 2𝜋𝑟ℎ + 2𝜋𝑟 2
= 2𝜋𝑟(ℎ + 𝑟)
22
= 2 × × 3.5(3.5 + 16)
7
= 429 m2

11. (e); Let, length and breadth of rectangle be ‘l’ and ‘b’
respectively
Given (ℓ × b) = 252 cm²
and, 4𝑙 = 72 cm
𝑙 = 18 cm
length of rectangle = 18 cm
252
breadth of rectangle, 𝑏 = = 14 𝑐𝑚
18
Radius of cone = 14 cm
Height of cone = 18 cm
1 1 22
Volume of cone = 𝜋𝑟 2 ℎ = × × 14 × 14 × 18 = 3696 cm²
3 3 7

12. (a); Length + breadth + height = 12 cm


and
√ℓ2 + 𝑏 2 + ℎ2 = 5√2
ℓ² + b² + h² = 50 cm
𝑙 + 𝑏 + ℎ = 12
Square both sides
(𝑙 + 𝑏 + ℎ)2 = 122
𝑙 2 + 𝑏 2 + ℎ2 + 2 (ℓb + bh + hℓ) = 122
12² = 50 + 2 (ℓb + bh + hℓ)
2 (ℓb + bh + hℓ) = 94 cm²
15 Adda247 Publications For any detail, mail us at
Publications@adda247.com
Cracker Book for Bank (IBPS | SBI | RRB PO | Clerk) Mains Exams

13. (a); Radius of circular garden


66×7 21
= = cm
2×22 2
Area of circular garden (πr²)
22 21 21
= × × = 346.5 sq cm
7 2 2
4
Perimeter of square = 346.5 ×
5
=277.2 cm
4a = 277.2 cm
a = 69.3 cm
Area of square hall
= 69.3 × 69.3
= 4802.49 sq cm

14. (d); r = 3 cm, R = 5 cm, h = 15 cm


Total surface area of pipe
= 2πrh + 2πRh + 2(πR² – πr²)
= (2π × 3 × 15) + (2π × 5 × 15) + 2(5²π – 3²π)
= 90π + 150π + 32π
= 272π cm²

15. (a); If the diagonal of cube is 6√3 cm


Then side of cube is 6cm.
Now let volume of irregular shape is x cubic centimeter.
4
216 = π((1)3 + (2)3 + (3)3 ) + x
3
4 22
216 = × × [36] + x
3 7
1056
𝑥 = 216 −
7
456
𝑥= = 65.14 cubic centimeter
7

16. (e); Let the radius and height of cylinder be are r & h respectively
Now 20% income in both mean
20
𝑛𝑒𝑤 𝑟𝑎𝑑𝑖𝑢𝑠 = (1 + ) 𝑟 = 1.2 𝑟
100
Also 1.2h.
2π × 1.2 r(1.2r + 1.2h)
44 × 2πr (h + r) = 67760

16 Adda247 Publications For any detail, mail us at


Publications@adda247.com
Cracker Book for Bank (IBPS | SBI | RRB PO | Clerk) Mains Exams

r(h+r)= 245
Let radius and height be x and 4x
𝑥 × 5𝑥 = 245
x = 7 cm

17. (e); Let ratio of cone, cylinder and hemisphere = r


Height = r (Because height of hemisphere is equal to its radius)
Required ratio →
2πr(r + h) : πr (ℓ +r) : 3πr²
⇒ 4r : √2r+r : 3r
4 : √2 +1 : 3

18. (d); 2πr + 2 (ℓ + b) = 220 cm


πr² = 1386 sq. cm
1386×7
r2 =
22
r = 21 cm
Length of rectangle
4
= 21 × = 28 cm
3
22
2× × 21 + 2(28 + b) = 220
7
132 + 56 + 2b = 220
32
b= = 16 cm
2
Area of rectangle = (28 × 16) = 448 𝑐𝑚2

19. (c); Let r be the radius of each circle.


Then by given condition,
πR2 = 2πR ⇒ R = 2
∴ The length of the side of the square = 8
Now the area covered by 4 coins = 4 × π (2)2 = 16 π
And area of the square = 64
∴ The area which is not covered by the coins
= 64 – 16π = 16 (4–π)

17 Adda247 Publications For any detail, mail us at


Publications@adda247.com
Cracker Book for Bank (IBPS | SBI | RRB PO | Clerk) Mains Exams

20. (e); Total surface area of sphere = 4πr²


Total surface area of hemisphere = 3πr²
Let radius of hemisphere and sphere be 3𝑥 cm
And 2𝑥 cm respectively.
ATQ—
3πr³ – 4πr² = 423.5 cm²
22 22
3 × × (3𝑥 )2 – 4 × × (2𝑥 )2 = 423.5
7 7
𝑥 = 3.5 cm
21
Radius of hemisphere = cm =10.5
2

21. (d); Let level of water will be increased by h.


4
π × (15)2 × h = π(10)3
3
4 10×10×10 25
⇒h= × =5 cm
3 15×15 27

22. (e); Volume of earth removed = 10 × 4.5 × 3


= 135m3
Remaining area = [20 × 9 − 10 × 4.5] = 180 − 45 = 135
Let, rise in height be h m
Therefore,
ℎ × 135 = 135
ℎ = 1m

23. (a); Lets length of two rectangles be 7x cm 6x cm respectively


ATQ
2 (7x + 16) – 2 (6x + 16) = 8 cm
2x = 8 cm
x = 4 cm
4×7
Radius of cylinder = = 14 cm
2
Height of cylinder = 4 × 6 = 24 cm
Total surface area of cylinder = 2πr (r + h)
22
= 2 × × 14(14 + 24)
7
= 88 × 38 = 3344 cm²

18 Adda247 Publications For any detail, mail us at


Publications@adda247.com
Cracker Book for Bank (IBPS | SBI | RRB PO | Clerk) Mains Exams

Solutions : (24-25); Let height of cylinder = 4𝑥


So radius of cylinder = 3𝑥
Height of cone = 4𝑥
4𝑥
Radius of sphere = = 2𝑥
2

24. (d); T.S.A of cylinder = 2𝜋(𝑟𝑎𝑑𝑖𝑢𝑠)(𝑟𝑎𝑑𝑖𝑢𝑠 + ℎ𝑒𝑖𝑔ℎ𝑡)


T.S.A of sphere = 4𝜋(𝑟𝑎𝑑𝑖𝑢𝑠)2
2𝜋×3𝑥×(4𝑥+3𝑥) 21
Required ratio = 2
= = 21 : 8
4𝜋(2𝑥) 8

1
25. (a); Volume of cone = × 𝜋(3𝑥 )2 × 4𝑥
3
4
Volume of sphere = 𝜋(2𝑥)3
3
1
×𝜋×36×𝑥 3
3
Required % = 4 × 100 = 112.5%
×𝜋×8×𝑥 3
3
26. (c); Volume of Equilateral triangle well = volume of cylindrical tank
filled.
Let h = height of cylinder
ATQ,
√3 22
× 8 × 8 × 44√3 = ×4×4×h
4 7
⇒ h = 42 cm

27. (d); Let, the side of square = 2a


⇒ Diameter of circle = 2a
ATQ,
22
4a2 – × a × a = 262.5
7
28a2 –22a2
⇒ = 262.5
7
⇒ 6a² = 262.5 × 7
⇒ a² = 306.25
⇒ a = 17.5 cm
22
Perimeter of circle = 2 × ×a
7
22
=2× × 17.5 = 110
7

19 Adda247 Publications For any detail, mail us at


Publications@adda247.com
Cracker Book for Bank (IBPS | SBI | RRB PO | Clerk) Mains Exams

Let, Breadth of rectangle = x


⇒ length of rectangle = 1.2x
ATQ,
2(1.2x + x) = 110
55
⇒ 𝑥= = 25
2.2
Required area = 1.2 × 25 × 25 = 750 cm²

28. (d); Area of the space left out = Area of square – Area of circle

22
= (28)2 − × 14 × 14
7
11×14
= 28 (28 − )
7
= 28(28 − 22)
= 28 × 6
= 168 m2

29. (c); (ℓ − 5)(𝑏 + 3) = ℓ𝑏 − 9 ……..(i)


(ℓ + 3)(𝑏 + 2) = ℓ𝑏 + 67 ……..(ii)
Solving (i) and (ii),
ℓ𝑏 + 3ℓ − 5𝑏 − 15 = ℓ𝑏 − 9
or, 3ℓ − 5𝑏 = 6 ………….(iii)
ℓ𝑏 + 2ℓ + 3𝑏 + 6 = ℓ𝑏 + 67
or, 2ℓ + 3𝑏 = 61 ………(iv)
Solving (iii) and (iv),
ℓ = 17m

20 Adda247 Publications For any detail, mail us at


Publications@adda247.com
Cracker Book for Bank (IBPS | SBI | RRB PO | Clerk) Mains Exams

30. (a); Total surface area of the toy = C.S.A of cone + C.S.A of
Hemisphere
Let, slant height of cone be l cm.
πrℓ + 2πr² = 858 cm2
πr(ℓ + 2r) = 858 cm2
ℓ =25 cm
height of cone = 24cm
volume of the toy
1 2
= πr 2 h + πr 3
3 3
1 2(
= πr h + 2r)
3
2
= 1950 cm3
3

31. (a); 2πr + 2(ℓ + b) = 204 cm


2 (ℓ + b) = 144 × ½
2 (ℓ + b) = 72
ℓ + b = 36
(b + 8) + b = 36
b = 14
ℓ = 22
Area of rectangle = 14 × 22 = 308 sq. cm
Circle radius
2πr = 204 – 72
πr = 66
r = 21 cm
22
Area of circle = × 21 × 21
7
= 1386 sq. cm
Required sum = (308 + 1386) sq. cm
= 1694 sq. cm

32. (d); Let the radius of large sphere is 3𝑥 and small sphere is 𝑥.
Then if large sphere is melted into n small spheres.
4 4
𝜋(3𝑥 )3 = 𝑛 𝜋(𝑥 )3
3 3
27𝑥 3
𝑛 = 3 = 27
𝑥
Surface area of large sphere = 4𝜋(3𝑥 )2 = 36𝜋𝑥 2
21 Adda247 Publications For any detail, mail us at
Publications@adda247.com
Cracker Book for Bank (IBPS | SBI | RRB PO | Clerk) Mains Exams

Surface area of 27 small spheres = 27 × 4 𝜋 (𝑥 )2


= 108𝜋𝑥 2
108𝜋𝑥 2 –36𝜋𝑥 2
Required %= × 100 = 200%
36𝜋𝑥 2

33. (d); Let the radius of circle is R


Atq,
πR2 21
= ⇒ R = 21
2πR 2
Volume of sphere of same radius
4
= πR3
3
Let, radius of cylinder is r and height is h
4
πR3 = πr 2 h
3
4
r 2 h = × 21 × 21 × 21
3
= 4 × 7 × 21 × 21
r 2 h = 28 × 21 × 21 …(i)
In question
h : D = 2 : 3 [D is diameter of cylinder]
h : r = 2 : 3/2
= 4: 3
4x: 3x
Put this in equation (i)
36x3 = 28 × 21 × 21
x³ = 7 × 7 × 7
x=7
height = 28 units

34. (a); Let radius of 1st cone be r1 and that of second cone be r2.
𝜋𝑟1 ℓ = 3(𝜋𝑟2 . 3ℓ)
or, 𝜋𝑟1 ℓ = 9𝜋𝑟2 ℓ
𝑟1 9
⇒ =
𝑟2 1
Ratio of areas = 81 : 1
22 Adda247 Publications For any detail, mail us at
Publications@adda247.com
Cracker Book for Bank (IBPS | SBI | RRB PO | Clerk) Mains Exams

35. (b); Let initial radius of both vessels r cm and volume is V cm3
V = 𝜋𝑟 2 12
For first vessel, 𝑉1 = 𝜋(𝑟 + 7)2 12
Second Vessel, 𝑉2 = 𝜋(𝑟 + 7)2 12
Second Vessel, 𝑉2 = 𝜋𝑟 2 (12 + 15)
Given,
𝑉1 = (𝑉 + 𝑄 ) 𝑐𝑚3 _______(I)
𝑉2 = (𝑉 + 𝑄 ) 𝑐𝑚3 ________(II)
From (i) and (ii) We get
𝑉1 = 𝑉2
𝜋(𝑟 + 7)2 12 = 𝜋𝑟 2 (27)
r = 14 cm
So increased capacity (Q)
22 22
= × (14 + 7)2 × 12 − × 14 × 14 × 12
7 7
= 16632 – 7392
= 9240 cm3

36. (e); Given, area of square park = 19600 cm²


a² = 19600
𝑎 = 140 cm
140
Radius of each light pole = = 2.8 cm
50
Area covered by One pole in park
22
= × 2.8 × 2.8 = 24.64 cm²
7
Total area covered by these nine poles
= 9 × (24.64)
= 221.76 cm²

37. (c); Given, Area of square = 306.25 cm2


Side of square = 17.5 cm
Breadth of square = 17.5 − 3.5 = 14 cm
252
Length of rectangle = = 18 𝑐𝑚
14
Radius & height of cylindrical vessel be 14 cm & 18 cm
respectively
22 1
Volume of cylindrical vessel = × 14 × 14 × 18 × = 2772 cm3
7 4

23 Adda247 Publications For any detail, mail us at


Publications@adda247.com
Cracker Book for Bank (IBPS | SBI | RRB PO | Clerk) Mains Exams

38. (c); let height of vessel = h


3
Let radius of bowl = h = radius of vessel
2
2
Volume of hot water = 𝜋ℎ3
3
Required percentage
2 3 3
𝜋( ℎ)
3 2
= 3 2
× 100
𝜋(2ℎ) ×ℎ

= 100%

39. (a); Let radius be 3r then height will be 2r


Value of figure formed
2
= π × (3r)3 + π(3r)2h
3
2
= π × 27r 3 + π9r 2 × 2r
3
= 36 πr³ = 3r = 21
r=7
22
= 36 × ×7×7×7
7
= 36 × 22 × 49 cm³
Required time
36×22×49
=
539
= 72 minute

40. (d); Let radius of cylinder and hemisphere is ‘r’


Also height of cylinder = ‘r’
Total milk = πr² × r = πr³
Milk poured in 1 hemi-sphere
2
= πr 3
3
1
Remaining milk = πr 3
3
When this milk poured in 2nd sphere then it remained empty
2 1 1
πr 3 – πr 3 = πr 3
3 3 3
Required% = 50%
24 Adda247 Publications For any detail, mail us at
Publications@adda247.com
Cracker Book for Bank (IBPS | SBI | RRB PO | Clerk) Mains Exams

41. (e); Perimeter of corridor and hall = 102 m


Let radius of hall is R m and corridor is r m
ATQ—
2πR + πr + 2r = 102 m
22 3r 22r
2× × + + 2r = 102 m
7 2 7
204r
= 102
14
r=7m
21
So, R = m
2
Radius of circular garden
21 400
=( + 7) ×
2 100
35
= × 4 = 70 m
2
Area of Garden = πr²
= 70 × 70 𝜋
= 4900 𝜋 sq. m

42. (c); Given, height of cylindrical vessel = side of square


a2 = 256 cm²
a = 16 cm
Let radius of hemisphere is 2r cm and sphere is r cm.
ATQ—
3π(2r)² – 4πr² = 4928 cm²
12r² – 4r² = 1568
r² = 196
r = 14
Radius of cylindrical vessel
28+14
= = 21 cm
2
Volume of cylindrical vessel
22
= × 21 × 21 × 16
7
= 22176 cm³

25 Adda247 Publications For any detail, mail us at


Publications@adda247.com
Cracker Book for Bank (IBPS | SBI | RRB PO | Clerk) Mains Exams

43. (b); Ratio between length, breadth and side of square = 21 : 6 : 14


Let length and breadth of rectangle be 21x cm and 6x cm
respectively.
And, side of square be 14x cm
ATQ –
4(14x) – 2 (21x + 6x) = 6
56x – 54x = 6
x = 3 cm
14×3
Radius of cylinder = = 21 cm
2
Height of cylinder = 6 × 3 = 18 𝑐𝑚
Volume of cylinder = πr2h
22
= × 21 × 21 × 18
7
= 24948 cm3
44. (b); Let the radius of sphere be x cm.
4 1 2
⇒ 𝜋(𝑥 )3 = 18 ( 𝜋 × (4√3) × 4) + 12(𝜋 × (4)2 × 6)
3 3
4
)3
⇒ (𝑥 = 1152 + 1152 cm³
3
⇒ x = 12 cm
Then, total surface area of sphere = 4πx²
22
= 4 × × (12)2
7
2
= 1810 cm²
7

45. (d); Let length & breadth of rectangle be 7x & 6x respectively


Given area of rectangle = 168 cm2
7x × 6𝑥= 168 cm2
42x2 = 168
x2 = 4
x = 2 cm
Radius of cylindrical vessel = 6 × 2 − 1.5 = 10.5 𝑐𝑚
Height of cylindrical vessel = 7 × 2 = 14 𝑐𝑚
Volume of cylindrical vessel = 𝜋 × 10.5 × 10.5 × 14
= 1543.5π cm3
Let think ness of each discs be x cm
So, 10 × 𝜋 × x × 10.5 × 10.5 = 1543.5π cm3
1543.5
x=
10.5×10.5×10
x = 1.4cm
26 Adda247 Publications For any detail, mail us at
Publications@adda247.com
Cracker Book for Bank (IBPS | SBI | RRB PO | Clerk) Mains Exams

46. (c); Let the radius of two hemisphere be 2x & 3x


ATQ,
2 2 836
π(3x)3 – π(2x)3 = cm3 [∵ volume of hemisphere = ⅔ π r³)
3 3 21
2 836
⇒ π[19x3 ] =
3 21

⇒x=1
∴ radius are 2 cm & 3 cm.
Now,
2 2 74
[ π(2)3 + π(3)3 ] + [ πcm3 ] = πR2 H
3 3 3
2 2
⇒ π[8 + 27] + π 37 = πR2 H
3 3
2
⇒ 𝜋[35 + 37] = 𝜋𝑅2 𝐻
3

⇒ R²H = 48
⇒ R : H = 4 : 3 (given)
Let R = 4a, H = 3a
⇒ 16a². 3a = 48
𝑎=1
∴ R = 4, H = 3
T.S.A. of cylinder = 2πR(R + H)
22
=2× × 4(7)
7

= 176 cm²

27 Adda247 Publications For any detail, mail us at


Publications@adda247.com
Cracker Book for Bank (IBPS | SBI | RRB PO | Clerk) Mains Exams

1 Adda247 Publications For any detail, mail us at


Publications@adda247.com
Cracker Book for Bank (IBPS | SBI | RRB PO | Clerk) Mains Exams

Chapter
Permutation, Combination and
11 Probability

BEST APPROACH TO SOLVE THE QUESTIONS

The best way to define probability is number of favorable outcomes divided


by total number of possible outcomes. Eg. When we conduct an experiment
of throwing a dice and we have to calculate probability of getting 2.

Now number of favorable outcome is 1 i.e. ‘2’


And total number of possible outcomes is 6 i.e it may appear ‘1, 2, 3, 4, 5 or
1
6’ on throwing a dice. Hence probability of getting a ‘2’ is =
6

If we were asked what is the probability of getting an even number.


Number of possible outcome = 3 (2, 4 or 6)
3 1
Probability of getting an even number on throwing a dice = =
6 2

But the probability in bank exam is not this much easy, we encounter with
way more complicated questions.
Example 1: A C++ program shuffles all the letters of a word. When the
word INDIA was entered, a new word was given as an output.
What is the probability that new word formed is having all the
vowels together?
Solution: We need to calculate two things
(i) Total number of possible outcomes
(ii) Total number of favorable outcomes
Imagine how many different words can be formed by re-
arranging the words of INDIA.
And then how many different possible words can be formed
when ‘IIA’ of ‘INDIA’ are together.
To solve these kind of questions we have PERMUTATION AND
COMBINATION.

2 Adda247 Publications For any detail, mail us at


Publications@adda247.com
Cracker Book for Bank (IBPS | SBI | RRB PO | Clerk) Mains Exams

Permutation and Combination:


Basic definition: Permutation means different possible arrangement of
things taken all or some of them together.
While Combination means different way of selection of things taken all or
some of them together.
Eg. Of arrangement/permutation:
There were two chairs numbered 1 and 2. In how many ways A, B and C
could sit on them?
SOLUTION: AB or BA, AC or CA and BC or CB. (First person denotes to sit on
chair named A). Hence there are 6 possible ways.
Eg. Of selection/ Combination
There were two chairs and three persons. In how many different ways they
can sit on chairs?
Solution: Either AB, BC or AC will sit on two chairs. Hence there are 3
possible ways.

The point is arrangement when sequence matters is known as permutation


or selecting some or all the persons is called combination.
Before further discussion, let us revise few terms of permutation and
combination.
! is called factorial.
N! where N is a natural number is N × (N-1) × (N-2)….. ×1
0! = 1
N N!
Pr = (N–r)!, it is used in permutation cases
N N!
Cr = , it is used in combination cases.
C!(N–r)!
E.g. In previous cases.
3 3! 3×2×1
P2 = (3–2)! = = 6 arrangements.
1
3 3!
C2 = = 3 cases of selection.
2!(3–2)!

NOTE:
Though permutation and combination in itself are too big, here we will
discuss few of the types of questions that are asked in exam. Look at the
previous example of INDIA.

3 Adda247 Publications For any detail, mail us at


Publications@adda247.com
Cracker Book for Bank (IBPS | SBI | RRB PO | Clerk) Mains Exams

Example 2: What is the probability that after rearranging the word ‘INDIA’
a new word is formed in which all vowels are together?
SOLUTION: When n things in which p are of one type, q are of one type ,
n!
then total no. of arrangements of those n things is =
p!q!
Taking two Is and one A as a single entity, we have in all 3
entities (N,D and the group of is and A).
Total no. of ways of arranging these three entities is 3!.
3!
Two Is and one A can be rearranged among themselves in
2!
ways.
Hence, total no. of ways in which all vowels are together is
3!
equal to 3!× =18.
2!
Total no. of ways in which all letters of word INDIA can be
5!
arranged = =60.
2!
18 3
Hence required probability= =
60 10

Example: What is the probability that a word formed after rearranging


“ARRANGEMENT’’ such that all vowels are together?
SOLUTION: Following the same method as that of previous question,
Total no. of ways of rearranging the letters of word
11! 11!
ARRANGEMENT= =
2!2!2!2! 16
No. of ways of arranging letters such that all vowels are
8! 4! 8!4!
together= × =
2!2! 2!2! 16
8!4!
16 4
Required probability= 11! =
165
16

KEY POINTS: Number of permutation of n things taken all at a time is n!


N
Pr = NCr × r!

4 Adda247 Publications For any detail, mail us at


Publications@adda247.com
Cracker Book for Bank (IBPS | SBI | RRB PO | Clerk) Mains Exams

Practice Exercise Based on New Pattern

1. There are five mangos and six oranges in a bucket. What will be
probability of Picking up four fruits which contains at least two orange?
53 43 59
(a) (b) (c)
66 66 66
49 3
(d) (e)
66 5

1 2 3
2. Three shooter having probability of shot a target , and respectively.
3 5 7
What is the probability that exactly one of them did not hit the target?
12 29 19
(a) (b) (c)
35 105 35
41 1
(d) (e)
105 5

3. Rahul, Sachin and Sanjeev speaks 2 times, 5 times and 3 times truth out
of 5 times, 6 times and 8 times respectively. Find the probability that
they contradict each other when asked to speak on a fact?
3 5 7
(a) (b) (c)
16 16 16
13 9
(d) (e)
16 16

4. Probability of choosing one boy from a group of 26 girls and ‘x’ boys is
3
. If one more boy joined the group then find the probability of choosing
5
one boy and one girl from that group?
8 6 4
(a) (b) (c)
33 11 11
14 16
(d) (e)
33 33

5. In how many ways word “EATEN” can be arranged with or without


meaning in which at least two vowels will come together?
(a) 48 (b) 54 (c) 72
(d) 108 (e) 90

6. Sum of digits of a 5-digit number is 43. Find the probability that such a
number is divisible by 11?
(a) 1/3 (b) 1/5 (c) 2/5
(d) 2/15 (e) 2/3
5 Adda247 Publications For any detail, mail us at
Publications@adda247.com
Cracker Book for Bank (IBPS | SBI | RRB PO | Clerk) Mains Exams

7. Three-unit squares are chosen at random on a chessboard. What is the


probability that two of them are of same color while remaining one is of
another color?
(a) 16/21 (b) 8/21 (c) 5/21
(d) 4/7 (e) 32/63
8. Two dice are thrown, find the probability of getting an even number
after multiplying numbers obtained from the dice.
(a) ¼ (b) ¾ (c) ⅔
(d) ½ (e) ⅙

9. In how many ways 9 hindi books are placed between 10 English books
such that no two Hindi book comes together?
(a) 60 (b) 55 (c) 45
(d) 96 (e) 75

10. Fourteen applications apply for a job out of which there are 9 men and
5 women. If only three applications selected for the job then find the
probability that at least one of the selected application is of a woman?
60 10 80
(a) (b) (c)
91 13 91
40 50
(d) (e)
91 91

11. In a bucket there are three type of fruits i.e, mango, banana and papaya.
1
Probability of selecting one mango out of total fruits in bucket is and
3
3
the probability of selecting one papaya out of total fruits in bucket is .
8
If number of banana in the bucket is seven, then find the number of
papaya in the bucket.
(a) 4 (b) 5 (c) 11
(d) 9 (e) 7

12. A bucket contains seventeen rectangular boxes, from number 1 – 17. If


three boxes drawn at random from bucket without repeated, then what
will be probability of that all three rectangular boxes represent even
number boxes?
9 6 7
(a) (b) (c)
85 85 85
11 21
(d) (e)
85 85
6 Adda247 Publications For any detail, mail us at
Publications@adda247.com
Cracker Book for Bank (IBPS | SBI | RRB PO | Clerk) Mains Exams

13. Two boxes contain 4 and 16 balls respectively. Two balls in the first box
and four in the second box are black. If a box is chosen randomly and
two balls are drawn at random from it, what is the probability that at
least one ball is black ?
11 43 77
(a) (b) (c)
20 120 120
9 7
(d) (e)
20 23

14. Probability of getting one Red ball from a bag full of balls is 2/13 and
Black balls in the bag is 5. If white ball is 30% less than Brown ball and
40% more then the black balls then find the number of Red balls.
(a) 5 (b) 4 (c) 6
(d) 10 (e) 13

15. Probability of choosing one boy from a group of 26 girls and ‘x’ boys is
3
. If one more boy joined the group then find the probability of choosing
5
one boy and one girl from that group?
8 6 4
(a) (b) (c)
33 11 11
14 16
(d) (e)
33 33

16. In a meeting of HCL company, in the month of June, there are 6 CEO’s ,
5
Y MD’s and 7 HR’s. Probability of calling one MD on stage is . When
18
the meeting is conducted in next month (Y + 1) CEO’s, (Y + 3) MD’s and
(Y + 2) HR’s attended the meeting. If two representatives are called
together on stage, then what will be probability of both being CEO’s?
1 1 1
(a) (b) (c)
16 14 12
1 1
(d) (e)
18 20

17. There are three groups of children in a school i.e. A, B and C. In group A,
there are (x + 2) girls and x boy. In group B, there are 2 boys and 2 girls,
whereas in group C, there are 1 girl and 2 boys respectively. One child
is selected at random from each group. Find the value of x, if the
3
probability of selecting 1 girl and 2 boys is .
8
(a) 2 (b) 3 (c) 1
(d) 4 (e) 5
7 Adda247 Publications For any detail, mail us at
Publications@adda247.com
Cracker Book for Bank (IBPS | SBI | RRB PO | Clerk) Mains Exams

18. In a room, 10 people sit in which 4 left hander and 6 right hander. Find
the probability that left hander shake hand with only left handers.
1 7 17
(a) (b) (c)
15 45 45
5 2
(d) (e)
9 15

19. In a cold drink cartoon 3 pepsi, 5 limca and 7 sprites bottels. If one
bottle is drawn at random from the cartoon what is the probability that
it is either sprite or pepsi ?
5 3 2
(a) (b) (c)
7 4 3
8
(d) (e) None of these
9
20. Find the difference between number of words formed from ALAIS
when all vowels are together, and words formed from ENGLISH when
no vowels are together?
(a) 3528 (b) 3618 (c) 3572
(d) 3592 (e) 3582

21. In a box some Munch chocolate, some Dairy milk chocolate and Kit Kat
chocolate. The probability of selecting one Munch chocolate out of box
3
is and the probability of selecting one Kit Kat chocolate out of box is
10
2
. The number of Dairy milk chocolate in box is 6. Find probability of
5
selecting three chocolate from the box such that at most two chocolate
are same?
64 64 42
(a) (b) (c)
85 95 85
87 67
(d) (e)
95 85

22. Find the probability of forming the word from ANANAB such that N
never comes together.
2 4 3
(a) (b) (c)
3 5 5
2 4
(d) (e)
5 5

8 Adda247 Publications For any detail, mail us at


Publications@adda247.com
Cracker Book for Bank (IBPS | SBI | RRB PO | Clerk) Mains Exams

23. A bag contains 5 red, 4 green and 3 black balls. If three balls are drawn
out of it at random, find the probability of drawing exactly 2 red balls?
5 7 6
(a) (b) (c)
22 24 25
2 7
(d) (e)
11 22

Directions (24-25): A man can send a message by using six flags. He had 4
black flags, 3 blue flags and 1 green flag.

24. In how many ways a message can be transmitted?


(a) 108 (b) 185 (c) 20
(d) 136 (e) 110

25. In how many ways a message can be transmitted by using all three
colored flags?
(a) 150 (b) 90 (c) 100
(d) 70 (e) 125

Directions (26-27): A box contains two types of ball i.e., red and Blue.
1
Probability of choosing one red and one blue ball is while probability of
2
3
choosing two red balls is .
20

26. Find the probability of selecting two balls without replacement from
the box such that at least one of them is red?
9 11 13
(a) (b) (c)
20 20 20
17 3
(d) (e)
20 4
27. If 5 Red balls and 6 blue balls are added in the box, then find the
probability of choosing two blue balls from the box at present?
1 1 1
(a) (b) (c)
2 3 4
1 1
(d) (e)
5 6

9 Adda247 Publications For any detail, mail us at


Publications@adda247.com
Cracker Book for Bank (IBPS | SBI | RRB PO | Clerk) Mains Exams

Directions (28-30): There are three sections A, B and C in a class. Every


section has some boy and some girl students in it. Probability of a girl being
2
selected when one student is selected randomly from section A is , that
5
4 5
from section B is and that from section C is .
9 9
28. If the ratio of total number of students in sections A, B and C is 10 : 12 :
9, then what is the probability of a girl being selected when one student
is selected randomly from the students from all the three sections
together? (2 Marks)
14 11 13
(a) (b) (c)
31 23 31
43
(d) (e) Cannot be determined
93
29. If the number of girls in sections A is same as the number of boys in
section C, then what is the ratio of number of boys in section A to the
number of boys in section C? (2 Marks)
(a) 2 : 3 (b) 3 : 4 (c) 3 : 2
(d) 4 : 3 (e) 3 : 5
30. If 20 girls leaves section B, then the probability of a boy being selected
5
when one student is selected randomly from this section will be . What
8
is the number of boys in section B? (2 Marks)
(a) 120 (b) 100 (c) 75
(d) 60 (e) Cannot be determined

Directions (31-32): There are some horses, hens and elephants in a park.
Ratio between number of heads in park to number of legs in park are 7 : 20.
31. Find the probability of choosing a hen over all the animals.
3 2 3
(a) (b) (c)
7 7 8
4 2
(d) (e)
7 9
9
32. If probability of choosing 2 horses over all the animals are , then find
119
the number of elephant present in park. (Number of horses is double
than that of the number of elephants in park).
(a) 5 (b) 10 (c) 20
(d) 15 (e) 25
10 Adda247 Publications For any detail, mail us at
Publications@adda247.com
Cracker Book for Bank (IBPS | SBI | RRB PO | Clerk) Mains Exams

Directions (33-34): There are two bags, one of which contains 3 black and
4 white balls, while the other contains 4 black and 3 white balls. A dice is
cast, if the face 1 or 3 turns up, a ball is taken from the first bag and if any
other face turns up, a ball is chosen from the second bag.

33. The probability of choosing a black ball is: (2marks)


11 11 11
(a) (b) (c)
24 21 27
11 3
(d) (e)
29 8

34. The probability of choosing a white ball is: (2marks)


11 11 10
(a) (b) (c)
21 29 21
14 11
(d) (e)
23 23

Directions (35-37) : India wins every 6 matches out of 10 matches against


England. Virat Kohli (Indian Player) make a century in every 3 matches out
of 5 matches and in which he makes century, India wins 80% of the
matches. Alastair cook (England’s player) make 1 century in every 4
matches and England wins 60% of that matches.

35. Find the probability of India’s winning against England when Kohli and
Cook both make centuries.
20 17 36
(a) (b) (c)
623 625 625
24 18
(d) (e)
625 625

36. Find the probability that India wins against England when Kohli failed
to make a century.
8 3 6
(a) (b) (c)
25 5 25
4 12
(d) (e)
25 25

37. Find the probability of winning England against India when Kohli make
a century but cook does not.
9 16 2
(a) (b) (c)
250 250 5
14 21
(d) (e)
250 250
11 Adda247 Publications For any detail, mail us at
Publications@adda247.com
Cracker Book for Bank (IBPS | SBI | RRB PO | Clerk) Mains Exams

Direction (38-39): In a stationary shop there are four types of colored


sheets of red, blue, green and white colors. The probability of selecting one
1
red sheet out of the total sheets is ,the probability of selecting one blue
3
2
sheet out of the total sheets is and the probability of selecting one white
7
1
sheet out of the total sheets is . The number of green sheets in the bag is
4
22.
38. If all the sheets are numbered as 1,2,3,….and so on and one sheet is
picked up at random, then find the probability of picking up a sheet
which is numbered as a multiple of 13 or 17. (2 marks)
1 1 5
(a) (b) (c)
8 7 8
9
(d) (e) none of these
10

39. What is the total number of sheets in the bag? (1 marks)


(a) 117 (b) 168 (c) 154
(d) 120 (e) none of these

Direction (40-41) : A coach invited 9 players of the team on dinner. All 10


persons seated in a circular table.
40. If in how many ways captain and vice-captain seat opposite to each
other?
(a) 80640 (b) 5040 (c) 3624
(d) 40320 (e) 10080
41. In how many ways team can seat, if caption and vice-captain seat either
side of coach.
(a) 80640 (b) 5040 (c) 3624
(d) 40320 (e) 10080
Directions (42-44): A joker have 10 balls, 10 ribbon and 10 plates of red,
yellow and blue color in ratio 2 : 3 : 5, 5 : 2 : 3 and 3 : 5 : 2 respectively. He
does his performance by using one item of each category.

42. What is the probability that he performs with only red color?
(a) 0.3 (b) 0.02 (c) 0.25
(d) 0.03 (e) 0.15
12 Adda247 Publications For any detail, mail us at
Publications@adda247.com
Cracker Book for Bank (IBPS | SBI | RRB PO | Clerk) Mains Exams

43. What is the probability that he chooses yellow color for ribbons?
(a) 0.0333 (b) 0.025 (c) 0.2
(d) 0.25 (e) 0.03

44. If he chooses 3 balls, 5 ribbons and 1 plate then find the probability that
all items are of same color.
(a) 1 (b) 0 (c) 0.05
(d) 0.5 (e) 0.002

Direction (45-47): - Kishan have two trays of eggs i.e. tray A and tray B.
Tray A have same number of fresh eggs as tray B have rotten eggs. Rotten
eggs in tray A is 40% less than that of in tray B. Fresh eggs in tray B is 14
more than that in tray A. Total number of eggs Kishan have is 140.
45. Fresh eggs in tray B is what percent more than that in tray A?
(a) 140% (b) 80% (c) 60%
(d) 40% (e) 20%

46. Find the probability of choosing two eggs from tray ‘A’ such that at least
one egg should be fresh?
19 17 27
(a) (b) (c)
22 22 44
25 21
(d) (e)
44 22

47. Find the probability of choosing one rotten egg?


23 7 19
(a) (b) (c)
48 16 48
25 9
(d) (e)
48 16

Direction (48- 50)- There are three bucket P, Q & R contains three fruits
Mangos, Bananas & Oranges.
In bucket P, contains x Mangos and y Bananas. Number of bananas are 3
more than the number of Mangos. If Ram selected one fruits for eating, then
3 1
the probability of getting orange is . The value of x is 37 % less than y.
16 2
In bucket Q, number of Mangos is 40% more than that of bucket P. If Satish
selecting two fruits at random for eating then the probability of getting both
18
Bananas is . Total number of fruits in bucket Q is 29.
203
13 Adda247 Publications For any detail, mail us at
Publications@adda247.com
Cracker Book for Bank (IBPS | SBI | RRB PO | Clerk) Mains Exams

In bucket R, the ratio of number of mangos to number of orange is 3 : 4.


Total number of bananas & orange is 33 . If Veer selecting one fruit for
3
eating the probability of getting mangos is .
14

48. What is the total number of bananas present in all the three buckets?
(a) 30 (b) 38 (c) 36
(d) 32 (e) 28

49. If all mangoes of bucket P is placed in the bucket Q then find the
probability of choosing a mango from bucket Q?
5 9 3
(a) (b) (c)
17 17 17
7 6
(d) (e)
17 17

50. If 25% oranges of bucket R are spoiled and thrown away then find the
probability of choosing a good orange from bucket R?
3 5 7
(a) (b) (c)
13 13 13
6 2
(d) (e)
13 13

Solutions

1. (a); Favorable case = (20, 2M) or (30, 1M) or 40


∴ Probability
6C × 5C 6C × 5C 6C
2 2 3 1
= 11C + 11C + 11 4
4 4 C4
15×10 20×5 15 265 53
= + + = =
330 330 330 330 66

2. (b); Required probability


1 2 4 1 3 3 2 3 2 29
= × × + × × + × × =
3 5 7 3 7 5 5 7 3 105

2
3. (d); Probability that Rahul speaks truth =
5
5
Probability that Sachin speaks truth =
6
3
Probability that Sanjeev speaks truth =
8

14 Adda247 Publications For any detail, mail us at


Publications@adda247.com
Cracker Book for Bank (IBPS | SBI | RRB PO | Clerk) Mains Exams

Rahul, Sachin and Sanjeev do not contradict with each other if they
all are correct or they all are incorrect means if they all speak truth
or they all speak lie.
2 5 3 1
Probability that they all speak truth = × × =
5 6 8 8
3 1 5 1
Probability that they all speak lie= × × =
5 6 8 16
1 1 16 –2 –1 13
Required probability = 1 – – = =
8 16 16 16

4. (e); ATQ,
x 3
=
26+x 5
⇒ 5x = 26 × 3 + 3x
⇒ 2x = 26 × 3
⇒ x = 39
Total no. of students now in group
= 26 + 39 + 1 = 66
26 40 16
Required probability = 2 × × =
66 65 33

5!
5. (b); Total number of ways = = 60 ways
2!
Selection of vowels can occur in three pair i.e. ( AE, EE, EA)
Number of ways in which two vowels will come together
3×4!
= = 3 × 4 × 3 = 36
2!
Above, we had not multiplied 2 in numerator because further
arrangement of vowels selected i.e.
(AE, EE, EA) will give the same arrangement
Number of ways in which three vowels will come together
3!×3!
= = 3 × 2 × 3 = 18
2!
Required no. of ways = 36 + 18 = 54
Alternate,
No. of ways in which no vowel will came together
3!
=2× =6
2!
Required no. of ways = 60 – 6 = 54
15 Adda247 Publications For any detail, mail us at
Publications@adda247.com
Cracker Book for Bank (IBPS | SBI | RRB PO | Clerk) Mains Exams

6. (b); Let, numbers in 5-digit number is a, b, c, d, e.


In order to get sum of 43, the combinations exist are
9, 9, 9, 9, 7 = 5 numbers
9, 9, 9, 8, 8 = 10 numbers
Total 15 numbers can be formed having sum 43.
For number which are divisible by 11 main condition is
(a + c + e) – (b + d) = 11 …(i)
and (a + b + c + d + e) = 43 …(ii)
On solving (i) and (ii)
a + c + e = 27, b + d = 16
a, c and e can be 9, 9, 9
while b, d can 7, 9 or 8, 8 or 9, 7 respectively
So, there are three numbers which are divisibly by 11 having sum
43 i.e., 97999, 99979, 98989.
3 1
Required probability = =
15 5

7. (a); Total number of ways = ⁶⁴C₃ = 41664


Favorable ways = 2 × ³²C₂ × ³²C₁
= 2 × 496 × 32 = 31744
31744 16
Required probability = =
41664 21
(³²C₂ × ³²C₁ is taken because if two are black, then one will be of
white or if two are white, then one will be of black)

8. (b); Favourable case →


(Dice 1 → even and dice 2 → odd)
(Dice 1 → even and dice 2 → even)
(Dice 1 → odd and dice 2 → even)
3 3 3 3 3 3 3
= × + × + × =
6 6 6 6 6 6 4

9. (b); Let us denote English book by E and Hindi books by H


The possible arrangements is
HEHEHEHEHEHEHEHEHEHEH
There are → English book, H can occupy 10 + 1 = places
So no. of ways → ¹¹C₉
11×10
= = 55 ways
2
16 Adda247 Publications For any detail, mail us at
Publications@adda247.com
Cracker Book for Bank (IBPS | SBI | RRB PO | Clerk) Mains Exams

10. (b); Total number of ways to select three persons = 14C3


Required case = 2 men 1 women or 1 man 2 women or 3 women
= ( 9C2 × 5C1 ) + ( 9C1 × 5C2 ) + ( 5C3 )
= (36 × 5) + (9 × 10) + (10)
= 180 + 90 + 10
= 280 14C3 = 14C3 = 364
280 10
Required probability = =
364 13

11. (d); Let number of papaya and mango in bucket be ‘P’ and ‘M’
respectively
Number of banana in bucket = 7
1
Probability of selecting one mango =
3
3
and selecting one papaya =
8
So,
M 1
(7+M+P)
=
3
3M = 7 + M + P
2M – P = 7 …(i)
Similarly—
P 3
(7+M+P)
=
8
8P = 21 + 3M + 3P
5P – 3M = 21 …(ii)
Solving (i) and (ii) we get
Number of Mango = 8, Banana = 7, Papaya = 9
Total number of papaya in bucket = 9

12. (c); Total even number boxes between 1 to 17


= 8 (2, 4, 6, 8, 10 …..16)
Total boxes = 17
8
Probability of first box represent even number box =
17
7
Probability of second box represent even number box =
16
6
Probability of third box represent even number box =
15
8 7 2 7
Required probability = × × =
17 16 5 85

17 Adda247 Publications For any detail, mail us at


Publications@adda247.com
Cracker Book for Bank (IBPS | SBI | RRB PO | Clerk) Mains Exams

13. (c); At least one black can be chosen in three ways:


1. first one is black, second is non-black
2. first one is non-black, second is black
3. both are black.
1
Probability of selecting a box is
2
Now, probability of choosing at least one black ball from first box
1 2 2 2 2 2 1 5
= ×( × + × + × )=
2 4 3 4 3 4 3 12
Probability of choosing at least one black ball from 2nd box
1 4 12 12 4 4 3 27
= ×( × + × + × )=
2 16 15 16 15 16 15 120
5 27 50+27 77
Required probability = + = =
12 120 120 120

14. (b); Let number of red balls is 2x


Total balls = 13x
Now ATQ
Black balls = 5
5×140
White balls = =7
100
7
Brown balls = × 100 = 10
70
Now→ 2x + 5 + 7 + 10 = 13x
x=2
Red balls = 4

15. (e); ATQ,


x 3
=
26+x 5
⇒ 5x = 26 × 3 + 3x
⇒ 2x = 26 × 3
⇒ x = 39
Total no. of students now in group
= 26 + 39 + 1 = 66
26 40 16
Required probability = 2 × × =
66 65 33
16. (b); In June —
Total CEO’s = 6
MD’s = y ⇒ HR’s = 7
Now,
18 Adda247 Publications For any detail, mail us at
Publications@adda247.com
Cracker Book for Bank (IBPS | SBI | RRB PO | Clerk) Mains Exams

ATQ,
y 5
= ⇒ y=5
13+y 18
In July —
CEO’s ⇒ 5 + 1 = 6
MD’s ⇒ 5 + 3 = 8
HR’s ⇒ 5 + 2 = 7
6C 6×5 1
2
Required probability = 21C = =
2 21×20 14

17. (c); Possible cases


Girl Boy Boy or Boy
Girls Boy or Boy Boy Girl
3 x+2 2 2 x 2 2 x 2 1
=( × × )+( × × )+( × × )
8 2x+2 4 3 2x+2 4 3 2x+2 4 3
(2x+2)×4×3 2(x+2) 2x x
⇒ =( + + )
2×8 3 3 3
(x+1)×9
⇒ = 5x + 4
2
⇒ 9x + 9 = 10x + 8
⇒ x=1

18. (e); Total no. of ways in which hand shakes


10(10 –1)
= = 45
2
No. of ways in which left hander shakes hand with left handers
4(4 –1)
= =6
2
6 2
Required probability = =
45 15

19. (c); Total number of bottle in cartoon


= 3 + 5 + 7 = 15
∴ Probability of either sprite bottle or pipe bottle
7C + 3C 7+3 2
1 1
= 15C = =
1 15 3

19 Adda247 Publications For any detail, mail us at


Publications@adda247.com
Cracker Book for Bank (IBPS | SBI | RRB PO | Clerk) Mains Exams

20. (e); AAILS when all vowels are together


3!
= 3! × = 18 words
2!
Total words from ENGLISH = 7! = 5040
Words from ENGLISH when no vowels are together
= 5040 – [when all vowels are together]
= 5040 – [6! × 2] = 3600
∴ Required difference = 3600 – 18 = 3582

21. (d); Let total chocolate in box be 10c. Munch chocolate be a and kit-kat
chocolate be b .
a 3
Probability of selecting Munch chocolate = =
10c 10
a = 3c ... (i)
Probability of selecting Kit Kat chocolate
b 2
= =
10c 5
b = 4c … (ii)
ATQ,
a + b + 6 = 10c
⇒ 3c + 4c + 6 = 10c ⇒ c = 2
So, total chocolate = 20
6C + 8C + 6C
3 3 3
Required probability = 1 − 20 C
3
(20+56+20) 1044 87
=1− = =
1140 1140 95

22. (a); Total number of words possible from ANANAB


6!
= = 60
3!2!
No. of word in which N is always together
5!
= = 4 × 5 = 20
3!
No. of in which N is never together = 60 – 20 = 40
40 2
Required probability = =
60 3

20 Adda247 Publications For any detail, mail us at


Publications@adda247.com
Cracker Book for Bank (IBPS | SBI | RRB PO | Clerk) Mains Exams

23. (e); Exactly 2 red balls can be selected in two ways.


(i) 2 red ball & 1 green ball
(ii) 2 red ball & 1 black ball
Total no. of possible outcome = ¹²C₃
12×10×11
= = 220 ways
3×2
Total favourable outcome
= ⁵C₂ × ⁴C₁ + ⁵C₂ × ³C₁
5×4 5×4
= ×4+ × 3 = 40 + 30 = 70 ways.
2 2
70 7
Probability = =
220 22

24. (b); Case I: When all 4 black flag selected


Two ways could be
1 green flag + 1 blue flag or 2 blue flags
6! 6!
No. of ways to arrange = +
4! 4!2!
Case II:
When all 3 blue flag selected
3 ways could be
2 black flags + 1 green flag or 3 black flags
6! 6!
No. of ways = +
3!2! 3!3!
Case III: When one green selected
6!
⇒ 3 black + 2 blue [other cases already included]
3!2!
Adding all
⇒ 30 + 15 + 60 + 20 + 60 = 185

25. (a); All colored used ⇒


1 green flag + 2 black flags + 3 blue flags
1 green flags + 3 black flags + 2 blue flags
1 green flag + 4 black flags + 1 blue flag
6! 6! 6!
⇒ + +
2!3! 3!2! 4!
⇒ 60+ 60 + 30 = 150

21 Adda247 Publications For any detail, mail us at


Publications@adda247.com
Cracker Book for Bank (IBPS | SBI | RRB PO | Clerk) Mains Exams

Solution (26-27): Let, Red balls = x


Blue balls =y
ATQ,
x y 1
2× × = ... (i)
(x+y) (x+y−1) 2
And,
x×(x−1) 3
= ... (ii)
(x+y)(x+y−1) 20
Dividing (ii) by (i)
x−1 3
⇒ =
y 5
⇒ 5x – 5 = 3y …(iii)
And, from (i)
4xy = (x + y) (x + y – 1) ... (iv)
From (iii) and (iv)
5x−5 5x−5+3x 5x−5
4x ( )=[ ] [x + − 1]
3 3 3
20x 8x−5 8x−8
⇒ (x − 1) = [ ][ ]
3 3 3
⇒ 60x² – 60x = 64x² - 64x – 40x + 40
⇒ -4x² + 44x – 40 = 0
⇒ x² – 11x + 10 = 0
⇒ x²– 10x – x + 10 = 0
⇒ x(x – 10) – 1 (x-10) = 0
⇒ x = 1, 10
x = 1 x = 10
⇒ y = 0 ⇒ y = 15But y can’t be zero.
⇒ Number of red balls = 10
Number of blue balls = 15

26. (c); Required probability


10 C + 10 C 15 C 45+150 195 13
2 1 1
y= 25 C = = =
2 300 300 20

21 C 21×20 1
2
27. (b); Required probability = 25+11 C = =
2 36×35 3
22 Adda247 Publications For any detail, mail us at
Publications@adda247.com
Cracker Book for Bank (IBPS | SBI | RRB PO | Clerk) Mains Exams

28. (d); Probability of a girl being selected from a section


Total girls in the section
=
Total students in the section
Let the number of girls, number of boys and total number of
students respectively:
For section A: 2x, 3x and 5x.
For section B: 4y, 5y and 9y.
For section C: 5z, 4z and 9z.
According to the question,
Ratio of total number of students in the three sections:
⟹ 5x : 9y : 9z = 10 : 12 : 9
⟹x:y:z=6:4:3
Let the values of x, y and z be 6k, 4k and 3k respectively.
Total number of girls in all the three sections = 2x + 4y + 5z = 12k +
16k + 15k = 43k
Total number of students in all the three sections
= 5x + 9y + 9z = 30k + 36k + 27k = 93k
Probability of a girl being selected from the students from all the
three sections together
Total girls in all sections 43k 43
= = =
Total students in all sections 93k 93

29. (c); According to the question,


Number of girls in sections A = Number of boys in section C
⟹ 2x = 4z ⟹ x = 2z
Number of boys in section A : Number of boys in section C = 3x : 4z
= 6z : 4z = 3 : 2

30. (b); Probability of a boy being selected from this section B after 20 girls
5
left the section =
8
Number of boys in section B 5
⟹ =
Total number of students in section B−20 8
5y 5
⟹ =
9y − 20 8
⟹ 40y = 45y – 100 ⟹ y = 20
Number of boys in section B = 5y = 100
23 Adda247 Publications For any detail, mail us at
Publications@adda247.com
Cracker Book for Bank (IBPS | SBI | RRB PO | Clerk) Mains Exams

Solution (31-32): Let number of hens, horses and elephant in park is — a,


b and c respectively.
Let number of heads and number of legs be 7x and 20x respectively
So,
a + b + c = 7x
b + c = 7x– a …(i)
2a + 4(b + c) = 20x …(ii)
Solving (i) and (ii)
2a + 4 × (7x– a) = 20x
2a + 28x – 4a = 20x
a = 4x

4x 4
31. (d); Probability of choosing hen over all the animals = =
7x 7
32. (a); No. of hens = 4x
No. of horses and elephants together = 7x– 4x = 3x
If number of horses is double than that of the number of elephants then
Number of horses = 2x
Number of elephant = x
ATQ
2xC 9
2
7xC =
2 119
2x(2x–1) 9
=
7x(7x–1) 119
x=5

2 3 1
33. (b); Probability of choosing a black ball from bag 1 = × =
6 7 7
4 4 8
Probability of choosing a black ball from bag 2 = × =
6 7 21
1 8 11
Total probability = + =
7 21 21

2 4 8
34. (c); Probability of choosing a white ball from bag 1 = × =
6 7 42
4 3 12
Probability of choosing a white ball from bag 2 = × =
6 7 42
10
Total probability =
21
24 Adda247 Publications For any detail, mail us at
Publications@adda247.com
Cracker Book for Bank (IBPS | SBI | RRB PO | Clerk) Mains Exams
6
35. (e); Probability that India wins against England =
10
Condition applied
3
Probability that Kohli makes century =
5
4
Probability that India win when Kohli makes century =
5
1
Probability of Cook’s century =
4
Probability of other country wining when Cook make century
3 2
= 1– =
5 5
6 3 4 1 2 18
Combining all = × × × × =
10 5 5 4 5 625

2
36. (c); Probability when Kohli failed to make a century =
5
So, Probability that India wins against England when Kohli failed to
6 2 6
make a century = × =
10 5 25

3
37. (a); Probability that Kohli will make a century =
5
1
Probability that India lose when Kohli make century =
5
3
Probability that Cook doesn’t make century =
4
4
Probability that of Winning of England against India =
10
4 3 3 1 9
Overall probability = × × × =
10 4 5 5 250

38. (a); Let, total no. of sheets be 84x,


1
Then no. of red sheets = × 84x = 28x
3
2
no. of blue sheets = × 84x = 24x
7
1
no. of white sheets = × 84x = 21x
4
Now,
28x + 24x + 21x + 22 = 84x
or, 11x = 22
or, x = 2
Total no. of sheets = 84x = 168
There are 12 multiples of 13 and 9 multiples of 17 from 1 to 168.

25 Adda247 Publications For any detail, mail us at


Publications@adda247.com
Cracker Book for Bank (IBPS | SBI | RRB PO | Clerk) Mains Exams

Also, there is no multiple of 13 and 17 from 1 to 168. (because,


13× 17 = 221)
21 1
Hence, required probability= =
168 8

39. (b); Total no. of sheets = 168

40. (e);

When two places fixed so we can arrange other 8 in → (8!) ways


and captain and vice-captain
Total ways ⇒ 8! ⇒ 40,320

41. (e);

3 places fixed ⇒
So total place ⇒ 10 – 3 = 7
Number of way to arrange ⇒ 7!
Captain and vice-captain change their place in two ways
So, total ways ⇒ 7 ! × 2 = 10,080

Solutions (42-44)

Balls = 10 Ribbon = 10 Plates = 10


Red = 2 Red = 5 Red = 3
Yellow = 3 Yellow = 2 Yellow = 5
Blue = 5 Blue = 3 Blue = 2
26 Adda247 Publications For any detail, mail us at
Publications@adda247.com
Cracker Book for Bank (IBPS | SBI | RRB PO | Clerk) Mains Exams

42. (d); Performance with only red color


One ball of red color, one ribbon of red color and one plate of red
color
2 5 3 3
Required Probability = × × = = 0.03
10 10 10 100

43. (c); Condition is yellow color for ribbon while plate and ball can be of
any color so, probability of choosing ball and plate is 1 while
probability of choosing one yellow color ribbon is
2
= = 0.2
10

44. (b); Now


Joker choose → 3 balls, 5 Ribbon and 1 plate
Condition → all are of some color
⇒ 5 ribbons of same color is only of red color
But, 3 red ball cannot be possible so
Required Probability = 0
Solution (45-47) : - Let fresh eggs in tray A = 100x = Rotten eggs in
tray B
Rotten eggs in tray A = 60x
Fresh eggs in tray B = 100x + 14
Total eggs = 100x + 100x + 60x + 100x + 14 = 140
360x = 126 ⇒ x = 0.35
Tray A Tray B
Fresh Rotten Fresh Rotten
35 21 49 35

49−35 14
45. (d); Required % = × 100 = × 100 = 40%
35 35

35 34 35×21 17 21 19
46. (a); Required probability = × +2× = + =
56 55 56×55 44 44 22

1 21 1 35 3 5 19
47. (c); Required probability = × + × = + =
2 56 2 84 16 24 48

27 Adda247 Publications For any detail, mail us at


Publications@adda247.com
Cracker Book for Bank (IBPS | SBI | RRB PO | Clerk) Mains Exams

Direction (48– 50): In bucket P –


Given, y = x + 3 … (i)
3y
x= … (ii)
8
By solving (i) & (ii) we get
Mangos (x) = 5
Bananas (y) = 8
Let the number of Orange be z.
z 3
Then, =
13+z 16
16z = 39 + 3z
z=3
In bucket Q –
140
Mangos = 5 × =7
100
Let the number of Bananas be ‘a’ and number of Orange be ‘b’
7 + a + b = 29 ⇒ a + b = 22
aC 18 a×(a−1) 18
2
29C = ⇒ =
2 203 2×406 203
a × (a − 1) = 72
Bananas (a) = 9 ⇒ Orange (b) = 12
In bucket R –
Let Mangos & Orange be 3x & 4x respectively
3xC 3 3x 3
1
= ⇒ =
33+ 3xC1 14 33+3x 14
14x = 33 + 3x ⇒ x = 3
Mangos = 9 ⇒ Orange = 12
Bananas = 33 – (9 + 12 ) = 21

48. (b); Total bananas in all three buckets = 8 + 9 + 21 = 38

5+7 6
49. (e); Required probability = =
29+5 17

3
50. (a); In bucket R Good orange = 12 × = 9
4
Total fruits = 9 + 21 + 9= 39
9 3
Probability = =
39 13

28 Adda247 Publications For any detail, mail us at


Publications@adda247.com
Cracker Book for Bank (IBPS | SBI | RRB PO | Clerk) Mains Exams

Adda247 Publications For any detail, mail us at


1
Publications@adda247.com
Cracker Book for Bank (IBPS | SBI | RRB PO | Clerk) Mains Exams

Chapter

12 Inequality

BEST APPROACH TO SOLVE THE QUESTIONS

In examination, we come across questions in which we need to find two


quantities and compare them to reach at the correct answer. Usually such
questions are asked in two ways. In one type of questions, a common data is
provided based on which we are supposed to find both quantities. In other
type of questions each quantity comes up with some data pertaining to it
only.

In these questions, it’s important not to mingle or use data of one


quantity in finding other quantity in cases where separate data is given for
both the quantities. Where common data is provided for both quantities, we
have to use it for calculating both the quantities. After calculating, we
compare both quantities and choose the correct answer from the options
available.

Example: A and B starting a business together. B invested 60% more than


that of A. Ratio of time period of investment of A and B is 8 : 9.
Quantity I: A’s share of profit out of total profit of Rs. 25,200.
Quantity II; B’s share of profit out of total profit of Rs. 14,000.
(a) Quantity I > Quantity II
(b) Quantity I < Quantity II
(c) Quantity I ≥ Quantity II (d) Quantity I ≤ Quantity II
(e) Quantity I = Quantity II or No relation

Solution: (e); Ratio of profit share of A and B


x×8 x×1.6×9
5 : 9
5
Quantity I: A’s profit = 25,200 × = 9000
14
9
Quantity II: B’s profit = 14,000 × = 9000
14
Quantity I = Quantity II

Adda247 Publications For any detail, mail us at


2
Publications@adda247.com
Cracker Book for Bank (IBPS | SBI | RRB PO | Clerk) Mains Exams

Over the years, questions that test a candidate’s basic algebraic


knowledge have been and are being asked. In these questions, two algebraic
equations are provided. The equation may be linear, quadratic or cubic. At
times, we may come across equations containing higher powers of variable,
but they aren’t too difficult to solve.

These equations generally contain one variable. From the given couple of
equations we need to find the values of two variables. Usually one of the
equations gives us the value of one of the variables and the other gives the
value of the other variable. After finding the values of two variables we
compare them and select the correct option accordingly.

Example: I. 21x² – 22x + 5 = 0


II. 63y² – 54y + 11 = 0

Solution (a); I. 21x² – 22x + 5 = 0


⇒ 21x² – 15x – 7x + 5 = 0
⇒ 3x (7x – 5) – 1 (7x – 5) = 0
⇒ (3x – 1) (7x – 5) = 0
1 5
⇒ 𝑥 = or
3 7
II. 63y² – 54y + 11 = 0
⇒ 63y² – 21y – 33y + 11 = 0
⇒ 21y (3y – 1) – 11 (3y – 1) = 0
⇒ (21y – 11) (3y – 1) = 0
11 1
⇒𝑦 = or
21 3
∴ no relation can be established.

Adda247 Publications For any detail, mail us at


3
Publications@adda247.com
Cracker Book for Bank (IBPS | SBI | RRB PO | Clerk) Mains Exams

Practice Exercise Based on new Pattern

PART A
1. B is thrice as efficient as C. B and C can compete a work together in
45/2 days. A takes 50% more days than the days taken by A and B to
complete the same work together.
Quantity 1: No. of days taken by fastest among them to complete the
work alone.
Quantity 2: Time taken by A and C to complete the work together.
(a) Quantity I > Quan tity II (b) Quantity I < Quantity II
(c) Quantity I ≥ Quantity II (d) Quantity I ≤ Quantity II
(e) Quantity I = Quantity II or No relation

2. Quantity I→ The profit earned by selling an item (in Rs) if the


2
difference b/w the SP and the CP is 117 % of 600.
3
Quantity II→ The cost price of an article (in Rs) if the selling price of
the article is 1000 Rs and he got 25% profit after selling the item.
(a) Quantity I > Quantity II (b) Quantity I < Quantity II
(c) Quantity I  Quantity II (d) Quantity I = Quantity II
(e) No relation

3. The largest possible right circular cylinder is cut out from a wooden
cube of edge 7 cm.
Quantity I: volume of the cube left over after cutting out the cylinder
Quantity II: Surface area of cube remained after cutting out the
cylinder.
Note: compare the magnitudes of both quantities.
(a) Quantity I > Quantity II (b) Quantity I < Quantity II
(c) Quantity I ≥ Quantity II (d) Quantity I ≤ Quantity II
(e) Quantity I = Quantity II or No relation
Adda247 Publications For any detail, mail us at
4
Publications@adda247.com
Cracker Book for Bank (IBPS | SBI | RRB PO | Clerk) Mains Exams

4. Quantity I- ‘Area of rectangle’ Sum of circumference of circle and


perimeter of rectangle is 220 cm. and area of circle is 1386 sq. cm. If
1
length of rectangle is 33 % more than radius of circle.
3
Quantity II- ‘Perimeter of square’ Circumference of a circle is 132 cm
and area of circle is 710 sq. cm more than area of square
(a) Quantity I > Quantity II (b) Quantity I < Quantity II
(c) Quantity I ≥ Quantity II (d) Quantity I ≤ Quantity II
(e) Quantity I = Quantity II or no relation

5. Quantity I – Veer takes two times more than the time taken by Sameer
to complete a work. Sameer and veer started to work alternatively
starting with Sameer on first day and Veer on second day. If both
completed work in 36 days, then find in how many days Sameer and
Veer will complete the work together, if both work with their double
efficiency?

Quantity II – Tap P can fill a swimming pool in 8 days and tap Q can fill
the same swimming pool in 24 days, while tap R can empty the
swimming pool in 16 days. If all three taps are opened together in
swimming pool, then in how much time swimming pool will filled
completely?
(a) Quantity I > Quantity II (b) Quantity I < Quantity II
(c) Quantity I ≥ Quantity II (d) Quantity I ≤ Quantity II
(e) Quantity I = Quantity II or no relation

6. Quantity I: P alone can do a piece of work in 72 days and Q alone can


do same work in 48 days, R is 50% more efficient than Q then find in
how many days total work will be completed if all three work on
alternate days starting with P and ending with R ?
Quantity II: A alone can do a task in 48 days B is 25% less efficient than
A and C can do the same task in 40 days lesser than B does. Then find in
how many days all three will complete the task together?
(a) Quantity I > Quantity II (b) Quantity I < Quantity II
(c) Quantity I ≥ Quantity II (d) Quantity I ≤ Quantity II
(e) Quantity I = Quantity II or no relation
Adda247 Publications For any detail, mail us at
5
Publications@adda247.com
Cracker Book for Bank (IBPS | SBI | RRB PO | Clerk) Mains Exams

7. Quantity I: A bucket carried one dozen of mangoes. One third of


mangoes become bad, if three mangoes taken out from the bucket at
random. Then what is probability that one mango out of three mangoes
picked up is good?
Quantity II: Find the probability of selecting a face card from a well
shuffled pack of card given that 2 additional Joker card also counts as a
face card?
(a) Quantity I > Quantity II (b) Quantity I < Quantity II
(c) Quantity I ≥ Quantity II (d) Quantity I ≤ Quantity II
(e) Quantity I = Quantity II or no relation

8. Quantity I – A train running at the speed of 72 km/hr passed a man in


14 sec and a platform in 32 sec. Find the length of platform ?
Quantity II – A train moves at the speed of 108 km/hr, passes a
platform and a bridge in 15 sec and 18 sec respectively. If the length of
platform is 50% of length of bridge, then find the length of train.
(a) Quantity I > Quantity II (b) Quantity I < Quantity II
(c) Quantity I ≥ Quantity II (d) Quantity I ≤ Quantity II
(e) Quantity I = Quantity II or no relation

9. Quantity I — Difference between the speeds of P and Q if 2 places A


and B are 60 km apart. P and Q start from A at same time & meet 1st
time at a place 12 km from B & they reach A after immediate return
from B. The speed of slower person is 48 km/hr.
Quantity II — Average speed of train if a distance of 600 km is to be
covered in 2 parts. In 1st phase 120 km is travelled by train and rest by
car and it took total of 8 hrs, but if 200 km is covered by train and rest
by car it takes 20 min more.
(a) Quantity I > Quantity II (b) Quantity I < Quantity II
(c) Quantity I ≥ Quantity II (d) Quantity I ≤ Quantity II
(e) Quantity I = Quantity II or No relation
Adda247 Publications For any detail, mail us at
6
Publications@adda247.com
Cracker Book for Bank (IBPS | SBI | RRB PO | Clerk) Mains Exams

10. Quantity I — Value of 𝑥, if ABCD is a rectangle and AB= 10 unit, AD= 6


unit

Quantity II — value of 𝑦, if volume of the cone is 16𝜋 unit3


Radius = 4 unit

(a) Quantity I > Quantity II (b) Quantity I < Quantity II


(c) Quantity I ≥ Quantity II (d) Quantity I ≤ Quantity II
(e) Quantity I = Quantity II or No relation

11. Quantity I – What is value of (A + B) if 25% of 48% of 55% of A is 1188


and 21% of 35% of B is equal to 5 more than square of 32.

Quantity II – Ratio of male population to female population of a town


in the year 2016 was 8 : 9. If in the year 2017 male population
increased by 25 % and female population increased by 40% as
compared to previous year so total population of town in the year 2017
was 45200. What was the total population of town in 2016.
(a) Quantity I > Quantity II (b) Quantity I < Quantity II
(c) Quantity I ≥ Quantity II (d) Quantity I ≤ Quantity II
(e) Quantity I = Quantity II or no relation
Adda247 Publications For any detail, mail us at
7
Publications@adda247.com
Cracker Book for Bank (IBPS | SBI | RRB PO | Clerk) Mains Exams

12. Quantity I – A man have two bucket which contains some fruits, first
bucket contains 5 mango and 7 banana and second bucket contains 4
mango and 5 banana. Also man have a pack of six card with numbering
1 - 6, if a card is taken out and it is even number, then he choose a fruit
from first bucket but if card is odd number, then man choose a fruit
from second bucket. Find the probability of choosing a mango.

Quantity II – In egg tray carried two dozen of eggs. One third of eggs
are rotten, if three eggs taken out from the tray at random. What is
probability that two eggs out of three eggs picked up is good?
(a) Quantity I > Quantity II
(b) Quantity I < Quantity II
(c) Quantity I ≥ Quantity II
(d) Quantity I ≤ Quantity II
(e) Quantity I = Quantity II or no relation

13. ‘A’ can complete a work alone in 12 days. A and B together can complete
2
the work in 6 days. C takes 5 more days than B to complete the work
3
alone. C is 50% more efficient than D.
Quantity I= ‘x’: A, B and C worked for alternate days starting from A
and finish the work in ‘x’ days

Quantity II= ‘y’: B and C worked together for 6 days and left and then D
completes the remaining work. ‘y’ is total time taken to complete the
work.
(a) Quantity I > Quantity II (b) Quantity I < Quantity II
(c) Quantity I ≥ Quantity II (d) Quantity I ≤ Quantity II
(e) Quantity I = Quantity II or No relation

14. Quantity I= ‘x’: A shopkeeper mark up an article 40% above its cost
price. At the time of sale, he gave 20% discount. By this, shopkeeper
earn Rs 168. ‘x’ is the difference between mark up price and cost price.
Quantity II= ‘y’ : Satish invested Rs.12000 in scheme ‘A’ at 12% S.I P.A.
for 4 years and Rs.25,000 in scheme B at 11% S.I P.A for 2 year. ‘y’ is the
difference between the interest earn from both schemes.
Adda247 Publications For any detail, mail us at
8
Publications@adda247.com
Cracker Book for Bank (IBPS | SBI | RRB PO | Clerk) Mains Exams

(a) Quantity I > Quantity II (b) Quantity I < Quantity II


(c) Quantity I ≥ Quantity II (d) Quantity I ≤ Quantity II
(e) Quantity I = Quantity II or No relation

15. ABCD is a rectangle and O is the midpoint of the line EF and diagonal
AC. AO=20cm and AB=24cm

Quantity I: Area of shaded region


Quantity II: 98 cm²
(a) Quantity I > Quantity II (b) Quantity I < Quantity II
(c) Quantity I ≥ Quantity II (d) Quantity I ≤ Quantity II
(e) Quantity I = Quantity II or No relation

16. Quantity I= ‘x’: Veer distributed Rs.80,000 among his wife, two
daughter and three sons. His wife got 300% more than the amount
every son got. If 30% of the total amount given to the sons then ‘x’ is the
amount every daughter got
Quantity II= ‘y’: Yogesh start a business with Rs 10800. After 4 months
veer join him with Rs 37800. After 2 months Amit join them with Rs
28800. After 1 year they got total Rs 54880 as profit. ‘y’ is the share of
Yogesh in profit.
(a) Quantity I > Quantity II (b) Quantity I < Quantity II
(c) Quantity I ≥ Quantity II (d) Quantity I ≤ Quantity II
(e) Quantity I = Quantity II or No relation

Adda247 Publications For any detail, mail us at


9
Publications@adda247.com
Cracker Book for Bank (IBPS | SBI | RRB PO | Clerk) Mains Exams

17. Quantity I: Profit share of ‘A’ out of total annual profit of Rs. 56,500. A,
B and C enter into a partnership. ‘A’ invests Rs. 4000 for the whole year,
‘B’ puts in Rs. 6000 at the first and increasing to Rs. 8000 at the end of 4
months, whilst C puts in at first Rs. 8000 but withdraw Rs. 2000 at the
end of 9 months.

Quantity II: Amount which when lend on C.I. at 20% interest being
compounded annually for 3 years, gives total interest equal to Rs.9100
(a) Quantity I > Quantity II
(b) Quantity I ≥ Quantity II
(c) Quantity I < Quantity II
(d) Quantity I = Quantity II or No relation
(e) Quantity I ≤ Quantity II

18. ‘A’, ‘B’ and ‘C’ together can complete a work in 48 days if they work
1
alternatively. ‘A’ is 25% more efficient than ‘B’ who is 33 % less
3
efficient than ‘C’.
Quantity I: Difference between days taken by ‘A’ alone and ‘C’ alone to
complete the work.
Quantity II: Days in which ‘A’, ‘B’ and ‘C’ together can complete half of
the work.
(a) Quantity I > Quantity II (b) Quantity I < Quantity II
(c) Quantity I ≥ Quantity II (d) Quantity I ≤ Quantity II
(e) Quantity I = Quantity II or No relation

19. X started from a point A towards point B. After 2 hours. Y started from
B towards A. By the time X travelled one-fifth of the total distance, Y had
also travelled the same. Y’s speed is thrice of that of X’s speed.
Quantity I: Difference in time (in hours) taken by X and Y to reach their
respective destinations.
Quantity II: 12 hours
(a) Quantity I = Quantity II or No relation
(b) Quantity I ≤ Quantity II
(c) Quantity I ≥ Quantity II
(d) Quantity I < Quantity II
(e) Quantity I > Quantity II
Adda247 Publications For any detail, mail us at
10
Publications@adda247.com
Cracker Book for Bank (IBPS | SBI | RRB PO | Clerk) Mains Exams
𝑋
20. 5X men can complete a work in days while 2Y men can complete same
2
4𝑌
work in days.
5
Quantity I: Value of ‘Y+20’.
Quantity II: Value of ‘1.25X'
(a) Quantity I ≥ Quantity II
(b) Quantity I = Quantity II or No relation
(c) Quantity I > Quantity II
(d) Quantity I < Quantity II
(e) Quantity I ≤ Quantity II

21. Total surface area of a cylinder is 200% more than that of its curved
surface area. Volume of cylinder is 2156 𝑐𝑚3
Quantity I: Volume of cone whose base radius and height is same as
that of radius and height of cylinder respectively.

Quantity II: Volume of hemisphere whose radius is same as that of


radius of cylinder.
(a) Quantity I > Quantity II
(b) Quantity I = Quantity II or No relation
(c) Quantity I ≥ Quantity II
(d) Quantity I < Quantity II
(e) Quantity I ≤ Quantity II

22. When two ships A & B are travelling in opposite direction crosses each
other in 2 second. Ship B is 50% faster than ship A in still water. On a
particular day, ship A was ahead of ship B by 9 meters and both of them
were travelling in downstream. if Ship B overtook Ship A in 11 seconds.
‘X’ is the speed of ship A and if length of ship B is 54 meters.
Quantity I: 16 m/s
Quantity II: Value of ‘X’ in m/s
(a) Quantity I = Quantity II (b) Quantity I ≥ Quantity II
(c) Quantity I ≤ Quantity II (d) Quantity I > Quantity II
(e) Quantity I < Quantity II
Adda247 Publications For any detail, mail us at
11
Publications@adda247.com
Cracker Book for Bank (IBPS | SBI | RRB PO | Clerk) Mains Exams

23. Quantity I= 𝒙

(o is the center of the circle)


Quantity II = 𝒚

(a) Quantity I ≥ Quantity II (b) Quantity I > Quantity II


(c) Quantity I ≤ Quantity II (d) Quantity I = Quantity II
(e) Quantity I < Quantity II

24. ‘B’ kg of Rs. 36 per/kg rice that mixed with 8 kg of Rs. 42 per/kg rice, in
order to earn profit of 10% while selling the mixture at Rs. 44 per/kg.
Quantity I: Value of ‘B+20’.
Quantity II: Value of ‘6B'

25. Quantity I –Vessel A contains (Q + 36) liter mixture of milk & water in
the ratio of 7 : 2, while vessel B contains (2Q + 42) liter mixture of milk
& water in the ratio of 2 : 3. If 40% & 46% of mixture from vessel A and
B taken out respectively, then remaining mixture in vessel B is 150% of
remaining mixture in vessel A. Find the total initial quantity of milk in
mixture of vessel A & vessel B together?

Adda247 Publications For any detail, mail us at


12
Publications@adda247.com
Cracker Book for Bank (IBPS | SBI | RRB PO | Clerk) Mains Exams

Quantity II – Two vessels contains mixture of mango juice and orange


juice in the ratio of 5 : 3 and 5 : 4 respectively. If 40 liter mixture from
first vessel taken out and mixed in second vessel, so new ratio of mango
juice and orange juice in second vessel becomes 25 : 19. Find initial
quantity of mixture in second vessel?
(a) Quantity I > Quantity II (b) Quantity I < Quantity II
(c) Quantity I ≥ Quantity II (d) Quantity I ≤ Quantity II
(e) Quantity I = Quantity II or no relation

26. Quantity I – Center of semicircle B is O1 and radius is 28 cm. find area


of shaded figure. BO1 is perpendicular to AC

Quantity II – Length of a rectangle is two times of side of square, while


ratio between side of square to breadth of rectangle is 8 : 7. If perimeter
of rectangle is 28 cm more than perimeter of square, then area of
rectangle.
(a) Quantity I > Quantity II (b) Quantity I < Quantity II
(c) Quantity I ≥ Quantity II (d) Quantity I ≤ Quantity II
(e) Quantity I = Quantity II or no relation

27. 5y² + 21y + 18 = 0 and


16(𝑥+2) ÷ 4(𝑥+3) = 64(𝑥+3) × 4(𝑥+4)
Quantity I: Value of y.
Quantity II: Value of x.
(a) Quantity I > Quantity II (b) Quantity I < Quantity II
(c) Quantity I ≥ Quantity II (d) Quantity I ≤ Quantity II
(e) Quantity I = Quantity II or no relation

Adda247 Publications For any detail, mail us at


13
Publications@adda247.com
Cracker Book for Bank (IBPS | SBI | RRB PO | Clerk) Mains Exams

28. Quantity I: Find the cost of B per kg if a product C is made of two


ingredients A and B in the proportion of 2 : 5. The price of A is three
times that of B. The overall cost of C is Rs. 5.20 per kg including labour
charges of 80 paisa per kg.

Quantity II: Find the cost of manufacturing each article if a


manufacturer estimates that on inspection 12% of the articles he
produces will be rejected. He accepts an order to supply 22,000 articles
at Rs. 7.50 each. He estimates the profit on his outlay including the
manufacturing of rejected articles, to be 20%.
(a) Quantity I > Quantity II (b) Quantity I < Quantity II
(c) Quantity I ≥ Quantity II (d) Quantity I ≤ Quantity II
(e) Quantity I = Quantity II or no relation

29. Quantity I: Value of x

Quantity II: Value of y


(20)3 +32 ×103 +25×50
=𝑦+1
54
(a) Quantity I > Quantity II (b) Quantity I < Quantity II
(c) Quantity I ≥ Quantity II (d) Quantity I ≤ Quantity II
(e) Quantity I = Quantity II or No relation

30. Quantity I: What is probability of getting two red balls when they are
drawn from a bag which contains 3 Red balls, 5 blue balls and 4 white
balls.
Quantity 2: Value of x.
1 2 1 1 3 11
5 +5 −5 = +5 −
2 7 14 𝑥 14 2
(a) Quantity I > Quantity II (b) Quantity I < Quantity II
(c) Quantity I ≥ Quantity II (d) Quantity I ≤ Quantity II
(e) Quantity I = Quantity II or No relation
Adda247 Publications For any detail, mail us at
14
Publications@adda247.com
Cracker Book for Bank (IBPS | SBI | RRB PO | Clerk) Mains Exams

31. Total surface area of cube and sphere are equal.


Quantity I: Volume of cube ÷ √𝜋
Quantity II: volume of sphere
(a) Quantity I > Quantity II (b) Quantity I < Quantity II
(c) Quantity I ≥ Quantity II (d) Quantity I ≤ Quantity II
(e) Quantity I = Quantity II or No relation

32. Figure I and II shows two semicircles of radius 14 cm with O and P as


their centres respectively.

Quantity I: Area of shaded region in figure I


Quantity II: Area of triangle in figure II
(a) Quantity I > Quantity II (b) Quantity I < Quantity II
(c) Quantity I ≥ Quantity II (d) Quantity I ≤ Quantity II
(e) Quantity I = Quantity II or No relation

33. Quantity I – A can complete a task in 24 days and B can do same task in
1
18 days. another two persons C & D complete 58 % of the same task in
3
7 days and efficiency of D is 40% more than that of C. Find in how many
days A, B and D will be complete the task together?

Quantity II – Ankit is 60% less efficient than Satish and complete a


piece of work in 22.5 days. Ankit and Saatish start work together and
after 4.5 days both left the work, if veer complete the remaining of
work in 4.5 days then find in how many days the whole work will be
completed, if all three work together ?
(a) Quantity I > Quantity II (b) Quantity I < Quantity II
(c) Quantity I ≥ Quantity II (d) Quantity I ≤ Quantity II
(e) Quantity I = Quantity II or no relation
Adda247 Publications For any detail, mail us at
15
Publications@adda247.com
Cracker Book for Bank (IBPS | SBI | RRB PO | Clerk) Mains Exams

34. Quantity I – A shopkeeper has two articles A & B. Marked price of


article B is 20% more than marked price of article A, shopkeeper sold
article A at 25% discount and article B at 20% discount. He made 20%
2
loss on article A and 6 % 𝑝𝑟𝑜𝑓𝑖𝑡 on article B. If total loss of shopkeeper
3
was Rs. 765, then find marked price of article B?

Quantity II – A shopkeeper gives a discount of 24% on marked price


of Shirt and cost price of Jeans is 25% more than selling price of Shirt. If
shopkeeper sold Jeans at 10 % profit and selling price of Jeans was Rs.
1140 more than selling price of Shirt , then find the cost price of article
Jeans ?
(a) Quantity I > Quantity II (b) Quantity I < Quantity II
(c) Quantity I ≥ Quantity II (d) Quantity I ≤ Quantity II
(e) Quantity I = Quantity II or no relation

35. Quantity I – A bag contains four green balls, three red balls and five
blue balls. If three balls taken out at random what is probability of at
least one ball is green and at least one ball is blue color.

Quantity II – There are five red toys and six green toys in a cartoon.
What will be the probability of selection of four toys which contains at
least two green toys?
(a) Quantity I > Quantity II (b) Quantity I < Quantity II
(c) Quantity I ≥ Quantity II (d) Quantity I ≤ Quantity II
(e) Quantity I = Quantity II or no relation

36. Quantity I – Six years ago ratio between age of Rohit and Prakash was
1 1
7 : 8, while six years hence ratio between 𝑡ℎ of Rohit age and 𝑟𝑑 of
6 3
Prakash age will be 9 : 20. The age of Rohit two years hence will be.

Quantity II – Ratio between age of A, B and C is 16 : 9 : 7. Three years


hence average of all three age will be 35 years. The age of A two years
hence will be?
(a) Quantity I > Quantity II (b) Quantity I < Quantity II
(c) Quantity I ≥ Quantity II (d) Quantity I ≤ Quantity II
(e) Quantity I = Quantity II or no relation
Adda247 Publications For any detail, mail us at
16
Publications@adda247.com
Cracker Book for Bank (IBPS | SBI | RRB PO | Clerk) Mains Exams

37. A diagram is given below. AC is the diameter of circle. AD || BC and


∠𝐴𝐶𝐷 = 90° ∠𝐴𝐶𝐵 ≤ 45°

Quantity I: -. ∠𝐴𝐷𝐶
Quantity II: - 45°
(a) Quantity I > Quantity II (b) Quantity I < Quantity II
(c) Quantity I ≥ Quantity II (d) Quantity I ≤ Quantity II
(e) Quantity I = Quantity II or No relation

38. Quantity I: ‘x’: There is an interview conducted by HCL. Out of total


applicants six are female. Probability of selecting two candidates such
7
that at least one is male is . ‘x’ is total number of males.
8

Quantity II: ‘y’ : A, B and C together start a business. Ratio between


investment of A and B is 15 : y while ratio between investment of B & C
is 9 : 11. A left business after 8 months of starting of business and B left
before 7 months of completion of year. Out of total profit of Rs. 2080, A
got Rs. 900.
(a) Quantity I > Quantity II (b) Quantity I < Quantity II
(c) Quantity I ≥ Quantity II (d) Quantity I ≤ Quantity II
(e) Quantity I = Quantity II or No relation

39. Quantity I: ‘x’: Two mutually perpendicular chords PQ and EF meet at a


point S inside a circle. PS = 12 cm, SQ = 8 cm and FS = 6 cm. ‘x’ is the
area of circle.
Quantity II: ‘y’: In a circle, two parallel chords are drawn on opposite
sides of diameter. Distance between these chords is 14cm. Length of
one chord is 16cm while length of another chord is 12cm. ‘y’ is the area
of circle

Adda247 Publications For any detail, mail us at


17
Publications@adda247.com
Cracker Book for Bank (IBPS | SBI | RRB PO | Clerk) Mains Exams

(a) Quantity I > Quantity II (b) Quantity I < Quantity II


(c) Quantity I ≥ Quantity II (d) Quantity I ≤ Quantity II
(e) Quantity I = Quantity II or No relation

40. Quantity I: ‘x’ :A dishonest shopkeeper sales his good by weighting


15% less and cheat his wholesaler by giving 15% less amount. ‘x’ is his
approximate total profit % if he sells it at cost price.
Quantity II: ‘y’ : A dishonest milkman, add 20 litre water in pure milk,
after selling half of mixture. He again added 20 liter of water in
remaining mixture now milk and water ratio in mixture becomes 4 : 3.
‘y’ is his overall profit % on selling total mixture.
(a) Quantity I > Quantity II
(b) Quantity I < Quantity II
(c) Quantity I = Quantity II or No relation
(d) Quantity I ≥ Quantity II
(e) Quantity I ≤ Quantity II

41. Quantity I: ‘x’ : A and B can do a work in 24 days together, in which


40% of work is done by B. C who is 25% more efficient than A, do the
same work alone in ‘x’ days.

Quantity II: ‘y’ : P, Q and S work for x, x+1 and x+2 days and their
efficiency are in the ratio of x : x+1 : x+2 respectively. If P got 16 Rs for
his work, then ‘y’ is the total money earned by all three in Rs..
(Compare Magnitude of x and y)
(a) Quantity I > Quantity II
(b) Quantity I ≥ Quantity II
(c) Quantity I = Quantity II or No relation
(d) Quantity I < Quantity II
(e) Quantity I ≤ Quantity II

42. Quantity I:’x’ -. Two circles are concentric with center ‘O’. Their radii
are 8cm and 10cm respectively. ‘B’ and ‘C’ are the points of contact of
two tangents drawn from bigger circle to smaller circle from point ‘A’
lying on bigger circle. ‘x’ is area of quadrilateral ABOC formed in 𝑐𝑚2

Adda247 Publications For any detail, mail us at


18
Publications@adda247.com
Cracker Book for Bank (IBPS | SBI | RRB PO | Clerk) Mains Exams

Quantity II: - ‘y’ - The lengths of two parallel sides of a trapezium are 6
cm and 8 cm. If the height of the trapezium be 6 cm, then its area is ‘y’
𝑐𝑚2
(a) Quantity I > Quantity II (b) Quantity I < Quantity II
(c) Quantity I ≥ Quantity II (d) Quantity I ≤ Quantity II
(e) Quantity I = Quantity II or No relation

43. Quantity I: ‘x’ -. Kishan invested Rs. 20,000 in a bank offering 22% p.a.
at Simple interest. After 2 years he withdraws his money and invested
in another bank which is offering ‘x%’ p.a. at compound interest. After 3
years, interest earned by him is Rs.1350 less than amount invested by
him in this bank.
5
Quantity II: - ‘y’ – Bhavya sell a diary at Marked price and earns 85 %
7
profit while if he gives 'y%’ discount on Marked price then he will earn
‘y%’ profit.
(a) Quantity I > Quantity II (b) Quantity I < Quantity II
(c) Quantity I ≥ Quantity II (d) Quantity I ≤ Quantity II
(e) Quantity I = Quantity II or No relation

44. Quantity I —‘x’ : B alone can do the work in ‘x’ days. A can complete a
work in 5 more days than B while A does the same work in 9 more days
than C. If A and B can complete the whole work in same time as time
taken by C alone to do the whole work.

Quantity II —‘y’ : ‘y’ is the days taken by 8 men and 14 women to reap
7
𝑝𝑎𝑟𝑡 of 360-hectare land by working 7 hrs per day if 6 men and 10
12
5
women can reap part of the land in 15 days by working 6 hrs per day.
12
It is also given that work of 2 men is equal to that of 3 women.
(a) Quantity I > Quantity II (b) Quantity I < Quantity II
(c) Quantity I ≥ Quantity II (d) Quantity I ≤ Quantity II
(e) Quantity I = Quantity II or No relation
Adda247 Publications For any detail, mail us at
19
Publications@adda247.com
Cracker Book for Bank (IBPS | SBI | RRB PO | Clerk) Mains Exams

45. Quantity I — ‘x’ : ‘x’ is the difference between the speeds of P and Q.
Distance between A and B are 60 km. P and Q start from A at same time
& meet 1st time at a place 12 km from B. They return to A immediately
after reaching B. The speed of slower person is 48 km/hr.

Quantity II —‘y’ : ‘y’ is the average speed of train if a distance of 600


km is to be covered in 2 parts. In 1st phase 120 km is travelled by train
and rest by car and it took total of 8 hrs, but if 200 km is covered by
train and rest by car it takes 20 min more.
(a) Quantity I > Quantity II (b) Quantity I < Quantity II
(c) Quantity I ≥ Quantity II (d) Quantity I ≤ Quantity II
(e) Quantity I = Quantity II or No relation

46. Quantity I: ‘x’ : ‘𝒙′ is the number of days taken by A alone to complete a
work. A and B together can complete the same work in 16 days while A
1
and C can complete the same work in 15 days. Ratio of efficiency of B
3
and C is 3 : 4.
Quantity II: ′𝒚′: P, Q and R takes ‘y’ days to complete a work. Ratio of
efficiency of P, Q and R are 2 : 3 : 4 respectively and P and Q together
takes 18 days to complete that work.
(a) Quantity I > Quantity II (b) Quantity I < Quantity II
(c) Quantity I ≥ Quantity II (d) Quantity I ≤ Quantity II
(e) Quantity I = Quantity II or No relation

47. Quantity I: ‘x’ : A article is sold at 100 Rs. profit whose cost price is
‘Rs.𝒙’. If article is sold at 20% more than its S.P. then S.P becomes 2𝑥.

Quantity II: ′𝒚′ : Article is sold at y% profit whose M.P. is 400% above
its C.P. and discount given on M.P. is 60%.
(a) Quantity I > Quantity II (b) Quantity I < Quantity II
(c) Quantity I ≥ Quantity II (d) Quantity I ≤ Quantity II
(e) Quantity I = Quantity II or No relation
Adda247 Publications For any detail, mail us at
20
Publications@adda247.com
Cracker Book for Bank (IBPS | SBI | RRB PO | Clerk) Mains Exams

48. Quantity I: ‘x’: Sum of ‘𝒙′ consecutive number is 4040. First number is
11.
Quantity II: ′𝒚′: Sum of ′𝒚′ consecutive even numbers is 7120. First
number is 10.
(a) Quantity I > Quantity II (b) Quantity I < Quantity II
(c) Quantity I ≥ Quantity II (d) Quantity I ≤ Quantity II
(e) Quantity I = Quantity II or No relation

49. Quantity I: ‘x’: A man invested 𝑹𝒔. ′𝒙′ in two scheme which offers 40%
S.I. for 2 years and 44% S.I. for 5 years respectively. Difference between
interest earned from both scheme is 18,900.

Quantity II: ′𝒚′: Interest earned in a scheme is ′𝑦′ which offers 12%
rate of interest for 3 years on a sum. 30% of the sum is 12000.
(a) Quantity I > Quantity II (b) Quantity I < Quantity II
(c) Quantity I ≥ Quantity II (d) Quantity I ≤ Quantity II
(e) Quantity I = Quantity II or No relation

50. The largest sphere that can fit in a cube of edge 7 cm.
Quantity I: ‘x’: ‘x’ is the volume of vacant space left over after putting
the sphere.(𝑖𝑛 𝑐𝑚3 )
Quantity II: ′𝒚′: ’y’ is the surface area of the sphere. (in 𝑐𝑚2 )
(a) Quantity I > Quantity II (b) Quantity I < Quantity II
(c) Quantity I ≥ Quantity II (d) Quantity I ≤ Quantity II
(e) Quantity I = Quantity II or No relation

PART B
Directions (Q1-5): In each of these questions, two equations I. and II. are
given. You have to solve both the equations and give answer
1. I. 3𝑥+5 . 92𝑥−4 = 95𝑥−14
II. 2𝑦 2 − 15𝑦 − 28 = 3𝑦 2 − 23𝑦 − 13
(a) x < y
(b) x ≥ y
(c) No relation can be established between x and y.
(d) x ≤ y
(e) x > y

Adda247 Publications For any detail, mail us at


21
Publications@adda247.com
Cracker Book for Bank (IBPS | SBI | RRB PO | Clerk) Mains Exams
25 15
2. I. − +2=0
x² x
40 13
II. + 1=
y² y
(a) x < y
(b) x ≥ y
(c) No relation can be established between x and y.
(d) x ≤ y
(e) x > y

3. I. 2(y+4) . 12(2y+4) = 9(y+2) . 16(y+4)


II. x 2 – 10x + 24 = 0
(a) x ≤ y
(b) x > y
(c) No relation can be established between x and y.
(d) x < y
(e) x ≥ y

48 14
4. I. − +1=0
x2 x
45 1
II. + =2
y² y

(a) No relation can be established between x and y.


(b) x ≤ y
(c) x < y
(d) x > y
(e) x ≥ y

5. I. (𝑥 − 4)2 = 9
II. (2𝑦 + 3)2 = 25
(a) No relation can be established between x and y.
(b) x < y
(c) x ≤ y
(d) x ≥ y
(e) x > y

Adda247 Publications For any detail, mail us at


22
Publications@adda247.com
Cracker Book for Bank (IBPS | SBI | RRB PO | Clerk) Mains Exams

Directions (6-10): In each of these questions, two equations I. and II. are
given. You have to solve both the equations and give answer
(a) if x>y
(b) if x≥y
(c) if x<y
(d) if x ≤y
(e) if x = y or no relation can be established between x and y.

6. I. 5x² + 3x – 36 = 0 II. 2y² – 13y + 20 = 0

7. I. x² – 7x + 12 = 0 II. 2y² – 11y + 15 = 0

8. I. 2x² + 11x + 15 = 0 II. 2y² + 9y + 10 =0

9. I. 3x² + 7x – 40 = 0 II. 5y² – 29y + 42 = 0

10. I. 3x² – 23x + 42 = 0 II. 2y² – 19y + 45 = 0

Directions (11-15): In each of these questions, two equations I. and II. are
given. You have to solve both the equations and give answer
(a) if x>y
(b) if x≥y
(c) if x<y
(d) if x ≤y
(e) if x = y or no relation can be established between x and y.

11. I. 𝑥 2 − 5𝑥 + 4 = 0 II. 𝑦 2 + 5𝑦 + 6 = 0

12. I. 2𝑥 2 − 𝑥 − 15 = 0 II. 3𝑦 2 − 23𝑦 + 42 = 0

13. I. 𝑥 2 − 15𝑥 + 54 = 0 II. 𝑦 2 + 15𝑦 − 54 = 0

14. I. 𝑥 2 + 14𝑥 + 40 = 0 II. 𝑦 2 − 5𝑦 − 24 = 0

15. I. 𝑥 2 − 225 = 0 II. 𝑥 2 + 𝑦 2 = 306

Adda247 Publications For any detail, mail us at


23
Publications@adda247.com
Cracker Book for Bank (IBPS | SBI | RRB PO | Clerk) Mains Exams

Directions (16-20): In each of these questions, two equations I. and II. are
given. You have to solve both the equations and give answer
(a) if x>y
(b) if x≥y
(c) if x<y
(d) if x ≤y
(e) if x = y or no relation can be established between x and y.

16. I. 2x² – 7x + 6 = 0 II. 3y² – 19y + 28 = 0

17. I. x² – 13x + 36 =0 II. 3y² – 19y + 30 = 0

25
18. I. x³– 120 = 96 II. y– =0
𝑦

19. I. x² – 2x – 48 = 0 II. y² – 15y + 54 = 0

20. I. 2x² + 25x + 72 = 0 II. 3y² + 22y + 40 = 0

Adda247 Publications For any detail, mail us at


24
Publications@adda247.com
Cracker Book for Bank (IBPS | SBI | RRB PO | Clerk) Mains Exams

Solutions

1. (a); Suppose B takes ‘x’ days,


Then C will take ‘3x’ days
1 1 1
+ =
3𝑥 𝑥 22.5
Or, x = 30
Now, Let A and B together take ‘2y’ days,
Then A will take ‘3y’ days,
1 1 1
+ =
3𝑦 30 2𝑦
Or, y = 5
Hence, A will take 3y = 3 × 5 = 15 days.
Quantity I = 15
90
Quantity II = = 90/7
6+1
Quantity I > Quantity II

117×3+2
2. (b); Quantity I→ × 600 = 706 Rs
300
25𝑥
Quantity II→ 𝑥 + = 1000
100
100
𝑥= × 1000 = 800 Rs
125
∴ Quantity I < Quantity II

3. (b); Quantity I: Volume of cube left


3 7 2
=7 −𝜋( ) ×7
2
22 49×7
= 343 − ×
7 4
= 343 − 269.5
= 73.5 cm3
Quantity II: Surface area of cube left
2 7 2 7
= 6 × 7 − 2. 𝜋 ( ) + 2𝜋 ( ) 7
2 2
= 294 − 77 + 154
= 371 cm2
Quantity II > Quantity I

Adda247 Publications For any detail, mail us at


25
Publications@adda247.com
Cracker Book for Bank (IBPS | SBI | RRB PO | Clerk) Mains Exams

4. (a); Quantity I – 2πr + 2 (ℓ + b) = 220 cm


πr² = 1386 sq. cm
1386×7
r2 =
22
r = 21 cm
Length of rectangle
4
= 21 × = 28 cm
3
22
2× × 21 + 2(28 + b) = 220
7
132 + 56 + 2b = 220
32
b= = 16 cm
2
Area of rectangle = (28 × 16) = 448 cm

Quantity II–2πr = 132


132×7
r=
2×22
r = 21 cm
πr² – a² = 710
22
× 21 × 21 – a2 = 710
7
a² = 1386 – 710
a² = 676 cm
a = 26 cm
Perimeter of square = 4 × 26 = 104 cm
Quantity I > Quantity II

5. (b); Quantity I— Let Veer take 3𝑥 days and Sameer take 𝑥 days.
Efficiency of Veer and Sameer be 𝑥 unit/day and 3𝑥 units/day
respectively
Total work = 3𝑥 × 18 + 𝑥 × 18 = 72x units
If both do with double efficiency
Then,
72𝑥
= (3𝑥×2+2𝑥) = 9 days

Adda247 Publications For any detail, mail us at


26
Publications@adda247.com
Cracker Book for Bank (IBPS | SBI | RRB PO | Clerk) Mains Exams

Quantity II—

When all three opened together


Total work in one day = (6 + 2 – 3) = 5 units
48 3
Required time = = 9 days
5 5
Quantity I < Quantity II

6. (a); Quantity I -

150
R efficiency = 3 × = 4.5 w⁄d
100
3 days work = (P + Q + R)
= 2 + 3 + 4.5 = 9.5 work
Required days = 9.5 × 15 = 142.5 work
Remaining work = 144 – 142.5 = 1.5
1.5 3
Next day by P = =
2 4
3 3
Total days = 15 × 3 + = 45 days
4 4

Quantity II. - A : B = 100 : 75 = 4 : 3


Total work = 4 × 48 = 192
192
B alone = = 64 days
3
C alone = 64 – 40 = 24 days
192
C efficiency = = 8 w⁄d
24
192 192 4
(A + B + C)together = = = 12 days
(4+3+8) 15 5
So, Quantity I > Quantity II

Adda247 Publications For any detail, mail us at


27
Publications@adda247.com
Cracker Book for Bank (IBPS | SBI | RRB PO | Clerk) Mains Exams

7. (b); Quantity I - No. of selection of 3 mango out of one dozen mangoes


= ¹²C₃ = 220
No. of selection of 2 bad mango out of 4 bad mangoes = ⁴C2 = 6
No. of selection of a good mango from 8 good mangoes = 8
8×6 12
Required probability = =
220 55
Quantity II - Face card = 12
Joker = 2
14 7
Probability = =
54 27
So, Quantity I < Quantity II

8. (e); Quantity I - Let length of train be L meter


ATQ—
5
Speed = 72 × = 20 m/sec
18
L
20 =
14
L = 280 meter
Let length of platform be P meter
P+280
20 =
32
P = 640 – 280
P = 360 meter

𝐐𝐮𝐚𝐧𝐭𝐢𝐭𝐲 𝐈𝐈 – Let’s length of train is X meter and length of bridge


and platform are 2L and L meter respectively
5
Speed of train = 108 × = 30 m⁄s
18
When train passed the bridge
Distance covered = X + 2L
X + 2L = 540 …(i)
When train passes the platform
Distance covered = X + L
X + L = 450 …(ii)
From (i) and (ii)
L = 90 meter
X = 360 meter
So, Quantity I = Quantity II
Adda247 Publications For any detail, mail us at
28
Publications@adda247.com
Cracker Book for Bank (IBPS | SBI | RRB PO | Clerk) Mains Exams

9. (b);

Let P is faster than Q


Then P covers 72 km distance in the same time as Q covers 48 km
distance
Ratio of the speed = 72 : 48 = 3 : 2
48
∴ Speed of faster train i.e., P = × 3 = 72 km/hr
2

Quantity 1→ Difference between P and Q


= 72 – 48 = 24 km/hr.
Let speed of train = T km/hr
Let speed of car = C km/hr
120 480
∴ + = 8 ………..(i)
𝑇 𝐶
200 400 1
+ = 8 ………(ii)
𝑇 𝐶 3
On solving (i) and (ii)
T = 60 km/hr
∴ Quantity I < Quantity II

6
10. (b); Quantity I → = 3
2
1
And, 𝜋𝑟 2 ℎ = 16𝜋
3
ℎ=3
∴ 𝑦 = √42 + 32 = √16 + 9
𝑦=5
∴ Quantity I < Quantity II

25 48 55
11. (a); Quantity I— 𝐴 × × × = 1188
100 100 100
1188×100×100×100
A=
25×48×55
A = 18000
21 35
𝐵× × = (32)2 + 5
100 100
21 25
𝐵× × = 1024 + 5
100 100

Adda247 Publications For any detail, mail us at


29
Publications@adda247.com
Cracker Book for Bank (IBPS | SBI | RRB PO | Clerk) Mains Exams
1029×100×100
𝐵=
21×25
B = 19600
(A + B) = 18000 + 19600 = 37600

Quantity II— Let male and female population of town in 2016 be


8x and 9x respectively
125 140
8𝑥 × + 9𝑥 × = 45200
100 100
63𝑥
10𝑥 + = 45200
5
113𝑥 = 226000
𝑥 = 2000
Total population of town in 2016 = 16000 + 18000
= 34000
Quantity I > Quantity II

12. (a); Quantity-I : Probability of choosing Mango


1 5 1 4 31
= × + × =
2 12 2 9 72
Quantity-II : Total no. of cases to choose 3 egg = ²⁴C₃
Favourable cases = ¹⁶C₂ × ⁸C₁
16C × 8C 15
2 1
Probability = 24C =
3 253
Quantity I > Quantity II

13. (e); A can complete work in= 12 days


3 1 9−5 4
‘B’ 1 day work is = − = =
20 12 60 60
60
B can complete the work in = = 15 days
4
C can complete the work in ‘20’ days
20×150
D can complete the work in = 30 days
100

Quantity I: ‘A + B + C three days’ work


1 1 1 5+4+3 12 1
= + + = = =
12 15 20 60 60 5
‘A + B + C’ can complete the work in total ’15 days
Adda247 Publications For any detail, mail us at
30
Publications@adda247.com
Cracker Book for Bank (IBPS | SBI | RRB PO | Clerk) Mains Exams

Quantity II: Remaining work for D


6 6 60−24−18 3
=1− − = =
15 20 60 10
3 30×3
of work is completed by D in
10 10
= 9days
Total time = 6 + 9 = 15 days
Quantity I = Quantity II

14. (a); Quantity I: let C.P = 100


Mark Up price = 140
80
Selling price = 140 × = 112
100
ATQ,
12 → 168
x = 40 → 560
x = 560

Quantity II: Interest from scheme A


12000×12×4
= = 5760
100
Interest from scheme B
25000×11×2
= = 5500
100
y= 5760 – 5500 = 260
x>y
Quantity I > Quantity II
15. (b); Quantity I:
OF=√202 − 12²=√400 − 144 = √256 = 16
Area of shaded region
Area of ∆AFE – Area of ∆ AFO
1 1
= × 12 × 32 − × 12 × 16
2 2
= 192 – 96 = 96 cm²
Quantity II > Quantity I

Adda247 Publications For any detail, mail us at


31
Publications@adda247.com
Cracker Book for Bank (IBPS | SBI | RRB PO | Clerk) Mains Exams

16. (a); Quantity I: Total amount sons got


30
= × 80000 = 24000
100
24000
Each son got = = 8000
3
400
Wife got = 8000 × = 32000
100
Total amount daughters got
80,000 – 24000 – 32000 = 24000
24000
Each daughter got = = 12000
2
x = 12,000
Quantity II: Ratio of investment or profit
= 10800 × 12 : 37800 × 8 : 28800 × 6
=3:7:4
54880
Share of Yogesh in profit = ×3
14
y = 11760
x>y
Quantity I > Quantity II

17. (c); ATQ,


Quantity I: Ratio of profit share
A : B : C
(4000×12) (6000×4)+(8000×8) (8000×9)+(6000×3)
24 : 44 : 45
24
Share of A = × 56,500 = Rs. 12,000
113
Quantity II:
20 3
9100 + P = P (1 + )
100
⇒ 9100 + P = 1.728P
or, P = Rs. 12,500
𝐐𝐮𝐚𝐧𝐭𝐢𝐭𝐲 𝐈𝐈 > 𝐐𝐮𝐚𝐧𝐭𝐢𝐭𝐲 𝐈
Adda247 Publications For any detail, mail us at
32
Publications@adda247.com
Cracker Book for Bank (IBPS | SBI | RRB PO | Clerk) Mains Exams

18. (e); ‘A’, ’B’ and ‘C’ alternatively can complete a work in 48 days
⇒ ‘A’, ‘B’ and ‘C’ together can complete same work in 16 days
Ratio between efficiency of A, B and C is 5 : 4 : 6
Ratio between days taken by A, B and C alone to complete the
same work is 12 : 15 : 10
Let A, B and C alone can complete work in 12x, 15x and 10x days
respectively.
1 1 1 1
⇒ + + = ⇒𝑥=4
12𝑥 15𝑥 10𝑥 16
Quantity I: Required difference
= (12 − 10)4 = 8 𝑑𝑎𝑦𝑠
Quantity II: A, B and C together can complete same work in 16
days, so they can complete half work in 8 days.
Quantity I = Quantity II

19. (d); Let the speed of X be x kmph. Distance travelled by X in 2 hours =


2x km.
1th
Suppose X takes ‘t’ hours to travel of the distance AB.
5
1
Y would take (t–2) hours to travel th of the distance AB.
5
As Y’s speed is thrice that of X’s speed.
𝑡−2 1
= ⇒ t=3
𝑡 3
1𝑡ℎ
of the distance AB = 3x km.
5
AB =15x km
15𝑥
Time taken by x to cover 15x km = = 15 hours
𝑥
15𝑥
Time taken by Y to cover 15x km =
3𝑥
= 5 ℎ𝑜𝑢𝑟𝑠.
∴ Difference in the time = 10 hours.
Quantity I : Difference in the time = 10 hours.
Quantity II : 12 hours
Quantity I < Quantity II
Adda247 Publications For any detail, mail us at
33
Publications@adda247.com
Cracker Book for Bank (IBPS | SBI | RRB PO | Clerk) Mains Exams
𝑋 4𝑌
20. (c); Total work = 5𝑋 × = 2𝑌 ×
2 5
𝑋2 16 𝑋 4
⇒ = ⇒ =
𝑌2 25 𝑌 5
Let X = 4a and Y = 5a
Quantity I: Y+20 = 5a+20
Quantity II: 1.25X = 5a
Quantity I > Quantity II

21. (b); T.S.A of cylinder = 2𝜋𝑟(𝑟 + ℎ)


C.S.A of cylinder = 2𝜋𝑟 2
ATQ,
3 2𝜋𝑟(𝑟+ℎ)
=
1 2𝜋𝑟 2
⇒ ℎ = 2𝑟
𝑉𝑜𝑙𝑢𝑚𝑒 𝑜𝑓 𝑐𝑦𝑙𝑖𝑛𝑑𝑒𝑟 = 𝜋𝑟 2 ℎ = 2156
⇒ 𝑟 = 7𝑐𝑚, ℎ = 14𝑐𝑚
1 2156
Quantity I: Volume of cone = 𝜋𝑟 2 ℎ =
3 3
Quantity II: Volume of hemisphere
2 2156
= 𝜋𝑟 3 =
3 3
Quantity I = Quantity II

22. (e); Let speed of ship A in still water is 2x m/s , and its length is ℓ m
And speed of ship B is 3x m/s and its length is 54 and speed of
water is Y m/s.
ATQ, when both of them are travelling in opposite direction.
Downstream speed of ship A = (2x + Y) m/s [assume ship A is
travelling in downstream and ship B in upstream]
Upstream speed of ship B = (3x – Y) m/s
Their relative speed = 2x + Y + 3x – Y = 5x m/s
ATQ,
54+ℓ
=2 …(i)
5x
Similarly when both are travelling in downstream their relative
speed is
= 3x + Y – 2x –Y= X m/s
Adda247 Publications For any detail, mail us at
34
Publications@adda247.com
Cracker Book for Bank (IBPS | SBI | RRB PO | Clerk) Mains Exams

ATQ,
54+9+ℓ
= 11
𝑥
63 + ℓ = 11x …(ii)
From (i)
ℓ = 10x – 54
put this value in eqn. (ii)
= 63 + 10x – 54 = 11x
x = 9 m/s
Hence speed of ship A in still water = 2 × 9 = 18 m/s
Quantity I = 16 m/s
Quantity II = 18 m/s
Quantity II > Quantity I

23. (b); 𝑥 = 180 − (90 + 30) = 90 − 30 = 60°


𝑦 = 180 − (60 + 40) (angles subtended by same arc in the same
segment are equal)
= 80°
∴ Quantity I < Quantity II

24. (e); Selling mixture at Rs. 44/kg with 10% profit means, the actual
price of mixture is Rs. 40/kg
Let B kg of Rs. 36/kg are mixed
Then
36𝐵+8×42
= 40
𝐵+8
36B + 336 = 40B + 320
4B = 16
B=4
Quantity I = B + 20 = 24
Quantity I = 𝟔 × 𝑩 = 𝟐𝟒
Quantity I = Quantity II
Adda247 Publications For any detail, mail us at
35
Publications@adda247.com
Cracker Book for Bank (IBPS | SBI | RRB PO | Clerk) Mains Exams

25. (b); Quantity I –


ATQ –
60 150 54
(Q + 36) × × = [(2Q + 42) × ]
100 100 100
3(60Q + 2160) = 2 (108Q + 2268)
180Q + 6480 = 216Q +4536
36Q = 1944
Q = 54 l
Total initial quantity of milk in mixture of vessel A & vessel B
7 2
= (54 + 36) × + (2 × 54 + 42) × = 70 + 60 = 130 l
9 5

Quantity II –
ATQ –
Let mixture of mango juice and orange juice in second vessel be 5y
liter and 4y respectively
5
5𝑦+40×8 25 5𝑦+25 25
3 = ⇒ =
4𝑦×40× 19 4𝑦+15 19
8
19y + 95 = 20y + 75
y = 20 l
Initial quantity of mixture in second vessel = 20 × 9 = 180 𝑙
Quantity I < Quantity II

26. (e); Quantity I–Area of semicircle – area of triangle


22 28×28 1
= × − × 56 × 28 = 1232 – 784 = 448 cm²
7 2 2

Quantity II – Ratio of Length, breadth of a rectangle & side of


square = 16 : 7 : 8
Let Length, breadth of a rectangle & side of square be 16x cm, 7x
cm & 8x cm respectively
ATQ –
2(16x + 7x) – 4 × 8𝑥 = 28 𝑐𝑚
46x – 32𝑥 = 28
x = 2 cm
Area of rectangle = (16× 2 ) × (2 × 7) = 448 cm2
Quantity I = Quantity II
Adda247 Publications For any detail, mail us at
36
Publications@adda247.com
Cracker Book for Bank (IBPS | SBI | RRB PO | Clerk) Mains Exams

27. (c); Quantity I: 5y² + 21y + 18 = 0


5y² + 5y + 15y + 18 = 0
Y (5y + 6) +3 (5y + 6) = 0
(y + 3) (5y + 6) = 0
–6
y = –3 or
3
Quantity II: 16(x+2)÷4(x+3) = 64(x+3)×4(x+1)
24x+8–2x–6 = 26x+18+2x+2
8x + 20 = 2x + 2
∴ x = –3.
∴ Quantity I ≥ Quantity II

28. (b); Quantity I: Let the price of B per kg be Rs. X. Then, the price of A
per kg = Rs. 3X
1kg of C contains 2/7 kg of A and 5/7 kg of B
Price of 1 kg of C = (2/7) × 3X + (5/7)X
= (11/7)X
By the given condition, 11X/7
= 5.20 – 0.80 = Rs. 4.40
⇒ X = 4.40 × (7/11) = Rs. 2.80
Hence the price of B per kg = Rs. 2.80.
Quantity II: If 100 articles are manufactured then 12 will be
rejected
Total selling price of 88 articles = 88 × 75
660×100
Total cost price = = 550
120
cost of manufacturing per article = 5.5 Rs
Quantity II > Quantity I

29. (b); Quantity I: ∠CAD = 90° – ∠ACD = 62


(x) = ∠CBD = 90° – ∠CAD
x = 28°
8000+9000+1250
Quantity II: =𝑦+1
625
y = 28.2
Quantity II > Quantity I
Adda247 Publications For any detail, mail us at
37
Publications@adda247.com
Cracker Book for Bank (IBPS | SBI | RRB PO | Clerk) Mains Exams
3C 1
30. (b); Quantity I: 12 2 =
C2 22
1 1 2 1 3 1
Quantity II: 5 ( + + − − )=
2 2 7 14 14 𝑥
1 1
5= ⇒𝑥 =
𝑥 5
Quantity II > Quantity I

31. (b); Let side of cube and radius of sphere be a and r respectively.
⇒ 6a² = 4πr²
2
a=√ π r
3

Quantity I: Volume of cube


=a×a×a
2 2
= π × √ π r 3 ÷ √𝜋
3 3
3
=(2/3 ) π r 3 2

4
Quantity II : volume of sphere πr 3
3
∴ Quantity II > Quantity I

32. (a); Quantity I: Area of shaded region


Area of circle – Area of triangle
Base of triangle = 2 × 14 = 28
22 14×14 1
= × – × 28 × 14
7 2 2
2
⇒ 112 cm

Quantity II: Radius = 14


1
Area of triangle = × 14 × 14 = 98 cm²
2
Quantity I > Quantity II

Adda247 Publications For any detail, mail us at


38
Publications@adda247.com
Cracker Book for Bank (IBPS | SBI | RRB PO | Clerk) Mains Exams

33. (a); Quantity I – Total work = 72 units (Lcm of days taken by A & B)
72
Efficiency of A = = 3 𝑢𝑛𝑖𝑡𝑠/𝑑𝑎𝑦
24
72
Efficiency of B = = 4 𝑢𝑛𝑖𝑡𝑠/𝑑𝑎𝑦
18
Let efficiency of C = x unit/day
175 1
7x + 7(1.4x) = 72 × = 42 𝑢𝑛𝑖𝑡𝑠
3 100
x = 2.5 units
Efficiency of D = 1.4 × 2.5 = 3.5 𝑢𝑛𝑖𝑡𝑠
72 6
(A + B + C) together = = 6 𝑑𝑎𝑦𝑠
(3+4+3.5) 7
Quantity II – Ratio of efficiency of Satish :
Ankit = 100x : 40x = 5x : 2x
Total work = 22.5 × 2x = 45x units
ATQ,
(Ankit + Satish) × 4.5 = 4.5 (5x + 2x) = 31.5x work
Remaining work = 45x– 31.5 x = 13.5x
13.5
Veer efficiency = = 3 units⁄day
4.5
45x
(Ankit + Satish + Veer) =
(5x+2x+3x)
= 4.5 days
So, Quantity I > Quantity II

34. (a); Quantity I – Let marked price of article


A = 100x
Marked price of article B = 120x
3
S. P. of article A = 100x × = 75x
4
4
S. P. of article B = 120 × = 96x
5
5
C. P. of article A = 75 × = 93.75x
4
15
C. P. of article B = 96x × = 90x
16
ATQ—
(93.75x + 90x) – (75x + 96x) = 765
183.75x – 171x = 765
12.75x = 765
x = 60
M.P. of article B = 60 × 120 = 7200 Rs.
Adda247 Publications For any detail, mail us at
39
Publications@adda247.com
Cracker Book for Bank (IBPS | SBI | RRB PO | Clerk) Mains Exams

Quantity II – MP of shirt = 100x Rs


76
SP of shirt = 100 × = 76𝑥 Rs
100
5
Cost price of jeans = 76𝑥 × = 95𝑥 Rs
4
110
Selling price of jeans = 95𝑥 ×
100
= 104.5x Rs
ATQ –
104.5x - 76x = 1140 Rs
28.5x = 1140 Rs.
X = 40 Rs.
Cost price of jeans = 95×40 = 3800 Rs.
So, Quantity I > Quantity II

35. (b); Quantity I – Required cases = (1 green, 1 red, I blue) or ( 2 green ,


1 blue) or (1 green , 2 blue)
4C × 3C × 5C 4C × 5C 4C × 5C 3 3 2 13
1 1 1 2 1 1 2
= 12C + 12C + 12C = + + =
3 3 3 11 22 11 22

Quantity II – Favorable case = (2G, 2R) or (3G, 1R) or 4G


∴ Probability
6C × 5C 6 C × 5C 6C
2 2 3 1 4
= 11C + 11C + 11C
4 4 4
15×10 20×5 15 265 53
= + + = =
330 330 330 330 66
So, Quantity I < Quantity II

36. (e); Quantity I – Let age of Rohit and Prakash six years ago be 7x years
and 8x years respectively
ATQ –
𝟕𝒙+𝟏𝟐
𝟔 𝟗
𝟖𝒙+𝟏𝟐 =
𝟐𝟎
𝟑
140x + 240 = 144x + 216
x = 6 years
Age of Rohit two years hence will be
= (7× 6 + 8) = 50 𝑦𝑒𝑎𝑟𝑠
Adda247 Publications For any detail, mail us at
40
Publications@adda247.com
Cracker Book for Bank (IBPS | SBI | RRB PO | Clerk) Mains Exams

Quantity II – Let age of A, B and C be 16y, 9y and 7y respectively.


ATQ –
(16y + 9y + 7y) = 35 × 3 − 3 × 3
32y = 96
y = 3 years
Age of A two years hence = 3 × 16 + 2 = 50 years
So, Quantity I = Quantity II

37. (c); ∠𝐴𝐵𝐶 = 90° (Semicircle property)


∠𝐶𝐴𝐵 + ∠𝐴𝐶𝐵 + ∠𝐴𝐵𝐶 = 180°
∠𝐶𝐴𝐵 + ∠𝐴𝐶𝐵 = 90°
As ∠𝐴𝐶𝐵 ≤ 45° So, ∠𝐶𝐴𝐵 ≥ 45°
∠𝐴𝐶𝐷 + ∠𝐶𝐴𝐷 + ∠𝐴𝐷𝐶 = 180°
∠𝐶𝐴𝐷 + ∠𝐴𝐷𝐶 = 90°
But ∠𝐴𝐶𝐵 = ∠𝐶𝐴𝐷 (AD || BC)
∠𝐴𝐶𝐵 + ∠𝐴𝐷𝐶 = 90°
As ∠𝐴𝐶𝐵 ≤ 45° So, ∠𝐴𝐷𝐶 ≥ 45°
Quantity I ≥ Quantity II

38. (a); Quantity I: Let, no. of males = x


ATQ,
x 6 x(x–1) 7
2× × + (x+6)(x+5) =
x+6 x+5 8
12x+x2 –x 7
(x+6)(x+5)
=
8
88x + 8x² = 7 (x² + 5x + 6x + 30)
x² + 11x – 210 = 0
x² + 21x – 10x – 210 = 0
x (x + 21) – 10 (x + 21) = 0
x = 10, –21

Adda247 Publications For any detail, mail us at


41
Publications@adda247.com
Cracker Book for Bank (IBPS | SBI | RRB PO | Clerk) Mains Exams

Quantity II: Ratio between investment of A, B and C is 135 : 9y :


11y
Profit sharing of A, B and C is
135 × 8 : 9𝑦 × 5 : 11𝑦 × 12
= 360 : 15𝑦 : 44𝑦
ATQ,
900 360
=
2080 360+59𝑦
⇒ (360 + 59y) = 104 × 8
⇒ 59y = 472
⇒y=8
Quantity I > Quantity II

39. (a); Quantity I:

As, PQ and EF are two line that intersect at S.


So, PS × SQ = ES × SF
12 × 8 = ES × 6
ES = 16 cm
From center O draw
OB ⊥ PQ and OA ⊥ EF
(12+8)
So, PB = PQ = = 10 cm
2
BS = OA = 2 cm
EF = 16 + 6 = 22 cm
22
EA = = 11 cm
2
In triangle ∆𝑂𝐴𝐸
OA² + EA² = OE²
2² + 11² = OE²
OE² = 125
Adda247 Publications For any detail, mail us at
42
Publications@adda247.com
Cracker Book for Bank (IBPS | SBI | RRB PO | Clerk) Mains Exams

OE = √125 = 5√5 cm
2
Area of circle = πr²= 𝜋(5√5)
22
= 125 × = 392.86 cm ²
7

Quantity II:

Let AB and CD are the chord drawn on opposite sides of diameter.


Length of CD is 16 cm and length of AB is 12 cm.
ATQ
𝐶𝐷 16
FC = = = 8𝑐𝑚
2 2
𝐴𝐵 12
AE = = = 6𝑐𝑚
2 2
EF = 14 cm
But AO = OC = radius of circle
𝑂𝐴2 = 𝐴𝐸 2 + 𝑂𝐸 2 = 𝑂𝐶 2 = 𝐶𝐹 2 + 𝑂𝐹 2
Let OE and OF be ‘a’ and ‘b’ respectively
𝑎2 − 𝑏2 = 82 − 62 = 28
And 𝑎 + 𝑏 = 14
So, 𝑎 = 8, 𝑏 = 6
Radius of circle = √62 + 82 = √36 + 64
= √100 = 10𝑐𝑚
Area of circle = 𝜋(10)2 = 314.28 𝑐𝑚2
Quantity I > Quantity II

Adda247 Publications For any detail, mail us at


43
Publications@adda247.com
Cracker Book for Bank (IBPS | SBI | RRB PO | Clerk) Mains Exams

40. (b); Quantity I:


Let cost price is 1000 of 1000 gm. he pays to whole seller ⇒ 850
Rs. for 1000 gm.
Effective cost price = 0.85 Rs. per gm.
Now he sells 850 gm. instead of 1000 gm.
Effective cost price of 850 gm. = 0.85 × 850 = 722.5
1000 –722.5
Profit(x) = × 100 = 38.40 ≈ 38%
722.5

Quantity II:
Let milkman have → q litre of milk
Initially he add → 20 litre of water
𝑞
Sale half Remaining mixture = + 10
2
Again add 20 litre water
𝑞
2 4
So, =
10+20 3
40
Total profit(y) = × 100 = 50%
80
Quantity II > Quantity I

41. (d); Quantity I: If B do 40% of work thus A do 60% of work


Ratio of efficiency of A and B = 3 : 2
Total work = 24 × (3 + 2) = 120 units
3×125 15
Efficiency of C = = units/day
100 4
120 × 4
Time taken by C = = 32 days
15

Quantity II: P work for → x day


Efficiency → x
So,
x × x ⇒ 16 Rs.
x ⇒ 4 Rs.
Money earned by P, Q and R
⇒ x² + (x + 1)² + (x + 2) ² = 4² + 5² + 6²
⇒ 77 Rs.
Quantity II > Quantity I

Adda247 Publications For any detail, mail us at


44
Publications@adda247.com
Cracker Book for Bank (IBPS | SBI | RRB PO | Clerk) Mains Exams

42. (a); Quantity I

OB = 8cm and OA = 10cm


As, AB and AC are tangents on smaller circle
⇒ ∠𝐴𝐵𝑂 = ∠𝐴𝐶𝑂 = 90°
⇒ 𝐴𝐵 = √102 − 82 = 6
Area of quadrilateral ABOC
= 2 × 𝐴𝑟𝑒𝑎 𝑜𝑓 𝑡𝑟𝑎𝑖𝑛𝑔𝑙𝑒 𝐴𝐵𝑂
Area of triangle ABO
1 1
= × 𝑂𝐵 × 𝐴𝐵 = × 8 × 6 = 24𝑐𝑚2
2 2
Area of quadrilateral ABOC
= 2 × 24 = 48𝑐𝑚2

Quantity II: Area of trapezium


1
= (sum of parallel sides) × perpendicular distance
2
1
= (6 + 8) × 6 = 42𝑐𝑚2
2
Quantity I > Quantity II

43. (b); Quantity I


20,000×22×2
Interest earned by Kishan after 2 years = = 8800
100
Amount invested by KIshan in another bank = 20,000 + 8800 =
28,800
ATQ,

Adda247 Publications For any detail, mail us at


45
Publications@adda247.com
Cracker Book for Bank (IBPS | SBI | RRB PO | Clerk) Mains Exams

𝑥 3
⇒ 28,800 [1 + ] = 28,800 + 28,800
100
−1350
𝑥 3
⇒ 28,800 [1 + ] = 56,250
100
𝑥 3 56,250
⇒ [1 + ] =
100 28,800
𝑥 3 125
⇒ [1 + ] =[ ]
100 64
𝑥 5
⇒ [1 + ] =[ ]
100 4
⇒ 𝑥 = 25%
5
Quantity II: Marked price is 185 % 𝑜𝑓 𝑡ℎ𝑒 𝑐𝑜𝑠𝑡 𝑝𝑟𝑖𝑐𝑒 𝑜𝑓 𝑎𝑟𝑡𝑖𝑐𝑙𝑒
7
Let Cost price = 100x and profit % is y% after y% discount
1300𝑥
⇒ 100𝑥 + 𝑥𝑦 = (100 − 𝑦)
700
1300𝑥
⇒ 100𝑥 + 𝑥𝑦 = (100 − 𝑦)
700
⇒ 700 + 7𝑦 = 1300 − 13𝑦
⇒ 20𝑦 = 600 ⇒ 𝑦 = 30%
Quantity II > Quantity I

44. (b); A B C
Time 𝑥 + 5 𝑥 𝑥 − 4
1 1 1
∴ + =
𝑥+5 𝑥 𝑥−4
𝑥 = 10
∴ Quantity I → 10 days
Given 2M = 3W
(8M+14W)×x×7 (6M+10W)×15×6
∴ 7 = 5
×360 ×360
12 12
171 2
𝑥= ⇒ = 13
13 13

2
Quantity II → 13
13
∴ Quantity I < Quantity II

Adda247 Publications For any detail, mail us at


46
Publications@adda247.com
Cracker Book for Bank (IBPS | SBI | RRB PO | Clerk) Mains Exams

45. (b);

Let P is faster than Q


Then P covers 72 km distance in the same time as Q covers 48 km
distance
Ratio of the speed = 72 : 48 = 3 : 2
48
∴ Speed of P = × 3 = 72 km/hr
2
Quantity I→ Difference between P and Q = 72 – 48 = 24 km/hr.
Let speed of train = T km/hr
Let speed of car = C km/hr
120 480
∴ + =8 ………..(i)
𝑇 𝐶
200 400 1
+ =8 ………(ii)
𝑇 𝐶 3
On solving (i) and (ii)
T = 60 km/hr
∴ Quantity I < Quantity II

46. (a); 𝑥: Let efficiency of A = 𝑎


and efficiency of B and C be 3𝑦 and 4𝑦
Now
1
(𝑎 + 3𝑦) × 16 = 15 (𝑎 + 4𝑦)
3
𝑎 = 20𝑦
(20𝑦+3𝑦)×16
Time taken by A = = 18.4 days
20𝑦
𝑦: Let efficiency of P, Q and R be 2𝑎, 3𝑎, 4𝑎 respectively
Total work = (2𝑎 + 3𝑎) × 18
5𝑎×18
Required time = = 10 days
9𝑎
Quantity I > Quantity II
Adda247 Publications For any detail, mail us at
47
Publications@adda247.com
Cracker Book for Bank (IBPS | SBI | RRB PO | Clerk) Mains Exams

47. (a); 𝑥:
Cost price = Rs. 𝑥
S.P. = 𝑥 + 100
S.P. after increasing it by 20%
120
2𝑥 = (𝑥 + 100)
100
𝑥 = 𝑅𝑠. 150
𝑦:
Let C.P. = Rs.100
So M.P. = Rs.500
500×40
S.P. = = 200
100
200–100
Profit % = × 100 = 100%
100
Quantity I > Quantity II

48. (e); 𝑥 :
𝑎 = 11
𝑑=1
𝑛
𝑆𝑛 = [2𝑎 + (𝑛– 1)𝑑]
2
𝑥
4040 = [2 × 11 + (𝑥– 1)1]
2
𝑥 = 80
𝑦:
𝑎 = 10
𝑑=2
So,
𝑦
7120 = [2 × 10 + (𝑦– 1)2]
2
𝑦 = 80
Quantity I = Quantity II

49. (b); 𝑥 :
Amount Invested = 𝑥
Time in first scheme = 2 years
Rate = 40%
𝑥×2×40 4𝑥
S.I. = =
100 5
Time in 2nd scheme = 5 years
Adda247 Publications For any detail, mail us at
48
Publications@adda247.com
Cracker Book for Bank (IBPS | SBI | RRB PO | Clerk) Mains Exams

Rate = 44%
𝑥×5×44 11𝑥
S.I. = =
100 5
11𝑥 4𝑥
Difference = – = 18,900
5 5
𝑥 = Rs. 13,500
𝑦:
𝑦 = Interest earned
12000
P= × 100 = 40,000
30
40000×3×12
Interest = = 14,400
100
Quantity II > Quantity I

50. (a); 𝑥 :
Edge of cube = 7 cm
7
So, largest sphere radius = cm
2
Volume of vacant space
4 22 7 7 7 490
=7×7×7− × × × × = cm³
3 7 2 2 2 3
𝑦 : Surface area of sphere = 4πr²
22 7 7
= 4 × × × = 154 𝑐𝑚2
7 2 2
Quantity I > Quantity II

PART B
1. (b); I. 3𝑥+5 . 92𝑥−4 = 95𝑥−14
⇒ 3𝑥+5 . 34𝑥−8 = 310𝑥−28
⇒ 3𝑥+5+4𝑥−8 = 310𝑥−28
⇒ 35𝑥−3 = 310𝑥−28
⇒ 5𝑥 − 3 = 10𝑥 − 28
⇒ 5𝑥 = 25
⇒ 𝑥=5
II. 2𝑦 2 − 15𝑦 − 28 = 3𝑦 2 − 23𝑦 − 13
⇒ 𝑦 2 − 8𝑦 + 15 = 0
⇒ 𝑦 2 − 3𝑦 − 5𝑦 + 15 = 0
⇒ 𝑦(𝑦 − 3) − 5(𝑦 − 3) = 0
⇒ (𝑦 − 5)(𝑦 − 3) = 0
⇒ 𝑦 = 5 ,3 ⇒ x ≥ y

Adda247 Publications For any detail, mail us at


49
Publications@adda247.com
Cracker Book for Bank (IBPS | SBI | RRB PO | Clerk) Mains Exams
25 15
2. (d); I. − +2=0
x² x
⇒2x 2 − 15x + 25 = 0
⇒2x 2 − 10x − 5x + 25 = 0
2x (x − 5) − 5(x − 5) = 0
(2x − 5)(x − 5) = 0
5
x = ,5
2
40 13
II. +1=
y² y
2
⇒ y − 13y + 40 = 0
⇒ y 2 − 8y − 5y + 40 = 0
⇒ y(y − 8) − 5(y − 8) = 0
(y − 5)(y − 8) = 0
y = 5, 8
y ≥ x

3. (e); 𝟐𝐲+𝟒 . 32y+4 . 42y+4 = 32y+4 . 22y+16


𝒚 + 4 + 4𝑦 + 8 = 4𝑦 + 16
𝐲=4
𝒙𝟐 – 10 + 24 = 0
𝒙𝟐 – 6𝑥– 4𝑥 + 24 = 0
𝒙(𝑥– 6)– 4(𝑥– 6) = 0
(𝑥– 4)(𝑥– 6) = 0
𝒙 = 4, 6
𝒙≥𝑦

48 14
4. (d); I. − +1=0
x² x
2
⇒ x − 14x + 48 = 0
⇒ x 2 − 8x − 6x + 48 = 0
⇒ x(x − 8) − 6(x − 8) = 0
⇒ (x − 8)(x − 6) = 0
x = 8, 6
45 1
II. + =2
y² y
2
⇒ 2y − y − 45 = 0
⇒ 2y 2 − 10y + 9y − 45 = 0

Adda247 Publications For any detail, mail us at


50
Publications@adda247.com
Cracker Book for Bank (IBPS | SBI | RRB PO | Clerk) Mains Exams

⇒ 2y(y − 5) + 9(y − 5) = 0
⇒ (2y + 9)(y − 5) = 0
9
𝐲 = 5, −
2
𝐱 > y

5. (d); I. (𝑥 − 4)2 = 9
⇒ 𝑥 − 4 = ±3
⇒ 𝑥 = 7, 1
II. (2𝑦 + 3)2 = 25
⇒ 2𝑦 + 3 = ±5
⇒ 𝑦 = 1, −4
⇒x≥y

6. (c); I. 5x² + 3x – 36 = 0
5x² + 15x – 12x – 36 = 0
5x (x + 3) – 12 (x + 3) = 0
(5x – 12) (x +3) = 0
x = 12/5, –3
II. 2y² – 13y + 20 = 0
2 y ² – 8 y – 5 y + 20 = 0
2 y (y – 4) – 5(y – 4) = 0
(2 y – 5) (y – 4) = 0
y = 5/2, 4
y>x

7. (b); I. x² – 7x + 12 = 0
x² – 4x – 3x + 12 = 0
x(x – 4) –3 (x – 4) = 0
(x – 3) (x – 4) = 0
x = 3, 4
II. 2y² – 11y + 15 = 0
2y² – 6y – 5y + 15 = 0
2y(x – 3) – 3 (y – 3) = 0
(2y – 5) (y – 3) = 0
y = 5/2, 3
x≥y
Adda247 Publications For any detail, mail us at
51
Publications@adda247.com
Cracker Book for Bank (IBPS | SBI | RRB PO | Clerk) Mains Exams

8. (d); I. 2x² + 11x + 15 = 0


2x² + 6x + 5x + 15 = 0
2x (x + 3) +5 (x + 3) = 0
(2x + 5) (x + 3) = 0
x = –5/2, –3
II. 2y² + 9y + 10 = 0
2y² + 4y + 5y + 10 = 0
2y (y + 2) +5 (y + 2) =0
(2y + 5) (y + 2) = 0
Y = –5/2, –2
y≥x

9. (c); I. 3x² + 7x – 40 = 0
3x² + 15x – 8x – 40 = 0
3x (x + 5) – 8x – 40 = 0
(3x – 8) (x + 5) = 0
x = 8/3, –5
II. 5y² – 29y + 42 = 0
5y – 14y – 15 y + 42 =0
y(5y – 14) – 3 (5y – 14) = 0
(y – 3) (5y – 14) = 0
y = 3, 14/5
y>x
10. (e); I. 3x² – 23x + 42 = 0
3x² – 9x – 14x + 42 = 0
3x( x – 3) – 14(x – 3) =0
(3x – 14) (x – 3) = 0
x =3, 14/3
II. 3x² – 19y + 45 = 0
2y² – 10y – 9y + 45 = 0
2y (y – 5) – 9 (y – 5) =0
(2y – 9) (y – 5) = 0
y=9/2, 5
No relation can be established between 𝑥 𝑎𝑛𝑑 𝑦

Adda247 Publications For any detail, mail us at


52
Publications@adda247.com
Cracker Book for Bank (IBPS | SBI | RRB PO | Clerk) Mains Exams

11. (a); I. 𝑥 2 − 5𝑥 + 4 = 0
𝑥 2 − 4𝑥 − 𝑥 + 4 = 0
𝑥(𝑥 − 4) − 1(𝑥 − 4) = 0
(𝑥 − 1)(𝑥 − 4) = 0
𝑥 = 1, 4

II. 𝑦 2 + 5𝑦 + 6 = 0
𝑦 2 + 3𝑦 + 2𝑦 + 6 = 0
𝑦(𝑦 + 3) + 2(𝑦 + 3) = 0
(𝑦 + 2)(𝑦 + 3) = 0
𝑦 = −2, −3
𝑥>𝑦

12. (d); I. 2𝑥 2 − 𝑥 − 15 = 0
2𝑥 2 − 6𝑥 + 5𝑥 − 15 = 0
2𝑥 (𝑥 − 3) + 5(𝑥 − 3) = 0
(2𝑥 + 5)(𝑥 − 3) = 0
−5
𝑥 = ,3
2

II. 3𝑦 2 − 23𝑦 + 42 = 0
3𝑦 2 − 14𝑦 − 9𝑦 + 42 = 0
𝑦(3𝑦 − 14) − 3(3𝑦 − 14) = 0
(𝑦 − 3)(3𝑦 − 14) = 0
14
𝑦 = 3,
3
𝑦≥𝑥

13. (a); I. 𝑥 2 − 15 + 54 = 0
𝑥 2 − 9𝑥 − 6𝑥 + 54 = 0
𝑥(𝑥 − 9) − 6(𝑥 − 9) = 0
(𝑥 − 6)(𝑥 − 9) = 0
𝑥 = 6, 9
II. 𝑦 2 + 15𝑦 − 54 = 0
𝑦 2 + 18𝑦 − 3𝑦 − 54 = 0
𝑦(𝑦 + 18) − 3(𝑦 + 18) = 0
(𝑦 + 18)(𝑦 − 3) = 0
𝑦 = −18, 3
𝑥>𝑦
Adda247 Publications For any detail, mail us at
53
Publications@adda247.com
Cracker Book for Bank (IBPS | SBI | RRB PO | Clerk) Mains Exams

14. (c); I. 𝑥 2 + 14𝑥 + 40 = 0


𝑥 2 + 10𝑥 + 4𝑥 + 40 = 0
𝑥(𝑥 + 10) + 4(𝑥 + 10) = 0
(𝑥 + 4)(𝑥 + 10) = 0
𝑥 = −4, −10
II. 𝑦 2 − 5𝑦 − 24 = 0
𝑦 2 − 8𝑦 + 3𝑦 − 24 = 0
𝑦(𝑦 − 8) + 3(𝑦 − 8) = 0
(𝑦 + 3)(𝑦 − 8) = 0
𝑦 = −3, 8
𝑦>𝑥

15. (e); I. 𝑥 2 − 225 = 0


𝑥 2 = 225
𝑥 = ±15

II. 𝑥 2 + 𝑦 2 = 306
225 + 𝑦 2 = 306
𝑦 2 = 306 − 225
𝑦 2 = 81
𝑦 = ±9
No relation can be established between x and y.

16. (c); I. 2x² – 7x + 6 = 0


2x² – 4x – 3x + 6 = 0
2x (x –2) –3 (x – 2) = 0
(2x – 3) (x – 2) =0
3
𝑥 = 𝑜𝑟 2
2
II. 3y² – 19y + 28 = 0
3y² – 12y – 7y + 28 = 0
3y (y – 4) –7 (y – 4) = 0
(3y – 7) (y – 4) = 0
7
y= ,4
3
y>x

Adda247 Publications For any detail, mail us at


54
Publications@adda247.com
Cracker Book for Bank (IBPS | SBI | RRB PO | Clerk) Mains Exams

17. (a); I. x² – 13x + 36 =0


x² – 9x – 4x + 36 =0
x (x – 9) – 4 (x – 9) =0
(x – 4) (x – 9) = 0
x = 4, 9

II. 3y² – 19y + 30 = 0


3y² – 10y – 9y + 30 = 0
y(3y – 10) – 3 (3y – 10)=0
(y – 3) (3y – 10) = 0
10
y = 3,
3
x>y
18. (a); I. x³ = 120 + 96 = 216
x=6
II. y² – 25 = 0
y² = 25
y = ±5
x>y

19. (e); I. x² – 2x – 48 = 0
x² – 8x + 6x – 48 = 0
x(x – 8) +6 (x – 8) =0
(x + 6) (x – 8) = 0
x = 8, –6

II. y² – 15y + 54 = 0
y² – 9y – 6y + 54 = 0
y(y – 9) –6 (y – 9) = 0
(y – 6) (y – 9) = 0
y = 6, 9
No relation can be established between x and y

20. (c); I. 2x² + 25x + 72 = 0


2x² + 16x + 9x + 72 = 0
2x (x + 8) +9 (x + 8) = 0
Adda247 Publications For any detail, mail us at
55
Publications@adda247.com
Cracker Book for Bank (IBPS | SBI | RRB PO | Clerk) Mains Exams

(2x + 9) (x + 8) = 0
9
x=− ,–8
2
II. 3y² + 22y + 40 = 0
3y² + 10 y + 12y + 40 = 0
y (3y + 10) + 4 (3y + 10) = 0
(y + 4) (3y + 10) = 0
10
y = – 4, –
3
y>x

Adda247 Publications For any detail, mail us at


56
Publications@adda247.com
Cracker Book for Bank (IBPS | SBI | RRB PO | Clerk) Mains Exams

Chapter

13
What is data interpretation: When data is organized into tables and charts
it is done with the purpose of making it meaningful. The objective of data
interpretation is to assess whether a student can understand bars and
charts and Answer some questions based on them. This act of organizing
and interpreting data to get meaningful information under a given set of
conditions is Data interpretataion.

About data interpretation: This is the calculation intensive portion, it


consists of a myrid of graph. charts and tables and analyze data. The key
to crack this area is to quickly Identify the key pieces of information that
you will require to work on.
Basic key that will help you to solve this topic:
• Calculation
• Square • cube
• table • BODMAS
• Percentage
• Profit and loss
• Ratio and proportion
• Average
Types of Data Interpretation:
• Data table
• Line graph
• Pie charts
• Bar graph
• Mixed graph
• Line with pie chart • Table with Bar
• Table with Line, etc. • Radar graph
• Triangular graph
• Case study (Puzzle)
• Venn Diagram • Table format
2 Adda 247 Publications For any detail, mail us at
Publications@adda247.com
Cracker Book for Bank (IBPS | SBI | RRB PO | Clerk) Mains Exams

Approach for data interpretation:


• First you look carefully at the table or graph and the direction. Note
the years to which, the data refers to and the units. Sometimes the
figures may be given in thousands. While the Answer may be millions
Resulting in mistakes.
• The level of approximation that can be done is assessed from the
choices. If the answer is wide, time should not be wasted in working
out exact figures. If the choice ‘none of the above exists, a close
approximation may be required’.
• Read the question carefully, it will give an indication as to which
row and column should be seen. A carefull reading of the question
will reveal exactly what is to be done and the units in which the
answer is required.
• There may be one or two very large question requiring calculations.
Attempt these at the last.
• Revise bar charts, table and line graphs before attempting D.I.
question remember that the D.I. section is a scoring one and also
time saving.
Data Table: The numbers in the bracket give the maximum marks in each
subject.
The following table gives the percentage of marks obtained by seven students in six
different subjects in an examination. Study it and answer the question given below
it.
Students Subject (Maximum Marks)
Maths Chemistry Physics Geography History Computer
(150) (130) (120) (100) (60) Science (40)
Golu 90 50 90 60 70 80

Mithi 100 80 80 40 80 70

Suraj 90 60 70 70 90 70

Gapplu 80 65 80 80 60 60

Mahi 80 65 85 95 50 90

Khushi 70 75 65 85 40 60

Sheetal 65 35 50 77 80 80

3 Adda 247 Publications For any detail, mail us at


Publications@adda247.com
Cracker Book for Bank (IBPS | SBI | RRB PO | Clerk) Mains Exams

Line Graph:
Answer the question based on the given line graph
Following line graph shows the ratio of export to import of company A
and company B over the year

2
Company A
1.75

1.5 Company B
1.25

.75

.5

.25

0
Years 2005 2006 2007 2008 2009 2010

1. In how many of the given years were the exports more than the imports
for company A?
(a) 2 (b) 3 (c) 4 (d) 5

2. If the imports of company A in 2007 were increased by 40%. What


would be the ratio of exports to the increased imports?
(a) 1.50 (b) 1.25 (c) 1.75 (d) 1.35

3. If the exports of company B in 2008 was Rs 237 Crore, what was the
amount of imports in that years ?
(a) 189.6 (b) 188.8 (c) 184.6 (d) 182.8
4 Adda 247 Publications For any detail, mail us at
Publications@adda247.com
Cracker Book for Bank (IBPS | SBI | RRB PO | Clerk) Mains Exams

4. In 2005, the export of company A was double that of company B. If the


imports of company A during the year was 180 crore. What was the
approximate amount of imports of company B during that year?
(a) 190 crore (b) 210 crore (c) 225 crore (d) 200 crore

5. In which year were the exports of company A minimum proportionate


to its imports.
(a) 2008 and 2009 (b) 2009 (c) 2008 (d) 2005

Solution 1: required ratio, more than 1 for the years; 2005, 2006 and 2007,
ie; for 3 years

Solution 2: In 2007, for company A the ratio of exports to import = 175 :


100
Let exports of company A = 175 x Þ and imports of company
A = 100x
New imports of company = 140% of 100x = 140x
175x
There for requied ratio = = 1.25
140x

Solution 3: Let import of company B in 2008 = x


237 237
Then, = 1.25 Þ x = Þ 189.6 crore
x 1.25

Solution 4: In 2005 for company A Þ Export = 1.75 × 180 crore


1.75 ´ 180
in 2005 for company B export = 2

Ex
= 157.5 crore, = .75
But
Im
157.5
Import of company B = = 210 crore
.75

Solution 5: 2008 and 2009, A option.


Pie Charts or Circle Graphs:
Distribution of candidate who were enrolled in MBA and the candidate
(out of those enrolled) who passed the exam in different institutes

5 Adda 247 Publications For any detail, mail us at


Publications@adda247.com
Cracker Book for Bank (IBPS | SBI | RRB PO | Clerk) Mains Exams

X X
P P
16% 12%
V 22% V 18%
12% 15%
T Q T Q
8% 15% 9% 17%
S R S R
17% 10% 16% 13%

Total number of candidates Total number of candidates


enrolled = 8550 who passed the exam = 5700

1. What percentage of candidates passed the exam from institute T out of


the total no. of candidate enrollled from the same institute?

(a) 50% (b) 52.5% (c) 75% (d) 80%


2. Which institute has the highest % of candidates passed to the candidates
enrolled?
(a) Q (b) R (c) V (d) T

3. The no. of candidates passed from institute S and P together exceeds


the no. of candidates enrolled from institutes T and R together by?
(a) 228 (b) 279 (c) 399 (d) 407

4. What is % of candidates passed to the candidate enrolled for institutes


Q and R together ?
(a) 68% (b) 80% (c) 74% (d) 65%

5. What is the ratio of candidates passed to the candidates enrolled from


institute P ?
(a) 9 : 11 (b) 14 : 17 (c) 6 : 11 (d) 9 : 17
6 Adda 247 Publications For any detail, mail us at
Publications@adda247.com
Cracker Book for Bank (IBPS | SBI | RRB PO | Clerk) Mains Exams

9%of 5700
Solution 1: Required % = ´ 100 = 75%
8%of8550
Solution 2: The % of candidates passed to candidates enrolled can be
determined for each institute as under.
17%of 5700
For Q Þ 15%of 8550 ´ 100 = 75.56% ;

13%of 5700
For R Þ ´ 100 = 86.67%
10%of 8550

9%of 5700
For T Þ ´ 100 = 75% ;
8%of 8550

15%of 5700
For V Þ ´ 100 = 83.33%
12%of 8550
So the highest of these is 86.67% corresponding to institutes
R.
Solution 3: Required Difference = [(16% + 18%) of 5700] – [(8% + 10%) of
8550] = 1938 – 1539 = 399
30%of 5700
Solution 4: 25%of 8550 ´ 100 = 80%

18%of 5700 6
Solution 5: = = 6 : 11
22%of 8550 11

7 Adda 247 Publications For any detail, mail us at


Publications@adda247.com
Cracker Book for Bank (IBPS | SBI | RRB PO | Clerk) Mains Exams

Bar Graph: Percentage profit earned by two companies X & Y over the
given years.
Income Expenditure
Pr ofit% = ´ 100
Expenditure
Company X
70 65
Company Y 60
60
55 55
50 50 50 50
Profit%

50 45 45
40
40
35
30

20

2006 2007 2008 2009 2010 2011


1. The income of two company X and Y in 2010 were in the rato of 3 : 4.
respectively. What is the respective ratio of their expenditures in 2010?
(a) 7 : 22 (b) 14 : 19 (c) 15 : 22 (d)27 : 35

2. If the expenditure of company Y in 2007 was Rs 220 crore, what was its
income in 2007?
(a) Rs. 312 crore (b) Rs. 297 crore (c) Rs. 283 crore (d)Rs. 275 crore

3. If the expenditures of company X and Y in 2006 were equal and the


total income of the two companies in 2006 was Rs 342 crore, what was
the total profit of the two company together in 2006?
(a) Rs. 240 crore (b) Rs. 171 crore (c) Rs. 120 crore (d)Rs. 102 crore

4. The expanditure of company X in the year 2008 was Rs 200 crore and
the income of company X in 2008 was the same as its expenditure in
2011. The income of company X in 2011 was?
(a) Rs. 465 crore (b) Rs. 385 crore (c) Rs. 335 crore (d)Rs. 295 crore

5. If the income of two company were equal in 2009, then what was the
ratio of expenditure of company X to that of company Y in 2009?
(a) 6 : 5 (b) 5 : 6 (c) 11 : 6 (d) 16 : 15
8 Adda 247 Publications For any detail, mail us at
Publications@adda247.com
Cracker Book for Bank (IBPS | SBI | RRB PO | Clerk) Mains Exams

165%of E1 3 E 1 15
Solution 1: = Þ Therefore, = = 15 : 22
150%of E 2 4 E 2 22

Solution 2: 220 + 35% of 220 Þ 220 + 77 = 297 crore

Solution 3: Let the expenditure of each companies x and y in 2006 be 100


Then we have 100 + 40 + 100 + 45 = 342 crore
342
Þ Required profit = 285
×85 = 102 crore

Solution 4: Expenditure of Company X in year 2008 is 200 crore, Then


income of company X in year 2008
= 200 + 55% of 200 = 310 crore
Hence, the expenditure of company X in year 2011 is 310 crore,
Therefore, the required income of X in year 2011 = 310 + 50%
of 310 = 465 crore
150 160
Solution 5: X= Y
100 100
x 160 16
be the incomes of two companies in 2009; y = 150 = 15 = 16
: 15

9 Adda 247 Publications For any detail, mail us at


Publications@adda247.com
Cracker Book for Bank (IBPS | SBI | RRB PO | Clerk) Mains Exams

Practice Exercise Based on new Pattern

Direction (1 – 5): Table given below shows total number of students in two
sections of five different schools, percentage distribution of students in
these different sections (X & Y) and also ratio between boys to girls in each
section. Read the table carefully and answer the questions:

Percentage Ratio between Ratio between


Total
Schools of students boys: girls boys : girls in
students
in Section ‘X’ in section ‘X’ Section ‘Y’
LPS 450 48% 2:1 4:5
CMS 360 55% 7:2 1:2
DPS 420 60% 5:4 11 : 10
SKD 300 55% 1:2 2:1
Loyola 250 50% 3:2 2:3

1. Total number of boys in section ‘X’ from CMS & Loyola together are
what percent of total girls in section ‘Y’ from DPS & SKD together?
(a) 183.2 % (b) 180.2 % (c) 188.2 %
(d) 189.2 % (e) 185.2%

2. Find the ratio between total number of girls in section ‘Y’ from CMS &
SKD together to total number of boys in section ‘X’ from DPS & SKD
together?
(a) 65 : 51 (b) 51 : 65 (c) 51 : 67
(d) 51 : 62 (e) 51 : 49

3. Out of total number of boys in section ‘X’ from D.P.S school ratio
between number of boys who got first, second and third division is 1 : 2
: 4. Find total number of boys who got first and third division together
in section ‘X’ from D.P.S school?
(a) 40 (b) 60 (c) 80
(d) 100 (e) 120
Adda247 Publications For any detail, mail us at
10
Publications@adda247.com
Cracker Book for Bank (IBPS | SBI | RRB PO | Clerk) Mains Exams

4. Find the difference between average number of girls in section ‘X’ from
SKD & Loyola together and average number of boys in section ‘Y’ from
LPS & CMS together?
(a) 5 (b) 4 (c) 3
(d) 2 (e) 1

5. Total girls in section ‘Y’ from CMS & LPS together are how much more
than total boys in section ‘X’ from both DPS & SKD together?
(a)33 (b) 43 (c)53
(d) 63 (e) 45

Direction (6 – 10): Table given below shows total number of applicants,


who have applied for CDS exam in the five years. Percentage of applicants
appeared in exam, percentage of applicants qualified in tier I and tier II.
Line graph shows percentage of applicants got selection in CDS exam. Read
the data carefully and answer the questions:

Appeared % of applicants % of students


Total
Years applicants qualified qualified in
Applicants
in tier I in tier I tier II
2013 2400 85% 25% 40%
2014 3200 75% 30% 25%
2015 4800 90% 20% 25%
2016 7500 80% 25% 20%
1
2017 8400 60% 12 % 40%
2

Note – All applicants qualified in tier I appeared for tier II.


50
(Percentage)

40

30

20
2013 2014 2015 2016 2017
Adda247 Publications For any detail, mail us at
11
Publications@adda247.com
Cracker Book for Bank (IBPS | SBI | RRB PO | Clerk) Mains Exams

6. Number of applicants, who have got the final selection in the year
2017, is how many percent less than number of applicants, who have
got the final selection in the year 2014?
2 2 2
(a) 22 % (b) 24 % (c) 28 %
9 9 9
2 2
(d) 18 % (e) 21 %
9 9

7. Find the difference between number of applicants, who have got final
selection in the years 2013 & 2016 together and number of applicants,
who have got final selection in the year 2014 & 2015 together?
(a) 5 (b) 8 (c) 3
(d) 6 (e) 11

8. Find the ratio between applicants qualified in tier II but did not get final
selection in the year 2017 to applicants qualified in tier I but did not
qualify for tier II in the year 2014?
(a) 7 : 20 (b) 7 : 22 (c) 7 : 19
(d) 7 : 18 (e)7 : 16

9. Total applicants appeared for tier I in the year 2017 is what percent
more than total applicants appeared for tier II in the year 2015?
1 1 1
(a) 498 % (b) 438 % (c) 485 %
3 3 3
1 1
(d) 483 % (e) 487 %
3 3

10. Find the total number of applicants, who did not qualify in tier I in the
year 2014, 2016 & 2017 together?
(a) 10390 (b) 10590 (c) 10390
(d) 10290 (e) 10190

Direction (11 – 15): Given below pie graph shows total number of tickets
of ‘Sanju’ movie sold by five multiplex on opening days, while bar graph
shows percentage distribution of three different type of class ticket sold by
each store. Read given data and answer the questions:
Adda247 Publications For any detail, mail us at
12
Publications@adda247.com
Cracker Book for Bank (IBPS | SBI | RRB PO | Clerk) Mains Exams

Total sold tickets - 12500

A
12%
E
B
36%
18%

D C
10% 24%

Jubali Silver Gold


100
90
80
70
60
Percentage

50
40
30
20
10
0
A B C D E
Multiplex

11. Total Jubali class ticket sold by multiplex C & E together is what percent
more than total Gold class ticket sold by multiplex A & C together?
1 1 1
(a) 105 % (b) 102 % (c) 100 %
7 7 7
1 1
(d) 97 % (e) 107 %
7 7

Adda247 Publications For any detail, mail us at


13
Publications@adda247.com
Cracker Book for Bank (IBPS | SBI | RRB PO | Clerk) Mains Exams

12. Find difference between total Silver class ticket sold by multiplex A, C &
E together and total silver gold class ticket sold by multiplex B, C & E
together?
(a) 400 (b) 200 (c) 650
(d) 600 (e) 450

13. Find sum of average number of Jubli class ticket sold by multiplex B & C
and average number of Gold class ticket sold by multiplex C & E?
(a) 2075 (b) 2175 (c) 2275
(d) 2375 (e) 2205

14. Total Silver class tickets sold by multiplex A & D is what percent less
than total Gold class ticket sold by B & E?
4 4 4
(a) 42 % (b) 40 % (c) 38 %
9 9 9
4 4
(d) 44 % (e) 36 %
9 9

15. The ratio of price of one Gold ticket in multiplex D, C & A is 10 : 6 : 7,


and all three multiplex earned Rs. 40000 from the selling of all Gold
Class tickets. Find difference between price of one Gold ticket in
Multiplex A and D?
(a) 10 Rs. (b) 15 Rs. (c) 5 Rs.
(d) 20 Rs. (e) 25 Rs.

Direction (16 – 20): Given below table shows total number of five different
brands of mobiles sold by a store on Sunday. Also given percentage of 2G
mobiles and ratio between 3G and 4G mobile phone sold out of total
number of sold mobiles. Read the table carefully and answer the following
questions:
Number of total Percentage of 2G Ratio of 3G : 4G
Brand
sold mobiles mobiles mobiles
Samsung 750 20% 2:3
Lava 840 25% 3:4
MI 1050 30% 1:4
Huwai 960 15% 1:2
Oppo 640 35% 1:3
Adda247 Publications For any detail, mail us at
14
Publications@adda247.com
Cracker Book for Bank (IBPS | SBI | RRB PO | Clerk) Mains Exams
2
16. Total number of Samsung mobiles sold by store on Saturday is 𝑟𝑑 of
3
the number of same brand of mobile sold on Sunday. If total number of
3G Samsung mobile sold on Saturday is 40 less than that of total
number of 3G Samsung mobile sold on Sunday, then find total number
of 4G Samsung mobile sold on Sunday is what percent more/less than
total number of 4G Samsung mobile sold on Saturday (on Saturday
store sold only 3G and 4G mobile)?
(a) 25% (b) 20% (c) 10%
(d) 15% (e) 5%

17. Total number of MI mobile sold on Monday is equal to average number


of Huwai & Oppo mobile sold on Sunday. If ratio between number of MI
2G, 3G and 4G mobiles sold on Monday is 4 : 5 : 7, then find the ratio
between total number of MI 3G mobile sold on Monday to total number
of Lava 3G mobile sold on Sunday?
(a) 25 : 29 (b) 25 : 27 (c) 25 : 36
(d) 25 : 31 (e) 25 : 33

18. If ratio of total number of Oppo 4G mobiles sold by store on Sunday,


Monday and Tuesday is 6 : 4 : 7, then find total number of Oppo 4G
mobile sold by store on Monday & Tuesday together is what percent
less/more than the total number of Samsung & Lava 4G mobile sold by
store on Sunday together?
5 5 5
(a) 18 % (b) 16 % (c) 24 %
9 9 9
5 5
(d) 20 % (e) 12 %
9 9

19. Find the difference between average numbers of Samsung & Lava 2G
mobile and average numbers of Huwai & Oppo 3G mobile sold by store
on Sunday?
(a) 12 (b) 16 (c) 10
(d) 8 (e) 4

20. If total number of Vivo mobile sold by store on Sunday is 56 more than
total number of 4G Lava & Huwai mobile sold by store on Sunday, then
find total number of Oppo mobile phone sold by store on Sunday is
Adda247 Publications For any detail, mail us at
15
Publications@adda247.com
Cracker Book for Bank (IBPS | SBI | RRB PO | Clerk) Mains Exams

what percent more/less than total number of Vivo mobile sold by store
on Sunday ?
1 1 1
(a) 35 % (b) 38 % (c) 33 %
3 3 3
1 1
(d) 39 % (e) 41 %
3 3

Direction (21- 25): Table given below shows total number of tickets which
were estimated to be sold in five stadiums. Out of total tickets sold,
percentage of A1 tickets sold, and ratio between A2 and A3 is given. Study the
data carefully and answer the following questions.

Stadiums Tickets which can be sold A1 tickets sold (in %) A2 : A3


Nehru 1800 15% 9:8
Patel 2600 25% 4:5
Eden 2200 20% 5:3
Kaloor 2000 20% 7:5
Raipur 2700 30% 3:4

Note: - 200 tickets remain unsold in each stadium


Tickets which can be sold = A1 tickets + A2 tickets + A3 tickets + Unsold
tickets
21. Total A1 and A3 tickets sold in Patel stadium is what percent more than
total number of A1 and A2 tickets sold in Kaloor stadium?
2 2
(a) 16 % (b) 50% (c) 66 %
3 3
1
(d) 25% (e) 33 %
3

22. Total A2 tickets sold in Nehru stadium is how much less than A3 tickets
sold in Raipur stadium?
(a) 220 (b) 240 (c) 260
(d) 280 (e) 300

23. Find the ratio between total A3 tickets sold in Eden and Kaloor stadium
together to total A2 tickets sold in Patel and Eden stadium?
(a) 3 : 2 (b) 2 : 3 (c) 20 :23
(d) 23 : 20 (e) 8 : 9
Adda247 Publications For any detail, mail us at
16
Publications@adda247.com
Cracker Book for Bank (IBPS | SBI | RRB PO | Clerk) Mains Exams

24. Find the average number of total A1 tickets sold by all five stadiums
together?
(a) 514 (b) 490 (c) 470
(d) 450 (e) 430

25. A1 and A2 tickets sold in Nehru stadium is what percent more/less than
A1 and A2 tickets sold in Raipur stadium?
(a) 36% (b) 64% (c) 56.25%
(d) 43.75% (e) 28%

Direction (26-30): Given below bar graph shows percentage distribution


of weekly income of four different Bollywood movies. Table shows total
income of these four movies and percentage of income earning by these
movies from foreign. Read the data carefully and answer the questions.

First week Second week Third week


100
80
60
40
20
0
A B C D
Movies Total income (in crores) % of income from foreign
A 210 26 ⅔%
B 180 25%
C 200 20%
D 240 30%

26. What is the ratio between income of movie B in second week to income
of movie C in third week?
(a) 13 : 5 (b) 13 : 3 (c) 27 : 5
(d) 27 : 10 (e) 27 : 13
Adda247 Publications For any detail, mail us at
17
Publications@adda247.com
Cracker Book for Bank (IBPS | SBI | RRB PO | Clerk) Mains Exams

27. Out of these four movies, which movie beard highest percentage drop
in its income in second week with respect to first week?
(a) A (b) B (c) C
(d) D (e) None of these

28. What is the ratio of income of movie A from foreign to the income of
movie C from foreign?
(a) 8 : 5 (b) 8 : 7 (c) 7 : 5
(d) 28 : 27 (e) 7 : 8

29. If ratio of foreign income of B of given three weeks is 1 : 2 : 2


respectively. Then find the income earn by movie B in second week in
India?
(a) 36 crores (b) 18 crores (c) 26 crores
(d) 30 crores (e) 32 crores

30. Income of movie D in second week and income of movie C in third week
together is approximately what percent more or less then total income
of movies A and B of second week?
(a) 55 (b) 78 (c) 62
(d) 44 (e) 38

Directions (31-35): Study the following graph carefully and answer the
question. The following bar graph shows the budget allocation (in crore) for
education, sport and Health care from year 2018-22.

Adda247 Publications For any detail, mail us at


18
Publications@adda247.com
Cracker Book for Bank (IBPS | SBI | RRB PO | Clerk) Mains Exams

Education Sport Health care


250
220 220
200
200 190 190
170 170
160 160
150
Budget (in crore)

150 125 130


125 130
105
100

50

0
2018 2020 2019
2021 2022
Year
31. What percent is the average budget of sport during 2018 to 2022 than
that of average budget of Education throughout all the years?
(approximately)
(a) 84% (b) 88% (c) 87%
(d) 90% (e) 94%

32. In 2022, budget allocated for sport for male to female are in the ratio of
3 : 2 and it proposed that budget allocated for males to be increased by
25% in 2023 compared to previous year for sport then, what is the
percentage increase in sport budget allocation for 2023 with no change
in sport budget allocated for female in 2023 as compared to previous
year?
(a) 20% (b) 18% (c) 15%
(d) 12% (e) 10%

33. Find the ratio between total budget allocated for Education from year
2018 to 2020 together to total budget allocated for Health care from
year 2020 to 2022 together ?
(a) 17 : 29 (b) 18 : 29 (c) 21 : 29
(d) 27 : 29 (e) 17 : 27
Adda247 Publications For any detail, mail us at
19
Publications@adda247.com
Cracker Book for Bank (IBPS | SBI | RRB PO | Clerk) Mains Exams

34. If in 2019 budget allocated for only three sports i.e. Cricket, Badminton
and Hockey in the ratio of 5 : 4 : 4 respectively and budget allocated for
Education in 2022 for Urban and rural area in the ratio of 5 : 6
respectively. Then find budget allocated for Education for rural area in
2022 how much more than budget allocated for Cricket & Badminton
together in 2019 ?
(a) 30 cr (b) 35 cr (c) 40 cr
(d) 45 cr (e) 25 cr

35. Find the sum of average of budget allocated for Health care throughout
all the year 2018-22 and average of budget allocated for Education
throughout all the year 2018-2022 ?
(a) 336 (b) 338 (c) 340
(d) 342 (e) 348

Direction (36 – 40): Given below table shows the percentage of number of
cars sold by four different motor companies, percentage of number of total
returned cars out of total number of sold cars in two successive years(2016,
2017) . Read the graph carefully and answer the following questions.

Note; (I) Total number of manufactured cars = number of Unsold cars +


number of Sold cars
(II) Total actual number of cars sold = Total number of sold cars
– number of returned cars

Percentage Percentage of Percentage of Percentage of


of number number of number of number of
of cars sold cars sold in cars returned cars returned
in 2016 2017 in 2016 in 2017
TATA 80% 75% 15% 10%
HYUNDAI 65% 70% 20% 15%
MARUTI-
80% 85% 12.5% 17.5%
SUZUKI
HONDA 75% 70% 15% 22.5%

Adda247 Publications For any detail, mail us at


20
Publications@adda247.com
Cracker Book for Bank (IBPS | SBI | RRB PO | Clerk) Mains Exams

36. The ratio between total number of cars manufactured by TATA and
HONDA in the year 2016 is 6 : 7 and the total actual number of cars sold
by HONDA is 4590 units more than that of TATA in the same year. If
total number of cars returned to HYUNDAI in the year 2017 is 1920
units less than total number of cars returned to TATA in 2016 and total
number of cars returned to MARUTI SUZUKI in the year 2017 is 4830
units more than total number of cars returned to HONDA in 2016, then
find the difference between total number of manufactured cars by
HYUNDAI and MARUTI SUZUKI in the year 2017?
(a) 36,000 (b) 42,000 (c) 44,000
(d) 40,000 (e) 32,000

37. Total number of cars manufactured by TATA in the year 2017 is 40%
more than that of total number of cars manufactured by MARUTI
SUZUKI in the year 2016 and total number of cars returned to TATA in
2017 is 120 units more than total number of cars returned to MARUTI
SUZUKI in the year 2016. If total number of cars returned to HONDA in
the year 2017 is 2010 units more than total number of cars returned to
MARUTI SUZUKI in the year 2016, then find total number of cars
manufactured by HONDA in the year 2017?
(a) 24,000 (b) 20,000 (c) 28,000
(d) 18,000 (e) 16,000

38. The ratio between total number of cars manufactured by HYUNDAI,


MARUTI SUZUKI & HONDA in the year 2016 is 2 : 3 : 4 and average
number of cars returned to these three companies in the same year is
4040 units. If total number of cars manufactured by HYUNDAI, MARUTI
SUZUKI & HONDA in the year 2017 is increased by 20%, 25% and
12.5% respectively over that of the previous year, then find the average
number of cars manufactured by HYUNDAI, MARUTI SUZUKI & HONDA
in the year 2017?
(a) 42,400 (b) 42,800 (c) 42,600
(d) 42,000 (e) 41,600
Adda247 Publications For any detail, mail us at
21
Publications@adda247.com
Cracker Book for Bank (IBPS | SBI | RRB PO | Clerk) Mains Exams

39. The ratio between total number of cars manufactured by HYUNDAI &
HONDA in the year 2016 is 8 : 9 and the ratio of number of cars
manufactured by HYUNDAI & HONDA in the year 2016 to 2017 is 2 : 3
and 3 : 5 respectively. If total actual number of cars sold by HYUNDAI &
HONDA together in the year 2016 is 39590 units then find total number
of cars manufactured by HONDA in the year 2017 is what percentage
more/less than total number of cars manufactured by HYUNDAI in the
same year?
(a) 20% (b) 15% (c) 10%
(d) 25% (e) 35%

40. The ratio between total number of cars manufactured by TATA,


HYUNDAI & HONDA in the year 2017 is 7 : 6 : 8 and total actual number
of cars sold by these companies in the year 2017 is 75810 units, then
find total number of cars manufactured by TATA, HYUNDAI & HONDA
together in the year 2017?
(a) 1,24,000 (b) 1,25,000 (c) 1,20,000
(d) 1,26,000 (e) 1,36,000

Direction (41 – 45): Pie-chart shows the distribution of total bikes


manufactured by five different companies. Also Table shows percentage
‘150 CC’ and ‘200 CC’ & total bike sold by these five companies. Study the
data carefully and solve the following questions

Hero
Yamaha 15%
22%

Bajaj
TVS
25%
18%
Honda
20%

Adda247 Publications For any detail, mail us at


22
Publications@adda247.com
Cracker Book for Bank (IBPS | SBI | RRB PO | Clerk) Mains Exams

Total sold
Companies 150CC Sold bikes 200CC Sold bikes
bikes
Hero 35% 65% —
Bajaj 40% 60% —
Honda — — 80%
TVS 75% — —
Yamaha 70% — 75%

Note All companies manufactured only two types of bike ‘150 CC’ & ‘200
CC’. Some values are missing you have to calculate it according to
questions.
Total bikes sold of any company= Total 150 CC bikes sold + Total 200
CC bikes sold

41. Total ‘200 CC’ Honda bikes sold by store is 25% more than total ‘150
CC’ bikes sold by same store. If difference between ‘200 CC’ & ‘150 CC’
bike sold by Honda is 640 then find total ‘200 CC’ bike sold by Yamaha?
(a) 1785 (b) 1782 (c) 1780
(d) 1787 (e) 1791

42. If Hero & Bajaj sold 80% & 90% of total manufactured bikes
respectively and difference between total ‘200 CC’ bikes sold by these
two companies is 4104, then find total ‘150 CC’ bike sold by Bajaj is
what percent more than total ‘150 CC’ bike sold by Hero?
10 10 2
(a) 88 % (b) 92 % (c) 94 %
21 21 7
2 10
(d) 114 % (e) 96 %
7 21

43. If Tvs sold 80% of total manufactured bikes and difference between
total ‘150 CC’ bikes sold by Tvs and Yamaha is 450, then find bike
manufactured by Hero & Bajaj together?
(a) 28000 (b) 26000 (c) 32000
(d) 24000 (e) 36000
Adda247 Publications For any detail, mail us at
23
Publications@adda247.com
Cracker Book for Bank (IBPS | SBI | RRB PO | Clerk) Mains Exams

44. If Hero, Bajaj & Tvs sold 75%, 80% & 90% of total manufactured bikes
respectively and total number of ‘150 CC’ bikes sold by these three
companies is 23124, then find the total bike sold by Honda?
(a) 15568 (b) 16164 (c) 17162
(d) 15360 (e) 17172

45. If total 84000 bikes manufactured by all five companies and ratio
between ‘150 CC’ to ‘200 CC’ bike manufactured by Honda is 3 : 2, then
find total ‘200 CC’ bike sold by Yamaha is what percent less than total
‘150 CC’ bike manufactured by Honda?
(a) 56.75% (b) 54.75% (c) 62.75%
(d) 59.75% (e) 58.75%

Directions (46-50): Given below are two pie-charts which shows the
percentage distribution of employees in Adda247 who travel to their office
in Gurgoan by two different means i.e. by metro and by cab on different
days of week. First pie chart shows data for Metro and second pie chart
shows data for Cab

Saturday Monday
15% 20%

Friday
(50/3)%
Tuesday
Thursday Wed. (70/3)%
10% 15%

Adda247 Publications For any detail, mail us at


24
Publications@adda247.com
Cracker Book for Bank (IBPS | SBI | RRB PO | Clerk) Mains Exams

Saturday
Monday
10%
25%
Friday
20%
Tuesday
(40/3)%
Thursday Wed.
(20/3)% 25%

Note: 1. Ratio of total employees travelling to Gurgaon by metro to by cab is


5 : 4.
2. If difference of persons travelling by metro and by cab on Saturday
is 420.
46. If number of employees travelling on Sunday by cab decreases by ‘Y’
with respect to employees travelling on Friday by cab and number of
employees travelling on Sunday by metro is twice than that of
travelling on Friday by metro . Then find value of ‘Y’?( Given that total
employees on Sunday is 75% of total employees on wed.)
(a) 1280 (b) None of these (c) 1385
(d) 1415 (e) 1255

47. What is the difference of number of employees travelling by metro on


Wednesday and Thursday together and number of employees travelling
by cab on same days together?
(a) 50 (b) 20 (c) None of these
(d) 40 (e) 60

48. If total fare per person travelling by metro & cab are Rs.120, then ratio
of total amount spent on Thursday by all employees travelling by cab to
total amount spent by all employees on same day travelling by metro?
(given that ratio of fare per person travelling by metro to by cab is 5 : 7)
(a) 21 : 23 (b) 56 : 73 (c) None of these
(d) 56 : 75 (e) 53 : 73
Adda247 Publications For any detail, mail us at
25
Publications@adda247.com
Cracker Book for Bank (IBPS | SBI | RRB PO | Clerk) Mains Exams

49. If total fare on Monday by all employees travelling by metro is Rs.


48000. Then find the total fare on same day by all employees travelling
by cab? (if per person fare for each employee is same)
(a) Rs. 48000 (b) Rs. 42000 (c) Rs. 56000
(d)None of these (e) Rs. 26000

50. Total number of employees travelling by metro on Friday and Saturday


together is approximately what percent more or less than number of
employees travelling by cab on same days together?
(a) 48% (b)28% (c) 42%
(d) 38% (e) 32%

Directions (51-55): Table given below shows number of students


appeared in preliminary examination of an exam ‘ASK’ in six different cities
and percentage of students failed in preliminary, mains and in only
reasoning. Those students who cleared the Preliminary examination can
give the Mains examination. In Mains examination of ASK, there are only
two subjects i.e. Reasoning and Quant. Student has to clear both the
subjects to pass in Mains examination of ‘ASK’.

Students Students
Students Students
Failed in Failed in
appeared in Failed in
Mains only
Preliminary Preliminary
examination Reasoning
exam exam (in %)
(in %) (in %)
A 96,000 25% 87.5% 25%
B 80,000 40% 75% 37.5%
2
C 1,20,000 35% 80% 16 %
3
D 1,60,000 15% 75% 25%
E 1,25,000 28% 70% 40%
1
F 72,000 33 % 68.75% 25%
3

Adda247 Publications For any detail, mail us at


26
Publications@adda247.com
Cracker Book for Bank (IBPS | SBI | RRB PO | Clerk) Mains Exams

51. Total number of students who failed only in Quant in City ‘A’ is three
times of total number of students passed the mains exams in city ‘B’
while total number of students who failed in both subjects in City ‘B’ is
3000 less then total number of students who failed is both subjects in
city ‘A’. Find number of students who failed only in Quant in city ‘A’ is
what percent more than number of students who failed in Reasoning in
city ‘B’?
(a) 250% (b) 50% (c) 150%
(d) 200% (e) 100%
52. In city D, total number of students who failed in Reasoning is ‘x’ more
than total number of students who failed in only Quant. If total number
of students who passed in mains exam in city D is ‘x’ then find the ratio
between total number of students who failed in Reasoning to total
number of students who failed in only Quant in city ‘D’.
(a) 1 : 3 (b) 3 : 1 (c) 1 : 1
(d) 1 : 2 (e) 2 : 1

53. Total number of students who failed in Quant in city E is how much
more than total number of students who failed in Quant in city F?
(a) 3000 (b) 6000 (c) 9000
(d) 12000 (e) 15000

54. Ratio between total number of students who failed in Quant in city ‘B’
to total number of students who failed in at most one subject in city ‘C’
is 30 : 91. Find what percent of students failed in both subject in city ‘C’
in mains exam?
(a) 25% (b) 15.5% (c) 17.5%
(d) 19.5% (e) 30%

55. Total number of students who failed in at most one subjects in city F is
24000 less than total number of students who failed in at least one
subject in city A. Find total number of students who failed in both
subjects in city ‘F’ is how much less than total number of students who
failed in only reasoning in city ‘A’.
(a) 3000 (b) 6000 (c) 9000
(d) 12000 (e) 15000
Adda247 Publications For any detail, mail us at
27
Publications@adda247.com
Cracker Book for Bank (IBPS | SBI | RRB PO | Clerk) Mains Exams

Direction (56-60): - A school has four hostels in which there are two
sections one for boys and one for girls. Line chart given below shows
increase/decrease in total number of students in 2013 in these hostels as
compare to 2012(previous year). Table given below shows ratio between
number of boys to number of girls in these hostels in 2013. Study the data
carefully and answer the following questions

300
Hostel Boys : Girls
250
2… Aravali 2:3
200
150 Nilgiri 4:1

100 Udaygiri 5:3

50 Shivalik 4:7
Aravali Nilgiri Udaygiri Shivalik

56. In Aravali, number of girls in 2013 is 20% more than that in 2012. If
change in the number of girls in Aravali is 25% greater than change in
number of boys in Aravali then find the Number of boys in Aravali in
2013.
(a) 900 (b) 600 (c) 500
(d) 750 (e) 800

57. Number of students is increases and decreases in Aravali and Nilgiri in


2013 respectively as compare to 2012. Number of girls who left Aravali
joins Nilgiri (No other change in Number of girls) such that ratio
between increase and decreases in Number of boys in Aravali and
Nilgiri in 2013 as compare to 2012 is 16 : 15, then find the change in
Number of boys in Nilgiri?
(a) 350 (b) 320 (c) 300
(d) 420 (e) 400

Adda247 Publications For any detail, mail us at


28
Publications@adda247.com
Cracker Book for Bank (IBPS | SBI | RRB PO | Clerk) Mains Exams

58. In 2012, Number of boys in Udaygiri and Shivalik are equal and
Number of girls in Shivalik is 540 more than the Number of girls in
Udaygiri. If in 2013, Number of boys in both hostels are equal then find
the Number of total student in Shivalik, If In 2013 strength of both
hostels increases as compare to previous year
(a) 1650 (b) 1200 (c) 960
(d) 1500 (e) 900

59. In 2012 and in 2013 ratio of total strength of Udaygiri to that of Nilgiri
is 4 : 5 and 5 : 8 respectively. Find the strength of Nilgiri in 2012 if
strength of Udaygiri decreases
(a) 1600 (b) 1200 (c) 1250
(d) 1400 (e) 1750

60. In2013, strength of all hostels increases as compare to 2012. If total


strength of hostels in 2012 is 1030, then find the Number of boys in
Aravali in 2013 is how much more than the Number of boys in Udaygiri
in 2013, given that ratio of strength of Aravali, Nilgiri, Udaygiri and
Shivalik is 2 : 3 : 2 : 3 respectively
(a) 80 (b) 100 (c) 120
(d) 90 (e) 10

Adda247 Publications For any detail, mail us at


29
Publications@adda247.com
Cracker Book for Bank (IBPS | SBI | RRB PO | Clerk) Mains Exams

Solutions

1. (a); Total boys in section ‘X” from CMS & Loyola


55 7 50 3
= 360 × × + 250 × ×
100 9 100 5
= 154 + 75 = 229
Total girls in section ‘Y’ from DPS & SKD
(100 –60) 10 (100–55) 1
= 420 × × + 300 × ×
100 21 100 3
= 80 + 45 = 125
229
Required percentage = × 100 = 183.2%
125

2. (b); Total girls in section ‘Y’ from CMS & SKD


(100–55) 2 (100 –55) 1
= 360 × × + 300 × ×
100 3 100 3
= 108 + 45 = 153
Total boys in section ‘X’ from DPS & SKD
60 5 55 1
= 420 × × + 300 × ×
100 9 100 3
= 140 + 55 = 195
153
Required ratio = = 51 : 65
195

3. (d); Total number of boys in section ‘X’ from D.P.S school


60 5
= 420 × × = 140
100 9
5
Required number of students = 140 × = 100
7

4. (e); Average number of girls in section ‘X’ from SKD & Loyola
55 2 50 2
300×100×3+250×100×5 110+50
= = = 80
2 2
Average number of boys in section ‘Y’ from LPS & CMS
(100–48) 4 (100–55) 1
450× 100
×9+360× 100 ×3
=
2
104+54
= = 79
2
Required difference = 80 – 79 = 1

Adda247 Publications For any detail, mail us at


30
Publications@adda247.com
Cracker Book for Bank (IBPS | SBI | RRB PO | Clerk) Mains Exams

5. (b); Total girls in section ‘Y’ from CMS & LPS together
(100–48) 5 (100–55) 2
= 450 × × + 360 × × = 130 + 108 = 238
100 9 100 3
Total boys in section ‘X’ from DPS & SKD
60 5 55 1
= 420 × × + 300 × ×
100 9 100 3
= 140 + 55 = 195
Required difference = 238 – 195 = 43
60 1
6. (a); Applicants got final selection in the year 2017= 8400 × × ×
100 8
40 25
× = 63
100 100
Applicants got final selection in the year 2014
75 30 25 45
= 3200 × × × × = 81
100 100 100 100
81−63
Required percentage = × 100
81
18 2 2
= × 1100 = × 100 = 22 %
81 9 9

7. (d); Total applicants got final selection in the year’s 2013 & 2016
together
85 25 40 25 80 25 20 30
= 2400 × × × × + 7500 × × × ×
100 100 100 100 100 100 100 100
= 51 + 90 = 141
Total applicants got final selection in the year 2014 & 2015
together
75 30 25 45 90 20 25 25
= 3200 × × × × + 4800 × × × ×
100 100 100 100 100 100 100 100
= 81 + 54 = 135
Required difference = 141 – 135 = 6

8. (a); Applicants qualified in tie II but did not get final selection in the
years 2017
60 1 40 (100−25)
= 8400 × × × × = 189
100 8 100 100
Applicants qualified in tier but did not qualify for tier II in the year
2014
75 30 (100−25)
= 3200 × × × = 540
100 100 100
189
Required ratio = = 7 : 20
540

Adda247 Publications For any detail, mail us at


31
Publications@adda247.com
Cracker Book for Bank (IBPS | SBI | RRB PO | Clerk) Mains Exams

9. (d); Total appeared applicants for tier I in the year 2017


60
= 8400 × = 5040
100
Total appeared applicants for tier II in the year 2015
90 20
= 4800 × × = 864
100 100
5040−864
Required percentage = × 100
864
4176 1
= × 100 = 483 %
864 3

10. (b); Total number of applications did not qualify in tier I in the year
2014, 2016 and 2017
75 (100−30) 80 (100−25) 60
= 3200 × × + 7500 × × + 8400 × ×
100 100 100 100 100
(100−12.5)
100
= 1680 + 4500 + 4410 = 10590

11. (e); Total Jubali class ticket sold by multiplex C & E


24 35 36 25
= 12500 × × + 12500 × ×
100 100 100 100
= 1050 + 1125 = 2175
Total gold class ticket sold by multiplex A & C
12 30 24 20
= 12500 × × + 12500 × × = 450 + 600 = 1050
100 100 100 100
2175−1050 1
Required percentage = × 100 = 107 %
1050 7

12. (d); Total silver class ticket sold by multiplex A, C and E


12 50 36 35 24 45
= 12500 × × + 12500 × × + 12500 × ×
100 100 100 100 100 100
= 750 + 1350 + 1575 = 3675
Total gold class ticket sold by multiplex B, C & E
18 30 24 20 36 40
= 12500 × × + 12500 × × + 12500 × ×
100 100 100 100 100 100
= 675 + 600 + 1800 = 3075
Required difference = 3675 – 3075 = 600
Adda247 Publications For any detail, mail us at
32
Publications@adda247.com
Cracker Book for Bank (IBPS | SBI | RRB PO | Clerk) Mains Exams

13. (b); Average number of jubali ticket sold by Multiplex B & C


18 40 24 35
12500× × +12500× ×
100 100 100 100 900+1050
= = 975
2 2
Average number of gold class ticket sold by multiplex C & E
24 20 36 40
12500× × +12500× × 600+1800
100 100 100 100
= = = 1200
2 2
Required sum = 975 + 1200 = 2175

14. (d); Total silver class ticket sold by multiplex A & D


12 50 10 50
= 12500 × × + 12500 × × = 750 + 625 = 1375
100 100 100 100
Total Gold glass ticket sold by B & E
18 30 36 40
= 12500 × × + 12500 × × = 675 + 1800 = 2475
100 100 100 100
2475−1375 1100 4
Required percentage = × 100 = × 100 = 44 %
2475 2475 9

15. (b); Let price of one Gold ticket in multiplex D, C and A be Rs.10x, Rs6x
and Rs7x respectively.
ATQ,
Total earning = 40000
10 10 24 20 12 30
12500× × ×10x+12500× × × 6x + 12500 × × × 7x
100 100 100 100 100 100
1250x + 3600x + 3150x = 40000
40000
x= ⇒ x = 5 Rs
8000
Required difference = 5 × 10 – 5 × 7 = 15 Rs.

16. (b); Total Samsung mobile sold by store on Saturday


2
= 750 × = 500
3
Total Samsung 3G mobile sold on Saturday
80 2
= 750 × × – 40 = 240 – 40 = 200
100 5
Total Samsung 4G mobile sold on Saturday
= 500 – 200 = 300
Total Samsung 4G mobile sold on Sunday
80 3
= 750 × × = 360
100 5
360–300 60
Required percentage = × 100 = × 100 = 20%
300 300

Adda247 Publications For any detail, mail us at


33
Publications@adda247.com
Cracker Book for Bank (IBPS | SBI | RRB PO | Clerk) Mains Exams

17. (b); Total MI mobile sold on Monday


960+640 1600
= = = 800
2 2
MI 3G mobile sold on Monday
5
= 800 × (4+5+7) = 250
250 250
Required ratio = 75 3 = = 25 : 27
840×100×7 270

18. (d); Total Oppo 4G mobile sold on Monday and Tuesday


640×65 3 (4+7)
= × × = 572
100 4 6
Total Samsung & Lava 4G mobile sold by store on Sunday
750×80 3 75 4
= × + 840 × ×
100 5 100 7
= 360 + 360 = 720
720 –572
Required percentage = × 100
720
148 5
= × 100 = 20 %
720 9

19. (d); Average number of Samsung & Lava 2G mobile sold by store on
Sunday
20 25
(750×100+840×100) 150+210
= = = 180
2 2
Average number of Huwai & Oppo 3G mobile sold by store on
Sunday
85 1 65 1
960×100×3+640×100×4 272+104
= = = 188
2 2
Required difference = 188 – 180 = 8

20. (c); Total Vivo mobile sold by store on Sunday


75 4 85 2
= (840 × × + 960 × × ) + 56
100 7 100 3
= (360 + 544) + 56 = 960
960–640
Required percentage = × 100
960
320 1
= × 100 = 33 %
960 3

Adda247 Publications For any detail, mail us at


34
Publications@adda247.com
Cracker Book for Bank (IBPS | SBI | RRB PO | Clerk) Mains Exams

21. (e); Total A1 and A3 tickets sold in Patel stadium


25 75 5
= 2400 × + 2400 × × = 600 + 1,000 = 1600
100 100 9
Total number of A1 and A2 tickets sold in Kaloor stadium
20 80 7
= 1800 × + 1800 × × = 360 + 840 = 1200
100 100 12
1600−1200 400 1
Required % = × 100 = × 100 = 33 %
1200 1200 3
22. (d); A2 tickets sold in Nehru stadium
85 9
= 1600 × × = 720
100 17
A3 tickets sold in Raipur stadium
70 4
= 2500 × × = 1,000
100 7
Required difference = 1000 − 720 = 280

23. (b); A3 tickets sold in Eden and Kaloor stadium together


80 3 80 5
= 2000 × × + 1800 × × = 600 + 600 = 1200
100 8 100 12
A2 tickets sold in Patel and Eden stadium together
75 4 80 5
= 2400 × × + 2000 × × = 800 + 1000 = 1800
100 9 100 8
1200 2
Required ratio = =
1800 3

24. (c); Total number of A1 tickets sold by all five stadiums together
15 25 20 20 30
= 1600 × + 2400 × + 2000 × + 1800 × + 2500 ×
100 100 100 100 100
= 240 + 600 + 400 + 360 + 750 = 2350
2350
Required average = = 470
5

25. (a); A1 and A2 tickets sold in Nehru stadium


15 85 9
= 1600 × + 1600 × × = 960
100 100 17
A1 and A2 tickets sold in Raipur stadium
30 70 3
= 2500 × + 2500 × × = 1500
100 100 7
1500−960
Required % = × 100 = 36%
1500

Adda247 Publications For any detail, mail us at


35
Publications@adda247.com
Cracker Book for Bank (IBPS | SBI | RRB PO | Clerk) Mains Exams

26. (d); Income of movie B in second week


30
= × 180 = 54 crores
100
Income of movie C in third week
10
= × 200 = 20 crores
100
Required ratio = 27 : 10

27. (d); Income of movie A in first week


50
= × 210 = 105 crores
100
Income of movie A in second week
30
= × 210 = 63 crores
100
42
% drop = × 100 = 40%
105
Similarly,
Income of movies B in first week
40
= 180 × = 72 crore
100
Income of movies B in second week
30
= 180 × =54 crore
100
18
% drop = × 100 = 25%
72
Income of movie C in first week
60
= 200 × = 120 crores
100
Income of movie C in second week
30
= 200 × = crores
100
60
% drop = × 100 = 50%
120
70
Income of movie D in week 1 = × 240
100
= 168 crore
10
income of movie D in week 2 = × 240
100
= 24 crore
144 5
% drop = × 100 = 85 %
168 7
Hence D is the answer.
Adda247 Publications For any detail, mail us at
36
Publications@adda247.com
Cracker Book for Bank (IBPS | SBI | RRB PO | Clerk) Mains Exams
80
28. (c); Foreign income of movie A = × 210 = 56 crores
300
20
Foreign income of movie C = × 200 = 40 crores
100
Required ratio = 7 : 5
25
29. (a); Total income of B from foreign = × 180 = 45 crores
100
2
Foreign income in second week = × 45 crores = 18 crores
5
Income earn by movie B in second week in India
30
= × 180 – 18 = 36 crores
100

30. (c); Second week income of movie D


10
= × 240 = 24 crore
100
10
Third week income of movie C = × 200 = 20 crore
100
Second week income of movie A
30
= × 210 = 63 crore
100
Second week income of movie B
30
= × 180 = 54 crore
100
117–44 73
Required % = × 100 =
117 117

31. (e); Average budget for sport


125+130+150+170+160 735
= = = 147
5 5
Average budget for Education
105+125+130+200+220 780
= = = 156
5 5
147
Required% = × 100 = 94.23 ≈ 94%
156

32. (c); Budget allocated for sport for male in 2022


3
= 160 × = 96 𝑐𝑟.
5
Budget allocated for sport for male in 2023
5
= 96 × = 120 cr
4
24
Required increase = × 100 = 15%
160

105+125+130 360
33. (b); Required ratio = = = 18 ∶ 29
170+190+220 580

Adda247 Publications For any detail, mail us at


37
Publications@adda247.com
Cracker Book for Bank (IBPS | SBI | RRB PO | Clerk) Mains Exams

34. (a); Budget allocated for Cricket and Badminton together in 2019
(5+4)
= 130 × (5+4+4) = 90 cr
Budget allocated for Rural Education in 2022
6
= 220 × = 120 cr
11
Required difference = 120 – 90 = 30 cr

35. (d); Average of budget allocated for Health care in the year 2018-22
160+190+170+190+220 930
= = = 186
5 5
Average of budget allocated for Education in year 2018-22
105+125+130+200+220 780
= = = 156
5 5
Required sum = 186 + 156 = 342

36. (e); Let, total number of manufactured cars by TATA & HONDA in the
year 2016 be 6x and 7x units respectively.
ATQ,
75 (100−15) 80 (100−15)
7𝑥 × × − 6𝑥 × ×
100 100 100 100
= 4590
75 85 80 85
7𝑥 × × − 6𝑥 × × = 4590
100 100 100 100
357𝑥 102𝑥
− = 4590
80 25
1785𝑥−1632𝑥
= = 4590
400
4590×400
𝑥= ⇒ x = 12,000 units
153
Total number of cars returned to HYUNDAI in 2017 = (12000 ×
80 15
6) × × − 1920
100 100
= 8640 – 1920 = 6720
Total number of cars returned to MARUTI SUZUKI in 2017
75 15
= (12000 × 7) × × + 4830 = 9450 + 4830 = 14,280 units
100 100
Let total number of cars manufactured by HYUNDAI in 2017 be x
units
70 15
𝑠𝑜, 𝑥 × × = 6720
100 100
6720×100×100
𝑥= ⇒ x = 64,000 units
70×15

Adda247 Publications For any detail, mail us at


38
Publications@adda247.com
Cracker Book for Bank (IBPS | SBI | RRB PO | Clerk) Mains Exams

Let total number of cars manufactured by MARUTI SUZUKI in 2017


be y units
so,
85 17.5
𝑦× × = 14280
100 100
14280×100×100
𝑦= ⇒ y =96,000 units
85×17.5
Required difference = 96000 – 64000 = 32,000 units

37. (c); Let total number of cars manufactured by MARUTI SUZUKI in the
year 2016 be 100x units
So, total number of cars manufactured by TATA in 2017 be 140x
units
ATQ,
75 10 80 12.5
140𝑥 × × − 100𝑥 × × = 120
100 100 100 100
21𝑥
− 10𝑥 = 120 ⇒ x = 240
2
Total number of cars returned to HONDA in 2017 = (240 × 100) ×
80 12.5
× + 2010 = 2400 + 2010 = 4410 units
100 100
Let total number of cars manufactured by HONDA in year 2017 be x
units
ATQ,
70 22.5
𝑥× × = 4410
100 100
4410×100×100
𝑥= ⇒ x = 28,000 units
70×22.5

38. (c); Let total number of cars manufactured by HYUNDAI, MARUTI


SUZUKI & HONDA in 2016 be 2y, 3y and 4y respectively.
ATQ,
65 20 80 12.5 75 15
2𝑦×100×100+3𝑦×100× 100 +4𝑦×100×100
= 4040
3
13𝑦 3𝑦 9𝑦
+ + = 12120
50 10 20
101y = 1212000 ⇒ y = 12,000
Required average
120 125 112.5
(12000×2)× +(12000×3)×100+(12000×4)× 100
100
=
3
28800+45000+54000 127800
= = = 42,600
3 3
Adda247 Publications For any detail, mail us at
39
Publications@adda247.com
Cracker Book for Bank (IBPS | SBI | RRB PO | Clerk) Mains Exams

39. (d); Let total number of cars manufactured by HYUNDAI & HONDA in
the year 2016 be 8x and 9x units respectively.
ATQ,
65 (100−20) 75 (100−15)
8𝑥 × × + 9𝑥 × × = 39590
100 100 100 100
104𝑥 459𝑥
+ = 39590
25 80
3959𝑥
= 39590 ⇒ x = 4,000 units
400
Total number of cars manufactured by HONDA in 2017
36000
= × 5 = 60,000
3
Total number of cars manufactured by HYNDUAI in 2017
32000
= × 3 = 48,000
2
60000−48000 12000
Required percentage = × 100 = × 100 = 25%
48000 48000

40. (d); Let total number of cars manufactured by TATA, HYUNDAI &
HONDA in the year 2017 is 7y , 6y & 8y units respectively
ATQ –
75 100−10 70 100−15 70 100−22.5
7𝑦 × × + 6𝑦 × × + 8𝑦 × × = 75810
100 100 100 100 100 100
189𝑦 357𝑦 217𝑦
+ + = 75810
40 100 50
945𝑦+714𝑦+868𝑦
= 75810
200
2527y = 75810 × 200 ⇒ y = 6,000
Total number of cars manufactured by TATA, HYUNDAI & HONDA
together in the year 2017
= 7 × 6000 + 6 × 6000 + 8 × 6000
= 42000 + 36000 + 48000 = 1,26,000

41. (b); Let total bike manufactured by all five companies = 100x
So, total bike manufactured by Honda
= 20x
Ratio between total ‘200 CC’ & ‘150 CC bikes sold by Honda = 5 : 4
80 5 80 4
20x × × − 20𝑥 × × = 640
100 9 100 9
80𝑥 64𝑥
− = 640 ⇒ x = 360
9 9
Total ‘200 CC’ bike sold by Yamaha
22 75 (100−70)
= 36000 × × × = 1782
100 100 100
Adda247 Publications For any detail, mail us at
40
Publications@adda247.com
Cracker Book for Bank (IBPS | SBI | RRB PO | Clerk) Mains Exams

42. (d); Let total bike manufactured by all five companies = 100x
Total ‘150 CC’ bike sold by Bajaj
25 90 40
= 100x × × × = 9x
100 100 100
Total ‘150 CC’ bike sold by Hero
15 90 35
= 100x× × × = 4.2x
100 100 100
9𝑥−4.2𝑥 2
Required percentage = × 100 = 114 %
4.2𝑥 7

43. (d); Let total bike manufactured by all five companies = 100x
ATQ –
75 70 80 75
22x × × − 18𝑥 × × = 450
100 100 100 100
11.55x − 10.8x = 450
0.75x = 450 ⇒ x = 600
Total bike manufactured by Hero & Bajaj
(15+25)
= 60000 × = 24000
100

44. (d); Let total bike manufactured by all five companies = 100x
75 35 80 40 90 75
15x × × + 25𝑥 × × + 18𝑥 × × = 23124
100 100 100 100 100 100
3.9375x + 8x + 12.15x = 23124
x = 960
Total bike sold by Honda
20 80
= 96000 × × = 15360
100 100

45. (e); Total ‘200 CC’ bike manufactured by Honda


20 3
= 84000 × × = 10080
100 5
Total ‘150 CC’ bike sold by Yamaha
22 75 (100−70)
= 84000 × × × = 4158
100 100 100
10080−4158
Required percentage = × 100 = 58. 75%
10080

Adda247 Publications For any detail, mail us at


41
Publications@adda247.com
Cracker Book for Bank (IBPS | SBI | RRB PO | Clerk) Mains Exams

Solutions (46-50):
Let total employees travelling by metro & by Cab be 5𝑥 & 4𝑥 respectevely.
15 10
5𝑥 × x – 4𝑥 × x = 420
100 100
75𝑥 40𝑥
− = 420 ⇒ X=1200
100 100
∴ Total employees travelling by metro = 1200 x 5 = 6000
& total employees travelling by cab = 1200 x 4 = 4800

50
46. (c); Total employees travelling on Friday by metro = x 6000 = 1000
300
20
Total employees travelling on Sunday by cab = x 4800–y=960 – y
100
ATQ,
960 – y + 2 x 1000
75 15 25
= [ × 6000 + × 4800]
100 100 100
2960 – y = .75 x 2100
Y = 2960 - 1575 = 1385

47. (b); No. of employees travelling by metro on Wednesday & Thursday


together.
15 10
= × 6000 + × 6000
100 100
= 900 + 600 = 1500
No. of employees travelling by Cab on same days together.
25 20
= × 4800 + × 4800
100 300
= 1200 + 320 = 1520
Required difference = 1520 – 1500 = 20

48. (d); Fare per person travelling by metro


5
= × 120 = Rs. 50
12
Fare per person travelling by cab .
7
= × 120 = 𝑅𝑠 70
12
20
300
×4800×70
Require ratio = 10 = 56 : 75
100
×6000×50

Adda247 Publications For any detail, mail us at


42
Publications@adda247.com
Cracker Book for Bank (IBPS | SBI | RRB PO | Clerk) Mains Exams

49. (a); Total employees travelling by metro on Monday


20
= × 6000 = 1200
100
48000
∴ Per person fare = = 𝑅𝑠. 40
1200
∴ Total fare on Monday of all employees travelling by cab
25
= × 4800 × 40 = 𝑅𝑠. 48000
100

50. (e); Total No. of employees travelling by metro on Friday & Saturday
together
50 15
= × 6000 + × 6000
300 100
= 1000 + 900 = 1900
Total no. of employees travelling by cab on same days together
20 10
= × 4800 + × 4800
100 100
= 960 + 480 = 1440
1900−1440
Required Percentage = × 100
1440
approximately 32%

51. (b); In city A,


Number of students appeared in mains exam
75
= 96000 × = 72000
100
Number of students passed in mains exam
12.5
= 72000 × = 9000
100
In city B,
Number of students appeared in mains exam
60
= 80,000 × = 48000
100
Number of students passed in mains exam
25
= 48000 × = 12000
100
Total number of students who failed only in quant in city A = 3 ×
12000 = 36000
Total number of students who failed in both subjects in city ‘A’
25
= 72000 – 9000 – 36000 – 72000 × = 9000
100
Total number of students who failed in both exam in city ‘B’ = 9000
– 3000= 6000
Adda247 Publications For any detail, mail us at
43
Publications@adda247.com
Cracker Book for Bank (IBPS | SBI | RRB PO | Clerk) Mains Exams

Total number of students who failed in Reasoning = 48000 ×


37.5
+ 6000
100
= 18000 + 6000 = 24000
36000−24000
Required % = × 1000 = 50%
24000
Vein diagram for failed students:-

52. (e); In city D,


Number of students appeared in mains exam
85
= 1,60,000 × = 1,36,000
100
Number of students passed in mains exam
25
= 1,36,000 × = 34,000 = 𝑥
100
Total number of students who failed in reasoning = 34,000 + x
Total number of students who failed in Quant
75
= 1,36,000 × − 34,000 = 68,000
100
Let Total number of students who failed in only Quant = a
And
Let Let Total number of students who failed in both subjects = b

Adda247 Publications For any detail, mail us at


44
Publications@adda247.com
Cracker Book for Bank (IBPS | SBI | RRB PO | Clerk) Mains Exams

ATQ,
𝑎 + 𝑏 = 68,000
And 34,000 + 𝑏 = 𝑎 + 34,000
So 𝑎 = 𝑏 = 34,000
Total number of students who failed in Reasoning = 34,000 +
34,000 = 68,000
Total number of students who failed in only Quant = 34,000
68,000 2
Required Ratio = =
34,000 1

53. (b); Total number of students who failed in Quant in city ‘E’ = Total
number of students who failed in Mains exam – Total number of
students who failed in only Reasoning
72 70−40
= 1,25,000 × × = 27,000
100 100
Total number of students who failed in Quant in city ‘F’ = Total
number of students who failed in Mains exam – Total number of
students who failed in only Reasoning
2 68.75−25
= 72,000 × × = 21,000
3 100
Required difference = 27,000 − 21,000 = 6,000

54. (e); Total number of students who failed in Quant in city ‘B’
60 75−37.5
= 80,000 × × = 18,000
100 100
Total number of students who failed in at most one subject in city
18,000
‘C’ = × 91 = 54,600
30
In city ‘C’
Total number of students who failed in at most one subject in city
‘C’ = Total number of students who failed in Reasoning only + Total
Adda247 Publications For any detail, mail us at
45
Publications@adda247.com
Cracker Book for Bank (IBPS | SBI | RRB PO | Clerk) Mains Exams

number of students who failed in only Quant + Total number of


students who passed in both subjects
Total number of students who passed in both subjects
65 20
= 1,20,000 × × = 15,600
100 100
Total number of students who failed in only Reasoning
65 50
= 1,20,000 × × = 13,000
100 300
Total number of students who failed in only Quant
= 54600 − 15,600 − 13,000 = 26,000
Total number of students who failed in Quant
65 80 50
= 120000 × ×[ − ] = 49,400
100 100 300
Total number of students who failed in both exams
= 49,400 − 26,000 = 23,400
23,400
Required % = × 100 = 30%
78000

55. (c); Total number of students who failed in atleast one subject in city ‘A’
75 87.5
= 96,000 × × = 63,000
100 100
Total number of students who failed in at most one subject in city
‘F’ = 63,000 − 24,000 = 39,000
Total number of students who failed in only Quant in city ‘F’
2 31.25+25
= 39,000 − 72,000 × × = 12,000
3 100
Total number of students who failed in both subjects
2 68.75−25
= 72,000 × × − 12,000 = 21,000 − 12,000 = 9,000
3 100
Total number of students who failed in only reasoning in city ‘A’
75 25
= 96,000 × × = 18,000
100 100

Adda247 Publications For any detail, mail us at


46
Publications@adda247.com
Cracker Book for Bank (IBPS | SBI | RRB PO | Clerk) Mains Exams

Required difference = 18,000 − 9,000 = 9,000

56. (b); Let Number of girls and boys in Aravali in 2012 be x and y
respectively.
In 2013
Number of girls = 1.2 x
Change in girls = 0.2x
0.2𝑥
Change in boys is = × 100 = 0.16x
125
Total increment = 0.2x + 0.16x
0.36x = 270
270×100
x= ⇒ x = 750
36
Number of boys in Aravali in 2013
2
= × 1.2 × 750 = 600
3

57. (c); Let Number of Girls in Aravali be x.


Let Number of girls who left Aravali and joins Nilgiri is ‘a’
Total strength of Aravali increases which means change in number
of boys is
= 270 + a
Adda247 Publications For any detail, mail us at
47
Publications@adda247.com
Cracker Book for Bank (IBPS | SBI | RRB PO | Clerk) Mains Exams

Total strength of Nilgiri decreases, which means change in number


of boys in Nilgiri is = 250 + a
270+𝑎 16
= ⇒ a = 50
250+𝑎 15
Number of boys increases in Nilgiri = 250 + 50 = 300

58. (a); Let Number of boys in Udaygiri in Shivalik in 2012 = x


Let Number of Girls in Udaygiri in 2012 = y
So, Number of Girls in Shivalik = y + 540
In 2013,
Let Number of boys in Udaygiri and Shivalik = 20a
⇒ Number of Girls in Shivalik = 35a
And, Number of Girls in Udaygiri = 12a
ATQ
x + y + 150 = 32a … (i)
x + (y + 540) + 300 = 55a … (ii)
Solving (i) and (ii)
a = 30
Shivalik strength = 55a = 55 × 30 = 1650

59. (e); Let in 2012


Total strength of Udaygiri and Nilgiri be 4x and 5x respectively
And total strength of Udaygiri and Nilgiri in 2013 be 5y and 8y
respectively
ATQ,
4x – 5y = 150 … (i)
Two cases formed
Either
8y – 5x = 250 ... (ii)
Or 5x – 8y = 250 ... (iii)
On solving (i) and (iii) it does give-ve integer value
So, on solving (i) and (ii)
x = 350 ⇒ y = 250
Strength of Nilgiri in 2012 = 1750

Adda247 Publications For any detail, mail us at


48
Publications@adda247.com
Cracker Book for Bank (IBPS | SBI | RRB PO | Clerk) Mains Exams

60. (d); Total strength in 2013 = 1030 + 970


= 2000
Number of boys in Aravali in 2013
2000 2
= × 2 × = 160
10 5
Number of boys in Udaygiri in 2013
2000 5
= × 2 × = 250
10 8
Required Difference = 250 – 160 = 90

Adda247 Publications For any detail, mail us at


49
Publications@adda247.com
Cracker Book for Bank (IBPS | SBI | RRB PO | Clerk) Mains Exams

1 Adda247 Publications For any detail, mail us at


Publications@adda247.com
Cracker Book for Bank (IBPS | SBI | RRB PO | Clerk) Mains Exams

Chapter
Arithmetic Data Interpretation
14
BEST APPROACH TO SOLVE THE QUESTIONS

INTRODUCTION: DI stands for DATA INTERPRETATION. Interpretation


means representing a data in a sensitive way after analyzing it well. In all
the banking exam or particularly quant section of bank, DI plays vital role.
The basic DI consists of pie table, bar graph, numerical value table or graph.
Look at this example:

900
800
700
Number of mobiles

600
500
400
300
200
100
0
Mon Tue Wed Thu Fri Sat

Instead of telling ‘number of mobiles sold’ for each day it is represented in


the graph. So that it is more visible and interested for a reader in this form.
Common form of DI are ratios, percentages or average.

ARITHMETIC DI

Arithmetic DI means representing arithmetic equations and conditions


in bar graphs, line graph, pie graph or in any other form of DI. The questions
on arithmetic DI also cover major portion in mains of any banking exam. So
2 Adda247 Publications For any detail, mail us at
Publications@adda247.com
Cracker Book for Bank (IBPS | SBI | RRB PO | Clerk) Mains Exams

it is necessary to understand the actual concept. Easy way to understand


this is through an example.

Total Road construct = 3600 meters


E A
46° 72°

D
90°
B
80°
C
72°
In this pie chart, portion of ‘Road construct’ is given for 5 different
72
companies. For eg. Company A constructs × 3600 = 720 meters road.
360
Now to make question more complicated different individual for each team
may be given. Let us consider through a bar graph.

32
28
24
No. of Labour in 1 team

20
16
12
8
4
0
A B C D E
Name of teams

3 Adda247 Publications For any detail, mail us at


Publications@adda247.com
Cracker Book for Bank (IBPS | SBI | RRB PO | Clerk) Mains Exams

Here we have considered number of individuals in each team. Now they


may ask you questions like “How many meters 1 member of team A
constructs”. It is definitely easy one. We just had to do
720
= 40 meters.
18
or to make it more complicated they may ask you
“How many day will 8 members of team C will take to construct 3600
meters road?”
Do it yourself.

HINT: Calculate road constructed by an individual of team C in 1 day as we


have calculated previously. Then calculate how many days 8 men will take
to construct given road.
A key point to solve any arithmetic DI is that to understand what had been
given in DI. Then consider these as an individual separate questions from
arithmetic, also these questions are way easier. Now let us consider another
example:

25% Series 1
20%
20%
15%
15% 12%
10%
10%
5%
5%

0%
A B C D E

Look carefully, here a % graph is given and nothing have been mentioned.
Examiner may mention it “% of distance covered by these (A, B, C, D & E)
individuals in 1 hour and total distance is 100 km” OR
“ % of work done by individuals in given time” OR
“ Profit/loss % earned on selling these items for shopkeeper”

4 Adda247 Publications For any detail, mail us at


Publications@adda247.com
Cracker Book for Bank (IBPS | SBI | RRB PO | Clerk) Mains Exams

“ Interest rate given by a bank to different individuals” or many other


things.
Point is arithmetic DI is not as hard as student consider it in exams, it is
always easier than actual arithmetic questions that are asked in exam.
Before solving a question, we must exactly know the information he had
given. For eg. Consider that this graph shows profit % for selling these
items. In question, if examiner ask to calculate actual selling price, then he
must mention the CP or some other clue in exam.

Example: Profit earned on selling item C is Rs. 160, what is amount of


profit earned for item D, if both items have same selling price?
Solution: This is a typical DI question. Now look, if we know that 20%
means Rs 160 we can calculate actual CP of item C. (Profit % is
always calculated on CP).
100
CP OF ITEM C = 160 × = 800.
20
It means SP of item C is Rs. 960
Also SP of item D is Rs. 960. And then we can calculate CP and
amount of profit of item D.

5 Adda247 Publications For any detail, mail us at


Publications@adda247.com
Cracker Book for Bank (IBPS | SBI | RRB PO | Clerk) Mains Exams

Practice Exercise Based on new Pattern

Direction (1 - 4): Given below bar graph shows number of hours taken by
six person to complete a task individually. Read the data carefully and
answer the questions:

50

40

30

20

10

0
Neeraj Aniket Saurabh Gopal Veer Sameer

1. Neeraj, Saurabh and Gopal start working together but due to bad health
1 1
the efficiency of Neeraj and Gopal decreases by 12 % and 33 %
2 3
respectively. Then find in how many hours task will be completed by all
three?
1 1 1
(a) 11 hours (b) 10 hours (c) 12 hours
4 4 4
1 1
(d) 9 hours (e) 13 hours
4 4

2. Veer and Sameer started working together on another task, where


Sameer work with 25% less efficiency. Veer and Sameer work for y
hours and remaining work complete by Aniket in (y + 1) hours, if ratio
of work done by Veer and Sameer together and by Aniket alone is 2 : 1,
then in how many hours Neeraj will complete same task alone?
1 1 1
(a) 15 hours (b) 13 hours (c) 17 hours
2 2 2
1 1
(d) 11 hours (e) 9 hours
2 2
6 Adda247 Publications For any detail, mail us at
Publications@adda247.com
Cracker Book for Bank (IBPS | SBI | RRB PO | Clerk) Mains Exams

3. If Divyaraj can do 50 % more work in one hour as Neeraj can do in one


hour, while Manish can do 25% less work in one hour as Aniket can do
in one hour. Saurabh started working alone and after some time he left
and remaining work completed by Divyaraj & Manish together in 11.75
hours more than time for which Saurabh worked. Then find total time
in which work is completed?
1 1 1
(a) 14 hours (b) 12 hours (c) 10 hours
4 4 4
1 1
(d) 16 hours (e) 8 hours
4 4
4. Neeraj, Aniket, Veer and Sameer worked together on a work in first
hour, while Saurabh & Gopal together destroyed the same work in
second hour. If this work continues till the total work completed. Find
how many hours required to complete the whole work?
270 270 270
(a) 22 hours (b) 20 hours (c) 18 hours
401 401 401
270 270
(d) 24 hours (e) 16 hours
401 401

Direction (5 - 9): The given bar-graph shows the number of marbels of


different colors which are contained by two different bags X and Y. Read the
data carefully and answered the following questions.

X Y
7
6
5
4
3
2
1
0
Yellow Red Black Blue

7 Adda247 Publications For any detail, mail us at


Publications@adda247.com
Cracker Book for Bank (IBPS | SBI | RRB PO | Clerk) Mains Exams

5. If two marbels are picked at random from bag Y then what is the
probability that both marbels are blue in color?
21 19 63
(a) (b) (c)
190 190 190
11 10
(d) (e)
95 99

6. If two marbels are picked at random from bag X then what is the
probability that both are yellow in color?
3 4 7
(a) (b) (c)
53 51 59
5 8
(d) (e)
51 59

7. If two marbels are picked at random from bag Y then what is the
probability that one is red and one is black in color?
1 6 7
(a) (b) (c)
19 95 95
8 13
(d) (e)
95 190

8. If two marbels are picked at random from bag X then what is


probability that either both are black or both are blue in color?
1 4 9
(a) (b) (c)
15 105 115
3 1
(d) (e)
37 17

9. If three marbels are picked at random from bag Y then what is the
probability that one is black, one is blue and one is yellow in color?
17 49 53
(a) (b) (c)
95 285 381
13 47
(d) (e)
95 360
Directions (10-14): Given below pie chart (I) shows percentage
distribution of total filled quantity of water in seven tanks. Line graph
shows time (minutes) taken by seven pipe A, B, C, D, E, F and G in filling
tanks P, Q, R, S, T, U and V respectively. Give the answer of the question
according to given data :
8 Adda247 Publications For any detail, mail us at
Publications@adda247.com
Cracker Book for Bank (IBPS | SBI | RRB PO | Clerk) Mains Exams

Total quantity of seven tanks = 1500 ℓ


P
V
15%
12%
U Q
18% 12%

R
T 8%
15% S
20%

65
60
55
Time (in minutes)

50
45
40
35
30
25
20
A B C D E F G
Pipes

10. A man has a tank which has quantity of water equal to the quantity of
water in P and T together. He opened pipe B and D together for filling
his tank. After X minutes man closed both pipes and opened pipe C and
E for filling remaining tank. If tank filled in next (x + 15) minutes then
find how many liter of water filled by B and D together ?
(a) 160 l (b)180 l (c)150l
(d)120l (e) 144 l
9 Adda247 Publications For any detail, mail us at
Publications@adda247.com
Cracker Book for Bank (IBPS | SBI | RRB PO | Clerk) Mains Exams

11. For filling tank S, man opened pipe D which fills the tank with 20%
more efficiently. After X minutes man close the pipe D and opened pipe
C and G together. Pipe C fills the tank with 25% more efficiently. If pipe
C and G fills remaining tank in 15 minutes then find the value of X ?
(a) 12 minutes (b) 10 minutes (c) 15 minutes
(d) 8 minutes (e) 18 minutes

12. Quantity of another tank K is 120% of total quantity of all seven tank.
For filling the bigger tank man opened pipes alternatively in such a way
that three pipe B, D and G together for first minute, pipe A alone for
second minute and pipe F and C together for third minutes. Find the
total time required in filling the tank alternatively?
(a) 151 7 minutes (b)141 7 minutes (c)151 minutes
9 9
(d)150 minutes (e) 170 minutes

13. A man have a tank M, which has equal quantity of water as tank S and U
together . For filling tank M man opened pipe A, C and G together for
17
some minutes man thought tank will be fully filled, but it takes 7
19
minutes more because of a leak. Find in what time leak will empty the
fully filled tank?
(a) 3hr (b) 3 2 hr (c)2 1 hr
3 6
1
(d) 3 1 hr (e) 4 hr
6 3

14. A man have a tank of 570 liter. Man opened three pipes B, D and C
together for filling the tank but pipe C fills the tank at 175% of its initial
Efficiency. There is a leak at a height of 2/3 from the bottom of tank
,which empty the tank with half of the efficiency of what B, D and C
filling the tank. Man noticed the leak after 32 minutes and closed all
pipes and leak. Find how many liter of tank remain unfilled?
(a) 90l (b) 76l (c) 78l
(d) 85l (e) 96l

10 Adda247 Publications For any detail, mail us at


Publications@adda247.com
Cracker Book for Bank (IBPS | SBI | RRB PO | Clerk) Mains Exams

Directions (15 − 𝟏𝟗): Given below the table shows Investment of five
person, time and share of profit. Some data are missing, calculate that
according to the question and answer the questions –

Person Investment (Rs) Time (months) Profit share (Rs.)


P — 12 65700
Q 32000 — —
R — — —
S 24000 — —
T — — 24300

15. P, Q and S invested in a business together, investment of P is 75% of


investment of S and all three invested for same time. If Q and S gets
extra 12% and 15% of total profit respectively and remaining profit is
distributed according to their share, then find the total profit.
(a) 270000 Rs. (b) 370000 Rs. (c) 375000 Rs.
(d) 275000 Rs. (e) 325000 Rs.

16. Person Q and R invested in the ratio of 4 : 3 and ratio between time
period of Investment Q to R is 5 : 3. If both person agree that 65% of the
total profit should be divided equally and remaining profit is to be
divided into ratio of their capital. If Q gets 10010 Rs. more to R. then
find the total profit share of R ?
(a) 27765 Rs. (b) 28190 Rs. (c) 42497 Rs.
(d) 34307 Rs. (e) 32695 Rs.

17. Q started a business with his investment, after some month S came to
joined with him and invest his amount in business. At the end of 3 year,
the ratio of profit of Q to S is 3 : 2. Find after how many month S joined
the business ?
(a) 4 months (b) 6 months (c) 3 months
(d) 5 months (e) 6 months
11 Adda247 Publications For any detail, mail us at
Publications@adda247.com
Cracker Book for Bank (IBPS | SBI | RRB PO | Clerk) Mains Exams

18. If T received Rs. 24300 as profit out of the total profit of Rs. 40500
which T and R earned at the end of one year. If T invested Rs. 81000 for
9 months, whereas R invested his amount for the whole year, what was
the amount invested by R ?
(a) 30500 Rs. (b) 40500 Rs. (c) 35500 Rs.
(d) 32500 Rs. (e) 40050 Rs.

19. If Q and R invested into the ratio of 8 : 9 and R and S Invested into the
ratio of 3 : 2. At the end of the year if they all got a total profit of 37030
Rs. then find the share of profit of Q, R and S individually ?
(a) 14490 Rs, 9660 Rs, 12880 Rs. (b) 9660 Rs, 17710
Rs, 11005 Rs.
(c) 12880 Rs, 14490 Rs, 9660 Rs. (d) 12434 Rs, 13594
Rs, 11006 Rs.
(e) None of these
Direction (20- 23): Data about investments of different persons is given
below. Study the data and solve the questions carefully.
→ Investment of Neeraj is 50% more than investment of Aman while
investment of Neeraj is 25% less than that of Sandeep.
Total Investment = 40,000

Rakesh,
30% Saurabh,
40%

Sandeep, 30%

20. Neeraj and Saurabh started a business together. After 8 months of


business they invested Rs 1000 per month for every month. If annual
profit is Rs 8000 then find Saurabh’s profit out of total profit?
(a) Rs 2950 (b) Rs 4000 (c) Rs 5050
(d) Rs 4450 (e) Rs 4850

12 Adda247 Publications For any detail, mail us at


Publications@adda247.com
Cracker Book for Bank (IBPS | SBI | RRB PO | Clerk) Mains Exams

21. Aman and Rakesh started a business together. Rakesh left the business
‘x’ months before the completion of year while Aman increased his
investment by Rs 2000 after 8 months and then after 2 months more he
increased his investment by Rs 2000 again. If ratio between profit
share of Aman and Rakesh is 7 : 8 then Rakesh worked for how many
months?
(a) 8 months (b) 4 months (c) 6 months
(d) 2 months (e) 10 months

22. Sandeep and Neeraj started a business together. After 8 months, Neeraj
is replaced by another partner, Satish whose investment is Rs 4000
more than investment of Saurabh. If Satish worked for 3 months, then
find the profit share of Sandeep if annual profit is Rs 11,500.
(a) Rs 2500 (b) Rs 3000 (c) Rs 4500
(d) Rs 5000 (e) Rs 6000

23. Aman invested his amount in a scheme which after 20% p.a. at C.I. for 2
years while Saurabh invested his amount in a car whose value will
depreciate at 20% p.a. every year. Find the total value of amount Aman
and Saurabh will have after 2 years.
(a) Rs. 31,680 (b) Rs. 18,880 (c) Rs. 26,880
(d) Rs. 16,320 (e) Rs. 22,880

Direction (24 – 25): Given below table shows quantity of four different
liquid in four different vessels. Read the data carefully and answer the
questions.

Vessels Milk Water Orange Juice Mango juice


(ml) (ml) (ml) (ml)
P 240 — 144 —
Q 160 80 — 210
R — 144 192 —
S 160 — — 120

13 Adda247 Publications For any detail, mail us at


Publications@adda247.com
Cracker Book for Bank (IBPS | SBI | RRB PO | Clerk) Mains Exams

24. In vessel P quantity of water is 50% of quantity of milk in same vessel,


while quantity of mango juice in vessel P is 6.25% less than quantity of
Orange juice in vessel R. In vessel S quantity of water is 50% more than
quantity of milk in same vessel and quantity of Orange juice in vessel S
is 40 ml less than quantity of water in same vessel. If 114 ml mixture
from vessel P and 180 ml from vessel S taken out and mixed in vessel T,
then find percentage of orange juice in vessel T?
25 25 25
(a) 29 % (b) 25 % (c) 27 %
147 147 147
25 25
(d) 23 % (e) 21 %
147 147

1
25. Quantity of milk in vessel R is 33 % less than quantity of water in
3
same vessel and quantity of orange juice in vessel Q is 25% less than
quantity of milk in same vessel. For making 1086 ml mixture solution
mixture of vessel Q & R mixed together, if ratio of quantity of total
mango juice in vessel R to total quantity of four liquid in vessel R is 7 :
43, then find the ratio between total mango juice in resulting mixture to
total given mixture?
49 49 49
(a) (b) (c)
161 151 181
49 49
(d) (e)
121 111

Direction (26-30): - Table given below shows details of 3 types of item of a


garment shop MRP, discount% (on MRP) and certain offer.
Shopkeeper have shoes, Jeans & T-Shirts in 2 types of each.
S₁, S₂; J₁, J₂; and T₁, T₂ represents the types of two types of shoes, jeans and
T-shirts respectively.

Items Shoes Jeans T-shirts


Types S₁ S₂ J₁ J₂ T₁ T₂
MRP (Rs) 1200 1647 1200 — 400 600
1
Discount 0% 11 % 0% 25% 10% 0%
9
Shop offer      

Note: Shop offer: Buy 2 get 2 free


 → Offer given  → Offer not given
14 Adda247 Publications For any detail, mail us at
Publications@adda247.com
Cracker Book for Bank (IBPS | SBI | RRB PO | Clerk) Mains Exams

26. If a person wants to buy T-shirts, what is minimum average price/t-


shirts he could get?
(a) Rs. 400 (b) Rs. 360 (c) Rs. 300
(d) Rs.600 (e) Rs. 500

27. If cost price of J₂ for shopkeeper is Rs. 1000. Find MRP of J₂ if he earns a
profit of 20% on selling a single piece.
(a) Rs. 1600 (b) Rs. 1800 (c) Rs. 1500
(d) Rs. 1700 (e) None of these
2
28. Find the cost price of S₂ if he earns a profit of 14 % on C.P.
7
(a) Rs. 1464 (b) Rs. 1098 (c) Rs. 1281
(d) Rs. 1379 (e) None of these

29. A boy buys 5 pieces of T₂, 4 pieces of J₁, & 1 piece of S₁ including offer.
Find the amount paid by him (in Rs.)
(a) 6000 (b) 7200 (c) 4800
(d) 3600 (e) 5400

30. If a person buys 4 pieces of T₂, 2 pieces of T₁ & 4 pieces of J₁ including


offer. Find overall discount% obtained by him.
(a) 40% (b) 44% (c) 42%
(d) 46% (e) 48%

Directions (31-35): Given below line graph shows the distance travelled
by car A in given time intervals and table shows ratio of average speed of
car A to average speed of car B in same time intervals. Both car started at
12:00 Noon.
90
80
70
60
50
40
30
12:00 – 1:00 1:00 – 2:00 2:00 – 3:00 3:00 – 4:00 4:00 – 5:00
15 Adda247 Publications For any detail, mail us at
Publications@adda247.com
Cracker Book for Bank (IBPS | SBI | RRB PO | Clerk) Mains Exams

Time Internal Ratio


12:00 – 1:00 6:5
1:00 – 2:00 14 : 11
2:00 – 3:00 6:7
3:00 – 4:00 4:3
4:00 – 5:00 4:7

31. What is average speed of car A during whole journey?


(a) 63 km/hr (b) 65 km/hr (c) 62 km/hr
(d) 70 km/hr (e) 68 km/hr

32. If both cars started from same point at same time, what is difference
between distance travelled by them till 3: 00 PM.
(a) 20 km (b) 15 km (c) 24 km
(d) 16 km (e) 12 km

33. Average speed of B between 2: 00 PM to 4: 00 PM is what % more or


less than average speed of A in same time interval?
1 2 4
(a) 7 % (b) 14 % (c) 28 %
7 7 7
5
(d) 35 % (e) None of these
7

34. If B have to cover a distance of 540 km till 9 : 00 PM, even after having a
rest of 45 minutes between 5 : 00 to 5 : 45 PM. What should be his
average speed for 4 hours between 5:00 PM to 9:00 PM.
1 4
(a) 60 km/hr (b) 59 km/hr (c) 58 km/hr
7 7
(d) 58 ¾ km/hr (e) 56 ⅔ km/hr

35. If A consumes 1liter petrol for every 15 km, and if A is 25% more
economic than B. What is the ratio of petrol consumed by them
till2:00PM?
(a) 91 : 92 (b) 93 : 97 (c) 104 : 105
(d) 106 : 107 (e) 103 : 106
16 Adda247 Publications For any detail, mail us at
Publications@adda247.com
Cracker Book for Bank (IBPS | SBI | RRB PO | Clerk) Mains Exams

Directions (36-40): The following line graph shows principal amount(in


10000) submitted by 5 persons in a scheme which offers simple interest
and table shows the rate of interest (ROI) per annum obtained by these
persons, time (in years) for which they invested their amount and third
column shows ratio of amount obtained after they withdraw their money to
that of interest obtained.
Some figures shown are missing. Calculate them according to question.

12
11
10
9
8
7
6
5
4
3
A B C D E

Persons ROI Time A:I


A — 5 8:3
B 16% — 41 : 16
C — 6 —
D 8% 5 —
E 15% — 29 : 9

36. What is the time period in months for which B invested his money?
(a) 60 (b) 48 (c) 64
(d) 72 (e) None of these

37. If the ratio of amount obtained by A to that of C is 4: 3. Find the ROI


obtained by C.
(a) 8% (b) 12% (c) 8.5%
(d) 10% (e) 12.5%
17 Adda247 Publications For any detail, mail us at
Publications@adda247.com
Cracker Book for Bank (IBPS | SBI | RRB PO | Clerk) Mains Exams

38. If a new person Vikas deposit half of the money invested by A, for 3
more years than C, calculate his ROI, if SI obtained by him is same as SI
obtained by D.
1 1 1
(a) 11 % (b) 9 % (c) 12 %
9 11 3
2
(d) 14 % (e) None of these
7

39. What is the ratio of time period for which E invested to that of B?
(a) 1 : 3 (b) 7 : 2 (c) 5 : 4
(d) 4 : 5 (e) 3 : 4

40. If after drawing his whole money, D invested 50% of this amount in CI
for 2 years at ROI/annum of 10% and remaining of the money he kept
with him. Calculate total money with him after 2 years.
(a) Rs. 169400 (b) Rs. 84700 (c) Rs. 154700
(d) Rs. 167400 (e) Can’t be determined

Directions (41-44): Bar-graph shows the percentage distribution of


distance covered in upstream and downstream by Rahul on different dates.

Upstream distance=6000 km Downstream distance=2000 km


32
28
24
20
16
12
8
4
0
1st Aug. 2nd Aug. 3rd Aug. 4th Aug. 5th Aug.

18 Adda247 Publications For any detail, mail us at


Publications@adda247.com
Cracker Book for Bank (IBPS | SBI | RRB PO | Clerk) Mains Exams

Table given below shows the speed of current in km/hr on different dates.

Date speed of current (in km/hr)


1st Aug. 10
2nd Aug. —
3rd Aug. 4
4th Aug. —
5th Aug. 3
1
41. If time taken by Rahul to swim upstream on 1st August. is 61 hr. more
3
than time taken by him to swim downstream on same date, then find
the speed of Rahul in still water?
(a) 16 km/hr (b)30 km/hr (c) 25 km/hr
(d) 20 km/hr (e) None of these

42. If speed of Rahul in still water on 2nd Aug. is 8 km/hr and speed of
Rahul in still water on 3rd Aug. is 25% more than his speed on 2nd Aug.
2
and time taken by him to travel upstream on 2 nd August is 166 % more
3
than time taken by him to travel downstream on same date, then find
the seven times of the speed of current on 2nd Aug.?
(a) 8 km/hr (b) 6 km/hr (c) 4 km/hr
(d)None of these (e) 2 km/hr

43. Speed of current on 4th Aug. is 50% of downstream speed of Rahul on


5th August, then time to cover 128 km upstream on 4th Aug. is ? [Given
that time of Rahul to cover upstream distance on 5th August is 80 hr and
assuming that speed of Rahul in still water is same on both days]
1 2
(a) 12 hr (b) 6 hr (c) 10 hr
3 3
(d) 21 hr (e)None of these

44. If ratio of speed of current on 2nd August to 4th August is 2 : 3 & speed of
Rahul on both days are same and time in upstream on 4th August. is 24
times the time taken in downstream on 2th August then find the speed
of Rahul in still water if speed of current on 3rd August is 50% of speed
of current on 2nd Aug.(approximately)?
(a) 21 km/h (b) 23km/h (c) None of these
(d) 25km/h (e) 20km/h
19 Adda247 Publications For any detail, mail us at
Publications@adda247.com
Cracker Book for Bank (IBPS | SBI | RRB PO | Clerk) Mains Exams

Directions (45-48): The following bar-graph shows the ratio of speed of


three different trains Rajdhani exp., Shatabadi exp. and Duranto exp. on five
different days of a week. And line graph shows the number of coaches
attached to Rajdhani express on different days of week. Length of each train
is the sum of the length of all coaches and length of engine.

Rajdhani Exp Shatabadi Exp Duranto Exp


7
6
5
4
3
2
1
0
Monday Tuesday Wednesday Friday Sunday

30

25

20

15

10
Monday Tuesday Wednesday Friday Sunday
Length of each coaches for every train = 15 m
And length of engine for every train = 20 m

20 Adda247 Publications For any detail, mail us at


Publications@adda247.com
Cracker Book for Bank (IBPS | SBI | RRB PO | Clerk) Mains Exams

45. On Tuesday, Rajdhani exp. crosses a tunnel of some length in 12.5 sec
while Duranto express having length 25% more than that of Rajdhani
exp. crosses the same tunnel on the same day in 11.9 sec. Then find the
time taken by Rajdhani exp. and Shatabadi exp. on the same day to
cross each other if they are running in opposite direction and the length
of Shatabadi exp. is 10 m less than that of Rajdhani exp.
(a) 8 sec (b) 9 sec (c) 10sec
(d) 11 sec (e) 12 sec

46. On Sunday, Duranto exp. having length 110 m less than that of Rajdhani
8
exp. crosses Rajdhani exp. travelling in opposite direction in 8 sec
9
then find the approximate time taken by Shatabadi exp. to cross
Duranto exp. on the same day when both are running in the same
direction and length of the Shatabadi exp. is 20% less than that of
Rajdhani exp.
(a) 79 sec (b) 81 sec (c) 75 sec
(d) 84 sec (e) 72 sec

47. On Friday, the speed of Rajdhani exp. was 90 km/hr and on the same
day, Shatabadi exp. started from station P and going to Q while Duranto
started from station Q and going to P. If speed of Shatabadi is increased
by 10% after first hour and 20% after second hour whereas the speed
of Duranto is increased by 20% after first hour and 25% after second
hour then they meet in 3 hours. Find the distance between P and Q.
(a) 947 km (b) 952 km (c) 955 km
(d) 957 km (e) 960 km

48. Duranto exp. having length 28% more than that of Rajdhani exp. on
Monday crosses a platform of same length in 12.8 sec. Then find the
difference between average speed of shatabadi and Rajdhani and the
speed of Duranto exp. on Monday.(in km/hr)
(a) 32.8 (b) 59.4 (c) 42.6
(d) 54.2 (e) 58.8
21 Adda247 Publications For any detail, mail us at
Publications@adda247.com
Cracker Book for Bank (IBPS | SBI | RRB PO | Clerk) Mains Exams

Directions (49-53): Pie chart given below shows length of six different
trains and table given below shows ratio between speed of six trains on
three different days. Study the data carefully and answer the following
questions.
Total length = 1600m

F, 8%
A, 16%

E, 22%
B, 18%

D, 12%

C, 24%

Train Speed on : Speed on : Speed on


Monday Tuesday Wednesday
A 2 : 3 : 2
B 3 : 4 : 5
C 4 : 6 : 5
D 4 : 4 : 7
E 6 : 9 : 5
F 4 : 5 : 3

49. On Wednesday, train ‘B’ crosses train ‘D’ coming from opposite
direction in 6 seconds. If speed of train ‘B’ on Monday is 97.2 km/hour
then in how much time train ‘F’ can cross train ‘D’ on Monday if train ‘D’
is coming from opposite direction and speed of train ‘F’ on Monday is
20 m/sec. (2 Marks)
(a) 6 seconds (b) 8 seconds (c) 10 seconds
(d) 12 seconds (e) 14 seconds
22 Adda247 Publications For any detail, mail us at
Publications@adda247.com
Cracker Book for Bank (IBPS | SBI | RRB PO | Clerk) Mains Exams

50. Train ‘C’ start from Delhi on Monday at 7:00 p.m and reach Kanpur on
next day at 3:00 pm. In return journey on Tuesday, train ‘C’ start from
Kanpur at 6 : 00 pm and reach Jaipur which is 180 km ahead of Delhi at
5 : 48 pm on Wednesday. Find the time taken by train ‘C’ to cross a pole
on Monday? (2 Marks)
(a) 8 seconds (b) 16 seconds (c) 19.2 seconds
(d) 14.4 seconds (e) 28.8 seconds

51. On Monday, train ‘A’ takes 2.5 hours more to cover 900 km distance
than train ‘C’. If train ‘A’ can cross a platform of length 128 in 12.8
seconds on Tuesday then find in how much time (in seconds) train ‘C’
can cross two poles 66 m apart from each other on Tuesday? (2 Marks)
(a) 12 seconds (b) 16 seconds (c) 20 seconds
(d) 24 seconds (e) 30 seconds
52. Ratio between speed of train ‘E’ to train ‘F” on Monday is 3 : 2. On
Tuesday train ‘E’ cross train ‘F’ running in same direction in 24 seconds
then find the time in which train ‘E’ can overtakes train ‘F’ on
Wednesday? (2 Marks)
(a) 48 seconds (b) 24 seconds (c) 12 seconds
(d) 36 seconds (e) 60 seconds

53. Ratio between time taken by train ‘B’ to train ‘D’ to cross a pole on
Monday is 1 : 1. The time taken by train ‘B’ to cross a pole on
Wednesday is what percent more/less than time taken by train ‘D’ to
cross a pole on Monday? (2 Marks)
(a) 30% (b) 40% (c) 50%
(d) 60% (e) 70%

Direction (54-58): Table given below shows profit percentage earned on


selling two different items X and Y and discount percentage offered by five
retailers on these items. Mark price of each article sold by each retailer is
same while cost price of article for each retailor may vary. Study the data
carefully & answer the following question

23 Adda247 Publications For any detail, mail us at


Publications@adda247.com
Cracker Book for Bank (IBPS | SBI | RRB PO | Clerk) Mains Exams

Items→ X Y
Retailor
Profit % Discount % Profit % Discount %

A 20% – – 15%
B – 26.5% 20% 32.5%
C 25% – 60% 24%
D 20% 34% – 37%
E 35% 46% – 28%

54. On article ‘X’, 28% discount is offered by ‘A’. If selling price of article ‘Y’
sold by ‘A’ is Rs 312 more than selling price of article ‘X’ sold by ‘A’ then
find profit percent earned by ‘A’ on selling article ‘Y’ given that average
of cost price of both article for ‘A’ is Rs 1520. (2 Marks)
(a) 18.5% (b) 22.5% (c) 27.5%
(d) 32.5% (e) 37.5%

55. Discount % on article ‘X’ offered by ‘C’ is 15% while profit % of article
1
‘Y’ sold by ‘E’ is 33 %. If difference between cost price of article ‘Y’ sold
3
by ‘B’ and ‘E’ together is Rs 216 more than cost price of article ‘X’ sold
by ‘C’ and ‘E’ together then find the cost price of article ‘Y’ sold by ‘C’? (2
Marks)
(a) Rs. 4940 (b) Rs. 3420 (c) Rs. 3800
(d) Rs. 4180 (e) Rs. 4560

56. Find the ratio between cost price of article ‘X’ for ‘B’ to cost price of
article ‘Y’ for ‘D’. If profit % of article ‘X’ sold by ‘B’ and profit % of
article ‘Y’ sold by ‘D’ is 68% and 20% respectively. (2 Marks)
(a) 2 : 3 (b) 5 : 6 (c) 1 : 2
(d) 5 : 8 (e) 5 : 7

57. Selling price of article ‘X’ sold by ‘E’ is same as cost price of article ‘Y’ for
‘E’. Find the profit % earned by ‘E’ on selling article ‘Y’? (1 Mark)
2
(a) 15% (b) 16 % (c) 25%
3
1 2
(d) 33 % (e) 41 %
3 3
24 Adda247 Publications For any detail, mail us at
Publications@adda247.com
Cracker Book for Bank (IBPS | SBI | RRB PO | Clerk) Mains Exams

58. Seller A marked article ‘X’ 100% above its cost price. If on selling both
article he earns total 25% profit, then find selling price of article ‘X’ sold
by ‘A’ is what percent less than cost price of article ‘Y’ for ‘A’. (2 Marks)
1 1
(a) 10% (b) 9 % (c) 8 %
11 3
(d) 20% (e) 15%

Directions (59-63): Line graph given below shows the distance between
Delhi to five different cities in kilometer and Table given below shows the
speed of five different cars in km/hr

3500

3000
Distance from Delhi (in km)

2500

2000

1500

1000

500

0
City A City B City C City D City E

Cars Speed (in kmph)


P 40
Q —
R 60
S —
T 75

NOTE: - Some data is missing you have to calculate according to question.

25 Adda247 Publications For any detail, mail us at


Publications@adda247.com
Cracker Book for Bank (IBPS | SBI | RRB PO | Clerk) Mains Exams

59. Time taken by car ‘P’ to travel from city ‘E’ to Delhi and then Delhi to
city ‘B’, is equal to the time taken by car ‘R’ to travel from Delhi to city
‘A’ and then city ‘A’ to city ‘B’. Find the distance between city ‘A’ and city
‘B’.
(a) 5650 km (b) 5750 km (c) 5450 km
(d) 5550 km (e) 5320 km

60. Find the approximate time car ‘T’ takes to reach city ‘E’ from city ‘A’ if
city ‘A’ and city ‘E’ is north and east direction of Delhi respectively.
(a) 24 hours (b) 27 hours (c) 20 hours
(d) 36 hours (e) 42 hours

61. Car Q and Car S start from Delhi for city B and city C respectively and
they reached in equal time. If Car Q and Car S starts from city B and city
D respectively at same time and move towards each other, then time
taken by car Q to cross car S is what percent of the time taken by car Q
to reach city B from Delhi. Distance between city B and city D is 1500
km.
(a) 25% (b) 20% (c) 30%
(d) 40% (e) 50%

62. A thief runs in a car S from Delhi to city E and after 6 hours of running,
a policeman started to catch him in a car R. Due to this, thief increases
the speed of his car by 100%. By this, the policeman is able to catch him
3
at th of the distance of city E from Delhi. Find the initial speed of car ‘S’.
5
(a) 15 km/hr (b) 27 km/hr (c) 20 km/hr
(d) 25 km/hr (e) 40 km/hr

63. Car P and Car Q start from Delhi for city A. Car Q first reaches at city A
and meets car P in between the way, 200 km from city ‘A’. Find after
how much time they will meet second time after first time meeting if
they continue their to and fro motion.
(a) 24 hours (b) 15 hours (c) 16 hours
(d) 25 hours (e) 20 hours

26 Adda247 Publications For any detail, mail us at


Publications@adda247.com
Cracker Book for Bank (IBPS | SBI | RRB PO | Clerk) Mains Exams

Direction (64-68): - Line chart given below shows time taken by five
different persons to complete a work ‘M’ alone. Ratio of efficiency of all five
persons remain same throughout any work. Study the data carefully and
answer the following questions.

70
60
50
40
30
20
10
0
Abhi Bhavya Neeraj Satish Veer

64. All five starts working together to complete work ‘X’. ‘Veer’ left after 8
days. Work done by ‘Bhavya’ is same as work done by ‘Neeraj’ while
‘Abhi’ and ‘Neeraj’ worked for same time. ‘Satish’ worked for ‘y’ days. If
‘Bhavya’, ‘Neeraj’ and ‘Satish’ together can complete work ‘X’ in 24 days
then find the value of ‘y’ if Bhavya worked for starting 10 days.
(a) 7 days (b) 9 days (c) 11 days
(d) 13 days (e) 15 days

65. Abhi and Neeraj together can complete work ‘Z’ in (A + 42) days while
Bhavya and Satish together can complete work ‘Z’ in (A + 15) days. All
start the work Z such that ratio between work done by Abhi, Bhavya
and Veer is 1 : 2 : 3, while ratio between days, Neeraj, Satish and Veer
worked is 2 : 2 : 1. Find how many days ‘Bhavya’ worked.
(a) 10 days (b) 15 days (c) 20 days
(d) 30 days (e) 40 days
27 Adda247 Publications For any detail, mail us at
Publications@adda247.com
Cracker Book for Bank (IBPS | SBI | RRB PO | Clerk) Mains Exams

66. All five persons started together to complete work ‘Y’. Veer worked for
starting 6 days and left the work. After 3 days more both Bhavya and
Satish left too. Remaining 40% work should be completed by Abhi and
Neeraj together but ‘Abhi’ left after ‘x’ days. Remaining work is
completed by ‘Neeraj’ in ‘z’ days. If ‘z – x = 3’, then number of days for
which ‘Neeraj’ worked is what percent more than number of days for
which ‘Abhi’ worked.
1 2
(a) 33 % (b) 50% (c) 66 %
3 3
(d) 75% (e) 100%

67. Abhi, Bhavya and Neeraj together starts to do work ‘M’. After 7 days
‘Neeraj’ left and after 3 days more ‘Abhi’ and ‘Bhavya’ left. Remaining
work is completed by Satish and Veer working alternatively in ‘y’ days.
If ‘y’ is integer then find ‘Veer’ worked for how many days?
(a) 3 days (b) 4 days (c) 5 days
(d) 6 days (e) Cannot be determined

68. Abhi, Bhavya and Satish starts working together to complete work ‘M’.
After 5 days, Bhavya and Satish replaced by Neeraj and Veer. After 5
more days Abhi left the work. After 1 more day Veer left too. Neeraj
worked for total ‘x’ days. In other case Abhi and Bhavya starts working
together to complete ‘M’. After 4 days both are replaced by Veer. Veer
worked for 5 days and replaced by Satish who worked for 8 days.
Remaining work is completed by Neeraj in ‘y’ days. Find (y − x)2 ?
(a) 25 (b) 36 (c) 49
(d) 64 (e) 81

28 Adda247 Publications For any detail, mail us at


Publications@adda247.com
Cracker Book for Bank (IBPS | SBI | RRB PO | Clerk) Mains Exams

Solutions

Solutions (1-4):
Total work = 5040 units (LCM of days taken by all)
5040
Efficiency of Neeraj = = 144 units/hour
35
5040
Efficiency of Aniket = = 168 units/hour
30
5040
Efficiency of Saurabh = = 112 units/hour
45
5040
Efficiency of Gopal = = 315 units/hour
16
5040
Efficiency of Veer = = 210 units/hour
24
5040
Efficiency of Sameer = = 280 units/hour
18
7
1. (a); New efficiency of Neeraj = 144 ×
8
= 126 units/hour
2
New efficiency of Gopal = 315 ×
3
= 210 units/hour
5040 1
Required time = = 11 hours
(126+112+210) 4
3
2. (c); New efficiency of Sameer = 280 ×
4
= 210 units/hour
ATQ –
(210+210 )(y) 2
=
168(y+1) 1
420y = 336y + 336
420y – 336y = 336
y = 4 hour
Total work = 420 × 4 + 168 × 5
= 2520 units
2520 1
Neeraj will complete alone in = = 17 hours
144 2

29 Adda247 Publications For any detail, mail us at


Publications@adda247.com
Cracker Book for Bank (IBPS | SBI | RRB PO | Clerk) Mains Exams

3. (d); Divyaraj work in one hour = 144 × 1.5


= 216 units
75
Manish work in one hour = 168 ×
100
= 126 units
Let Saurabh work for n hours and Divyaraj & Manish work for (n +
11.5) hours
ATQ –
N × 112 + (216 + 126)(n + 11.75)
= 5040
112n + 342n = 5040 – 4018.5
1021.5
n=
454
n = 2.25 hours
Total time = (2.25 + 2.25 + 11.75 )
1
= 16 hours
4

4. (d); Total work completed by Neeraj, Aniket, Veer and Sameer together
in one hour
= (144 + 168 + 210 + 280) = 802 units
In Second hour total work destroyed by Saurabh & Gopal together
= −(315 + 112) = − (427)
Total work in 2 hours = 802 – 427
= 375 units
24
Total work in 24 hours = 375 ×
2
= 4500 unit
On 25th hour remaining work by Neeraj, Aniket, Veer and Sameer
together
5040 −4500 270
= = hours
802 401
270
Required time = 24 hours
401

5. (a); Total marbels in bag Y = 20


7C 7×6 21
Required probability = 20 2 = =
C2 20×19 190

30 Adda247 Publications For any detail, mail us at


Publications@adda247.com
Cracker Book for Bank (IBPS | SBI | RRB PO | Clerk) Mains Exams

6. (d); Total marbels in bag X = 18


6C 6×5 5
Required probability = 18 2 = =
C2 18×17 51

7. (c); Total marbels in bag y = 20


2C × 7C 7
1 1
Required probability = 20C =
2 95

8. (e); Total marbels in bag X = 18


3C 4C
Required probability = 18 2 + 18 2
C2 C2
3 4×3 9 1
= + = =
9×17 18×17 153 17

9. (b); Total marbels in bag Y = 20


7C × 7C × 4C 49
1 1 1
Required probability = 20C
=
3 285

10. (b); Quantity of tank P and T together


(15+15)
= 1500 × = 450 ℓ
100
Efficiency of pipe B and D
12
1500×
Pipe B = 100
= 4.5 ℓ⁄m
40
20
1500×
Pipe D = 100
= 7.5 ℓ⁄m
40
Efficiency of pipe C and E
8
1500×
Pipe C = 100
= 4 ℓ⁄m
30
15
1500 
Pipe E  100  5l / m
45
According to question
= 12X + 9(X + 15) = 450
= 21X = 450 – 135
315
=X= = 15 minutes
21
Liter of water filled by pipe B and D together
= 15 (4.5 + 7.5) = 15 × 12 = 180ℓ
31 Adda247 Publications For any detail, mail us at
Publications@adda247.com
Cracker Book for Bank (IBPS | SBI | RRB PO | Clerk) Mains Exams

11. (c); Quantity of tank S


20
= 1500 × = 300ℓ
100
New efficiency of Pipe D
300 6
= × = 9 ℓ⁄m
40 5
Efficiency of pipe C and G
8 12
1500×100 1500×100
= × 125 +
30 30
= 5 + 6 = 11 l/ m
According to question = 9X + 11 × 15 = 300
= 9X = 300 – 165 = 9X = 135
135
X= = 15 minutes
9

12. (a); Given, total quantity of all seven tank


= 1500ℓ
Quantity of bigger tank K
120
= 1500 × = 1800ℓtr
100
Efficiency of pipe B, D and G together
12 20 12
1500× 1500× 1500×
100 100 100
= + +
40 40 30
= 4.5 + 7.5 + 6 = 18 ℓ/m
15
1500×
Efficiency of pipe A = 100
= 9 ℓ⁄m
25
Efficiency of Pipe F and C
18 8
1500× 1500×
100 100
= +
60 30
= 4.5 + 4 = 8.5 ℓ/m
According to question
Tank filled in three minutes alternatively
= (18 + 9 + 8.5) = 35.5 litre
In (50 × 3) minutes = 35.5 × 50 = 1775 litre
Next by (B + D + G) = 18 ℓtr/m
After 151 minutes remain quantity of bigger tank
= 1800 – (1775 + 18) = 7 ℓtr
7
Tank K, total filled in = 151 +
9
7
= 151 minutes
9

32 Adda247 Publications For any detail, mail us at


Publications@adda247.com
Cracker Book for Bank (IBPS | SBI | RRB PO | Clerk) Mains Exams

13. (d); Total quantity of tank M = (total quantity of tank S + U together)


20 18
= 1500 × + 1500 ×
100 100
= 300 + 270 = 570ℓ
Efficiency of pipe A, C and G together
15 8 12
1500×100 1500×100 1500×100
= + +
25 30 30
= 9 + 4 + 6 = 19 ℓ/m
(A + C + G) filled tank M together in
570
= = 30 minutes
19
But according to question —
17
tank 7 minutes more due to leak
19
leak empty quantity of water in 30 minutes
17
= (A + C + G) filled in 7 minutes
19
17 150
= 19 × 7 = 19 × = 150 ℓtr
19 19
150
Efficiency of leak = = 3 ℓ⁄m
30
Leak empty tank M in
570
= = 190 minutes
3
1
= 3 hr
6

14. (b); Given, total quantity of tank = 570 ℓ


Efficiency of pipe B, D and E together
12 20 8
1500× 1500× 1500× 175
100 100 100
= + + ×
40 40 30 100
= 4.5 + 7.5 + 7 = 19 ℓ/m
Leak at a height of 2/3 from the bottom of tank
2
= 570 × = 380 ℓtr
3
380 ℓ filled without leak
380
= = 20 minutes
19
Given, man notice after 32 minutes
Tank filled in 32 minutes = (20 + 12) minutes
1
= 380 + 12 × (19 – 19 × ) = 380 + 114 = 494
2
Remain = 570 – 494 = 76 ℓ

33 Adda247 Publications For any detail, mail us at


Publications@adda247.com
Cracker Book for Bank (IBPS | SBI | RRB PO | Clerk) Mains Exams

15. (b);
Q S P
75
32000 : 24000 : 24000 ×
100
16 : 12 : 9
Let total profit = 100X
12
Q extra profit share = 100X × = 12X
100
15
S extra profit share = 100X × = 15X
100
Remaining profit = 100X – (12X + 15X)
= 73X
73X
Share of P = (16+12+9) × 9 = 65700
657X
= = 65700
37
65700×37
X= = 3700
657
Total profit = 3700 × 100 = 370000 Rs.

16. (e); Ratio of investment by Q & R = 4 : 3


⇒ 4x : 3x
Ratio of time period = 5 : 3 ⇒ 5y : 3y
Ratio of profit share between Q & R
= 4x × 5y ∶ 3x × 3y = 20xy ∶ 9xy
= 20a ∶ 9a [say]
ATQ,
20a – 9a = 100%
10010
a= = 910
11
Now 35% profit = 29a = 29 × 910
65
65% profit = 29 × 910 ×
35
65 29×910×13
% profit = = 24505
2 2×7
R’s total profit share = 24505 + 9a
= 24505 + 8190 = 32695

34 Adda247 Publications For any detail, mail us at


Publications@adda247.com
Cracker Book for Bank (IBPS | SBI | RRB PO | Clerk) Mains Exams

17. (a); Lets S joined the business after X months


According to question
32000×36 3 36
= ⇒ x = = 4 months
24000×(36 –X) 2 9
18. (b); Profit share of R = 30500 – 24300 = 16200 Rs.
Lets R invested X Rs. for 12 months
ATQ -
81000×9 24300 81000×3 3
= = =
X×12 16200 4X 2
X = 40500 Rs.

19. (c); Q : R = 8 : 9
R:S=3:2
Q:R:S=8:9:6
Total profit = 37030
Q’s share
8
× 37030 = 12880
23
R’s share
9
× 37030 = 14490
23
S’s share
6
× 37030 = 9660
23
40
20. (c); Investment of Saurabh = 40,000 × = 16,000
100
30 75
Investment of Neeraj = 40,000 × × = 9,000
100 100
Ratio of profit share of Neeraj to Saurabh

8000 8000
Saurabh’s profit = × 101 = × 101 = 5050
59+101 160

35 Adda247 Publications For any detail, mail us at


Publications@adda247.com
Cracker Book for Bank (IBPS | SBI | RRB PO | Clerk) Mains Exams
30
21. (a); Investment of Rakesh = 40,000 × = 12,000
100
Investment of Aman
30 75 100
= 40,000 × × × = 6,000
100 100 150
Ratio of profit share of Aman to Rakesh
Aman : Rakesh
6000  8  8000  2  10, 000  2 : 12000(12  x)
84000 : 12000(12  x)
7 : (12  x)
ATQ,
7 7
= ⇒ x = 4 months
12−x 8
Rakesh worked for (12-x) = 12 -4
= 8 months

30
22. (e); Investment of Sandeep = 40000 × = 12000
100
30 75
Investment of Neeraj = 40000 × × = 9000
100 100
40
Investment of Satish = 40000 × + 4000 = 20000
100
Ratio of profit share of Sandeep, Neeraj and Satish

11500
Sandeep’s profit share = (12+6+5) × 12 = Rs 6000

23. (b); Investment of Aman


30 75 100
= 40,000 × × × = 6,000
100 100 150
40
Investment of Saurabh = 40,000 × = 16,000
100
Total amount they have after 2 years
120 120 80 80
= 6000 × × + 16000 × ×
100 100 100 100
= 8,640 + 10,240 = 18,880

36 Adda247 Publications For any detail, mail us at


Publications@adda247.com
Cracker Book for Bank (IBPS | SBI | RRB PO | Clerk) Mains Exams
50
24. (b); Total quantity of in vessel P = 240 × = 120 ml
100
Quantity of Mango juice in vessel P
93.75
= 192 × = 180 ml.
100
150
Quantity of water in vessel S = 160 ×
100
= 240 ml
Quantity of Orange juice in vessel S
= 240 – 40 = 200 ml
Ratio of (Milk : Water : Orange juice : Mango juice) in vessel P
= 240 : 120 : 144 : 180 = 20 : 10 : 12 : 15
Ratio of (milk : Water : Orange juice : Mango juice) in vessel S
= 160 : 240 : 200 : 120 = 4 : 6 : 5 : 3
Total orange juice in vessel T
12 5
= 114 × (20+10+12+15) + 180 × (4+6+5+3)
= 24 + 50 = 74 ml
74
Required % = (114+180) × 100
74 25
= × 100 = 25 %
294 147

2
25. (c); Quantity of milk in vessel R = 144 × = 96 ml
3
Quantity of Orange juice in vessel Q
3
= 160 × = 120 ml
4
Total solution in vessel Q
= (160 + 80 + 120 + 210) = 570 ml
Total solution in vessel R = 1086 – 570 = 516 ml
Let mango juice and total solution in vessel R be 8n and 43 n
respectively
ATQ—
96 + 144 + 192 + 7n = 43n
36n = 432
n = 12
Total mango juice in vessel R = 12 × 7 = 84 ml
210+84 294 49
Required ratio = = =
1086 1086 181

37 Adda247 Publications For any detail, mail us at


Publications@adda247.com
Cracker Book for Bank (IBPS | SBI | RRB PO | Clerk) Mains Exams

26. (c); Minimum average is obtained when he will buy 2T₂ t- shirts.
Price for 2 shirts = 600 × 2 = Rs. 1200
He will get 4 t-shirts in this price
1200
Price/t-shirt = = Rs. 300
4
NOTE: If he buys T₁ t-shirt, price will be
90
400 × = Rs. 360
100

27. (a); 20% profit means he is selling it in


20
1000 (1 + ) = Rs. 1200
100
This amount is obtained after 25% discount
4
Hence MRP = 1200 × = Rs. 1600
3

1
11 9 8
28. (c); SP of S₂ = 1647 [1– ] = 1647 [ ]
100 9

= 183 × 8 = Rs. 1464


ATQ,
2
14
7
CP [1 + ] = Rs. 1464
100
7
CP = 1464 × = Rs. 1281
8

29. (e); Price paid for 5 pieces of T₂ is actually price paid for
2 pieces of T₂ + 1 piece of T₂.
As he will get 2 – t-shirts (T₂) free after buying two = 2 × 600 + 600
= 1800
Similarly price for 4 pieces of J₁ = 2 × 1200
1 piece of S₁ = 1 × 1200 = Rs. 1200
Total = Rs. 1800 + Rs. 2400 + Rs. 1200 = Rs. 5400

30. (d); Actual price of 4 pieces of T₂ = 4 × 600 = 2400 Rs.


Actual price of 2 pieces of T₁ = 2 × 400 = 800 rs.
Actual price of 4 pieces of J₁ = 4 × 1200 = Rs 4800
Total actual price is Rs (2400 + 800 + 4800) = Rs 8000
Price paid for 4 pieces of T₂ = 2 × 600 = Rs. 1200
38 Adda247 Publications For any detail, mail us at
Publications@adda247.com
Cracker Book for Bank (IBPS | SBI | RRB PO | Clerk) Mains Exams

90
For 2 pieces of T₁ = 400 × ( ) × 2 = Rs. 720
100
For 4 pieces of J₁ = 2 × 1200 = Rs. 2400
Discount amount obtained
= (8000 – 4320) Rs. = 3680
3680
D% = × 100 = 46%
8000

Total distance
31. (c); Average speed =
Total time
60+70+60+80+40 310
= = = 62 km/hr
5 5

32. (b); Distance travelled by A till 3 PM is = 60 + 70 + 60 = 190 km


Now note that if A is travelling a distance of 60 km in 1 hour, then
its speed is also 60 km/hr.
Also, we know that ratio of distance travelled by two objects in
same time is same as ratio of their speed.
D S
∴ A= A
DB SB
Distance travelled by B between 12:00 to 1:00 PM
60 6
= ⇒ DB =50 km
DB 5
Similarly distance travelled by B between 1:00 PM – 2:00 PM
70 14
= ⇒ DB = 55 km
DB 11
Distance travelled between 2:00 – 3:00 PM
60 6
= = DB = 70 km
DB 7
Required difference = 190 – [50 + 55 + 70]
= 15 km

33. (a); Average speed of A between 2:00 to 4:00 pm


60+80
= = 70 km/hr
2
Average speed of B between 2:00 to 3:00 pm
7
= × 60=70 km/hr
6
Similarly between 3:00 to 4:00 pm
3
= 80 × = 60 km/hr
4

39 Adda247 Publications For any detail, mail us at


Publications@adda247.com
Cracker Book for Bank (IBPS | SBI | RRB PO | Clerk) Mains Exams

Average speed of B between 2:00 to 4:00 pm


60+70
= = 65 km/hr
2
70–65
Required answer = ( ) × 100
70
1 1
= × 100 = 7 %
14 7

34. (d); Distance covered by B till 5:00 PM is


5 11 7 3 7
(60 × ) + (70 × ) + (60 × ) + (80 × ) + (40 × )
6 14 6 4 4
= 50 + 55 + 70 + 60 + 70 = 305 km
Remaining distance = 540 – 305 = 235 km
235
Required average = = 58¾ km/hr
4

35. (c); A consumes 1 litre/15 km


100
B consume 15 ×
125
4
15 × = 1 litre/12 km
5
Distance travelled by A till 2:00 PM = 60 + 70 = 130 km
130 26
Petrol consumed = = litre
15 3
Distance travelled by B till 2:00 PM = 50 + 55 = 105 km
105 35
Petrol consumed = = litre
12 4
26 35
Required Ratio = ∶
3 4
⇒ 104: 105

36. (b); Let the amount obtained by B is 41x and S.I. obtained is 16x.
Therefore, principal is 41x – 16x = 25x = Rs75000
⇒ x = 3000/–
Therefore, SI obtained 16x = 16 × 3000 = 4800/-
Using formulae
PRT 48000×100
= 48000 ⇒ T =
100 16×75000
= 4 years = 48 months
40 Adda247 Publications For any detail, mail us at
Publications@adda247.com
Cracker Book for Bank (IBPS | SBI | RRB PO | Clerk) Mains Exams

37. (d); If we assume amount of A 8x and SI obtained 3x, then we can


calculate principal of A (5x) = Rs. 80000
x = 16000
and amount obtained by A = 16000 × 8 = 128000
3
∴ amount obtained by C = × 128000 = Rs. 96000
4
Let ROI obtained by C is m%/annum and from line graph, we know
that Principal submitted by him is Rs 60000.
Then,
60000(m)×6
= Rs. (96000 – 60000) = 36000
100
m = 10%

38. (a); Time for which Vikas invested is 6 + 3 = 9 years.


80000
Sum deposit by him = = Rs. 40000
2
100000×8×5
SI obtained by D = = Rs. 40000
100
40000×R×9
∴ ROI of Vikas = = 40000
100
100 1
R= = 11 %
9 9

39. (e); Let amount obtained by E is 29y & SI obtained is 9y


∴ 29y – 9y = 20y = 50000
[∵ 50000 is principal]
⇒ y = 2500
Interest obtained = 2500 × 9 = Rs. 22500
50000×15%×T
∴ = 22500
100
22500
T= = 3 years.
500×15
Similarly,
For B principal is Rs. 75000
(41–16) = 25 units = Rs. 75000
75000
⇒ 16 units (SI) = × 16 = Rs. 48000
25
∴ time for which B invested
75000×16×T
= 48000
100
48×100
T= = 4%
75×16
Required ratio = 3 : 4
41 Adda247 Publications For any detail, mail us at
Publications@adda247.com
Cracker Book for Bank (IBPS | SBI | RRB PO | Clerk) Mains Exams

40. (c); Amount obtained by D after drawing his whole money


100000×8×5
= 100000 + = Rs. 140000
100
Now he invested 50% of this = 70000
Amount obtained after two years from this
110 2
= 70000 × ( ) = Rs. 84700
100
Total amount with him = Rs. 84700 + Rs.70000
= Rs. 154700
41. (d); Upstream distance travelled on 1st Aug.
12
= × 6000 = 720km.
100
Downstream distance travelled on 1st Aug.
16
= × 3000 = 320 km
100
Let speed of Rahul in still water be x km/hr.
ATQ.
720 320 184
= = +
x−10 x+10 3
On solving
x = 20km/hr.
∴ speed of Rahul in still water = 20Km/hr.

42. (a); Speed of Rahul in still water on 3rd Aug.


125
= × 8 = 10Km/hr.
100
ATQ.
8 12
×600 8 ×2000
100 100
= ×
8−x 3 8+x
Solving we get
x = 8/7 km/hr.
∴ seven times Speed of Current = 8 Km/hr

43. (c); Let speed of Rahul in still water be x Km/hr.


32
100
×6000
∴ 80 = ⇒ x = 27Km/hr.
x−3
Downstream speed on 5 August =27 + 3 = 30 km/h
Speed of current on 4th Aug.
50
= (27+3) × = 15 Km/hr.
100
Time to cover 128 Km up stream on 4th Aug.
128 2
= = 10 hr.
27−15 3
42 Adda247 Publications For any detail, mail us at
Publications@adda247.com
Cracker Book for Bank (IBPS | SBI | RRB PO | Clerk) Mains Exams

44. (e); Let speed of current on 2nd Aug. & 4th Aug. be 2x km/hr & 3xkm/hr.
Let speed Rahul in still water be y km/hr.
ATQ.
If speed of current on 3 August is 4 km/h then
2x = 8
Therefore
3x = 12
Now,
12 28
100
×2000 100
×6000
24× = 2x ×
y+8 y−12
7y + 56 = 24y − 288
y = 20.22 km/h
Approximately 20km/h
45. (c); On Tuesday,
Length of Rajdhani exp → 24 × 15 + 20
= 380 m
Let the speed of Rajdhani exp on Tuesday be 8 x m/s
Speed of shatabadi exp = 7x m/s
Speed of duranto exp = 10 x m/s
Let the length of tunnel be y m
ATQ,
380+y
= 12.5
8x
⇒ 380 + y = 100x
⇒ 100x – y = 380 … (i)
125
And, length of Duranto exp = 380 ×
100
= 475 m
475+y
= 11.9
10x
⇒ 475 +y = 119x
⇒ 119x – y = 475 … (ii)
From (i) and (ii)
x = 5 m/s
380+370 750
Required time = = = 10 sec
15×5 75

43 Adda247 Publications For any detail, mail us at


Publications@adda247.com
Cracker Book for Bank (IBPS | SBI | RRB PO | Clerk) Mains Exams

46. (a); Length of Rajdhani on Sunday


= 29 × 15 + 20 = 455m
Length of Duranto exp = 345 m
Let the speed of Rajdhani, shatabadi and Duranto exp be 6x m/s, 5x
m/s and 4x m/s respectively.
ATQ,
455+345 80
= ⇒ x = 9 m/s
10x 9
80
Length of Shatabadi = 455× = 364 m
100
364+345
Required time = ≈ 79 sec
9

47. (d); Speed of Shatabadi exp on Friday


= 150 km/hr
Speed of Duranto exp on Friday
= 120 km/hr
Distance travelled by Shatabadi exp in 3 hours
= 150 + 150 × 1.10 + 150 × 1.1 × 1.2
= 513 km
Distance travelled by Duranto exp in 3 hours
= 120 + 120 × 1.2 + 120 × 1.2 × 1.25
= 444 km
Total distance between P and Q
= 513 + 444 = 957 km
48. (b); On Monday,
Length of Rajdhani exp. = 15 × 17 + 20
= 275 m
Length of Duranto exp. = 275 × 1.28
= 352 m
Let the speed of Rajdhani, shatabadi and Duranto
exp. on Monday be 3x m/s, 4x m/s, 5x m/s respectively.
ATQ,
352+352
= 12.8 ⇒ 704 = 64x ⇒ x = 11
5x
7x 3 18
Required difference = 5x − = ×11×
2 2 5
= 59.4 km/hr

44 Adda247 Publications For any detail, mail us at


Publications@adda247.com
Cracker Book for Bank (IBPS | SBI | RRB PO | Clerk) Mains Exams
18
49. (b); Length of train B = × 1600 = 288m
100
12
Length of Train D = × 1600 = 192m
100
8
Length of Train F = × 1600 = 128m
100
5
Speed of train B on Monday = 97.2 ×
18
= 27 m/sec
Speed of train ‘B’ on Wednesday
27
= × 5 = 45 m/sec
3
ATQ,
288 + 192 = (45 +y)×6
where y is the speed of train ‘D’ on Wednesday
⇒ y = 80 – 45 = 35 m/sec
35
Speed of train ‘D’ on Monday = ×4
7
= 20 m/sec
192+128
Time required to cross train F =
20+20
= 8 seconds

50. (e); Let speed of train ‘C’ on Monday, Tuesday and Wednesday be 4x, 6x
and 5x respectively.
Train ‘C’ travel 5 hours on Monday and 15 hours on Tuesday.
∴ Total distance = 5 × 4x + 15 × 6x = 110x
On the same day i.e, Tuesday, train ‘C’ start from Kanpur. It travels
6 hours on Tuesday and 17.8 hours on Wednesday.
∴ total distance travel = 6 × 6x + 17.8 × 5x
= 36x + 89x = 125x
ATQ, 125x = 110x + 180
⇒ 15x = 180 ⇒ x = 12
speed of train ‘C’ on Monday
40
= 12 × 4 = 48 km/hour = m/sec
3
24
Length of train ‘C’ = × 1600 = 384
100
384
Required time = × 3 = 28.8 sec
40

45 Adda247 Publications For any detail, mail us at


Publications@adda247.com
Cracker Book for Bank (IBPS | SBI | RRB PO | Clerk) Mains Exams

51. (a); Let, speed of train ‘A’ and train ‘C’ on Monday be ‘4x’ and ‘4y’
respectively
ATQ,
900 900
2.5 = −
4x 4y
1 1
2.5 = 225 [ − ]
x y
xy = 90 (y – x)
16
length of train ‘A’ = × 1600 = 256
100
256+128
speed of train ‘A’ on Tuesday =
12.8
384
= = 30 m/sec
12.8
⇒ Speed of train ‘A’ on Monday
30
= × 2 = 20 m/sec = 72 km/hr
3
⇒ 4x = 72 ⇒ x = 18
xy = 90(y – x)
y = 5(y – 18)
⇒ y = 22.5
Speed of train ‘C’ on Monday = 4y
= 4 × 22.5 = 90 km/hr
90
Speed of train ‘C’ on Tuesday = × 6
4
= 135 km/hr = 37.5 m/sec
24
Length of train ‘C’ = × 1600 = 384
100
384+66
Required time = = 12 seconds
37.5

22
52. (a); Length of train ‘E’ = × 1600 = 352
100
8
Length of train ‘F’ = × 1600 = 128
100
Let speed of train ‘E’ and train ‘F’ on Monday be 6x and 4y
respectively.
6x 3 x 1
⇒ = ⇒ =
4y 2 y 1
Let speed of train ‘E’ on Tuesday = 9x
So speed of train ‘F’ on Tuesday = 5y = 5x
ATQ,
46 Adda247 Publications For any detail, mail us at
Publications@adda247.com
Cracker Book for Bank (IBPS | SBI | RRB PO | Clerk) Mains Exams
352+128
9x – 5x = = 20 ⇒ 4x = 20
24
⇒x=5
Speed of train ‘E’ on Wednesday = 5 × 5
= 25 m/sec
Speed of train ‘F’ on Wednesday = 3 × 5
= 15 m/sec
352+128 480
Required time = =
25−15 10
= 48 seconds

53. (b); Let, speed of train ‘B’ on Monday, Tuesday & Wednesday be 3x, 4x
& 5x respectively.
And speed of train ‘D’ on Monday, Tuesday & Wednesday be 4y. 4y
& 7y respectively.
18
Length of train ‘B’ = × 1600 = 288
100
12
Length of train ‘D’ = × 1600 = 192
100
ATQ,
288
3x 1 3 4y 1
192 = ⇒ × =
1 2 3x 1
4y
y 1
⇒ = ⇒ x = 2y
x 2
288 57.6
Time taken by train ‘B’ on Wednesday to cross pole = =
5x x
192 96
Time taken by train ‘D’ on Monday to cross a pole = =
4y x
96 57.6
( − )×100 38.4
x x
Required % = 96 = × 100 = 40%
96
x

54. (c); Let, marked price of both articles be 600x


72
S.P. of article X = 600x × = 432x
100
85
S.P. of article Y = 600x × = 510x
100
ATQ,
510x − 432x = 312 ⇒ 78x = 312
⇒x=4

47 Adda247 Publications For any detail, mail us at


Publications@adda247.com
Cracker Book for Bank (IBPS | SBI | RRB PO | Clerk) Mains Exams
432×4
Cost price of article ′X ′ = × 100 = 1440
120
Cost price of article ‘Y’ = 1520 × 2 – 1440
= 3040 – 1440 = 1600
Profit % earned on selling article Y
510×4−1600 440
= × 100 = × 100 = 27.5%
1600 1600

55. (e); Let M.P. of each article sold by each seller be 800x
800x×67.5
Cost price of article Y sold by B =
120
= 450x
Cost price of article Y sold by E
800x×72
= × 3 = 432x
4×100
800x×85
Cost price of article X sold by C =
125
= 544x
800x×54
Cost price of article X sold by E =
135
= 320x
ATQ,
(450x + 432x) – 544x – 320x = 216
882x – 864x = 216
216
⇒x= = 12
18
M.P. of each article = 800 × 12 = 9600
9600×76
Cost price of article Y sold by C =
160
= Rs 4560

56. (b); Let M.P. of each article be 400x


400x×73.5
Cost price of article X sold by B = = 175x
168
400x×63
Cost price of article Y sold by D = = 210x
120
175x 5
Required ratio = =
210x 6

48 Adda247 Publications For any detail, mail us at


Publications@adda247.com
Cracker Book for Bank (IBPS | SBI | RRB PO | Clerk) Mains Exams

57. (d); Let M.P. of each article = 400x


400x×54
S.P. of article X sold by E = = 216x
100
Cost price of article Y sold by E = 216x
400x×72
Selling price of article Y sold by E =
100
= 288x
288x−216x
Profit % = × 100
216x
72x 1
= × 100 = 33 %
216x 3

58. (b); Let, M.P. of each article be 400x


400x
C.P. of article X sold by A = = 200x
2
120
S.P of article X sold by A = 200x ×
100
= 240x
85
SP of article Y sold of A = 400x ×
100
= 340x
Let, CP of article Y sold by A = y
ATQ,
125
(200x + y) × = 240x + 340x
100
580x
(200x + y) = × 4 ⇒ y = 264x
5
264x−240x 1
Required % = × 100 = 9 %
264x 11

59. (b); Distance travel by car P = 1500 + 3000


= 4500 km
4500
Total Time taken = = 112.5 hour
40
Time taken by car R from Delhi to City A
1000 50
= = hours
60 3
50 287.5
Time taken from city A to city B = 112.5– =
3 3
Distance from between City A to city B
287.5
= × 60 = 5750 km
3

49 Adda247 Publications For any detail, mail us at


Publications@adda247.com
Cracker Book for Bank (IBPS | SBI | RRB PO | Clerk) Mains Exams

60. (a);

Distance between city A and city E


= √10002 + 15002
= √1000000 + 2250000
= √3250000 = 500√13km
Approximate time taken by car ‘T’
500√13
= ≈ 24 hours
75

61. (c); Let speeds of car Q and car S be x and y respectively.


ATQ—
3000 2000 x 3
⇒ = ⇒ =
x y y 2
Let speed of car Q and car S be 3a and 2a respectively
Distance between city B and city D
= 1500 km
1500 300
Time taken to cross each other = =
5a a
3000 1000
Time taken by car Q to reach city B from Delhi = =
3a a
300×100
Required% = = 30%
1000

1500×3
62. (d); = 900 km
5
Time taken by car R to cover this distance
900
= = 15 hour
60
Let initial speed of car S = x km/hr
So, ATQ
6x + 15(2x) = 900
6x + 30x = 900
36x = 900 ⇒ x = 25 km/hr
50 Adda247 Publications For any detail, mail us at
Publications@adda247.com
Cracker Book for Bank (IBPS | SBI | RRB PO | Clerk) Mains Exams

63. (e); Distance between Delhi and city A = 1000 km


Distance covered by Car Q before first meeting = 1200 km
Distance covered by Car P before first meeting = 800 km
Speed of car P = 40 km/hr
800
⇒ Time for first meeting = = 20hr
40
1200
Speed of car Q = = 60km/hr
20
When car P reaches city ‘A’ distance covered by car ‘Q’
200
= × 60 = 300km
40
500 25
Time taken by car ‘Q’ to reach Delhi = = hr
60 3
25
Distance covered by car ‘P’ in hour
3
25 1000
= × 40 = km
3 3
Distance between car ‘Q’ and car ‘P’
1000 2000
= 1000 − =
3 3
2000
3 20
Time to meet = = hour
60+40 3
200 25 20
Total time = + + = 20 hours
40 3 3

64. (d); Total work = 24 × (4a + 2a + 4a) = 240a


Bhavya worked for10 days
⇒ Work done by ‘Bhavya’ = 40a
40a
⇒ Time taken by ‘Neeraj’ = = 20 days
2a
⇒ ‘Neeraj’ and ‘Abhi’ worked for 20 days
And ‘Veer’ worked for 8 days
20 × 3a + 10 × 4a + 20 × 2a + y × 4a + 8 × 6a = 240a
⇒ 118a + y × 4a = 240a
240a−188a 52a
⇒y= ⇒ y= = 13 days
4a 4a

65. (b); 5a × (A + 42) = 8a × (A + 15)


⇒ 5A + 210 = 8A + 120
⇒ A = 30
Total work = 5a (30 + 42) = 5a(72) = 360a

51 Adda247 Publications For any detail, mail us at


Publications@adda247.com
Cracker Book for Bank (IBPS | SBI | RRB PO | Clerk) Mains Exams

Let, Abhi, Bhavya and Veer worked for x, y and z days


ATQ,
3x : 4y : 6z = 1 : 2 : 3
Ratio between working days of Abhi, Bhavya and Veer
⇒x:y:z=2:3:3
Ratio between working days of Neeraj, Satish and Veer
=2:2:1
Let working days of Abhi, Bhavya, Neeraj, Satish and Veer be 2m,
3m, 6m and 6m and 3m days respectively
ATQ,
3a × 2m + 4a × 3m + 2a × 6m + 4a × 6m + 6a × 3m = 360a
⇒ 72am = 360a ⇒ m = 5
Bhavya worked for 5 × 3 = 15 days

66. (b); Veer, Bhavya and Satish worked for 6 days, 9 days and 9 days
respectively. Total 60% of work completed by them
⇒ 60% of work = 4a × 9 + 4a × 9 + 6a × 6
= 108a
108a
⇒ Total work = × 5= 180a
3
‘Abhi’ worked for (9 + x) days & ‘Neeraj’ worked for (9+ x + z) days
and completed 40% of work
⇒ 72a = 3a (9 + x) + 2a(9 + x+ z)
72 = 27 + 3x + 18 + 2x + 2z
27 = 5x +2z … (i)
And,
z–x=3 … (ii)
On solving (i) & (ii)
z = 6, x= 3
‘Abhi’ worked for (9 +3) = 12 days
‘Neeraj’ worked for (9 + x+ z) = 9 + 3 + 6
= 18
18−12
Required % = × 100
12
6
= × 100 = 50%
12

52 Adda247 Publications For any detail, mail us at


Publications@adda247.com
Cracker Book for Bank (IBPS | SBI | RRB PO | Clerk) Mains Exams

67. (b); There are two possibilities


First – Satish worked first
Second – Veer worked first
When satish worked first
remaining work
→ 120a – 10(3a+4a) – 7(2a) = 36a
Satish and Veer worked in 3 days = 30a
4th day satish’s worked = 4a
Veer’s worked for = 1/3 day
Now ‘y’ cannot be in fraction
2nd case-
When Veer worked first-
Veer and Satish worked for first 3 day
= 30a
Remaining work = 6/6= 1 day
So Veer worked for 4 days.

68. (d); In first case


ATQ,
10 5 x 5 6
+ + + + =1
40 30 60 30 20
x 53 x 7
⇒ + =1 ⇒ = ⇒ x=7
60 60 60 60
In second case
ATQ,
4 4 y 8 5 y 3
+ + + + =1 ⇒ + =1
40 30 60 30 20 60 4
y 1
⇒ = ⇒ y = 15
60 4
(y − x) = (15 − 7)2 = 82 = 64
2

53 Adda247 Publications For any detail, mail us at


Publications@adda247.com
Cracker Book for Bank (IBPS | SBI | RRB PO | Clerk) Mains Exams

1 Adda247 Publications For any detail, mail us at


Publications@adda247.com
Cracker Book for Bank (IBPS | SBI | RRB PO | Clerk) Mains Exams

Chapter
Data Sufficiency
15
BEST APPROACH TO SOLVE THE QUESTIONS

Data Sufficiency
These questions formerly asked in only MAINS Examination but now-a-
days these questions were seen in PRELIMINARY Examinations too. So,
student can’t deny the importance of DATA SUFFICIENY Questions.

In these questions some data are given in form of statements and one
question is asked. Student must choose that statement/statements which
is/are sufficient to solve the given question.

They consist of two types. In first type, two statements are given while in
second type, three statements are given, and student is expected to choose
the statement/statements among the given statements which is/are
required to solve the question. Five questions of each type are given below
to practice this topic.

Example1.
What is the length of train ‘A’?
A. Train A takes 8 second to cross a pole
B. Train A cross train B coming from opposite side in 8 second, speed of
train B is 25 m/sec and length of train B is half of train A.
C. Train A takes 10 sec to cross two pole 100 m apart.
(a) Only A and B together
(b) Only A and C together
(c) Any two of them
(d) Either B alone or A and C together
(e) All statements are required

2 Adda247 Publications For any detail, mail us at


Publications@adda247.com
Cracker Book for Bank (IBPS | SBI | RRB PO | Clerk) Mains Exams

Solution:

1. (c); Let length of train A = L


Speed of train A = x
L
From A = = 8
x
L+0.5L
From B = =8
x+25
L+100
From C = = 10
x
From any of these two-statement question be solved.

Example 2:
In how many days B and C together can complete work?
A. Per day efficiency of A, B and C is in the ratio 3 : 2 : 4
B. A and B together can complete the work in 7.2 days
C. Time taken by A, B and C alone to complete the work is in the ratio
4:6:3.
(a) Only A and B together
(b) Either A and B or B and C
(c) Any two of them
(d) Either B alone or A and C together
(e) All statements are required

Solution:
2. (b); From A per day efficiency = 3 : 2 : 4
⇒ ratio of time taken by A, B and C alone to complete work = 4 : 6 :
3 same as (C)
From B) time taken by A and B together = 7.2 days
Either B and A or B and C questions can be solved

Example3:
What is the base radius of the cone?
A. Total surface area of the cone is 462 cm²

3 Adda247 Publications For any detail, mail us at


Publications@adda247.com
Cracker Book for Bank (IBPS | SBI | RRB PO | Clerk) Mains Exams

1078√3
B. Volume of the given cone is 𝑐𝑚3 and height = 7√3 cm
3
C. Ratio of height and radius of the given cone is √3 : 1
(a) Only A and B together
(b) Only A and C together
(c) Any two of them
(d) Either B alone or A and C together
(e) All statements are required

Solution:
3. (d) From A) T.S.A = πrℓ + πr² = 462 cm2
1078√3 h
From B) V = = πr 2
3 3
And h = 7√3 cm
From C) h : r = √3 ∶ 1
Either B alone or A and C together, question can be solved.

Example 4:
Find the share of Satish in profit after one year among Satish, veer and
Yogesh
A. Investment of Veer, Satish and Yogesh is in the ratio 6: 8: 9 and share of
Yogesh in profit is Rs.900
B. Satish and Veer invested Rs.8000 and Rs.6000 for 9 months and one
year respectively. Difference b/w profit earned by Satish and Veer is
Zero.
C. Satish, Veer and Yogesh invested for 9 months, 1 year and 8 months
respectively and share of Veer in profit is 900.
(a) Either A and B or B and C
(b) Either A and B or A and C
(c) Any two of them
(d) Either B alone or A and C together
(e) Either A and C or B and C

4 Adda247 Publications For any detail, mail us at


Publications@adda247.com
Cracker Book for Bank (IBPS | SBI | RRB PO | Clerk) Mains Exams

Solution:
4. (e); Veer : Satish : Yogesh
From A and C) 6×12 : 8×9 : 9×8
72 : 72 : 72
1 : 1 : 1
Share of Veer = Share of Satish Rs 900
Satish : Veer
From (B and C) 8000×9 : 6000×12
1 : 1
Share of Veer share of Satish = Rs 900
Either A and C or B and C question can be solved.

Example 5:
What will be the probability of choosing two fresh eggs simultaneously
from tray A given that there are only two trays i.e. tray A and tray B?
A. Tray B contains 30 Eggs in which fresh and rotten eggs in the ratio 7 : 3.
Ratio of fresh eggs in Tray A and Tray B is 2 : 3
B. Tray A contains 20 Eggs in total out of which 14 are fresh and rest are
rotten.
C. Tray B contains 30 Eggs and Tray A contains 20 Eggs respectively. Ratio
of Rotten eggs and fresh eggs in Tray A is 7 : 3
(a) Either A or B
(b) Either B or C
(c) Any one of them
(d) A alone
(e) All statements are required

Solution:
5. (b); From A) Tray B = 30
Rotten eggs in Tray B = 9 eggs & fresh eggs in Tray B = 21 eggs
Tray A, Fresh eggs = 14 eggs
From B) Tray A = 20 eggs
Fresh eggs in Tray A = 14 Rotten eggs in Tray A = 6
From C)
Tray B = 30 eggs
Tray A = 20 Eggs
Rotten eggs in Tray A =6 eggs
Fresh eggs in Tray A = 14 eggs
From Either B or C question can be solved.
5 Adda247 Publications For any detail, mail us at
Publications@adda247.com
Cracker Book for Bank (IBPS | SBI | RRB PO | Clerk) Mains Exams

Practice Exercise Based on new Pattern

Directions (1-5): Given below in each question there two statements (I)
and (II). You have to determine, which statement is sufficient to give the
answer of question. Also there are five alternatives given, you have choose
one alternative as your answer of the questions:

1. What will perimeter of smaller rectangle?


I. Ratio between length of smaller and larger rectangle is 4 : 5 and
breadth of both rectangle is equal. Difference between Perimeter of
both rectangle is 8 cm.
II. Breadth of both rectangle is equal to side of square, whose area is
196 cm2 .
(a) Only statement I is sufficient
(b) Only statement II is sufficient
(c) Statement I and II both together sufficient
(d) Either statement I or Statement II alone sufficient
(e) Neither statement I or statement II sufficient

2. How many Students in college?


I. Ratio between girls to boy is 9 : 11.
II. Out of total girls in the college 20% are belongs to below eighteen
years age group. Total girls belongs to below eighteen years age
group are 9% of total students in college
(a) Only statement I is sufficient
(b) Only statement II is sufficient
(c) Statement I and II both together sufficient
(d) Either statement I or Statement II alone sufficient
(e) Neither statement I or statement II sufficient

3. What was profit shopkeeper made on article?


I. Shopkeeper sold article on 5% discount in the Rs. of 3800.
II. If shopkeeper sold article on marked price, he would made a profit
of 25%.
6 Adda247 Publications For any detail, mail us at
Publications@adda247.com
Cracker Book for Bank (IBPS | SBI | RRB PO | Clerk) Mains Exams

(a) Only statement I is sufficient


(b) Only statement II is sufficient
(c) Statement I and II both together sufficient
(d) Either statement I or Statement II alone sufficient
(e) either statement I or statement II sufficient

4. What is speed of boat?


I. Speed of boat in still water is two times more that speed of current.
II. Boat takes equal time to cover a distance downstream to 50% of
that distance upstream.
(a) Only statement I is sufficient
(b) Only statement II is sufficient
(c) Statement I and II both
(d) Either statement I or Statement II alone sufficient
(e) Neither statement I or statement II sufficient

5. Find the amount invested at the rate of 10%?


I. Total amount of Rs. 4500 invested in two different parts at the rate
of 20% p.a. and 10% p.a. for two years. Simple Interest obtained
from both parts are equal.
II. A man invested an amount in two schemes A and B in the ratio of 2 :
1 respectively. Scheme A offered simple interest at the rate of 10%
p.a. and Scheme B offered compound interest at the rate of 20% p.a.
and man got a total interest of Rs. 1260 after two years from both
scheme. Amount invested on SI is same as amount invested at the
rate of 10% in statement I?
(a) Only statement I is sufficient
(b) Only statement II is sufficient
(c) Statement I and II both
(d) Either statement I or Statement II alone sufficient
(e) Neither statement I or statement II sufficient

7 Adda247 Publications For any detail, mail us at


Publications@adda247.com
Cracker Book for Bank (IBPS | SBI | RRB PO | Clerk) Mains Exams

Directions (6-10): The following questions are accompanied by two


statements (I) and (II). You have to determine which statements(s) is/are
sufficient/necessary to answer the questions.
(a) Statement (I) alone is sufficient to answer the question but statement
(II) alone is not sufficient to answer the questions.
(b) Statement (II) alone is sufficient to answer the question but statement
(I) alone is not sufficient to answer the question.
(c) Both the statements taken together are necessary to answer the
questions, but neither of the statements alone is sufficient to answer
the question.
(d) Either statement (I) or statement (II) by itself is sufficient to answer
the question.
(e) Statements (I) and (II) taken together are not sufficient to answer the
question.

6. Ratio between length of two trains is 4 : 3. What will be difference


between lengths of both trains?
I. Speed of larger trains and smaller train is 72 km/hr and 90 km/hr
28
respectively. Both trains cross each other in 𝑠𝑒𝑐, when running
3
in opposite direction.
II. Speed of smaller train is 90 km/hr and it can cross a pole in 7.2 sec.

7. There are three men P, Q and R. Find the difference between time taken
by P & Q together to complete a task and time taken by Q & R together
to complete the same task?
I. ‘R’ takes twice as much time as ‘Q’ and thrice as much time as ‘P’
takes alone.
II. If they all three works together work will be completed in 4 days.

8. Satish sold an article to Ayush at 20% profit. If Ayush purchased article


from Satish in Rs. 1440, then find the profit percentage of Veer if
Satish bought this article form Veer?
I. Veer sold the article on Rs. 240 more than its cost price to Satish.
II. If Veer sold article to Ayush on same price as Satish sold to Ayush,
then he made overall profit of 50%.

8 Adda247 Publications For any detail, mail us at


Publications@adda247.com
Cracker Book for Bank (IBPS | SBI | RRB PO | Clerk) Mains Exams

9. If x : y = 11 : 9 and y : z = 3 : 4, then find (x + y) – 1.5z = ?


I. Average of all three is two more than average of x and y.
1 1
II. Sum of 9 % of x and 11 % of y is equal to (36)0.5.
11 9

10. Ratio between length and breadth of rectangle ‘X’ is 7 : 4. Find area of a
square ‘Y’?
I. Length of rectangle ‘X’ is two times of radius of circle, whose area is
616 cm2.
II. Perimeter of rectangle ‘X’ is 20 cm more than perimeter of square
‘Y’.

Directions (11-14): The following questions are accompanied by three


statements (A) or (I) and (B) or (II), You have to determine which
statement(s) is/are sufficient/necessary to answer the questions.

11. What is the minimum passing percentage in a test?


I. Raman scored 25% marks in the test and Sunil scored 288 marks
which is 128 more than that of Raman.
II. Raman scored 64 marks less than the minimum passing marks.
(a) Both I and II together are not sufficient
(b) Both I and II together are needed.
(c) Only I alone is sufficient.
(d) Only II alone is sufficient.
(e) Either I or II alone is sufficient

12. Whose body weight is second highest among the five boys Arun, Vinay,
Suraj, Raju and Pratap?
I. Average weight of Arun, Suraj and Vinay is 68 kg and average
weight of Raju and Pratap is 72 kg. Also Suraj is 78 kg. Raju is 68 kg
and Vinay is 46 kg.
II. Average weight of Arun, Suraj, Vinay and Raju is 68 kg and also
Suraj is 78 kg. Raju is 68 kg and Vinay is 46 kg. All of them have
different weights.
(a) Both I and II together are not sufficient
(b) Both I and II together are needed.
(c) Only I alone is sufficient.
(d) Only II alone is sufficient.
(e) Either I or II alone is sufficient
9 Adda247 Publications For any detail, mail us at
Publications@adda247.com
Cracker Book for Bank (IBPS | SBI | RRB PO | Clerk) Mains Exams

13. What is the population of the city A?


I. The ratio of the population of males and females in city A is 27 : 23
and the difference between their population is 100000.
II. The population of city A is 80% of that of city B. The difference
between populations of city A and city B is 312500.
(a) Both I and II together are not sufficient
(b) Both I and II together are needed.
(c) Only I alone is sufficient.
(d) Only II alone is sufficient.
(e) Either I or II alone is sufficient

14. How many students did participate in Singing?


I. The students who participated in dancing were 150% more than
that who participated in Singing.
II. 150 students participated in dancing.
(a) Both I and II together are not sufficient
(b) Both I and II together are needed.
(c) Only I alone is sufficient.
(d) Only II alone is sufficient.
(e) Either I or II alone is sufficient

Directions (15-19): The following questions are accompanied by two


statements A and B. You have to determine which statements(s) is/are
sufficient/necessary to answer the questions.
(a) Statement A alone is sufficient to answer the question but statement B
alone is not sufficient to answer the questions.
(b) Statement B alone is sufficient to answer the question but statement A
alone is not sufficient to answer the question.
(c) Both the statements taken together are necessary to answer the
questions, but neither of the statements alone is sufficient to answer
the question.
(d) Either statement A or statement B by itself is sufficient to answer the
question.
(e) Statements A and B taken together are not sufficient to answer the
question.
10 Adda247 Publications For any detail, mail us at
Publications@adda247.com
Cracker Book for Bank (IBPS | SBI | RRB PO | Clerk) Mains Exams

15. An equilateral triangle is inscribed in a circle. What will be the


difference between the area of circle and area of triangle?
(A) Radius of circle is given
(B) Sum of the perimeter of triangle and circle is given.

16. Is ‘n’ even or odd, if n is a natural number.


(A) 2n + 1 is divisible by 3
(B) 4n − 1 is divisible by 3

17. Five positive naturals numbers are given which may or may not be
equal the sum of these numbers.
(A) If numbers are arranged is ascending order then median is 18
while sum of first and last number and second and fourth number
is equal
(B) Median of these numbers is 18 while mode of these numbers is 12
and 24.

18. ‘X’ mark up an article 50% above its cost price. Find the cost price of
article.
(A) ‘X’ gave Rs 60 discount on mark price and earn 20% profit.
(B) If ‘X’ gave two successive discounts of 10% each then he will earn
Rs 43 as profit.

19. Amit tells truth 2 times out of x times while Ankush tells truth 5 times
out of ‘y’ times. Find the product of ‘x’ and ‘y’
(A) Probability that Amit and Ankush contradict with each other on
facts is 50%
(B) Probability that both tells truth is 20%

Directions (20-24): The following questions are accompanied by two


statements (A) and (B). You have to determine which statements(s) is/are
sufficient/necessary to answer the questions.

11 Adda247 Publications For any detail, mail us at


Publications@adda247.com
Cracker Book for Bank (IBPS | SBI | RRB PO | Clerk) Mains Exams

20. O is center, find area of shaded region

A. Length of AO is given.
B. AC is given in multiple of radius of semicircle.
(a) Only A
(b) Only B
(c) Either Only A or Only B
(d) A and B together
(e) A and B together are not sufficient

21. In a box three type of balls are there, Black, Red and White. If no. of
white balls is given then find out the probability of getting one white
ball.
A. Probability of getting one Red ball is given.
B. Probability of getting one black ball is given.
(a) Only A
(b) Only B
(c) Either Only A or Only B
(d) A and B together
(e) A and B together are not sufficient

22. What is the volume of the sphere?


A. Surface area of hemisphere is equal to the total surface area of the
cylinder having radius and height in ratio 3 : 4.
B. When we cut sphere into two hemi-sphere then total surface area
is equal to the area of a circle whose radius is 21 cm.
(a) Only A
(b) Only B
(c) Either Only A or Only B
(d) A and B together
(e) A and B together are not sufficient

12 Adda247 Publications For any detail, mail us at


Publications@adda247.com
Cracker Book for Bank (IBPS | SBI | RRB PO | Clerk) Mains Exams

23. PR is diameter of circle. Find, ∠QPO – ∠SRO = ?

A. ∠SPO = 40°
B. ∠PQR + ∠QRO = 120°
(a) Only A
(b) Only B
(c) Either Only A or Only B
(d) A and B together
(e) A and B together are not sufficient

24. A shopkeeper gets a loss of 70 Rs. when he sold an article at 20%


discount on M.P. Find cost price of Article.
A. % of mark up above cost price is equal to % discount given on M.P.
B. When no discount is given, article sold at profit of 350 Rs.
(a) Only A
(b) Only B
(c) Either Only A or Only B
(d) A and B together
(e) A and B together are not sufficient
Directions (25-29): The following questions are accompanied by three
statements (I), (II), and (III). You have to determine which statement(s)
is/are sufficient /necessary to answer the questions

25. What is distance between A and B ?


I. Two persons Amit and Abhi started simultaneously from A to B with
their speed in ratio 4 : 5.
13 Adda247 Publications For any detail, mail us at
Publications@adda247.com
Cracker Book for Bank (IBPS | SBI | RRB PO | Clerk) Mains Exams

II. Abhi reached reached Q one hour earlier than Amit.


III. Difference between speed of Amit and Abhi is 20 km/hr.
(a) Only I and II.
(b) Only II and III
(c) All I, II and III
(d) Cannot be answered even including all three statement
(e) None of these

26. What is the area of rectangle ?


I. If ratio of length and breadth of the rectangle is 3 : 2.
II. Circumference of a circle is 440 m and breadth of rectangle is 1/7 th
of diameter of circle.
III. If length is 50% more than breadth.
(a) Only III
(b) Only I and II or II and III.
(c) Only II
(d) All I, II and III
(e) None of these

27. How many students failed in class 11th ?


I. 400 Students passed in class 11th.
II. No. of students failed in class 11th is 20% of those failed in class 12th.
III. Ratio of student appeared to that of failed in class 11th is 5 : 3.
(a) Only I and III
(b) Only II
(c) Only I and II
(d) All I, II and III
(e) Cannot be answered even including all three statement

28. What is the rate of interest on some amount?


I. S.I. accrued in two years on same amount at same rate of interest is
Rs. 44000.
II. The amount after some years on S.I. is Rs. 154000.
III. Difference between the C.I. and S.I. earned in two years on the same
amount invested is Rs. 120.
14 Adda247 Publications For any detail, mail us at
Publications@adda247.com
Cracker Book for Bank (IBPS | SBI | RRB PO | Clerk) Mains Exams

(a) Only I and III


(b) Only III
(c) Only II and III
(d) Cannot be answered even including all statement
(e) None of these

29. What is the sum of two number?


I. The bigger no. is 6 more than the smaller no.
II. 40% of smaller no. is equal to 30% of bigger no.
III. The ratio b/w half of the bigger no. & one-third of smaller no. is 2 :
1.
(a) Only II & III
(b) Only I & II
(c) Any two of the three statement
(d) All statement is required
(e) I and II or I and III

Directions (30-33): In the following questions three statements either A, B


and C or I, II and III are given. You have to use your knowledge of
mathematics to answer which statement(s) is/are sufficient to answer the
question.
30. What is the cost price of wrist watch?
A. Shopkeeper gives 20% discount on a speaker.
B. The marked price of wrist watch is 25% more than that of speaker.
C. The shopkeeper earns a profit of 10% after selling the speaker.
(a) Any two of them are sufficient
(b) All the three statements are required
(c) Only C is sufficient.
(d) Even using all statements, answer cannot be found
(e) Only A is sufficient
31. What is the total quantity of milk in final mixture of milk and water
after adding some milk ?
A. 68 liters of initial mixture has ratio of milk and water as 11:6.
B. A certain amount of milk is added to this mixture to make the ratio
of water to milk as 6:13.
C. Selling the mixture at a certain rate, 35 % profit is obtained.
15 Adda247 Publications For any detail, mail us at
Publications@adda247.com
Cracker Book for Bank (IBPS | SBI | RRB PO | Clerk) Mains Exams

(a) Any two of them is sufficient


(b) A and B together are sufficient
(c) All the three statements are required
(d) Answer cannot be found even using all the three statements
(e) Only B and C are required

32. The speed of a train A and time taken by it to cross a tunnel is known.
Find length of tunnel.
A. Another train B is running in opposite direction to A with a speed
40% more than A.
B. Train B crosses a platform X and a pole in 24 sec and 8 sec
respectively before crossing the train A and tunnel.
C. The ratio of length of train A and platform X is 3 : 5.
(a) Only A and B are sufficient.
(b) Only B and C are sufficient
(c) Even using all the three statements answer cannot be found.
(d) All the three statements are required.
(e) Only B is sufficient

33. In how many ways 2 green balls can be chosen from box ‘A’ which
contains red, green and black balls?
A. Ratio of total number of balls in boxes A and B is 2 : 3 and box A
contains 5 green balls.
B. Total balls in box B is 18 while ratio of red and black balls in box A
is 3:4.
C. Box ‘B’ contains 5 red 7 black and 6 green balls only while ratio of
red and green balls in box ‘A’ is 3:5.
(a) Statement A and either B or C are sufficient
(b) Any two statements are sufficient
(c) All the three statements are required
(d) Statement B and either A or C are sufficient
(e) None of these

Directions (34-38): The following questions are accompanied by three


statements (I), (II), and (III). You have to determine which statements(s)
is/are sufficient/necessary to answer the questions.

16 Adda247 Publications For any detail, mail us at


Publications@adda247.com
Cracker Book for Bank (IBPS | SBI | RRB PO | Clerk) Mains Exams

34. Making a new t-shirt, consists of two parts first is cutting of cloth and
second is stitching the cloth. In how many days Nitin and Vishal
together can make 100 t-shirts.
I. Nitin can cut cloth for 20 t-shirts in a day and he is 20% slow as
that of Vishal in cutting clothes and 25% faster than Vishal for
stitching the cloth.
II. Time taken by Nitin alone to make 60 t-shirts is 9 days.
III. The ratio of days taken by Vishal to cut cloth for 50 t-shirts and to
stich 100 t-shirts is 4 : 25.
(a) Only I and III together or only II and III together
(b) All I, II and III together
(c) Any two of the three
(d) Only I and III together
(e) Either from I & II or I & III

35. Ratio of age of Aashish & Kullu is 4 : 5 and that of Nikhil and Yash is 7 :
8. What is their average age after 4 years.
I. Age difference of Nikhil and Aashish is 18 years.
II. Ratio of 12 years ago age of Aashish & Nikhil is 2 : 5 and Kullu &
Yash is 1 : 2.
III. 12 years later Aashish will be 150% of his present age.
(a) Any two of I, II and III are required.
(b) Only I and II are required
(c) Only II and III are required
(d) All I, II and III together are required
(e) II alone or from I and III together are required

36. Three athletes practice running on a circular track of 400 m. who runs
fastest among them?
I. When A starts running clockwise and B starts running
1
anticlockwise, they meet 2nd time at a distance of 57 meter from
7
6
starting point in clockwise direction after 22 seconds.
7
II. B runs 5 m/s faster than C. A is not twice or more fast than B and
neither B is twice or more fast than A or C.
III. When A, C and B, all runs in same direction, A and C only meet at
starting point, while ratio of speed of B to C is 3 : 2.

17 Adda247 Publications For any detail, mail us at


Publications@adda247.com
Cracker Book for Bank (IBPS | SBI | RRB PO | Clerk) Mains Exams

(a) Any one of them


(b) Only I and II together are sufficient
(c) Any two of the three together are sufficient
(d) None of the above
(e) All the three statements are not sufficient.

37. A man has 16 balls with him, in 3 colors i.e. Red, green and blue. Three
balls are drawn at random, what is the probability that all are blue.
1
I. The probability of drawing a blue ball is greater than and less than
8
5
. Also, the number of green balls he has is odd and number of red
16
balls he has is even.
3 1
II. Probability of drawing red ball is which is lesser than that of
8 16
green balls.
III. If a man lost one ball, probability of drawing a blue ball is ⅓.
(a) Only I and III or only II and III
(b) All I, II and III together
(c) Any one of them is sufficient
(d) Only I and III together sufficient
(e) None of the above

38. A man bought two bats and 6 identical balls, he sold all of them in a day,
calculate his overall profit %.
I. He sold one bat at a price of Rs. 600 and other at Rs. 420 and profit
is 20% on both bats. Each ball is sold at 12% profit.
II. Profit earned from 6 balls is Rs. 36 and profit earned from both bats
is Rs. 170.
III. Profit earned on each bat is 20% while cost price of each ball is Rs.
50.
(a) Any one of them
(b) Only I and II together are sufficient
(c) Any two of the three together are sufficient
(d) None of the above
(e) All three together are sufficient
Directions (39-42): The following questions are accompanied by three
statements A, B and C. You have to determine which statement(s) is/are
necessary/sufficient to answer the question.

18 Adda247 Publications For any detail, mail us at


Publications@adda247.com
Cracker Book for Bank (IBPS | SBI | RRB PO | Clerk) Mains Exams

39. Find the area of the shaded region?

A. Radius of bigger circle is given.


B. Diagonal of square ABCD is given.
C. Difference between area of bigger circle and area of smaller circle
is given.
(a) Either A or B alone is sufficient to answer the question
(b) Either B or C alone is sufficient to answer the question
(c) Either A or C alone is sufficient to answer the question
(d) Any of A, B or C alone is sufficient to answer the question
(e) Either only A or B and C together is sufficient to answer the
question

40. Find the surface area of cone?


A. Ratio between height and radius of cone is 8 : 7.
B. Radius of cone is half of the radius of hemisphere, which volume is
19404 cm³.
C. Radius of cone is 75% of radius of cylinder, which total surface area
of 2640 cm³ and ratio between height and radius of cylinder is 8 : 7.
(a) Either A and B or B and C are sufficient to answer the question
(b) Either A and B or A and C are sufficient to answer the question
(c) Either A and C or B and C are sufficient to answer the question
(d) A, B and C together are sufficient to answer the question
(e) Either only A or B and C together are sufficient to answer the
question
41. What will be 26 times of 26th term of Arithmetic progression (AP)?
A. If eight times of the 8th term of that A.P. is equal to 18th times the
18th term of that A.P.
19 Adda247 Publications For any detail, mail us at
Publications@adda247.com
Cracker Book for Bank (IBPS | SBI | RRB PO | Clerk) Mains Exams

B. If first term of AP is a prime number.


C. If difference between two terms in A.P. is an odd number.
(a) Only statement ‘A’ alone is sufficient to answer the question
(b) Only statement ‘B’ alone is sufficient to answer the question
(c) Only statement ‘C’ alone is sufficient to answer the question
(d) Any of two statements are sufficient to answer the question
(e) All three together are sufficient to answer the question

42. What is cost price of the article?


A. On selling the article at 119 Rs. shopkeeper earn profit % equal to
cost price of article.
2
B. If 14 % discount is given on the mark price, then its selling price
7
will be 102 Rs.
C. If article sold on profit percent half of the mark price, then profit is
41.65 Rs.
(a) Either A and B or B and C are sufficient to answer the question
(b) Either A and B or A and C are sufficient to answer the question
(c) Either A and C or B and C are sufficient to answer the question
(d) A, B and C together are sufficient to answer the question
(e) Either only A or B and C together are sufficient to answer the
question

43. A joker has three identical boxes having same number of balls but are
of different colors. In each box ‘x’ balls are red, 7 balls are blue and ‘y’
balls are green. Find (𝒙 + 𝒚)?
A. Probability of choosing one ball which is either red or blue from
𝟑
any one of the box, is .
𝟒
B. Number of ways to select two balls from any one of the box is 6.
C. Probability of choosing one ball which is either red or green from
𝟗
any one of the box is .
𝟏𝟔
(a) Only statement ‘A’ alone is sufficient to answer the question
(b) Only statement ‘B’ alone is sufficient to answer the question
(c) Only statement ‘C’ alone is sufficient to answer the question
(d) Any of two statements are sufficient to answer the question
(e) Both A and C are required together

20 Adda247 Publications For any detail, mail us at


Publications@adda247.com
Cracker Book for Bank (IBPS | SBI | RRB PO | Clerk) Mains Exams

44. Is (bc − ab) even integer or odd integer if a, b and c are integers?
A. a, b and c are in arithmetic progression.
B. Set {a,b,c} is co-prime and a,b and c are consecutive integers.
C. a, b and c all three are prime numbers.
(a) Either statement ‘A’ or ‘B’ alone is sufficient to answer the question
(b) Either statement ‘B’ or ‘C’ alone is sufficient to answer the question
(c) Either statement ‘C’ or ‘A’ alone is sufficient to answer the question
(d) Any of two statements are sufficient to answer the question
(e) Any one of them is sufficient to answer the question

45. Find the area of a rhombus?


A. The ratio of the diagonals of the rhombus is 3 : 4.
B. The perimeter of the rhombus is equal to that of a square whose
diagonal length is 20√2m.
C. The difference between the squares of the diagonals of rhombus is
448 m2.
(a) Statement ‘A’ and ‘B’ together are sufficient to answer the question
(b) Statement ‘B’ and ‘C’ together are sufficient to answer the question
(c) Statement ‘C’ and ‘A’ together are sufficient to answer the question
(d) Any of two statements are sufficient to answer the question
(e) Any one of them is sufficient to answer the question

46. What will be the sum of the ages of father and the son after five years?
A. Father’s present age is twice son’s present age
B. After ten years the ratio of father’s age to the son’s age will become
12 : 7.
C. Five years ago the difference between the father’s age and son’s age
was equal to the son’s present age.
(a) Statement ‘A’ and ‘B’ together are sufficient to answer the question
(b) Statement ‘B’ and ‘C’ together are sufficient to answer the question
(c) Statement ‘C’ and ‘A’ together are sufficient to answer the question
(d) Any of two statements are sufficient to answer the question
(e) Either ‘A’ and ‘B’ together or ‘B’ and ‘C’ together
47. In how much time Bhavya can row 64km in a stream to reach a
destination ‘A’ and return back?

21 Adda247 Publications For any detail, mail us at


Publications@adda247.com
Cracker Book for Bank (IBPS | SBI | RRB PO | Clerk) Mains Exams

A. Time taken by Bhavya to reach destination ‘A’ is twice than that of


to return back.
B. Ratio between speed of boat in still water to speed of boat in
downstream is 3 : 4
C. Upstream speed of boat is 4kmph less than downstream speed.
(a) Either statement ‘A’ and ‘B’ together or ‘B’ and ‘C’ together are
sufficient
(b) Either statement ‘A’ and ‘C’ together or ‘B’ and ‘C’ together are
sufficient
(c) Either statement ‘A’ and ‘B’ together or ‘A’ and ‘C’ together are
sufficient
(d) Any of two statements are sufficient to answer the question
(e) All three are required to solve the question

48. A, B and C together can complete work ‘X’ in 5 days. Find in how many
day ‘C’ alone can complete work ‘X’?
A. A and B together can complete work ‘X’ in 15 days if both worked
with half of their efficiencies.
B. B, who is 25% more efficient than ‘C’, takes 8 days more than ‘A’ to
complete work alone
C. ‘A’ is 25% less efficient than ‘B’.
(a) Either statement ‘A’ alone or ‘B’ alone is sufficient to answer the
question
(b) Either statement ‘B’ alone or ‘C’ alone is sufficient to answer the
question
(c) Either statement ‘C’ alone or ‘A’ alone is sufficient to answer the
question
(d) Any of two statements are sufficient to answer the question
(e) Any one of them is sufficient to answer the question

22 Adda247 Publications For any detail, mail us at


Publications@adda247.com
Cracker Book for Bank (IBPS | SBI | RRB PO | Clerk) Mains Exams

49. Find the sum of ‘x’ and ‘y’ ?


A. H.C.F and L.C.M of ‘x’ and ‘y’ is 21 and 3003 respectively.
B. Difference between x and y is 42
C. Both ‘x’ and ‘y’ are multiple of 21.
(a) Either statement ‘A’ and ‘B’ together or ‘B’ and ‘C’ together are
sufficient
(b) Either statement ‘A’ and ‘C’ together or ‘B’ and ‘C’ together are
sufficient
(c) Either statement ‘A’ and ‘B’ together or ‘A’ and ‘C’ together are
sufficient
(d) Any of two statements are sufficient to answer the question
(e) All three are required to solve the question

50. Find the area of shaded region if radius of each circle is same?

A. Area of circle is given.


B. Perimeter of triangle is given.
C. Area of triangle is approximately 310% more than area of a circle.
(a) Either statement ‘A’ or ‘B’ alone is sufficient to answer the question
(b) Either statement ‘B’ or ‘C’ alone is sufficient to answer the question
(c) Either statement ‘C’ or ‘A’ alone is sufficient to answer the question
(d) Any of two statements are sufficient to answer the question
(e) Any one of them is sufficient to answer the question
23 Adda247 Publications For any detail, mail us at
Publications@adda247.com
Cracker Book for Bank (IBPS | SBI | RRB PO | Clerk) Mains Exams

Solutions

1. (c); From I.
Lets length of larger rectangle and smaller rectangle be 5x and 4x
respectively.
2(5x+b) – 2(4x+b) = 8
Form II.
Breadth of rectangle = side of square
Side of square = 14 cm
From I & II we get
10x – 8x = 8
x = 4cm
perimeter of smaller rectangle
= 2 (4×4+14) = 60 cm
So, I and II both together sufficient to give answer

2. (e); From I,
Lets number of girls and boys be 9x and 11x respectively
From II,
Let total students 100x
Girls below eighteen years group
9
= 100x × = 9x
100
9𝑥
Given × 100 = 45x
20
Boys : girl = (100x–45x) : 45x = 11 : 9
From I and II we get same equation but we can’t get the answer.
So, Neither I nor II both sufficient to give answer of question.

3. (c); Form I,
Selling price of article = 3800 Rs.
3800
Marked price of article = × 100 = 4000
95
From II
4000
Cost price = × 100 = 3200
125

24 Adda247 Publications For any detail, mail us at


Publications@adda247.com
Cracker Book for Bank (IBPS | SBI | RRB PO | Clerk) Mains Exams

From I & II
Profit of shopkeeper = 3800 – 3200
= 600 Rs.

4. (e); From I,
Let speed of boat in
still water x km/hr and speed of current y km/hr
x = 3y …(1)
From II,
′𝑑′
Let bot cover ‘d’ distance downstream and distance in upstream
2
d
d 2 1 1
= ⇒ =
3y+y 3y−y 4y 4y
So, From I & II both not sufficient to answer of the question–

5. (d); From I,
Let amount invested on 20% is x Rs and on 10% is (4500–x) Rs.
𝑥 × 20 × 2 (4500– 𝑥 ) × 2 × 10
=
100 100
40x = 90000 – 20x
60x = 90000
x = 1500 Rs.
amount invested on 10% = (4500–1500) = 3000 Rs.
From II,
Lets man invested Rs 3x
Equivalent CI of two years on 20%
20×20
= 20 + 20 + = 44%
100
ATQ –
20 44
2𝑥 × + 𝑥× = 1260
100 100
108x = 126000
x = 1500 Rs.
Amount invested on 10% = 2 × 1500 = 3000 𝑅𝑠.
So, Either statement I alone or statement II alone sufficient is to
give answer of question
25 Adda247 Publications For any detail, mail us at
Publications@adda247.com
Cracker Book for Bank (IBPS | SBI | RRB PO | Clerk) Mains Exams

6. (d); Let length of two trains be 4x meter and 3x meter


From I,
5 (4𝑥+3𝑥)3
(90 + 72) × =
18 28
21x = 1260
x = 60 meters
Required difference = 60 × 4 − 60 × 3 = 60 meters
From II,
5 3𝑥
90 × =
18 7.2
3x = 180 meters
180
Length of larger train = 4x = × 4 = 240
3
Required difference = 60 meters
So, either I or II alone sufficient to give answer of question.

7. (c); From I and II together


Let P, Q and R takes 2x days, 3x days and 6x days respectively
So efficiency of P, Q and R is 3x, 2x and x unit/day
Total work = 4 × (3𝑥 + 2𝑥 + 𝑥 ) = 24x
So, we can determine required difference with I and II together

8. (d); Given, Cost price of article for Satish


1440
= × 100 = 1200 Rs.
120
From I,
Cost price of article for Veer = 1200 – 240
= 960 Rs.
240
Profit percentage of Veer = × 100 = 25%
960
From II,
100
Cost price of article for Veer = 1440 ×
150
= 960
Veer profit percentage
1200−960
= × 100 = 25%
960
So, either I or II alone sufficient to give answer of question.

26 Adda247 Publications For any detail, mail us at


Publications@adda247.com
Cracker Book for Bank (IBPS | SBI | RRB PO | Clerk) Mains Exams

9. (d); x : y : z = 11 : 9 : 12
Let x, y and z be 11a, 9a and 12a respectively
From I,
11a + 9a + 12a 11a + 9a
− =2
3 2
32a
− 10a = 2
3
𝑎=3
So,
(x + y) – 1.5z = (11 × 3 + 9 × 3)–1.5×3×12 = 6
From II,
1 1
11𝑎 × + 9𝑎 × = (62)0.5
11 9
2𝑎 = 6
𝑎=3
So, we can determine (x + y) – 1.5z from II also
So, Either statement I or Statement II alone sufficient

10. (c); Let length and breadth of rectangle be 7x and 4x respectively


From I,
Given, πr2 = 616
616×7
r2 =
22
r = 14 cm
length of rectangle = 14 × 2 = 28 𝑐𝑚
28
breadth of rectangle = × 4 = 16 𝑐𝑚
7
From I and II together,
2( l + b) – 4a = 20
2 (28 + 16) – 4a = 20
4a = 88 – 20
a = 17 cm
Area of Square = (17)2
= 289 cm2
So, Statement I and II both together sufficient

27 Adda247 Publications For any detail, mail us at


Publications@adda247.com
Cracker Book for Bank (IBPS | SBI | RRB PO | Clerk) Mains Exams

11. (b); If the max marks of exam = x


x
Raman =
4
x
⇒ = 288 − 128 = 160
4
x = 640
∴ Minimum passing marks
= 160 + 64 = 224
224
Required %= × 100 = 35%
640

12. (c); From I,


A + S+ V = 3 × 68 = 204 kg
R + P = 144 kg
A ⇒ 204 – 46 – 78 = 80 kg
P ⇒ 144 – 68 = 76 kg
S = 78 kg

13. (e); From I


27x − 23x = 100000
x = 25000
Population of city A = 50x
= 1250000
From II
Population of city B = x
4x
Population of city A =
5
4x
x− = 312500
5
x = 1562500
4
Population of city A = × 1562500
5
= 1250000

14. (b); From I and II


Students participating in dance = 150
Students who participate in singing
150×100
= = 60
250

28 Adda247 Publications For any detail, mail us at


Publications@adda247.com
Cracker Book for Bank (IBPS | SBI | RRB PO | Clerk) Mains Exams

15. (d); Form A → Let radius of circle = r


Then side of equilateral ∆ = √3𝑟
Area of equilateral triangle and circle can be find out and required
difference can also be find out easily.
From B → Let radius of circle =r
⇒ side of equilateral triangle = √3 r
Sum of peremeter of triangle and circle is given. By this value of ‘r’
can be find out and after this required difference can be find out
easily. Hence, either only A or only B is sufficient to answer the
question

16. (a); From A → 2𝑛 + 1 is divisible by 3


⇒ n =1, 3, 5, 7….
⇒ n = odd
From B) → 4𝑛 − 1 is divisible by 3
⇒ n =1, 2, 3…….
⇒ ‘n’ can be even as well as odd
Hence, Only A is sufficient to answer the question

17. (b); From A → Let numbers are a, b, c, d, e in ascending order


⇒ Median = 18 = c
And a+e=b+d
From B → let number are a, b, c, d, e is ascending order.
Median = 18 = c
Mode = 12 and 24
This means two number are 12 and two number are 24
⇒ Numbers are 12, 12, 18, 24, 24
Required sum 12 + 12 + 18 + 24 + 24 = 90
Hence, Only B is sufficient to answer the question.

18. (d); Let CP = 200x


⇒ MP = 300x
From A → SP = 300x – 60
120
200𝑥 × = 300𝑥 − 60
100
⇒ 60𝑥 = 60

29 Adda247 Publications For any detail, mail us at


Publications@adda247.com
Cracker Book for Bank (IBPS | SBI | RRB PO | Clerk) Mains Exams

⇒ CP = 200x = 200
From B → Two successive discount
10 × 10
= 10 + 10 − = 19%
100
81
S.P =200x + 43 = 300𝑥 ×
100
200x + 43 = 243x
x=1
CP = 200x = 200
Hence, either alone A or alone B is sufficient to answer the
question.

2
19. (b); Probability that Amit tells truth =
𝑥
𝑥−2
Probability that Amit tells lie =
𝑥
5
Probability that Ankush tells truth=
𝑦
𝑦−5
Probability that Ankush tells lie =
𝑦
2 𝑦−5 𝑥−2 5 1
From A→ × + × =
𝑥 𝑦 𝑥 𝑦 2
2 5 1
From B → × =
𝑥 𝑦 5
⇒ 𝑥𝑦 = 50
Hence only B is sufficient to answer the question.

20. (d); Given


AO + OC = OB
From A = AO → find
From B → AC = x (AO)
When ‘x’ multiple
From A & B together area of triangle and Area of semi circle find
out.
Required area
π(AO)2
= − √S(S – AO). (S – OC). (S – AC)
2
AO+OC+AC
S=
3
∴ A & B together sufficient to answer the question

30 Adda247 Publications For any detail, mail us at


Publications@adda247.com
Cracker Book for Bank (IBPS | SBI | RRB PO | Clerk) Mains Exams

21. (d); Given no. of white ball


Let → a
x
From A let probability →
y
Let no. of red ball → px, total balls → py
𝑠
From B → Let probability =
t
Let no. of black ball = qs, total balls = qt
From A & B
px + a + qs = qt = py
we know the values of x, y, s, t and a so we can find the value of p
and q
a a
So probability of white ball found = or
𝑞𝑡 py
∴ A & B together sufficient to answer the question

22. (b); Only B is sufficient to answer the question


When we cut sphere into hemisphere total surface area of two
hemisphere
Total surface area of two hemisphere
3πr² + 3πr² = π × 21 × 21
r = find out
So, volume of sphere can be find out.

23. (d); Given


∠PQR = ∠PSR = 90° {Angle of diameter}
A → ∠SRO = 180°– 90°(∠PSR) – 40°(∠ SPO)
∠SRO = 50°
B→ ∠PQR + ∠QRO = 120°
∠QRO = 120°– 90 = 30°
∠QPO = 60°
From B & A together, required difference can be find out.

24. (c); From A→ Discount % = 20% = Mark up%


If cost price is 100x then Markup price 120x and selling price is →
96x
So ATQ,
100x – 96x = 70

31 Adda247 Publications For any detail, mail us at


Publications@adda247.com
Cracker Book for Bank (IBPS | SBI | RRB PO | Clerk) Mains Exams
70
C. P = 100x = × 100x = 1750
4x
From B→ Let mark up price is → 100x
Then selling price is → 80x
ATQ,
100x – 80x = 350 + 70
20x = 420
100x = 2100
80x = 1680
C.P. → 1680 + 70 = 1750
So Either A or B alone required.
Solutions (25-29)

25. (c); From I, II & III


Let speed of Amit and Abhi be 4x and 5x km/hr respectively.
5x – 4x = 20
∴ x = 20 km/hr
Let distance be d km
d d
– =1
80 100
80×100
∴d= = 400 km
20

26. (b); From I and II


Let length and breadth be 3x and 2x
2πr = 440 [r → radius of circle]
r = 70 m
∴ breadth = 10 m
& length = 15 m
∴ Area = 10 × 15 = 150 m²
Statement I and III are same.

27. (a); From I


Passed = 400
From III
Let appeared & Passed student be 5x and 3x respectively
2x = 400 ⇒ x = 200
32 Adda247 Publications For any detail, mail us at
Publications@adda247.com
Cracker Book for Bank (IBPS | SBI | RRB PO | Clerk) Mains Exams

∴ failed = appeared – passed


= 1000 – 400
= 600

28. (a); From I


PRT(2)
= 44000
100
PR = 2200000
From II
PRT
P+ = 15400
100
From III
PR2
Difference =
1002
PR2
= 120
1002
From I and III R can be found.

29. (e); Let the smaller no. is x & bigger no. is y.


From I
y=x+6
From II,
40 30
×𝑥 = ×𝑦
100 100
From III,
𝑦
2 =2
𝑥 1
3
⇒ 3y = 4x
∴ from I and II or I and III

30. (d); From statement A,


Let MP of speaker = Rs. 100
∴ SP = 80 rupees
From B,
MP of wrist watch = 1.25 M.P of speaker
= Rs. 125 (From st. A)
From C,
33 Adda247 Publications For any detail, mail us at
Publications@adda247.com
Cracker Book for Bank (IBPS | SBI | RRB PO | Clerk) Mains Exams
100 800
C. P. of speaker = 80 × = (from st. A)
110 11
Here, there is no information about S.P. and Profit of watch. So,
answer cannot be found.

31. (b); From A,


11
Milk = 68 × = 44 ℓ
17
6
Water = 68 × = 24 ℓ
17
From B, let x ℓ milk is added to the mixture.
24 6
= ⇒x=8ℓ
44+x 13
∴ Total milk in final mixture = 44 + 8 = 52 ℓ

32. (d); From st. A,


Speed of train A = P m/sec (Given)
140 14P
Speed of train B = of P =
100 10
From st. B+A,
Length of platform
14P 14P
24 × – × 8
10 10
14P
= 16 × = 22.4 P
10
From C, Length of train A
3
= × 22.4P (with help of st. A and B together)
5
3
∴ Length of tunnel = Pt(given) – × 22.4P
5

33. (e); With help of statement A only, required number of ways can be
found out.

34. (e); From statements (I)


Nitin can cut cloth for 20 t-shirts in a day
5
∴ Vishal can cut cloth for 20 × = 25 t-shirts in a day
4
And if Vishal can stich 4x t-shirts in a day, Nitin will stich 5x t-
shirts.
It can’t be solved further.
34 Adda247 Publications For any detail, mail us at
Publications@adda247.com
Cracker Book for Bank (IBPS | SBI | RRB PO | Clerk) Mains Exams

From statement (II)


We only can obtain time taken by Nitin to build 100 t shirts which
is15 days
Nothing more can be obtained from (II) alone.
From statement (III)
Question can’t be solved from this statement alone.
So, by using (I) & (II)
Nitin will take 3 days to cut cloth for 60 t-shirts, Hence 6 days to
stich them.
Therefore, Nitin stiches 10 t-shirts per day
And Vishal stiches 8t-shirts per day. (From ratio obtained in (I))
Total time required for Nitin alone to make 100 t-shirts
100 100
= + = 15 days
20 10
Total time required for Vishal alone to make 100 t-shirts
100 100
= + = 4 + 12.5 = 16.5 days
25 8
Hence time can be calculated for both.

Using statement (I) and (III)


Days required by Vishal to cut cloth for 50
t-shirts is 2 days
∴ Days required by Vishal to stich 50
t-shirts in 12.5 days
Hence days required for both Nitin and Vishal can be calculate,
[first we will calculate, days required individually and then for both
of them]
Hence question can be solved through I & II or I & III.

35. (e); From Question,


Let age of Aashish and Kullu is 4x and 5x and that of Nikhil & Yash
is 7y and 8y.
From (I) statement
7y – 4x = 18 …eqn. (A)
It can’t be solved further.
From (II) statement

35 Adda247 Publications For any detail, mail us at


Publications@adda247.com
Cracker Book for Bank (IBPS | SBI | RRB PO | Clerk) Mains Exams
4𝑥–12 2
=
7𝑦–12 5
⇒ 20x – 60 = 14y – 24
⇒ 10x – 30 = 7y – 12
⇒ 10x – 7y = 18 …eq. (B)
5x–12 1
=
8y–12 2
⇒ 10x – 24 = 8y – 12
⇒ 5x – 12 = 4y – 6
5x – 4y = 6 …eqn. (C)
Solving eqn. (B) & eqn. (C)
We will get
x=y=6
Hence their age can be calculated. Also their average age after 4
years.
Therefore, statement (II) alone is efficient to solve question.
From (III) statement
4𝑥+12 3
= ⇒ x = 6, present age of Aashish is 24 years as that of Kullu
4𝑥 2
is 30 years.
Nothing more can be calculated
Now combining statement (III) & (I)
We will get x = y = 6.
Hence question can be solved.
Therefore, this question can be solved by statement (II) alone
or combining (I) and (III).

36. (b);

36 Adda247 Publications For any detail, mail us at


Publications@adda247.com
Cracker Book for Bank (IBPS | SBI | RRB PO | Clerk) Mains Exams

Let us draw a circular track of 400 meters with 4 point U, V, W and


X at anti-clockwise distance of 100m, 200 m and 300 meter.
From statement (i)
160
They meet second time in seconds that is somewhere between
7
U and X.
Therefore their sum of speed [as both were travelling in opposite
direction] is
800×7
= = 35 ms¯¹
160
As they meet between U and X.
There are two chances, either A have only covered a distance of 57
1 1
meter or total distance of (400 + 57 ) meter, hence, we can’t
7 7
determine who is more faster.
From statement (II)
It is clearly given that B is faster than C, but we can’t determine
who is faster between A and C.
From statement III
A and C only meet at starting points means either their speed is co-
prime of each other or one of their speed is integral multiple of
other and ratio of speed of B and C is given.
Now combining statement (i) and statement (ii)
It is given that B doesn’t run faster at twice or more speed. Hence
1
chances that A only travelled (400 + 57 ) meter distance, hence
7
faster than B, and it is given that B runs faster than C in statement
(ii).
Hence question can be answered from statement (i) and (ii)
We can’t answer question from (i) and (iii) or (ii) or (iii), as we
don’t know who is faster among A and C.

37. (c); From statement …(I)


2 5
Probability is greater than ⅛ i.e. and less than . Therefore, he
16 16
could either have 3 balls or 4 balls. Now sum of green balls and red
balls is odd. [Odd + Even = Odd]. 16 is even.
Therefore, blue balls must be odd. [Odd + Odd = Even].
Hence there are 3 blue balls.

37 Adda247 Publications For any detail, mail us at


Publications@adda247.com
Cracker Book for Bank (IBPS | SBI | RRB PO | Clerk) Mains Exams
3
(II) Probability of drawing red ball is
8
Therefore, he has 6 red balls.
3 1 7
Probability of drawing green ball = + =
8 16 16
He has 7 green balls.
∴ No. of blue balls he has = 16 – (6 + 7) = 3
Hence question can be answered.
(III) Let he have x balls. If one ball is lost, probability of drawing a
blue ball is

Which gives us x = 3
Hence question can be answered.
Question can be answered from either statement alone.

38. (c); From (I)


Total SP of bats = 1020
5
Therefore, CP of bats = 1020 × = Rs. 850
6
Profit % on ball = 12%
And more can’t be calculated.
From (II)
Profit on 1 ball = Rs.6
Profit on selling both bats = Rs. 170
From (III)
Cost price of each ball = Rs.50
Profit earned on each bat = 20%
From (I) and (II)
Total SP of bats = Rs. 1020
And CP of bats = Rs. 850
Total profit of 36 on balls is 12% of total CP of balls.

38 Adda247 Publications For any detail, mail us at


Publications@adda247.com
Cracker Book for Bank (IBPS | SBI | RRB PO | Clerk) Mains Exams
36
Hence profit on each ball is 12% which is = Rs. 6
6
Hence CP = Rs. 50/ ball
And question can be answered.
From (I) and (III)
Question can be answered, as we can calculate both CP and SP of
both balls & bat.
From (II) and (III)
Total profit earned is Rs. 206 [from (II)]
We can say that Rs. 170 is 20% of total CP of but, hence SP and CP
of both bats can be calculated.
CP of balls is given in statement (III), Hence question can be
answered from (II) and (III).
Hence, we can answer question from any two statements.

39. (d);

By joining the centers of all four circles, a square is formed which


contain 4 quadrant of circles which together makes a complete
circle.
Area of shaded region = Area of square(EFGH) - Area of bigger
circle – Area of smaller circle
Let radius of smaller circle and bigger circle be ‘r’ and ‘R’
respectively
Then relation between ‘r’ and ‘R’ is
𝑟 = 𝑅(√2 − 1)
A → Radius of bigger circle is given

39 Adda247 Publications For any detail, mail us at


Publications@adda247.com
Cracker Book for Bank (IBPS | SBI | RRB PO | Clerk) Mains Exams

By this side of square, radius of smaller circle can be find out. So,
area of shaded region can be find out
B → Diagonal of square is given
By this, side of square then radius of bigger circle and then radius
of smaller circle can be find out
After that shaded region can be find out
C → by using this we can find out the radius of the both circles and
side of square.
𝜋𝑅2 − 𝜋𝑟 2 = 𝑔𝑖𝑣𝑒𝑛
Relation between R and r is:-
𝑟 = 𝑅(√2 − 1)
Hence, Any of A, B or C alone is sufficient to answer the question

40. (b); From A,


Given 𝑟 ∶ ℎ = 7 ∶ 8
From B
Radius of hemisphere
Radius of cone =
2
2
Volume of hemisphere = 𝜋𝑟 3
3
2 3
𝜋𝑟 = 19404
3
19404 × 3 × 7
𝑟3 =
22 × 2
3
𝑟 = 9261
𝑟 = 21 cm

From A & B together —


21
Radius of cone = cm
2
10.5
Height of cone = × 8 = 12 cm
7
So from A and B we can determine the surface area of cone.
From A and C,
Given, radius of cone : height of cone = 7 : 8
Ratio between height and radius of cylinder = 8 : 7
2πr (r + h) = 2640 cm³
2π7x (8x + 7x) = 2640
40 Adda247 Publications For any detail, mail us at
Publications@adda247.com
Cracker Book for Bank (IBPS | SBI | RRB PO | Clerk) Mains Exams

x = 2 cm
75
Radius of cone = 7 × 2 × = 10.5 cm
100
10.5
Height of cone = × 8 = 12 cm
7
So, from A and B or from A and C, we can determine the surface
area of cone

41. (a); From A,


8𝑡8 = 18𝑡18
We have to find 26𝑡26
8(𝑎 + 7𝑑 ) = 18(𝑎 + 17𝑑)
8𝑎 + 56𝑑 = 18𝑎 + 306𝑑
10𝑎 + 250𝑑 = 0
(𝑎 + 25𝑑) = 0
So 26th term of = (𝑎 + 25𝑑 ) = 0
26 times = 26 × 0 = 0
From B,
No result determines
From C
No result determines
So, only statement A sufficient to give answer of question.

42. (e); From A:


Let cost price = profit percent = x
So,
𝑥×(100+𝑥)
= 119 Rs.
100
x² + 100x = 11900
x² + 100x – 11900 = 0
x = 70 Rs.
From B:
Selling price = 102 Rs.
102
M.P. ⇒ × 7 = 119 Rs.
6
From C:
If M.P. ⇒ 119 Rs.

41 Adda247 Publications For any detail, mail us at


Publications@adda247.com
Cracker Book for Bank (IBPS | SBI | RRB PO | Clerk) Mains Exams

Let cost price = x Rs.


From B and C together:
𝑥 119
× = 41.65 Rs.
100 2
x = 70 Rs.
Either only A or B and C together.

43. (c); From A,


ATQ,
𝑥+7 3
= ⇒ 3y – x = 7
𝑥+𝑦+7 4
From B,
As all boxes are identical and all colored balls are identical then
number of ways to choose balls is 6 ⇒ 𝑅𝑅, 𝐺𝐺, 𝐵𝐵, 𝑅𝐺, 𝑅𝐵, 𝐺𝐵
Where GG = Green | Green
By this value of x and y cannot be find out
From C
ATQ,
𝑥+𝑦 9
= ⇒x+y=9
𝑥+𝑦+7 16
So, only statement ‘C’ is sufficient to give answer of question.

44. (a); From A,


a, b and c are in A.P
Let common difference = ‘d’ and
⇒a = b−d
And c = b + d
(bc − ab) = b2 + bd − b2 + bd = 2bd
⇒ (bc − ab) is divisble by 2
From B,
Set {a, b, c} is coprime
⇒ all three have ′1′ as common multiple
And they are consecutive numbers
So , either b is even or a and c are both even.
In both cases-
b(c-a)= even

42 Adda247 Publications For any detail, mail us at


Publications@adda247.com
Cracker Book for Bank (IBPS | SBI | RRB PO | Clerk) Mains Exams

From C
All three are prime numbers but we can’t say whether in these
prime numbers 2 is taken or not. And if 2 is taken then b = 2 or not.
SO, in some cases (bc − ab) is divisble by 2 and in some cases
(bc − ab) is not divisble by 2. Correct conclusion cannot be inferred
from statement ‘C’ alone
So, either statement ‘A’ or ‘B’ alone is sufficient to give answer of
question.

45. (d); From A,


diagonal = 3x, 4x
From B,
4 a = 80, a = 20 ,𝑑12 + 𝑑22 = 1600
From C,
(4𝑥 )2 − (3𝑥 )2 = 448
x = 8, diagonal = 24, 32
1
Area = × 24 × 32 = 384 𝑚2
2
So, using any two of the three statement we can find area of
rhombus

46. (e); Let age of father = 𝑥


Age of son = 𝑦
From A,
𝑥 = 2𝑦
From B,
𝑥+10 12
=
𝑦+10 7
From C,
𝑥 − 𝑦 = 𝑦, 𝑥 = 2𝑦
So, Either A and B together or B and C together are sufficient to
answer the question

43 Adda247 Publications For any detail, mail us at


Publications@adda247.com
Cracker Book for Bank (IBPS | SBI | RRB PO | Clerk) Mains Exams

47. (b); Distance = 64 km


From A
Ratio between time to reach destination A and to return back = 2 :
1
1
Upstream to downstream Speed ratio = ∶ 1 ⇒ 1 : 2
2
From this we can find out the ratio between speed of boat in still
water to speed of stream
=3:1
From B
Ratio of speed of boat in still water to speed of boat in downstream
=3:4
From this we can find out the ratio between speed of boat in still
water to speed of stream
=3:1
From C
Difference between upstream speed to downstream speed = 4
km/hr
By using ‘A’ and ‘C’ together or ‘B’ and ‘C’ together, answer can be
find out.

48. (a); Let A, B and C can complete the work ‘X’ in a, b and c days
Respectively
1 1 1 1
⇒ + + =
a b c 5
From A
1 1 1
+ =
2a 2b 15
From B
4
b= c
5
and
b=a+8
From C
5
a=b
4
So, either statement ‘A’ alone or ‘B’ alone is sufficient to solve the
question.

44 Adda247 Publications For any detail, mail us at


Publications@adda247.com
Cracker Book for Bank (IBPS | SBI | RRB PO | Clerk) Mains Exams

49. (c); From A,


𝑥 × 𝑦 = 21 × 3003 = 21 × 21 × 11 × 13
L.C.M is 21 ⇒ Minimum value of ‘x’ or ‘y’ should be 21
From B,
(𝑥 − 𝑦) 𝑜𝑟 (𝑦 − 𝑥 ) = 42
From C
Both x and y are multiple of 21 ⇒ L.C.M of x and y is 21.
Hence, either statement ‘A’ and ‘B’ together or ‘A’ and ‘C’ together
are sufficient

50. (a); In the figure Triangle ABC is an equilateral triangle


And relation between radius or circle (r) and side of triangle (a) is
given below
𝑟 1
=
𝑎 2(1+√3)
From A,
Area is given ⇒ radius is given ⇒ side of triangle can be find out ⇒
Area of shaded region can be find out
From B,
Perimeter of triangle is given ⇒ radius of circle can be find out ⇒
Area of shaded region can be find out
From C
It is a basic condition of this figure and anything cannot be
concluded from this statement
Hence, either statement ‘A’ or ‘B’ alone is sufficient.

45 Adda247 Publications For any detail, mail us at


Publications@adda247.com
Cracker Book for Bank (IBPS | SBI | RRB PO | Clerk) Mains Exams

1 Adda247 Publications For any detail, mail us at


Publications@adda247.com
Cracker Book for Bank (IBPS | SBI | RRB PO | Clerk) Mains Exams

Chapter

16 Caselets

BEST APPROACH TO SOLVE THE QUESTIONS

A Caselet is another way of providing data, more often, explicitly. Unlike


various graphical methods of data representation, a caselet requires to be
solved completely before arriving at the answers to the questions that
follow it. Usually, in a caselet, data is provided through statements that
imply the logic or relation of various components of data. Some caselets are
based on Set Theory and concepts of Set theory are applied to them. All
other caselets are based on basic mathematical understanding.
A sample example of a caselet is as follows:

‘PO’ exam in Lucknow is 60% more than candidates who are appearing
for same exam in Agra. Number of candidates appearing for ‘PO’ exam in
Allahabad is 48% of total candidates appearing for ‘PO’ exam. Ratio
between total candidates appearing for ‘PO’ exam and ‘Clerk’ exam is 5 : 9.
Difference between candidates appearing for ‘PO’ exam in Lucknow and
Allahabad is 400. Ratio between candidates appearing for ‘PO’ exam to
‘Clerk’ exam in Agra and Lucknow is 5 : 12 and 8 ; 9 respectively.

Let total candidates appeared for ‘PO’ exam and ‘Clerk’ exam is 500x and
900x

Number of candidates appearing for ‘PO’ exam in Allahabad


48
= × 500x = 240x
100

Remaining candidates appeared for ‘PO’ exam in Agra and Lucknow


together = 500x – 240x = 260x
Let total candidates appeared for ‘PO’ exam in Agra = y
Then total candidates appeared for ‘PO’ exam in Lucknow = 1.6y

ATQ,
y + 1.6y = 260x ⇒ y = 100x
2 Adda247 Publications For any detail, mail us at
Publications@adda247.com
Cracker Book for Bank (IBPS | SBI | RRB PO | Clerk) Mains Exams
12
total candidates appeared for ‘Clerk’ exam in Agra = 100x × = 240x
5
9
total candidates appeared for ‘Clerk’ exam in Lucknow = 160x × = 180x
8
total candidates appeared for ‘Clerk’ exam in Allahabad
= 900x – 240 – 180x = 480x

EXAM AGRA LUCKNOW ALLAHABAD


PO 100x 160x 240x
CLERK 240x 180x 480x

Now ATQ
240x – 160x = 400
400
⇒𝑥 = =5
80

EXAM AGRA LUCKNOW ALLAHABAD


PO 500 800 1200
CLERK 1200 900 2400

Practice Exercise Based on new Pattern

Directions (1-5): Study the given passage carefully and answer the
questions.

Rahul, Sandy and Sati invested in ratio 2 : 3 : 4. After 4 months Sandy


added Rs. 1500 more in his investment and Rahul withdrew Rs. 800 from
his investment. After six months more Sati invested half of the investment
done by Rahul in first four months and Sandy invested 50% more than the
investment done by Sati in first 10 months. Rahul invested same as
investment done by Sandy in first four months. Ratio of profit of Sati to total
profit at the end of year is given as 125 : 376.

1. Profit of Sandy is approximately what percent of total profit?


(a) 64% (b) 48% (c) 72%
(d) 68% (e) 42%

3 Adda247 Publications For any detail, mail us at


Publications@adda247.com
Cracker Book for Bank (IBPS | SBI | RRB PO | Clerk) Mains Exams

2. What is the difference between profit share of Rahul and Sandy if total
profit is Rs.37,600?
(a) 12,000 (b) 16,400 (c) 18,500
(d) 22,900 (e) 20,000

3. Veer have 250% more than initial investment of Sati for a year. Find
total interest earned by him if he invested his amount in a scheme
which offers 20% p.a. for 2 years?
(a) Rs. 1400 (b) Rs. 1500 (c) Rs. 1540
(d) Rs.1600 (e) Rs.1640

4. What is the average of profit share of Sandy and Sati out of total profit
of Rs. 37,600?
(a) 18,220 (b) 18,250 (c) 16,420
(d) None of these (e) 12,490

5. If initial investment of Bhavya is one-third of initial investment of


Rahul, Sandy and Sati together then find the difference between initial
investment of Bhavya and Rahul.
(a) Rs.1000 (b) None of these (c) Rs.750
(d) Rs.500 (e) Rs.250

Directions (6-10): There are 2000people live in a village. Out of 2000


people some people are watching ‘3 idiots’ ‘Ravan’ and ‘PK’ on Sunday and
some are not watching any movie. Number of people did not watch any
movie is equal to number people watching ‘PK’ only. Number of people who
are watching ‘Ravan’ is 720. Ratio between number of people watching ‘3
Idiots’ and ‘Ravan’ only to number of people watching ‘Ravan’ and ‘PK’ only
is 2 : 3. Number of people watching ‘3 Idiots ’ and ‘PK’ both is half of number
of people watching only ‘PK’. Number of people watching ‘3 idiots’ only is
50% more than number of people watching ‘Ravan’ only. Number of people
watching all the three movies is 4% of the total number of people in the
village. Number of people watching ‘Ravan’ only is same as number of
people watching ‘Ravan’ and ‘PK’ only.

4 Adda247 Publications For any detail, mail us at


Publications@adda247.com
Cracker Book for Bank (IBPS | SBI | RRB PO | Clerk) Mains Exams

6. Number of people who did not watch any movie are what percent less
than number of people who watching only Ravan, only PK and only 3
Idiots together? (approx.)
(a) 55% (b) 50% (c) 65%
(d) 62% (e) 70%

7. Find the ratio between people who watching only Ravan and only PK
together to people who watching only Ravan & 3 idiots and only PK & 3
idiots together ?
(a) 16 : 5 (b) 16 : 7 (c) 16 : 13
(d) 16 : 11 (e) 76 : 43

8. Out of total people watching 3 idiots only, ratio between male to female
is 4 : 1 and that of people watching Ravan only is 5 : 3. Find total
number of males , who watching 3 idiots and Ravan only ?
(a) 436 (b) 428 (c) 440
(d) 438 (e) 442

9. People watching only Ravan & PK are what percent more than people
watching only Raven & 3 idiots?
(a) 50 % (b) 45% (c) 55%
(d) 40% (e) 60%

10. People who are watching only 3 Idiots, only PK and only 3 Idiots and PK
both is how much more than people who are watching Ravan?
(a) 252 (b) 192 (c) 672
(d) 488 (e) 272

Directions (11-15): Given below is the data about students appeared in


two exams i.e., A and B in six different years i.e., 2011 to 2016. Total
students appeared in both exam in 2016 is 8000 while in 2013 it is 5800.
Average number of students appeared in exam B in 2011 and 2013 is 3100
and is in the ratio 18 : 13. Students appeared in exam A in year 2015 is
1
33 % more than students appeared in exam B in same year. Total students
3
appeared in 2016 is 25% more than total students appeared in 2011.

5 Adda247 Publications For any detail, mail us at


Publications@adda247.com
Cracker Book for Bank (IBPS | SBI | RRB PO | Clerk) Mains Exams
26
Students appeared in exam A in 2016 is 62 % more than students
27
appeared in exam B is 2015. Ratio of total students appeared in 2016 &
2014 is 16 : 13. Total number of students appeared in exam A in all six
years is 21,100. Students appeared in exam B in 2011 is same as student
appeared in exam A in 2015.Students appeared in exam A in 2012 is 700
more than that of students appeared in same exam in 2014. Students
appeared in exam B in 2014 is 1200 less than that of in same exam in 2012.

11. In which year total students appeared in both exam is 3rd highest?
(a) 2012 (b) 2014 (c) 2016
(d) 2011 (e) Other than the given options

12. What is the respective ratio between students appeared in exam A in


year 2011, 2012 and 2014 together to the students appeared in exam B
in year 2013, 2014 and 2016 together?
(a) 95 : 97 (b) 99 : 97 (c) 98 : 97
(d) 99 : 95 (e) None of these

13. Find the difference between average number of students appeared in


exam A and average of students appeared in exam B in starting four
years?
(a) 250 (b) 225 (c) 215
(d) 200 (e) None of these

14. Students appeared in exam A in 2013 is how much less than students
appeared in exam B in 2012?
(a) 1400 (b) 1000 (c) 1100
(d) 1200 (e) 1300

15. Total number of students appeared in both exam in 2012 is what


percent more than total number of students appeared in both exam in
2011?
(a) 25.25% (b) 28.25% (c) 31.25%
(d) 34.25% (e) 37.25%
6 Adda247 Publications For any detail, mail us at
Publications@adda247.com
Cracker Book for Bank (IBPS | SBI | RRB PO | Clerk) Mains Exams

Directions (16-20): There are 1000 students in a college. Out of 1000


students some appeared in exams ‘X’, ‘Y’ and ‘Z’ while some not. Number of
student not appeared in any exam is equal to number of students appeared
in exam ‘Z’ only. Number of students appeared in exam ‘Y’ is 360. Ratio of
number of students appeared in exam ‘X’ and ‘Y’ only to number of students
appeared in exam ‘Y’ and ‘Z’ only is 2 : 3. Number of student appeared in
exam ‘X’ and ‘Z’ both is half of number of students appeared in only exam
‘Z’. Number of students appeared in exam ‘X’ only is 50% more than
number of students appeared in ‘Y’ only. Number of students appeared in
all the three exam is 4% of the total number of students in the college.
Number of students appeared in ‘Y’ exam only is same as number of
students appeared in ‘Y’ and ‘Z’ only.

16. How many students appeared in at least two exams?


(a) 240 (b) 260 (c) 300
(d) 360 (e) 500

17. How many students appeared in two exams only?


(a) 280 (b) 220 (c) 340
(d) 300 (e) 260

18. How many students appeared in at most two exams?


(a) 240 (b) 260 (c) 300
(d) 500 (e) 960

19. How many students not appeared in exam Y?


(a) 440 (b) 360 (c) 540
(d) 640 (e) None of these

20. How many students appeared in exam X or in exam Z?


(a) 240 (b) 360 (c) 500
(d) 680 (e) 760

Directions (21-25): Number of songs recorded by Sonu Nigam in 2008 is


300. And number of songs recorded by Sandeep and Arijit Singh in 2010 are
in ratio 5 : 4. Total number of songs recorded in 2009 is 250% more than
songs recorded by Arijit Singh in 2009. Total songs recorded by Sandeep in
7 Adda247 Publications For any detail, mail us at
Publications@adda247.com
Cracker Book for Bank (IBPS | SBI | RRB PO | Clerk) Mains Exams
4
all three years is 650. Average of songs recorded in 2010 is th of songs
3
recorded by Arijit Singh in 2008. Songs recorded by Arijit Singh in 2009 is
2th
of songs recorded by Sonu Nigam in 2008. Total songs recorded in 2008
5
is 6 times of songs recorded by Arijit Singh in 2009 and songs recorded by
Sandeep in 2008 is 25% less than songs recorded by Arijit Singh in same
year. Ratio of songs recorded by Sonu Nigam and Sandeep in 2009 is 2 : 3.

21. Number of songs recorded by Arijit Singh in all years is what percent
more/less than total songs recorded in 2008 ?
2 7 2
(a) 12 % (b) 17 % (c) 21 %
4 9 3
2
(d) 27 % (e) None of these
5

22. What is the difference of average of total number of songs recorded by


Sandeep in 2008 and 2009 and average number of songs recorded by
Sonu Nigam and Arijit Singh in 2010?
(a) 155 (b) 165 (c) None of these
(d) 150 (e) 145

23. What is the ratio of total number of songs recorded in 2010 to total
number songs recorded in 2011, if total number of songs recorded in
2011 is 50% more than total number of songs recorded in 2009 ?
(a) 32 : 17 (b) None of these (c) 21 : 19
(d) 33 : 19 (e) 32 : 21

24. Average number of songs recorded by Sonu Nigam is approximately


what percent more/less than average number of songs recorded in
2009?
(a) 104% (b) None of these (c) 124%
(d) 114% (e) 110%

25. Average number of songs recorded by all in all three years and is how
much more than average number of songs recorded by Sandeep and
Arijit Singh in 2009?
(a) None of these (b) 525 (c) 650
(d) 550 (e) 450
8 Adda247 Publications For any detail, mail us at
Publications@adda247.com
Cracker Book for Bank (IBPS | SBI | RRB PO | Clerk) Mains Exams

Directions (26-30): Study the following passage & answer the


questions that follows.
There are three schools in a town named A, B & C. In each school students
knows only Hindi, Only English and both language.
Total number of student in school A is 1600. The no. of student who know
only Hindi in school C is 40% of the total students in school A. And total
students who know only single language in school A are equal to student
who knows both language in same School and ratio of student who know
only Hindi to only English in school A is 2 : 3. Student who know both
language in all three schools is 50% more than student who knows only
English in school B. Student who knows only Hindi in all schools is equal to
the total students in school A except student who knows only English. Ratio
of students who know only English in school B to C is 1 : 4 and average of
student who know only English in all the school is 460. Student who know
both language in school C is half of student who know only Hindi in school
3
A. Total student in school B is th of total student from all the school who
4
knows only Hindi.

26. Total student in school A who know only one language is what percent
more/less than total student in school B who knows both language?
(a) 50% (b) 60% (c) 70%
(d) None of these (e) 45%

27. What is ratio of total student who knows only English in school B and C
together to total student in school B ?
(a) 3 : 2 (b) 7 : 5 (c) None of these
(d) 5 : 3 (e) 15 : 14

28. What is sum of average of student who knows only English in all the
3th
school and of student who knows only single language in school B
4
and C together.
(a) 1735 (b) 1335 (c) 1220
(d) 1525 (e) 1785
9 Adda247 Publications For any detail, mail us at
Publications@adda247.com
Cracker Book for Bank (IBPS | SBI | RRB PO | Clerk) Mains Exams

29. 40% of total student in school B is what percent more/less than half of
student who know only Hindi in school A ?
(a) 140% (b) 110% (c) 90%
(d) None of these (e) 130%

30. What is difference of average of total student in all three schools and
average of student who knows both language in school B and C ?
(a) 920 (b) 840 (c) 890
(d) 990 (e) None of theses

Directions (31-35): Data regarding investment of three different persons


in three different schemes is given below. Study the data carefully and
answer the following questions.

→ Out of total amount invested by ‘Rahul’ in all the three schemes, 25% is
invested in scheme ‘X’. Remaining amount is invested in scheme ‘Y’ and
‘Z’ equally.

→ ‘Veer’s’ investment in scheme ‘X’ is 37.5% less than that of ‘Anurag’s’ in


same scheme. Total amount invested by ‘Rahul’, ‘Veer’ and ‘Anurag’ in
all the three schemes is in the ratio of 4 : 5 : 6. Ratio between amount
invested by ‘Veer’ in scheme ‘Y’ to in ‘Z’ is 3 : 2. Amount invested by
‘Veer’ in scheme ‘Y’ is 80% more than that in scheme ‘X’.

→ Amount invested by Anurag in scheme ‘Y’ and ‘Z’ together is Rs. 20,000
more than the amount invested by ‘Rahul’ in both the same schemes
together. Amount invested by Anurag in scheme ‘Y’ is 200% more than
that in scheme ‘Z’.

31. Rate of interest on scheme ‘X’, ‘Y’ and ‘Z” is 10%, 20% and 30% p.a. at
Simple interest. Find total interest earned by Rahul after 2 years.
(a) Rs. 44,000 (b) Rs. 41,000 (c) Rs. 34,000
(d) Rs. 30,000 (e) Rs. 24,000
10 Adda247 Publications For any detail, mail us at
Publications@adda247.com
Cracker Book for Bank (IBPS | SBI | RRB PO | Clerk) Mains Exams

32. Veer and Anurag both invested in scheme ‘X’. After 8 months Veer
withdraw his total amount while Anurag withdraw his total amount
after 12 months. If total profit of both is Rs. 7,650 then find the profit
share of Veer.
(a) Rs. 2,250 (b) Rs. 3,150 (c) Rs. 4050
(d) Rs. 4950 (e) Rs. 6750

33. Total amount invested in scheme ‘Y’ by all three together is what
percent more than total amount invested by all three in scheme ‘Z’ all
together?
(a) 50.25% (b) 56.75% (c) 62.75%
(d) 68.75% (e) 72.25%

34. Find the ratio between amount invested by Veer on scheme ‘X’ to
amount invested by Rahul in scheme ‘Z’?
(a) 4 : 3 (b) 1 : 1 (c) 2 : 3
(d) 5 : 8 (e) 5 : 6

35. Scheme ‘Z’ offers 20% p.a. at Compound interest. Find interest earned
by Veer is what percent more than interest earned by Anurag in that
scheme?
(a) 75% (b) 50% (c) 100%
(d) 150% (e) 200%

Direction (36-40): Data given below about five vessels M, N, O, P & Q


contains mixture of different liquid read the data carefully and answer the
questions.

Vessel M contains mixture of milk and water, vessel N contains mixture of


water and orange juice, vessel O contains mixture of orange juice and
Vodka, vessel P contains mixture of Vodka and apple juice and vessel Q
contains mixture of apple juice and Scotch.

Mixture of milk and water in vessel M in the ratio of 7 : 2 and quantity of


water in vessel N equal to quantity of milk in vessel M. Mixture of orange
juice and vodka into the vessel O in the ratio of 5 : 3 and total quantity of
mixture in vessel O is 30 liter more than total quantity of mixture in vessel
N.
11 Adda247 Publications For any detail, mail us at
Publications@adda247.com
Cracker Book for Bank (IBPS | SBI | RRB PO | Clerk) Mains Exams
2
Total quantity of mixture in vessel Q in 66 % of total quantity of mixture in
3
vessel M. Total quantity of mixture in vessel O is 50% more than total
quantity of mixture in vessel P.

Ratio of orange juice in vessel N and O is 7 : 15, while ratio of vodka in


vessel O and P is 3 : 2. Total quantity of Scotch in vessel Q is 30 liter and
total quantity of water in vessel M is 40 liter. Quantity of orange juice in
vessel N is 70 liters.

36. If 72 𝑙 of mixture from vessel M and 36 mixture taken out from vessel N
and mixed in another vessel G, then what will be ratio of milk, `water
and orange juice in vessel G?
(a) 15 : 5 : 8 (b) 14 : 10 : 3 (c) 14 : 7 : 3
(d) 15 : 8 : 3 (e) 14 : 10: 7

37. 36 liter of mixture from vessel Q taken out and mixed in a vessel C,
which contains 21 liter scotch, what quantity of apple juice should
mixed in vessel C to obtain ratio of apple juice and scotch 2 : 1:
(a) 36 (b) 33 (c) 38
(d) 37 (e) 32

38. If vessel N and O mixed together in a big vessel Z and 22.5 liter of
mixture taken out from vessel Z, find quantity of Vodka in 22.5 liter of
mixture?
(a) 4.5 liter (b) 9 liter (c) 2.25 liter
(d) 1.5 liter (e) 3.6 liter

39. If 64 liter of mixture from vessel O, 48 liter of mixture from P and 60


liter of mixture from vessel Q taken out and mixed together in vessel A,
find the ratio of orange juice, vodka, apple juice and scotch in vessel A?
(a) 40 : 42 : 75 : 25 (b) 40 : 25 : 75 : 52 (c) 40 : 52 : 75 : 15
(d) 30 : 52 : 75 : 15 (e) 40 : 42 : 75 : 15
12 Adda247 Publications For any detail, mail us at
Publications@adda247.com
Cracker Book for Bank (IBPS | SBI | RRB PO | Clerk) Mains Exams

Directions (40-42): Given below is the data about players participated for
state level and National level championship in six different years i.e., 2001
to 2006. Total players participated for both Championship in 2006 is 6000
while in 2003 is 4800. Total number of players participated for national
level championship in 2001 and 2003 is 3600 and is in ratio of 11 : 7. Player
participated for state championship in 2005 is 25% more than player
participated for national Championship in same year. Total players
participated in 2006 is 20% more than total players participated in 2001.
1
Players participated for state Championship in 2006 is 62 % more than
2
players participated for national championship in 2005. Ratio of total
players participated in 2006 to 2004 is 15 : 13. Total number of players
participated for state championship in all six years is 20000. Players
participated for national championship in 2001 is same as players
participated for state championship in 2005. Players participated for state
championship in 2002 is 800 more than that of players participated for
same championship in 2004. Players participated for National level
championship in 2004 is 1200 less than that of for same Championship in
2002.?

40. What is the respective ratio between players participated for state level
championship in year 2001, 2002 and 2004 together to the players
participated for national level championship in year 2003, 2004 and
2005 together?
(a) 1145:493 (b) 1154:441 (c) 1154:439
(d) 439:1154 (e) 1105:439

41. Find the difference between average number of players participated for
state level championship and average of players participated for
national level championship for starting four years?
(a) 1900 (b) 1920 (c) 1820
(d) 1780 (e) 1940

42. Players participated for state level championship in 2003 is how much
more than players participated for national level championship in
2002?
(a) 930 (b) 940 (c) 950
(d) 960 (e) 970
13 Adda247 Publications For any detail, mail us at
Publications@adda247.com
Cracker Book for Bank (IBPS | SBI | RRB PO | Clerk) Mains Exams

Direction (43-47): Given below statistical data of top five wheat


producer states of India in the three successive years. Read the data
carefully and answer the question given below.
Year 2015– Uttar Pradesh and Punjab together produced 15860 thousand
ton, while Punjab production was 1180 ton less than that of Uttar Pradesh.
Madhya Pradesh produced 25% less than Uttar Pradesh, Haryana produced
690 thousand ton less than that of Madhya Pradesh. Production of
Rajasthan was 70% of total production of Haryana
Year 2016– Production of Punjab decreased by 20% compare to previous
year, while production of Haryana increased by 800 thousand ton compare
1
to previous year. Madhya Pradesh produced 33 % more than previous
3
year, while Rajasthan produced 2390 thousand ton more wheat than
previous year. All five states produced total 36612 thousand ton wheat in
2016.
Year 2017– Uttar Pradesh and Haryana together produced 16520
thousand ton, while Uttar Pradesh production was 2840 ton more than that
of Haryana. Punjab produced 628 thousand ton more wheat than previous
year and Madhya Pradesh produced 360 thousand ton more wheat than
previous year. All five states produced total 38200 thousand ton wheat in
2017.

43. Total wheat produced by Uttar Pradesh is how much more than total
wheat produced by Madhya Pradesh in the all three given years?
(a) 3550 thousand ton (b) 3750 thousand ton
(c) 3650 thousand ton (d) 3250 thousand ton
(e) 3150 thousand ton

44. Total wheat produced by Madhya Pradesh in the year 2016 & 2017
together is what percent more than total wheat produced by Haryana
in the year 2015 & 2016 together?
38 38 38
(a) 40 % (b) 44 % (c) 36 %
61 61 61
38 38
(d) 42 % (e) 48 %
61 61

14 Adda247 Publications For any detail, mail us at


Publications@adda247.com
Cracker Book for Bank (IBPS | SBI | RRB PO | Clerk) Mains Exams

45. Find the difference between average quantity of wheat produced by


Uttar Pradesh, Madhya Pradesh and Haryana in the year 2015 and
average quantity of wheat produced by Madhya Pradesh, Haryana and
Rajasthan in the year 2017?
(a) 360 thousand ton (b) 400 thousand ton (c) 420 thousand ton
(d) 450 thousand ton (e) 470 thousand ton

46. Find the ratio between total wheat produced by Rajasthan & Punjab
together in the year 2017 to total wheat produced by Haryana &
Rajasthan together in the year 2016?
(a) 160 : 161 (b) 150 : 161 (c) 170 : 171
(d) 160 : 169 (e) 160 : 171

47. Find the total wheat produced by Rajasthan and Uttar Pradesh in the
year 2016 and 2017 together?
(a) 30700 thousand ton (b) 30800 thousand ton
(c) 31700 thousand ton (d) 32400 thousand ton
(e) 33400 thousand ton

Direction (48–51): Given below data is about the number of


candidates who got final selection into UPSC exam from five different
states (Uttar Pradesh, Kerala, Maharashtra, Madhya Pradesh and
Bihar) in three different successive years. Read the data carefully and
answer the questions:
2012– Total number of Candidates selected from Uttar Pradesh are 20%
more than total number of Candidates selected from Kerala and total
number of selected Candidates from these two states are 440. Average
number of Candidates selected from Uttar Pradesh, Kerala and Maharashtra
is 220 and total number of Candidates selected from Madhya Pradesh is 70
less than that of from Maharashtra. Total number of Candidates selected
from these five states in the year 2012 is 930.
2013– Total number of Candidates selected from Uttar Pradesh is 33
1
% more than that of total number of Candidates selected from same state
3
in previous year, while total number of Candidates selected from Kerala is
30 less than average number of Candidates selected from Kerala &
Maharashtra in the year 2012. Total number of Candidates selected from
15 Adda247 Publications For any detail, mail us at
Publications@adda247.com
Cracker Book for Bank (IBPS | SBI | RRB PO | Clerk) Mains Exams

Maharashtra is 55 more than that of total number of Candidates selected


from same state in previous year and total number of candidates selected
from Madhya Pradesh is equal to total number of candidates selected from
Kerala. Total number of Candidates selected from these five states in the
year 2013 is 1130.
2014– Total number of candidates selected from Uttar Pradesh and
Maharashtra is 40 more and 25 less than total number of candidates
selected from Uttar Pradesh and Maharashtra in previous year respectively.
Average number of candidates selected from Uttar Pradesh, Kerala and
Maharashtra is 280, while average number of candidates selected from
Madhya Pradesh & Bihar is 195. Total number of selected candidates from
Bihar is 20 more than that of Madhya Pradesh.

48. Total number of candidates selected from Kerala & Maharashtra


together in the year 2014 is what percent more/less than total number
of candidates selected from Uttar Pradesh in the year 2012 & 2014
together?
(a) 10% (b) 20% (c) 15%
(d) 25% (e) 5%

49. If total number of candidates selected from Karnataka in the year 2014
is 50% more than total number of candidates selected from Madhya
Pradesh in the year 2013, then find average number of candidates
selected from Karnataka, Uttar Pradesh, Madhya Pradesh & Bihar in the
year 2014?
(a) 225 (b) 255 (c) 205
(d) 215 (e) 200

50. Find the ratio between total number of candidates selected from Kerala
to total number of selected candidates from Bihar in the given three
years?
(a) 63 : 50 (b) 71 : 50 (c) 61 : 50
(d) 67 : 50 (e) 69 : 50
16 Adda247 Publications For any detail, mail us at
Publications@adda247.com
Cracker Book for Bank (IBPS | SBI | RRB PO | Clerk) Mains Exams

51. Total number of candidates selected from Maharashtra, Madhya


Pradesh and Bihar together in the year 2013 is what percent more/less
than total number of candidates selected from Madhya Pradesh in the
year 2012 & 2013 together?
10 10 10
(a) 92 % (b) 90 % (c) 94 %
11 11 11
10 10
(d) 91 % (e) 95 %
11 11

Directions (52-56): Neeraj have some toys which are in the form of
different structures. These are cylindrical, conical, spherical. Other than
solid conical structure, all two are of both types i.e., hollow as well as solid.

→ Volume of a conical toy is three times of the volume of a solid


cylindrical toy while radius of a solid spherical toy is half than that the
radius of a conical toy. Outer radius of hollow cylindrical toys is same as
radius of solid spherical toy while average of outer radius and inner
radius of hollow cylindrical toys is equal to radius of solid cylindrical
toy. Height of cylindrical, conical and hollow cylindrical toys is same i.e,
14c.m
→ Number of solid spherical toys is 20% of total number of toys Neeraj
have. Number of hollow spherical toys is 150% more than number of
conical toys. Ratio between number of solid cylindrical toys to number
of conical toys is 3 : 2. Total number of hollow cylindrical toys is 40% of
total number of toys Neeraj have and also ‘20’ more than the total
number of solid spherical toys Neeraj have.
→ Volume of a hollow spherical toy is 33,957 cm² whose inner radius is
half of its outer radius. Volume of a hollow spherical toy is 5.25 time of
volume of conical toy.

52. Find the total space taken by all solid spherical toys? (in cm³)
(a) 97020 (b) 48510 (c) 72765
(d) 14553 (e) 24255

53. Find the number of conical toys Neeraj have?


(a) 40 (b) 20 (c) 15
(d) 12 (e) 8

17 Adda247 Publications For any detail, mail us at


Publications@adda247.com
Cracker Book for Bank (IBPS | SBI | RRB PO | Clerk) Mains Exams

54. Find the curved surface area of one hollow cylindrical toy? (in cm²)
(a) 616 (b) 1232 (c) 924
(d) 462 (e) 1386

55. Find the ratio between outer radius of hollow spherical toy to radius of
solid cylindrical toy?
(a) 4 : 1 (b) 3 : 2 (c) 3 : 1
(d) 4 : 3 (e) 2 : 1

56. Volume of one hollow cylindrical toy is how much more then volume of
one cylindrical toy?(in cm3 )
(a) 4312 (b) 3234 (c) 2696
(d) 2156 (e) 1078

Direction (57-61): - Data about speed and length of six different trains is
given below. Study the data carefully and answer the following question.

Speed of six trains A, B, C, D, E and F are a, b, c, d, e and f respectively while


time taken by six trains to cross a pole is 𝑃𝐴 , 𝑃𝑏 , 𝑃𝑐 , 𝑃𝑑 , 𝑃𝑒 and 𝑃𝑓 respectively.
Length of six trains A, B, C, D, E and F are 𝑙𝑎 , 𝑙𝑏 , 𝑙𝑐 , 𝑙𝑑 , 𝑙𝑒 and 𝑙𝑓 respectively

→ Train C can cross a pole in double time in which train E can cross a pole.
Time taken by train D to cross a platform of 180 m length is twice than
that time taken by same train to cross a pole. Train F can cross a
platform double of its length in 36 seconds. Speed of train ‘E’ is 20%
more than speed of train ‘C’. Train ‘A’ can cross a platform of 100 m
length in same time in which train ‘F’ can cross a pole.

→ Train ‘B’ can cross two poles 60 m apart from each other in 24 seconds.
Train ‘B’ can cross train ‘C’ in 10 seconds if they move towards each
other. Time taken by train ‘D’ to cross a platform of 300 m length is
same time in which train ‘C’ can cross a pole.

Given that
a:b→5:3 ⇒d:f→4:3
𝑃𝐴 ∶ 𝑃𝑏 → 2 ∶ 5 ⇒ 𝑙𝑑 ∶ 𝑙𝑒 → 1 ∶ 2
18 Adda247 Publications For any detail, mail us at
Publications@adda247.com
Cracker Book for Bank (IBPS | SBI | RRB PO | Clerk) Mains Exams

57. Train ‘C’ and Train ‘D’ enter in a tunnel from opposite sides at same
time. Find the length of tunnel if both trains completely take exit from
the tunnel at same time?
(a) 900 m (b) 1200 m (c) 1500 m
(d) 1800 m (e) 2100 m

58. Find the time in which train ‘B’ can cross a platform of same as length
as of train ‘F’?
(a) 50 seconds (b) 52 seconds (c) 54 seconds
(d) 56 seconds (e) 58 seconds

59. Train ‘A’ starts from station Rewari and move towards Gurgaon. After 4
hours Train ‘F’ starts from station Rewari and move towards Gurgaon.
After how much time Train ‘F’ will overtake train ‘A’ if length of both
trains is taken as negligible?
(a) 5 hours after start of Train ‘F’
(b) 5 hours after start of train ‘A’
(c) 9 hours after start of train ‘F’
(d) 9 hours after start of train ‘A’
(e) Both (a) and (d)

60. Time taken by train ‘A’ to cross a platform of length 250 m length is
what percent less than time taken by train ‘D’ to overtake train ‘F’ when
they move in same direction?
(a) 37.5% (b) 62.5% (c) 50%
2 2
(d) 166 % (e) 266 %
3 3

61. Train ‘C’ starts from Gurgaon station while train ‘D’ starts from Delhi
station at same time. Both starts to move towards each other. After how
much time trains will completely cross each other if distance between
Delhi and Gurgaon is 30km?
(a) 208 seconds (b) 218 seconds (c) 223 seconds
(d) 228 seconds (e) 238 seconds

19 Adda247 Publications For any detail, mail us at


Publications@adda247.com
Cracker Book for Bank (IBPS | SBI | RRB PO | Clerk) Mains Exams

Directions (62-66) Satish, Abhi and Bhavya are three persons who each
invested some amount in three different schemes (P, Q and R). Data tells
about amount invested and time of investment by them. Study the data
carefully & answer the following questions.

→ Amount invested by Satish in scheme ‘P’ is half than that amount


invested by Bhavya in same scheme. Abhi invested Rs 80,000 in scheme
‘P’ and time for which he invested in scheme ‘P’ is 5 months more than
time for which Satish invested in same scheme. Ratio of amount
invested by Satish and Bhavya in Scheme ‘R’ is same as ratio of amount
invested by Satish and Bhavya in scheme ‘P’. Abhi invested 20,000 less
in scheme ‘R’ than that of amount invested by Bhavya in scheme ‘R’.

→ Time of investment of Abhi and Bhavya is same in scheme ‘Q’. Amount


invested by Abhi in scheme ‘Q’ is 96% of amount invested by Satish in
scheme ‘Q’. Bhavya invested Rs 1,00,000 more than Abhi in scheme ‘Q’.
Ratio of amount invested by Satish in scheme ‘P’ to scheme ‘Q’ is 3 : 5.
Ratio of amount invested by Bhavya in scheme ‘P’ to scheme ‘Q’ is 15 :
22.

→ Out of total profit earned from scheme ‘Q’ Bhavya got 50%. Ratio of
profit share of Satish and Abhi is 3 : 4 in scheme ‘R’ while time of
investment of Satish and Abhi is scheme ‘R’ is 4 : 3. Bhavya invested for
10 month in scheme ‘R’.

Satish got 25% of total profit both in scheme ‘R’ as well as in scheme ‘P’.
Time of investment of Satish in scheme ‘P’ is 2 months less that of in
scheme ‘R’. Satish invested for 16 months in scheme ‘Q’

62. Abhi and Bhavya both invested same amount they invested in scheme
‘Q’ in two different schemes i.e, S1 and S2 respectively. S1 and S2 offers
20% p.a at C.I and 25% p.a at S.I respectively. Find the difference
between interest earned by both after 2 years?
(a) 36,800 (b) 41,900 (c) 47,000
(d) 52,100 (e) 57,200
20 Adda247 Publications For any detail, mail us at
Publications@adda247.com
Cracker Book for Bank (IBPS | SBI | RRB PO | Clerk) Mains Exams

63. If Bhavya earned Rs. 28,980 profits from scheme ‘R’ which is 125%
more than profit earned by him from scheme ‘P’, then find total profit
earned by Satish from scheme ‘P’ and ‘R’ together?
(a) 33,948 (b) 26,588 (c) 30,268
(d) 22,908 (e) 37,628

64. Find interest earned by Satish if scheme ‘Q’ offers S.I. at the rate of 15%
p.a?
(a) Rs26500 (b) Rs29000 (c) Rs28000
(d) Rs25000 (e) Rs27500

65. Amount invested by Bhavya in scheme ‘R’ is what percent more than
amount invested by Satish in scheme ‘P’?
(a) 140% (b) 100% (c) 144%
(d) 50% (e) 20%

66. If Abhi doubles his investment in scheme ‘P’ than before, then find what
percent of decrement is seen in Bhavya’s profit percentage?
4
(a) 20% (b) 40% (c) 28 %
7
6 1
(d) 42 % (e) 57 %
7 7

21 Adda247 Publications For any detail, mail us at


Publications@adda247.com
Cracker Book for Bank (IBPS | SBI | RRB PO | Clerk) Mains Exams

Solutions
Solutions (1-5)
Let investment of Rahul, Sandy and Sati be 2x, 3x and 4x respectively.
Ratio of profit

Rahul : Sandy : Sati


2x × 4 : 3x × 4 : 4x × 10
+(2x – 800)×6 +(3x+1500)× 6 +(5x × 2)
+(5x–800)× 2 (9x +1500) × 2
30x – 6400 : 48x + 12000 : 50x

ATQ,
50x 125
= ⇒ x = 250
128x+5600 376
Ratio of profit share of Rahul, Sandy and Sati is
1100 ∶ 24000 ∶ 12500 → 11 ∶ 240 ∶ 125

240
1. (a); Required percentage = × 100
376
= 63.829% ≃ 64%

2. (d); Required difference


240−11
= × 37600 = 22,900
376

3. (c); Investment of Veer


350
= 4 × 250 × = 3500
100
Interest earned by Veer
20 2
= 3500 [1 + ] − 3500 = 1540
100

240+125 37600
4. (b); Required average = × =18,250
2 376

2x+3x+4x
5. (e); Investment of Bhavya = = 3x
3
= 3 × 250 = Rs.750
Required profit = 750 - 2× 250 = 750 – 500 = 250
22 Adda247 Publications For any detail, mail us at
Publications@adda247.com
Cracker Book for Bank (IBPS | SBI | RRB PO | Clerk) Mains Exams

Solutions (6–10):

Total people = 2000


Let total people watching only ‘PK’ = a
Total number of people watching ‘Ravan’ = 720
Ratio between number of people watching only ‘3 Idiiots’ and ‘Ravan’ to
number of people watching only ‘Ravan’ and ‘PK’ = 2 : 3

𝑎
Number of people watching ‘3 Idiot’ and ‘PK’ =
2
Number of people watching all three movies
4
= 2000 × = 80
100

Number of people watching ‘Ravan’ only = number of people watching


‘Ravan’ and ‘PK’ = 3x
2
Number of people = watching ‘3 Idiots’ and ‘Ravan’ only = × 3x = 2x
3

Number of people watching ‘3 Idiots’ only


150 3
= Number of people watching ‘Ravan’ only × = 3𝑥 × = 4.5x
100 2

Now, 3x + 3x + 2x + 80 = 720
640
8x = 720 – 80 ⇒ x = ⇒ x = 80
8

23 Adda247 Publications For any detail, mail us at


Publications@adda247.com
Cracker Book for Bank (IBPS | SBI | RRB PO | Clerk) Mains Exams

Now,
a
12.5x + a + + a = 2000 - 80
2
5a
= (1920 – 1000) ⇒ a = 368
2

6. (d); People watching only Ravan, only PK and only 3 Idiots


= 240 + 360 + 368 = 1000
968 −368
Required percentage = × 100 = 62%
968

240+368
7. (e); Required ratio = = 76 : 43
160+184

8. (d); Total number of males , who watching 3 idiots and Ravan only
4 5
= 360 × + 240 = 438
5 8

240 −160 80
9. (a); Required percentage = × 100 = × 100 = 50%
160 160

10. (b); Required difference


= 360 + 184 + 368 – 240 – 160 – 80 – 240
= 912 – 720 = 192

Solutions (11-15):

Total students appeared in 2016 = 8000


Total students appeared in 2013 = 5800
Total students appeared in exam B is 2011 & 2013 = 6200

24 Adda247 Publications For any detail, mail us at


Publications@adda247.com
Cracker Book for Bank (IBPS | SBI | RRB PO | Clerk) Mains Exams
6200
Total students appeared in exam B in 2011 = × 18 = 3600
31
6200
Total students appeared in exam B in 2013 = × 13 = 2600
31
Total students appeared in exam A in 2013 = 5800 – 2600 = 3200
8000
Total students appeared 2011 = × 100 = 6400
125
Total students appeared in exam A in 2011 = 6400 – 3600 = 2800
8000
Total students appeared in 2014 = × 13= 6500
16
Students appeared in exam B in 2011
= Students appeared in exam A in 2015 = 3600
3600
Students appeared in exam B in 2015 = × 3 = 2700
4
1700
Students appear in exam A in 2016 = [1 + ] × 2700 = 4400
2700
Students appear in exam B in 2016 = 8000 – 4400 = 3600
Let, student appeared in exam A in 2014 = x
student appeared in exam A in 2012 = x + 700
⇒ x + x + 700 + 2800 + 3200 + 3600 + 4400 = 21,100
2x = 6400 ⇒ x = 3200
Students appeared in exam A in 2014 = 3200
Students appeared in exam A in 2012 = 3200 + 700 = 3900
Students appeared in exam B in 2014 = 6500 – 3200 = 3300
Students appeared in exam B in 2012 = 3300 + 1200 = 4500

A B Total
2011 2800 3600 6400
2012 3900 4500 8400
2013 3200 2600 5800
2014 3200 3300 6500
2015 3600 2700 6300
2016 4400 3600 8000
Total 21,100 20,300

11. (b); According to table its in 2014.

2800 + 3200 + 3900 9900 99


12. (d); Required ratio = = =
2600 + 3300 + 3600 9500 95

25 Adda247 Publications For any detail, mail us at


Publications@adda247.com
Cracker Book for Bank (IBPS | SBI | RRB PO | Clerk) Mains Exams

13. (b); Average students appeared in exam A in starting four years


2800+3900+3200+3200
= = 3275
4
Average students appeared in exam B in starting four years
3600+4500+2600+3300
= = 3500
4
Required difference = 225.

14. (e); Required difference = 4500 – 3200 = 1300

8400 –6400 2000


15. (c); Required % = × 100 = × 100 = 31.25%
6400 6400

Solutions (16-20):

Total students = 1000


Let, students appear in exam Z only = a
Total students appeared in exam Y = 360
Ratio of number of students appeared in exam X and Y only to students
appeared in exam Y and Z only = 2 : 3
Students appeared in exam X and Z both = a/2
4
Number of students appeared in all three exams = × 1000 = 40
100
Number of students appeared in Y exam only
= No. of students appeared in Y and Z only = 3x
2
Number of students appeared in exam X and Y only = × 3x = 2x
3

Now, 2x + 3x + 3x + 40 = 360 ⇒ x = 40
a
and, 12.5x + a + + a = 1000
2

26 Adda247 Publications For any detail, mail us at


Publications@adda247.com
Cracker Book for Bank (IBPS | SBI | RRB PO | Clerk) Mains Exams
5a
= 500 ⇒ a = 200
2

16 (c); Students appeared in atleast two exams = 80 + 60 + 40 + 120 = 300

17. (e); Students appeared in two exams only = 80 + 60 + 120 = 260

18. (e); Students appeared in atmost two exams


= 180 + 120 + 200 + 60 + 80 + 120 + 200 = 960

19. (d); Student not appeared in exam Y = 1000 – 360 = 640

20. (d); Students appeared in exam X or in exam Z


= 180 + 60 + 40 + 80 + 200 + 120 = 680

Solutions (21-25):

No. of songs recorded by Sonu Nigam in 2008 = 300


2
No. of songs recorded by Arijit Singh in 2009 = × 300 = 120
5
Total number of songs recorded in 2008 = 6 × 120 = 720
Let number of songs recorded by Arijit Singh in 2008 be 𝑥.
75
∴𝑥+ 𝑥 = (720– 300)
100
∴ 𝑥 = 240
∴ No. of songs recorded by Arijit Singh and Sandeep in 2008 are 240 & 180
respectively

27 Adda247 Publications For any detail, mail us at


Publications@adda247.com
Cracker Book for Bank (IBPS | SBI | RRB PO | Clerk) Mains Exams
350
Total no. of songs recorded in 2009 = × 120 = 420
100
2
No. of songs recorded by Sonu Nigam in 2009 = × (420– 120) = 120
5
∴ No. of songs recorded by Sandeep in 2009 = 180
4
Total no. of songs recorded in 2010 = 3 × × 240 = 960
3
No. of songs recorded by Sandeep in 2010 = 650 – 180 – 180 = 290
290
No. of songs recorded by Arijit Singh in 2010 = × 4 = 232.
5
No. of songs recorded by Sonu Nigam in 2010
= 960 – 290 – 232 = 438

Years
2008 2009 2010
Singers
Sonu Nigam 300 120 438
Arijit Singh 240 120 232
Sandeep 180 180 290
Total 720 420 960

720–(240+120+232) 7
21. (b); Required percentage = × 100 = 17 %
720 9

438+2321 180+180
22. (a); Required difference = ( )–( ) = 335 – 180 = 155
2 2

960
23. (e); Required ratio = 150 = 32 : 21
420 ×
100

24. (a); Average number of songs recorded by Sonu Nigam


300+120+438 858
= = = 286
3 3
420
Average number of songs recorded in 2009. = = 140
3
286–140
Required percentage = × 100 ≃ 104%
140

25. (d); Required difference


720+420+960 120+180
=( )–( ) = 700 – 150 = 550
3 2

28 Adda247 Publications For any detail, mail us at


Publications@adda247.com
Cracker Book for Bank (IBPS | SBI | RRB PO | Clerk) Mains Exams

Solutions (26-30):

Total student in school A = 1600


40
Student who knows only Hindi in C = × 1600 = 640
100
Let student who knows only Hindi & only English in school A be 2𝑥 and 3𝑥
respectively.
∴ Total student who knows both language in school A = 5𝑥

ATQ,
10𝑥 = 1600 ⇒ 𝑥 = 160
∴ Student who knows only Hindi in A = 320
& only English in A = 480
Both language in A = 800
Student who knows only Hindi in B = 800 + 320 – 640 – 320 = 160
Total student who knows Only English = 3 × 460 = 1380
1
Student who knows only English in B = (1380– 480) = 180
5
Student who knows only English in C = 900 – 180 = 720
320
Student who knows both language in C = = 160
2
2
Total student in school B = 1120 × = 840
4
∴ Student who knows both language in B = 840 – 160 – 180 = 500

School Only Hindi Only English Both Hindi and English Total
A 320 480 800 1600
B 160 180 500 840
C 640 720 160 1520
Total 1120 1380 1460

(320+480)–500 300
26. (b); Required percentage = × 100 = × 100 = 60%
500 500

180+720 900
27. (e); Required ratio = = = 15 : 14
840 840

1380 3
28. (a); Required sum = + [160 + 180 + 640 + 720]
3 4
= 460 + 1275 = 1735
29 Adda247 Publications For any detail, mail us at
Publications@adda247.com
Cracker Book for Bank (IBPS | SBI | RRB PO | Clerk) Mains Exams
40 1
×840– ×320 336–160
100 2
29. (b); Required percentage = 1 × 100 = × 100 = 110%
×320 160
2

1600+840+1520 500+160
30. (d); Required difference= ( )–( ) = 1320 – 330 = 990
3 2

Solutions (31-35):

Total amount invested by ‘Rahul’, ‘Veer’ and ‘Anurag’ is in the ratio 4 : 5 : 6.


Let total amount invested by Rahul, Veer and Anurag in all the three
schemes be 16x, 20x and 24x
25
Amount invested by Rahul in scheme ‘X” = × 16𝑥 = 4𝑥
100
16𝑥−4𝑥
Amount invested by Rahul in scheme ‘Y’ or ‘Z’ = = 6𝑥
2
Let Amount invested by Veer in scheme Y and Z be 3y and 2y respectively
3𝑦 5𝑦
Then amount invested by Veer in scheme X is = × 100 =
180 3
5𝑦
Ratio between amount invested by Veer in scheme ‘X’, ‘Y’ and ‘Z’ = ∶ 3𝑦 ∶
3
2𝑦 → 5 ∶ 9 ∶ 6
Amount invested by Veer in scheme ‘X’, ‘Y’ and ‘Z’ is 5x, 9x and 6x
respectively.
5𝑥
Amount invested by Anurag in scheme ‘X’ = × 8 = 8𝑥
5
Amount invested by Anurag in scheme ‘Y’ and ‘Z’ together = 24𝑥–8𝑥 = 16𝑥
Amount invested by Rahul in scheme ‘Y’ and ‘Z’ together = 12𝑥

ATQ,
16𝑥 − 12𝑥 = 20,000 ⇒ 𝑥 = 5,000
Total amount invested by Anurag in scheme ‘Y’ and ‘Z’ together = 80,000
Let amount invested by Anurag in scheme ‘Z’ = ‘a’
Amount invested by Anurag in scheme ‘Y’ = 3𝑎
⇒ 𝑎 + 3𝑎 = 80,000 ⇒ 𝑎 = 20,000

X Y Z
Rahul 20,000 30,000 30,000
Veer 25,000 45,000 30,000
Anurag 40,000 60,000 20,000

30 Adda247 Publications For any detail, mail us at


Publications@adda247.com
Cracker Book for Bank (IBPS | SBI | RRB PO | Clerk) Mains Exams

31. (c); Required interest


20,000×10×2 30,000×20×2 30,000×30×2
= + +
100 100 100
= 4,000 + 12,000 + 18,000 = Rs. 34,000

32. (a); Ratio between profit share of Veer and Anurag


Veer : Anurag
→ 25,000 × 8 ∶ 40,000 × 12 → 5 ∶ 12
5
Profit share of Veer = × 7650 = Rs 2,250
17

33. (d); Total amount invested in scheme ‘Y’


= 30,000 + 45,000 + 60,000 = 1,35,000
Total amount invested in scheme ‘Z’
= 30,000 + 30,000 + 20,000 = 80,000
1,35,000−80,000 55,000
Required % = × 100 = × 100 = 68.75%
80,000 80,000

25,000 5
34. (e); Required Ratio = =
30,000 6

20 2
35. (b); Interest earned by Veer = 30,000 × [1 + ] − 30,000 = 13,200
100
20 2
Interest earned by Anurag = 20,000 × [1 + ] − 20,000 = 8,800
100
13,200−8,800
Required % = × 100 = 50%
8,800

Solutions (36 – 40)

Vessel M
Water ⇒ 40 liters
40
So milk in M ⇒ × 7 = 140 liters
2
Total quantity in vessel M = 180 liters
Total quantity quantity in vessel Q
180×2
= = 120 liters
3
Scotch in vessel Q = 30 liters
Apple juice in vessel Q = 90 liters

31 Adda247 Publications For any detail, mail us at


Publications@adda247.com
Cracker Book for Bank (IBPS | SBI | RRB PO | Clerk) Mains Exams

In vessel N
Total quantity = 70 + 140 = 210 liter
Water = 140 liters
Orange juice = 70 liters

In Vessel O
Total quantity = 210 + 40 = 240 liters
Orange juice in vessel O : Orange juice in vessel N = 7 : 15
70
Orange juice in vessel O = × 15 = 150 liters
7
Vodoka in vessel = 240 – 150 = 90 liters

In Vessel P
2
Total quantity = 66 % of total quantity of vessel P
3
2
= × 240 = 160
3
2
Vodka = of vodka in vessel O
3
2
90 × = 60 liters
3
Apple juice = 160 – 60 = 100 liters

M N O P Q
Milk 140 — — — —
Water 40 140 — — —
Orange juice — 70 150 — —
Vodka — — 90 60 —
Apple Juice — — — 100 90
Scotch — — — — 30

36. (b); In vessel G—


7
Milk in 72 liter of mixture = 72 × = 56 liter
9
2
Water in 72 liter of mixture = 72 × = 16 liter
9
2
Water in 36 liter of mixture =36 × =24 ltr.
3
1
Orange juice in 36 liter of mixture = 36 × = 12 liter
3
Ratio of milk : water : Orange juice in vessel G = 56 : (16 + 24) : 12 =
14 : 10 : 3
32 Adda247 Publications For any detail, mail us at
Publications@adda247.com
Cracker Book for Bank (IBPS | SBI | RRB PO | Clerk) Mains Exams
3
37. (b); Apple juice in vessel C = 36 × = 27 liter
4
1
Scotch in vessel C = 36 × + 21 = 9 + 21 = 30 liter
4
27+𝑥 2
Let 𝑥 liter of apple juice mixed = =
30 1
𝑥 = 60 – 27 ⇒ 𝑥 = 33 liter

38. (a); Ratio of water, Orange juice and Vodka vessel Z


= 140 : (70 + 150) : 90 = 14 : 22 : 9
Vodka in 22.5 liter of mixture
9 9
= 22.5 × (14+22+9) = 22.5 × = 4.5 liter
45

39. (e); Ratio of orange juice, Vodka, apple juice & scotch in vessel A
5 3 3 5 3 1
=[(64 × ) : (64 × + 48 × ) : (48 × + 60 × ) ∶ 60 × ]
8 8 8 8 4 4
= [ 40 : (24 + 18) : (30 + 45) : 15) = 40 : 42 : 75 : 15

Solutions (40-42):

Total players participated in 2006 = 6000


Total players participated in 2003 = 4800
Total players participated for national level Championship in 2001 & 2003
= 3600
Total players participated for National level championship in 2001
11
= 3600 × = 2200
18
Total players participated for national Level championship in 2003
7
= 3600 × = 1400
18
Total player participated for state level championship in 2003
= 4800 – 1400 = 3400
Total player participated in 2001
100
= 6000 × = 5000
120
Total player participated in 2001 for state level championship
= 5000 – 2200 = 2800
Total player participated in 2004
6000
= × 13 = 5200
15

33 Adda247 Publications For any detail, mail us at


Publications@adda247.com
Cracker Book for Bank (IBPS | SBI | RRB PO | Clerk) Mains Exams

Players participated for national championship in 2001


= Players participated for state championship in 2005 = 2200
2200
Player participated for national championship in 2005 = × 4 = 1760
5
Player participated for state level championship in 2006
5 13
= [1 + ] × 1760 = × 1760 = 220 × 13 = 2860
8 8
Player participated for national championship in 2006 = 6000–2860 = 3140
Let players participated for state level championship in 2004 = X
Players participated for state level championship in 2002 = X + 800
⇒ X + X + 800 + 3400 + 2800 + 2200 + 2860 =20000
2X = 20000 – 12060
7940
X= ⇒ X = 3970
2
Player participated for state championship level in 2004 = 3970
Player participated for state level championship in 2002= 3970+800 = 4770
Players participated for national level championship in 2004 = 5200 – 3970
= 1230
Players participated for national level championship in 2002 = 1230 + 1200
= 2430

state level National Level


Years Total
championship Championship
2001 2800 2200 5000
2002 4770 2430 7200
2003 3400 1400 4800
2004 3970 1230 5200
2005 2200 1760 3960
2006 2860 3140 6000

2800+4770+3970 11540
40. (c); Required ratio = = = 1154 : 439
1400+1230+1760 4390

41. (b); Required difference


2800+4770+3400+3970 2200+2430+1400+1230
=( )–( )= 3735 – 1815 = 1920
4 4

42. (e); Required difference = 3400 – 2430 = 970

34 Adda247 Publications For any detail, mail us at


Publications@adda247.com
Cracker Book for Bank (IBPS | SBI | RRB PO | Clerk) Mains Exams

Solutions (43-47):

15860−1180
2015 → Production of wheat in Punjab = = 7340 thousands ton
2
Production of wheat in Uttar Pradesh = 734 + 1180 = 8520 thousands ton
75
Production of wheat in Madhya Pradesh= 8520 × = 6390 thousands ton
100
Production of wheat in Haryana = 6390 – 690 = 5700 thousands ton
70
Production of wheat in Rajasthan = 5700 × = 3990 thousands ton
100

80
2016 → Production of wheat in Punjab = 7340 × = 5872 thousands ton
100
Production of wheat in Haryana = 5700 + 800 = 6500 thousands ton
4
Production of wheat in Madya Pradesh= 6390 × = 8520 thousands ton
3
Production of wheat in Rajasthan = 3990 + 2390 = 6380 thousands ton
Production of wheat in Uttar Pradesh
= 36612 – (5872 + 6500 + 8520 + 6380) = 9340 thousands ton

16520−2840
2017 → Production of wheat in Haryana = = 6840 thousands ton
2
Production of wheat in uttar Pradesh = 6840 + 2840 = 9680 thousands ton
Production of Wheat in Punjab = 5872 + 628 = 6500 thousands ton
Production of wheat in Madhya Pradesh= 8520 + 360 = 8880 thousands ton
Production of wheat in Rajasthan
= 38200 – (6840 – 9680 + 6500 + 8880) = 6300 thousands ton

Production of wheat (thousands ton)


States
2015 2016 2017
Uttar Pradesh 8520 9340 9680
Punjab 7340 5872 6500
Madhya Pradesh 6390 8520 8880
Haryana 5700 6500 6840
Rajasthan 3990 6380 6300

35 Adda247 Publications For any detail, mail us at


Publications@adda247.com
Cracker Book for Bank (IBPS | SBI | RRB PO | Clerk) Mains Exams

43. (b); Total wheat produced by Uttar Pradesh in all three years
= 8520 + 9340 + 9680
= 27540 thousands ton
Total wheat produced by Madhya Pradesh in all three years
= 6390 + 8520 + 8880
= 23790 thousands ton
Required difference = 27540 – 23790
= 3750 thousands ton

44. (d); Total wheat produced by Madhya Pradesh in year 2016 & 2017
together
= 8520 + 8880 = 17400 thousands ton
Total wheat produced by Haryana in the year 2015 & 2016
together = 5700 + 6500
= 12200 thousands ton
17400 –12200 5200 38
Required% = × 100 = × 100 = 42 %
12200 12200 61

45. (e); Average quantity of wheat produced by Uttar Pradesh, Madhya


Pradesh and Haryana in the year 2015
8520+6390+5700 20610
= = = 6870 thousands ton
3 3
Average quantity of wheat produced by Madhya Pradesh, Haryana
and Rajasthan in the year 2017
6840+6300+8880 22020
= = = 7340 thousands ton
3 3
Required difference = 7340 – 6870 = 470 thousands ton

6300+6500 12800
46. (a); Required ratio = = = 160 : 161
6500+6380 12880

47. (c); Total Wheat produced by Rajasthan and Uttar Pradesh in the year
2016 & 2017 together = (9340 + 6380) + (9680 + 6300)
= 31700 thousands ton
36 Adda247 Publications For any detail, mail us at
Publications@adda247.com
Cracker Book for Bank (IBPS | SBI | RRB PO | Clerk) Mains Exams

Solutions (48-51):

2012 →
Let total number of candidates selected from Kerala be 5𝑥 and from Uttar
Pradesh be 6𝑥

ATQ—
5𝑥 + 6𝑥 = 440 ⇒ 𝑥 = 40
Total number of selected candidate from Uttar Pradesh = 240
Total number of selected candidate from Kerala = 200
Total number of candidate selected from Maharashtra
= 220 × 3 – (240 + 200) = 660 – 440 = 220
Total number of candidates selected from Madhya Pradesh = 220 – 70= 150
Total number of candidates selected from Bihar
= 930 – (240 + 200 + 220 + 150) = 930 – 810 = 120

2013 →
4
Total number of candidates selected from Uttar Pradesh = 240 × = 320
3
Total number of candidates selected from Kerala
(200+220)
= – 30 = 210 – 30 = 180
2
Total number of candidates selected from Maharashtra = 220 + 55 = 275
Total number of candidates selected from Madhya Pradesh = 180
Total number of candidates selected from Bihar
= 1130 – (320 + 180 + 275 + 180) = 1130 – (955) = 175

2014 →
Total number of candidates selected from Uttar Pradesh = 320 + 40 = 360
Total number of candidate selected from Maharashtra = 275 – 25 = 250
Total number of candidate selected from Kerala
= 280 × 3 – (360 + 250) = 840 – 610 = 230
Total number of candidate selected from Madhya Pradesh and Bihar = 195
× 2 = 390
390–20
Total number of candidate selected from Madhya Pradesh = = 185
2
Total number of candidate selected from Bihar = 185 + 20 = 205

37 Adda247 Publications For any detail, mail us at


Publications@adda247.com
Cracker Book for Bank (IBPS | SBI | RRB PO | Clerk) Mains Exams

48. (b); Total number of candidates selected from Kerala & Maharashtra
together in 2014
= (250 + 230) = 480
Total number of candidates selected from Uttar Pradesh in 2012 &
2014 together
= 240 + 360 = 600
600 –480
Required percentage = ×100 =20%
600

49. (b); Total number of candidate selected from Karnataka in the year
2014
150
= 180 × = 270
100
270+360+185+205 1020
Required average = = = 255
4 4

(200+180+230) 610
50. (c); Required ratio = (120+175+205) = = 61 : 50
500

51. (b); Total number of candidates selected from Maharashtra, Madhya


Pradesh and Bihar together in 2013 = (275 + 180 + 175) =630
Total number of candidates selected from Madhya Pradesh in the
year 2012 & 2013 together = (150 + 180) = 330
630 –330 300 1000 10
Required percentage = × 100 = × 100 = = 90 %
330 330 11 11

Solution (52-56):

Volume of hollow spherical toy = 33,957𝑐𝑚2


Let Outer radius of hollow spherical toy = R
𝑅
Inner radius of hollow spherical toy =
2

38 Adda247 Publications For any detail, mail us at


Publications@adda247.com
Cracker Book for Bank (IBPS | SBI | RRB PO | Clerk) Mains Exams

ATQ,
4 𝑅 3
𝜋 (𝑅3 − ( ) ) = 33,957
3 2
3
⇒ R = 9261 ⇒ R = 21
Outer radius of hollow spherical toy = 21cm
Inner radius of hollow spherical toy = 10.5 cm
Volume of conical toy
33957 14
= = 6468 = 𝜋(𝑟𝑎𝑑𝑖𝑢𝑠 𝑜𝑓 𝑐𝑜𝑛𝑒)2 ×
5.25 3
⇒ Radius of cone = 21 cm
6468
Volume of solid cylindrical toy = = 2156
3
= π (radius of cylinder)² × (height of cylinder)
⇒ Radius of Cylinder = 7cm
21
Radius of Solid Spherical toy = = 10.5𝑐𝑚
2
Outer Radius of hollow cylindrical toy = 10.5𝑐𝑚
Inner radius of hollow cylindrical toy = 7 × 2 − 10.5 = 3.5 cm

Let total number of toys = 100x


ATQ,
Number of solid spherical toys = 20x
Number of hollow cylindrical toys = 40x

ATQ,
40𝑥 − 20𝑥 = 20 ⇒ 𝑥 = 1
Let Number pf conical toys = 2𝑦
Number of hollow spherical toys = 5𝑦
Number of solid cylindrical toys = 3𝑦

ATQ,
2𝑦 + 5𝑦 + 3𝑦 = 100 − 40 − 20 = 40 ⇒ 𝑦 = 4

39 Adda247 Publications For any detail, mail us at


Publications@adda247.com
Cracker Book for Bank (IBPS | SBI | RRB PO | Clerk) Mains Exams

Toy Number of Toys


Conical 8
Solid Cylindrical 12
Solid Spherical 20
Hollow Cylindrical 40
Hollow Spherical 20

52. (a); Space taken by one solid spherical toy


= Volume of one solid spherical toy
4
= 𝜋(10.5)3 = 4851𝑐𝑚3
3
Total space taken by solid spherical toys
= 20 × 4851 = 97020𝑐𝑚2

53. (e); Number of conical toys Neeraj have = 8

54. (b); Curved surface area of one hollow cylindrical toy


= 2𝜋 × (3.5 + 10.5) × 14 = 1232 cm2
21 3
55. (c); Required Ratio = =
7 1

56. (d); Volume of one hollow cylindrical toy


= 𝜋 × 14 × (10.52 − 3.52 ) = 4312 cm3
Required difference
= 4312 − 2156 = 2156𝑐𝑚3

Solution (57-61): Time taken by train ‘D’ to cross a platform of 180 m


length is twice than that time taken by same train to cross a pole.
ℓ +180 2ℓ
⇒ d = d ⇒ ℓd = 180m
d d
But, ℓd ∶ ℓe = 1 ∶ 2 ⇒ ℓe = 360m
Train ‘F’ can cross a platform double of its length in 36 seconds
ℓ +2ℓ ℓ
⇒ f f = 36 ⇒ f = 12 seconds
f f
⇒ Train ‘F’ can cross a pole in 12 seconds
Train ‘A’ can cross a platform of 100m length in same time in which train ‘F’
can cross a pole i.e, 12 seconds
ℓ +100
⇒ A = 12
a

40 Adda247 Publications For any detail, mail us at


Publications@adda247.com
Cracker Book for Bank (IBPS | SBI | RRB PO | Clerk) Mains Exams

Train ‘B’ can cross two poles 60 m apart from each other in 24 seconds
ℓ +60
⇒ 24 = B
b
But a : b → 5 : 3
And, PA : PB → 2 ∶ 5
ℓa
a 2 ℓa b ℓa 2 2
ℓb ⇒ = × ⇒ = ⇒ ℓa = ℓb
5 a ℓb ℓb 3 3
b

And,
2(ℓA +100) (ℓb +60) 5 3ℓa
= ⇒ 2(ℓa + 100) = ( + 60)
a b 3 2
15 3
6ℓa + 600 = ℓa + 300 ⇒ 300 = ℓa
2 2
⇒ ℓa = 200m

And, ℓb = 300m
200+100 300
a= = = 25 m/sec
12 12
25
b = × 3 = 15 m/sec
5
Train ‘C’ can cross a pole in double time in which train ‘E’ can cross a pole.
ℓ 2ℓ
⇒ C= e
c e
But,
e = 1.2c
ℓ 2ℓ 5
⇒ c = e ⇒ ℓc = ℓe
c 1.2c 3

But ℓe = 360 m
⇒ ℓc = 600m
Train ‘B’ can cross train ‘C’ in 10 seconds if they move in opposite direction
ℓ +ℓ 300+600
⇒ b c=b+c⇒ = 15 + c ⇒ 90 – 15 = C ⇒ c = 75 m/sec
10 10
Time train by train ‘D’ to cross a platform of 300m is same time in which
train ‘C’ can cross a pole.
ℓd +300 ℓ 180+300 600 480
⇒ = c⇒ = ⇒d= = 60 m/sec
d c d 75 8
d:f→4:3
60
⇒ f = × 3 = 45 m/sec
4
ℓf
= 12 ⇒ ℓf = 540 m
f

41 Adda247 Publications For any detail, mail us at


Publications@adda247.com
Cracker Book for Bank (IBPS | SBI | RRB PO | Clerk) Mains Exams

But
e = 1.2c ⇒ e = 1.2 × 75 = 90 m/sec

Train Speed (in m/sec) Length (in m)


A 25 200
B 15 300
C 75 600
D 60 180
E 90 360
F 45 540
57. (c); Let length of tunnel is ‘x’ meter
Time taken by train ‘C’ to cross tunnel is same as time taken by
train ‘D’ to cross tunnel
180+𝑥 600+𝑥
= ⇒ 900 + 5𝑥 = 2400 + 4𝑥
60 75
𝑥 = 1500 𝑚
300+540 840
58. (d); Required time = = = 56 seconds
15 15

59. (e); Speed of train ‘A’ = 25m/sec = 90km /hr


Distance covered by train ‘A’ in 4 hours
= 90 × 4 = 360 km
Let Train ‘F’ will overtake train ‘A’ in ‘x’ time
360×5
Then, 𝑥 = (45−25)×18 =5 hours
Train ‘F’ meet train ‘A’ after 5 hours start of train ‘F’ or after 9
hours start of train ‘A’

60. (b); Time taken by train ‘A’ to cross a platform of 250m length
200+250
= = 18
25
Time taken by train ‘D’ to overtake train ‘F’
180+540
= = 48sec
60−45
48−18
Required % = × 100 = 62.5%
48

30,000+600+180
61. (d); Required time = = 228 seconds
75+60
42 Adda247 Publications For any detail, mail us at
Publications@adda247.com
Cracker Book for Bank (IBPS | SBI | RRB PO | Clerk) Mains Exams

Solutions (62-66):
Ratio of amount invested by Satish and Bhavya in scheme ‘P’ and in ‘R’ is
same that is 1 : 2
Let Bhavya and Satish invested 2z and z in scheme ‘R’
⇒ Amount invested by Abhi in scheme ‘R’ is (2z – 20,000)
Ratio of profit share of Satish and Abhi in scheme ‘R’ is 3 : 4 while time of
investment of Satish and Abhi in scheme ‘R’ is 4 : 3.
3 𝑧×4
⇒ = ⇒ 18z – 1,80,000 = 16z
4 (2𝑧−20,000)×3
z = 90,000

Amount invested Amount invested Amount invested by


Scheme
by Satish by Abhi Bhavya
R 90,000 1,60,000 1,80,000

In scheme R,
Bhavya invested for 10 months while Satish got 25% of profit out of total
profit
If ratio between Satish’s and Abhi’s profit share is 3 : 4, then Bhavya’s profit
3
share is × 100 − 7 = 5
25
⇒ Ratio of profit share of Satish, Abhi and Bhavya is 3 : 4 : 5.
Let Satish invested for ‘y’ month
1,80,000×10 5
= ⇒ y = 12 month
90,000×𝑦 3
12
Abhi invested for = × 3 = 9 month
4
Time of investment of Satish in scheme ‘P’ is 2 months less than that of in
scheme ‘Q’
⇒ Satish invested for 10 months in scheme ‘P’.
Time of investment of Abi in scheme ‘P’ is 5 months more than time for
Satish invested
⇒ Abhi invested for 15 months in scheme ‘P’
Let amount invested by Satish in scheme ‘Q’ is = 5b
5𝑏
⇒ Amount invested by Satish in scheme ‘P’ is = × 3 = 3𝑏
5
96
And, Amount invested by Abhi in scheme ‘Q’ is = 5𝑏 × = 4.8𝑏
100
Amount invested by Bhavya in scheme ‘Q’ is = 4.8b + 1,00,000

43 Adda247 Publications For any detail, mail us at


Publications@adda247.com
Cracker Book for Bank (IBPS | SBI | RRB PO | Clerk) Mains Exams

Amount invested by Bhavya in scheme ‘P’ is = 3b × 2 = 6b

ATQ,
6𝑏 15
= ⇒ 132b = 72b + 15,00, 000
4.8𝑏 + 1,00,000 22
b= 25,000

Amount Invested
Scheme Satish Abhi Bhavya
P 75,000 80,000 1,50,000
Q 1,25,000 1,20,000 2,20,000
R 90,000 1,60,000 1,80,000

Let, Bhavya invested for ‘y’ month in scheme ‘P’.

ATQ,
1 75,000×10
= ⇒ y = 7 months
4 75,000×10+80,000×15+1,50,000×𝑦
Time of investment of Abhi and Bhavya is same in scheme ‘Q’ and ratio
between profit sharing of Bhavya to total profit in scheme ‘Q’ is 1 : 2.
Let, Abhi and Bhavya invested for ‘5a’ months and Satish invested for ‘x’
month.

ATQ,
1 2,20,000×5𝑎
=
1 1,25,000×16+1,20,000×5𝑎
⇒ 1,25,000 × 16 = 1,00,000 × 5a ⇒ 5a= 20 months
So Bhavya and Veer invested in scheme ‘Q’ for 20 months

44 Adda247 Publications For any detail, mail us at


Publications@adda247.com
Cracker Book for Bank (IBPS | SBI | RRB PO | Clerk) Mains Exams
120 120
62. (e); Interest earned by Abhi = 1,20,000 × × − 1,20,000 = 52,800
100 100
Interest earned by Bhavya
25
= 2,20,000 × 2 × = 1,10,000
100
Required difference = 1,10,000 –52,800 = 57,200

63. (b); Ratio of profit sharing between Satish, Abhi and Bhavya in scheme
‘P’ and scheme ‘R’ is 5 : 8 : 7 and 3 : 4 : 5 respectively.
28,980
Profit earned by Satish from scheme ‘R’ = × 3 = 17,388
5
28980
Profit earned by Bhavya from scheme ‘P’ = × 100 = 12,880
225
12,880
Profit earned by Satish from scheme ‘P’ = × 5 = 9,200
7
Total profit earned by Satish from scheme ‘P’ and ‘R’ together
= 17,388 + 9,200 = 26,588
20
64. (d); Time of investment of Satish = × 4 = 16 months
5
15 16
Interest earned by Satish = 1,25,000 × × = 25,000
100 12

1,80,000−75,000 105
65. (a); Required % = × 100 = × 100 = 140%
75,000 75

66 (c); Ratio of profit sharing between Satihs, Abhi and Bhavya in scheme
‘P’ is 5 : 8 : 7
Let total profit = 20
7
Bhavya’s profit = × 100
20
= 35%
When Abhi doubles his investment then ratio of profit sharing
between Satish, Abhi and Bhavya in scheme ‘P’ is 5 : 16 : 7
Let total profit = 28
7
Bhavya’s profit = × 100 = 25%
28
35−25 4
% Decrement in profit % ×100 = 28 %
35 7

45 Adda247 Publications For any detail, mail us at


Publications@adda247.com
Cracker Book for Bank (IBPS | SBI | RRB PO | Clerk) Mains Exams

1 Adda247 Publications For any detail, mail us at


Publications@adda247.com
Cracker Book for Bank (IBPS | SBI | RRB PO | Clerk) Mains Exams

Chapter

1 Seating Arrangement

BEST APPROACH TO SOLVE THE QUESTIONS

Being a student, you should know about the recent changes that have been
seen in the recent exams i.e SBI PO/ IBPS PO/RRB Clerk/SBI Clerk/RBI
Grade B/Insurance etc. In recent exams, puzzles are tricky but not lengthy
or difficult. So, we are providing you some questions based on the latest
exams and also providing you the detail solution of it. So that you may get to
know how to solve these questions. Here is the approach to solve such
questions.

Example-1:

Directions: Study the information carefully and answer the questions given
below.
(SBI PO Mains-2018)
Certain number of persons are sitting around a circular table, which has a
circumference of 546 cm. All the persons are facing towards the center.
They are sitting at distances to each other which are consecutive multiple of
six. A is 3rd to the left of I. Two persons are sitting between K and I. M is
immediate right to L. H sits to the left of G at a distance of 72 cm. The
distance between A and D is 18 cm. The number of persons sitting between
J and B is same as between B and F. The distance between E and F is LCM of
6 and 5. Neither M nor L is neighbor of K and H. The number of persons
sitting between C and I is same as between I and E. The distance between K
and I is not more than 162cm. Either C or E is neighbor of K.

Explanation (The Approach):


Certain number of persons are sitting around a circular table but not given
the number of persons, but from some other information we can conclude
the number of persons in the circle. As it is given that all persons are sitting

2 Adda247 Publications For any detail, mail us at


Publications@adda247.com
Cracker Book for Bank (IBPS | SBI | RRB PO | Clerk) Mains Exams

at distances to each other which are consecutive multiple of six and


circumference of circle is 546 cm, so between persons distances are 6, 12,
18, 24, 30, 36, 42, 48, 54, 60, 66, 72 and 78. From that condition we deduce
that there are only 13 persons in the circle.
Now we try to solve Puzzle by using some other given information-
A is 3rd to the left of I. Two persons are sitting between K and I. The distance
between A and D is 18 cm. H sits to the left of G at a distance of 72 cm. The
distance between K and I is not more than 162cm. also persons are sitting
at distances to each other which are consecutive multiple of six, so we can
get-

Now from the other conditions-


The distance between E and F is LCM of 6 and 5 that means distance
between E and F is 30. From that Case- 3 is eliminated. Either C or E is
neighbor of K. The number of persons sitting between C and I is same as
between I and E. from that condition Case-2 and Case-4 is eliminated. So
rest Case-1 is left.
3 Adda247 Publications For any detail, mail us at
Publications@adda247.com
Cracker Book for Bank (IBPS | SBI | RRB PO | Clerk) Mains Exams

Neither M nor L is neighbor of K and H. M is immediate right to L. The


number of persons sitting between J and B is same as between B and F. So
the final arrangement are-

Example-2:
Directions: Study the following information carefully and answer the
questions given below.
(SBI Clerk Mains-2018)
There are eight members i.e. A, B, C, D, E, F, G and H are sitting around a
square table such that four of them likes flowers i.e. Lily, Rose, orchid and
Sunflower and four of them likes fruits i.e. Mango, Kiwi, Apple, Banana but
not necessarily in the same order. Those who likes Fruits sits at the corner
and those who like flower sits at the middle of the table. Some of them face
inside and some of them face outside.
The one who likes Orchid sits third to the right of H. A sit second to the right
of the one who likes Orchid. A is not the immediate neighbour of H. The one
4 Adda247 Publications For any detail, mail us at
Publications@adda247.com
Cracker Book for Bank (IBPS | SBI | RRB PO | Clerk) Mains Exams

who likes Banana is an immediate neighbour of A. The one who likes


Banana sits opposite to the one who likes Kiwi. H does not like Kiwi. The
one who likes Apple sits second to the left of the one who likes Kiwi, who is
not the immediate neighbour of A. B sits third to the right of the one who
likes Apple. F likes Lily. Only one person sits between B and the one who
likes Sunflower. F faces the one who likes Sunflower. E and G sits opposite
to each other. E does not like Banana. C sits second to the right of G. C does
not face Inside. C and D face same direction as G.

Explanation (The Approach):


Sol. The one who likes Orchid sits third to the right of H. A sit second to
the right of the one who likes Orchid. A is not the immediate neighbour
of H. The one who likes Banana is an immediate neighbour of A. The
one who likes Banana sits opposite to the one who likes Kiwi. H does
not like Kiwi. Those who likes Fruits sits at the corner and those who
like flower sits at the middle of the table. So we can get that-

From the other conditions, the one who likes Apple sits second to the left of
the one who likes Kiwi, who is not the immediate neighbour of A.

5 Adda247 Publications For any detail, mail us at


Publications@adda247.com
Cracker Book for Bank (IBPS | SBI | RRB PO | Clerk) Mains Exams

Now, B sits third to the right of the one who likes Apple. Only one person
sits between B and the one who likes Sunflower. F faces the one who likes
Sunflower. F likes Lily. From that conditions Case-3 and Case-4 are
eliminated. So, case-1 is continued.

E and G sits opposite to each other. E does not like Banana, So G likes
Banana. C sits second to the right of G. C does not face Inside. C and D face
same direction as G. From that conditions Case-1 is eliminated. Case-2 is
continued.
So, the final arrangement are-
6 Adda247 Publications For any detail, mail us at
Publications@adda247.com
Cracker Book for Bank (IBPS | SBI | RRB PO | Clerk) Mains Exams

Practice Exercise Based on new Pattern

Directions (1-5): Study the following information carefully and answer the
questions that follow:

Seven people having their name as consecutive alphabets, are sitting in


an alphabetical order from west to east direction in a straight line. Four of
them are facing south and remaining of them are facing north. All of them
are of different age. The person whose age is square of four sits second to
the left of K. Two persons sit between the one whose age is square of four
and the one whose age is six years. Both the immediate neighbours of the
one, whose age is six years faces opposite direction to each other (i.e. if one
faces to north then the other faces to south vice versa). L’s age is twice of
the age of the one who sit second left of K. The one whose age is four years
more than the half of L’s age sits second to the right to L. Only one person
sits between M and the one, whose age is 5/4 of L’s age. The persons whose
age is six years and the one whose age is 0.5 times of 18 are immediate
neighbours. The one, whose age is square of five sits third to the right of the
person whose age is four years more than the half of L’s age. L and M face
opposite direction to each other (i.e. If one faces north then the other faces
south vice versa). The one whose age is six years faces south. The person
whose age is 5/4 of L’s age faces north.

7 Adda247 Publications For any detail, mail us at


Publications@adda247.com
Cracker Book for Bank (IBPS | SBI | RRB PO | Clerk) Mains Exams

1. Who among the following is sitting in the row?


(a) G (b) P (c) Q
(d) J (e) H

2. Who among the following person faces north direction?


(a) K (b) J (c) M
(d) Q (e) O

3. Which of the following is the age of M?


(a) 16 (b) 40 (c) 9
(d) 25 (e) None of these

4. If 25 is related to K and 16 is related to M then 32 is related to whom?


(a) L (b) J (c) Q
(d) G (e) None of these

5. What will be the age of the one who sit third to the right of L?
(a) 20 (b) 6 (c) 40
(d) 25 (e) 9

Directions (6-10): Study the following information carefully and answer


the given questions.
P, Q, R, S, T, U, V and W are eight cricketers sitting around a circular table.
Some of them are facing the center while others are facing opposite to the
center. Each of them plays for different team viz. M, K, J, X, N, L, Y and Z but
not necessarily in the same order. Each of them except U and R played
different number of matches for their team. S played two more matches
than his immediate right neighbour. S sits third to the left of P. R sits second
to the left of P. The difference between the number of matches played by Q
and the one who plays for J is two less than the number of person sitting
between them. Q who plays for N is an immediate neighbour of both S and
V. The one who sits to the immediate right of Q plays for X. V who plays for
M is sitting third to the right of P. The sum of the total number of matches
played by the immediate neighbours of P is equal to 9. W who plays for Y
sits third to the right of T. Neither P nor W is an immediate neighbour of the

8 Adda247 Publications For any detail, mail us at


Publications@adda247.com
Cracker Book for Bank (IBPS | SBI | RRB PO | Clerk) Mains Exams

person who plays for L. U does not play for J. Only two such persons are
there who do not have anyone sitting in front of them. The number of
matches played by T is twice the sum of the number of matches played by
his immediate neighbors. The person who plays for Z is sitting opposite T.
The one who plays for N played 5 matches. P and Q are not facing each
other. The one who played for K played 3 matches. No one played more
than 8 matches.

6. Who among the following sits fourth to the right of P?


(a) Q (b) R (c) S
(d) T (e) None of these

7. What is the sum of the total number of matches played by the


immediate neighbours of W?
(a) 11 (b) 8 (c) 7
(d) 9 (e) None of these

8. Who played the least number of matches?


(a) P (b) Q (c) R
(d) T (e) None of these

9. What is the difference between the number of matches played by the


immediate neighbours of U?
(a) 6 (b) 3 (c) 2
(d) 4 (e) None of these
10. Who among the following sits fifth to the right of Q?
(a) P (b) U (c) T
(d) V (e) None of these

Directions (11-15): Study the following information carefully and answer


the questions given below.
Some students are sitting in two rows waiting for their turn to deliver their
speech in the annual function such that all are facing towards north
direction. Some are sitting in row 1 which has more number of seats than
row 2. The student sitting on the left end of the row 1 is the first one to
deliver the speech.
9 Adda247 Publications For any detail, mail us at
Publications@adda247.com
Cracker Book for Bank (IBPS | SBI | RRB PO | Clerk) Mains Exams

Note: The student on the left end of row 1 is at the 1st position and when he
goes to deliver his speech then the student sitting (next) at position 2 will
shift to the first position of the row 1 and all the students will shift in the
same manner. Only three students got the chance to deliver the speech on
the day of the annual function.

All the students will deliver speech according to their seating arrangement
as 1st student seating at left end of row-1 will deliver the speech first then
his immediate neighbour deliver and so on. Total 8 students are sitting in
both rows.

The student sitting at the extreme left end of row 2 is the first student to
shift to row 1 from the extreme right end of row-1. Initially the seating
arrangement of students is based on the given information. Student sitting
on the immediate left of B is the student who is going to deliver the speech
second. B is the last student to deliver the speech on the first day. A and B
does not sit in the same row. No two students sits adjacent to each other
according to the English alphabet(i.e. A does not sit adjacent to B and B does
not sits adjacent to C and A and so on). C does not get the chance to deliver
the speech on the first day of the annual function. C is not the first person to
shift from row 2 to row 1. E is not the first student to deliver the speech nor
he sits in the same row with A. Both F and C sits in the same row and at
least one students sits between them. F is not the first student to deliver the
speech. G is the last student who shift from row 2 to row 1 and is sitting at
extreme right end of row. Only one person sits on the immediate right of A.
D and H does not sit in the same row. H does not deliver the speech on first
day.

11. Who among the following pair sits in the row 2 initially?
(a) C and A (b) B and G (c) G and D
(d) H and G (e) None of these

12. Who among the following is the first to deliver the speech on the
annual day?
(a) D (b) H (c) E
(d) F (e) None of these
10 Adda247 Publications For any detail, mail us at
Publications@adda247.com
Cracker Book for Bank (IBPS | SBI | RRB PO | Clerk) Mains Exams

13. Who among the following student replaces B after the second student
goes to deliver the speech?
(a) H (b) E (c) A
(d) G (e) None of these

14. How many students are there in the first row?


(a) 5 (b) 6 (c) 7
(d) 4 (e) None of these

15. Who among the following sits second to the right of F?


(a) A (b) D (c) E
(d) H (e) None of these

Directions (16-20): Study the following information carefully to answer


the questions given below:

Ten people K, L, M, N, O, P, Q, R, S and T are sitting in a cabin as per the


given formation in the figure such that four of them are facing in west
direction, four are facing east while the other two are facing north direction.

R is sitting second to the right of T. Neither R nor T sits opposite to P. K is an


immediate neighbour of P. N sits opposite to the one who sits immediate
left of L. Only one person sits between P and S. Either three persons sit to
the left of N or no one sits to the right of N. S is not an immediate neighbour
of N and L. P does not face north direction. Q sits to the immediate right of
M. O sits opposite to the one who sits third to the left of N. Neither N nor T
is an immediate neighbour of Q.

11 Adda247 Publications For any detail, mail us at


Publications@adda247.com
Cracker Book for Bank (IBPS | SBI | RRB PO | Clerk) Mains Exams

16. Who among the following sit opposite to O?


(a) L (b) R (c) N
(d) S (e) None of these

17. What is the position of P with respect to N?


(a) Immediate left (b) Second to the left (c) Second to the right
(d) Immediate right (e) None of these

18. How many persons sit between Q and K, counting from the left of K?
(a) One (b) Three (c) Two
(d) No one (e) None of these

19. Which of the following pair faces north direction?


(a) L, O (b) M, Q (c) S, K
(d) R, K (e) None of these

20. How many persons sits between O and T?


(a) Two (b) One (c) Three
(d) No One (e) None of these

Directions (21-25): Study the following information carefully to answer


the questions given below:
Seven boxes B, Y, R, I, O, G, V are placed one above another in a loaded truck.
All these boxes are placed in a linear row from west to east when truck
unloaded. Also after unloading each of the box assigned a unique number.
Initially when boxes are arranged vertically in the truck, the arrangement
is-
More than three boxes are placed between V and R. Only one box is placed
between G and O. Box O is placed above G. Three boxes are placed between
Y and I. Box O is not placed above B and Y. Box V is above box I.
After the unloading of the boxes, Now Only two boxes are placed between
the boxes which was third from the bottom vertically and the box which
was at top. The box which is third from the west end is assigned a number
300
which is % of the number of the box which is two places towards east of
6
it. Only one box is placed between the boxes which was at top and the box
12 Adda247 Publications For any detail, mail us at
Publications@adda247.com
Cracker Book for Bank (IBPS | SBI | RRB PO | Clerk) Mains Exams

which was at bottom. Box I is not placed next to box R. The number
assigned to the box which was at top is 44. The box which was exactly in the
middle is placed adjacent to box R. More than three boxes are placed
between the boxes which was second from the top and box O. The number
assigned to box which is at east end is average of the number assigned to
boxes B, Y and O. Box V is not in the east of box O. Box G is placed
immediate next to box B. Box G is not placed immediate next to the box
which was third from the bottom. The number assigned to box O is 15/11 of
the number assigned to box B. The sum of the number assigned to the boxes
placed at both the ends is 67. The number assigned to the box placed
immediate next to V is 4/7 of the number assigned to it. The number
assigned to box G is twice of the difference of the numbers assigned to box Y
and I.

21. How many boxes are placed between box Y and O when boxes are
placed in vertical manner?
(a) One (b) Three (c) Two
(d) More than three (e) None of these

22. Which of the following box is placed second to the right of box B
horizontally?
(a) V (b) O (c) Y
(d) R (e) None of these

23. What will be the difference of the number assigned to box G and I?
(a) 25 (b) 26 (c) 28
(d) 24 (e) None of these

24. What is the new position of the box which was placed at the bottom?
(a) Second from right end
(b) Immediate left of box G
(c) Third from the left end
(d) Immediate right of box O
(e) None of these

13 Adda247 Publications For any detail, mail us at


Publications@adda247.com
Cracker Book for Bank (IBPS | SBI | RRB PO | Clerk) Mains Exams

25. What is the number assigned to the box which is placed exactly in the
middle horizontally?
(a) 22 (b) 44 (c) 48
(d) 24 (e) None of these

Directions (26-30): Study the information carefully and answers the


question given below.
There are nine-person U V T Z S X W Y and E Sitting in a row where some of
them are facing North and remaining are facing South direction.
Note: Opposite direction means (If one face South then another person
faces north and vice versa). Also, their ages are between 35-65 (means all
nine members age between 35-65, and none of them have same age). W sits
middle of the row, and one person sit between Y and W. E sits second to left
of Y, and face opposite direction with respect of both W and Y. S and T both
are immediate neighbours and one of them sits extreme end of row and
both face same direction with respect of E. Number of persons sit between
T and W and Z and W are same. One person sit between Z and the one
whose age is a perfect square of an even number which is more than 6. U
and V are immediate neighbours and difference between the ages of them is
one year. Either U’s or V’s age is a perfect square of a number which is more
than 5. The ages of both the immediate neighbours of W are perfect
squares. U, who is 48-year-old sits third to left of X. The person who is 55-
year-old sit sixth to right of T. E’s age is 10 years less than his neighbour. Z
faces opposite direction with respect to both U and Y. W age is 3 years more
than V. Difference between ages of Y and T is 5 years. T is older than Y, who
is 2 years younger than Z. Not more than three person faces south direction
and S is youngest amongst Nine. The one whose age is 37 sits at extr8eme
end. E’s age is not 37. There is only one person sits between S and U. There
are only 2 persons sit between the one whose age is 37 and the one who sits
immediate right of W.

26. Which of the following persons sits third to the left of the one whose
age is 64 years?
(a) S (b) T (c) U
(d) V (e) None of these.

14 Adda247 Publications For any detail, mail us at


Publications@adda247.com
Cracker Book for Bank (IBPS | SBI | RRB PO | Clerk) Mains Exams

27. How many persons sit to the right of Y?


(a) One (b) Two (c) Three
(d) None (e) More than Three

28. What is the age of the person who is immediate left of V?


(a) 48 (b) 49 (c) 53
(d) 64 (e) None of these

29. Which group of persons represent whose ages are perfect square?
(a) S T (b) X V (c) X W
(d) V U (e) None of these

30. How many persons sit between W and the person whose age is 45
years?
(a) One (b) Two (c) Three
(d) None (e) More than three

Directions (31-35): Study the following information carefully and answer


the questions that follow:

In the given figure the four-line segments 1,2,3,4 are PA, QB, RC, SD
respectively. The lengths of the line are 50m,35m,24m and 62m
respectively.
Six people Dev, Ram, Ravi, Raju, Sonu and Monu are standing in line 1-PA.
All of them are facing north. With distance between them increasing in
multiples of 9 from the left end (i.e. suppose Dev is left end of the line at
point P then the remaining people will stand at a distance as follows -9m,
18m, 27m…. from the end)

15 Adda247 Publications For any detail, mail us at


Publications@adda247.com
Cracker Book for Bank (IBPS | SBI | RRB PO | Clerk) Mains Exams

Two persons are standing between Dev and Ram. Dev stands to the left of
Ram. Ravi stand second to the left of Ram. Monu is an immediate neighbor
of Ravi. Only one person stands between Monu and Raju. Dev is not an
immediate neighbour of Monu. Raju is neither an immediate neighbour of
Dev nor stand at an extreme end.
Similarly, six people Diya, Riya, Piya, Jiya, Tia and Kiya are standing in line-4
SD. All of them are facing south. with distance between them increasing in
multiples of 11 from the end (i.e. if first person is at right end of the line at
point S and the remaining will be stand at the distance as follows-11m,
22m, 33 m…. from point S).
Piya is standing at 3rd position from the right end. Tia is an immediate
neighbor Of Piya. Two people are standing between Tia and Kiya. More than
three persons stand between Kiya and Diya. Jiya stands to the immediate
left of Kiya. Riya and Tia are immediate neighbours.
* Point Q and Point R is left end of the row in line QB and line RC.

31. In the line-4 SD, what is the distance between Tia and point D?
(a) 29m (b) 16m (c) 22m
(d) 24m (e) None of these

32. If in line-1 PA, if peoples stand at distance of multiples of 11 in the


same order, and rest of them move to line-2 QB and stands from left
end in the same order. And if in line 4-SD, if people stand at distance
of multiples of 13 in the same order and rest of them move to line-3
RC, stands from left end in the same order. Then total together how
many people are standing in lines 2 and 3?
(a) 1 (b) 2 (c) 3
(d) 4 (e) None of these

33. If in line 4-SD, if people stand in multiples of 14 in the same order and
rest of them move to line-3 RC and stands from left end in the same
order. Then the distance between Diya and point C is?
(a) 2 (b)12 (c)24
(d)14 (e) None of these

16 Adda247 Publications For any detail, mail us at


Publications@adda247.com
Cracker Book for Bank (IBPS | SBI | RRB PO | Clerk) Mains Exams

34. Suppose line-1 PA is joined to line2-QB to form a new straight-line PB


in such a way that distance between point A and Q is 5m. If the people
of line-1 PA are made to stand in multiples of 12 in that new line, then
how far is Ram from point B?
(a) 16 (b) 21 (c) 26
(d) 54 (e) None of these

35. If in line-1 PA people stand in distance at multiples of 12 in the same


order and rest of them move to line-2 QB in the same order from left
end, then who will be standing in line-2?
(a) Raju (b) Monu (c) Ram
(d) Ram and Monu (e) None of these

Direction (36-40): Study the following information carefully and answer


the question below-
* Numbers are given in the figure considered as their positions.

Eight persons Ravi, Arjun, Rahul, Ram, Raj, Ayush, Sahil, Shyam are sitting
around the circular table having eight chairs such that some of them are
facing towards the center while some are facing outside the center.
Shyam sits second to the left of Ravi. Only two persons sit between Shyam
and Raj. Raj is not an immediate neighbour of Ravi. Both Raj and Ram are
facing outside the center. Ayush sits on the immediate right to Raj. Sahil sits
third to the right of Arjun. Only Rahul sits between Shyam and Sahil. Both
Arjun and Shyam are facing towards the center. Both Ayush and Rahul are
facing the same direction as Shyam. Sahil is facing opposite direction of
Ravi.
17 Adda247 Publications For any detail, mail us at
Publications@adda247.com
Cracker Book for Bank (IBPS | SBI | RRB PO | Clerk) Mains Exams

Now they start playing cards game. They shuffled a pack of cards. 1. Ravi
draws one card and changes his place according to the given conditions, 2.
Ram draws one card and changes his place according to the given
conditions similarly others draw cards given in the condition below-
Conditions:
1. If the card drawn is spade, the person who draws it first moves to the
outer square at position 5 facing opposite direction of his current
direction, then the second person who also draw spade moves to
position 6, similarly 7 and 8.
2. If card drawn is heart, person who draws it moves to the immediate
left of his current position facing same direction.
3. If the card drawn is diamond, the person who draws it first moves to
the inner square at corner 1 facing same direction, then the second
person, who also draw diamond moves to corner 2, similarly 3 and 4.
4. If card drawn is club, person remains on the same position facing
opposite direction of current direction.
Draws:
1. Ravi draws queen of Club
2. Ram draws Jack of Club
3. Shyam draws 3 of Spade
4. Rahul draws a King of Diamond
5. Sahil draws Ace of Heart
6. Raj draws 9 of Heart
7. Ayush draws 7 of Diamond
8. Arjun draws 4 of Spade
* Note: All the persons draws card in a serial way as given above.
(ex- First Ravi draws then Ram draws ….. so on.)

36. After all persons changed their places based on the above conditions,
who among the following is sitting at the corner 2 of the square?
(a) Arjun (b) Rahul (c) Sahil
(d) Ayush (e) No one

37. After all persons changed their places based on the above conditions,
how many persons will remain in the circle?
(a) Two (b) Four (c) One
(d) Three (e) None of these
18 Adda247 Publications For any detail, mail us at
Publications@adda247.com
Cracker Book for Bank (IBPS | SBI | RRB PO | Clerk) Mains Exams

38. After all persons changed their places based on the above conditions,
then who among the following is sitting to the immediate right of
Sahil?
(a) Raj (b) Ravi (c) Rahul
(d) Shyam (e) None of these

39. After all persons changed their places based on the above conditions,
then how many persons facing away from the center?
(a) Two (b) One (c) Five
(d) Three (e) None of these

40. After all persons changed their places based on the above conditions,
then who among the following is sitting at the position 6 of the
square?
(a) Sahil (b) Ravi (c) Arjun
(d) Shyam (e) None of these

Directions (41-45): Study the following information carefully and answer


the question given below:
Eight students namely A, B, C, D, E, F, G and H study in the school. The Class
teacher asked them to sit in a circle in one arrangement and after lunch in a
straight line in another arrangement. While sitting around a circle each
student are facing outward and while sitting in a straight line each student
are facing north. But in both arrangements each student likes same
cartoons. The cartoons are i.e. Chota bheem, Ben 10, Tom & Jerry, Oggy,
Doraemon, Samurai jack, Johnny Bravo and Generator Rex, but not
necessarily in the same order.
F likes Johnny Bravo. F and C are the immediate neighbours of B in both the
arrangements, but C is not at the extreme ends of the line. The one who
likes Samurai jack sits immediate right of G in the circle. One of the
immediate neighbours of H in the straight line sits opposite to H in the
circle. The one who likes Chota bheem sits third to the right of the one who
likes Johnny Bravo in the Straight line. The one who likes Tom & Jerry sits
second to the left of the one who likes Samurai jack in the straight line. The
19 Adda247 Publications For any detail, mail us at
Publications@adda247.com
Cracker Book for Bank (IBPS | SBI | RRB PO | Clerk) Mains Exams

one who sits on the extreme left end sits second to the right of E in the
circle. H is not on the immediate left of F in both the arrangements. The
Persons who like Chota bheem and Ben 10 sits opposite to each other in the
circle. D sits third to the right of F in the straight line. The one who sits on
the immediate left of B in the straight line is sitting on the immediate right
of B in the circle. G sits on the immediate left of H in the circle, but both are
not immediate neighbours of each other in the straight line. Person likes
Doraemon sits second to the left of the one who likes Oggy in the straight
line. E sits third to the right of B in the circle, while fourth to his left in the
straight line.

41. Which of the following pairs sit at the extreme ends of the row?
(a) The Person who likes Samurai jack, E
(b) F, G
(c) Can’t be determined
(d) G, The Person who likes Chota bheem
(e) None of these

42. The one who is sitting at the extreme left end of the straight line likes
which of the following cartoon?
(a) Tom & Jerry (b) Chota bheem (c) Ben 10
(d) Generator Rex (e) None of these

43. Who among the following sits on the immediate right of E in the
circle?
(a) A (b) G (c) C
(d) D (e) None of these

44. The one who sits third to the right of C in the circle is at what position
in the straight line with respect to G?
(a) Third to the left (b) Third to the right (c) Immediate right
(d) Fourth to the right (e) None of these

20 Adda247 Publications For any detail, mail us at


Publications@adda247.com
Cracker Book for Bank (IBPS | SBI | RRB PO | Clerk) Mains Exams

45. The person who is sitting between E and G in the circle is sitting at
what position in the straight line?
(a) Extreme left end of the straight line
(b) Fourth from the right end
(c) Third from the left end
(d) Extreme right end of the straight line
(e) None of these

Directions (46-50): Study the following information carefully and answer


the questions given below:
Nine people are sitting in a straight line, some are facing south and some
are facing north. Each of them has got married during the years 1972-1980
in the months of Jan to Sept but not necessarily in the same order and only
one person got married in one year and in one month.
C is not the youngest. C got married in a leap year and in a month of 31
days. I is older than B and faces north direction. E got married in Sept. D is
older than H but H is younger than F. C sits 3rd left of H while I sits 2nd
right of D. The persons sitting adjacent to the ones who got married in a
month of 31 days except May but not sitting at extreme ends are facing
opposite directions. Oldest of them sit at an extreme end and got married in
the month which has 31 days. The person who got married in 1977 sits
immediate left of the person, who got married on September. The persons
adjacent to the youngest person face opposite direction. While youngest of
them got married in July sit adjacent to those, who got married in 31 days
month but none of them sit at any extreme position. A sit on the extreme
end. And both the youngest and the oldest face the same direction. A sit 3rd
left of G and face in same direction as G who is not sitting adjacent to I. The
one who got married in Jan is facing south and is sitting exactly in the
middle and his immediate neighbours face opposite directions. I got
married one of the year before B. G got married in 1980. The one who got
married in Aug is facing south and got married in 1974. The ones who got
married in a month having 30 days are sitting adjacent to each other but
none of them is E. E got married in 1978. H who is facing north, got married
in a year between the years of D and A got married and none of them got
married in a leap year. The marriage difference between B and I is 7 years

21 Adda247 Publications For any detail, mail us at


Publications@adda247.com
Cracker Book for Bank (IBPS | SBI | RRB PO | Clerk) Mains Exams

and they got married in May and March respectively. H and D sit adjacent to
each other facing same direction. B and E faces same direction. Persons
sitting adjacent to A face north direction.

46. Who among the following person sits fourth to the right of C?
(a) H (b) A (c) E
(d) I (e) None of these

47. In which of the following year B got married?


(a) 1978 (b) 1980 (c) 1975
(d) 1979 (e) None of these

48. How many persons sit between the person who got married in 1979
and the person who got married in the month of June?
(a) Four (b) Two (c) Three
(d) Can’t be determined (e) Five

49. In which of the following month D got married?


(a) April (b) August (c) June
(d) September (e) Either (a) or (c)

50. Who among the following person got married in the month of August?
(a) Person, who sits immediate left of C
(b) G
(c) Person, who sits immediate left of D
(d) Both (a) and (c)
(e) A

Directions (51-55): Read the following information carefully and answer


the questions given below:
There are six cars – K, L, M, N, O, P – parked in a row facing north direction,
but not necessarily in the same order. The distances between two adjacent
cars are multiples of three.
Information regarding all the cars:
22 Adda247 Publications For any detail, mail us at
Publications@adda247.com
Cracker Book for Bank (IBPS | SBI | RRB PO | Clerk) Mains Exams

The total distance between all the parked cars is 198m.


The distance between ‘N’ and ‘K’ is 105 m and car ‘N’ is second to the left of
car ‘K’.
The distance between the cars ‘N’ and ‘M’ is 132 m.
The car ‘P’ is 84m away from car ‘O’. Car ‘P’ is at one of the position to the
left of car ‘O’.
Only one Car is parked between L and P. P is to the immediate left of K and
distance between them is 36m.
Car L is not in the right of car N and more than two cars are parked between
car N and car O.
Car ‘L’ starts moving towards south and after going 10 m, it turns left, then
it moves 114 m and then it goes another 5 m to its left and stops at point ‘X’.
An another Car ‘C’ moves 15 m in a certain direction, and then takes a left
turn and goes 27 m straight. Then it turns to its right and moves another 5
m and halts at point ‘Y’.
Point Y is in the east of point X and is 36m apart from each other.
Now car ‘N’ starts moving in north direction. After moving 15m it turns to
its right move 105m and stopped at point Z.

51. How many cars are parked there in between cars ‘M’ and ‘L’?
(a) Two (b) None (c) More than three
(d) One (e) Three

52. What is the distance between point ‘Z’ and point ‘Y’?
(a) 25 m
(b) 18 m
(c) They don’t align in the same straight line
(d) 20 m
(e) 27 m

53. What is the distance and direction of final position of Car ‘C’ with
respect to the Car ‘P’?
(a) √1334 m towards north-west
(b) 50 m towards south-east
(c) √1321 m towards south-east
(d) 5√34 m towards south-east
(e) None of these
23 Adda247 Publications For any detail, mail us at
Publications@adda247.com
Cracker Book for Bank (IBPS | SBI | RRB PO | Clerk) Mains Exams

54. Which car will be met first, if ‘L’ moves through the shortest distance
from point ‘X’?
(a) N (b) O (c) K
(d) P (e) None of these

55. What is the current position of car ‘M’ with respect to car ‘O’?
(a) 75 m towards right
(b) 21m to the immediate left
(c) 36 m towards left
(d) 69 m towards left
(e) None of the above

Directions (56-60): Study the following information to answer the given


questions:
Ten persons are sitting in 2 parallel rows containing 5 persons in each row.
In 1st row G, F, H, I and J are seated and face south and in 2nd row E, D, A, B
and C are seated and face north, but not necessarily in the same order.
Therefore, in the given seating arrangement, each member seated in a row
faces another member of the other row. They own different dogs i.e. Great
Dane, Dobermann, Pug, Golden Retriever, German sphered, doodle, Pitbull,
beagle, Labrador, bull dog but not necessarily in the same order.
E sits 2nd left to D, none of them is sitting at the corner. G faces one of the
immediate neighbor of D.H and J sit together and only one of them is
neighbor of G. The one who owns Pitbull sits at one of the corner. Neither D
nor his neighbors face the one who owns Dobermann. B owns pug. Two
people sit between B and the one who owns golden retriever. E does not
own golden retriever. J faces E. The one who owns German sphered sits
opposite to F, who owns Labrador. F does not sit next to the one who owns
Great Dane. The one who owns great Dane does not face north. A does not
face G. Neither I nor his neighbors face the one who owns doodle. The one
who owns Dobermann faces south. I does not own Doodle.

56. Who among the following faces the one who owns Pitbull?
(a) The one who owns Pug
(b) I
(c) D
(d) the one who owns bulldog
(e) none of these
24 Adda247 Publications For any detail, mail us at
Publications@adda247.com
Cracker Book for Bank (IBPS | SBI | RRB PO | Clerk) Mains Exams

57. Who are immediate neighbors of the one who owns Doodle?
(a) E, J (b) B, C (c) C, D
(d) C, A (e) None of these

58. Four of the following five from a group, which among the following
does not belong to this group?
(a) H (b) B (c) C
(d) A (e) F

59. G owns which of the following dogs?


(a) Pug (b) Labrador (c) golden retriever
(d) bulldog (e) none of these

60. How many persons sit to right of the person who owns German
sphered?
(a) one (b) two (c) three
(d) no one (e) four

Directions (61-65): Study the following information to answer the given


questions:
There are eight persons namely P, Q, R, S, T, U, V and W sitting around a
square table in such a way that two persons sit on each side. All the persons
are facing towards the center. They like different cafes in Himachal Pradesh
i.e. Johnson, Evergreen, Cafe 1947, Freedom cafe, Moon dance, Jim Marison,
Moonpeak espresso, Shiva but not necessarily in the same order.
S and P sit on the same side of the table. Neither P nor his neighbors like
Shiva and Moon dance. The one who likes Jim Marison sits 2nd right to S.
Three people sit between the one who likes Jim Marison and W. Neither W
nor his neighbors like Johnson. R does not sit next to the one who like
Johnson. R who likes evergreen faces S, who likes Cafe 1947.T sits opposite
to Q.V sits 3rd right to U. More than three persons sit between the one who
likes Johnson and the one who like Moon dance, when counted in clockwise
direction from of the one who likes Johnson. The one who likes Jim Marison
does not face the ones who like Moonpeak espresso and Johnson. No one
sits between T and the one who likes freedom café.

25 Adda247 Publications For any detail, mail us at


Publications@adda247.com
Cracker Book for Bank (IBPS | SBI | RRB PO | Clerk) Mains Exams

61. Which among the following represents the correct combination?


(a) V-Cafe 1947
(b)T-Johnson
(c) U-Moon dance
(d) P-Moonpeak espresso
(e) none of these

62. Four of the following five form a group, which among the following
does not belongs to that group?
(a) S-R (b) V-P (c) W-U
(d) T-Q (e) S-V

63. What is the position of W with respect to R?


(a) 5th from the left (b) 2nd to the right (c) 3rd to the left
(d) 4th to the right (e) none of these

64. If S and the one who likes Jim Marison exchange their positions, then
who among the following sits 4th right to S?
(a) R (b) P (c) U
(d) T (e) None of these

65. Which the following is not true regarding T?


(a) he likes Shiva
(b) he faces the one who likes Jim Marison
(c) only V sits between T and R.
(d) S sits 3rd left to T
(e) none of these

Directions (66-70): Study the following information to answer the given


questions :
There are eight head of NGO’s namely A, B, C, D, E, F, G and H sitting in a
linear row facing north. All of them distribute free meals to poor on
different festivals i.e. Republic day, Independence Day, Christmas, Diwali,
Lohri, Budh Purnima, Eid-ul-fitr, Mahavir Jayanti but not necessarily in the
same order. No two persons sit next to each other according to the English

26 Adda247 Publications For any detail, mail us at


Publications@adda247.com
Cracker Book for Bank (IBPS | SBI | RRB PO | Clerk) Mains Exams

alphabetical order (i.e. A does not sit adjacent to B and B does not sit next to
A & C and so on).
A sits at one of the ends of the row. More than four persons sit between A
and D. C who is one of the immediate neighbors of H, sits 3rd left to the one
who distributes meal on Mahavir Jayanti. Only one person sits between the
ones who distribute meal on Christmas and Eid-ul-fitr and none of them sit
at the corner. More Than one person sits between the ones distributing
meal on Independence Day and Eid-ul-fitr. The ones who distribute meal on
Diwali and Lohri sit together. Neither A nor C distributes meal on Lohri and
Diwali. H sits 3rd left to B, who does not distribute meal on republic day. A is
not neighbor of H. Odd number of persons sit between H and D. The one
who distributes meal on Budh Purnima sits at extreme left end. There is one
person less sits between A and G as compared to the number of persons
sitting between G and F. The one who distributes meal on Mahavir Jayanti
sits to the immediate left of B. The one who distributes meal on republic
day sits 3rd right to the one who distributes meal on Lohri, who sits
immediate left to the one who distributes meal on Diwali.

66. Who among the following sit to immediate right to the one who
distributes free meal on republic day?
(a) G (b) C (c) D
(d) F (e) none of these

67. Who among the following sit immediate right of D?


(a) C (b) F (c) E
(d) G (e) B

68. How many persons are sitting between the one who distributes on
Eid-al-fitr and F?
(a) no one (b) one (c) two
(d) three (e) more than three

69. Which among the following represents incorrect combination?


(a) C-Eid-ul-fitr
(b) F-christmas
(c) B-mahavir jayanti
(d) H-republic day
(e) none of these
27 Adda247 Publications For any detail, mail us at
Publications@adda247.com
Cracker Book for Bank (IBPS | SBI | RRB PO | Clerk) Mains Exams

70. Which among the following pairs of person represents 2nd right to H
and 4th left to H respectively?
(a) D, A (b) A, D (c) G, C
(d) B, E (e) none of these

Solutions

Directions (1-5):
The person whose age is square of four sits second to the left of K. Two
persons sit between the one whose age is square of four and the one whose
age is six years. The one whose age is six years faces south.

Both the immediate neighbours of the one, whose age is six years faces
opposite direction to each other.

28 Adda247 Publications For any detail, mail us at


Publications@adda247.com
Cracker Book for Bank (IBPS | SBI | RRB PO | Clerk) Mains Exams

As there is only one immediate neighbour of the one whose age is 6 years,
so case-4 will be eliminated. Now, L’s age is twice of the age of the one who
sit second left of K. from this, case 1a, 2a, 3a will be eliminated as it is clear
that L’s age is 32 years.

The one whose age is four years more than the half of L’s age sits second to
the right to L. From this case-1b will be eliminated. Further, Only one
person sits between M and the one, whose age is 5/4 of L’s age.
29 Adda247 Publications For any detail, mail us at
Publications@adda247.com
Cracker Book for Bank (IBPS | SBI | RRB PO | Clerk) Mains Exams

The persons whose age is six years and the one whose age is 0.5 times of 18
are immediate neighbours. The one, whose age is square of five sits third to
the right of the person whose age is four years more than the half of L’s age.
From this case-2b will be eliminated. Now, with case-3b, 5a, 5b------

L and M face opposite direction to each other. The person whose age is 5/4
of L’s age faces north. From this case-5a will be eliminated. Both the
immediate neighbours of the one, whose age is six years faces opposite
direction to each other. So, from this case-5b also get eliminated as only
four persons are facing south. So, the final solution is------------

30 Adda247 Publications For any detail, mail us at


Publications@adda247.com
Cracker Book for Bank (IBPS | SBI | RRB PO | Clerk) Mains Exams

1. (d); 2. (e); 3. (a);

4. (a); 5. (c);

Directions (6-10):
Step 1. From the information given in the question,
S sits third to the left of P. R sits second to the left of P. There can be two
possible cases i.e. case 1 when P is facing towards the center and case 2
when P is facing outside the center. Q who plays for N is an immediate
neighbour of both S and V. It means Q and P are sitting opposite to each
other. P and Q are not facing each other. It means there will three possible
cases.
So we have,

Step 2. The one who sits to the immediate right of Q plays for X. V who
plays for M is sitting third to the right of P. So our case 1 and case 2 will be
eliminated and we will proceed with case 3. in which Q and P are facing
opposite to the center. Only two such persons are there who do not have
anyone sitting in front of them. It means all the other persons except P and
31 Adda247 Publications For any detail, mail us at
Publications@adda247.com
Cracker Book for Bank (IBPS | SBI | RRB PO | Clerk) Mains Exams

Q will be facing towards the center. W who plays for Y sits third to the right
of T. It means T sits to the immediate right of V. The person who plays for Z
is sitting opposite T. It means R plays for Z and U is sitting between T and P.
The one who plays for N played 5 matches. S played two more matches than
his immediate right neighbour.
So we have,

Step 3. Neither P nor W is an immediate neighbour of the person who plays


for L. So, clearly T plays for L. U does not play for J. Means P plays for J and U
plays for K. The difference between the number of matches played by Q and
the one who plays for J is two less than the number of person sitting
between them. The one who played for K played 3 matches. The sum of the
total number of matches played by the immediate neighbours of P is equal
to 9. It means W played 6 matches. U and R are the only two persons who
played the same number of matches. It means R played 3 matches and P
played four matches. The number of matches played by T is twice the sum
of the number of matches played by his immediate neighbors. No one
played more than 8 matches. It means V played 1 match while T played
eight matches.
So we have our final solution as,

32 Adda247 Publications For any detail, mail us at


Publications@adda247.com
Cracker Book for Bank (IBPS | SBI | RRB PO | Clerk) Mains Exams

6. (a); 7. (c); 8. (e);

9. (d); 10. (b);

Solutions (11-15):
Student sitting on the immediate left of B is the student who goes to deliver
the speech second. B is the last student to deliver the speech on the first
day. A and B does not sit in the same row. G is the last students who shift
from row 2 to row 1 and sits at an extreme right end of row-2. Only one
person sits on the immediate right of A. No two students sits adjacent to
each other according to the English alphabet (i.e. A does not sit adjacent to
B and B does not sits adjacent to C and A and so on). So, A sits in row 2.

C does not get the chance to deliver the speech on the first day of the annual
function. C is not the first person to shift from row 2 to row 1. E is not the
first student to deliver the speech nor he sits in the same row with A. Both F
and C sits in the same row and at least one student sit between them. F is
not the first student to deliver the speech. Total 8 students sit in both the
rows.

D and H does not sit in the same row. H does not deliver the speech on first
day.

33 Adda247 Publications For any detail, mail us at


Publications@adda247.com
Cracker Book for Bank (IBPS | SBI | RRB PO | Clerk) Mains Exams

Now, the students shift according to their chance to deliver the speech as it
is given that only three students get the chance to deliver speech so clearly
1st D goes to deliver speech then 2nd F goes and 3rd B goes and accordingly
the shifting takes place as 1st H shifts to row-1 at the right end, 2nd A shifts
at right end as H moves to immediate left of A, 3rd G shifts at the right end as
accordingly. So, the arrangement will be-------

11. (d); 12. (a); 13. (b);

14. (a); 15. (c);

Solutions (16-20):
Either three persons sit to the left of N or no one sits to the right of N. N sits
opposite to the one who sits immediate left of L. O sits opposite to the one
who sits third to the left of N. From this there will be two possible cases-

Only one person sits between P and S. S is not an immediate neighbour of N


and L. K is an immediate neighbour of P. P does not face north direction.
From the given conditions there will be further three possibilities in case2
for the position of P.

34 Adda247 Publications For any detail, mail us at


Publications@adda247.com
Cracker Book for Bank (IBPS | SBI | RRB PO | Clerk) Mains Exams

Q sits to the immediate right of M. From this case-2(c) will be eliminated. R


is sitting second to the right of T. Neither R nor T sits opposite to P.

Neither N nor T is an immediate neighbour of Q. So, from this case-1 and


case-2(a) will be eliminated. So, the final arrangement is—

16. (d); 17. (a); 18. (a);

19. (b) 20. (a);

35 Adda247 Publications For any detail, mail us at


Publications@adda247.com
Cracker Book for Bank (IBPS | SBI | RRB PO | Clerk) Mains Exams

Solutions (21-25):
First we start with vertical arrangement of boxes- More than three boxes
are placed between V and R. Only one box is placed between G and O. Box O
is placed above G. Three boxes are placed between Y and I. Box V is above
box I.

Box O is not placed above B and Y. So, from this Case-2 and Case-3 will be
eliminated. So, the final vertical arrangement before unloading is----

Boxes
Y
V
B
O
I
G
R
Now, after unloading the linear arrangement of the boxes will be as follows-
-- Now Only two boxes are placed between the boxes which was third from
the bottom vertically and the box which was at top. That means two boxes
are placed between box Y and I. Only one box is placed between the boxes
which was at top and the box which was at bottom. That means one box is
placed between box Y and R. Box I is not placed next to box R. The number
assigned to the box which was at top is 44. So, there will be two possible
cases---

36 Adda247 Publications For any detail, mail us at


Publications@adda247.com
Cracker Book for Bank (IBPS | SBI | RRB PO | Clerk) Mains Exams

The box which was exactly in the middle is placed adjacent to box R. That
means Box O is placed adjacent to box R. More than three boxes are placed
between the boxes which was second from the top and box O i.e. more than
three boxes placed between V and O. Box V is not in the east of box O.
So, from this it is clear that box V is placed at west end, box R is at the east
end and rest box O is adjacent to box R. Further Box G is placed immediate
next to box B. Box G is not placed immediate next to the box which was
third from the bottom which means G is not placed next to I. So, the linear
arrangement will be----

The box which is third from the west end is assigned a number which is
300
% of the number of the box which is two places towards east of it. So,
6
clearly number of box B is 22. The number assigned to box O is 15/11 of the
number assigned to box B. Clearly number of box O is 30. The number
assigned to box which is at east end is average of the number assigned to
boxes B, Y and O. So, number of box R is 32. The sum of the number
assigned to the boxes placed at both the ends is 67. So, number of box V is
35. The number assigned to the box placed immediate next to V is 4/7 of
the number assigned to it. So, number of box I is 20. The number assigned
to box G is twice of the difference of the numbers assigned to box Y and I.
So, number of box G is 48. So, the final arrangement is-------

37 Adda247 Publications For any detail, mail us at


Publications@adda247.com
Cracker Book for Bank (IBPS | SBI | RRB PO | Clerk) Mains Exams

21. (c); 22. (c); 23. (c);


24. (d); 25. (c);

Solutions (26-30):

26. (c); 27. (e); 28. (a);

29. (b); 30. (c);

Solutions (31-35):
Step I:Let us start with the detail solution. As given in the diagram that
there is a line-1 PA and Six people Dev, Ram, Ravi, Raju, Sonu and Monu are
standing in line 1-PA. All of them are facing north. With distance between
them increasing in multiples of 9 from the left end (i.e. suppose Dev is left
end of the line at point P then the remaining people will stand at a distance
as follows -9m, 18m, 27m…. from the end). So, the arrangement will be----

In this arrangement the distance between two persons standing next to


each other is nine whereas from first person standing at left end the
distance will be in multiple of 9. And as the total length of the line is 50m
and all persons are standing in the line at a distance of multiple of 9 so it
will be 45m, remaining distance will be 50-45=5m from the right end.
Step II: Now, let us start with the sitting arrangement of line 1-PA. Two
persons are standing between Dev and Ram. Dev stands to the left of Ram.
Ravi stand second to the left of Ram. Monu is an immediate neighbor of
Ravi. Only one person stands between Monu and Raju. Dev is not an
immediate neighbour of Monu. Raju is neither an immediate neighbour of
Dev nor stand at an extreme end. So, the final arrangement of line1-PA is----
----

38 Adda247 Publications For any detail, mail us at


Publications@adda247.com
Cracker Book for Bank (IBPS | SBI | RRB PO | Clerk) Mains Exams

Step IV: Similarly, six people Diya, Riya, Piya, Jiya, Tia and Kiya are standing
in line-4 SD. All of them are facing south. with distance between them
increasing in multiples of 11 from the end (i.e. if first person is at right end
of the line at point S and the remaining will be stand at the distance as
follows-11m, 22m,33 m….from point S)

In this arrangement the distance between two persons standing next to


each other is eleven whereas from first person standing at right end the
distance will be in multiple of 11. And as the total length of the line is 62m
and all persons are standing in the line at a distance of multiple of 11 so it
will be 55m, remaining distance will be 62-55=7m from the left end.
Step V: Piya is standing is at 3rd position from the right end. Tia is an
immediate neighbor Of Piya. Two people are standing between Tia and
Kiya. More than three persons stand between Kiya and Diya. Jiya stands to
the immediate left of Kiya. Riya and Tia are immediate neighbours.

Now, final solution is-----------

31. (a);

32. (b); As only five people can stand in both lines PA and SD at a
distance in multiples of 11m and 13 respectively, So one person
from each line will move to line 2-QB and line 3-RC respectively.
39 Adda247 Publications For any detail, mail us at
Publications@adda247.com
Cracker Book for Bank (IBPS | SBI | RRB PO | Clerk) Mains Exams

33. (c); As only four people can stand in line 4- SD at a distance in


multiples of 14m, So one person from line 4-SD will move to line
3-RC respectively.

34. (d);

40 Adda247 Publications For any detail, mail us at


Publications@adda247.com
Cracker Book for Bank (IBPS | SBI | RRB PO | Clerk) Mains Exams

35. (e);

Solutions (36-40):
From the given condition first we make the sitting arrangement of the
persons—
Step I: Shyam sits second to the left of Ravi. Only two persons sit between
Shyam and Raj. Raj is not an immediate neighbour of Ravi. Both Raj and
Ram are facing outside the center. Ayush sits on the immediate right to Raj.
Sahil sits third to the right of Arjun. Only Rahul sits between Shyam and
Sahil. Both Arjun and Shyam are facing towards the center. Both Ayush and
Rahul are facing the same direction as Shyam. Sahil is facing opposite
direction of Ravi. So, the sitting arrangement is-----

Step II: Now, according to the given conditions - 1. Ravi draws queen of
Club, 2. Ram draws Jack of Club and If card drawn is club, person remains
on the same position facing opposite direction of current direction. So, Ravi
and Ram remains on the same position facing opposite direction of current
direction.

41 Adda247 Publications For any detail, mail us at


Publications@adda247.com
Cracker Book for Bank (IBPS | SBI | RRB PO | Clerk) Mains Exams

Step III: 3.Shyam draws 3 of Spade, If the card drawn is spade, the person
who draws first moves to the outer square at position 5 facing opposite
direction of current direction, then the second person who also draw spade
moves to position 6, similarly 7 and 8. So, it is clear that Shyam will move to
position-5 and will now face outside the center. Further 4. Rahul draws a
King of Diamond, If the card drawn is diamond, the person who draws first
moves to the inner square at corner 1 facing same direction, then the
second person who also draw diamond moves to corner 2, similarly 3 and 4.
So, it is clear that Rahul will move to corner-1 facing same direction.

Step IV:5. Sahil draws Ace of Heart, 6. Raj draws 9 of Heart and If card
drawn is heart, person who draws it moves to the immediate left of his
current position facing same direction. So, both Sahil and Raj moves to the
immediate left of their current position facing same direction.
42 Adda247 Publications For any detail, mail us at
Publications@adda247.com
Cracker Book for Bank (IBPS | SBI | RRB PO | Clerk) Mains Exams

Step V: Further, Ayush draws 7 of Diamond and If the card drawn is


diamond, the person who draws first moves to inner square at corner 1
facing same direction, then the second person who also draw diamond
moves to corner 2, similarly 3 and 4. So, Ayush move to corner 2 of the
inner square. Now, 8. Arjun draws 4 of Spade and If the card drawn is
spade, the person who draws it first moves to the outer square at position 5
facing opposite direction of current direction, then the second person who
also draw spade moves to position 6, similarly 7 and 8. So, Arjun moves to
the position-6 of the outer square.
So, the final arrangement is------------

36. (d); 37. (b); 38. (a);

39. (c); 40. (c);

43 Adda247 Publications For any detail, mail us at


Publications@adda247.com
Cracker Book for Bank (IBPS | SBI | RRB PO | Clerk) Mains Exams

Solutions (41-45):

41. (d); 42. (a); 43. (d);

44. (b); 45. (d);

Directions (46-50):

46. (b); 47. (d); 48. (d);

49. (e); 50. (d);

Solutions (51-55):
The distance between ‘N’ and ‘K’ is 105 m and car ‘N’ is second to the left of
car ‘K’. Only one Car is parked between L and P. P is to the immediate left of
K and distance between them is 36m. The car ‘P’ is 84m away from car ‘O’.
Car ‘P’ is at one of the position to the left of car ‘O’. Car L is not in the right of
car N and more than two cars are parked between car N and car O. The
distance between the cars ‘N’ and ‘M’ is 132 m. The total distance between
all the parked cars is 198m.

44 Adda247 Publications For any detail, mail us at


Publications@adda247.com
Cracker Book for Bank (IBPS | SBI | RRB PO | Clerk) Mains Exams

Car ‘L’ starts moving towards south and after going 10 m, it turns left, then
it moves 114 m and then it goes another 5 m to its left and stops at point ‘X’.
An another Car ‘C’ moves 15 m in a certain direction, and then takes a left
turn and goes 27 m straight. Then it turns to its right and moves another 5
m and halts at point ‘Y’.
Point Y is in the east of point X and is 36m apart from each other.
Now car ‘N’ starts moving in north direction. After moving 15m it turns to
its right move 105m and stopped at point Z.

51. (e); 52. (d); 53. (c);

54. (d); 55. (b);

Solutions (56-60):
Given, “E sits 2nd left to D, none of them is sitting at the corner. G faces one
of the immediate neighbor of D.H and J sit together and only one of them is
neighbor of G.”, J faces E. So, only place left for H and J is 2nd right and
immediate right to G respectively.
B owns pug. Two people sit between B and the one who owns golden
retriever. Either B can sit at extreme left or extreme right end but since E
does not own golden retriever, B sits at left end and D owns golden
retriever.

45 Adda247 Publications For any detail, mail us at


Publications@adda247.com
Cracker Book for Bank (IBPS | SBI | RRB PO | Clerk) Mains Exams

A does not face G. So, A will sit at the extreme right end of the row. Further
The one who owns German sphered sits opposite to F, who owns Labrador.”
Therefore, F faces A, who owns German sphered, we get the following
arrangement:

Given, “The one who owns Pitbull sits at one of the corner.” which means H
owns Pitbull. From, “Neither D nor its neighbor faces Dobermann.” Means
Neither I nor G owns Doberman. The one who owns Dobermann faces
south. So, J owns Dobermann. Given, “F, who owns Labrador and does not
sit next to the one who owns Great Dane. The one who owns great Dane
does not face north.”, we get, that G owns Great Dane.
Given, “Neither I nor its neighbor faces the one who owns doodle.”, I does
not own Doodle. E owns doodle. Rest Either C or I owns Bulldog and beagle.
So, the final arrangement is:

56. (a); 57. (b); 58. (c);

59. (e); 60. (d);


46 Adda247 Publications For any detail, mail us at
Publications@adda247.com
Cracker Book for Bank (IBPS | SBI | RRB PO | Clerk) Mains Exams

Solutions (61-65):
Given, “S and P sit on same side of the table. The one who likes Jim Marison
sits 2nd right to S. Three people sit between the one who likes Jim Marison
and W. R who likes evergreen faces S, who likes Cafe 1947. T sits opposite
to Q.V sits 3rd right to U”, we get the following arrangement:

Given, “Neither P nor its neighbors likes Shiva or Moon dance. More than
three persons sit between the one who likes johnson and the one who like
Moon dance, when counted in clockwise direction from of the one who likes
Johnson. Neither W nor his neighbors like johnson. T R does not sit next to
the one who like Johnson he one who likes Jim marison does not face the
ones who like moonpeak espresso and johnson.”
From these conditions only place left for the ones who likes Johnson that U
likes Johnson, V likes Moon dance, Also, since, neither P nor its neighbor
likes shiva café so T likes shiva café.
Also, No one sits between the one who likes freedom cafe and T. Only
position left, T is immediate left to W, who likes freedom cafe and P likes
moonpeak espresso.

47 Adda247 Publications For any detail, mail us at


Publications@adda247.com
Cracker Book for Bank (IBPS | SBI | RRB PO | Clerk) Mains Exams

61. (d); 62. (e); 63. (a);

64. (e); 65. (d);

Solutions (66-70):
A sit at one of the ends of the row. More than four persons sit between A
and D.H sits 3rd left to B. A is not neighbor of H. Odd number of persons sit
between H and D. we get four possibilities:

C who is one of the immediate neighbors of H, sits 3rd left to the one who
distributes meal on Mahavir Jayanti. The one who distributes meal on
Mahavir Jayanti sits immediate left of B, So, case4 gets eliminated as C
cannot be placed next to D. The one who distributes meal on Budh Purnima
sits at extreme left end, therefore in all the remaining cases A distributes
meal on Budh Purnima.

There is one person less sit between A and G as compared to the number of
persons who are sitting between G and F, as there is no place left for G and F
in case 1 according to this condition, therefore case1 gets eliminated.
48 Adda247 Publications For any detail, mail us at
Publications@adda247.com
Cracker Book for Bank (IBPS | SBI | RRB PO | Clerk) Mains Exams

The ones who distribute meal on Diwali and Lohri sit together. Neither A
nor C distributes on Lohri and Diwali. The one who distributes meal on
republic day sits 3rd right to the one who distributes on Lohri, who sits
immediate left to the one who distributes on Diwali, but given that B does
not distribute meal on republic day so case2 gets eliminated.
Continuing with case 3, one person sit between A and G and two persons
are there in between G and F. So, only place left for E is immediate right to
A. E distributes on Lohri, G on Diwali and H on republic day.
Only one person sits between the ones who distribute meal on Christmas
and Eid-ul-fitr and none of them sit at the corner. So, B distributes meal on
Independence Day. More Than one person sits between the one who
distribute meal on Independence Day and Eid-ul-fitr. So, C distributes on
Eid-al-fitr .
The final arrangement is:

66. (d); 67. (e); 68. (b);

69. (c); 70. (a);

49 Adda247 Publications For any detail, mail us at


Publications@adda247.com
Cracker Book for Bank (IBPS | SBI | RRB PO | Clerk) Mains Exams

 Key Points to Remember for New Pattern Questions


➢ Solving of these questions is similar to a chain process in
which each condition or information in some way or the
other is linked with each other. If you get that one key root
or node of it you will be able to solve it easily.
➢ New concept of undefined number of persons is introduced
in the recent examination. In this type of questions first you
should start adjusting the defined persons in the
arrangement and after going forward with the given
conditions you will be able to find the total number of
persons in that arrangement.
➢ There is also a new pattern introduced of linear
arrangement in the mains exam in which the total length of
the row is given and you have to adjust the given persons in
that row and all the persons are at a given multiple from
each other, so while solving this the factor of distance
should be kept in mind as it plays a major role.
➢ Some times you will see in the exam that the liking of the
persons differ according to their position. For ex- those
who are sitting at the corner like fruit while those who are
sitting in the middle like colours so be careful while reading
the direction also solve the arrangement accordingly.
➢ One new thing that has been seen in the arrangement is the
figure based arrangement in which a figure is given and you
have to place the persons as per the given conditions within
that figure, so the method should be that first we should go
on with the seating arrangement as start placing the
persons around that figure.
➢ A new concept of undefined persons has also been seen in
which you get to know that there are seven or eight
persons and they are in alphabetical order but those
alphabets are not defined so in this type you have to take all
the possible combinations as if K, L and M are given then
you have to take J, K, L, M… or K, L, M, N…. and so on…

50 Adda247 Publications For any detail, mail us at


Publications@adda247.com
Cracker Book for Bank (IBPS | SBI | RRB PO | Clerk) Mains Exams

Adda247 Publications For any detail, mail us at


1
Publications@adda247.com
Cracker Book for Bank (IBPS | SBI | RRB PO | Clerk) Mains Exams

Chapter

2 Puzzle

BEST APPROACH TO SOLVE THE QUESTIONS

Being a student you should know about the recent changes that have been
seen in the recent exams.
So, we are providing you some questions based on the latest exams and also
providing you the detail solution of it. So that you may get to know how to
solve these questions. Here is the approach to solve such questions.

Example-1:

Directions: Study the following information carefully and answer the


questions given below. (IBPS Clerk Mains-2017)
There are five doctors of different types viz., Ophthalmologist, Orthopedic
surgeon, Psychiatrist, General physician and Dentist in a hospital but not
necessarily in same order. They attend their patients at different timing in a
single day. The duration of their meeting with the patient is different. The
total duration of the meeting with the patients by all doctors is of 11 hour.
There is no gap between the meeting time of all doctors. The duration of
meeting either full hour or half an hour but not in one third or one fourth.
The one who is orthopedic surgeon attends their patients from 12:30 to
3pm.The dentist attends their patients immediate before or immediate after
the Orthopedic surgeon. The meeting hour of Dentist with the patients is
three hours. The duration of meeting with patients of General physician is
more than duration of meeting of Psychiatrist with patients by 1 hour.
General physician attends meeting just after the meeting of Psychiatrist.
The general Physician does not attend patients at the last. The
Ophthalmologist attends their patients before the Orthopedic surgeon but
not immediate before. The duration of meeting of Ophthalmologist is of 1.5
hours. The timing of Psychiatrist meeting is after 6:00 am.

Adda247 Publications For any detail, mail us at


2
Publications@adda247.com
Cracker Book for Bank (IBPS | SBI | RRB PO | Clerk) Mains Exams

Solutions: The one who is orthopedic surgeon attends their patients from
12:30 to 3pm.The dentist attends their patients immediate before or
immediate after the Orthopedic surgeon. The meeting hour of Dentist with
the patients is three hours. So, from this we get that----

Case-1:
Doctors Meeting hour
Orthopedic surgeon 12:30- 3:00 pm
Dentist 3:00- 6:00 pm

Case-2:
Doctors Meeting hour
Dentist 9:30am-12:30pm
Orthopedic surgeon 12:30- 3:00 pm

The Ophthalmologist attends their patients before the Orthopedic surgeon


but not immediate before. General physician attends meeting just after the
meeting of Psychiatrist. The duration of meeting of Ophthalmologist is of
1.5 hours. The general Physician does not attend patients at the last. The
duration of meeting with patients of General physician is more than
duration of meeting of Psychiatrist with patients by 1 hour.
Case-1:
Doctors Meeting hour
Ophthalmologist 7:00-8:30am
Psychiatrist 8:30-10:00am
General physician 10:00am-12:30pm
Orthopedic surgeon 12:30- 3:00 pm
Dentist 3:00- 6:00 pm
Case-2:
Doctors Meeting hour
Psychiatrist 4:00-5:30am
General physician 5:30-8:00am
Ophthalmologist 8:00-9:30 am
Dentist 9:30am-12:30pm
Orthopedic surgeon 12:30- 3:00 pm

Adda247 Publications For any detail, mail us at


3
Publications@adda247.com
Cracker Book for Bank (IBPS | SBI | RRB PO | Clerk) Mains Exams

Case-3:
Doctors Meeting hour
Ophthalmologist 4:00-5:30am
Psychiatrist 5:30-7:00am
General physician 7:00-9:30am
Dentist 9:30am-12:30pm
Orthopedic surgeon 12:30- 3:00 pm

The timing of Psychiatrist meeting is after 6:00 am. So, from this case-2 and
3 gets eliminated. So, the final arrangement is----
Doctors Meeting hour
Ophthalmologist 7:00-8:30am
Psychiatrist 8:30-10:00am
General physician 10:00am-12:30pm
Orthopedic surgeon 12:30- 3:00 pm
Dentist 3:00- 6:00 pm

Example-2:

Directions: Study the following information carefully and answer the


questions:
(SBI Clerk Mains-2018)
There are three compartments A, B, C such as compartment A is in west of
compartment B and compartment A and compartment B is in west of
compartment C. Twelve boxes P, Q, R, S, V, X, Y, Z, K, L, M, N are placed in
three different compartments such as four boxes are placed in each
compartment . And these four boxes are placed one above another. Only
one box is placed in between V and Z in compartment B. R is placed on top
in compartment A. Box Y is in the immediate west of L. Box L is placed
between box M and N. M is placed above N. Two boxes are placed between
R and S in same compartment. Box Q is placed immediately above P in the
same compartment. Box X is placed above box V and Z in the same
compartment. Box V is placed above box Z.

Adda247 Publications For any detail, mail us at


4
Publications@adda247.com
Cracker Book for Bank (IBPS | SBI | RRB PO | Clerk) Mains Exams

These boxes are shifting in other compartments as per the cards drawn and
only two cards drawn at a time -
I. If both the card drawn is heart then the box placed at the top in
compartment B will be interchanged with the box placed at the bottom
of Compartment C.

II. If among the card drawn one is diamond and another is spade then the
box which is second from the bottom in Compartment A will be inter
changed with box placed at second from the top in Compartment C.

III. If among the two cards drawn one is Club and another is Heart then the
box placed at top and the box placed at the bottom will be interchanged
in compartment B.

IV. If among the two cards drawn one is club and another is diamond then
the box which is third from the bottom in compartment B is
interchanged with the box which is third from the top in compartment
A.

The Cards Drawn are---


1. Club-Heart
2. Heart-Heart
3. Club-Diamond
4. Spade-Diamond
Note- The cards will be drawn in the given serial order.

Sol. Only one box is placed in between V and Z in compartment B. R is


placed on top in compartment A. Box Y is in the immediate west of L.
Box L is placed between box M and N. M is placed above N. Two boxes
are placed between R and S in same compartment. Box Q is placed
immediately above P in the same compartment. Box X is placed above
box V and Z in the same compartment. Box V is placed above box Z.

Adda247 Publications For any detail, mail us at


5
Publications@adda247.com
Cracker Book for Bank (IBPS | SBI | RRB PO | Clerk) Mains Exams

Compartment Compartment Compartment


A B C
R X K
Q V M
P Y L
S Z N

Step II: According to the given conditions, The cards drawn is- 1. Club-
Heart, for this If among the card drawn one is Club and another is Heart
then the box placed at top and the box placed at the bottom will be
interchanged in compartment B. So, from given conditions, Box X and Z will
be interchanged with each other.

Compartment Compartment Compartment


A B C
R Z K
Q V M
P Y L
S X N

Step III:2. Heart-Heart, If both the card drawn is heart then the box placed
at the top in compartment B will be interchanged with the box placed at the
bottom of Compartment C. So, from given conditions, Box Z and N will be
interchanged with each other.

Compartment Compartment Compartment


A B C
R N K
Q V M
P Y L
S X Z

Adda247 Publications For any detail, mail us at


6
Publications@adda247.com
Cracker Book for Bank (IBPS | SBI | RRB PO | Clerk) Mains Exams

Step IV: 3. Club-Diamond, If among the card drawn one is club and another
is diamond then the box which is third from the bottom in compartment B
is interchanged with the box which is third from the top in compartment A.
So, from the given conditions, Box P and box V will be interchanged.

Compartment Compartment Compartment


A B C
R N K
Q P M
V Y L
S X Z

Step V: 4. Spade-Diamond, If among the card drawn one is diamond and


another is spade then the box which is second from the bottom in
Compartment A will be inter changed with box placed at second from the
top in Compartment C. So, from the given conditions, Box V will be
interchanged with M.

Compartment Compartment Compartment


A B C
R N K
Q P V
M Y L
S X Z

Adda247 Publications For any detail, mail us at


7
Publications@adda247.com
Cracker Book for Bank (IBPS | SBI | RRB PO | Clerk) Mains Exams

Practice Exercise Based on new Pattern

Directions (1-5): Read the given information carefully to answer the


following questions.
There are seven cars viz. A, B, C, D, E, F and G on two parallel roads X and Y.
Four of the given seven cars parked while rest of the three are moving along
the road. The parked cars are facing either north or south direction. The
roads are narrow and are to be considered as a straight line. Distance
between two cars on road X is a multiple of 5, while the distance between
two cars on road Y is a multiple of 3. There are three cars to the right of A.
Distance between B and F is 45 meters. Distance between E and the car in
front of it is two-third of the distance between car E and the car behind it.
Distance between G and A is half of the distance between G and D. Distance
between car C and D is a multiple of 3 but less than 28m. C is 25 meters
ahead of G. B is to the left of G. The moving car on road Y is moving towards
B. Car F is to the North-East of car D. E is moving in a direction opposite to G
and C. Car D is not anywhere behind car G. Only three cars are there on road
Y. F is not behind car G. Distance between G and D is more than 20. No Car is
moving towards F.

1. Which of the following car is facing/moving towards West direction?


(a) A (b) C (c) F
(d) Cannot be determined (e) None of these
2. What is the shortest distance between car F and A?
(a) More than 65 meters (b) 65 meters (c) 57 meters
(d) 55 meters (e) 45 meters

3. Which of the following car is to the left of F?


(a) E (b) B (c) Both B and E
(d) Cannot be determined (e) None of these

4. What is the direction of car E with respect to A?


(a) North East (b) North West (c) South West
(d) South East (e) None of these
Adda247 Publications For any detail, mail us at
8
Publications@adda247.com
Cracker Book for Bank (IBPS | SBI | RRB PO | Clerk) Mains Exams

5. What is the distance between car A and D?


(a) 60 meters (b) 75 meters
(c) 33 meters (d) 55 meters
(e) None of these

Directions (6-10): Study the following information carefully and answer


the given questions.
Seven people A, B, C, D, E, F, G participated in a race which is of a certain
duration, also each of them got different ranks. The track is a linear one and
is of 100m. The one who got first rank run complete 100m but rest all run
different distance but less than 100m in that time. C ran 10 m more than G.
Both A and B run less distance than D. One of the person covers 85m
distance. The one who got II rank is only 5m behind the border line. C
covers less distance than F. The difference between the distance covered by
C and D is 5m. None of the given person runs 90m and nobody covers less
distance than 75m. The difference between the distance covered by C and G
is same as the difference between the distance covered by F and A. F runs
less distance than E. The difference between the sum of the distance
covered by A and D and sum of the distance covered by E and F is 23. B
covers 3m less distance than G.

6. Who among the following got III rank?


(a) D (b) C (c) F
(d) Cannot be determined (e) None of these

7. What is the sum of the distance covered by A and B?


(a) 190 m (b) 165 m (c) 164 m
(d) 155 m (e) None of these

8. What is the difference between the distance covered by D and E?


(a) 15 m (b) 16 m (c) 14 m
(d) 13 m (e) None of these

9. Who got fifth rank?


(a) G (b) A (c) D
(d) C (e) F
Adda247 Publications For any detail, mail us at
9
Publications@adda247.com
Cracker Book for Bank (IBPS | SBI | RRB PO | Clerk) Mains Exams

10. How much distance covered by G?


(a) 92m (b) 79m (c) 95m
(d) 82m (e) None of these

Directions (11-15): Study the following information carefully to answer


the given questions.
Eight football players R, Q, P, O, N, M, S and T have different annual earnings
viz. one million, three million and ten million. Out of these, two are females.
They all live in a building having eighteen floors with top floor being
numbered as 18th , second top floor is number 10th and bottom-most being
numbered as 1st . Only one player lives on each floor. It is given that top
floor and tenth floor is occupied by two of these players, one by each player.
Each of the given eight players likes the following boxers viz. A, B, C, U and
V. Two of them likes A, two likes U, two likes B and one likes C and one likes
V. Q earns three millions and likes U, and her friend N likes A and earns ten
millions. O, M and N lives on consecutive floors. The one who earns three
millions and likes A lives on topmost floor. S lives below the one of the man
who earns three millions. None of the players lives between T and S. M
likes U, and earns the same amount of money as S earns, who likes B. No
two players liking same boxers earns the same amount of money. O lives
above M who live below N. O likes B and earns one million while his friend
T who likes A earns three millions with only one another person. Not more
than three of them earns ten millions. S, R and the one who likes U are living
on consecutive floors respectively. The players who earns one million lives
on the floors which are divisible by three. No one live on fourth and fifth
floor. No female likes C and V. Females players earns either three millions
or ten millions. Neither T nor M is a female. There are only two such pairs of
three persons who live on consecutive floors. The one who earns ten million
lives on the bottom most floor. R lives above the floor of Q.

11. R likes which of the following boxers?


(a) A
(b) V
(c) C
(d) Can’t be determined
(e) None of these

Adda247 Publications For any detail, mail us at


10
Publications@adda247.com
Cracker Book for Bank (IBPS | SBI | RRB PO | Clerk) Mains Exams

12. Which of the following pair represents females?


(a) QS (b) NQ (c) NS
(d) Either (a) or (b) (e) Either (a) or (c)

13. Who among the following earns three millions?


(a) O (b) M (c) S
(d) Q (e) R

14. Which of the following is correctly matched?


(a) P – one million – B (b) Q – ten millions– U
(c) S –three millions – A (d) M –ten millions– U
(e) None of these

15. Who amongst the given eight players stays between Q and P?
(a) M (b) Q (c) S
(d) None of these (e) No one

Directions (16-20): Read the following information carefully and answer


the questions.

In a family 12 members i.e. A, B, C, D, E, F, G, H, I, K, L and J are from 4


different rooms, only 3 members live in each room. Rooms are numbered as
14, 15, 16, 11. Each room is painted with different colours Red, Green, Blue,
Yellow (not necessarily in same order). Each room is situated in different
apartments i.e. Parsavanath, Pragati, Antariksh and Ansal (not necessarily
in same order).

A and G are from same room and their room is an odd numbered room. J’s
room colour is Green. B’s and K’s room colour is red. E and D, are from
different rooms but they are from even numbered room. Yellow and blue
are colours of even numbered room. B neither share the room with E nor
with D. H is from odd numbered room. F and C are not from same room. C is
from blue coloured room. Room number 14 colour is not yellow. C’s room in
Pragati apartment. 11 numbered room is in Antariksh apartment but it is
not red coloured room. D and L share same room in Ansal apartment.

Adda247 Publications For any detail, mail us at


11
Publications@adda247.com
Cracker Book for Bank (IBPS | SBI | RRB PO | Clerk) Mains Exams

16. What is the number of B’s room and name of B’s apartment?
(a) 15, Pragati (b) 16, Ansal (c) 15, Ansal
(d) 15, Parsavanath (e) None of these

17. What is the colour of the room which is situated in Parsavanath


apartment?
(a) Blue (b) Yellow (c) Red
(d) Green (e) None of these

18. Which of the following is a colour of room number 11?


(a) Red (b) Green (c) Blue
(d) Yellow (e) None of these

19. What is the number of E’s room?


(a) 11 (b) 16 (c) 14
(d) 15 (e) None of these

20. Which of the following combination is not true?


(a) A- Green-Antariksh (b) E- Blue-Pragati (c) C- Blue-Ansal
(d) F-Yellow-Ansal (e) J- Green- Antariksh

Direction (21-25): Study the following information carefully and answer


the question given below-

There are seven boxes i.e. A, B, C, D, E, F and G which are kept one above the
other but not necessarily in the same order. They all contains different topic
books i.e. Puzzle, Machine Input (MI), English, Logical, Math’s, Computer,
and Ranking but not necessarily in the same order. All the box contains
different number of books. The total number of books in all the boxes is
192. No two boxes have same number of books.
Three boxes are kept between box F having odd number of books which is a
perfect cube and the box which contains computer book. Box C having 21
books kept immediately below the box having 34 books in it. Only one box
is kept between box A which contains Logical books and the box having
Computer books. Box G has 1 book less than box D. Box containing Machine

Adda247 Publications For any detail, mail us at


12
Publications@adda247.com
Cracker Book for Bank (IBPS | SBI | RRB PO | Clerk) Mains Exams

Input book kept immediately above the box containing English books,
which is 6 less than the number of logical Book. Box A contains even
number of books which is perfect square. Box F has more books than box C.
Box D is kept immediately above box A. Only two boxes are kept between
box D and box E which has total 34 books in it. More than three boxes are
kept between box containing Ranking book and the box containing Puzzle
book, which is kept above Ranking book. Neither box E nor box F contains
Puzzle book. The box containing puzzle books have total 15 books in it. Box
B has 15 books and is kept above box G. Box A has 9 more books than box F.

21. How many boxes are kept above the box B?


(a) One (b) Four (c) More than four
(d) Three (e) None

22. Which of the following box is kept at the bottom?


(a) C – Ranking (b) E – Computer (c) G – Math’s
(d) B – Puzzle (e) None of these

23. How many books does box A contains in it?


(a) 32 (b) 36 (c) 25
(d) 21 (e) None of these

24. Box F contains which of the following book?


(a) Math’s (b) Reasoning (c) Machine Input
(d) Computer (e) None of these

25. How many books does Box G contains?


(a) 21 (b) 29 (c) 30
(d) 27 (e) None of these

Direction (26-30): Study the following information carefully and answer


the question given below-
There are seven boxes i.e. A, B, C, D, E, F and G which are kept one above the
other. The boxes are of different colour i.e. Blue, white, Red, Black, Green,
Pink and Yellow but not necessarily in the same order. The boxes contains
different number of same items i.e. 16, 25, 9, 19, 10, 28, 32 but not
necessarily in the same order. The box are of different sizes. The total size
of all the boxes is 80 sq. cm.
Adda247 Publications For any detail, mail us at
13
Publications@adda247.com
Cracker Book for Bank (IBPS | SBI | RRB PO | Clerk) Mains Exams

Only two boxes are kept between box E, which is of Red colour and the box
which contains 28 items which is 15 sq.cm in length. Box B contains items
which is perfect square number but does not contain even number of items.
Only one box is kept between box F which is 12 sq.cm in size and the box
which is Red in colour. There are three boxes kept between box D which of
Green colour and the box F which is of Blue colour. Box A contains 4 items
less than box C. Only one box is kept between Box C and box D. Box D is kept
below box E but not kept at the bottom. Box C is of yellow colour and is kept
at the bottom. Only one box is kept between box C and box which is of 13
sq.cm in size. G is of Black in colour and is 14 sq.cm in size is Kept
immediately below the box containing 9 items which is 7 sq.cm in size. Box
F contains more item than box D but less items than box G. Box B is 1 Sq.cm
more in size than box C and is not of pink in colour.

26. Which of the following box is white in colour?


(a) Box A (b) Box D (c) Box B
(d) Box F (e) None of these

27. What is the total size of Box C, B and D?


(a) 32 Sq.cm (b) 31 Sq.cm (c) 36 Sq.cm
(d) 34 Sq.cm (e) None of these

28. Which of the following box is kept at the top?


(a) Box B (b) Box F (c) Box E
(d) Box G (e) None of these

29. How many items does box B contains?


(a) 16 (b) 9 (c) 28
(d) 25 (e) None of these

30. Which of the following combination of ‘Box-colour-no. of items’ is


correct?
(a) B- White- 25 (b) A- Pink- 28 (c) D- Green- 10
(d) F- Blue- 16 (e) All are correct
Adda247 Publications For any detail, mail us at
14
Publications@adda247.com
Cracker Book for Bank (IBPS | SBI | RRB PO | Clerk) Mains Exams

Directions (31-35): Study the following information carefully and answer


the question given below.

Six persons A, B, C, P, Q, R attend a seminar on three different months of the


same year on different dates i.e. 15th and 22nd. Months are January, April
and August. Each person attends only one seminar on each day.

B attends the seminar on 15th of the month, but not attends in the month
which has 30 days. Two persons attend seminar between B and B’ son. B’s
father attend seminar in January. Two persons attends seminar between B’s
wife and P, who is in the third generation of the family. B’s wife attends
seminar before B’s son-in-law. P is the daughter of A. Q attends seminar on
15th of the month. Two persons attend seminar between R and B’s son-in-
law. R has only one son.

31. How many persons attend seminar between Q’s wife and B’s wife?
(a) Three (b) Two (c) Four
(d) One (e) None of these

32. Who attends seminar on 22nd April?


(a) A’s daughter (b) R (c) B
(d) Q’s brother-in-law (e) None of these

33. R attends seminar on which of the following date?


(a) 15th April (b) 15th January (c) 22nd August
(d) None of these (e) 22nd January

34. How is Q related to A?


(a) Son-in-law (b) Daughter (c) Father
(d) Daughter-in-law (e) None of these

35. Four of the following five are alike in a certain way and hence they form
a group. Which one of the following does not belong to that group?
(a) B (b) C (c) A
(d) Q (e) R

Adda247 Publications For any detail, mail us at


15
Publications@adda247.com
Cracker Book for Bank (IBPS | SBI | RRB PO | Clerk) Mains Exams

Direction (36-40): Study the following information carefully and answer


the question below-
Seven persons A, B, C, D, E, F, G are going to participate in seven different
races all of which are of different milestones viz. 200, 350, 500, 800, 1000,
1200 and 1500. Each of them got different ranks in their respective races as
1st, 2nd, 3rd…and so on till 7th. Also each of them participated in the race in
different years viz. 1995, 1998, 2000, 2002, 2007, 2010, 2011. The one who
got 1st rank participated in 1200m race. C participated 2 years after A. D
got 4th rank. The milestone of F’s race was 4 times of the milestone of the
who participated in race in 2011. The one who participated first got 3rd
rank. There is a difference of one year between B and G’s participation year.
F got the lowest rank. Only two person got lower rank than G. A’s milestone
of race is twice of the one whose rank is just lower than him. The one who
got 6th rank participated in odd numbered year. The one whose rank is just
higher than G participated in an odd numbered year. The one whose rank is
last participated after the one whose rank is first. The one who participated
in 2007 does not have milestone which is a multiple of the one whose rank
is 3rd. B does not get higher rank than A.

36. Who among the following participated in 1995?


(a) B (b) G (c) A
(d) D (e) E

37. Who among the following got 6th rank?


(a) B (b) G (c) A
(d) D (e) E

38. What is the milestone of C?


(a) 350m (b) 1500m (c) 1000m
(d) 1200m (e) 500m

39. Who among the following participated second in the race?


(a) B (b) G (c) A
(d) D (e) E
Adda247 Publications For any detail, mail us at
16
Publications@adda247.com
Cracker Book for Bank (IBPS | SBI | RRB PO | Clerk) Mains Exams

40. What will be the difference between the milestones of the one who got
3rd rank and the one who got 6th rank?
(a) 300m (b) 700m (c) 1000m
(d) 200m (e) None of these

Directions (41-45): Study the following information carefully to answer


the given questions.
In Chennai, seven persons namely K, L, M, N, O, P and Q are living in an
apartment of a seven storey building. They are living on separate different
floors. The ground floor of the building is numbered 1, the floor above it 2
and so on until the topmost floor is numbered 7. Each person is of different
age. Also Each person likes different cars – BMW, Audi, Acura, Bugatti,
Buick, Desire and Safari, but not necessarily in the same order.
The person who likes Bugatti lives on floor numbered four. The one who
lives immediately above the one who likes Desire Car is 28 years old. P is 33
years old. K does not live on the lowermost floor. K lives on any odd
numbered floor below the one who likes Bugatti. Only two persons live
between K and the person who likes Safari. Only one person lives between L
and P. The difference between the ages of O and Q is 2 years. P lives on an
even numbered floor and does not like Bugatti. Only three persons live
between the persons who like BMW and Acura respectively. M is 9 years
older than K. The person who likes BMW lives on any floor above the L’s
floor. The person who likes BMW does not live on the topmost floor. The
one who lives on the middle floor of this building is 35 years old. Q lives on
an even numbered floor but neither immediately above nor immediately
below the floor of K. The one who likes BMW, is 30 years old but younger
than Q. M does not like BMW or Acura. Only two persons live between N
and the one who likes Bugatti. The person who likes Buick lives on the floor
immediately above the floor of the person who likes Desire. The sum of the
ages of those who live on 1st floor and 2nd floor is 64 years.

41. How many persons live between the floors on which Q and the one who
is 31 years old lives?
(a) Three (b) Two (c) Four
(d) Five (e) No one

Adda247 Publications For any detail, mail us at


17
Publications@adda247.com
Cracker Book for Bank (IBPS | SBI | RRB PO | Clerk) Mains Exams

42. Which of the following statements is/are true according to the given
information?
(a) O lives on floor numbered 5 and he does not like Bugatti
(b) K likes Buick and he does not live on floor numbered 4
(c) M likes Audi and he lives on the topmost floor
(d) Only two persons live between the floors of O and P
(e) All the statements are true.

43. Who among the following lives on the floor immediately above the floor
of the one who is 28 years old?
(a) L (b) P (c) Q
(d) M (e) No one

44. Who among the following lives exactly between the floors on which Q
and L live?
(a) P (b) O (c) N
(d) M (e) No one

45. Who among the following like Acura?


(a) K (b) M (c) L
(d) N (e) No one

Directions (46-50): Study the following information carefully and answer


the question given below:

Eight persons A, B, C, D, E, F, G, H work in different departments of a


company viz. Sales, IT, HR. Not more than three persons work in a single
department. Also each of them like different colours viz. Purple, Yellow,
White, Red, Orange, Pink, Green, Black. C likes green color and work in HR
department. B works in the same department with only one who likes
Purple color. Neither D nor E work in the same department as B. F likes
Orange color. Both A and H work in the same department. The one who
works in HR department like White color. Neither A nor D works in HR
department. The one who works in IT department like Pink color. The one
who works in the same department as G does not like Pink colour. H does
not like Pink color. B neither likes Red nor Black color. H does not like Red
color. D does not like Pink color.
Adda247 Publications For any detail, mail us at
18
Publications@adda247.com
Cracker Book for Bank (IBPS | SBI | RRB PO | Clerk) Mains Exams

46. G works in which of the following department?


(a) IT (b) HR (c) Sales
(d) Either (a) or (b) (e) None of these

47. Who among the following like Black color?


(a) H (b) E (c) D
(d) B (e) None of these

48. Who among the following works in HR department?


(a) H (b) A (c) F
(d) B (e) None of these

49. In which of the following department does exactly two persons are
working?
(a) IT (b) HR (c) Sales
(d) Either (a) or (b) (e) None of these

50. Who among the following likes Red color?


(a) H (b) A (c) D
(d) B (e) None of these

Direction (51-55): Study the following information carefully and answer


the question given below-
Eight persons i.e. A, B, C, D, E, F, G and H are sitting around a circular table
while having dinner. Some of them face inside and some of them face
outside the centre. Each of them is at equal distance from each other. B is
sitting third to the right of A. H sits third to the right of G. D is not the
immediate neighbour of A. H is the immediate neighbour of D. D sits second
to the right of B. E sits second to the right of F, who sits second to the left of
H. C and E faces same direction. The table is arranged in such a way that
two of the given eight persons are facing towards north. D and C faces
opposite direction. C sits second to the left of D. E faces H. After some time,
each of them started leaving the table one by one as one finished his dinner.
F finished first and left the table and walked in the same direction he is
facing to reach point M which is two meters away from his initial position.
After F, E got up from the table, turned 180 degrees and walked four meters
Adda247 Publications For any detail, mail us at
19
Publications@adda247.com
Cracker Book for Bank (IBPS | SBI | RRB PO | Clerk) Mains Exams

to reach point P which is five meters to the south of point Q. Point Q is in the
east of point M. Now, A left the table and walks for 7m in west to reach
point L. After A, H leaves the table and walks 4m in the direction opposite to
in which he was facing initially and reach point S which is 3m north of point
L. At this instant the remaining five persons are still sitting at the initial
positions around the circular table.

51. What is the shortest distance between point P and point S?


(a) 12 meters
(b) 13 meters
(c) 14 meters
(d) More than 16 meters
(e) None of these

52. What is the shortest distance between A and F’ initial position when
they all are sitting around the circular table?
(a) 3 meters
(b) 5 meters
(c) 7 meters
(d) Cannot be determined
(e) None of these

53. In which direction is point L with respect to the E’s Initial position at
the time of dinner?
(a) South
(b) Southwest
(c) Northwest
(d) West
(e) None of these
54. Who among the following is sitting third to the left of C at the instant
when they all are sitting around the circular table?
(a) A (b) E (c) B
(d) F (e) D

55. Who among the following is sitting farthest from point M at the instant
when they all were sitting around the circular table?
(a) A (b) B (c) C
(d) D (e) None of these
Adda247 Publications For any detail, mail us at
20
Publications@adda247.com
Cracker Book for Bank (IBPS | SBI | RRB PO | Clerk) Mains Exams

Direction (56-60): Read the following information carefully and answer


the questions given below:
Eight persons L, M, N, O, P, Q, R, S are working on an eight storey building
where ground floor is numbered as 1 and top floor is numbered as 8. Each
person works on a different floor and also ordered different snacks from
the restaurant viz. Aloo tikki, Masala Vada, Noodles, Pizza, Pakoda, Pav
bhaji, Sweet corn, Samosa. They also drink different varieties of drink from
CCD machine viz. Tea, Coffee, Lemon Tea, and Soup. 3 persons drink tea, 2
persons drink coffee and soup each and 1 person drinks Lemon tea.
There are three floors in between the one who ordered Masala Vada and Q
and both of them work on an odd numbered floor. L works immediate
above O. S drinks Tea. The one who ordered Pakoda also drinks Soup. Both
the persons, who drinks Coffee, works on two consecutive floors. N ordered
Sweet corn and lives below the one who ordered Samosa. P drinks neither
Tea nor Coffee and works on any floor below Q. The one who works on
lowermost floor ordered Pav Bhaji. Only the persons who work on even
number floor drinks Tea. The one who works on 4th floor ordered Pizza but
did not drink Tea. Neither O nor L ordered Pizza. M drinks Soup and both
the persons who work on a floor which is immediately above and
immediately below him drinks Tea. Samosa is ordered with Coffee but not
order by Q. Only one person works in between the floors of the one who
ordered Noodles and L.

56. Who among the following ordered Noodles?


(a) L (b) S (c) P
(d) R (e) Q

57. Which of the following drink is taken with Aloo Tikki?


(a) Lemon Tea
(b)Tea
(c) Either Coffee or Lemon Tea
(d) Soup
(e) Data inadequate
58. R ordered which snack?
(a) Pakoda (b) Pav Bhaji (c) Noodles
(d) Samosa (e) Pizza
Adda247 Publications For any detail, mail us at
21
Publications@adda247.com
Cracker Book for Bank (IBPS | SBI | RRB PO | Clerk) Mains Exams

59. On which of the following floor does O work?


(a) 7th (b) 4th (c) 3rd
(d) 5th (e) 2nd

60. How many persons lives between the one who ordered Pakoda and the
one who ordered Sweet corn?
(a) Five (b) Six (c) Two
(d) Three (e) None
Direction (61-65): Study the following information carefully and answer
the question below-
Six persons A, B, C, D, E and G who all belongs to a family having three
generation and all are sitting around two inscribed squares table such
that all the persons sit at the three corners of each outer and the inner
squares. Some of them face inside and some of them Face outside. F is
the daughter of G. Brother-in- law of G sits second to the right of B and
both faces opposite direction. B is the son-in-law of D and does not sits
in the same square with his daughter. E is the daughter of D, and is not
married to B. There is only one vacant seat in each square. The vacant
seats are not in front of each other. No one sits on the immediate right
of B. Daughter of B sits on the immediate left of G, who faces E. E does
not sit in the same square table with F. D is not the immediate
neighbour of G. Parent of E does not sit in the same square with his son-
in-law. B has no brother. F and A face same direction. F does not face
vacant seat. E and F faces same direction. B and F does not face each
other. A is married to E.

61. How is D related to F?


(a) Maternal Grand-father or Maternal Grandmother
(b) Grandmother (c) Aunt
(d) Father (e) None of these

62. How many persons faces inside?


(a) One (b) Three
(c) Two (d) Four
(e) Can’t be determine

Adda247 Publications For any detail, mail us at


22
Publications@adda247.com
Cracker Book for Bank (IBPS | SBI | RRB PO | Clerk) Mains Exams

63. How is G is related to D?


(a) Son-in-law (b) Son
(c) Daughter-in-law (d) Daughter
(e) None of these

64. Which among the following pair sits in the same square?
(a) B, D (b) A, F
(c) F, E (d) None of these
(e) G, D

65. Who among the following are the immediate neighbours of the vacant
seat?
(a) D, E (b) F, D
(c) A, E (d) E, F
(e) G, D

Directions (66-70): Study the following information carefully and answer


the questions below:

Ten ASEAN countries members (Brunei, Cambodia, Indonesia, Laos,


Malaysia, Myanmar, Philippines, Singapore, Thailand, Vietnam) and its six
FTA partners (Australia, New Zealand (NZ), India, China, Japan and Korea)
are living on different floors in a ten-storey hotel such that 6 of the floors
are doubly occupied. None of the two FTA’s or ASEAN members live on
same floor. Also, they are sitting around a circular table facing the centre in
the following manner as only all the ASEAN members are sitting around the
outer circular table.

Adda247 Publications For any detail, mail us at


23
Publications@adda247.com
Cracker Book for Bank (IBPS | SBI | RRB PO | Clerk) Mains Exams

The arrangement of the persons sitting in the inner circle is irrespective of


the positions of the persons sitting on the outer circle.
The one from the Laos lives on an odd numbered floor and lives just above
the one from India. None of the FTA lives on 10th and 9th floor. Two floors
are there between the one from Vietnam and Philippines. The one from
Australia sits immediate left to the one from New Zealand, who lives above
the one from china and there are three floors between the one from New
Zealand and the one from China. More than three floors are there between
the ones from Laos and Korea, who sits immediate left to the one from
Australia. Three persons sit between the ones from Philippines and
Indonesia (when counted from the left of the one from Philippines). No one
lives with the representatives of Philippines and Myanmar. The one from
Indonesia lives just below the one from Cambodia. Two floors are between
the ones from India and china. The one from the host country and the one
from Thailand live on even numbered floor together. More than two
persons between the ones from Myanmar and Brunei (when counted from
the left of Myanmar.) All the members in both the circles live exactly in the
same sequence as according to the floors they are living in. More than 1
person lives between the ones from Malaysia and Vietnam, who lives above
no one. The one from India does not live on top three floor. The one from
India does not sit next to Korea and the host country representative. The
ones from Malaysia and India live together on same floor. The one from
Myanmar does not lives on the top Floor. The one from Philippines sits
third to the left of the one who is from Vietnam. The one from Singapore
lives on a doubly occupied floor.

66. On which of the following floors does the representative of Singapore


lives?
(a) 4th (b) 2nd
(c) 3rd (d) 7th
(e) None of these

67. Who among the following sits 3rd to the right of the one who is from
Brunei?
(a) Vietnam (b) Laos
(c) Cambodia (d) Singapore
(e) none of these
Adda247 Publications For any detail, mail us at
24
Publications@adda247.com
Cracker Book for Bank (IBPS | SBI | RRB PO | Clerk) Mains Exams

68. Who among the following is the host country?


(a) Australia (b) Japan
(c) Cambodia (d) Singapore
(e) None of these

69. Four of the five are alike in a certain way, who among the following
does not belongs to this group?
(a) Australia (b) Japan
(c) Myanmar (d) India
(e) Brunei

70. Which of the following represents the incorrect combination of persons


living on the same floor?
(a) Australia -Indonesia
(b) Japan-Thailand
(c) Myanmar- New Zealand
(d) India-Malaysia
(e) Vietnam-Korea
Solutions

Solutions (1-5):
Step 1. From the information given in the question,
Only three cars are there on road Y. There are only three moving cars.
Distance between two cars on road X is a multiple of 5, while the distance
between two cars on road Y is a multiple of 3. There are three cars to the
right of A. C is 25 meters ahead of G. It means cars A, C and G lies are on
road X. B is to the left of G. It means car B is on road Y. Distance between E
and the car in front of it is two-third of the distance between car E and the
car behind it. The moving car on road Y is moving towards B. E is moving in
a direction opposite to G and C. It means car E is moving towards car B on
road Y. No Car is moving towards F. Car F is to the North-East of car D. Car D
is not anywhere behind car G. Distance between G and A is half of the
distance between G and D.
There can be two possible cases,
Adda247 Publications For any detail, mail us at
25
Publications@adda247.com
Cracker Book for Bank (IBPS | SBI | RRB PO | Clerk) Mains Exams

Case 1. When G is moving in the West direction.

Case 2. When G is moving in the East direction.

But it is given that car F is not behind car G. So, our case 1 will be
eliminated.
There will be two more possible cases when Car A is facing south direction
but both of the cases will be eliminated by the above given conditions.

Step 2. Proceeding with the remaining information,


Distance between B and F is 45 meters. So the distance between car E and B
will be 18 meters while the distance between E and F will be 27 meters.
Distance between G and A is half of the distance between G and D. Distance
between car C and D is a multiple of 3 but less than 28m. It means the
distance between car C and D is equal to 15 meters. Further the distance
between G and D is more than 20 and distance between C and D is multiple
of 3, so from this it is clear that D is ahead of Car C.

Adda247 Publications For any detail, mail us at


26
Publications@adda247.com
Cracker Book for Bank (IBPS | SBI | RRB PO | Clerk) Mains Exams

So, we have our final solution as,

1. (e); 2. (a); 3. (d); 4. (a); 5. (a);


Solutions (6-10):
The one who got first rank run complete 100m but rest all run different
distance but less than 100m in that time. The one who got II rank is only 5m
behind the border line. Means the one who got II rank runs 95m. The
difference between the distance covered by C and D is 5m. C covers less
distance than F. F runs less distance than E. Both A and B run less distance
than D. Means either F or D got second rank but None of the given person
runs 90m. C run 10 km more than G. So, clearly F got II rank.

Rank Persons Distance


I E 100m
II F 95m

The difference between the distance covered by C and G is same as the


distance covered by F and A. So, clearly A covers 85m. The difference
between the sum of the distance covered by A and D and sum of the
distance covered by E and F is 23. From this we get that D covers 87m.

Rank Persons Distance


I E 100m
II F 95m
D 87m
A 85m

Adda247 Publications For any detail, mail us at


27
Publications@adda247.com
Cracker Book for Bank (IBPS | SBI | RRB PO | Clerk) Mains Exams

The difference between the distance covered by C and D is 5m. C run 10 km


more than G. Nobody covers less distance than 75m. It means C runs 92m
and G runs 82m and also C & D got III and IV rank respectively. B covers 3m
less distance than G. So, the final arrangement is---

Rank Persons Distance


I E 100m
II F 95m
III C 92m
IV D 87m
V A 85m
VI G 82m
VII B 79m

6. (b); 7. (c); 8. (d);


9. (b); 10. (d);

Solutions (11-15):

Floor Players Earnings Boxer


18 T(+) three millions A
10 S(+/-) ten millions B
9 R(+) one million C/V
8 Q(-) three millions U
6 P(+) one million V/C
3 O(+) one million B
2 N(+/-) ten millions A
1 M(+) ten millions U

11. (d); 12. (d); 13. (d)


14. (d); 15. (e);
Adda247 Publications For any detail, mail us at
28
Publications@adda247.com
Cracker Book for Bank (IBPS | SBI | RRB PO | Clerk) Mains Exams

Solutions (16-20):

Room Members Colour Apartment


11 AGJ Green Antariksh
14 ECI Blue Pragati
15 BHK Red Parsavanath
16 DFL Yellow Ansal

16. (d); 17. (c); 18. (b);


19. (c); 20. (c);

Solutions (21-25):
Three boxes are kept between box F and the box which contains computer
book. Only one box is kept between box A which contains Logical books and
the box having Computer books. Box D is kept immediately above box A.

Case 1 Case 2
Box No. of Books Book Topic Box No. of Books Book Topic
F Computer
D D
A Logical A Logical

Computer F

Case 3
Box No. of Books Book Topic
F

Computer
D
A Logical

Adda247 Publications For any detail, mail us at


29
Publications@adda247.com
Cracker Book for Bank (IBPS | SBI | RRB PO | Clerk) Mains Exams

Only two boxes are kept between box D and box E which has total 34 books
in it. Box C having 21 books kept immediately below the box having 34
books in it.

Case 1a Case 1b Case 2


No. of Book No. of Book No. of Book
Box Box Box
Books Topic Books Topic Books Topic
F E 34 E 34
D C 21 C 21
A Logical F Computer
D D
E 34 Computer A Logical A Logical
C 21
Computer F

Case 3
Box No. of Books Book Topic
F

E 34
C 21
Computer
D
A Logical

More than three boxes are kept between box containing Ranking book and
the box containing Puzzle book, which is kept above Ranking book. Neither
box E nor box F contains Puzzle book. The box containing puzzle books
have total 15 books in it. Box B has 15 books and is kept above box G. Now,
it is clear that box B contain puzzle book and has 15 books. So, case 1a, 1b, 2
and case 3 is eliminated.
Adda247 Publications For any detail, mail us at
30
Publications@adda247.com
Cracker Book for Bank (IBPS | SBI | RRB PO | Clerk) Mains Exams

Case 1c
Box No. of Books Book Topic
B 15 Puzzle
F
D
A Logical
G
E 34 Computer
C 21 Ranking

Box G has 1 book less than box D has. Box containing Machine Input book
kept immediately above the box containing English books , which is 6 less
than the number of logical Book. So, case 1a and case 2 is eliminated. Box F
have odd number of books which is a perfect cube. Box A contains even
number of books which is perfect square. Box F has more books than box C.
Box A has 9 more books than box F.

Case 1c
Box No. of Books Book Topic
B 15 Puzzle
F 27 Machine Input
D 30 English
A 36 Logical
G 29 Math’s
E 34 Computer
C 21 Ranking

21. (e); 22. (a); 23. (b);


24. (c); 25. (b);

Adda247 Publications For any detail, mail us at


31
Publications@adda247.com
Cracker Book for Bank (IBPS | SBI | RRB PO | Clerk) Mains Exams

Solutions (26-30):
Only two boxes are kept between box E, which is of Red colour and the box
which contains 28 items which is 15 sq.cm in size. Only one box is kept
between box F which is 12 sq.cm in size and the box which is of Red in
colour.

CASE 1 CASE 2
No. of No. of
Box Sizes(Sq.cm) Colour Box Sizes(Sq.cm) Colour
Items Items
E Red 15 Sq.cm 28
F 12 Sq.cm
F 12 Sq.cm
15 Sq.cm 28 E Red

CASE 3 CASE 4
No. of No. of
Box Sizes(Sq.cm) Colour Box Sizes(Sq.cm) Colour
Items Items
F 12 Sq.cm 15 Sq.cm 28

E Red
E Red

15 Sq.cm 28 F 12 Sq.cm

There are three boxes kept between box D which of Green colour and the
box F which is of Blue colour. Box A contains 4 items less than box C i.e. Box
A contains 28 items and box C contains 32 item. Only one box is kept
between Box C and box D. Box D is kept below box E but not kept at the
bottom. So, case 1 and case 4 is eliminated

Adda247 Publications For any detail, mail us at


32
Publications@adda247.com
Cracker Book for Bank (IBPS | SBI | RRB PO | Clerk) Mains Exams

CASE 2
Box Sizes(Sq.cm) Colour No. of Items
A 15 Sq.cm 28
F 12 Sq.cm

E Red

D Green

CASE 3
Box Sizes(Sq.cm) Colour No. of Items
F 12 Sq.cm Blue

E Red

D Green
A 15 Sq.cm 28

Box C is of yellow colour and is kept at the bottom. Only one box is kept
between box C and D. Only one box is kept between box C and box which is
of 13 sq.cm in size. So, case-2 gets eliminated.

Box Sizes(Sq.cm) Colour No. of Items


F 12 Sq.cm Blue

E Red

D 13Sq.cm Green
A 15 Sq.cm 28
C Yellow 32
G is of Black in colour and is 14 sq.cm in size is Kept immediately below the
box containing 9 items which is 7 sq.cm in size. Box B contains items which
is perfect square number but does not contain even number of items. Box F
contains more item than box D but less items than box G. Box B is 1 Sq.cm
more in size than box C and is not of pink in colour. So, the final
arrangement is----

Adda247 Publications For any detail, mail us at


33
Publications@adda247.com
Cracker Book for Bank (IBPS | SBI | RRB PO | Clerk) Mains Exams

Box Sizes(Sq.cm) Colour No. of Items


F 12 Sq.cm Blue 16
B 10 Sq.cm White 25
E 7 Sq.cm Red 9
G 14 Sq.cm Black 19
D 13 Sq.cm Green 10
A 15 Sq.cm Pink 28
C 9 Sq.cm Yellow 32
Total 80 Sq.cm

26. (c); 27. (a); 28. (b);


29. (d); 30. (e);
Solutions (31-35):

Dates
Months 15th 22nd
January B R
April A C
August Q P

31. (b); 32. (d); 33. (e);


34. (a); 35. (c);
Adda247 Publications For any detail, mail us at
34
Publications@adda247.com
Cracker Book for Bank (IBPS | SBI | RRB PO | Clerk) Mains Exams

Solutions (36-40):
D got 4th rank. F got the lowest rank. Only two person got lower rank than G.
The milestone of F’s race was 4 times of the milestone of the who
participated in race in 2011. There is a difference of one year between B
and G’s participation year. So, from this we will get two possible cases---

Persons Rank Milestone Participation Year


A
B 200/ 2011/2010
C
D 4
E
F 7 800
G 5 200/ 2010/2011

Rank based comparison---


>>>D>G>>F

The one who participated first got 3rd rank means the one who participated
in 1995 got 3rd rank. A’s milestone of race is twice of the one whose rank is
just lower than him. C participated 2 years after A. So, it is clear that A got
2nd rank and his milestone of race is 1000. And also we get that the one who
participated in 1995 got 3rd rank and has milestone 500 of race. The one
who got 1st rank participated in 1200m race. B does not get higher rank
than A. So, we get that C got 1st rank and E got 3rd rank.

Persons Rank Milestone Participation Year


A 2 1000
B 200/ 2011/2010
C 1 1200
D 4
E 3 500 1995
F 7 800
G 5 200/ 2010/2011

Rank based comparison---


C> A > E > D > G > > F

Adda247 Publications For any detail, mail us at


35
Publications@adda247.com
Cracker Book for Bank (IBPS | SBI | RRB PO | Clerk) Mains Exams

Now, we get that B got 6th rank. The one who got 6th rank participated in
odd numbered year. So, B participated in 2011. The one whose rank is just
higher than G participated in an odd numbered year. So, D participated in
2007.

Persons Rank Milestone Participation


Year
A 2 1000
B 6 200 2011
C 1 1200
D 4 2007
E 3 500 1995
F 7 800
G 5 2010

Rank based comparison---


C> A > E > D > G >B > F
C participated 2 years after A. The one whose rank is last participated after
the one whose rank is first. So, clearly C participated in 2000 and A
participated in 1998. Further F participated in 2002. Rest, the one who
participated in 2007 does not have milestone which is a multiple of the one
whose rank is 3rd. So, D has milestone 350 and G has milestone 1500. So, the
final arrangement is---

Persons Rank Milestone Participation Year


A 2 1000 1998
B 6 200 2011
C 1 1200 2000
D 4 350 2007
E 3 500 1995
F 7 800 2002
G 5 1500 2010

Rank based comparison---


C> A > E > D > G >B > F

36. (e); 37. (a); 38. (d);


39. (c); 40. (a);
Adda247 Publications For any detail, mail us at
36
Publications@adda247.com
Cracker Book for Bank (IBPS | SBI | RRB PO | Clerk) Mains Exams

Solutions (41-45):

Floors Person Age Car


7 M 37 Audi
6 Q 32 Safari
5 O 30 BMW
4 L 35 Bugatti
3 K 28 Buick
2 P 33 Desire
1 N 31 Acura

41. (c); 42. (e); 43. (a);


44. (b); 45. (d);

Solutions (46-50):
C likes green color and work in HR department. B works in the same
department with only one who likes Purple color. Neither D nor E work in
the same department as B. F likes Orange color. Both A and H work in the
same department. From this it is clear that G works in same department
with only B and likes Purple color.

Persons Colour Department


A
B IT/Sales
C Green HR
D
E
F Orange
G Purple IT/Sales
H

The one who works in HR department like White color. Neither A nor D
works in HR department. So, from this we get that remaining E works in HR
department and likes White color as both A and H work in same
department.
Adda247 Publications For any detail, mail us at
37
Publications@adda247.com
Cracker Book for Bank (IBPS | SBI | RRB PO | Clerk) Mains Exams

Persons Colour Department


A
B IT/Sales
C Green HR
D
E White HR
F Orange
G Purple IT/Sales
H

The one who works in IT department like Pink color. The one who works in
the same department as G does not like Pink colour. H does not like Pink
color. D does not like Pink color. So, it is clear that A likes Pink color and
works in IT department with H and D. And rest B and G works in Sales
department.

Persons Colour Department


A Pink IT
B Sales
C Green HR
D IT
E White HR
F Orange
G Purple Sales
H IT

Not more than three persons work in a single department. So, rest F works
in HR department. B neither likes Red nor Black color. So, B likes Yellow
color. H does not like Red color. So, H likes Black color and D like Red color.

Adda247 Publications For any detail, mail us at


38
Publications@adda247.com
Cracker Book for Bank (IBPS | SBI | RRB PO | Clerk) Mains Exams

Persons Colour Department


A Pink IT
B Yellow Sales
C Green HR
D Red IT
E White HR
F Orange HR
G Purple Sales
H Black IT

46. (c); 47. (a); 48. (c);


49. (c); 50. (c);

Solutions (51-55):
B is sitting third to the right of A. D is not the immediate neighbour of A. D
sits second to the right of B.
H is the immediate neighbour of D. H sits third to the right of G.

E sits second to the right of F, who sits second to the left of H. So, case 1 and
case 3 are eliminated.
C and E faces same direction. D and C faces opposite direction. C sits second
to the left of D. E faces H. So, case 4 is eliminated

Adda247 Publications For any detail, mail us at


39
Publications@adda247.com
Cracker Book for Bank (IBPS | SBI | RRB PO | Clerk) Mains Exams

The table is arranged in such a way that two of the given eight persons are
facing towards north. So F and A are the only persons who faces opposite
direction and sits opposite to each other. After some time, each of them
started leaving the table one by one as one finished his dinner. F finished
first and left the table and walked in the same direction he is facing to reach
point M which is two meters away from his initial position. After F, E got up
from the table, turned 180 degrees and walked four meters to reach point P
which is five meters to the south of point Q. Point Q is in the east of point M.

Now, A left the table and walks for 7m in west reach point L. After, A H
leaves the table and walks 4m in the direction opposite to in which he was
facing initially and reach point S which is 3m north of point L. So, A walks in
the west direction.

51. (c); 52. (e); 53. (b);


54. (d); 55. (a);
Adda247 Publications For any detail, mail us at
40
Publications@adda247.com
Cracker Book for Bank (IBPS | SBI | RRB PO | Clerk) Mains Exams

Solutions (56-60):
There are three floors in between the one who ordered Masala Vada and Q
and both of them lives on an odd numbered floor. The one who works on
lowermost floor order Pav Bhaji. The one who works on 4th floor ordered
Pizza but did not drink Tea. Only the persons who works on even number
floor drinks Tea. Now, it is clear that the persons live on 2nd, 6th and 8th floor
drinks Tea. So, there can be three possible cases-

Case:1-
Floor Person Drinks Snacks
8 Tea
7 Masala Vada
6 Tea
5
4 Tea(×) Pizza
3 Q
2 Tea
1 Pav Bhaji

Case:2-
Floor Person Drinks Snacks
8 Tea
7 Q
6 Tea
5
4 Tea(×) Pizza
3 Masala Vada
2 Tea
1 Pav Bhaji

Adda247 Publications For any detail, mail us at


41
Publications@adda247.com
Cracker Book for Bank (IBPS | SBI | RRB PO | Clerk) Mains Exams

Case:3-
Floor Person Drinks Snacks
8 Tea
7
6 Tea
5 Masala Vada
4 Tea(×) Pizza
3
2 Tea
1 Q Pav Bhaji

Now, P neither drink Tea nor Coffee and works on any floor below Q. So,
Case: 3 will be eliminated as Q works on floor no.1. M drinks Soup and both
the persons who work on a floor which is immediately above and
immediately below him drinks Tea. From this Case:2 will be eliminated.
Now, L lives immediate above O. Neither O nor L ordered Pizza. So, only
possible condition will be as shown below-

Case:1-
Floor Person Drinks Snacks
8 Tea
7 M Soup Masala Vada
6 L Tea
5 O
4 Tea(×) Pizza
3 Q
2 Tea
1 P Pav Bhaji

Now, Samosa is ordered with Coffee but not order by Q. So, it will be the
person who works on 5th floor who ordered Samosa. Both the persons, who
drinks Coffee, works on two consecutive floors. N ordered Sweet corn and
lives below the one who ordered Samosa. So, N works on 2nd floor. S drinks
Tea so S works on 8th floor and remaining R will work on 4th floor. The one
who ordered Pakoda also drinks Soup.
Adda247 Publications For any detail, mail us at
42
Publications@adda247.com
Cracker Book for Bank (IBPS | SBI | RRB PO | Clerk) Mains Exams

Floor Person Drinks Snacks


8 S Tea
7 M Soup Masala Vada
6 L Tea
5 O Coffee Samosa
4 R Coffee Pizza
3 Q Soup Pakoda
2 N Tea Sweet corn
1 P Pav Bhaji

So, rest P will drink lemon tea. Only one person lives between the one who
ordered Noodles and L. So, the final arrangement will be-----

Floor Person Drinks Snacks


8 S Tea Noodles
7 M Soup Masala Vada
6 L Tea Aloo Tikki
5 O Coffee Samosa
4 R Coffee Pizza
3 Q Soup Pakoda
2 N Tea Sweet corn
1 P Lemon Tea Pav Bhaji

56. (b); 57. (b); 58. (e);


59. (d); 60. (e);

Solutions (61-65):
B is the son-in-law of D. E is the daughter of D, and is not married to B. F is
the daughter of G. Daughter of B sits on the immediate left of G who faces E.
B has no brother. Brother-in- law of G sits second to the right of B. A is
married to E. So, the blood relation tree

Adda247 Publications For any detail, mail us at


43
Publications@adda247.com
Cracker Book for Bank (IBPS | SBI | RRB PO | Clerk) Mains Exams

Brother-in- law of G sits second to the right of B and both faces opposite
direction. No one sits on the immediate right of B.

So, the blood relation tree

There is only one vacant seat in each square. The vacant seats are not in
front of each other. Daughter of B sits on the immediate left of G, who faces
E. E does not sit in the same square with F.

D is not the immediate neighbour of G. F and A face same direction. F does


not face vacant seat. So, case 2 and Case 4 is eliminated.

Adda247 Publications For any detail, mail us at


44
Publications@adda247.com
Cracker Book for Bank (IBPS | SBI | RRB PO | Clerk) Mains Exams

E and F faces same direction. B and F does not face each other. So, case 3
will be eliminated.

61. (a); 62. (e); 63. (d);


64. (e); 65. (e);

Solutions (66-70):
All the members in both the circles live exactly according to the sequence of
the floors they are living in. The one from Philippines sits third to the left of
the one who is from Vietnam. Two floors are there between the ones from
Vietnam and Philippines. The one from Australia sits immediate left to the
one from New Zealand, who lives above the one from china and there are
three floors between the one from New Zealand and the one from China.
The one from Korea sits immediate left to the from Australia. The one from
Vietnam lives above no one. Three persons sit between the ones from
Philippines and Indonesia, (when counted from the left of the one from
Philippines). The one from Indonesia lives just below the one from
Cambodia.

Adda247 Publications For any detail, mail us at


45
Publications@adda247.com
Cracker Book for Bank (IBPS | SBI | RRB PO | Clerk) Mains Exams

Floor ASEAN members FTA


10
9 Cambodia
8 Indonesia
7
6
5
4 Philippines
3
2
1 Vietnam

The one from India does not sit next to Korea and the host country
representative. None of the FTA lives on 10th and 9th floor. No one lives with
the representatives of Philippines and Myanmar. The one from the Laos was
allotted an odd numbered floor and lives just above the one from India.
More than 1 person lives between Malaysia and Vietnam. The one from
Malaysia and India live together on same floor. So, the one from Laos lives
on 7th floor and the one from India lives on 6th floor with the one from
Malaysia. ‘

Adda247 Publications For any detail, mail us at


46
Publications@adda247.com
Cracker Book for Bank (IBPS | SBI | RRB PO | Clerk) Mains Exams

Floor ASEAN FTA


members
10
9 Cambodia
8 Indonesia
7 Laos
6 Malaysia India
5
4 Philippines
3
2
1 Vietnam

Two floors are between the representatives of India and China, Since no
FTA lives on 9 & 10th floor. The one from china lives on 3rd floor. The one
from New Zealand, who lives above the one from china and there are three
floors between the one from New Zealand and the one from China The host
and Thailand live on even numbered floor together. No one lives with the
representatives of Philippines and Myanmar. More than two persons sits
between Myanmar and the one from Brunei(when counted from the left of
the one who is from Myanmar). . The one from Myanmar does not lives on
the top Floor. The one from Singapore lives on a doubly occupied floor.
The final arrangement is:

Adda247 Publications For any detail, mail us at


47
Publications@adda247.com
Cracker Book for Bank (IBPS | SBI | RRB PO | Clerk) Mains Exams

Floor ASEAN members FTA


10 Brunei
9 Cambodia
8 Indonesia Australia
7 Laos New Zealand
6 Malaysia India
5 Myanmar
4 Philippines
3 Singapore China
2 Thailand Japan
1 Vietnam Korea

66. (c); 67. (b); 68. (b);


69. (c); 70. (c);

 Key Points to Remember for New Pattern Questions

➢ Puzzles are not a hard nut to crack. These are given in


exams basically to judge your analytical, logical and
mental ability amalgamate with your intelligence.
➢ Try to solve this as a game that you are playing like a
Jigsaw puzzle. You just have to place all the objects on a
right place and you will find all your answers.
➢ There is modified concept of floor-based arrangement in
which flats are also incorporated in it. This will become
simple if we start following the instructions of puzzle for
the alignment of flats.
➢ The time-based puzzle which is also a new concept consist
of the consecutive timing which has no gap, is totally a
computation based puzzle. You have to adjust the timing
for which you have to start with the defined timing slot.
Adda247 Publications For any detail, mail us at
48
Publications@adda247.com
Cracker Book for Bank (IBPS | SBI | RRB PO | Clerk) Mains Exams

➢ The date and month puzzle is commonly seen topic now a


days. It required basic attention on even/odd date and
also number of days present in the given months. So, Just
go with these points and you will get your answer.
➢ Mathematical condition is a hot topic for puzzles in recent
exams. Most of the puzzles consist of these conditions so
you have to be strong enough in your mathematical
calculation.
➢ Box arrangement with different concepts as uncertain
number of boxes, different number of an item consist by
the box or the linear arrangement of boxes are recently
seen puzzles. These require a bit focus on the number of
boxes above or below the given one.

Adda247 Publications For any detail, mail us at


49
Publications@adda247.com
Cracker Book for Bank (IBPS | SBI | RRB PO | Clerk) Mains Exams

Adda247 Publications For any detail, mail us at


1
Publications@adda247.com
Cracker Book for Bank (IBPS | SBI | RRB PO | Clerk) Mains Exams

Chapter

3 Input-Output

BEST APPROACH TO SOLVE THE QUESTIONS

Recently many changes were seen in the machine Input questions asked in
various mains exams i.e. SBI Po/ IBPS Po/RRB Clerk/SBI Clerk/Insurance
etc. So, below there are some examples based on the new pattern Machine
Input asked in SBI Po Mains and IBPS Po mains 2017 which have been
explained with details solution to help you understand the approach of
these type of questions.

Example 1:
A word and number arrangement machine when given an input line of
words and numbers rearranges them following a particular rule in each
step. The following is an illustration of input and rearrangement. (IBPS Po
Mains 2017)

Input- njes glfu 53 18 31 44 wird dect


Step I: yktf 106 njes glfu 18 31 44 dect
Step II: plgu 88 yktf 106 glfu 18 31 dect
Step III: inhw 62 plgu 88 yktf 106 18 dect
Step IV: fgev 36 inhw 62 plgu 88 yktf 106

In this new pattern machine input question only one word and one number
is arranged in each step.

Let us understand the logic behind it:


We can see that in each step the words and the numbers both are arranged
from the left end.
For words- The word which has highest place value according to
alphabetical series is arranged first from left end and each letter of each
word is replaced by its second succeeding letter according to alphabetical
series and same will be followed in each step.
For numbers- Numbers are arranged in descending order from left end in
such a way that each number is multiplied by two.
Adda247 Publications For any detail, mail us at
2
Publications@adda247.com
Cracker Book for Bank (IBPS | SBI | RRB PO | Clerk) Mains Exams

Example 2:
Study the following diagram and convert it into other diagrams by
implementing the instructions which is given in each step to get next step.
(SBI Po Mains 2017)

Interchange the Alphabets to get step 1 as arrows mention in the above


figure.

For Step-2:
(i) If the alphabets contain one consonant and one vowel and the number
with them is greater than 3, then subtract 3 from the given number.
(ii) If the alphabets are two consonant and the number with them is
greater than 5, then change the letters with the previous letter in
alphabetical series.

Adda247 Publications For any detail, mail us at


3
Publications@adda247.com
Cracker Book for Bank (IBPS | SBI | RRB PO | Clerk) Mains Exams

For Step-3: step 3 is coded in some special pattern.

As, In the above question nothing has mentioned about the step 3. So, we
must understand the logic behind this.

For Step 3:
The elements arranged in the first and third column in such way that the
element in third row placed in first row and first row element is placed in
second row and second row element is placed in third row. The alphabet of
first row second column is replaced with the alphabet which is three places
after the alphabet in alphabetical series. The alphabet of third row second
column is replaced with the alphabet which is five places after the alphabet
in alphabetical series.

Adda247 Publications For any detail, mail us at


4
Publications@adda247.com
Cracker Book for Bank (IBPS | SBI | RRB PO | Clerk) Mains Exams

Practice Exercise Based on new Pattern

Directions (1-5): Study the following diagram and convert it into other
diagrams by implementing the instructions which is given in each step to
get next step.

For Step-1:
Add the number of the last alphabet which is in the alphabetical series with
the element.

For Step-2:
(i) If the alphabets contain one consonant and one vowel -
If the number with them is a whole square, then replace vowel with the
opposite letter in alphabetical series and add 2 in the given number.
If the number with them is not a whole square, then replace vowel with
the second previous letter in the alphabetical series.
(ii) If the alphabets are two consonants –
If the number with them is a whole square, then replace alphabets with
the opposite letter in alphabetical series and subtract 2 in the given
number.
If the number with them is not a whole square, then replace alphabets
with the second next letter in the alphabetical series.

Adda247 Publications For any detail, mail us at


5
Publications@adda247.com
Cracker Book for Bank (IBPS | SBI | RRB PO | Clerk) Mains Exams

For Step-3:
step 3 is coded in some special pattern.

As per the rules followed in the above step, find out the appropriate steps
for the given input.
And answer the following questions.

1. Which of the following element replaces the alphabets SN14 from step-
2 to in step 3?
(a) MJ10 (b) KD20 (c)T O9
(d) XW2 (e) None of these

2. Which of the following element replaces HY25 from step-1 to in step 2?


(a)FZ3 (b)SB23 (c)RT18
(d)GZ26 (e)None of these
Adda247 Publications For any detail, mail us at
6
Publications@adda247.com
Cracker Book for Bank (IBPS | SBI | RRB PO | Clerk) Mains Exams

3. Which of the following element is placed in 2nd column 3rd row in step
3?
(a) DM11 (b) XW2 (c) TC18
(d) RT18 (e) None of these
4. Which of the following is diagonally opposite to the element FZ3 in step
3?
(a) GZ26 (b)JA25 (c)TC18
(d)MJ10 (e)None of these

5. Which of the following replaces CD4 from step-1 to in step 2?


(a) XW2 (b) XN14 (c) MJ10
(d) GZ26 (e) None of these
Directions (6-10): Study the following information to answer the given
questions:
A word and number arrangement machine when given an input line of
words, rearranges them following a particular rule. The following is an
illustration of input and rearrangement.
Input: code word right you thing like
Step I: youv code word right thing like 211
Step II: worde youv code right thing like 211 44
Step III: thingh worde youv code right like 211 44 77
Step IV: rightu thingh worde youv code like 211 44 77 200
Step V: likef rightu thingh worde youv code 211 44 77 200 55
Step VI: codef likef rightu thingh worde youv 211 44 77 200 55 55
Step VI is the last step of the rearrangement.
As per the rules followed in the above steps, find out in each of the
following questions the appropriate steps for the given input.
Input: quite similar dull go test vice

6. How many steps would be needed to complete the arrangement?


(a) X (b) VIII (c) IX
(d) VI (e) None of these

7. What will the difference of the numbers which is third from the right
end in step III and 1st from the right end in step IV?
(a) 12 (b) 0 (c) 18
(d) 11 (e) None of these
Adda247 Publications For any detail, mail us at
7
Publications@adda247.com
Cracker Book for Bank (IBPS | SBI | RRB PO | Clerk) Mains Exams

8. Which of the following would be the cube root of the sum of the
numbers which is 2nd from right end in step IV and 2nd from right end in
Step VI?
(a) 9 (b) 8 (c) 13
(d) 7 (e) None of these

9. Which of the following element will be 6th to the left of 3rd from the
right end in step V?
(a) quitef (b) dull (c) similars
(d) 200 (e) None of these

10. In Step IV, which of the following word/number would be on 4th


position (from the left end)?
(a) vicef (b) 188 (c) 55
(d) quite (e) None of these

Directions (11-15): Study the following information carefully and answer


the given questions.
When a word and number arrangement machine is given an input line of
words and numbers, it arranges them following a particular rule. The
following is an illustration of Input and rearrangement.
Input: camps 59 to 91 concentration 48 including 85 Auschwitz 35.
Step I: Auschwitz 35 camps 59 to 91 concentration 48 including 85.
Step II: Auschwitz 35 camps 91 59 to concentration 48 including 85.
Step III: Auschwitz 35 camps 91 to 48 59 concentration including 85.
Step IV: Auschwitz 35 camps 91 to 48 concentration 85 59 including.
Step V: Auschwitz 35 camps 91 to 48 concentration 85 including 59.

And step V is the last step of the above input. As per the rules followed in
the above step, find out the appropriate step for the given output.
Input: 84 Warsaw has 72 14 already 49 come 83 under sharp 37 from
21.

11. Which element is exactly between the elements which are fourth from
left end and third from right end in step V?
(a) Sharp (b) 72 (c) Under
(d) 37 (e) None of these
Adda247 Publications For any detail, mail us at
8
Publications@adda247.com
Cracker Book for Bank (IBPS | SBI | RRB PO | Clerk) Mains Exams

12. Which step number would be the following output?


“already 21 Warsaw 14 has 72 under 37 sharp 83 from 84 come
49.”
(a) Step VII (b) Step IV (c) Step V
(d) Step VI (e) Step III

13. What is the position of ‘Under’ in the step IV?


(a) Fourth from the right end (b) Sixth from the left end
(c) None of these (d) Fourth from the left end
(e) Eighth from the right end

14. In step VI, what is the sum of 4th element from the left end and 7th
element from the right end?
(a) 56 (b) 51 (c) 48
(d) 54 (e) None of these
15. In which of the following step ’37 84 49’ found in the same order?
(a) Step II (b) Step VII (c) Step IV
(d) Step VI (e) None of these

Direction (16-20): A word and number arrangement machine when given


an input line of words and numbers rearranges them following a particular
rule in each step. The following is an illustration of input and
rearrangement.
Input- utys hltk 37 21 19 48 gmrd fexn
Step I: gdwo 100 utys hltk 37 21 48 gmrd
Step II: hlqe 100 gdwo 100 utys hltk 21 48
Step III: iksl 9 hlqe 100 gdwo 100 utys 48
Step IV: vsxt 144 iksl 9 hlqe 100 gdwo 100
Answer the following questions based on the following input: -
Input- qewb 23 plsf 17 15 cjrx 36 ytvd

16. What will be the difference of the number which is second from the left
end of step I and which is third from the right end of step III?
(a) 52 (b) 28 (c) 0
(d)91 (e)None of these

Adda247 Publications For any detail, mail us at


9
Publications@adda247.com
Cracker Book for Bank (IBPS | SBI | RRB PO | Clerk) Mains Exams

17. What will be the third step of the given input?


(a) Step III- rdvc 36 qkrg 25 diqy 64 32 ytvd
(b) Step III- qkrg 36 25 rdvc diqy 64 36 ytvd
(c) Step III- rdvc 36 qkrg 25 diqy 64 36 ytvd
(d) Step III- qkrg 36 rdvc 25 diqy 64 ytvd 36
(e) None of these

18. Which of the following word/number will be third to the left of sixth
from the left end in step II?
(a) 25 (b) rdvc (c) 64
(d) diqy (e)None of these

19. What will be the twice of the difference of sixth number from the left
end of step I and fifth number from right end of step IV?
(a) 34 (b) 42 (c) 28
(d) 44 (e) None of these

20. In step IV, what is the sum of 2nd element from the left end and 5th
element from the right end?
(a) 107 (b) 89 (c)117
(d) 106 (e) None of these

Directions (21-25): Study the following information carefully and answer


the given questions.
When a word and number arrangement machine is given an input line of
words and numbers, it arranges them following a particular rule. The
following is an illustration of Input and rearrangement.
Input: 18 quora 26 diagnose 89 maths 27 eat
Step I: fkcipqug quora 26 89 maths 27 eat 18
Step II: gcv fkcipqug quora 26 89 maths 18 27
Step III: ocvju gcv fkcipqug quora 89 18 27 26
Step IV: swqtc ocvju fkcipqug 18 27 26 89
And step IV is the last step of the above input. As per the rules followed in
the above step, find out the appropriate step for the given output.
INPUT: Queen 79 apple 38 vowel 19 jungle 26

Adda247 Publications For any detail, mail us at


10
Publications@adda247.com
Cracker Book for Bank (IBPS | SBI | RRB PO | Clerk) Mains Exams

21. Which element is exactly between the elements which are fourth from
left end and third from right end in step IV?
(a) 79 (b) 26 (c) 38
(d) 19 (e) None of these

22. Which step number would be the following output?


“swggp lwping crrng 79 vowel 19 38 26”
(a) Step III (b) Step IV (c) Step I
(d) Step II (e) None of the Above

23. What is the position of ‘vowel’ in the step II?


(a) Seventh from the right end (b) Sixth from the left end
(c) Fourth from the right end (d) Fourth from the left end
(e) None of these

24. In step II, what is the sum of 7th element and 6th element from left end?
(a) 56 (b) 54 (c) 48
(d) 45 (e) None of these

25. Which of the following would be at the third position from the left end
in step IV?
(a) swggp (b) lwping (c) crrng
(d) 19 (e) None of these

Direction (26-30): A word and number arrangement machine when given


an input line of words and numbers rearranges them following a particular
rule in each step. The following is an illustration of input and
rearrangement.
Input- juhs elok 65 17 32 15 pnir veox
Step I: ednc 6511 juhs elok 17 32 15 pnir
Step II: kmhi 325 ednc 6511 juhs elok 17 15
Step III: qtsh 178 kmhi 325 ednc 6511 elok 15
Step IV: donp 156 qtsh 178 kmhi 325 ednc 6511
Answer the following questions based on the following input: -
Input- wder 76 yqok 33 54 isxv 13 zcjf
Adda247 Publications For any detail, mail us at
11
Publications@adda247.com
Cracker Book for Bank (IBPS | SBI | RRB PO | Clerk) Mains Exams

26. What will be the difference of the number which is second from the left
end of step II and which is second from the left end of step III?
(a) 252 (b) 218 (c) 213
(d)191 (e)None of these
27. What will be the step II of the given input?
(a) Step II- bjnp 549 axqu 76 wder 33 isxv 13
(b) Step II- bjnp 549 axqu 7613 wder 33 isxv 13
(c) Step II- 549 bjnp axqu 7613 wder 33 isxv 13
(d) Step II- bjnp 546 axqu 7613 wder 33 isxv 13
(e) None of these
28. Which of the following word/number will be third to the left of sixth
from the left end in step III?
(a) 134 (b) bjnp (c) 336
(d) dwdi (e) None of these

29. What will be the twice of the difference of second number from the left
end of step IV and fifth number from right end of step IV?
(a) 384 (b) 424 (c) 828
(d) 404 (e) None of these
30. In which of the following step ’7613 isxv 13’ found in the same order?
(a) Step II (b) Step II (c) Step IV
(d) Step III (e) None of these
Directions (31-35): Study the following information to answer the given
questions.
A word arrangement machine when given an input line of words rearranges
them by following different operation in each step. The following is an
illustration of input and rearrangement.
Input: Duplicate YouTube Google Jaboong Myntra Strengths
Step I: 13*4 11*4 9%3 10*3 7%1 10*1
Step II: 3 4 1
Step III: es ea eg
Step IV: 6
Step IV is the last step of the arrangement of the above input as the
intended arrangement is obtained.
Now, answer the questions based on the following input.
Input: chamber satellite international domestic ambassador conference

Adda247 Publications For any detail, mail us at


12
Publications@adda247.com
Cracker Book for Bank (IBPS | SBI | RRB PO | Clerk) Mains Exams

31. Which of the following will be the sum of all the digits in step I?
(a) 58 (b) 43 (c) 81
(d) 62 (e) None of these

32. Which of the following is the final step?


(a) 7 (b) 9 (c) 8
(d) 6 (e) None of these

33. If there was one more word “transfer” placed right to conference in the
input then what will be step I ?
(a) 9*2 13*4 18*5 11%3 14%4 14%4 10%2
(b) 9*2 13*4 18*5 11%3 14%3 14%4 10*5
(c) 9*2 13*4 18*5 11%2 14%4 14%4 10%2
(d) 9*2 13*4 19*6 11%3 14%4 14%4 10%2
(e) None of these

34. Which of the following represents the sum of all the digits in step II?
(a) 14 (b) 10 (c) 2
(d) 13 (e) None of these

35. What is the total sum of the numbers before and after the symbol in
step I?
(a) 102 (b) 103 (c) 100
(d) 105 (e) None of these

Directions (36-40): Study the following information carefully and answer


the given questions.
A word and number arrangement machine when given an input line of
words and numbers rearranges them following a particular rule in each
step. The following is an illustration of input and rearrangement.
Input- Draft 95 alliance 67 ideological 58 complex 62
Step-I: revlmlhrdzm draft 95 alliance 58 complex 62 67
Step-II: revlmlhrdzm eszgu alliance 58 complex 62 67 95
Step-III: revlmlhrdzm eszgu dlnqmvy alliance 62 67 95 58
Step-IV: revlmlhrdzm eszgu dlnqmvy zmmrzodv 67 95 58 62
Input- tactics 89 constrained 73 macro 56 hardline 42

Adda247 Publications For any detail, mail us at


13
Publications@adda247.com
Cracker Book for Bank (IBPS | SBI | RRB PO | Clerk) Mains Exams

36. How many steps are required to rearrange the given input?
(a)III (b)VI (c)V
(d)IV (e)None of these

37. What will be the second step of the given input?


(a) Step II- uzdurdt nzdsl constrained 56 hardline 73 42 89
(b) Step II- uzdurdt nzdsl constrained 42 hardline 56 73 89
(c) Step II- uzdurdt nzdsl constrained 56 hardline 42 73 89
(d) Step II- uzdurdt nzdsl constrained 73 56 hardline 42 89
(e) None of these

38. Which of the following word/number will be at fifth position from right
end in III step?
(a) 73 (b) nzdsl (c) 56
(d) constrained (e)None of these

39. What is the addition of 2nd and 5th element from the right end in step II?
(a)130 (b)129 (c)128
(d)121 (e)None of these

40. Which step gives following output?


“uzdurdt nzdsl izsemrov constrained 56 73 89 42”
(a) Step-I (b) Step-II (c) Step-III
(d) Step-IV (e)None of these

Adda247 Publications For any detail, mail us at


14
Publications@adda247.com
Cracker Book for Bank (IBPS | SBI | RRB PO | Clerk) Mains Exams

Solutions

Directions (1-5):

The alphabets are arranged according to the directions are given for step 1.

The alphabets are arranged according to the directions are given for step 2.

For Step-3:
(i) The elements arranged in a serial way in first and third row as element
of third column is placed in first column, element of first column is
placed in second column and element of second column is placed in
third column.
(ii) In the second row the elements are arranged in the given arrangement-

Adda247 Publications For any detail, mail us at


15
Publications@adda247.com
Cracker Book for Bank (IBPS | SBI | RRB PO | Clerk) Mains Exams

The Consonant are replaced with the letter in the alphabetical series with
the next letter of the given letter in alphabetical series. There is a
subtraction of 5 from the given number.

1. (c) 2. (b) 3. (a)


4. (a) 5. (a)

Directions (6-10):
Logic: - There are six words in the input. In each of the step the words are
arranged in reverse alphabetical order such that next letter of the last
letter of that word is also placed with it. Also, in each step a number is
placed at the right end which is the place value of the last letter of the
word which is arranged, with the last digit of that number (place value)
is repeated once in it. For example, Vice (place value of e = 5) so 55 is
placed at the rightmost end.

Input: quite similar dull go test vice


Step I: vicef quite similar dull go test 55
Step II: testu vicef quite similar dull go 55 200
Step III: similars testu vicef quite dull go 55 200 188
Step IV: quitef similars testu vicef dull go 55 200 188 55
Step V: gop quitef similars testu vicef dull 55 200 188 55 155
Step VI: dullm gop quitef similars testu vicef 55 200 188 55 155 122
6. (d); 7. (b); 8. (d);
9. (c); 10. (a);
Adda247 Publications For any detail, mail us at
16
Publications@adda247.com
Cracker Book for Bank (IBPS | SBI | RRB PO | Clerk) Mains Exams

Direction (11-15):
Students let us understand the Logic behind this Question and let’s
understand how to solve it. When we see each step, then we can find that
The machine rearranges one word and one number in each step
simultaneously, words and numbers both are arranged from left end.

(i) Words are arranged in decreasing alphabetical order according to the


last letter of the word and same will be followed in further steps.
(ii) Numbers are arranged in increasing order, according to addition of
their digits. (For example: 19 = 9+1=10).

INPUT: 84 Warsaw has 72 14 already 49 come 83 under sharp 37 from


21.
Step I: already 21 84 Warsaw has 72 14 49 come 83 under sharp 37
from.
Step II: already 21 Warsaw 14 84 has 72 49 come 83 under sharp 37
from.
Step III: already 21 Warsaw 14 has 72 84 49 come 83 under sharp 37
from.
Step IV: already 21 Warsaw 14 has 72 under 37 84 49 come 83 sharp from.
Step V: already 21 Warsaw 14 has 72 under 37 sharp 83 84 49 come from.
Step VI: already 21 Warsaw 14 has 72 under 37 sharp 83 from 84 49 come.
Step VII: already 21 Warsaw 14 has 72 under 37 sharp 83 from 84 come 49.

11. (d); 12. (a); 13. (e);


14. (b); 15. (c);

Direction (16-20):
In the given Input-Output one word and one number is being arranged
simultaneously in each step.
For Words- In first step- The word which comes first according to
alphabetical series will be arranged from left end such that its first and last
letter of the word will be replaced with its next letter and rest of the letters
are replaced with their preceding letter. Then the next word according to
alphabetical order will be arranged in second step from the left end and so
on………..

Adda247 Publications For any detail, mail us at


17
Publications@adda247.com
Cracker Book for Bank (IBPS | SBI | RRB PO | Clerk) Mains Exams

For numbers- All the numbers are arranged with the one word
simultaneously in each step, such that first the prime numbers are arranged
in ascending order and then non-prime numbers will be arranged. Each of
the number will be arranged by following logic as number will be replaced
with the square of the addition of the digits of that number after
arrangement.
Input- qewb 23 plsf 17 15 cjrx 36 ytvd
Step I: diqy 64 qewb 23 plsf 15 36 ytvd
Step II: qkrg 25 diqy 64 qewb 15 36 ytvd
Step III: rdvc 36 qkrg 25 diqy 64 36 ytvd
Step IV: zsue 81 rdvc 36 qkrg 25 diqy 64

16. (c); 17. (c); 18. (d);


19. (b); 20. (c);

Direction (21-25):
Students let us understand the Logic behind this Question and let’s
understand how to solve it. When we see each step, then we can find that
The machine rearranges one word and one number in each step
simultaneously, words are arranged from left end and numbers are
arranged from right end.

(i) Words are arranged in increasing alphabetical order with each letter of
word is replaced by its 2nd succeeding letter according to alphabetical
series and same will be followed in further steps.
(ii) Numbers are arranged in decreasing order, according to difference of
their digits. (For example: 19 = 9-1=8).

INPUT: Queen 79 apple 38 vowel 19 jungle 26


Step I: crrng queen 79 38 vowel jungle 26 19
Step II: lwping crrng queen 79 vowel 26 19 38
Step III: swggp lwping crrng 79 vowel 19 38 26
Step IV: xqygn swggp lwping crrng 19 38 26 79

21. (d); 22. (a); 23. (c);


24. (d); 25. (b);
Adda247 Publications For any detail, mail us at
18
Publications@adda247.com
Cracker Book for Bank (IBPS | SBI | RRB PO | Clerk) Mains Exams

Direction (26-30):
For words- All the words are arranged in descending order in alphabetical
series. The logic of the arrangement of words is that the word which is last
according to the alphabetical series is arranged first from the left end in
step I, then the word which is second last according to the alphabetical
order is arranged from the left end in step II and so on….Also in the words
each of the consonant is replaced by its opposite letter and each vowel is
replaced by its previous letter while arrangement.
For Numbers-Only One number is arranged along with one word in each
step. The logic of the arrangement of numbers is that the numbers are
arranged in decreasing order as the highest number is arranged in step I
with the word then second highest number is arranged in step II with the
next word and so on... Each number is arranged along with one word in
such a way that the sum of the digits of that number is placed next to it.
Input- wder 76 yqok 33 54 isxv 13 zcjf
Step I: axqu 7613 wder yqok 33 54 isxv 13
Step II: bjnp 549 axqu 7613 wder 33 isxv 13
Step III: dwdi 336 bjnp 549 axqu 7613 isxv 13
Step IV: hhce 134 dwdi 336 bjnp 549 axqu 7613

26. (c); 27. (b); 28. (b);


29. (d); 30. (d);

Directions (31-35):
In the given machine input, different operation are performed in different
steps:
In step I-

(a) the number before the symbol- Total number of letters in the word +
number of vowels in the word.
(b) the number after the symbol- Total number of vowels in the word.
(c) Symbol- if the resultant of number in {(a) +(b)} is even then % and if it
is odd then ‘*’.
Adda247 Publications For any detail, mail us at
19
Publications@adda247.com
Cracker Book for Bank (IBPS | SBI | RRB PO | Clerk) Mains Exams

In step II. The difference between the first numbers of the words according
to the following pattern.

Step III. The last letters of the words combinedly used in the step II.

Step IV. Adding the place value of all the letters obtained in step III and
then adding the digits obtained after the sum .
Es Ea Eg ---- 5+19+5+1+5+7=42=6

INPUT: chamber satellite international domestic ambassador conference


Step I: 9*2 13*4 19*6 11%3 14%4 14%4
Step II: 5 1 8
Step III: re er lc
Step IV: 7
31. (a); 32. (a); 33. (d);
34. (a); 35. (b);

Directions (36-40):
In this new pattern Input-Output question only one word and one number
is arranged in each step. Let us understand the logic behind it- In each step
the words are arranged from the left end such that in 1st step word are
arranged in extreme end than 2nd word are arranged in 2nd left end and so
on… while the numbers are arranged from the right end.

Adda247 Publications For any detail, mail us at


20
Publications@adda247.com
Cracker Book for Bank (IBPS | SBI | RRB PO | Clerk) Mains Exams

For words- words are arranged according to reverse alphabetical order


(English dictionary) from left end with each vowel of word is replaced by its
opposite letter according to alphabetical series while each consonant of
word is replaced by its succeeding letter according to alphabetical series
and same will be followed in further steps.
For numbers- Numbers are arranged in ascending order from right end in
such a way that first all odd numbers are arranged after that all even
numbers are arranged.
Input- tactics 89 constrained 73 macro 56 hardline 42
Step-I: uzdurdt 89 constrained macro 56 hardline 42 73
Step-II: uzdurdt nzdsl constrained 56 hardline 42 73 89
Step-III: uzdurdt nzdsl izsemrov constrained 56 73 89 42
Step-IV: uzdurdt nzdsl izsemrov dlotuszrove 73 89 42 56
36. (d); 37. (c); 38. (d);
39. (b); 40. (c);



Key Points to Remember for New Pattern Questions
In your daily life you are required to put some efforts, then some
processing has to be done so that output will get received.
Similarly, In Input-Output, Input that has been given, processed
by a particular logic within some steps and then finally you will
get the output.
➢ The machine input-output is keep on changing recently, as it
includes the logic related to place values of the letters of the
word or it may some numbers and there will be some
mathematical operations applied on it.
➢ In some recent exam the new pattern that has been seen
includes a diagram having some elements. Some operations are
applied on it in a stepwise manner and then the diagram
changed accordingly.
➢ In some of the input-output questions, now a days there can be
seen some different logics which includes omission and addition
of letters in the words.

Adda247 Publications For any detail, mail us at


21
Publications@adda247.com
Cracker Book for Bank (IBPS | SBI | RRB PO | Clerk) Mains Exams

1 Adda247 Publications For any detail, mail us at


Publications@adda247.com
Cracker Book for Bank (IBPS | SBI | RRB PO | Clerk) Mains Exams

Chapter

4 Data Sufficiency

BEST APPROACH TO SOLVE THE QUESTIONS

Data Sufficiency, a topic that has emerged as mostly based on seating and
puzzle arrangement recently. It requires attention while solving as each and
every statement contains some direct and indirect information in it. So, you
are required to solve every statement individually and also by combining it
with other one.

Example-1:
Each of the questions below consists of a questions and two statements
numbered I and II given below it. You have to decide whether the data
provided in the statements are sufficient to answer the question. Read both
the statements and find;
(IBPS PO Mains 2017)
(a) if the data in statement I alone are sufficient to answer the question,
while the data in statement II alone are not sufficient to answer the
question.
(b) if the data in statement II alone are sufficient to answer the question,
while the data in statement I alone are not sufficient to answer the
question.
(c) if the data either in statement I alone or in statement II alone are
sufficient to answer the question.
(d) if the data even in both statements I and II together are not sufficient to
answer the question.
(e) if the data in both statements I and II together are necessary to answer
the question.

2 Adda247 Publications For any detail, mail us at


Publications@adda247.com
Cracker Book for Bank (IBPS | SBI | RRB PO | Clerk) Mains Exams

1. Five friends Q, R, S, T, V are married on different dates 12, 19, 21, 28, 31
but not necessarily in the same order in the month of either January or
September. T is married on which of the following date?
(I) Both T and Q are not married on even date of the month. T is
married before Q. Two persons are married in between T and V.
(II) No body is married after R. S is married on a date in between T and
Q. S is married on an even date. More than three persons are married
in the same month.

2. Six boxes A, B, C, D, E, F of different colours are placed one above


another. Also each box have different number of toffees. Only two boxes
are placed in between B and Green box. No box is placed above B. Box D
is placed immediately above Blue box. Only Red box is placed in between
Green box and A. Only one box is placed between Red and Blue box. Only
one box is placed in between D and E. Only one box is placed between
Orange box and C. How many number of toffees does Blue box have?
(I) Box E has more number of toffees than 8 while box C has more
number of toffees than 20. Box D has 21 toffees. The box which has
lowest and 2nd lowest number of toffees has 10 and 12 toffees
respectively. Box A, C, D and F has odd number of toffees.
(II) A has more number of toffees than B but not more than D. The
difference in the number of toffees in box F and E is 7. The box
which has highest number of toffees has 8 more toffees than box F.
Total number of toffees in box B and A is 31.

Explanation (The Approach)


1. (e); From statement II- S can be married on either 28 January or 12
September.
From statement I and II- When S is married on 28 January

When S is married on 12 September

3 Adda247 Publications For any detail, mail us at


Publications@adda247.com
Cracker Book for Bank (IBPS | SBI | RRB PO | Clerk) Mains Exams

But it is given in statement II that More than three persons are


married in the same month. So, we will get that S is married on 12
September and final arrangement from both statement I and II-

2. (e);
Boxes Colours
B Orange
D -
C Blue
E Green
F Red
A -
So, it is clear that, Box C is of Blue color.
From Statement I-

From Statement II-


Total Number of toffees in box B+A= 31
Difference in Number of toffees in box F-E=7
Number of toffees in box F+8= Box contains highest number of
toffees
Now combining both Statement I and II-

So, Box C of Blue color contain 25 toffees.

Example-2:

Study the following information and answer the questions given below:
(IBPS Clerk Mains 2017)
1. Four friends are sitting around a circular table named M, N, O and P. Are
they facing to the center of table? If-
I. N is sitting second to the right of P. P is facing centre. O is sitting
immediate right of N and P.
4 Adda247 Publications For any detail, mail us at
Publications@adda247.com
Cracker Book for Bank (IBPS | SBI | RRB PO | Clerk) Mains Exams

II. M is sitting immediate left of N. O is not sitting immediate left of M. O


is sitting immediate right of P.
III. P is sitting immediate of M and O. N is sitting immediate left of M. O
is sitting immediate left of P.
(a) If the data in statement I and II are sufficient to answer the question,
while the data in statement III are not sufficient to answer the
question.
(b) If the data in statement II and III are sufficient to answer the
question, while the data in statement I is not sufficient to answer the
question.
(c) If the data in statement I alone or in the statement II alone or in the
statement III alone is sufficient to answer the question.
(d) If the data in all the statement I, II and III are necessary to answer
the question.
(e) If the data in statement I and III are sufficient to answer the
question, while the data in statement II is not sufficient to answer
the question.

2. Six people A, B, C, D, E and F are living in six floor building each live in
one of apartment of building. In building there are from I to VI floor.
Ground floor is given number I, then II and so on. Who is living on
Ground floor?
I. In which floor C and B are living there is only one floor between
them. A is living in even number floor.
II. E is not living in even number floor. B is living in even number floor.
B is not living in top floor.
III. D is living in odd number floor. In which floor D and A are living,
there are two between them. E is living immediate next to C from
down.
(a) If the data in statement I and II are sufficient to answer the question,
while the data in statement III are not sufficient to answer the
question.
5 Adda247 Publications For any detail, mail us at
Publications@adda247.com
Cracker Book for Bank (IBPS | SBI | RRB PO | Clerk) Mains Exams

(b) If the data in statement I and III are sufficient to answer the
question, while the data in statement II is not sufficient to answer
the question.
(c) If the data in statement II and III are sufficient to answer the
question, while the data in statement I is not sufficient to answer the
question.
(d) If the data in all the statement I, II and III are necessary to answer
the question
(e) If the data in statement I alone or in the statement II alone or in the
statement III alone is sufficient to answer the question.

3. Who is youngest among six family members G, H, I, J, K and L? (Each


member is of different ages.)
I. G is the daughter-in-law of J, K is grandson of L, who is the father of
H.
II. G is not youngest while L is the eldest.
III. H is the father of I, who is grandson of L. J is the wife of K’s son and
I’s father
(a) If the data in statement I and II are sufficient to answer the question,
while the data in statement III are not sufficient to answer the
question.
(b) If the data in statement I alone or in the statement II alone or in the
statement III alone is sufficient to answer the question.
(c) If the data in statement I and III are sufficient to answer the
question, while the data in statement II is not sufficient to answer
the question.
(d) If the data in statement II and III are sufficient to answer the
question, while the data in statement I is not sufficient to answer the
question.
(e) If the data in all the statement I, II and III are necessary to answer
the questio

6 Adda247 Publications For any detail, mail us at


Publications@adda247.com
Cracker Book for Bank (IBPS | SBI | RRB PO | Clerk) Mains Exams

Explanation (The Approach):


1. (c); I.

II.

III.

I, II or III statement alone are sufficient to conclude that M, N, O and


P are all not facing to centre of table.
7 Adda247 Publications For any detail, mail us at
Publications@adda247.com
Cracker Book for Bank (IBPS | SBI | RRB PO | Clerk) Mains Exams

2. (c); From II and III-


6. A
5. E
4. C
3. D
2. B
1. F
It is clear that F is living on the Ground floor

3. (d);

From statements II and III, I is the youngest because G is not


youngest.

Example-3:
In each of the following questions, a question is followed by three
statements numbered I, II and III. Read all the statements to find the answer
to given question and then answer accordingly that which statement/s can
give the answer alone/together.
(SBI PO Mains 2017)
1. What is the direction of point U with respect to point X?
Statement I: Point R is 7 m to the North of point Q. Point P is 8 m to
the West of point Q. Point R is 6 m to the West of point U.
Statement II: Point B is 9 m to the North of point A. Point P is 5 m to the
North of point Z. Point Z is 4 m to the West of point A.
Statement III: Point C is 7 m to the East of point A. Point X is 2 m to the
East of point F. Point F is 3 m to the North of point C.
(a) Both I and III
(b) Both II and III
(c) All I, II and III
(d) II and either I or III
(e) Even I, II and III together are also not sufficient
8 Adda247 Publications For any detail, mail us at
Publications@adda247.com
Cracker Book for Bank (IBPS | SBI | RRB PO | Clerk) Mains Exams

2. What does the code ‘bp’ stand for in the given code language?
Statement I: In the language, ‘black white red’ is coded as ‘df dc or’
and ‘green blue grey’ is coded as ‘st hn wo’
Statement II: In the language, ‘blue pink brown’ is coded as ‘er bp hn’
and ‘pink blue white’ is coded as ‘hn or bp’
Statement III: In the language, ‘green violet orange’ is coded as ‘pa wo
kl’ and ‘yellow pink brown’ is coded as ‘bp bi er’
(a) Both II and III
(b) I and either II or III
(c) II and either I or III
(d) Both I and III
(e) All I, II and III

Explanation (The Approach):


1. (c); From all the statements

2. (c); From I and II, we get- ‘hn’ stands for ‘Blue’ So,
The code ‘bp’ stands for—Pink
From II and III, we get—
The code ‘bp’ stands for—Pink

9 Adda247 Publications For any detail, mail us at


Publications@adda247.com
Cracker Book for Bank (IBPS | SBI | RRB PO | Clerk) Mains Exams

Practice Exercise Based on new


Pattern
Directions (1): Each of the questions below, consist of a question and three
statements numbered I, II and III. You have to decide whether the data
provided in the statements are sufficient to answer the question. Read the
three statements and Give answer
(a) If the data in statement I and II together are sufficient to answer the
question, while the data in statement III are not required to answer the
question.
(b) If the data in statement I and III together are sufficient to answer the
question, while the data in statement II are not required to answer the
question.
(c) If the data in statement II and III are sufficient to answer the question,
while the data in statement I are not required to answer the question.
(d) If the data in all three statements I, II and III together are necessary to
answer the question.
(e) If the data in all the statements, I, II and III even together are not
sufficient to answer the question.

1. Six employees P, Q, R, S, T, U attend office on six different days starting


from Monday to Saturday and each of them have their office on
different floors viz. 1st, 4th, 5th, 6th, 8th, 9th. The one who has office on 1st
floor attend office on which of the following day?
(I) Only two persons attend office in between R and S. The one who
attend office on Wednesday has office on 8th floor. Q has office on
an even numbered floor. P attend office immediately before T. R
attend office before S and has office on an odd numbered floor.
(II) No one attend office after U. Only three persons attend office in
between the one who has office on 1st floor and the one who has
office on 5th floor. The one who attend office immediately before T
has office 1st floor.
(III) R has office neither on 1st nor on 5th floor. Q attend office
immediately after S. The one who attend office on Thursday does
not has office on 4th floor.
10 Adda247 Publications For any detail, mail us at
Publications@adda247.com
Cracker Book for Bank (IBPS | SBI | RRB PO | Clerk) Mains Exams

Directions (2-3): Each of the questions below consists of a question and


two statements numbered I and II given below it. You have to decide
whether the data provided in the statement are sufficient to answer the
question. Read both the statements and Give answer:
(a) If the data in statement I alone are sufficient to answer the question,
while the data in statement II alone are not sufficient to answer the
question.
(b) If the data in statement II alone are sufficient to answer the question,
while the data in statement I alone are not sufficient to answer the
question.
(c) If the data either in statement I alone or in statement II alone are
sufficient to answer the question.
(d) If the data even in both statements I and II together are not sufficient to
answer the question.
(e) If the data in both statements I and II together are necessary to answer
the question.

2. If given Output: bold 71 keen 212 tough 353 unit 634 then which of
the following will the input?
I. If step II of the given input is---
Step II: bold 71 keen 212 tough 63 35 unit
II. If Step III of the given input is---
Step III: bold 71 keen 212 tough 353 63 unit

3. Six persons A, B, C, D, E, F born in six different months of same year.


Only one person was born in one month. C was born in which of the
following month?
I. E was born in a month having least number of days. A was born in a
month having 30 days. A was not born in April, June and November.
C was born in on one of the month between D and A.
II. No person was born after B. C was born in a month having 31 days.
F was born in a month immediately after A. There is gap of 4
months in between the months in which F and D were born.
11 Adda247 Publications For any detail, mail us at
Publications@adda247.com
Cracker Book for Bank (IBPS | SBI | RRB PO | Clerk) Mains Exams

Directions (4-5): Each of the questions below, consist of a question and


three statements numbered I, II and III. You have to decide whether the
data provided in the statements are sufficient to answer the question. Read
the three statements and Give answer
(a) If the data in statement I and II together are sufficient to answer the
question, while the data in statement III are not required to answer the
question.
(b) If the data in statement I and III together are sufficient to answer the
question, while the data in statement II are not required to answer the
question.
(c) If the data in statement II and III are sufficient to answer the question,
while the data in statement I are not required to answer the question.
(d) If the data in all three statements I, II and III together are necessary to
answer the question.
(e) If the data in all the statements, I, II and III even together are not
sufficient to answer the question.

4. Six persons A, B, C, D, E, F are sitting around a circular table facing


towards the center. Each of them like different color Blue, White, Pink,
Purple, Black, Red. The one who is sitting second to the left of C like
which of the following color?
I. F sits opposite to E. The one who likes Blue color sits second to the
right of E. The one who likes Purple is an immediate neighbour of
the one who likes Blue color. B does not like Purple color. C is an
immediate neighbour of A.
II. C sits second to the left of E and likes Black color. The one who likes
Pink color sit between the one who likes Red color and the one who
likes White color. F sits to the immediate right of D.
III. Only D sits between F and B. B likes White color and sits opposite
to C.

5. Ten persons P, Q, R, S, T, U, V, W, X, Y are living in a five storey building


such as ground floor is numbered as 1, above it is floor 2 then top floor
is numbered as 5. Each of the floor has 2 flats in it as flat-1 and flat-2.
Flat-1 of floor-2 is immediately above flat-1 of floor-1 and immediately
below flat-1 of floor-3 and so on. In the same way flat-2 of floor-2 is
immediately above flat-2 of floor-1 and immediately below flat-2 of
floor-3 and so on. Who among the following lives in flat-2 of floor-4?
12 Adda247 Publications For any detail, mail us at
Publications@adda247.com
Cracker Book for Bank (IBPS | SBI | RRB PO | Clerk) Mains Exams

I. Q lives on floor-2 and T lives to the west of Q. There is two floors


gap between Q and V.
II. X lives to the east of W. V does not live in the same flat number as
W. There is a two floors gap between S and U.
III. P lives to the west of S but does not live on ground floor. Y lives
above R. Y and R live in same flat number.

Directions (6-8): Each of the questions below, consist of a question and


three statements numbered I, II and III. You have to decide whether the
data provided in the statements are sufficient to answer the question. Read
the three statements and Give answer
(a) If the data in statement I and II together are sufficient to answer the
question, while the data in statement III are not required to answer the
question.
(b) If the data in statement I and III together are sufficient to answer the
question, while the data in statement II are not required to answer the
question.
(c) If the data in statement II and III are sufficient to answer the question,
while the data in statement I are not required to answer the question.
(d) If the data in all three statements I, II and III together are necessary to
answer the question.
(e) If the data in all the statements, I, II and III even together are not
sufficient to answer the question.

6. Some persons are sitting in a row such that all are facing in the north
direction. Who among the following sits third to the left of M (less than
15 people sits in the row)?
I. M sits fourth to the right of A. Only two people sits between A and
K. O sits on the sixth right of N
II. Only 2 person sits between N and D. M sits at the end of the row.
III. D sits eighth to the left of K. B sits third to the right of O.
13 Adda247 Publications For any detail, mail us at
Publications@adda247.com
Cracker Book for Bank (IBPS | SBI | RRB PO | Clerk) Mains Exams

7. In the question given below two conclusions followed by three


statements. You have to choose the correct set of statements that
logically satisfies the given conclusions
Conclusions:
Some Rhombus is not Square.
No Ring are Rhombus.
Statements:
I. All Ring are Square. No Rhombus is rectangle.
II. All ring are square. Some square are Rhombus.
III. Only Rectangle are square. Some ring are circle.
8. Some students are sitting in a row such that all are facing in the same
direction. How many persons sits in the row.
I. A sits fifth to the right of G, who sits seventh to the right of M. Two
persons sits between D and A. M does not sits at the end of the row.
A, sits at the end of the row.
II. D sits sixth to the right of B. B sits third to the right of M.
III. A sits seventh to the right of M. M sits at the end of the row. Three
persons sits between O and A, who does not sit fifth place from the
end of the row.

Directions (9-10): Each of the questions below consists of a question and


two statements numbered I and II given below it. You have to decide
whether the data provided in the statements are sufficient to answer the
question.
(a) If statements I alone is sufficient to answer the question, but statement
II alone is not sufficient to answer the question.
(b) If statement II alone is sufficient to answer the question, but statement I
alone is not sufficient to answer the question.
(c) If statement either I or II is sufficient to answer the question.
(d) If both the statements I and II taken together are not sufficient to
answer the questions.
(e) If both the statements I and II taken together are sufficient to answer
the questions.
14 Adda247 Publications For any detail, mail us at
Publications@adda247.com
Cracker Book for Bank (IBPS | SBI | RRB PO | Clerk) Mains Exams

9. In a class of certain number of students, all of them are sitting in a row


according to their rank. J’s rank is 10th from the left end. What will be
L’s rank from the left end?
(I) K’s rank is 5th from the right end and he is 4th from the right of H.
Only three persons sit between H and D. D is 7th from the left end.
(II) O is exactly in between J and L. O is third to the left of H.

10. Six persons P, Q, R, S, T, U are sitting around a circular table. Some of


them are facing the center while some are facing away from the center.
How many persons are facing inside?
(I) P sits third to the right of Q, who is facing outside. Both R and U are
immediate neighbour of each other. S is not an immediate
neighbour of Q.
(II) U sits second to the left of Q, who is sitting third to the left of P. R
sits second to the right of T.

Directions (11-12): Each of the questions below consists of three


statements numbered I, II and III given below it. You have to decide
whether the data provided in the statements are sufficient to answer the
question.

11. Five persons P, Q, R, S and T are standing in a row facing north in


increasing order of their heights from left to right. That means the
shortest one stands at the leftmost end while the tallest one stands at
the rightmost end. Who among the following is the fourth tallest?
I. Q and S are shorter than R. S height is 172 cm. Only two persons
are standing to the right of S.
II. Q’s height is 178 cm. T is not an immediate neighbour of R.
III. Q is an immediate neighbour of T and S. More than two persons are
standing to the left of P.
(a) Statement (I) and Statement (II) together are sufficient.
(b) Statement (II) and Statement (III) together are sufficient.
(c) Statement (I) and Statement (III) together are sufficient.
(d) All the statement (I), statement (II) and statement (III) together are
sufficient.
(e) Either two of the given statements are sufficient.

15 Adda247 Publications For any detail, mail us at


Publications@adda247.com
Cracker Book for Bank (IBPS | SBI | RRB PO | Clerk) Mains Exams

12. A, B, C, D and E are five friends among the 15 persons who participated
in quiz. Each of the 15 participants obtained different marks (a whole
number). One of the participant among the given five friends obtained
11 marks. Who obtained 11 marks?
I. B obtained 4 marks. Least marks obtained by a person is 2. Third
lowest scorer obtained either 5 or 6 marks.
II. Marks of C lies somewhere between D and A. There are only five
participants whose marks lies between E and D. A obtained 3 more
marks than B.
III. D scored 5 more marks than A. E scored 18 marks. Neither A nor B
score more marks than D. There are only two candidates whose
marks lies between C and A.
(a) Statement (I) and Statement (II) together are sufficient.
(b) Any two statements together are sufficient.
(c) Either statement (I) and statement (II) together or statement (III)
are statement
(d) All the statement (I), statement (II) and statement (III) together are
sufficient.
(e) None is sufficient.

Directions (13-15): Each of the questions below consists of three


statements numbered I, II and III given below it. You have to decide
whether the data provided in the statements are sufficient to answer the
question.
Given answer:
(a) If the data in statement I alone are sufficient to answer the question,
while the data in statement II alone are not sufficient to answer the
question.
(b) If the data in statement II alone are sufficient to answer the question,
while the data in statement I alone are not sufficient to answer the
question.
(c) If the data either in statement I alone or in statement II alone are
sufficient to answer the question.
(d) If the data even in both statements I and II together are not sufficient to
answer the question.
(e) If the data in both statement I and II together are necessary to answer
the question.
16 Adda247 Publications For any detail, mail us at
Publications@adda247.com
Cracker Book for Bank (IBPS | SBI | RRB PO | Clerk) Mains Exams

13. Six persons P, Q, R, S, T and U are sitting around a circular table, are
they all facing the center?
I. P sits second to the left of U. S sits second to the right of U. T does
not sit opposite to S who sit second to the left of P. Both T and R are
immediate neighbour of P. U is facing the center. R sits second to
the right of T. Q sits second to the right of R. Both Q and S are facing
same direction as P.
II. Q is second to the left of T. Only S is between Q and T. R is to
immediate left of U. Q sits opposite to P. P is second to the right of
S. T is second to the right of Q and second to the left of R. Both Q
and P faces each other.

14. Five boxes D, V, F, N, K are placed in a row from west to east direction
according to their weight in an increasing order. And also each of them
is of different color.
The box which is second lightest is of which color?
(I) F placed at one of the extreme end. Purple box is heavier than Red
box. Neither box K nor V is the heaviest. Box D is not of Blue and
Red color. Pink box is not heavier than Red box.
(II) Only one box is placed in between box K and V. Box D is heavier
than Pink box. Box V is not placed adjacent to F. Box N is of Black
color. Blue box is lighter than N. Neither K nor F is of Blue color.
Blue box is heavier than Purple box.
15. Five persons K, L, M, N, O lives on five different floors such as bottom
floor is numbered as 1 and top floor is numbered as 5. Also each of
them born in different months of a year but no two person born in two
consecutive months. The one who lives on fourth floor born in which of
the following month?
(I) Three person lives between L and O. O is older than M and born in
a month having 31 days. M lives on third floor and born in April. L
is born in one of the month after July but in a month having 30
days.
(II) Only one person lives between K and N and K is younger than N. O
does not live above M. N is born before L. K was not born in last
month of the year and is younger than L. N was not born in a month
of 30 days.

17 Adda247 Publications For any detail, mail us at


Publications@adda247.com
Cracker Book for Bank (IBPS | SBI | RRB PO | Clerk) Mains Exams

Directions (16-20): Each of the questions given below consists of a


question and two statements numbered I and II. You have to decide
whether the data provided in the statements are sufficient to answer the
question.
(a) If statement I alone is sufficient to answer the question, but statement
II by itself is not sufficient to answer the question.
(b) If statement II by itself is sufficient to answer the question, but
statement I alone is not sufficient to answer the question.
(c) If statement either I or II is sufficient to answer the question.
(d) If both the statements I and II taken together are not sufficient to
answer the question.
(e) If both the statements I and II taken together are sufficient to answer
the question.

16. Conclusion: Some Milk are Coffee. Some Coffee are not Cold-drinks.
Which of the following set of statement is required that logically
satisfies given conclusions (Given statements to be true even if they
seem to be at variance from commonly known facts).
Statement I: Some Milk are Drink. Some Coffee are Tea.
Statement II: All drink are Coffee. No Tea is Cold-drink.

17. Six persons A, B, C, D, E and F are sitting around a circular table. Are all
facing towards the center?
Statement I: E sits second to the left of A, who is facing towards the
center. C sits opposite to D. F sits to the immediate right of A. A sit
second left of C and C does not sit between A and E. F sits second to
right of D. F sits third left of E.
Statement II: F sits second left of B. E sits second right of C and is not
an immediate neighbour of F. D sits third left of C. B is facing towards
the center.

18. What is code of ‘Allegation’?


Statement I: In certain code language, ‘Speech audience popularity’ is
coded as- ‘kl da bp’ and ‘Conference debate report’ is coded as- ‘nu ga
tr’
Statement II: In certain code language, ‘Deliver patience Speech’ is
coded as- ‘bt hd kl’ and ‘Allegation conference popularity’ is coded as-
‘wa ga bp’.
18 Adda247 Publications For any detail, mail us at
Publications@adda247.com
Cracker Book for Bank (IBPS | SBI | RRB PO | Clerk) Mains Exams

19. Five persons P, Q, R, S, T are sitting in a row facing north direction. Who
among the following sits at extreme left end?
Statement I: Q is third to the left of P. T is not an immediate neighbour
of P. S sits second to the right of R.
Statement II: T sits second to the left of P. More than two persons sit
between Q and S. Q sits to the left of R.

20. Who among the following is the father of K?


I. L is the father of R. R is the brother-in-law of D. D is the wife of K. K
is the son of M, who is the wife of L.
II. F is the son of K. F is married to G, who is the daughter-in-law of D.
F is the grandson of M.

Directions (21-23): Each of the questions below, consist of a question and


three statements numbered I, II and III. You have to decide whether the
data provided in the statements are sufficient to answer the question. Read
the three statements and Give answer
(a) If the data in statement I and II together are sufficient to answer the
question, while the data in statement III are not required to answer the
question.
(b) If the data in statement I and III together are sufficient to answer the
question, while the data in statement II are not required to answer the
question.
(c) If the data in statement II and III are sufficient to answer the question,
while the data in statement I are not required to answer the question.
(d) If the data in all three statements I, II and III together are necessary to
answer the question.
(e) If the data in all the statements, I, II and III even together are not
sufficient to answer the question.
21. Some persons are sitting in a row such that all are facing in the north
direction. Who among the following sits fourth to the left of D?
I. M sits third to the right of D. Only five persons sit between M and A.
More than two person sits between M and N.
II. D sits second to the left of M, who sits sixth to the left of N. A sits third
to the right of N.
III. More than six person sits between R and D. B sits exactly between N
and A. B sits fifth to the right of R. Only one person sits between N and
A.
19 Adda247 Publications For any detail, mail us at
Publications@adda247.com
Cracker Book for Bank (IBPS | SBI | RRB PO | Clerk) Mains Exams

22. In the question given below two conclusions followed by three sets of
statement. You have to choose the correct set of statement that logically
satisfies the given conclusions
Conclusion: I. Some hands are not Socks.
II. No legs are socks.
I. All legs are Hands. Some Shoes are legs. No Legs are fingers.
II. Some watches are arm. All arms are fingers. All legs are Hands.
III. No Socks are Shoes. All legs are Shoes. Some Hands are finger.

23. Eight persons i.e. A, B, C, D, E, F, G and H are sitting around a circular


table such that some of them face inside and some face outside. How
many persons face outside?
I. A sits second to the right of E and second to the right of D. G and D
faces opposite direction. C and G are the immediate neighbour of E.
F does not face inside.
II. Not more than two persons sitting adjacent to each other faces
same direction. H sits second to the left of C, who sits third to the
left of B. H and A face same direction. C and G face opposite
direction.
III. A sits third to the left G, who sits second to the right of C. E is not
the immediate neighbor of G.

Directions (24-25): Each of the questions below, consist of a question and


three statements numbered I, II and III. You have to decide whether the
data provided in the statements are sufficient to answer the question. Read
the three statements and Give answer
(a) If the data in statement I and II together are sufficient to answer the
question, while the data in statement III are not required to answer the
question.
(b) If the data in statement I and III together are sufficient to answer the
question, while the data in statement II are not required to answer the
question.
(c) If the data in statement II and III are sufficient to answer the question,
while the data in statement I are not required to answer the question.

20 Adda247 Publications For any detail, mail us at


Publications@adda247.com
Cracker Book for Bank (IBPS | SBI | RRB PO | Clerk) Mains Exams

(d) If the data in all three statements I, II and III together are necessary to
answer the question.
(e) If the data in all the statements, I, II and III even together are not
sufficient to answer the question.

24. Six lectures on different subjects i.e. Physics, Chemistry, Math’s,


Biology, Economics and English delivered by different professor i.e. A,
B, C, D, E and F on the different days of the week starting from Monday
to Saturday but not necessarily in the same order. Who among the
following professor delivered the lecture on Wednesday and on which
subject?
I. More than two professors delivered the lecture before the one who
deliver the lecture of English. Three professors deliver the lecture
between B and C, who delivered the lecture before B. C delivered
the lecture of Biology. Only one professor delivered the lecture
between B and the one who deliver the lecture of physics.
II. Two professor deliver the lecture between C and the one who
deliver the lecture of English. Only two lectures were delivered
between Physics and Math’s. Two professors deliver the lecture
between D and E. B does not delivered the lecture of economics. A
delivered the lecture before F.
III. Only one professor deliver the lecture between F and the one who
deliver the lecture of Math’s. B delivered the lecture before the one
who delivered the lecture on Math’s. D deliver the lecture
immediately after A, who deliver the lecture on Economics.

25. There are eight persons i.e. A, B, C, D, E, F, G and H are sitting in row
such that some are facing towards north and some are facing towards
south direction. How many persons faces north direction?
I. Only one person sits on the right of A. Only three persons sits
between C and A. A and C faces opposite direction. B sits second to
the left of C. There are two vacant seats and none of the vacant
seats are at the end of the row. G and B are not the immediate
neighbour of the vacant seats.

21 Adda247 Publications For any detail, mail us at


Publications@adda247.com
Cracker Book for Bank (IBPS | SBI | RRB PO | Clerk) Mains Exams

II. D sits third to the left of G, who faces same direction as A. G does
not sits at the end of the row. H and B face opposite direction. E
faces North. H does not face north direction.
III. Immediate neighbours of G face opposite direction. Four persons
sits between the two vacant seats.
E sits third to the left of F. F does not sits at the end of the row. H
and D faces opposite direction.

Directions (26-30): Each of the questions below consists of a question and


two statements numbered I and II given below it. You have to decide
whether the data provided in the statement are sufficient to answer the
question. Read both the statements and Give answer:
(a) If the data in statement I alone are sufficient to answer the question,
while the data in statement II alone are not sufficient to answer the
question.
(b) If the data in statement II alone are sufficient to answer the question,
while the data in statement I alone are not sufficient to answer the
question.
(c) If the data either in statement I alone or in statement II alone are
sufficient to answer the question.
(d) If the data even in both statements I and II together are not sufficient to
answer the question.
(e) If the data in both statements I and II together are necessary to answer
the question.

26. In a certain code language ‘fi ku le ja’ means ‘Indian Cricket Premier
League’. Then what is the code of ‘league’? If,
I. In the given code language ‘Premier league shifted venue’ is written
as ‘le hi nu ku’.
II. In the given code language ‘league venue cricket pune’ is written as
‘nu le fi un’.

27. How is N related to D?


I. N is the only child of R. D is father of M, who is brother-in-law of S.
P is the brother of M. M is married to R. P is unmarried.

22 Adda247 Publications For any detail, mail us at


Publications@adda247.com
Cracker Book for Bank (IBPS | SBI | RRB PO | Clerk) Mains Exams

II. D is married to A. P is the son of N. Q is brother of M. N is married


to M. Q is the only male child of A. S is the sister of N. A is a male
28. On which of the following day of the week (starting from Monday and
ends on Sunday) match of CSK is scheduled?
I. Dhoni captain of CSK completely remembers that match is
scheduled after Wednesday but not on Sunday.
II. Jadeja correctly remembers that the match of CSK is scheduled
before Friday but after Monday.

29. Some persons i.e. P, Q, A, C, M, D and S are sitting in a row such that all
are facing in the north direction, who among the following sits on the
right end of the row?
I. Q sits third to the left of D and both does not sit at the end of the
row. S sits fourth to the right of A and one of them sits at the end of
the row. C sit third to the right of P. M is not the immediate
neighbour of D.
II. D sits second to right of Q. One of the immediate neighbour of D sits
fifth to the right of M. S sits second to the right of A. Neither P nor C
sits at the end of the row. P and C are not the immediate
neighbours of each other.

30. How many people are sitting around the circle (all are facing inside)?
I. Z is sitting third to the right of C, who is sitting third to the right of
A. Three persons are sitting in between F and Z.
II. Two persons are sit in between A and F who is sitting second to the
right of Z. C is sitting third right of A and Three person sit between
Z and C.

Directions (31-34): Each of the questions below consists of a question and


two statements numbered I and II given below it. You have to decide
whether the data provided in the statements are sufficient to answer the
question. Read both the statements. Give answer-
(a) If the data in statement I alone are sufficient to answer the question,
while the data in statement II alone are not sufficient to answer the
question.

23 Adda247 Publications For any detail, mail us at


Publications@adda247.com
Cracker Book for Bank (IBPS | SBI | RRB PO | Clerk) Mains Exams

(b) If the data in statement II alone are sufficient to answer the question,
while the data in statement I alone are not sufficient to answer the
question.
(c) If the data either in statement I alone or in statement II alone are
sufficient to answer the question.
(d) If the data in both statements I and II together are not sufficient to
answer the question.
(e) If the data in both statements I and II together are necessary to answer
the question.
31. A, B, C, D, E and F are sitting in a circle, some of them facing towards the
centre while some are facing away from the center of the circle. How
many persons are facing inside?
I F is on the immediate left of E. Only D is between B and E. C is
immediate right of F. C is second to the left of B. A does not faces
same direction as E. D and C faces opposite direction of A.
II. A is facing D. Only C is between A and B. Only F is between E and A.
E sits second to the right of B. Both F and C faces same direction but
opposite to B. F does not sits to the immediate left E.

32. Amongst P, Q, R, S and T all of them are of different price. Also they are
placed one above another. Which book is placed at the top?
I. Price of the book R is more than the price of book P. Only two
books are placed between book R and P. Price of the book S is
second costliest among all and is placed above T.
II. Price of the book Q is more than only P and T and costliest book is
not placed at top. The third costliest book is exactly in the middle.

33. Six people viz. I, J, K, L, M and N lives in a Building on different floors


from top to bottom (such as ground floor numbered as 1 and top is
numbered as 6). Each of them likes different colours Black, White, Blue,
Pink, Purple, Grey. Which color is liked by the one who lives on 4th
floor?
(I) Only one person lives between L and M. J lives above I who likes
Black color. There is a gap of three floors between J and L and both
of them lives on odd number of floor. The one who lives on top
floor likes Purple color.
(II) K likes White color. The one who likes Grey color lives above I. J
does not like Grey color.
24 Adda247 Publications For any detail, mail us at
Publications@adda247.com
Cracker Book for Bank (IBPS | SBI | RRB PO | Clerk) Mains Exams

34. Six people viz. D, F, L, M, Q, R are sitting in a row some of them are
facing north while some of them are facing south direction. Who sits to
the immediate right of D?
(I) M sits third to right of R.Q sits second to the left of M. R is not an
immediate neighbour of Q. D is not an immediate neighbor of Q.
(II) L sits third to the left of Q who is facing north direction. Both the
immediate neighbours of M faces same direction as M.

Directions (35): Each of the questions below consists of a question and


two statements numbered I and II given below it. You have to decide
whether the data provided in the statement are sufficient to answer the
question. Read both the statements and Give answer:
(a) If the data in statement I alone are sufficient to answer the question,
while the data in statement II alone are not sufficient to answer the
question.
(b) If the data in statement II alone are sufficient to answer the question,
while the data in statement I alone are not sufficient to answer the
question.
(c) If the data either in statement I alone or in statement II alone are
sufficient to answer the question.
(d) If the data even in both statements I and II together are not sufficient to
answer the question.
(e) If the data in both statements I and II together are necessary to answer
the question.

35. Five friends J, K, L, M, N born on different dates 8, 13, 19, 24, 31 but not
necessarily in the same order in the month of March and June in the
same year. Only one person born on one of the given date. M is born on
which of the following date?
I. Both M and J are not born on even date of the month. M is born
before J. Two persons are born in between M and N.
II. Nobody is born after K. L is born on a date in between M and J. L is
born on an even date. M does not born on 13 and N does not born
on 19.
25 Adda247 Publications For any detail, mail us at
Publications@adda247.com
Cracker Book for Bank (IBPS | SBI | RRB PO | Clerk) Mains Exams

Directions (36-37): Each of the questions below, consist of a question and


three statements numbered I, II and III. You have to decide whether the
data provided in the statements are sufficient to answer the question. Read
the three statements and Give answer
(a) If the data in statement I and II together are sufficient to answer the
question, while the data in statement III are not required to answer the
question.
(b) If the data in statement I and III together are sufficient to answer the
question, while the data in statement II are not required to answer the
question.
(c) If the data in statement II and III are sufficient to answer the question,
while the data in statement I are not required to answer the question.
(d) If the data in all three statements I, II and III together are necessary to
answer the question.
(e) If the data in all the statements, I, II and III even together are not
sufficient to answer the question.

36. Seven persons i.e. A, B, C, D, E, F and G are sitting around a circular


table. Are all the persons facing towards the centre of the table?
I. A sits third to the right of C, who sits second to the left of D. Only two
person sits between G and A. B is not the immediate neighbour of C. C
faces inside.
II. D sits third to the left of B. and second to the right of E. C sits third to
the right of F, who is not the immediate neighbour of E.
III. F sits third to the right of B. A and G faces same direction. F sits second
to the left of A. E sits third to the right of F, and on the immediate left of
B. E and B faces same direction. B is not an immediate neighbour of A.

37. There are 9 members i.e. A, B, D, G, H, R, Z, I and K in a family having


three generation. How is R related to Z?
I. Z has three children. C is the unmarried brother of G. B is the sister-in-
law of D. There are three married couple in the family.
II. D is the daughter in law of A, who is the father in law of I. K is the
cousin of R.
III. Z has only one daughter. I is the brother-in-law of C. B is the mother of
K.

26 Adda247 Publications For any detail, mail us at


Publications@adda247.com
Cracker Book for Bank (IBPS | SBI | RRB PO | Clerk) Mains Exams

Directions (38-40): Each of the questions below consists of a question and


two statements numbered I and II given below it. You have to decide whether
the data provided in the statements are sufficient to answer the question.
Read both the statements and give answer.
(a) If the data in statement I alone are sufficient to answer the question, while
the data in statement II alone are not sufficient to answer the question.
(b) If the data in statement II alone are sufficient to answer the question, while
the data in statement I alone are not sufficient to answer the question.
(c) If the data either in statement I alone or in statement II alone are sufficient
to answer the question.
(d) If the data in both statements I and II together are not sufficient to answer
the question.
(e) If the data in both statements I and II together are necessary to answer
the question.
38. Eight people viz. G, H, I, J, K, L, M and N lives in a Building on different
floors from top to bottom(such as ground floor numbered as 1 and top
is numbered as 8) where I lives on floor number 6. Who among the
following lives on 4th floor?
(I) Only one person lives between L and M. J lives above I. There is a
gap of three floors between J and L and both of them lives on odd
number of floor.
(II) Three persons live between K and H. N lives just above H who lives
on even numbered floor.

39. Seven people viz. A, D, F, L, M, Q, R are sitting in a row some of them are
facing north while some of them are facing south direction. R sits at an
extreme end of the row. F is not facing south direction. D sits third to
the left of F. Who sits to the immediate right of F?
(I) M sits third to right of R.Q sits second to the left of M.D is not an
immediate neighbor of Q.
(II) A sits third to the left of Q. F is an immediate neighbor of M.F sits
second to the right of L.

40. In a certain code language -‘benefits life in loss’ is coded as ‘hlt sa rtv
mlp’ then what will be the code of ‘Benefits’?
(I) The code of ‘live life trouble benefits’ is ‘mlp hlt ngi snk’,
(II) The code of ‘live happy feeling at’ is ‘ngi riy nop hus’.
27 Adda247 Publications For any detail, mail us at
Publications@adda247.com
Cracker Book for Bank (IBPS | SBI | RRB PO | Clerk) Mains Exams

Solutions

1. (a); From I

Case-1 Case-2
Days Person Floors Person Floors
Monday R
Tuesday P/ R
Wednesday T/ 8th P 8th
Thursday S T
Friday P/ S
Saturday T/

From II,
Case-1 Case-2
Days Person Floors Person Floors
Monday 1st
Tuesday T 1st
Wednesday T
Thursday
Friday 5th
Saturday U U 5th

Now, from I, II-

Days Person Floors


Monday R 9th
Tuesday P 1st
Wednesday T 8th
Thursday S
Friday Q
Saturday U 5th
So, clearly P has office on 1st floor attend office on Tuesday.

28 Adda247 Publications For any detail, mail us at


Publications@adda247.com
Cracker Book for Bank (IBPS | SBI | RRB PO | Clerk) Mains Exams

2. (d); Even by combining both I and II we cannot find the input of the
output given in the question.

3. (d); From I,
Person Month
E February
A September

From II,
Person Month
B December
C (31)

From I and II,


Persons Months
A September
B December
D May
C July/August
E February
F October

So, even by combining both I and II we cannot find that C was born
in which of the following month.
4. (a); From I, From II,

29 Adda247 Publications For any detail, mail us at


Publications@adda247.com
Cracker Book for Bank (IBPS | SBI | RRB PO | Clerk) Mains Exams

From III,

From I and II,

So, we get that the one who is sitting second to the left of C is D and
like Blue color.

5. (d); From I,
Floors Flat-1 Flat-2
5 V/ V/
4
3
2 T Q
1

30 Adda247 Publications For any detail, mail us at


Publications@adda247.com
Cracker Book for Bank (IBPS | SBI | RRB PO | Clerk) Mains Exams

From I, II and III-

Floors Flat-1 Flat-2


5 Y V
4 P S
3 W X
2 T Q
1 R U

6. (d); The data in all three statements I, II and III together are necessary
to answer the question.
From statement I and III M sits at the end of the row and fourth to
the right of A. B sits third to the right of O. less than 15 people sits
in the row. Only two people sits between A and K and D sits eighth
to the left of K so K sits third to the right of A because if K sits third
to the left of A and we placed D then there will be sixteen persons
in the row which can’t be possible. Now by using statement II that
Only 2 person sits between N and D. M sits at the end of the row.
We get our final answer B sits third to the left of M.

7. (b); Using statement I and III we can find the definite relation between
Rhombus and Square and between Ring and Rhombus which is not
possible if we use statement II.

31 Adda247 Publications For any detail, mail us at


Publications@adda247.com
Cracker Book for Bank (IBPS | SBI | RRB PO | Clerk) Mains Exams

8. (e); From the statements, I, II

As it is given that M does not sits at the end of the but it is also not
mention in any statement that how many persons sits on the left of
M. Hence, it cannot get confirmed how many person sits in the row.
From statement II and III,

From these we cannot conclude that O sitting at the end of the row
or not.
9. (e); From I,

From I and II,

10. (d); From I,

From II,

32 Adda247 Publications For any detail, mail us at


Publications@adda247.com
Cracker Book for Bank (IBPS | SBI | RRB PO | Clerk) Mains Exams

Even by combining both the statements I and II together we get that


both the are not sufficient to answer the questions.

11. (c); From (I) and (III), The arrangement of person facing north in
increasing order of their heights from left to right is--------------
T< Q < S < P/R < P/R

12. (d); Using the information given in statement (I), (II) and (III),
From (I) B obtained 4 marks. Least marks obtained by a person is
2. Third lowest scorer obtained either 5 or 6 marks. It means B is
the second least scorer.
From (II) A obtained 3 more marks than B. It means A obtained
seven marks he could be either the fourth or the fifth lowest scorer.
Marks of C lies somewhere between D and A.
From (III) D scored 5 more marks than A. It means D scored 12
marks. There are only two candidates whose marks lies between C
and A. E scored 18 marks.

Rank Person Marks


1 2
2 B 4
3 5/6
4 A 7
5
6
7 C
8 D 12
9
10
11
12
13
14
15

33 Adda247 Publications For any detail, mail us at


Publications@adda247.com
Cracker Book for Bank (IBPS | SBI | RRB PO | Clerk) Mains Exams

Or

Rank Person Marks


1 2
2 B 4
3 5
4 6
5 A 7
6
7
8 C
9 D 12
10
11
12
13
14
15

And as we know that One of the participant among the given five
friends obtained 11 marks it means C must have scored 11 marks.

13. (c);
From II, From II,

34 Adda247 Publications For any detail, mail us at


Publications@adda247.com
Cracker Book for Bank (IBPS | SBI | RRB PO | Clerk) Mains Exams

14. (e); From I,

From II,

Purple < Blue < Black (N) and Pink < D


From both I and II,
We get that F is the lightest box and is of Pink color and K is second
lightest and is of Red color.

15. (d); From I,

And L can be born either in September or November. And O is born


in January.
From II,
K<L<N
From both I and II,
Even by combining both the statements we only know that K is
born in November and N is born in July but we cannot find that
either K or N live on 4th floor, so both together are not sufficient.

16. (e); By using both statement I and II-

35 Adda247 Publications For any detail, mail us at


Publications@adda247.com
Cracker Book for Bank (IBPS | SBI | RRB PO | Clerk) Mains Exams

17. (d); Even by combining both statement I and II the direction of facing of
F cannot be determined. So, both the statements I and II taken
together are not sufficient to answer the question.

18. (e); By using both statement I and II the code of Allegation will be- wa.

19. (c); From either I or II we will get that Q sits at extreme left end of the
row.

20. (a); Statement I alone is sufficient to answer the question.

But from Statement II we cannot find that M is either father or


mother of K. So, statement II by itself is not sufficient to answer the
question.
36 Adda247 Publications For any detail, mail us at
Publications@adda247.com
Cracker Book for Bank (IBPS | SBI | RRB PO | Clerk) Mains Exams

21. (b); The statement I and III together are sufficient to answer the
question;
Step 1: M sits third to the right of D. Only five persons sits between
M and A. More than two person sits between M and N. Only one
person sits between N and A.

Step 2: B sits exactly between N and A. B sits fifth to the right of R.


More than six person sits between R and D. So, case 1 Case 2 and
case 4 gets eliminated.

Hence, B sits fourth to the left of D.

22. (c); Combining both the statements II and III we get,

I. From the venn diagram it is clear that all legs are hand, all legs
are shoes and no shoes is socks. Hence, we can conclude that
some hands are not socks.
II. From the venn diagram it is clear that all legs are shoes and no
shoes are socks. Hence, we can conclude that no legs are socks.

37 Adda247 Publications For any detail, mail us at


Publications@adda247.com
Cracker Book for Bank (IBPS | SBI | RRB PO | Clerk) Mains Exams

23. (a); Using statements, I and II.


A sit second to the right of E and second to the right of D. C and G
are the immediate neighbour of E. H sits second to the left of C, who
sits third to the left of B.

Now, not more than two persons sitting adjacent to each other
faces same direction. F does not face inside. H and A face same
direction. C and G face opposite direction. So, case 1 gets
eliminated.

Therefore, five person faces outside.


24. (b); From statement I and III
More than two professors delivered the lecture before the one who
deliver the lecture of English. Three professors deliver the lecture
between B and C, who delivered the lecture before B. C delivered
the lecture of Biology. B delivered the lecture before the one who
delivered the lecture on Math’s. Only one professor delivered the
lecture between B and the one who deliver the lecture of physics.
38 Adda247 Publications For any detail, mail us at
Publications@adda247.com
Cracker Book for Bank (IBPS | SBI | RRB PO | Clerk) Mains Exams

Only one professor deliver the lecture between F and the one who
deliver the lecture of Math’s. D deliver the lecture immediately
after A, who deliver the lecture on Economics. So, the final
arrangement is--

Days Professor Subject


Monday C Biology
Tuesday A Economics
Wednesday D Physics
Thursday F English / Chemistry
Friday B Chemistry / English
Saturday E Math’s

25. (d); Using the given informations from I, II and III,


Four persons sits between the two vacant seats. Only one person
sits on the right of A. There are two vacant seats and none of the
vacant seats are at the end of the row. G and B are not the
immediate neighbour of the vacant seats. B sits second to the left of
C. A and C faces opposite direction.

39 Adda247 Publications For any detail, mail us at


Publications@adda247.com
Cracker Book for Bank (IBPS | SBI | RRB PO | Clerk) Mains Exams

D sits third to the left of G, who faces same direction as A. G does


not sits at the end of the row. Immediate neighbours of G face
opposite direction. E sits third to the left of F. F does not sits at the
end of the row. H and B face opposite direction. E faces North. H
and D faces opposite direction.

H does not face north direction. So, from this case-1 will be
eliminated. And the final arrangement is-----

26. (e); Using Statement I and II together,

Words Codes
Indian ja
Cricket fi
League le
Venue nu
Premier ku
Shifted hi
pune un

The code for League is ‘le’.

40 Adda247 Publications For any detail, mail us at


Publications@adda247.com
Cracker Book for Bank (IBPS | SBI | RRB PO | Clerk) Mains Exams

27. (b); From statement I, we can’t determine the gender of N.

From the statement II,

N is the son-in-law of D.

28. (e); From statement I, Dhoni remembers that the match is scheduled on
either Thursday, Friday or on Saturday.
From Statement II, Jadeja remembers that the match is scheduled
either on Tuesday, Wednesday or on Thursday.
So, using both the statement CSK match is scheduled on Thursday.

29. (c); From Statement 1:


Step 1: From the given statement I, Q sits third to the left of D and
both does not sit at the end of the row. S sits fourth to the right of A
and of them sits at the end of the row. There will be two possible
cases

41 Adda247 Publications For any detail, mail us at


Publications@adda247.com
Cracker Book for Bank (IBPS | SBI | RRB PO | Clerk) Mains Exams

Step 2: Now, C sit third to the right of P. M is not the immediate


neighbour of D. So, case 1 will be eliminated and we get our final
answer,

From Statement II,


Step 1: As it is given that, D sits second to right of Q. One of the
immediate neighbour of D sits fifth to the right of M. So, there will
be two possible cases,

Step 2: Now, S sits second to the right of A. Neither P nor C sits at


the end of the row. Neither P nor C are the immediate neighbours
of each other. So, case 2 will be eliminated and we get our final
answer,

30. (b); From statement II,


Step 1: Using the given conditions, two persons are sit in between
A and F who is sitting second to the right of Z. There will be two
possible cases

42 Adda247 Publications For any detail, mail us at


Publications@adda247.com
Cracker Book for Bank (IBPS | SBI | RRB PO | Clerk) Mains Exams

Step 2: Now, C is sitting third right of A, So, case 1 will be


eliminated. Continuing with step 2, Three person sit between Z and
C. Hence, we get our final answer

31. (c); From I, we get two possible case-

From both of the above cases we get that three persons are facing
inside.
From II,

From both of the above cases we get that four persons are facing
inside.
So, either I or II is sufficient to answer the question.
43 Adda247 Publications For any detail, mail us at
Publications@adda247.com
Cracker Book for Bank (IBPS | SBI | RRB PO | Clerk) Mains Exams

32. (d); From I,


R>P and > S> > >

Books
R/P

R/P

From II, Q> P, T


Books

From I and II,


R>S>Q>P/T>P/T

As, even by combining both we can say either book S or P is placed


at top. So, data in both statements I and II together are not
sufficient to answer the question.

33. (e); From I,


Floors Persons Colours
6 Purple
5 J
4 I/ Black/
3 M
2 I/ Black/
1 L
44 Adda247 Publications For any detail, mail us at
Publications@adda247.com
Cracker Book for Bank (IBPS | SBI | RRB PO | Clerk) Mains Exams

From II, K likes White color. The one who likes Grey color lives
above I.

From I and II,

Floors Persons Colours


6 Purple
5 J Grey/
4 K White
3 M Grey/
2 I Black
1 L

So, From both I and II we get that K lives on 4th floor and likes
White colour.

34. (d); From I,

From II,

From I and II,

Even by combining I and II we cannot get the direction of D, so data


in both statements I and II together are not sufficient to answer the
question.
45 Adda247 Publications For any detail, mail us at
Publications@adda247.com
Cracker Book for Bank (IBPS | SBI | RRB PO | Clerk) Mains Exams

35. (e); From statement II- L can be born on either 24 March or 8 June.
From statement I and II- When L is born on 24 March

When L is born on 8 June

But it is given in statement II that M does not born on 13 and N


does not born on 19 So, final arrangement from both statement I
and II-

36. (b); A sits third to the right of C, who sits second to the left of D. Only
two person sits between G and A. B is not the immediate neighbour
of C. F sits third to the right of B. C faces inside. B is not an
immediate neighbour of A.

A and G faces same direction. F sits second to the left of A. E sits


third to the right of F, and on the immediate left of B. E and B faces
same direction.

46 Adda247 Publications For any detail, mail us at


Publications@adda247.com
Cracker Book for Bank (IBPS | SBI | RRB PO | Clerk) Mains Exams

37. (e);

38. (b); Only statement II is sufficient to answer the question as from II


statement it is clear that H lives on 4th floor. Statement I is not
required to answer the question.
Floors Persons
8 K
7
6 I
5 N
4 H
3
2
1

39. (a); From Only statement I there will be two possible cases-----
Case 1-

Case 2-

From both possible cases it is clear that Q sits to the immediate


right of F. So only Statement I alone is sufficient to answer the
questions but Statement II alone is not sufficient to answer the
question.

47 Adda247 Publications For any detail, mail us at


Publications@adda247.com
Cracker Book for Bank (IBPS | SBI | RRB PO | Clerk) Mains Exams

40. (d); Statement I and II together are not sufficient to answer the
question as by combining both I and II together we get the code of
Benefit can be either mlp or hlt.

 Key Points to Remember for New Pattern Questions


➢ Today’s era is a race in which everybody wants to put
minimum efforts to get maximum output, Data sufficiency
is similar concept in which you have to find the minimum
information which is required to get the answer of your
question.
➢ The topic of data sufficiency is bit molded as seen in the
recent exams as most of its part is either in the form of a
puzzle or seating arrangement.
➢ While solving the data sufficiency questions based on
puzzles you have to draw separate arrangement from
every statement as you have to find the answer of your
question even if you are not be able to draw the complete
arrangement from that particular statement.
➢ If there are two possible solutions occur from one
statement or from all, then it will be an insufficient
information as we are not be able to get a single solution.
➢ Now a days the questions of data sufficiency as seen in the
exam are bit tricky and you have to be smart enough to
extract the required information from it.

48 Adda247 Publications For any detail, mail us at


Publications@adda247.com
Cracker Book for Bank (IBPS | SBI | RRB PO | Clerk) Mains Exams

Adda247 Publications For any detail, mail us at


1
Publications@adda247.com
Cracker Book for Bank (IBPS | SBI | RRB PO | Clerk) Mains Exams

Chapter

5 Coding-Decoding

BEST APPROACH TO SOLVE THE QUESTIONS

Coding decoding is the topic which shows continuous changes in it. In each
and every exam this topic has gone through various changes. So, to get
updated we are providing you some questions based on recent exams. By
solving these you may get to know about the latest pattern and you will be
able to prepare for the upcoming exams.

Example-1:
Directions (1-5): Study the information and answer the following
questions: (IBPS PO Mains-2016)
In a certain code language
"fresh mind happy life " is coded as " H#14 M@17 K#33 U@17 "
"kashi city of temple" is coded as " S#20 G@28 L@21 O@25"
"Sarnath belongs to varanasi" is coded as"G#27 T#21 G@35 H@31"
"One airport in banaras " is coded as "M#20 I#21 R@23 Z#21 "

1. What is the code for ‘Excellence’ in the given code language?


(a) X@10 (b) X#10 (c) E@10
(d) E#10 (e )None of these

2. What is the code for ‘University' in the given code language?


(a) G#46 (b) T@46 (c) N@46
(d) G@46 (e) None of these

3. What may be the possible code for ‘One army ’ in the given code
language?
(a) M#20 M@26 (b) M#20 T@26 (c) N@26 M#20
(d) M#20 N@26 (e) None of these

Adda247 Publications For any detail, mail us at


2
Publications@adda247.com
Cracker Book for Bank (IBPS | SBI | RRB PO | Clerk) Mains Exams

4. What may be the possible code for ‘Veracity’ in the given code
language?
(a) T@47 (b) G@47 (c) E@47
(d) G#47 (e) None of these

5. What is the code for ‘Ghats in Kashi' in the given code language?
(a) R@21 T#20 G#26 (b) G#26 R@23 S#20 (c) G@26 S#20 R@21
(d) G@26 S#20 R#21 (e) None of these

Explanations (The Approach):


These are the latest pattern of coding-decoding questions. In these
questions we are applying following concept:-

1. (a); X@10 2. (d); G@46 3. (d); M#20 N@26


4. (b); G@47 5. (b); G#26 R@23 S#20
Adda247 Publications For any detail, mail us at
3
Publications@adda247.com
Cracker Book for Bank (IBPS | SBI | RRB PO | Clerk) Mains Exams

Example-2:
Direction (1-5): Study the following information and answer the given
questions. (IBPS PO Mains-2017)
In alphabetical series A-Z each letter except vowels is assigned a different
number from 1-8 (for ex- B is coded as 1, C-2………..K-8)and again those
numbers get repeated(for ex- L-1, M-2……..so on).
Also each vowel is assigned a different symbol viz. #, $, %, @, &.
For example-
In coded language-
“She is girl” is coded as - 76% #7 5#61
“What did you like” is coded as - 26$8 3#3 4@& 1#8%
“It is Opinion” is coded as - #8 #7 @4#3#@3

(i) If both first and last letter of a word is vowel then the codes of both the
vowels are interchanged.
(ii) If first letter of a word is vowel and last letter is consonant then both
are to be coded as *.
(If the word does not satisfy the conditions given above then the letters
of that word are to be coded as per the directions given above)

1. What can be the code of ‘Nothing perfect’?


(a) 3@86#53 4%64%28 (b) 3@68#35 4%64%28
(c) 3@86#35 4%64%38 (d) 3@86#35 4%64%28
(e) None of these

2. What can be the code of ‘Exam was easy’?


(a) *3$* 2$7 *$7* (b) *3$* 2$7 %$74 (c) %3$2 2$7 *$7*
(d) *3$* 2$7 $74% (e) None of these

3. What can be the code of ‘Create style’?


(a) 2#6$8% 7841% (b) 26%$8% 7814% (c) 26%$8% 7841%
(d) 26%$6% 7841% (e) None of these
4. What can be the code of ‘Strength of god’?
(a) 768%3586 4@ 5@3 (b) 786%3586 ** 5@3(c) 786%3586 @* 5@3
(d) 786%3856 4@ 5@3(e) None of these

Explanations (The Approach):


Adda247 Publications For any detail, mail us at
4
Publications@adda247.com
Cracker Book for Bank (IBPS | SBI | RRB PO | Clerk) Mains Exams

Direction (1-4): In this new pattern coding decoding each letter, except
vowel, is assigned a number from 1-8 So, B-1, C-2, D-3, F-4, G-5, H-6, J-7, K-
8, L-1, M-2, N-3, P-4, Q-5, R-6, S-7, T-8, V-1, W-2, X-3, Y-4, Z-5.
Each vowel is assigned a different symbol as-%, #, $, @, &. So, for vowels
the symbols are - A-$, E-%, I-#, O- @, U-&.

1. (d);

2. (a); The code will be- *3$* 2$7 *$7*

3. (c);

4. (b); The code will be- ‘786%3586 ** 5@3’

Example 3:
Directions (1-3): Study the information carefully answers the questions
given below. (SBI PO Mains-2017)
@ means either hour hand or minute hand is at 8
# means either hour hand or minute hand is at 5
$ means either hour hand or minute hand is at 4
% means either hour hand or minute hand is at 12
& means either hour hand or minute hand is at 2
£ means either hour hand or minute hand is at 3
Note: if two symbols are given than by default first symbol is consider as
hour hand and second one is consider as minute hand. And all time are
consider at PM.

1. If A takes 25 min to reach railway station and his train is scheduled at


#& then at what time should he leave to reach the station 5 minute
earlier?
(a) $% (b) $& (c) &S
(d) $@ (e) £$
Adda247 Publications For any detail, mail us at
5
Publications@adda247.com
Cracker Book for Bank (IBPS | SBI | RRB PO | Clerk) Mains Exams

2. If a train departed from a station at &£ and it takes 2 hours to reach the
destination then when it will reach to the destination?
(a) $£ (b) $% (c) #$
(d) $# (e) £$

3. A person has to catch a train that is scheduled to depart at ‘@%’. It


takes the person 4 hours and 15 minutes to reach the railway station
from his home. At what time should he leave from his home for the
railway station to arrive at the station at least 25 minutes before the
departure of the train?
(a) %@ (b) £$ (c) %+
(d) +@ (e) None of these

Explanations (The Approach):


Directions (1-3):
1. (d); train is scheduled at= 5 hour 10 minute = #&
Time should he leave to reach the station 5 minute earlier= 5 hour
10 minute-(25+5) minute= 4 hour 40 minute

2. (a); 4 hour 15 minute

3. (b); Scheduled time of departure of train= @%= 8:00 PM


Time for travel+ early arrival= 4 hour 15 minutes + 25 minutes = 4
hour 40 minutes
8:00 PM- 4 hour 40 minutes= 3:20 PM= £$

Adda247 Publications For any detail, mail us at


6
Publications@adda247.com
Cracker Book for Bank (IBPS | SBI | RRB PO | Clerk) Mains Exams

Practice Exercise Based on new Pattern

Direction (1-3): Study the following information carefully and answer the
given questions:

In alphabetical series A-Z each letter except vowels is assigned a different


number from 1-5 (for ex- B is coded as 1, C-2………..G-5)and again those
numbers get repeated(for ex- H-1, J-2……..so on).
Also each vowel is assigned a different symbol viz. #, $, %, @, &.
In coded language-
“Solar Power Energy” is coded as – 5&4@4 2&3#4 #1#455
“Need to Change” is coded as – 1##3 1& 21@15#
“Less Economic Revenue” is coded as – 4#55 #2&1&5$2 4#2#1%#
Besides the above example, following operations are to be applied for
coding the words given in the questions below.

(i) If both first and last letter of a word is consonant then the codes of both
the consonant are interchanged.
(ii) If first letter of a word is vowel and last letter is consonant then both
are to be coded as *.
(If the word does not satisfy the conditions given above then the letters
of that word are to be coded as per the directions given above)

1. What can be the code of ‘Nuclear Bomb’?


(a) 4%24#@1 15&1 (b) 4%42#@1 1&51 (c) 4%24#@1 1&51
(d) 4%24@#1 1&51 (e) None of these

2. What can be the code of ‘War and Peace’?


(a) 3@4 *1* 2#@2# (b) 4@3 *1* 2#@2# (c) 4@3 @13 2#@2#
(d) 4@3 *1* 2#2@# (e) None of these

3. What can be the code of ‘Own life’?


(a) &3* 4$4# (b)*3* 44$# (c) &33 4$4#
(d)*3* 4$4# (e)None of these
Adda247 Publications For any detail, mail us at
7
Publications@adda247.com
Cracker Book for Bank (IBPS | SBI | RRB PO | Clerk) Mains Exams

Directions (4-5): Study the following information carefully to answer the


given questions. In a certain code language:-
“mount serious challenge has ended” is written as ‘ nu re ch is de’.
“inform wants credible alternative has ” is written as ‘ di lt wu is fu’.
“Has Serious Challenge Credible” is written as ‘re ch is di’.
“Credible alternative mount inform” is written as ‘ nu lt di fu’.

4. What is the code for ‘alternative’?


(a) fu (b) di (c) lt
(d) re (e) Either (a) and (c)

5. Which of the following words are coded as ‘Serious challenge has


ended’ ?
(a) de ch is wu (b) wu re ch is (c) re is de ch
(d) re is de di (e) None of these

Directions: (6-10): Study the information carefully and answer the


questions given below.
In a certain code language,
“country first casino resorts” is coded as “GL18 ER15 FZ9 GV12”
“policies may deterrent investors” is coded as “HL11 CZ22 IV11 IM10”
“filing process for small businesses” is coded as “FR1 GI12 CL15 EN7 JF9”

6. Which of the following could be the code for “Sanctuary”?


(a) IZ16 (b) DZ18 (c) HZ16
(d) EA18 (e) None of these

7. Which of the following words could be coded as ‘EL13’?


(a) Mountain (b) Courier (c) Power
(d) Banker (e) Police

8. Which of the following could be the code for “annual turnover”?


(a) FM6 HU10 (b) FM6 HF10 (c) KP13 LN13
(d) KP13 NL13 (e) None of these

9. Which of the following words could be coded as ‘GV12’?


(a) Beast (b) Lucky (c) Roller
(d) Returns (e) Ready
Adda247 Publications For any detail, mail us at
8
Publications@adda247.com
Cracker Book for Bank (IBPS | SBI | RRB PO | Clerk) Mains Exams

10. Which of the following could be the code for “Dairy Milk”?
(a) EZ10 DR10 (b) EZ20 DR9
(c) EZ10 DR17 (d) CL10 DR7
(e) EZ20 DR7

Directions (11-13): Study the following information carefully to answer


the given questions.
In a certain code language: -
“tell tale heart” is written as ‘V5M V5F I5U’
“raven black cat” is written as ‘S5O C1L D1U’
“premature burial” is written as ‘Q21F F7M’
“pit and pendulum” is written as ‘Q9U B1E F9N’

11. In the given coding language, which of the following will be the code for
“raven and cats”?
(a) S50 B1E D1U (b) Q9U B1E D1U
(c) C1L B1E D1U (d) S50 Z5T B1E
(e) None of these

12. In the given coding language, which of the following will be the code for
“dark cave”?
(a) D5R V5F (b) D6N Z5L (c) Z5L V5F
(d) Z4L V4L (e) None of these

13. Which of the following words could be coded as “T15E O9U”?


(a) Death Magnetic
(b) Sword Fish
(c) Dark Knight
(d) Sword Night
(e) Last Knight

Directions (14-15): Study the information and answer the following


questions:
In a certain code language
“Words Meaning Enhanced” is coded as “6&14 4@4 1&5”.
“Vowels Support Education” is coded as “3#12 1#18 9%15”
“Sometime Mentioned Tongue” is coded as “14$13 9$5 15&21”.
“Device Made Function” is coded as “8&15 1&3 8#4”.
Adda247 Publications For any detail, mail us at
9
Publications@adda247.com
Cracker Book for Bank (IBPS | SBI | RRB PO | Clerk) Mains Exams

14. What is the code for ‘supported'?


(a) 15#5 (b) 15&5 (c) 14$5
(d) 14&5 (e) None of these

15. What is the code for ‘Baking Powder'?


(a) 5#14 2#5 (b) 2#13 5#5
(c) 5#13 2%5 (d) 5&13 2#5
(e) None of these

Directions (16-20): These questions are based on the following


informations.
In a certain code:
‘severe cash crunch again’ is coded as ‘M*Z S@X V#H S#X’.
‘Finance Ministry has emphasis’ is coded as ‘B$N H&S H$V V©U’.
‘Reserve Bank India has claimed’ is coded as ‘Z*R V©I H&S W©X P@Y’.
16. What will be the code for ‘Emphasis’?
(a) V©U (b) H&S (c) B$N
(d) H$V (e) None of these

17. What will be the code for ‘Bankers’?


(a) Y©H (b) Y&B (c) H©Y
(d) B$Y (e) M35 C12

18. Which of the following will be the code for ‘Renews’?


(a) H#I (b) H&I (c) I#H
(d) I$O (e) None of these

19. Which of the following can be coded as ‘L*A’?


(a) Jaxes (b) Zumbo (c) Judge
(d) April (e) None of these

20. What could be the code for ‘Court has judges’?


(a) G*X H$S Q#H (b) G*X H$S H#Q (c) G@X H$S H#Q
(d) G*X H$S H@Q (e) G*X H&S H#Q
Adda247 Publications For any detail, mail us at
10
Publications@adda247.com
Cracker Book for Bank (IBPS | SBI | RRB PO | Clerk) Mains Exams

Direction (21-25): Study the following information and answer the given
questions:
In alphabetical series each consonant is assigned a different number from
1-7 (for ex- B is coded as 1, C-2……….J-7)and again those numbers get
repeated(for ex- K-1, L-2…….so on).
Besides the above information, following operations are to be applied for
coding the words given in the questions below.
Each letters of the given questions will be coded as per the given
conditions:
I. Vowels appearing before ‘M’ in the Alphabetical series will be coded as
‘**’.
II. Vowels appearing after ‘M’ in the alphabetical series will be coded as
‘$$’.
III. Number immediately preceded by vowel will be coded as ‘#1’.
IV. Number immediately followed by vowel will be coded as ‘@#’.

21. What will possibly be the code for ‘NORMAL’?


(a) ‘#1$$@##2**@# (b) ‘#1$$@##1**@# (c) ‘#3$$@##1**@#
(d) ‘#1$#@##1**@# (e) None of these

22. What will possibly be the code for ‘EMBARKS’?


(a) **@##1**@#12 (b) **@##1**@#31 (c) **@##1**@#41
(d) **@##1**@#11 (e) None of these

23. What will possibly be the code for ‘SMITTLE’?


(a) 1#1**@#2#1** (b) 1#1**@#2#11* (c) 1#1**@#2#1*1
(d) 2#1**@#2#1** (e) None of these

24. What will possibly be the code of ‘ANNUAL’?


(a) **@##1$$**@# (b) *1@##1$$**@# (c) **##@1$$**@#
(d) *2@##1$$**@# (e) None of these

25. What is the code for ‘PROM’?


(a) 5#1$$@# (b) 2#1$$@# (c) 4#1$$@#
(d) 6#1$$@# (e) None of these

Adda247 Publications For any detail, mail us at


11
Publications@adda247.com
Cracker Book for Bank (IBPS | SBI | RRB PO | Clerk) Mains Exams

Directions (26-30): Study the information and answer the following


questions:
In a certain code language
“Seemed peer attend” is coded as “18BV 20BZ 19CW”.
“Arrive Assessing file” is coded as “22BZ 19DZ 12RV”
“Double systems possible” is coded as “19BY 21LY 25CV”.

26. What is the code for ‘support'?


(a) 22DL (b) 21CL (c) 21BL
(d) 22BL (e) None of these

27. What is the code for ‘asking'?


(a) 19ZH (b) 20ZH (c) 21XH
(d) 19HX (e) 19HZ

28. What is the code for ‘Announced'?


(a) 21CZ (b) 21ZC (c) 23BZ
(d) 22CZ (e) None of these

29. What is the code for ‘Less’?


(a) 20BZ (b) 19BV (c) 21CV
(d) 19BZ (e) None of these

30. What is the code for ‘peer loom'?


(a) 15CO 18BC (b) 15BO 18BV
(c) 15BO 18CV (d) 18CO 15BV
(e) None of these

Direction (31-35): Study the following information and answer the given
questions:
In alphabetical series A-Z each letter except vowels is assigned a different
number from 1-6 (for ex- B is coded as 1, C-2………..H-6)and again those
numbers get repeated(for ex- J-1, K-2……..so on).
Also each vowel is coded with different letters viz. g, w, q, c, k.
In coded language-

Adda247 Publications For any detail, mail us at


12
Publications@adda247.com
Cracker Book for Bank (IBPS | SBI | RRB PO | Clerk) Mains Exams

“Election Live” is coded as – g3g24kq5 3k5g


“start new” is coded as – 34c24 5g6
“under process” is coded as – w53g2 62q2g33
Besides the above example, following operations are to be applied for
coding the words given in the questions below.

(i) If first letter is consonant and last letter is vowel then the codes of both
of them will be interchanged.
(ii) If both first and last letter are vowel then they are to be coded as $.
(iii) If first letter of a word is vowel and last letter is consonant then both
are to be coded as @.
(iv) If both first and last letter are consonant then they are to be coded as &.
(If the word does not satisfy the conditions given above then the letters
of that word are to be coded as per the directions given above)

31. What can be the code of ‘global’?


(a) 53q1c& (b) 53q1c3 (c) &q31c&
(d) &3q1c& (e) None of these

32. What can be the code of ‘Title’?


(a) gk434 (b) gk344 (c) 4k43g
(d) kg434 (e) None of these

33. What can be the code of ‘Adviser’?


(a) c35k3g2 (b) c35k3g@
(c) @35k3g@ (d) c53k3g2
(e)None of these

34. What can be the code of ‘Insurance’?


(a) k53w2c52c (b) $53w25c2$
(c) $35w2c52$ (d) $53w2c52$
(e)None of these

35. What can be the code of ‘Marine’?


(a) gc2sk4 (b) gc2k54 (c) 4c2k54
(d) 4c2k5g (e)None of these

Adda247 Publications For any detail, mail us at


13
Publications@adda247.com
Cracker Book for Bank (IBPS | SBI | RRB PO | Clerk) Mains Exams

Directions (36-38): Answer these questions based on the following


information.
In a certain code:
“ review time slot” is coded as - “e%5 o#11 a@27”
“moment question answer” is coded as - “g#29 c&18 q#19”
“tension paper mailing” is coded as - “f%10 b#3 f%15”
“ design layout home” is coded as - “j#6 h%11 c#14”

36. What is the code for “purchase railway”?


(a) k%20 g%1 (b) b#13 h@35 (c) b@13 u%12
(d) k%20 g%2 (e) none of these

37. What is the code for “crosscheck”?


(a) g#21 (b) h#21 (c) p%16
(d) b@14 (e) None of these

38. Which among the following may be coded as “q#9”?


(a) assets (b) recover (c) sector
(d) connect (e) none of these

Directions (39-40): Triangle represents (1) and circle represents (0). If


triangle appears in unit's place, then its value is 1. If it appears in 10's place
its value is doubled to 2 like that it continues. Questions based on this
For example:

39. How will you represent ‘11’ in this code language?


(a) (b)
(c) (d)
(e)
40. What will be the code for ?
(a) 19 (b) 23 (c) 22
(d) 27 (e) 25
Adda247 Publications For any detail, mail us at
14
Publications@adda247.com
Cracker Book for Bank (IBPS | SBI | RRB PO | Clerk) Mains Exams

Solutions

Solutins (1-3): In this new pattern coding decoding each letter, except
vowel, is assigned a number from 1-5 So, B-1, C-2, D-3, F-4, G-5, H-1, J-2, K-
3, L-4, M-5, N-1, P-2, Q-3, R-4, S-5, T-1, V-2, W-3, X-4, Y-5, Z-1.
Each vowel is assigned a different symbol as-%, #, $, @, &. So, for vowels
the symbols are - A-@, E-#, I-$, O- &, U-%.

1. (c); Nuclear Bomb – Condition (i) applied- 4%24#@1 1&51

2. (b); War – Condition (i) applied-4@3


And- Condition (ii) applied- *1*
Peace- No condition applied- 2#@2#

3. (d); Own- Condition (ii) applied- *3*


Life- No Condition applied- 4$4#

Solutions (4-5):
WORDS CODES
Mount nu
Has is
Serious/Challenge re/ch
Credible di
Alternative Lt/fu
Wants Wu
Inform Fu/lt
Ended de

4. (e); 5. (c);

Solutions (6-10): This coding decoding question is based on the latest


pattern, following logic are applied in this question to decode the code.
(i) The first letter of the code represents an alphabet which has the same
place value as the number of letters in the given word.
(ii) The second letter of the code is the reverse of the second letter of the
word(A-Z, B-Y…..)
Adda247 Publications For any detail, mail us at
15
Publications@adda247.com
Cracker Book for Bank (IBPS | SBI | RRB PO | Clerk) Mains Exams

(iii) The number at the end of the code is the difference between the place
value of the last letter of the word and the total number of letters in the
word.

6. (a); 7. (c); 8. (b);


9. (d); 10. (e);

Solutions (11-13): This is question of Coding-Decoding based on new


pattern. In these questions, following logic is applied to decode the code:-
If the total number of letters in the word is even,
1st letter of the code:- Reverse(A-Z, B-Y….) of the greatest(according to the
alphabetical series) vowel in the word.
2nd letter of the code:- Total number of letters in the word + 1
3rd letter of the code:- Next letter(according to the alphabetical series) of
the last letter of the word.

If the total number of letters in the word is odd,


1st letter of the code:- Next letter(according to the alphabetical series) of the
first letter of the word.
2nd letter of the code:- Rank/Place value of the greatest vowel in the word.
3rd letter of the code:- Next letter(according to the alphabetical series) of
the last letter of the word.

Adda247 Publications For any detail, mail us at


16
Publications@adda247.com
Cracker Book for Bank (IBPS | SBI | RRB PO | Clerk) Mains Exams

11. (d); 12. (c); 13. (d);

Solutions (14-15): Let us understand the logic behind the given coding
decoding:
For the I digit of the code – I Number in the code will be the difference of
the of place value of the first letter and last letter present in the word.
For the symbol
For different number of vowels present in the word, the code will be
accordingly assigned to it.

No. of vowels in the word Code


1 @
2 #
3 &
4 $
5 %

For the last digit of the code – The place value of the second last letter
present in the word.

14. (b); 15. (a);

Solutions (16-20): The given words are coded as per following pattern:
(i) First letter of the code represents the opposite letter of the last letter of
the given word.
For example. Again- ‘N’=M
(ii) Last letter of the code represents the opposite letter of the first letter of
the given word.
Adda247 Publications For any detail, mail us at
17
Publications@adda247.com
Cracker Book for Bank (IBPS | SBI | RRB PO | Clerk) Mains Exams

For example. Again- ‘A’=Z


(iii) The symbol of the code is depending on the total number of letters in
the given word.
No. of letters - Symbol
3-&
4-@
5-*
6-#
7–©
8-$
For ex. Again – The code is ‘M*Z’.

16. (d); 17. (c); 18. (a);


19. (b); 20. (e);

Solutions (21-25):
Logic: The different number codes for all the consonant as per the given
condition are,

Step 1: The consonants of the word ‘NORMAL’ are to be coded as the


number allotted to the consonant:

Step 2: The numbers immediately preceded and followed by the vowels are
to be coded as per the given conditions;
So, the code for consonant for word ‘NORMAL’ is coded as ‘4O73A2’,
numbers 4 and 7 is immediately followed and preceded
respectively by ‘O’ so, ‘4’ is coded as ‘#1’ and ‘7’ is coded as ‘@#’.
Similarly, ‘3’ and ‘2’ is immediately followed and preceded
respectively by ‘A’ so, ‘3’ is coded as ‘#1’ and ‘2’ is coded as ‘@#’.

Adda247 Publications For any detail, mail us at


18
Publications@adda247.com
Cracker Book for Bank (IBPS | SBI | RRB PO | Clerk) Mains Exams

Step 3: Now the vowels are to be coded as per the given conditions, as ‘O’
comes after ‘M’ in the alphabetical series so ‘O’ is coded as ‘$$’ and
‘A’ comes before ‘M’ in the alphabetical series so, ‘A’ is to be coded
as ‘**’.

So, the final code for the word ‘NORMAL’ is ‘#1$$@##1**@#’.

21. (b); Therefore, the code for the word ‘NORMAL’ is ‘#1$$@##1**@#’.

22. (d) Step 1: The consonants of the word ‘EMBARKS’ are to be coded as
the number allotted to the consonant:

Step 2: The numbers immediately preceded and followed by the


vowels are to be coded as per the given conditions;
So, the code for consonant for word ‘EMBARKS’ is coded as
‘E31A711’, number ‘3’ is followed by vowel so the code for ‘3’ is
‘@#’ and numbers ‘1’ and ‘7’ is immediately preceded and followed
respectively by ‘A’ so, ‘1’ is coded as ‘#1’ and ‘7’ is coded as ‘@#’.
But the numeric code of ‘K’ and ‘S’ is ‘1’ is neither followed by nor
preceded by any vowel. Hence, there code will remain the same.

Adda247 Publications For any detail, mail us at


19
Publications@adda247.com
Cracker Book for Bank (IBPS | SBI | RRB PO | Clerk) Mains Exams

Step 3: Now the vowels are to be coded as per the given conditions,
as ‘E’ comes before ‘M’ in the alphabetical series so ‘E’ is coded as ‘**’
and ‘A’ comes before ‘M’ in the alphabetical series so, ‘A’ is to be
coded as ‘**’.

So, the final code for the word ‘EMBARKS’ is ‘**@##1**@#11’.

23. (a); The code for ‘’SMITTLE’ is ‘1#1**@#2#1**’.

24. (a); The code for ‘ANNUAL’ is ‘**@##1$$**@#’.

25. (a);

Solutions (26-30):
Let us understand the logic behind the given coding decoding:
For the digit of the code – Number in the code will be the place value of the
highest place value of letter present in the word.
For the first letter of the code-

Adda247 Publications For any detail, mail us at


20
Publications@adda247.com
Cracker Book for Bank (IBPS | SBI | RRB PO | Clerk) Mains Exams

Case-1
If the given word has some common letter then the code will be according
to the given order:

No. of common letter in the word Code


2 B
3 C
4 D

For ex: Arrive -In this word two common letters ‘r’ and in Assessing has
four common letters ‘s’. So the first letter of code for Arrive is ‘B’
and for Assessing is ‘D’.

Case II
If the given word has no common letter then first letter of the code will be
coded as opposite letter of the second letter of the word.
For the last letter of the code – The opposite letter of the smallest place
value letter present in the word according to the English alphabet.

26. (c); 27. (e); 28. (a);


29. (b); 30. (b);

Direction (31-35):
In this new pattern coding decoding each letter, except vowel, is assigned a
number from 1-6 So, B-1, C-2, D-3, F-4, G-5, H-6, J-1, K-2, L-3, M-4, N-5, P-6,
Q-1, R-2, S-3, T-4, V-5, W-6, X-1, Y-2, Z-3.
Also each vowel is assigned different letters. So, for vowels the letters are -
A-c, E-g, I-k, O-q, U-w.

31. (d); global- Condition (iv) applied- &3q1c&

32. (a); Title- Condition (i) applied- gk434

33. (c); Adviser- Condition (iii) applied- @35k3g@

34. (d); Insurance- Condition (ii) applied $53w2c52$


Adda247 Publications For any detail, mail us at
21
Publications@adda247.com
Cracker Book for Bank (IBPS | SBI | RRB PO | Clerk) Mains Exams

35. (b); Marine- Condition (i) applied- gc2k54

Solutions (36-38):

36. (a); “purchase railway” ------ “k%20 g%1”

37. (b); “crosscheck” ------ “h#21”

38. (d); “q#9” --- “connect”

Solutions (39-40):
39. (e); As given in the Directions
If triangle appears in unit's place then its value is 1. If it appears in
10's place its value is doubled to 2 like that it continues.
Then to represent the 11 by 2^3 0^2 2^1 2^0
= 8+0+2+1 = 11

Adda247 Publications For any detail, mail us at


22
Publications@adda247.com
Cracker Book for Bank (IBPS | SBI | RRB PO | Clerk) Mains Exams

40. (b); As given in the Directions If triangle appears in unit's place then its
value is 1. If it appears in 10's place its value is doubled to 2 like
that it continues
= 2^4 0^3 2^2 2^1 2^0
=16+0+4+2+1 = 23

Key Points to Remember for New Pattern Questions


➢ To expect the unexpected shows a thoroughly modern
intellect, it completely goes with the coding decoding now-
a-days.
➢ The coding decoding has changed completely as its pattern
has gone through lots of changes and the logic behind it, is
always a mystery to go around with. Just go through the
given pattern completely otherwise there will be a chance of
ambiguity in finding the logic.
➢ Recent exam consist of different logics as you may see that
the logic varies as per the vowels or consonants present in
the word.
➢ Sometimes the logic goes around the first or last letter of the
word so try to initiate accordingly.
➢ It is not necessary that all the words present in the word
may follow same logic as it may varies according to the
number of letters present in word as if it is odd one then
there may be a different logic for it as it is for the even one.

Adda247 Publications For any detail, mail us at


23
Publications@adda247.com
Cracker Book for Bank (IBPS | SBI | RRB PO | Clerk) Mains Exams

1 Adda247 Publications For any detail, mail us at


Publications@adda247.com
Cracker Book for Bank (IBPS | SBI | RRB PO | Clerk) Mains Exams

Chapter

6 Syllogism

BEST APPROACH TO SOLVE THE QUESTIONS


Every day changing pattern of examination makes us realize that any
topic has its own importance and you may see some new type of questions
introduced in it at any time, the same goes with the syllogs. We have seen
some changes that has already happed as reverse syllogism, does not follow
and so on. And in future exams also we may see some changes as some
coded syllogism or any thing new. Here are some glimpse of the already
seen syllogism question in the given below example.

Example-1:
In each of the questions below are given three statements followed by three
conclusions number I, II and III. You have to take the given statements to be
true even if they seem to be at variance from commonly known facts. Read
the entire conclusion and then decide which of the given conclusions
logically follows from the given statements disregarding commonly known
facts. (IBPS Clerk Mains 2017)

1. Statements: All pigs are Zebra.


All Zebra are Rabbits.
No Ant is are Pig.
Conclusions: I. Some rabbit is Ant.
II. Some Zebra are Ant is a possibility.
III. Some Ant are Pig.
(a) Only I follows (b) Only II follows (c) Only III follows
(d) All follow (e) None of these

2. Statements: Some bat are pen.


Some pen are Shirt.
No shirt is Coat.
Conclusions: I. Some Coat is Pen.
II. Some Coat is shirt is a possibility.
III. All pen can be Coat
(a) None follows (b) only II follows (c) only III follows
(d) only I follows (e) only either II or III follow
2 Adda247 Publications For any detail, mail us at
Publications@adda247.com
Cracker Book for Bank (IBPS | SBI | RRB PO | Clerk) Mains Exams

3. Statements: All Men are Rock.


Some men are pearl.
No Rock is Angel.
Conclusions: I. Some men are not Angel.
II. All Pearl can be Angel is a possibility.
III. Some Angel can be Pearl is a possibility.
(a) only I and II follow (b) only I and III follow
(c) only II and III follow (d) All I, II and III follow
(e) None of these

1. (b);

2. (a);

3. (b);

Example-2:
In each question below are given three statements followed by four
conclusions numbered I, II, III and IV. You have to take the given statements
to be true even if they seem to be at variance with commonly known facts.
Read all the conclusions and then decide which of the given conclusions
logically follows from the given statements, disregarding commonly known
facts. (RBI Grade-B 2016)

3 Adda247 Publications For any detail, mail us at


Publications@adda247.com
Cracker Book for Bank (IBPS | SBI | RRB PO | Clerk) Mains Exams

1. Statements: A. No spoons are plates


B. All plates are bowls
C. All bowls are utensils
Conclusions: I. No spoons are bowls
II. No spoons are utensils
III. Some bowls are not plates
IV. Some utensils are not plates
(a) Only I follow (b) Only III follows (c) Only IV follows
(d) None follows (e) None of these

2. Statements: A. Some roses are leaves


B. Some leaves are plants
C. Some plants are flowers
Conclusions: I. No roses are flowers
II. No plants are roses
III. Some leaves are flowers
IV. Some plants are roses
(a) None follows (b) Only I and II follow
(c) Only II and III follow (d) All follow (e) None of these

3. Statements: A. Some sweets are chocolates


B. All chocolates are fruits
C. Some fruits are vegetables
Conclusions: I. Some sweets are vegetables
II. Some chocolates are vegetables
III. Some fruits are not sweets
IV. Some fruits are not vegetables
(a) Only I follow (b) Only II follows (c) Only III follows
(d) None follows (e) None of these

4 Adda247 Publications For any detail, mail us at


Publications@adda247.com
Cracker Book for Bank (IBPS | SBI | RRB PO | Clerk) Mains Exams

4. Statements: A. No reynolds are jetters


B. No jetters are finegrips
C. All finegrips are pens
Conclusions: I. No jetters are reynolds
II. Some jetters are not finegrips
III. Some finegrips are pens
IV. Some pens are finegrips
(a) All follows
(b) Only I, II and III follow
(c) Only II, III and IV follow
(d) Only I, II and IV follow
(e) None of these

5. Statements: A. Some officers are gentlemans


B. All graduates are officers
C. All intelligents are graduates
Conclusions: I. Some gentlemans are officers
II. All intelligents are officers
III. Some officers are not intelligents
IV. All officers are intelligents
(a) Only I, II and III follow (b) Only I, II and IV follow
(c) Only I and II follow (d) Only I and III follow
(e) None of these

1. (d);

5 Adda247 Publications For any detail, mail us at


Publications@adda247.com
Cracker Book for Bank (IBPS | SBI | RRB PO | Clerk) Mains Exams

2. (e);

3. (d);

4. (a);

5. (e);

6 Adda247 Publications For any detail, mail us at


Publications@adda247.com
Cracker Book for Bank (IBPS | SBI | RRB PO | Clerk) Mains Exams

Practice Exercise Based on new Pattern

Direction (1-3): In the following questions, the symbols #, @, $ and © are


used with the following meanings as illustrated below. Study the following
information and answer the given questions. In each of the questions given
below statements are followed by some conclusions. You have to take the
given statements to be true even if they seem to be at variance from
commonly known facts. Read all the conclusions and then decide which of
the given conclusions logically does not follows from the given statements
disregarding commonly known facts.
P#Q – All P is Q.
P@Q - Some Q is P.
P©Q – No P is Q.
P$Q – Some P is not Q.

1. Statements: Rat @ Cat # Dog © Fish @ Goat $ Hen


Conclusions: (a) Rat @ Dog (b) Goat @ Fish
(c) Cat © Fish (d) Rat $ Fish
(e) Fish $ Hen
2. Statements: Air @ Car # Drive $ Exit © Ride @ Air
(a) Air $ Exit (b) Drive @ Air
(c) Car @ Ride (d) Car @ Drive
(e) Exit $ Ride

3. Statements: Rubber © Pencil # Eraser @ Scale # Copy $ Book


Conclusions: (a) Scale @ Eraser (b) Eraser @ Copy
(c) Eraser @ Pencil (d) Eraser $ Rubber
(e) Rubber $ Book

Directions (4-5): In each group of questions below are two conclusions


followed by five set of statements. You have to choose the correct set of
statements that logically satisfies given conclusions. Given statements to be
true even if they seem to be at variance from commonly known facts.

7 Adda247 Publications For any detail, mail us at


Publications@adda247.com
Cracker Book for Bank (IBPS | SBI | RRB PO | Clerk) Mains Exams

4. Conclusion: At least some Caps being Pens is a possibility.


Some Copies are Caps.
Statements:
Statements-1: All Caps are Copies. All Caps are Papers. No Cap is a Pen.
Statements-2: All Caps are Copies. All Copies are Papers. No Paper is a
Pen.
Statements-3: Some Caps are Copies. Some Caps are Papers. No Cap is
a Pen.
Statements-4: Some Caps are Copies. All Copies are Papers. No Cap is a
Pen.
Statements-5: All Caps are Copies. Some Caps are Papers. No Paper is a
Pen.
(a) Only Statements – 1
(b) Both Statements – 2 and 4
(c) Only Statements – 3
(d) Both Statements – 3 and 5
(e) Only Statements – 5

5. Conclusion: Some Reds being Black is a possibility


Some Blue being Red is a possibility
Statements:
Statements-1: All Black are Pink. Some Red are Pink. No Blue is Red.
Statements-2: All Black are Pink. No Red is Pink. All Blue are Black.
Statements-3: Some Black are Pink. Some Red are Pink. No Blue is Red.
Statements-4: Some Black are Pink. Some Red are Pink. No Pink is
Blue.
Statements-5: Some Black are Pink. Some Red are Pink. All Blues are
Black.
(a) Only Statements – 1
(b) Both Statements – 2 and 3
(c) Only Statements – 4
(d) Both Statements – 4 and 5
(e) Only Statements – 5
8 Adda247 Publications For any detail, mail us at
Publications@adda247.com
Cracker Book for Bank (IBPS | SBI | RRB PO | Clerk) Mains Exams

Directions (6-8): In each of the questions below. Some statements are


given followed by conclusions/group of conclusions. You have to assume all
the statements to be true even if they seem to be at variance from the
commonly known facts and then decide which of the given conclusions
logically does not follows from the information given in the statements.
6. Statements: Some air is ball No ball is cat
Some cat is dog All dog is egg.
Conclusions: (a) some dogs are not ball.
(b) some egg are not ball
(c)All air is egg is a possibility.
(d) All egg can be ball.
(e) some air is dog is a possibility.

7. Statements: All watch is clock All clock is table


No watch is pen No clock is chair.
Conclusions: (a) Some table are watch
(b) Some table can be chair.
(c) No watch is chair
(d) No pen is chair.
(e) some table is not pen.

8. Statements: All plastic is rubber Some plastic is Teflon.


Some Teflon is cotton.
All cotton is woolen.
Conclusions: (a) some woolen is Teflon.
(b) No plastic is woolen is a possibility.
(c) All Teflon can be woolen.
(d) some Teflon is rubber.
(e) All rubber is plastic.

9 Adda247 Publications For any detail, mail us at


Publications@adda247.com
Cracker Book for Bank (IBPS | SBI | RRB PO | Clerk) Mains Exams

Directions (9-10): In each group of questions below are two conclusions


followed by five set of statements. You have to choose the correct set of
statements that logically satisfies given conclusions. Given statements to be
true even if they seem to be at variance from commonly known facts.

9. Conclusion-Some Students are Teachers. Some Principal are not


Students.
(a) Statement- No Study is School. All Teachers is Students. Some
Teachers is Study. Some School are Principal.
(b) Statement- No School is Study. Some Teachers is Students. Some
Teachers is Study. Some School are Principal.
(c) Statement- All Students is Teachers. Some Teachers is School. Some
Study are Principal. Some School is Study.
(d) Statement- All Students is Teachers. Some Teachers is Study. No
Students is School. Some School are Principal.
(e) Statement- Some Teachers is Students. No Teachers is School.
Some School is Study. Some School are Principal.

10. Conclusion: Some Enjoy is not Comedy. Some Fun are Circus.
(a) Statement: Some Joke is Enjoy. No Enjoy is Fun. Some Fun is Circus.
Some Joke are Comedy.
(b) Statement - All Fun is Circus. No Circus is Comedy. Some Joke are
Comedy. Some Joke is Enjoy.
(c) Statement - All Comedy is Circus. Some Joke are Circus. All Joke is
Fun. No Enjoy is Fun.
(d) Statement - No Circus is Joke. Some Joke are Fun. Some Fun is
Circus. Some Enjoy are Comedy.
(e) Statement -Some Joke is Enjoy. Some Enjoy are Circus. All Circus
are Fun. All Comedy are not fun.

Directions (11-13): This question consists of four statements followed by


five conclusions. Consider the given statements to be true even if they seem
to be at variance with commonly known facts. Read all the conclusions and
then decide which of the given conclusions does not logically follow from
the given statements using all statements together.

10 Adda247 Publications For any detail, mail us at


Publications@adda247.com
Cracker Book for Bank (IBPS | SBI | RRB PO | Clerk) Mains Exams

11. Statements: Some Pens are Papers.


All copies are Pencil
No Papers is a pencil
All pencil is Inks.
Conclusions: (a) Some Copies are Inks.
(b) All Inks being Pens is a possibility.
(c) At least some Pens are not Copies.
(d) All Papers can be Inks.
(e) Some Pens are Copies.

12. Statements: Some Book are Note.


All Book are Stationery.
All Note are Stationery.
No Book are Test.
Conclusions: (a) Some Stationery are not Test.
(b) All Test being Stationery is a possibility.
(c) No Test is Stationery.
(d) No Test being Stationery is a possibility.
(e) Some Note are not Test.
13. Statements: All blades are sharp.
Some blades are blunt.
No blunt is a sword.
Some sword are sharp.
Conclusions: (a) Some sharp are not blunt
(b) All blunt being sharp is a possibility.
(c) some blades are not sword.
(d) All sword can be blade.
(e) All sharp can be blunt.
Directions (14-15): In each of the questions below are given three
statements followed by three conclusions number I, II and III. You have to
take the given statements to be true even if they seem to be at variance
from commonly known facts. Read the entire conclusion and then decide
which of the given conclusions logically follows from the given statements
disregarding commonly known facts.

11 Adda247 Publications For any detail, mail us at


Publications@adda247.com
Cracker Book for Bank (IBPS | SBI | RRB PO | Clerk) Mains Exams

14. Statements: All PO are graduate.


All clerk are PO.
Some clerk are intermediate.
Conclusions: I. Some clerk are graduate.
II. Some intermediate are graduate is a possibility.
III. All intermediate can be graduate
(a) only I and II follow (b) only I and III follow
(c) only II and III follows (d) All I, II and III follows
(e) None of these

15. Statements: Some Hotels are 5-Star.


No 5-Star are Cheap.
Some Cheap are luxurious.
Conclusions: I. Some luxurious are hotel.
II. No hotel are luxurious.
III. All hotel are luxurious.
(a) None follows (b) only II follows
(c) only III follows (d) only I follows
(e) only either I or II follow

Direction (16-18): In the following questions, the symbols *, +, - and / are


used with the following meanings as illustrated below. Study the following
information and answer the given questions. In each of the questions given
below statements are followed by some conclusions. You have to take the
given statements to be true even if they seem to be at variance from
commonly known facts. Read all the conclusions and then decide which of
the given conclusions logically does not follows from the given statements
disregarding commonly known facts.
A*B – All A is B.
A+B - Some B is A.
A/B – No B is A.
A-B – Some A is not B.
?- possibility case
Note: if ? is placed after any of the symbols mentioned above then it will be
considered as possibility case of the symbol. E.g. A+?B means some B being
A is a possibility.

12 Adda247 Publications For any detail, mail us at


Publications@adda247.com
Cracker Book for Bank (IBPS | SBI | RRB PO | Clerk) Mains Exams

16. Statements: mall+ plaza* market- complex


Conclusions: (a) mall- complex
(b) mall+ market
(c) market+ plaza
(d) complex+? market
(e) plaza-? complex

17. Statements: road/traffic/light*crossing


Conclusions: (a) road+? light
(b) crossing- light
(c) traffic*? crossing
(d) road+? crossing
(e) crossing-traffic

18. Statements: pen+ eraser* paper; pen*pencil


Conclusions: (a) paper+? pencil
(b) paper+ pencil
(c) pencil*? paper
(d) pencil+ eraser
(e) paper+ pen

Directions (19-20): In each group of questions below are two conclusions


followed by five set of statements. You have to choose the correct set of
statements that logically satisfies given conclusions. Given statements to be
true even if they seem to be at variance from commonly known facts.

19. Conclusion-Some Light are Dark. Some Dull are not Light.
Statement I- No Bright is Glow. All Dark is Light. Some Dark is
Bright. Some Glow are Dull.
Statement II- No Glow is Bright. All Dark is Light. Some Dark is Glow.
Some Light are Dull.
Statement III- All Light is Dark. Some Dark is Glow. Some Bright are
Dull. Some Glow is Bright.
Statement IV- All Light is Dark. Some Dark is Bright. No Light is Glow.
Some Glow are Dull.
Statement V- Some Dark is Light. No Dark is Glow. Some Glow is
Bright. Some Glow are Dull.
13 Adda247 Publications For any detail, mail us at
Publications@adda247.com
Cracker Book for Bank (IBPS | SBI | RRB PO | Clerk) Mains Exams

20. Conclusion-Some Above not being Upper is a possibility. Some Below


are Odd.
Statement I- Some Above is Below. No Below is Even. Some Even is
Odd. Some Above are Upper.
Statement II- All Even is Odd. No Odd is Upper. Some Above are
Upper. Some Above is Below.
Statement III- All Upper is Odd. Some Above are Odd. All Above is
Even. No Below is Even.
Statement IV- No Odd is Above. Some Above are Even. Some Even is
Odd. Some Below are Upper.
Statement V- Some Above is Below. Some Below are Odd. All Odd are
Even. All Upper are not even.

Direction (21-22): In each group of questions below are some conclusions


followed by five set of statements. You have to choose the correct set of
statements that logically satisfies given conclusions. Given statements to be
true even if they seem to be at variance from commonly known facts.

21. Conclusion: Some Science are not English. Some Reasoning can be
Computer.
Statements:
(a) All Computer are Math. No Math is reasoning. Some Reasoning are
English. Some Science are Computer.
(b) Some Math are English. No English is Computer. No Computer is
Reasoning. Some Reasoning is Science.
(c) No Computer is Science. Some Science are English. No English is
Reasoning. Some Reasoning is Math.
(d) Some Computer are Science. All Science is Math. No Math are
English. Some English are Reasoning.
(e) All English are Math. No Math is Reasoning. Some Reasoning are
Computer. Some Science are Computer.

14 Adda247 Publications For any detail, mail us at


Publications@adda247.com
Cracker Book for Bank (IBPS | SBI | RRB PO | Clerk) Mains Exams

22. Conclusion: Some Physics can never be Geography. Some Biology are
not Geography.
Statements:
(a) All Biology are Geography. No Geography is History. Some History
are Physics. Some Chemistry are Biology.
(b) Some Biology are Chemistry. No Geography is History. Some
History are Physics. All Physics are Biology.
(c) No Biology is Chemistry. Some Chemistry are Physics. No Physics
is History. Some History is Geography.
(d) Some Biology are Geography. All Geography is History. No History
are Physics. Some Chemistry are Biology.
(e) Some Geography are Physics. No Physics is Biology. No Biology is
History. Some History is Chemistry.

Directions (23-25): In each question below are given some statements


followed by some conclusions numbered I, II, III and IV. You have to take
the given statements to be true even if they seem to be at variance with
commonly known facts. Read all the conclusions and then decide which of
the given conclusions definitely does not logically follow from the given
statements, disregarding commonly known facts. Give answer

23. Statements: Some Pepsi are Cocacola.


No Cocacola is a Mirinda.
All Mirinda are Dew.
Conclusions: I. Some Cocacola not being Mirinda is a possibility.
II. Some Dew are not Cocacola.
III. All Pepsi are Mirinda.
IV. Some Pepsi can be Dew.
(a) Only I & II do not follow
(b) Only III & IV do not follow
(c) Only III does not follow
(d) All follow except IV
(e) None of these
15 Adda247 Publications For any detail, mail us at
Publications@adda247.com
Cracker Book for Bank (IBPS | SBI | RRB PO | Clerk) Mains Exams

24. Statements: All Puma are Reebok.


All Adidas are Reebok.
Some Adidas are Woodland.
Conclusions: I. No Puma is Adidas.
II. Some Reebok being Woodland is a possibility.
III. Some Reebok are Puma.
IV. Some Reebok are Adidas.
(a) Only I & III do not follow
(b) Only II does not follow
(c) All follow except III
(d) Only II & I do not follow
(e) All follow except I
25. Statements: Some tibbet are China
All china are nepal.
No nepal is bhutan.
Conclusions: I. Some bhutans are not tibbet.
II. All tibbet being nepal is a possibility.
III. At least some bhutans are nepal.
IV. All china can be nepal.
(a) II & IV do not follow
(b) I & IV do not follow
(c) I & III do not follow
(d) II & III do not follow
(e) None of these

Directions (26-28): In each of the questions below are given some


statements followed by some conclusions. You have to take the given
statements to be true even if they seem to be at variance from commonly
known facts. Read the entire conclusion and then decide which of the given
conclusions logically follows from the given statements disregarding
commonly known facts.

26. Statements: No Boys are Girls.


All Boys are Graduate
All Girls are graduate
Some Girls are IAS
16 Adda247 Publications For any detail, mail us at
Publications@adda247.com
Cracker Book for Bank (IBPS | SBI | RRB PO | Clerk) Mains Exams

Conclusions: I. Some Boys are not IAS.


II. All IAS can be Boys.
III.All Graduate being IAS ia a possibility.
(a) Only I follows
(b) Only II follows
(c) Only III follows
(d) All follow
(e) None of these

27. Statements: All milk is water.


Some milk is Tea.
No Coffee is Tea.
Conclusions: I. Some milk is Coffee.
II. No milk is Coffee
III. All milk being coffee is a possibility.
(a) None follows
(b) only II follows
(c) only III and either I or II follow
(d) only I follows
(e) only either I or II follow

28. Statements: Some Pant are Shirt.


Some Shirt are Suit.
No Suit is Button.
Conclusions: I. Some shirt is not Button.
II. Some Pant is Button.
III. Some Buttton is not Shirt
(a) Only I follows (b) Only II follows (c) Only III follows
(d) All follow (e) None of these

17 Adda247 Publications For any detail, mail us at


Publications@adda247.com
Cracker Book for Bank (IBPS | SBI | RRB PO | Clerk) Mains Exams

Directions (29-30): In each group of questions below are two conclusions


followed by five set of statements. You have to choose the correct set of
statements that logically satisfies given conclusions. Assume the given
statements to be true even if they seem to be at variance from commonly
known facts. Mark your answer as,

29. Conclusion: I. All mango can be papaya.


II. Some grapes are papaya.
(a) All grapes are guavas. Some mangoes are grapes. Some guavas are
papaya.
(b) Some grapes are guavas. No mangoes are grapes. Some guavas are
papaya.
(c) Some guavas are papaya. No grapes are guavas. No mangoes are
grapes.
(d) No mangoes are grapes. All guavas are papaya. Some grapes are
guavas.
(e) No grapes are guavas. No mangoes are grapes. No guavas are
papaya.

30. Conclusions: All cups being plate is a possibility. Some glasses can be
cups.
Statements:
(a) Some plates are Steels. All Steels are glasses. All cups are plate.
(b) Some plates are Steels. Some glasses are Steels. No plate is a cup.
(c) Some glasses are Steels. Some plates are Steels. Some plates are not
cups.
(d) All glasses are plates. Some Steels are glasses. No cup is a glass.
(e) None of these

Directions (31-35): Some statements are given followed by some


conclusions. You have to consider the statements to be true even if they
seem to be at variance from commonly known facts. You have to decide
which of the following conclusions follow from the given statements:

31. Statements: All bags are books.


Some books are pencils.
No pencils are boxes.
18 Adda247 Publications For any detail, mail us at
Publications@adda247.com
Cracker Book for Bank (IBPS | SBI | RRB PO | Clerk) Mains Exams

Conclusion: (i) Some books are not boxes


(ii) Some books being boxes is a possibility
(iii) Some bags are not boxes
(a) Only (i) follow (b) Only (ii) and (iii) follow
(c) Only (i) and (ii) follow (d) All follow
(e) None of these

32. Statements: Some dolls are barbie.


Some barbie are famous.
All famous are player.
No famous is actor.
Some actors are barbie.
Conclusion: (i) Some barbie are not actors
(ii) Some barbie are players
(iii) Some players are famous
(a) Only (i) follow (b) Only (ii) and (iii) follow
(c) Only (i) and (ii) follow (d) All follow
(e) None of these

33. Statements: All cats are dogs.


Some dogs are not cows.
No cow is a rat.
Some rats are dogs.
No rat is horse.
Conclusion: (i) All cats are horses.
(ii) Some dogs being cows is a possibility.
(iii) All cats being cows is a possibility.
(a) Only (i) follow (b) Only (ii) and (iii) follow
(c) Only (i) and (ii) follow (d) All follow
(e) None of these

34. Statements: All good are bad.


All bad are best.
All best are worst.
Some worst is fair.
No best is normal.
Some normal are worst.
19 Adda247 Publications For any detail, mail us at
Publications@adda247.com
Cracker Book for Bank (IBPS | SBI | RRB PO | Clerk) Mains Exams

Conclusion: (i) All fair being normal is a possibility


(ii) Some best are fair.
(iii) Some normal being good is a possibility
(a) Only (i) follow (b) Only (ii) and (iii) follow
(c) Only (i) and (iii) follow (d) All follow
(e) None of these

35. Statements: All eyes are legs.


No eyes are nose.
All noses are hands.
Some hands are legs.
No legs are ears.
Conclusion: (i) Some hands are ears
(ii) Some hands are not nose
(iii) Some hands are not ears
(a) Only (i) follow
(b) Either (i) or (ii) and (iii) follow
(c) Either (i) or (iii) and (ii) follow
(d) All follow
(e) None of these

Directions (36-38): In each group of questions below are two conclusions


followed by five set of statements. You have to choose the correct set of
statements that logically satisfies given conclusions.
Given statements to be true even if they seem to be at variance from
commonly known facts.

36. Conclusions: All summer being autumn is a possibility.


Some Autumn are Winters is a possibility
Statements:
Statements I: All summer are winters. No winter is Spring. All Springs
are Autumn.
Statements II: No Winter is summer. No summer is Autumn. No
Autumn is spring.
Statements III: All summer are winters. No summer is Spring. No winter
is autumn.
20 Adda247 Publications For any detail, mail us at
Publications@adda247.com
Cracker Book for Bank (IBPS | SBI | RRB PO | Clerk) Mains Exams

Statements IV: All Winters are summer. No winter is Spring. No summer


is Autumn.
Statements V: All summer are winters. No winter is Autumn. All
Springs are Autumn.
(a) Only Statements I (b) Only Statements II
(c) Only Statements III (d) Only Statements IV
(e) Only Statements V

37. Conclusions: At least some samsung being motorola is a possibility.


Some apple are samsung.
Statements:
Statements I: All Samsung are Apple. All samsung are lenovo. No
samsung is a motorola.
Statements II: All Samsung are Apple. All Apple are lenovo. No lenovo
is a motorola.
Statements III: Some Samsung are Apple. Some samsung are lenovo. No
samsung is a motorola.
Statements IV: Some Samsung are Apple. All Apple are lenovo. No
samsung is a motorola.
Statements V: All Samsung are Apple. Some samsung are lenovo. No
lenovo is a motorola.
(a) Only Statements I (b) Only Statements II
(c) Only Statements III (d) Only Statements IV
(e) Only Statements V

38. Conclusions: At least some purse are bag.


Some moneys being bag is a possibility
Statements:
Statements I: All bag are purse. Some purse are wallet. No money is
bag.
Statements II: No bag is purse. Some purse are moneys. No money is
wallet.
Statements III: All bag are purse. Some purse are moneys. No money is
wallet.

21 Adda247 Publications For any detail, mail us at


Publications@adda247.com
Cracker Book for Bank (IBPS | SBI | RRB PO | Clerk) Mains Exams

Statements IV: No bag is purse. Some purse are moneys. No money is


wallet.
Statements V: All bag are purse. Some purse are wallet. No money is
bag.
(a) Only Statements I (b) Only Statements II
(c) Only Statements III (d) Only Statements IV
(e) Only Statements V

Directions (39-40): In the questions below are given two conclusions


followed by four set of statements. You have to choose the correct set of
statements that logically satisfies given conclusions. Assume the given
statements to be true even if they seem to be at variance from commonly
known facts.

39. Conclusions: Some Purse is wallet. Some money is not purse.


Statements:
(a) Some ATM is wallet. No wallet is money. All money is coin. No coin
is purse.
(b) All ATM is wallet. Some wallet is money. No money is coin. Some
coin is purse.
(c) Some ATM is purse. All purse is money. No money is wallet. All
wallet is coin.
(d) All ATM is money. Some money is coin. No coin is purse. Some
wallet is purse.
(e) None of these.

40. Conclusions: Some wall is paint. Some fan is not paint.


Statements:
(a) No window is fan. All wall is fan. No floor is paint. Some paint is
brush.
(b) All window is fan. No window is brush. Some brush is wall. No wall
is paint.
(c) Some window is brush. All brush is floor. No floor is paint. All paint
is fan.

22 Adda247 Publications For any detail, mail us at


Publications@adda247.com
Cracker Book for Bank (IBPS | SBI | RRB PO | Clerk) Mains Exams

(d) All wall is window. Some paint is wall. No paint is brush. All fan is
brush.
(e) None of these.

Directions (41-45): Each question consists of three or four statements


followed by some conclusions. Consider the given statement to be true even
if they seem to be at variance with commonly known facts. Read all the
conclusions and then decide which of the given conclusions logically follow
from the given statements using all statements together. Mark your answer
as-

41. Statements: Some telephones are mobiles


All tablets are laptops
No mobiles are tablets
Some palmtops are tablets
Conclusions:
I. Some laptops are mobiles is a possibility
II. All telephones can be laptops
III. Some palmtops are laptops is a possibility
(a) Only I follow
(b) Only II and III follow
(c) Only I and II follow
(d) Only II follow
(e) None of these

42. Statements: Some bottels are mugs


All mugs are tups
No tups are balty
Conclusions:
I. No balty is mug is a possibility
II. Some baltys are tups
III. Some bottels are not balty
(a) Only II and III follow
(b) Only I follow
(c) None follow
(d) Only I and III follow
(e) Only III follow
23 Adda247 Publications For any detail, mail us at
Publications@adda247.com
Cracker Book for Bank (IBPS | SBI | RRB PO | Clerk) Mains Exams

43. Statements: No mangoes are banana


No bananas are apple
All apples are litchi
No litchis are sweet
Conclusions:
I. No apple are litchi
II. No bananas are sweet
III. All apples can be mango
(a) All follow
(b) Only I and II follow
(c) Only I follow
(d) Only III follow
(e) Only II and III follow

44. Statements: Some tables are chairs


All beds are sofas
No chairs are beds
All bed-sheets are Tables
Conclusions:
I. All bed-sheets can be sofas
II. Some chairs are bad-sheets
III. All sofas can be chairs
IV. Some bed-sheets are tables
(a) All follow
(b) Only IV follows
(c) Only I and IV follow
(d) Only II and III follow
(e) None follow

45. Statements: All books are pens


Some pens are pencils
No pencils are dots
Some dots are pens
Conclusions: I. Some pens are not pencils
II. Some books are dots

24 Adda247 Publications For any detail, mail us at


Publications@adda247.com
Cracker Book for Bank (IBPS | SBI | RRB PO | Clerk) Mains Exams

III. Some pens are not dots


IV. No dots are books
(a) Only III follow
(b) Only I and III follow
(c) Only II follow
(d) Only II and IV follow
(e) None of these

Directions (46-50): In each of the questions below are given three


statements followed by three Conclusions numbered I, II and III. You have
to take the given statements to be true even if they seem to be at variance
from commonly known facts. Read all the Conclusions and then decide
which of the given Conclusions logically follows from the given statements
disregarding commonly known facts.
46. Statements: All flowers are toys.
Some toys are trees.
Some angels are trees.
Conclusions:
I. Some angels are toys.
II. Some trees are flowers.
III. Some flowers are angels.
(a) Only I follows
(b) Only II follows
(c) Only I and III follow
(d) Only III follows
(e) None follows
47. Statements: Some pigeons are dogs.
All dogs are cats.
All pigeons are horses.
Conclusions: I. Some horses are dogs.
II. Some cats are pigeons.
III. Some horses are cats.
(a) All follows
(b) Only II and III follow
(c) Only I and III follow
(d) Only I and II follow
(e) None of these
25 Adda247 Publications For any detail, mail us at
Publications@adda247.com
Cracker Book for Bank (IBPS | SBI | RRB PO | Clerk) Mains Exams

48. Statements: All tables are streets.


All streets are bottles.
All parrots are bottles.
Conclusions: I. Some streets are parrots.
II. Some bottles are tables.
III. All tables are bottles.
(a) Only I follows
(b) Only II follows
(c) Only III follows
(d) Only II and III follow
(e) None of these

49. Statements: Some cartoons are boys.


All boys are windows.
All jokers are windows.
Conclusions: I. All jokers are boys.
II. Some cartoons are windows.
III. Some jokers are cartoons.
(a) Only I follows
(b) Only II follows
(c) Only III follows
(d) Only I and II follow
(e) None of these

50. Statements: Some saints are balls.


All balls are bats.
Some tigers are balls.
Colnclusions: I. Some bats are tigers.
II. Some saints are bats.
III. All bats are balls.
(a) Only I and III follow
(b) Only II follows
(c) Only I and II follow
(d) Only III follows
(e) None of these

26 Adda247 Publications For any detail, mail us at


Publications@adda247.com
Cracker Book for Bank (IBPS | SBI | RRB PO | Clerk) Mains Exams

Solutions

Direction (1-3):
1. (e);

2. (c);

3. (e);

Direction (4-5):
4. (e);

27 Adda247 Publications For any detail, mail us at


Publications@adda247.com
Cracker Book for Bank (IBPS | SBI | RRB PO | Clerk) Mains Exams

5. (d);

Direction (6-8)
6. (d);

Among all the conclusion, conclusion (d) does not follows as it is


clear from the venn diagram some part of egg is cat and no cat is
ball. Therefore, all egg cannot be ball.

7. (d);

Among all the conclusion, conclusion (d) does not follows as


there is no direct relation between the elements pen and chair.
Therefore, we cannot conclude that no pen is chair.

28 Adda247 Publications For any detail, mail us at


Publications@adda247.com
Cracker Book for Bank (IBPS | SBI | RRB PO | Clerk) Mains Exams

8. (e);

Among all the conclusions, conclusion (e) does not follows as


some rubber is definitely plastic but nothing can be said about
all rubber being plastic.

Direction (9-10):
9. (d);

10. (e);

Directions (11-13):
11. (e);

12. (c);

29 Adda247 Publications For any detail, mail us at


Publications@adda247.com
Cracker Book for Bank (IBPS | SBI | RRB PO | Clerk) Mains Exams

13. (e);

Directions (14-15):
14. (b);

15. (e);

Directions (16-18):
16. (a);

Conclusion (a) does not follows as there is no direct relation


between the elements mall and complex. Hence, we cannot
conclude that some mall are not complex.

17. (b);

30 Adda247 Publications For any detail, mail us at


Publications@adda247.com
Cracker Book for Bank (IBPS | SBI | RRB PO | Clerk) Mains Exams

Conclusion (b) does not follows as from the venn diagram it is


clear some light is crossing but nothing can be said about some
crossing not being light. Hence, some crossing is not light does
not follow.

18. (a);

Conclusion (a) does not follows as from the venn diagram it is


clear that definitely some paper is pencil. Hence, possibility case
will not hold true.
Directions (19-20):
19. (d);

20. (e);

Direction (21-22):
21. (d);

31 Adda247 Publications For any detail, mail us at


Publications@adda247.com
Cracker Book for Bank (IBPS | SBI | RRB PO | Clerk) Mains Exams

22. (b);

23. (e);

24. (d);

25. (e);

26. (c)

27. (e)

28. (a);

32 Adda247 Publications For any detail, mail us at


Publications@adda247.com
Cracker Book for Bank (IBPS | SBI | RRB PO | Clerk) Mains Exams

Directions (29-30):
29. (d);

30. (c);

Directions (31-35):
31. (c);

32. (d);

33. (b);

33 Adda247 Publications For any detail, mail us at


Publications@adda247.com
Cracker Book for Bank (IBPS | SBI | RRB PO | Clerk) Mains Exams

34. (a);

35. (e);

36. (a);

37. (e);

38. (c);

34 Adda247 Publications For any detail, mail us at


Publications@adda247.com
Cracker Book for Bank (IBPS | SBI | RRB PO | Clerk) Mains Exams

Directions (39-40):
39. (d);

40. (d);

Directions (41-45):
41. (c);

42. (e);

43. (d);

35 Adda247 Publications For any detail, mail us at


Publications@adda247.com
Cracker Book for Bank (IBPS | SBI | RRB PO | Clerk) Mains Exams

44. (c);

45. (e);

46. (e);

47. (a);

48. (d);

36 Adda247 Publications For any detail, mail us at


Publications@adda247.com
Cracker Book for Bank (IBPS | SBI | RRB PO | Clerk) Mains Exams

49. (b);

50. (c);

 Key Points to Remember for New Pattern Questions


➢ Syllogism topic is a quite scoring one and also quite easy
one to deal with but one should only have to be clear with
the concepts.
➢ The most common points seen in the exam are ‘can’, ‘can
never’ and ‘possibility’. As all these are the recently seen
key words in exam.
➢ Reverse Syllogism is also a mostly seen topic. In this the
diagram of all the statements should be drawn to find out
the one which follows the given conclusion.
➢ An another type comprises of syllogism in which one of
the conclusion does not follow the given statement and
you have to find that by drawing the diagram first.
➢ The probable changes that can be seen in the upcoming
examination is the coded form of Syllogism. You should
draw the diagram by following the codes.

37 Adda247 Publications For any detail, mail us at


Publications@adda247.com
1 Adda247 Publications For any detail, mail us at
Publications@adda247.com
Chapter

7 Inequalities

BEST APPROACH TO SOLVE THE QUESTIONS

It is a quiet easy topic as compared to the others and also can help you to
score well in exams. One should only has to follow the given statement and
just go with the basic knowledge of this topic and you will find your answer.
If you find the questions of inequality in the exams try not to skip this as it
will help you to attain good marks.

Example-1: In the following question each of the following questions,


assuming the given statements to be true, find which of the two conclusions
I and II given below them is/are definitely true. Give answer.
(RBI Grade B-2016)
(a) If only conclusion I is true.
(b) If only conclusion II is true.
(c) If either conclusion I or II is true.
(d) If neither conclusion I nor II is true.
(e) If both conclusions I and II are true.

1. Statements: P=R, Y≥T, U≥V


Conclusions: I. V>U II. Y=R

2. Statements: P≤D, D>Z, D≤X


Conclusions: I. D≥P II. P≤Z

3. Statements: L≤M, N=O, N≥A


Conclusions: I. N≥M II. O≥M

4. Statements: P≥H, Z≤J, H≥J


Conclusions: I. J≤P II. Z≤H

5. Statements: A>B, P≥L, L=S


Conclusions: I. B≥L II. L<B
2 Adda247 Publications For any detail, mail us at
Publications@adda247.com
Solutions:
1. (d); V > U does not follow Y = R does not follow
2. (a); D ≥ P follows P ≤ Z does not follows
3. (d); N ≥ M does not follow O ≥ M does not follow
4. (e); J ≤ P follows Z ≤ H follows
5. (d); L ≤ B does not follow B > L does not follow

Example-2: In these questions, a relationship between different elements is


shown in the statements. The statements are followed by two conclusions.
Give answer.
(Canara PO-2017)

1. Statement: L≤T≤I≥M<X, W<P≤L≥B≥K


I. K<X II. W>M
(a) if only conclusion II is true.
(b) if only conclusion I is true.
(c) if neither conclusion I nor II is true.
(d) if either conclusion I or II is true.
(e) if both conclusions I and II are true.
2. Statement: Z<U≤D≤A≤M<S, Q>A≤Y<G
I. Z<Y II. S>Q
(a) if both conclusion I and II are true.
(b) if only conclusion I is true.
(c) if neither conclusion I nor II is true.
(d) if either conclusion I or II is true.
(e) if only conclusion II is true.

3. Statement: L≤T≤I≥M<X, W<P≤L≥B≥K


I. K≥M II. P>M
(a) if only conclusion II is true.
(b) if either conclusion I or II is true.
(c) if neither conclusion I nor II is true.
(d) if only conclusion I is true.
(e) if both conclusions I and II are true.

3 Adda247 Publications For any detail, mail us at


Publications@adda247.com
4. Statement: Z<U≤D≤A≤M<S, Q>A≤Y<G
I. M≥U II. G>Z
(a) if only conclusion II is true.
(b) if only conclusion I is true.
(c) if neither conclusion I nor II is true.
(d) if either conclusion I or II is true.
(e) if both conclusions I and II are true.

5. Statement: J>K≥H=U≥B≤T<F≤R
I. J>B II. H<R
(a) if only conclusion II is true.
(b) if either conclusion I or II is true.
(c) if neither conclusion I nor II is true.
(d) if only conclusion I is true.
(e) if both conclusions I and II are true.

1. (c); Statement-L≤T≤I≥M<X, W<P≤L≥B≥K


I. K<X (false) II. W>M (false)

2. (b); Statement: Z<U≤D≤A≤M<S, Q>A≤Y<G


I. Z<Y(True) II. S>Q(false)

3. (c); Statement: L≤T≤I≥M<X, W<P≤L≥B≥K


I. K≥M(false) II. P>M(false)

4. (e); Statement: Z<U≤D≤A≤M<S, Q>A≤Y<G


I. M≥U(True) II. G>Z(True)

5. (d); Statement: J>K≥H=U≥B≤T<F≤R


I. J>B(True) II. H<R(false)

4 Adda247 Publications For any detail, mail us at


Publications@adda247.com
Practice Exercise Based on new Pattern

Directions (1-5): Study the following information carefully to answer the


questions given below.
P+Q means ‘P’ is greater than ‘Q’.
P-Q means ‘P’ is smaller than ‘Q’.
P/Q means ‘P’ is either greater than or equal ‘Q’.
P&Q means ‘P’ is either smaller than or equal to ‘Q’.
P*Q means ‘P’ is equal to ‘Q’

1. Statements: A-B, B&C, C*D, D+E


Conclusions: (I) A+E (II) B*D
(a) If only conclusion I is true
(b) If only conclusion II is true
(c) If either conclusion I or II is true
(d) If neither conclusion I nor II is true
(e) If both conclusions I and II are true

2. Statements: A/B, B&C, C*D, D+E


Conclusions: (I) A + C (II) C + E
(a) If only conclusion I is true
(b) If only conclusion II is true
(c) If either conclusion I or II is true
(d) If neither conclusion I nor II is true
(e) If both conclusions I and II are true

3. Statements: A+B, B&C, C+P, D/A


Conclusions: (I) D+B (II) A+P
(a) If only conclusion I is true
(b) If only conclusion II is true
(c) If either conclusion I or II is true
(d) If neither conclusion I nor II is true
(e) If both conclusions 1 and 2 are true
5 Adda247 Publications For any detail, mail us at
Publications@adda247.com
4. Statements: A&B, C+D, D*B, A&E
Conclusions: (I) D/A (II) C+B
(a) If only conclusion I is true
(b) If only conclusion II is true
(c) If either conclusion I or II is true
(d) If neither conclusion I nor II is true
(e) If both conclusions I and II are true

5. Statements: B+A, C*B, D*C, D+E


Conclusions: (I) A+C (II) B+E
(a) If only conclusion I is true
(b) If only conclusion II is true
(c) If either conclusion I or II is true
(d) If neither conclusion I nor II is true
(e) If both conclusions I and II are true

Directions (6-10): In each of these questions, relationship between some


elements is shown in the statement (s). These statements are followed by
two conclusions. Read the statements and give answer:
(a) If only conclusion I follows.
(b) If only conclusion II follows.
(c) If either conclusion I or II follows.
(d) If neither conclusion I nor II follows.
(e) If both conclusions I and II follow.

6. Statement: K < E = L, T ≥ G < 𝐿 ≥ D


Conclusions: I. E > T II. T ≥ L

7. Statement: P ≥ I ≤ O < Z < R > A


Conclusions: I. P ≤ R II. I > A

8. Statement: U < Q, V = E, R ≥ V, N < R, N > Q


Conclusions: I. Q > R II. R > E

9. Statement: S ≤ E, N ≤ Z, E ≥ N, N > Q
Conclusions: I. S > Z II. Q < E

10. Statement: N > A ≥ M = B < E ≥ R ≤ D


Conclusions: I. N > B II. M ≥ D
6 Adda247 Publications For any detail, mail us at
Publications@adda247.com
Directions (11 –15): In the following questions, the symbols @, $, , #,
and % are used with the following meaning as illustrated below.
‘P $ Q’ means ‘P is not smaller than Q’.
‘P @ Q’ means ‘P is neither smaller than nor equal to Q’.
‘P # Q’ means ‘P is neither greater than nor equal to Q’.
‘P %Q’ means ‘P is neither greater than nor smaller than Q’.
‘P  Q’ means ‘P is not greater than Q’.
Now, in each of the following questions assuming the given statements to
be true, find which of the four Conclusions I, II, III and IV given below them
is/are definitely true and give your answer accordingly.

11. Statements: N % B, B $ W, W # H, H  M
Conclusions: I. M@W II. H@N
III. W%N IV. W # N
(a) Only I is true
(b) Only III is true
(c) Only IV is true
(d) Either III or IV is true
(e) Either III or IV and I are true

12. Statements : RD, D $ J, J # M, M @ K


Conclusions: I. K # J II. D @ M
III. R # M IV. D @ K
(a) None is true
(b) Only I is true
(c) Only II is true
(d) Only III is true
(e) Only IV is true

13. Statements: H @ T, T # F, F % E, E V
Conclusions: I. V $ F II. E @ T
III. H @ V IV. T # V
(a) I, II and III are true
(b) I, II and IV are true
(c) II, III and IV are true
(d) I, III and IV are true
(e) All are true
7 Adda247 Publications For any detail, mail us at
Publications@adda247.com
14. Statements: D # R, RK, K @ F, F $ J
Conclusions: I. J # R II. J # K
III. R # F IV. K @ D
(a) I, II and III are true
(b) II, III and IV are true
(c) I, III and IV are true
(d) All are true
(e) None of these

15. Statements: M $ K, K @ N, NR, R # W


Conclusions: I. W @ K II. M $ R
III.K @ W IV. M @ N
(a) I and II are true
(b) I, II and III are true
(c) III and IV are true
(d) II, III and IV are true
(e) None of these

Directions (16-20): In the following questions, the symbols $, %, @, © and


 are used with the following meaning as illustrated below.
‘P%Q’ means ‘P is neither greater than nor smaller than Q’.
‘P$Q’ means ‘P is neither smaller than nor equal to Q’.
‘P©Q’ means ‘P is neither greater than nor equal to Q’.
‘PQ’ means ‘P is not greater than Q’.
‘P@Q’ means ‘P is not smaller than Q’.
Now in each of the following questions assuming the given statements to be
true, find which of the three conclusions I, II and III given below them is/are
true and give your answer accordingly.

16. Statements: V©K, K@B, B$M


Conclusions:
I. V©B II. M©K III. M©V
(a) None is true (b) Only I is true
(c) Only II is true (d) Only III is true
(e) Only II and III are true
8 Adda247 Publications For any detail, mail us at
Publications@adda247.com
17. Statements: N@D, DK, K$A
Conclusions:
I. K@N II. A©D III. N$A
(a) None is true (b) Only I is true
(c) I and II are true (d) I and III are true
(e) All are true
18. Statements: K@T, T$N, N©R
Conclusions:
I. R$K II. NK III. K$N
(a) None of true (b) Only I is true
(c) Only II is true (d) Only III is true
(e) All are true.

19. Statements: DR, R%F, F$T


Conclusions:
I. F%D II. F$D III. T©R
(a) Only I is true
(b) Only II is true
(c) Only III is true
(d) Only either I or II is true
(e) Only either I or II and III are true

20. Statements: W%K, K©F, D$F


Conclusions:
I. D$K II. D$W III. F@W
(a) Only I and III are true
(b) Only I and II are true
(c) Only II and III are true
(d) All, I, II and III are true
(e) None of above

Directions (21-25): In these questions, the relationship between different


elements is shown in the statements. The statements are followed by two
conclusions. Give answer
9 Adda247 Publications For any detail, mail us at
Publications@adda247.com
(a) if only conclusion I is true.
(b) if only conclusion II is true.
(c) if either conclusion I or II is true.
(d) if neither conclusion I nor II is true.
(e) if both conclusions I and II are true.

21. Statements: A ≥ B > C, D ≤ A < P


Conclusions: I. P > C II. D < P

22. Statements: A ≤ B < C, C ≥ M > N


Conclusions: I. M > C II. B < N

23. Statements: M = N ≥ O > P < R


Conclusions: I. M < R II. R ≥ M

24. Statements: A ≥ B > C, D ≤ A < P


Conclusions: I. B > P II. P > B

25. Statements: F ≥ G = H ≥ M ≥ C
Conclusions: I. F ≥ M II. G = C

Directions (26–30): In these questions relationship between different


elements is shown in the statements. These statements are followed by
conclusions. You have to find out which of the conclusions logically follows
from the given statements.
26. Statements: X ≤ Y, Y ≥W, W < P , P≤M
Conclusions: I. M > W II. P > X III. X ≤ M
(a) Only I follows
(b) Only II follows
(c) Only III follows
(d) Only I and III follow
(e) None of these

27. Statements: P ≥ Q, N ≤ S, Q > N, S<T


Conclusions: I. T > Q II. P ≥S III.Q>T IV. P > N

10 Adda247 Publications For any detail, mail us at


Publications@adda247.com
(a) Only I and II follow
(b) Only I, II and III follow
(c) Only III and IV follow
(d) Only II, III and IV follow
(e) None of these

28. Statements: C < Q, Q ≤ J, J > E, E ≥ I


Conclusions: I. J < Q II. I < J III. Q <E IV. J > C
(a) Only I, II and III follow
(b) Only II, III and IV follow
(c) Only I, III and IV follow
(d) All I, II, and IV follow
(e) None of these

29. Statements: X ≥ Y= Z ≤ A < B > C


Conclusions: I. C < Z II. B < Y
(a) Only I follows
(b) Only II follows
(c) Only either I or II follows
(d) Neither I nor II follow
(e) Both I and II follow

30. Statements: P < Q ≤ R < S < T


Conclusions: I. R ≤P II. T > R
(a) Only I follows
(b) Only II follows
(c) Only either I or II follows
(d) Both follow
(e) None follows

Directions (31-35): In these questions, relationship between different


elements is shown in the statements. These statements are followed by two
conclusions:
(a) If only conclusion I follows.
(b) If only conclusion II follows.

11 Adda247 Publications For any detail, mail us at


Publications@adda247.com
(c) If either conclusion I or II follows.
(d) If neither conclusion I nor II follows.
(e) If both conclusion I and II follow.

31. Statements: K≥Q, U>V=I, K >J≤V, U<S


Conclusion: I. I<S II. I≤Q

32. Statements: D≥Q, P>J, O>Q>J, T>D


Conclusion: I. O>T II. T>J

33. Statements: Z<W=V, Y≥W, Z≥U, X≤V


Conclusion: I. Y≥U II. W≥X

34. Statements: A≥U, C=B<U, D≥A>O


Conclusion: I. B≤D II. O<U

35. Statements: Q=B, K>E≥B, J≥E, R≤Q


Conclusion: I. J>R II. R=J

Direction (36-40): Read the information /statement given in each question


carefully and answer the questions.

36. Which of the following expressions will be true if the expression ‘W ≤


V > T ≥ S’ is definitely true?
(a) W < S (b) T < W (c) V > S
(d) W ≤ T (e) None is true

37. Which of the following expressions will not be true if the expression
‘A ≥ B = C > D’ is definitely true?
(a) A D (b) CA (c) D<A
(d) B>D (e) All are true

38. In which of the following expression will the expression F  R’ be


definitely true?
(a) F < Q  G < R (b) R  Q < G  F (c) R  G = Q  F
(d) F = Q  G < R (e) None of these
12 Adda247 Publications For any detail, mail us at
Publications@adda247.com
39. In which of the following expressions does the expressions L < P’ does
not hold true?
(a) P > N  M = L (b) L = N  M < P (c) P < N  M  L
(d) L < M = N  P (e) L < M < N < P

40. Which of the following expressions will be true, if the expression ‘J  H


= I > K’ is definitely true?
(a) J  K (b) K < H (c) I  J
(d) K = J (e) None of These

Directions (41-45): In these questions, relationship between different


elements is shown in the statements. These statements are followed by two
conclusions.
Mark answer as
(a) If only conclusion I follows.
(b) If only conclusion II follows.
(c) If either conclusion I or II follows.
(d) If neither conclusion I nor II follows.
(e) If both conclusions I and II follow.

41. Statements: J>K≤S, Y≤T>Q, T≥S


Conclusions: I. K ≤ T II. Q < J

42. Statements: A>B<C, G≥Q, Q>A


Conclusions: I. G > B II. Q >B

43. Statements: I>R, S>Q≤T, R≤Q<K


Conclusions: I. R > S II. K > R

44. Statements: Y>Z≥W, U>Q>W>R


Conclusions: I. Y > R II. U > R

45. Statements: A>B<C, Q>A, G≥Q


Conclusions: I. C > G II. B ≥ Q

13 Adda247 Publications For any detail, mail us at


Publications@adda247.com
Directions (46-50): In each question, a relationship between different
elements is shown in the statements. The statements are followed by two
conclusions. Study the conclusion for the given statement and select the
appropriate answer.

46. Statements: P<L≤A=N≥E≥D; Q≥N<O


Conclusions: I. L≤E II. P<Q
(a) Only conclusion I is true.
(b) Only conclusion II is true.
(c) Neither conclusion I nor II is true.
(d) Both conclusion I and II are true.
(e) Either conclusion I or II is true.

47. Statements: P≤U=N≤C≥H>S; K≥C


Conclusions: I. P≤C II. U>H
(a) Only conclusion II is true.
(b) Only conclusion I is true.
(c) Both conclusion I and II are true.
(d) Neither conclusion I nor II is true.
(e) Either conclusion I or II is true.

48. Statements: P<L≤A=N≥E≥D; Q≥N<O


Conclusions: I. Q≥D II. A<D
(a) Both conclusion I and II are true.
(b) Either conclusion I or II is true.
(c) Only conclusion II is true.
(d) Only conclusion I is true.
(e) Neither conclusion I nor II is true.

49. Statement: D≥I>S≥M≤A<L


Conclusions: I. D≥A II. L>I
(a) Only conclusion I is true.
(b) Either conclusion I or II is true.
(c) Only conclusion II is true.
(d) Neither conclusion I nor II is true.
(e) Both conclusion I and II are true.

14 Adda247 Publications For any detail, mail us at


Publications@adda247.com
50. Statements: P≤U=N≤C≥H; K≥C
Conclusions: I. K>U II. U = K
(a) Both conclusion I and II are true.
(b) Either conclusion I or II is true.
(c) Only conclusion II is true.
(d) Only conclusion I is true.
(e) Neither conclusion I nor II is true.

Solutions

1. (d); (I) A+E (False) (II) B*D (False)

2. (b); (I) A + C (False) (II) C + E (True)


3. (a); (I) D+B (True) (II) A+P (False)
4. (e); (I) D/A (True) (II) C+B(True)
5. (b); (I) A+C (False) (II) B+E (True)

Directions (6-10):

6. (c); 7. (d); 8. (d);

9. (b); 10. (a);

Directions (11-15):
11. (e); 12. (a); 13. (b);

14. (e); 15. (e);

Directions (16-20):

16. (c); I. V < K ≥ B(FALSE)


II. M < B ≤ K(TRUE)
III. M < B ≤ K > V(FALSE)
15 Adda247 Publications For any detail, mail us at
Publications@adda247.com
17. (a); I. K ≥ D ≤ N(FALSE)
II. A < K ≥ D(FALSE)
III. N ≥ D ≤ K > A(FALSE)
18. (d); I. R > N < T ≤ K(FALSE)
II. N < T ≤ K(FALSE)
III. K ≥ T > N(TRUE)
19. (e); I. F = R ≥ D(FALSE)
II. F = R ≥ D(FALSE)
III. T < F = R(TRUE)

20. (b); I. D > F > K(TRUE)


II. D > F > K = W(TRUE)
III. F > K = W(FALSE)

Directions (21-25):

21. (e); I. P > C (True) II. D < P (True)

22. (d); I. M > C (False) II. B < N (False)

23. (d); I. M < R (False) II. R ≥ M (False)

24. (b); I. B > P (False) II. P > B (True)

25. (a); I. F ≥ M (True) II. G = C (False)

Directions (26 – 30):


26. (a); 27. (e); 28. (e);

29. (d); 30. (b);


Direction (31-35):

31. (a); I. I<S (True) II. I≤Q (False)

32. (b); I. O>T (False) II. T>J (True)


16 Adda247 Publications For any detail, mail us at
Publications@adda247.com
33. (b); I. Y≥U (False) II. W≥X (True)

34. (d); I. B≤D (False) II. O<U (False)

35. (c); I. J>R (False) II. R=J (False)

Directions (36 – 40):


36. (c); 37. (a); 38. (c);

39. (c); 40. (b);

Directions (41-45):

41. (a); I. K ≤ T (True) II. Q < J (False)

42. (e); I. G > B (True) II. Q >B (True)

43. (b); I. R > S (False) II. K > R (True)

44. (e); I. Y > R (True) II. U > R (True)

45. (d); I. C > G (False) II. B ≥ Q (False)

Directions (46 – 50):


46. (b); I. 𝐿 ≤ 𝐸 (False) II. 𝑃 < 𝑄 (True)

47. (b); I. 𝑃 ≤ 𝐶 (True) II. 𝑈 > 𝐻 (False)

48. (d); I. 𝑄 ≥ 𝐷 (True) II. 𝐴 < 𝐷 (False)

49. (d); I. 𝐷 ≥ 𝐴 (False) II. 𝐿 > 𝐼 (False)

50. (b); I. 𝐾 > 𝑈 (False) II. 𝑈 = 𝐾 (False)

17 Adda247 Publications For any detail, mail us at


Publications@adda247.com
 Key Points to Remember for New Pattern Questions
➢ Inequality consist of different mathematical symbols
which define the relation among different elements.
➢ In some cases there may be more than one statement so
to solve such question you have to combine these to get
the exact relation between two elements.
➢ An another variety in questions of Inequality is coded
form of it in which all the relations are given in coded
form. You first have to decode those operators to find the
relation between the element.

18 Adda247 Publications For any detail, mail us at


Publications@adda247.com
1 Adda247 Publications For any detail, mail us at
Publications@adda247.com
Chapter

8 Logical Reasoning

Practice Exercise Based on New Pattern

1. Statement: A sting operation by a news organisation called Cobrapost


claims to have revealed a deeply engrained bias towards the ruling
BJP within many of India's leading media groups, as well as a
willingness among some of the country's most senior media
executives and journalists to take money in return for pushing a
political agenda.
Which of the following can be inferred from the above statement?
(i) India’s leading media companies are more concerned with TRP
rather than showing the truth.
(ii) The ruling party is trying to manipulate the content provided by
the media.
(iii) Some of the journalists are corrupt.
(a) Both (i) and (ii) (b) Both (ii) and (iii) (c) Only (ii)
(d) Only (iii) (e) None of these

2. Statement: In county A, there have been numerous reports over the


past three months about people falling sick after consuming the beer
imported from country B. The beer manufacturers in county B are
claiming a conspiracy by the local manufacturers of country A.
Which of the following is a most appropriate course of action in light
of the above statement?
(a) Country A should stop all its imports from country B.
(b) Heavy taxes should be implemented by country A on the beer
that is imported from country B.
(c) People should be advised to cut down on their beer
consumption.
(d) The samples of beer imported from country A should be sent for
chemical analysis.
(e) A ban should be imposed on the sales of beer imported from
country B until the matter is investigated.
2 Adda247 Publications For any detail, mail us at
Publications@adda247.com
3. Statement: Some of the recent researches points to the addictive
nature of grains. Neurologist and author of Grain Brain, Dr. David
Perlmutter, discussed the addictive effects grains have on our brain in
his book. While the book doesn’t directly say “carbs are addictive,” it
does say that when you eat carbs you will crave more carbs. And
absolutely, wheat for many people feels like something they could
never give up.
Which of the following most strengthens the above statement?
(a) Emotional eating doesn’t fix your emotional problems, in fact, it
usually makes them worse.
(b) Grains are not meant for human consumption, a close look at our
digestive autonomy suggest that we are “frugivores” or “fruit
eaters”.
(c) Many birds have a distinct organ, called the gizzard, which helps
grind grains and seeds. Grains are meant to be bird food for
parrots, pigeons, and doves.
(d) Our digestive tract cannot handle the mixture of starch and
sugar found in most recipes of cereals, breads, bagels, etc. And as
a consequence, most cooked grain we eat tends to ferment
inside of us producing gas and alcohol.
(e) Most people could easily go for months without eating certain
fruits or vegetables and it wouldn’t be a problem for them but
that’s not the case with processed grains.

Direction (4): Study the following information in which a statement is


followed by five assumptions, read carefully and answer the question
below-
The banking sector is facing tough times. But the sector is facing only short-
term pain as the rise in non-performing assets (NPAs) is proving a key
challenge for the industry at large. The regulator wanted some stringent
measures to be taken, and the government is coming out with so many
reforms and policies.
Which of the following can be assumed from the given statement?

3 Adda247 Publications For any detail, mail us at


Publications@adda247.com
(a) NPA was 2.4% more in last two years as compared to the present one.
(b) The steps taken by the government are not enough to revamp the
banking sector.
(c) Manufacturing sector is performing far better than the banking sector
for last five years.
(d) After the implementation of suitable expedient, the things are likely to
be changed for banking sector.
(e) The increase in NPA is the only factor responsible for the difficult time
of banking industry.

Direction (5): Study the following information in which a statement is


followed by five conclusions, read carefully and answer the question below-
Other than being an essential source of water for Indian agriculture, the
monsoon plays a critical role in flushing out pollutants over Asia. However,
recently increased pollution — particularly from coal burning — could
potentially weaken this ability
of the monsoon.

Which of the following can be deduced from the given statement?


(a) The unusually high concentration of particulate matter in the last few
days in India clearly shows that air pollution is emerging as a big
problem.
(b) As the climate gets warmer and frequency of rains reduces, such
spurts in coarse particles making breathing difficult will become a
new normal and the government is not waking up to the alarm.
(c) Both the periodicity and duration of dry spells in the country were
rising as total rainfall events in a year had fallen even though the
average rainfall in a year has not changed much, a direct consequence
of climate change.
(d) The annual average rainfall has remained the same because the
frequency of heavy downpours increased in the past two decades.
(e) The air quality in the region deteriorated because of dust storms in
western India, particularly Rajasthan.

4 Adda247 Publications For any detail, mail us at


Publications@adda247.com
6. Statement-The "kidnapping" case of a retired Pakistan army officer in
Nepal could not be taken to the ICJ or the UN on the basis of
"assumptions" as there is no "concrete evidence" suggesting India's
hand in it, a minister has told Parliament.
Which of the following can be inferred from the given statement?
(I) The issue appeared to have a link with the conviction of
Kulbushan Jadhav.
(II) It can be said that Indian intelligence agency RAW is behind the
kidnapping.
(III) The FO (foreign officers) is trying to obtain solid evidence and
the Nepalese government had been asked to get in touch with
three Indians who had received the Pakistan army officer at the
airport.
(a) Only (I) (b) Only (III) (c) Only (II)
(d) Both (II) and (III) (e) None of these

7. Statement-Despite stagnant foreign direct investment (FDI) inflow of


$44 billion in 2016, India will most likely remain most favoured
destination due to its attractiveness among MNCs for cross-border
mergers and acquisitions, a UN trade report has said.

Which of the following negates the following statement?


(I) An improved economic outlook in major economies, such as
ASEAN, China and India, will likely boost investor confidence,
propping up the region's FDI prospects for 2017.
(II) There are tax related concerns in India that may pose as
deterrent to some foreign investors.
(III) Cross-border mergers and acquisitions deals have become
increasingly important for foreign multinational enterprises to
enter the rapidly-growing Indian market.
(a) Only (I) (b) Only (III) (c) Only (II)
(d) Both (II) and (III) (e) None of these

5 Adda247 Publications For any detail, mail us at


Publications@adda247.com
Directions (8-9): In each of the following questions, three statements
numbered I, II and III are given. There may be cause and effect relationship
among the statements. These statements may be the effect of the same
cause or independent causes. These statements may be independent causes
without having any relationship. Read all the statements in each question
and mark your answer as-
(a) Statement I will be cause and Statement II and III will be its effects.
(b) Statement II and III will be causes and I will be its effect.
(c) Statement III is cause and Statement I is its effect and Statement II is
effect of independent cause.
(d) Statement II is cause and Statement I is its effect and Statement III is
effect of independent cause.
(e) Statement II is cause and Statement III is its effect and Statement I is
an independent cause.

8. Statement I- China has approved a plan to build an underwater


observation network across the disputed East and South China seas,
in yet another provocative move that will anger Beijing's neighbors.
Statement II-China is likely to establish additional military bases in
Pakistan and other countries with which it has longstanding friendly
ties and similar strategic interests.
Statement III-China's military build-up initiative, along with regular
naval vessel visits to foreign ports, both reflects and amplifies China's
growing influence, extending the reach of its armed forces.
Which of the following may be cause/effect among the above
statements?

9. Statement I-International pop sensation Justin Bieber arrived in


Mumbai on Wednesday for his maiden concert in India.
Statement II-A rage among the youth, especially teenagers, Bieber is
touring to promote his critically acclaimed album ‘Purpose.’
Statement III-Grammy Award winning singer Justin Bieber fought
back tears during a short speech, after he performed acoustic versions
of Love Yourself and Cold water at the One Love Manchester concert.
Which of the following may be cause/effect among the above
statements?
6 Adda247 Publications For any detail, mail us at
Publications@adda247.com
10. Statement-The menacing spread, starting last Friday, of the malicious
software Wanna Cry, is a frightening reminder of the vulnerabilities of
a connected world.
Which of the following can be postulated from the given statement?
(I) It is more than obvious now that cyber vulnerabilities have
massive global implications.
(II) While the state of preparedness is a cause for worry, the likely
origin of Wanna Cry forces stakeholders to revisit a long-
standing issue and raises uneasy questions regarding the actions
of governments.
(III) The hackers have used cyber weapons and believed to have
linkage with terrorist organizations.
(a) Only (I) (b) Only (III) (c) Only (II)
(d) Both (I) and (II) (e) None of these

11. In the following question a statement is given followed by three


statements numbered as I, II and III. You have to read all three
statements and decide which of them strengthens or weakens the
statement.
Statement: Among the governments, there is a strong policy emphasis
on improving living conditions in urban areas as more and more
people are looking to migrate to semi-urban or rural areas. People are
willing to compromise on their standard of living in order to live in a
healthier/pollution free environment.
(I) Cities are economically vibrant spaces around the world and
drawing a large number of rural migrants looking for better
prospects. This is a sustained trend, particularly in developing
countries now a day.
(II) A fresh look at urban governance is necessary as migration from
rural areas picks up pace.
(III) The rate of migration has shown a consecutive decline in the last
three Censuses. It was possible that this slow-down signalled the
worsening conditions in urban slums, where most of the rural
immigrants were concentrated.

7 Adda247 Publications For any detail, mail us at


Publications@adda247.com
(a) Only statement (I) strengthen but statement (II) and (III) is the
neutral statement.
(b) Both statement (I) and statement (III) strengthen the given
statement but statement (II) is neutral statement.
(c) Both statement (I) and statement (II) are neutral statements and
statement (III) strengthen the statement.
(d) Both statement (I) and (II) weakens but statement (III)
strengthen the given statement.
(e) All statement (I), (III) and statement (II) weaken the given
statement.

12. The revenue of Company A has been decreasing from past one year
while the prices of the products offered by the company have also
decreased from past six months.
Which of the following cannot be the reason of downfall of the
revenue of Company A?
(a) Company B is offering same product at much lower prices than
Company A.
(b) The quality of the products of Company A is not upto a desirable
level.
(c) The demand of the product offered by Company A has been
decreasing due to the usage of an old technology in it.
(d) There is an abrupt rise in the availability of the product offered
by Company A but the demand has not changed accordingly.
(e) Chinese companies offered better and cheap products than
Indian Companies.

13. Today’s children are different from the children of previous


generations. They are more interested in playing video games or
sitting hours on the play stations than in playing outdoor games. They
are more technology oriented. These things are depriving the child
from developing their personality. They lack the stamina and strength
of the previous generations.
Which of the following statement weakens the given statement?

8 Adda247 Publications For any detail, mail us at


Publications@adda247.com
(I)
The children of today are mentally active as this has made
children much more organized and mature than in previous
times.
(II) These gadgets are addicting. They have made the children so
much addicted to PC and PS that they do not enjoy the outdoor
games anymore.
(III) The children of present time are not physically active. These
children are not being a team player anymore, they grow up to
become shy and introvert instead of being bold and extrovert.
(a) Only I (b) Both III and I (c) Only II
(d) Both II and III (e) All I, II and III
14. A survey conducted recently shows that religious people are more
financially successful than atheists. This proves that following
religious practices or having religious beliefs is not only important for
your spiritual health but also beneficial in maintaining a healthy bank
account.
Which of the following, if true, would most seriously weaken the
above conclusion?
(a) The survey does not tell us if those people became financially
successful after they started practicing religion or they started
practicing religion after they became successful.
(b) Some people embrace religion and spirituality mainly because
they believe that somehow being religious or spiritual can help
them achieve financial success.
(c) Most of the poor people are too busy thinking about earning
their daily livelihood.
(d) Percentage of atheists among the total population is highest in
most of the developed countries.
(e) Spiritual and religious people lead a more disciplined life.

15. A committee on reduction of air pollution in the cities of country X has


suggested a ban on the use and sales of all the diesel based electricity
generators in the cities.
Which of the following will be the repercussions, if the suggestion of
the committee is implemented?

9 Adda247 Publications For any detail, mail us at


Publications@adda247.com
(a) The prices of diesel will reduce sharply.
(b) Pollution levels will come down to safer level in all the cities.
(c) Residents living in the cities will face frequent power outages.
(d) There will be a reduction in the revenues of the companies
manufacturing diesel generators.
(e) All of the above

16. Statement: Around 1,100 cinemas across Tamil Nadu will be closed
from Monday as the theatre owners have called an indefinite strike
against a new municipal tax of 30 per cent in addition to the Goods
and Services Tax or GST.
Which of the following could be the repercussions of the above-
mentioned strike?
(i) People may not be able to watch movies.
(ii) Theatre owners in other parts of the country may follow the
suite.
(iii) The central government may be forced to reduce the tax rates.
(a) Only (i) (b) Only (ii)
(c) Only (ii) and (iii) (d) Only (i) and (iii)
(e) None of these

Directions (17-18): In each of the following questions, two statements


numbered I and II are given. There may be cause and effect relationship
between the two statements. These two statements may be the effect of the
same cause or independent causes. These statements may be independent
causes without having any relationship. Read both the statements in each
question and mark your answer as-
(a) If statements I is the cause and statement II is its effect.
(b) If statements II is the cause and statement I is its effect.
(c) If both the statements I and II are independent causes.
(d) If both the statements I and II are effects of independent causes.
(e) If both the statements I and II are effects of some common cause.

10 Adda247 Publications For any detail, mail us at


Publications@adda247.com
17. Statement I: The former UB group chief Mr. Mallya fled the country
last year and has unpaid debts close to the tune of Rs. 9000 crore
taken as loans from various Indian banks.
Statement II: Fugitive businessman Vijay Mallya was booed by a
section of India supporters as he entered the Oval arena to watch
India’s crucial ICC Champions Trophy encounter against South Africa.

18. Statement I: The National Council of Educational Research and


Training (NCERT) will review all its books.
Statement II: NCERT books were created over a decade ago and need
to be updated, the books were created in 2007.

19. Statement: Highlighting his achievements in present in rooting out


corruption, Prime Minister Narendra Modi said that there has been
"not even one taint or blot" on his government in these past three
years.
Which of the following abrogates the above statement?
(I) Technology has brought about transparency and is being used to
achieve great heights across sectors in India in Modi governance.
(II) In several parameters, the country is moving at a rapid pace
under PM Modi’s government. Noting that infrastructure is
essential for sustainable development. The increase in
aspiration of the common people is the greatest cause of this fast
track development of the country.
(III) The billion Indians in India are still below poverty line and they
are also finding the way to get rid of biggest problem of India
which is corruption.
(IV) India is now receiving record number of foreign direct
investment and all credit agencies and multilateral fora are
giving a positive rating about India.
(a) Only II (b) Only III and IV (c) Only III
(d) Only IV (e) Only I and III

11 Adda247 Publications For any detail, mail us at


Publications@adda247.com
20. Statement: “In a big relief to liquor companies which are holding
their stocks worth more than Rs. 200 crore in Bihar, the Supreme
Court on Monday granted them two months more time to transport
liquor out of the state and directed them to dispose off their stocks by
July 31.”
I. Beverages Companies, contended that it was possible even to
destroy the existing stocks by the end of this month.
II. The apex court had earlier fixed a deadline of May 31 for the
companies to dispose off their stocks.
III. Seeking more time for transporting the stocks out of state, the
confederation said in its petition that delay in disposing off
stocks was beyond its control as clearances were required from
various agencies.
Which of the following can be the Presupposition of the given
statement?
(a) Only I (b) Only II and III (c) Only I and III
(d) All of the above (e) None of the above

21. In the following question a statement is given followed by some more


statements. You have to read all the statement and decide which of
them weakens the statement.
The NITI Aayog released the results of a study warning that India is
facing its “worst” water crisis in history and that the demand for
potable water will outstrip supply by 2030, if steps are not taken.
Which of the following most weakens the above statement?
(a) Currently 600 million Indians face high to extreme water
shortage and about two lakh people die every year due to
inadequate access to potable water.
(b) People may face water scarcity and the country will suffer a loss
of up to 6 per cent of the GDP.
(c) The country's nearly 70 per cent of water is contaminated,
which puts India at the 120th position among 122 countries in
the water quality index.
12 Adda247 Publications For any detail, mail us at
Publications@adda247.com
(d) The ranks of all states in the index on the composite water
management, comprising 9 broad sectors with 28 different
indicators covering various aspects of ground water, restoration
of water bodies, irrigation, farm practices, drinking water, policy
and governance.
(e) The condition of availability of water currently in India is better
than the water crisis which India had faced in 1960.

Direction (22): Study the following information in which a statement is


followed by five assumptions, read carefully and answer the question
below-
Statements: All the population of village A is relocating to city B, near to
village A.
Assumption: (I) City B is rich in terms of resources and employment
opportunities.
(II) A critical virus has been spread in village A and more than 10 people
died from it.
Which of the following can be assumed from the given statement?
(a) Both I and II follow
(b) Neither II nor I follow
(c) Only II follow
(d) Only I follow
(e) Either I or II follow

Direction (23): Study the following information in which a statement is


followed by five assumptions, read carefully and answer the question
below-
“If we slightly increase the price of our product then we can improve the
quality of our product to increase the number of customers from our only
rival company V,”-A statement by owner of company U.
Which of the following can be assumed from the given statement?
(a) Without increasing the price, the quality of the product offered by
company U can’t be increase.
(b) Quality offered by company V of the product is better than the quality
offered by company U.

13 Adda247 Publications For any detail, mail us at


Publications@adda247.com
(c) Improvising the quality is the only way to attract more number of
customers.
(d) The price offered by the rival company V is already higher than the
price offered by company U.
(e) Superior quality even for bit higher price will definitely increase the
demand of the product in the market.

Direction (24): Study the following information in which a statement is


followed by some statement, read carefully and answer the question below-
A company has X decided to increase the HRA allowances of its employees
due to reallocation of the company to NCR region.
Which of the following can be inferred from the given statement?
(I) The HRA allowances directly depend on the region of living of the
employee.
(II) The employees of the company X are on strike due to the reallocation
of the company.
(III) The profit amount of company X has increased so it has also increased
the amount of HRA of its employees.
(a) Both (I) and (II) (b) Both (II) and (III) (c) Only (I)
(d) Only (III) (e) None of these

25. In the following question a statement is given followed by some more


statements. You have to read all the statement and decide which of
them strengthen the statement.
Out of all the major quadrennial international sporting events, the
Football World Cup is the one that gives Indians the purest pleasure.
The Cricket World Cup douses Indians in a broth of tension. The
Olympics continue to visit humiliation on the idea of India as an
athletic nation. The FIFA jamboree, though, is devoid of any such
freight.
Which of the following most strengthen the above statement?
(a) Cricket has always been the first love of Indians. Indians breath
Cricket and enjoy every bit of it.
(b) Cricket is a religion here and it makes a lot of us happy. People
need a Sachin in football. Indians want to talk about the
popularity of football players in the national side as well.
14 Adda247 Publications For any detail, mail us at
Publications@adda247.com
(c) With the people expressing their support, this surely earmarks a
turning point for the rise of football in India and exploit the
potential of our youth and give impetus to the rising status of
this sport.
(d) So far, Hockey is the superior sport. Its players took pride in
'their' way of playing - of their touch and superb body language.
(e) The performance of Indian Women who reach the Olympics are
thus likely to already have exceptional skills, relative to men in
India. Therefore, India should field more make an effort to field
more women and give them better access to trainers and
facilities.

26. Google is trying to turn its search engine into an employment engine.
Job hunters will be able to go to Google and see help-wanted listings
that its search engine collects across the internet. The results will aim
to streamline such listings by eliminating duplicate jobs posted on
different sites. Google will also show employer ratings from current
and former workers, as well as typical commute times to job locations.
(I) Now a days Google market is not good that’s why Google
launched new service.
(II) Google is no more interested in providing other services to its
consumer, now Google is focused only on turning its search
engine into an employment engine.
(III) Google’s search engine results will aim to well-organized
employment listings by removing duplicate jobs.
(IV) Google is not interested to show any kind of ratings in its new
employment search engine.
Which of the following can be deduced from the given passage?
(a) Only II and IV (b) Only I and III (c) Only II and III
(d) Only III (e) None of these

Directions (27-28): Travis Kalanick spent most of the past decade turning
a taxi app (Uber) into the world's most valuable startup. Uber is ride-hailing
service that he helped found in 2009 and built into a transportation
colossus.

15 Adda247 Publications For any detail, mail us at


Publications@adda247.com
Travis Kalanick stepped down Tuesday as chief executive of Uber, after a
seemingly endless series of scandals raised doubts over his leadership.
Kalanick’s resignation doesn’t come as a surprise -- after he was caught on
video chewing out an Uber driver who questioned the company’s
compensation policies in February. Meanwhile, Shareholders released a
damning report on the firm's management culture.
(I) Uber is a riding service that Chief Executive of Uber helped built into
transportation prodigious.
(II) Shareholders ordered Travis Kalanick to relinquish position of chief
executive of Uber.
(III Holder released a report against firm’s management culture that was
condemnatory.

27. Which of the following can be hypothesized from the above passage?
(a) Only II (b) Only I and III (c) Only I and II
(d) All of these (e) None of these

28. Which of the followings is not in the line with the passage?
(I) Kalanick’s resignation came under the pressure of damning
report which is released by the Shareholders on management’s
culture, this one of the reason of resignation of Chief executive of
Uber.
(II) Travis Kalanick stepped down as chief executive of Uber,
because endless series of scandals raised doubts over his
leadership as well as he was caught on video chewing out an
Uber driver and shareholder’s report was not in a favour of
management.
(III) Travis Kalanick spent most of the past decade turning a taxi app
(Uber) into the world's most valuable startup.
(a) Only II (b) Only I and III (c) Only I and II
(d) All of these (e) None of these
16 Adda247 Publications For any detail, mail us at
Publications@adda247.com
29. Statement: An elite Delhi Golf club has apologised after its staff told a
woman who turned up dressed in a traditional north-eastern Indian
outfit to leave. A report said- The staff asked her to leave because they
said she looked "like a maid".
Which of the following is an appropriate course of action in light of the
above incident?
(i) The club should seek an explanation from the staff member for
such a behavior and take disciplinary action against that person.
(ii) Club should apologize for its actions pay fine of 1000 rs to the
women.
(iii) Club should set up a system to verify the actual profiles of new
entrants, so that the staff members won’t have to judge the
entrants on the basis of their dresses.
(a) Only (iii) (b) Only (ii) (c) Only (i)
(d) Both (ii) and (iii) (e) All of these

30. Statement: Low-risk category Indian travellers to the US from now on


would experience speedy entry into the country after landing, with
India making a formal entry into an American initiative. US President
Donald Trump welcomed India's entry into the International
Expedited Traveler Initiative (Global Entry Program).
Which of the following could be the repercussions of India’s entry into
IETI?
(i) It would facilitate closer business and educational ties between
the citizens of India and the US.
(ii) India too will launch a similar programme for tourists entering
the country,
(iii) Travelling to United States will become less risky for Indians.
(a) Only (iii) (b) Only (ii) (c) Only (i)
(d) Both (ii) and (iii) (e) All of these

Direction (31): Study the following information in which a statement is


followed by some course of action read carefully and answer the question
below-

17 Adda247 Publications For any detail, mail us at


Publications@adda247.com
The Uttarakhand high court has banned paragliding and white-water
rafting and other water sports. This move aimed at safeguarding both the
environment and people engaging in these activities. The court’s decision
comes as a jolt to adventure tourism and water sports in the Himalayan
state.

Course of Action
I. An appeal should be filed against this decision as It’s an industry that
provides indirect jobs to no less than 10,000 families.
II. The tourism must be promoted but it is required to be regulated and
for this the State government should be directed to prepare the
transparent policy.
III. All the villages and cities near the banks of the rivers should be ceased
to reduce the pollution in the rivers.
Which of the following is an immediate course of action for the given
statement?
(a) Only I and III
(b) Only II and III
(c) All of the above
(d) Only II
(e) None of these

Direction (32): Study the following information in which a statement is


followed by some assumptions, read carefully and answer the question
below-
National Crime Records Bureau (NCRB) Director Ish Kumar on Thursday
requested the government to provide limited access to the Aadhaar card
database, which would help the investigating agencies in conducting a
probe into inter-State criminals more effectively with the help of
fingerprints.
Assumption: I. The Aadhar card database consist of basic information of a
citizen including his fingerprints.
II. If the number of explorers to a confidential database increases, its
authenticity will be subjected questionable.
III. NCRB do not have its own maintained record containing the realizable
information of criminals.
Which of the following can be assumed from the given statement?
(a) Only I and II (b) Only II and III (c) All of the above
(d) Only II (e) None of these
18 Adda247 Publications For any detail, mail us at
Publications@adda247.com
Direction (33): Study the following information which is followed by some
statement read carefully and answer the question below-
The number of hungry people in the world has risen for the first time in
more than a decade, according to a United Nations report released on
Wednesday. After a prolonged decline, world hunger appears to be on the
rise again. Conflict, drought and disasters linked to climate change are
among the key factors causing this reversal in progress,” said the report.
Which of the following statement strengthen the give statement?
(a) The increasing impact of extreme events related to a changing climate,
economic losses attributed to disasters were estimated at over $300
billion.
(b) More than nine out of 10 people living in urban areas around the
world are breathing polluted air, with southern Asia scoring the worst
in this area.
(a) Only I (b) Only II (c) All of the above
(d) Either I or II (e) None of these

Direction (34): Study the following information which is followed by some


statement read carefully and answer the question below-
Delhiites breathed the cleanest air this week, with monsoon winds bringing
the air quality in the satisfactory level for the first time in nearly a year,
authorities said.
Which of the following is the reason of the given statement?
(I) The monsoon winds cleaned the dusty air and brought the air quality
to satisfactory level.
(II) There was a dip in the air quality level due to dust storms in western
India that pushed the air quality level to "severe plus," bringing to
light that emergency level pollution could be a "summer-time
problem.
(III) Strong anti-cyclonic winds blowing from Rajasthan caused ‘hazy’
conditions in Delhi.
(a) Only I and II (b) Only II and III (c) All of the above
(d) Only I (e) None of these
19 Adda247 Publications For any detail, mail us at
Publications@adda247.com
Direction (35): In each of the following questions, two statements
numbered I and II are given. There may be cause and effect relationship
between the two statements. These two statements may be the effect of the
same cause or independent causes. These statements may be independent
causes without having any relationship. Read both the statements in each
question and mark your answer as-
(a) If statements I is the cause and statement II is its effect.
(b) If statements II is the cause and statement I is its effect.
(c) If both the statements I and II are independent causes.
(d) If both the statements I and II are effects of independent causes.
(e) If both the statements I and II are effects of some common cause.
(I) DMRC's 9,000-strong workforce of non-executive employees has been
taking part in sit-in protests at different metro stations.
(II) Demand of Delhi metro employees is a revision of salaries and pay-
grades and payment of arrears, apart from other demand like the
right to form a union.

Direction (36-38): In making decisions about important questions, it is


desirable to be able to distinguish between ‘strong’ arguments and ‘weak’
arguments. 'Strong’ arguments are those which are important and directly
related to the question. ‘Weak’ arguments are those which are of minor
importance and also may not be directly related to the question or may be
related to a trivial aspect of the question. Each question below is followed
by two arguments numbered I and II. You have to decide which of the
arguments is a strong argument and which is a weak argument.
Give answer
(a) if only Argument I is strong
(b) if only Argument II is strong
(c) if either Argument I or II is strong
(d) if neither Argument I nor II is strong
(e) if both Argument I and II are strong
20 Adda247 Publications For any detail, mail us at
Publications@adda247.com
36. Statement: Should banks be allowed to offer different interest rates
in India?
Arguments:
I. Yes, this is the only way to eventually lower interest rates in the
country.
II. No, this will discourage the customers and they might be
compelled to switch over from one bank to another.

37. Statement: Should the NCERT withdraw its directive for deletion of
passages from History books in the CBSE curriculum?
Arguments:
I. Yes, it will adversely affect school children.
II. No, History should not be taught to school children in a one-
sided manner and any distortions would have to be rectified.

38. Statement: Should Indian government allow the neighbouring


country X to dump huge quantities of Vanaspati ghee into India at a
price which is cheaper than market price?
Arguments:
I. No. it is a violation of the fundamental right of the farmer who
grow crops and that of the industry to operate with a level
playing held.
ll. Yes, it essential for India to provide shelter to all the industries
of X at all costs because it is a neighbouring country.

Directions (39-40): Below in each question are given two statements I and
II. These statements may be either independent causes or may be effects of
independent causes or a common cause. One of these statements may be
the effect of the other statement. Read both the statements and decide
which of the following answer choice correctly depicts the relationship
between these two statements.
Given answer
(a) if statements I is the effect and Statement II is the immediate cause
(b) if statements I is the immediate cause and statement II is its effect
(c) if Statement I is the effect but Statement II is not its immediate cause
(d) if Statement II is the effect but Statement I is not its immediate cause
(e) if Both the Statements I and II are independent cause
21 Adda247 Publications For any detail, mail us at
Publications@adda247.com
39. I. In the world’s biggest conference on AIDS in Barcelona, officials
in their declaration called for $ 10 billion per year to be donated
to the global AIDS fund and for at least two million people with
HIV in developing countries to receive anti-retroviral treatment
by 2012.
II. According to recent 2012 UN report, about 70 million people
will die of AIDS over the next two decades in the 45 worst
affected countries.
40. I. 500000 people were forced to flee to Rwanda from the Eastern
Congo town of Goma.
II. Fires burned out of control in the Eastern Congo town of Goma,
after a nearby volcano sent 100-foot-wide rivers of lava flowing
through the centre of the town and into Lake Kivu.

Solutions

1. (d); Statement (i) cannot be inferred as nothing relating to the TRP is


mentioned in the statement.
Statement (ii) can be inferred as the statement is only concerned
with the bias of media executives and journalists as they take
money in return for pushing a political agenda it means they are
trying to show news which they want.
The fact that some of the senior media executives and journalists
were willing to take money in return for pushing a political
agenda clearly points to their corrupt intentions. So statement
(iii) can be inferred.

2. (e); It is not appropriate to ban all the imports from country B which
might include some essential commodities so (a) not correct.
Option (b) is not appropriate as implementing heavy taxes might
reduce the consumption of beer imported from country B but
the consumption by even one person can be dangerous to him.
Similarly we don’t know the quantity of beer consumed by
people who fell sick after drinking the beer imported from
country B so (c) is not appropriate. Option (d) is irrelevant as
22 Adda247 Publications For any detail, mail us at
Publications@adda247.com
nothing has been mention in the statement about the imports
from country A. Option (e) will be most effective as a corrective
and preventive measure.

3. (e); Statement (b), (c) and (d) talks about the digestibility of grains
or their bad effects while the statement is only concerned with
the addictive nature of grains.
Statement (a) is irrelevant. Statement (e) points to a fact which
proves the addictive nature of grains.
4. (d); All the statement except (d), cannot be assumed from the given
statement as data provided in the statement is not enough to
assume the increase or decrease in NPA from the previous year
also it is not clear whether NPA is the only cause of pain to
banking sector. Further we also cannot assume from the given
statement that steps taken by the government are not enough or
some other sector is performing better than it. But it can be
clearly assumed from the given statement that after the proper
implementation of the policies and reforms there will be some
improvement in the condition of the banking sector.

5. (a); Only statement (a) can be deduced from the given statement as
by burning the coal the concentration of its particles is increased
which lead to excessive amount of air pollution. But for others
(b) and (e) can be assumed but cannot be deduced. While for (c)
and (d) the amount of average rainfall and its consistency in not
directly given in the statement.

6. (e); In the above question we have to find the statement which can
be indirectly deduced from the given statement. None of the
above statement can be inferred because this issue requires
solid evidences regarding Pakistan army officer’s kidnapping in
Nepal so, both (I) and (II) are false because we cannot say that if
this issue has a link with Kulbushan Jadhav’s case or this issue
has some involvement of RAW and in statement (III) it is given
that FO was trying obtain solid evidence which is more of an
assumption.

23 Adda247 Publications For any detail, mail us at


Publications@adda247.com
7. (c); In the above question we have to find the statement which will
make it ineffective or nullifies the given statement.
For I-False, because It supports the given statement as it shows
it will push investors towards FDI.
For II-True, because It opposes the given statement as it states
that tax related issues will intend to discourage the foreign
investors regarding investment in India which contradicts the
given statement.
For III- False, because It also supports the given statement as it
describes the importance of mergers and deals across the
border for foreign investors in growing Indian market.

Direction (8-9):
8. (e); In this question Statement II is the cause and Statement III will
be its effect because China is going to establish military bases in
Pakistan and other countries as its effect will be that the reach of
its armed forces is going to be enhanced. Statement I will be a
independent cause because it discusses about the plans of China
regarding its disputed East and South China seas.

9. (d); In the above question statement II will be the cause and


statement I will be its effect as Justin Bieber is touring to
promote his album will be the cause of his arrival to Mumbai for
his concert in India which is its effect. And Statement III will be
the effect of a different cause as it discusses about him being
emotional at a concert.

10. (d); In the above question we have to find which statement can be
assumed from the given statement.
For I- True, because cyber-attacks had a tremendous impact on
all the countries as given in statement that it infected thousands
of computer system.
For II-True, as it is clear from the above statement that it is a
frightening reminder all over the world which raises questions

24 Adda247 Publications For any detail, mail us at


Publications@adda247.com
regarding government’s action and it’s preparations to deal with
this issue.
For III- False, because it is not clear from the above statement
about its linkage with any terrorist organizations

11. (d); For I- Statement I weaken the given statement as according to it


large number of people migrating to urban areas for better job
opportunities whereas the given statement suggests that people
are willing to compromise on their standard of living in order to
live in a healthier/pollution free environment
For II- Statement II weaken the given statement as according to
it migration from the rural areas is increasing whereas
according to the statement, more and more people are willing to
migrate to rural/semi-urban areas.
For III- Statement III strengthens the statement is it states that
there has been a decline in number of migrants but it is due to
the poor condition of urban slum areas, mostly acquired by the
migrants and because of the policies of the government.

12. (e); As all options other than (e) can be the reason of the downfall of
the revenue of a company but option (e) cannot be a reason as
nothing regarding the Chinese/Indian companies have been
stated in the given statement and is not related with the
downfall of revenue of company A.

13. (a); Only I weakens the given statements as the statement suggest
that the today’s children are weaker than previous generation
and are not as much active and also lack in developing their
personality whereas the statement I states that today’s children
are mentally more active so it weakens the given statement.

14. (a);
(a) mention a flaw in the survey which weakens the
conclusion drawn from it.

25 Adda247 Publications For any detail, mail us at


Publications@adda247.com
(b) supports the statement by pointing out that some people
embrace religion to achieve financial success.
(c) poor people might not have enough time to engage in
religious practices but it does not eliminate the possibility
of them having religious beliefs.
(d) mentions a fact which is irrelevant as the given statement
talks about the general population of financially successful
people which includes both developed and developing
countries.
(e) is irrelevant to the statement.

15. (d);
(a) The prices of diesel are governed by various factors and
we cannot say if the reduction in the prices will be sharp or
minor.
(b) We don’t know the current levels of pollution in the cities
and how much will be the effect of the ban.
(c) There is no doubt the ban will increase the demand for
electricity but we don’t know if the government or the
private agencies involved in the production will or will not
be able to meet the increased demands.
(d) As there will be no further sale of diesel generators in the
cities, there will definitely be a reduction in the revenues.

16. (e); In this question, we have to find out the consequences of the
strike by the theater owners.
Statement (i) is not a correct option as people may be able to
watch movies in mediums other than cinema halls.
Statement (ii) is also not a correct option as the strike will be
against the municipal tax which is being levied by the local state
government.
Statement (iii) is also not true for the same reason as mentioned
above.
26 Adda247 Publications For any detail, mail us at
Publications@adda247.com
Directions (17-18):
17. (a); In the above question it is clear that statement I will be the cause
and statement II will be its effect. Because Mallya has taken loan
from various banks in India and has not been able to repay it for
some time. So this will be the reason of Vijay Mallya’s
disapproval by the audience at the stadium

18. (b); It is clear from the above statement that statement II will be
cause and statement I will be its effect. Because books were
created in 2007 quite a while before it will be a cause and it
need to be updated now so it will be its effect.

19. (c); In the above question we have to find which statement negates
the given statement.
For I: This statement supports the given statement as it
describes about the revolution that technology has bring in India
which is a mark of achievement of Modi’s government.
For II-This statement also supports the given statement as it
describes the country’s growth in Modi government’s tenure
which is considered as victory of his government.
For III-This negates of the given statement as it defines the
problems of poverty and corruption that India is still facing
which can be considered as flaw of Modi’s government.
For IV-It supports the given statement as number of FDI’s has
increased in India in current scenario as given in the statement.

20. (b); In the statement we have to pick the statement which can be
assumed from above statement.
For Statement I. Wrong, Because it was not possible to destroy
the existing stocks by the end of this month that is why the
companies pleaded to the Supreme Court.
For Statement II. Right, Because it is clear from the above
statement that the court had fixed the deadline 31 may earlier
and there after extended it to 31 july.
For Statement III. Right, Because to dispose off stock and
transporting it out of the state requires clearance.

27 Adda247 Publications For any detail, mail us at


Publications@adda247.com
21. (e); Except (e), all the statement strengthens the given statement by
pointing out the effects of water scarcity India is currently facing
and problems related to it but option (e) states that the crisis
was more in 1960 from now which contradicts the given
statement as it states that India is facing its “worst” water crisis
in history. So (e) weakens the given statement.

22. (e); Either (I) or (II) can be assumed from the given statement as
better opportunities or virus can be the reason behind the major
step taken by the population of village A.

23. (e); Statement (e), can be assumed from the given statement as
better quality is always a major factor of concern for customers
and for it paying a bit higher amount is convenient. All other
option except (e) cannot be assumed quality and price offered
by company V for the product is not mention in the given
statement. And also quality is only parameter to increase the
number of customer or it is the only way also cannot be
hypothesized from the given statement.

24. (c); Only statement I can be inferred from the given statement as
HRA allowance increase after reallocation which clearly
indicates that both are directly related to each other. But II and
III cannot be inferred as we cannot state that strike is the reason
behind the rise in allowance. And also increase in profit cannot
be a factor to increase the allowance which the statement also
states that the decision has been taken after the reallocation.

25. (c); Only (c) strengthen the given statement as it states that interest
of people of India is rising in football which will help the skilled
youth to indulge in this game. But (a) and (b) suggest that
Cricket is the most lovable game and football require star
players to become as popular as cricket players among Indians.
Further (d) is about the extraordinary players of Hockey in India
and (e) is about superiority of Indian Women players over Men
players in Olympics game

28 Adda247 Publications For any detail, mail us at


Publications@adda247.com
26. (d); In the above question we have to find which statement can be
concluded from the given passage.
For I-This is not true because it is not given in the above passage
that Google’s market is not good that’s why Google has decided
to launch new service.
For II-This statement is irrelevant. Because Google is going to
launch new service it does not mean that Google is not
interested to provide other services, we can’t conclude this from
the given passage.
For III- There is a sentence in the passage “The results will aim
to streamline such listings by eliminating duplicate jobs posted
on different sites.” It clarifies the statement III. So we can
conclude statement III from the given passage.
For IV- This is vague in the given context because Google will
also show employer ratings from current and former workers, it
is given in the passage.

Direction (27-28):
27. (b); In this question we have to choose that statement which can be
assumed from the passage.
For Statement I: Right, As passage says that Travis Kalanick
helped Uber built into colossus (enormous importance).
Statement I says same thing that Travis Kalanick helped Uber
built into prodigious.
For Statement II: Wrong, from passage we can’t assume that
shareholders forced or ordered Travis Kalanick to give-up from
the post of chief executive of Uber. He stepped down as chief
executive, after a seemingly endless series of scandals raised
doubts over his leadership, no one forced him to do so.
For Statement III: Right, There is a sentence in a passage
“Meanwhile, Shareholders released a damning report on the
firm's management culture”, it clarifies that the Shareholders
released damning (fatal, condemnatory) report against
management’s culture.

29 Adda247 Publications For any detail, mail us at


Publications@adda247.com
28. (e); In this question, we have to select that statement which is out of
the context in regarding to the above passage.
For Statement I: from passage we can get an idea that Travis
Kalanick stepped down as chief executive because of the reason
and there were some other reasons behind the resignation.
For Statement II: As in passage these all are the reasons behind
the Travis Kalanick stepped down as chief executive of Uber.
For Statement III: As in passage it is mentioned, Travis Kalanick
spent most of the past decade turning a taxi app (Uber) into the
world's most valuable startup.

29. (c); In this question we have to find out which of the given courses of
actions will be appropriate in light of the statement given.
Statement (i) is an appropriate course of actions as it is
necessary to take some disciplinary actions against the staff
member so that such incident do no repeat in future.
Statement (ii) is not appropriate as the club has already
apologized the woman as stated in the given media report.
Statement (iii) is also not an appropriate course of action as the
main issue was discrimination. The club apologized not because
the given lady was not a maid but because of a discriminatory
remark made by its employee.

30. (c); In this question we have to find out the possible consequences of
India’s entry into IETI.
Statement (i) is a possible outcome as speedy entry would
definitely encourage more people to visit US.
Statement (ii) not correct as there is not enough evidence in the
statement to make that deduction, Just because any other
country did so does not mean it has to be repeated by India also.
Statement (iii) is also not a correct option as it lacks sufficient
evidence.

30 Adda247 Publications For any detail, mail us at


Publications@adda247.com
31. (d); Only II course of action follows as safety of the people and
environment protection require transparent reforms and
policies. But I and III does not follow as even by considering the
fact that this tourism industry provide employment to so many
people but the lives of so many others cannot be put at stake for
it. And also, III is not feasible as we cannot relocate app. lakhs of
people to reduce the pollution level near rivers.

32. (a); Only I and II can be assumed from the given statement as I is
directly given in the statement that investigating agencies are
going to use the Aadhar database for their investigations which
includes the fingerprints of criminals. And II can also be
assumed from the given statement which states that limited
access leads to assurance and clearly with increase in access,
vulnerability also increases. While III cannot be assumed as it is
not mentioned in the statement.

33. (a); Statement I strengthen the given statement as is states about the
amount invested on disasters and climate related
mishappenings which is also a cause for rising the number of
hungry people. But statement II cannot be assumed from the
given statement as it is about increase in pollution level.

34. (d); Only I can be the reason as the monsoon cleans the air particles
which caused the satisfactory level of wind blow. But II and III
cannot be the reason as it states about the dusty or severe air
flow due to emergency level of air pollution.
35. (b); Clearly Statement I is the effect of statement II as the Delhi
metro employees are protesting which is the effect and the
cause of this protest is hike in the salaries and pay grades.

Direction (36-38):
36. (d); Argument I is not strong due to the word ‘only’. Also it cannot be
said that Different interest rates encourage the customers to
switch the banks. So, Argument II is also not strong.

31 Adda247 Publications For any detail, mail us at


Publications@adda247.com
37. (b); Argument I is not strong because it is not directly related with
the statement. Argument II is strong because it states that
History should not be taught to school children in a one-sided
manner and also if its content is twisted than it should be
corrected.

38. (a); Argument I is strong because when huge quantity of cheaper


Vanaspati ghee will enter in the market, then it will affect the
interest of farmers and industries. Argument II is absurd so it is
not strong.

Directions (39-40):
39. (a); Statements II is cause and I is its effect. As statement II states
that 70 million people will die of AIDS in upcoming time in worst
affected countries and statement I states that officials donated
some amount to the global AIDS fund and it will help 2 million
people for their treatment so it is an effect which will turn out
from the report of UN.

40. (a); Statements II is the cause and Statements I is its effect as after
the fire burned out in eastern Goma, its people were forced to
relocate to Rwanda.

32 Adda247 Publications For any detail, mail us at


Publications@adda247.com
 Key Points to Remember for New Pattern Questions
➢ These questions are of much importance for the mains
exam. As now a days these questions are mostly related
to the general life or current issues.
➢ In the recent exams you would have seen the questions
which strengthen or weakens the given statement.
These also includes the neutral statements, so you have
to be careful and analytical while answering the
questions.
➢ You have to stick to the statement as you cannot be
judgmental, politically correct or emotional while
answering the question as you have to answer the
questions only by considering the statement 100% true.
➢ In recent exams sometimes, you would have seen
reverse in pattern, as the course of action or the
assumption is given, and you have to find the statement
which should be followed by the given course of action
or assumption.
➢ There is a point that requires attention i.e. there is a
difference between conclusion, inference and
assumption. Conclusion is something that you can
directly obtain from a statement. Inference is something
which can be inferred indirectly i.e. on the basis of
evidence and reasoning.

33 Adda247 Publications For any detail, mail us at


Publications@adda247.com
Cracker Book for Bank (IBPS | SBI | RRB PO | Clerk) Mains Exams

1 Adda247 Publications For any detail, mail us at


Publications@adda247.com
Cracker Book for Bank (IBPS | SBI | RRB PO | Clerk) Mains Exams

Chapter

9 Direction

BEST APPROACH TO SOLVE THE QUESTIONS

Sometimes a topic can help you to score good marks in that particular
section. So, you should not miss any of them. The topic of direction may also
play an important role for you to score good marks. These days the
questions asked from this topic are quiet tricky but if you have practiced
enough and your concepts are clear then it can be a game changer for you.

Example-1:

Directions: In the following questions, the symbols #, &, @ and $ are used
with the following meanings as illustrated below. Study the following
information and answer the given questions:
(IBPS PO Mains 2017)
Note: The directions which are given indicates exact directions.
P#Q:- Q is in the south direction of P.
P@Q:- Q is in the north direction of P
P&Q:- Q is in the east direction of P at distance of either 12m or 6m
P$Q:- Q is in the west direction of P at distance of either 15m or 3m.
P#&Q:- Q or P is in the southeast direction of P or Q.
P@&Q:- P or Q is in the northeast direction of Q or P.

1. If A&B#&C$D&E@F are related to each other such that F is placed


exactly between A and B on line AB. Similarly D and A are vertically
inline then what is the possible shortest distance between F and B
when DE (length of segment DE) < DC/2 and EC < 10m?
(a) 12m (b) 5 m (c) 4 m
(d) 6m (e) None of these

2 Adda247 Publications For any detail, mail us at


Publications@adda247.com
Cracker Book for Bank (IBPS | SBI | RRB PO | Clerk) Mains Exams

2. If A&B#&C$D&E@F are related to each other such that D, A are inline,


DE=6 m and a perpendicular drawn from E on AB divides AB in two
equal parts and D@&B then what is the probable direction of A with
respect to F?
(a) North-west (b) West (c) South-west
(d) east (e) Can’t be determined

3. K#&T$M#&S&K&Z are related to each other such that K is in north of


M then what is the probable direction of Z with respect to M?
(a) North-east (b) West (c) South-west
(d) east (e) Can’t be determined

4. K#&T$M#&S&K&Z are related to each other such that K is in north of


M. M and K are inline vertically when MT>SK then what is the distance
between S and Z When KZ=12 m?
(a) 24m (b) 18m (c) 15m
(d) Either (a) or (b) (e) None of these

5. K#&T$M#&S&K&Z are related to each other such that K is in north of


M. M and K are inline vertically when MT>SK then what is the sum of
SK and MT?
(a) 27m (b) 21m (c) 15m
(d) Either (a) or (b) (e) Either (b) or (c)

Explanations (The Approach)

1. (d); It is given that DE < DC/2 so the value of DC = 15m and AB= 12m
or 6m
D and A are inline so DE=AF,
If DE = 6m, EC = 15-6 = 9m
If DE= 3M, EC= 15-3 = 12m
It is given that EC<10 so DE= 6m and AB = 12m
Hence FB =6m
3 Adda247 Publications For any detail, mail us at
Publications@adda247.com
Cracker Book for Bank (IBPS | SBI | RRB PO | Clerk) Mains Exams

2. (e); The direction of F with respect to E is given but the exact


distance from E is not given so we can’t find out the exact
position of F hence the direction of F with respect to A can’t be
determined.

3. (a); There are four possible possibilities but two will cancel out by
the condition that K is in east of S and K is in north of M so the
final figure is given below.
It is clear that Z is in north east from M.

4. (d); It is given that MT>SK so MT=15m KZ= 12m and SK =12m or 6m


so the value of SZ
= (12+12) or (12+6) = 24m or 18 m
4 Adda247 Publications For any detail, mail us at
Publications@adda247.com
Cracker Book for Bank (IBPS | SBI | RRB PO | Clerk) Mains Exams

5. (d); It is given that MT>SK so MT=15m and SK =12m or 6m


SK + MT = (12+15) or (15+6) =27m or 21m

Example-2:

Directions: Study the following information and answer the questions


given below:
There are AB axis in such a way that A is in north and B is in south
direction. There is XY axis in such a way that X is in west direction and Y is
in east direction. AB axis and XY axis intersect at a point Q in such a way
that AQ is 15m, QB is 17m, QX is 12m, QY is 24 m.

Mehul starts from point X and walks 20m in south direction and then he
turns his left and walks 32m. Arun starts from point A and walks 20m in
east direction. Raju starts from point Y and walks 5m in north direction and
then he turns his left and walk 4m and again he turns his left and walks
22m. (IBPS Clerk Mains 2017)

1. Point B is in which direction with respect to Arun’s current position?


(a) south (b) south-east (c) south-west
(d) west (e) north-west
5 Adda247 Publications For any detail, mail us at
Publications@adda247.com
Cracker Book for Bank (IBPS | SBI | RRB PO | Clerk) Mains Exams

2. Point Y is in which direction with respect to Mehul’s current position?


(a) north (b) east (c) north-east
(d) north-west (e) south

3. What is distance between Raju’s current position and Mehul’s current


position?
(a) 3m (b) 5m (c) 13m
(d) 22m (e)27m

Explanation (The Approach):

Solutions (1-3):

1. (c); 2. (c); 3. (a);

Example-3:
Directions: Study the following information carefully and answer the
questions given below.
Six cars C, D, P, Q, R, S are parked in a row facing north at a distance which
is successive multiple of 4m in an increasing order from the left end from
each other. Car S is second to the right of car P. The total distance between
car R and Q is 52m. Only one car is parked in between car D and R. Car S
6 Adda247 Publications For any detail, mail us at
Publications@adda247.com
Cracker Book for Bank (IBPS | SBI | RRB PO | Clerk) Mains Exams

and car Q are parked next to each other. Car C and car S are not parked next
to D. Now Car D starts moving towards north direction after moving 10m its
takes a right turn and stops at point T after moving 28m. Car C starts
moving in east direction and after going 12m it turns right and move 20m
and then again turn right and move 72m and stops there at point H. Car P
starts moving in south direction and after moving 10m it takes a left turn
and moves 20m then it again takes a left turn and moves 5m. From there it
takes a right turn and moves 24m and stops at point V. (SBI Clerk Mains
2018)

1. How many cars are parked there in between cars ‘D’ and ‘Q’?
(a) Two (b) None (c) More than three
(d) One (e) Three

2. What is the distance between point ‘V’ and point ‘H’?


(a) 25 m
(b) 15 m
(c) They don’t align in the same straight line
(d) 20 m
(e) 10 m

3. What is the distance and direction of initial position of Car ‘C’ with
respect to the initial position of Car ‘P’?
(a) 100 m towards east
(b) 50 m towards west
(c) 104 m towards east
(d) 72 m towards south-east
(e) None of these

4. Which car will be met first, if ‘car D’ moves through the shortest
distance from point ‘T’?
(a) R (b) S (c) Q
(d) P (e) None of these

5. What is the current position of car ‘R’ with respect to initial position of
car ‘C’?
(a) Immediate right (b) Second to the left (c) Third to the left
(d) Second to the right (e) None of the above
7 Adda247 Publications For any detail, mail us at
Publications@adda247.com
Cracker Book for Bank (IBPS | SBI | RRB PO | Clerk) Mains Exams

Explanations (The Approach):

Solutions (1-5):
Car S is second to the right of car P. The total distance between car R and Q
is 52m. Only one car is parked in between car D and R. Car S and car Q are
parked next to each other. Car C and car S are not parked next to D.

Now Car D starts moving towards north direction after moving 10m its
takes a right turn and stops at point T after moving 28m. Car C starts
moving in east direction and after going 12m it turns right and move 20m
and then again turn right and move 72m and stops there at point H.

Car P starts moving in south direction and after moving 10m it takes a left
turn and moves 20m then it again takes a left turn and moves 5m. From
there it takes a right turn and moves 24m and stops at point V.

1. (e); 2. (b); 3. (c);

4. (a); 5. (c);
8 Adda247 Publications For any detail, mail us at
Publications@adda247.com
Cracker Book for Bank (IBPS | SBI | RRB PO | Clerk) Mains Exams

Practice Exercise Based on new Pattern

Directions (1-2): Read the given information carefully to answer the


following questions.

Tom start walking in a certain direction to catch Jerry. After walking 5m


in the same direction he turned left and walk 4m. From there he turns
towards his right and walk 3m. Then he takes another right turn and
walked 6m. Now Jerry and Tom are facing each other as they are inline 6m
apart from each other at point B and A respectively. From point B, Jerry
immediately takes a left turn and after walking a certain distance, he
stopped. Now the shortest distance between Tom and Jerry is 10m and Tom
who is still at point A, is to the north-west of Jerry.

1. In which direction did Tom started walking?


(a) North (b) West (c) South
(d) East (e) None of these

2. What is the distance between the points at Jerry stopped from the point
he starts walking?
(a) 7m (b) 5m (c) 10m
(d) 8m (e) None of these

Direction (3-4): In the following questions, the symbols #, *, @ and $ are


used with the following meanings as illustrated below. Study the following
information and answer the given questions:

Note: The directions which are given indicated exact directions.


P*Q :– Q or P is in the south direction of P or Q at distance of 6m.
P@Q :– Q or P is in the north direction of P or Q at distance of 3m
P#Q :– Q or P is in the east direction of P or Q at distance of 5m
P$Q :– Q or P is in the west direction of P or Q at distance of 4m.
P*#Q :– Q or P is in the southeast direction of P or Q.
P*$Q :– Q or P is in the southwest direction of P or Q
P@#Q :– Q or P is in the northeast direction of P or Q.
P@$Q :– Q or P is in the northwest direction of P or Q.
9 Adda247 Publications For any detail, mail us at
Publications@adda247.com
Cracker Book for Bank (IBPS | SBI | RRB PO | Clerk) Mains Exams

3. If B@$C#D*A is related to each other, point A, D and B are inline and


Point B is the midpoint of AD and C is not in the northwest of A then
find out the probable distance between point B and C?
(a) √35 m (b) √39 m (c) 5√2 m
(d) √34 m (e) None of these

4. If B@#C#D*A is related to each other, point B is to the east of point A


and the distance between them is 8m and B is to the northeast of D
then find out the how far and in which direction is B with respect to C?
(a) 2√5 m, South
(b) 3√5 m, North-east
(c) √35 m, West
(d) 5 m, Southeast
(e) None of these

Directions (5-6): Study the following information carefully and answer the
questions given below.

Six persons i.e. A, B, C, D, E and F are sitting in a open ground such that
they are at certain distance with their immediate neighbours. F sits 4m
northeast direction to B. B is in the east of E, who is 4m in the east of person
A. C is 4m in the north of E. The distance between B and E is same as the
distance between D and A. D is 3m south of A.

5. In which direction and what distance is D with respect to E?


(a) North 5m (b) South 4m (c) Southwest, 5m
(d) Southeast, 5m (e) None of these

6. What is the shortest distance between B and A?


(a) 6m (b) 7m (c) 4m
(d) 3m (e) None of these

10 Adda247 Publications For any detail, mail us at


Publications@adda247.com
Cracker Book for Bank (IBPS | SBI | RRB PO | Clerk) Mains Exams

Directions (7-9): Study the following information carefully to answer the


questions given below:

Six cars P, Q, R, S, U, V are parked in the parking and each of them is of


different lengths. Four cars are parked in a rectangular formation such that
the car which is just longer than Q is in north east of it and the car which is
just longer than V is in north west of it. Both Q and V are 8km apart and are
horizontally inline. Only two cars are shorter than R. Car S is longer than P
and car V is longer than Q. Two cars are in north of car U. Car R and U are
not in line. The longest car is in north-west of P. U is not the second longest
car. Longest car is not in the west of the third shortest car. The distance
between the second shortest and second longest car is 6km.

7. What is the shortest distance between car R and Q?


(a) 6km (b) 7km (c) 10km
(d) 14km (e) None of these

8. If the shortest distance between car U and V is 10km then what is the
distance between car P and U?
(a) 16km (b) 17km (c) 20km
(d) 12km (e) None of these

9. The longest car is in which direction with respect to the shortest car?
(a) North (b) South (c) northwest
(d) Southeast (e) None of these

Directions (10-12): Study the following information carefully and answer


the questions given below:

A man was initially walking along the diagonal of a rectangular playground


such that the longer side was in horizontal axis and the shorter on the
vertical axis. He walked only along the perimeter of the rectangle, diagonal
or along the median of the rectangle. He started from the midpoint of the
ground towards southwest and walked 6.5km, then took a turn and walked
for 6km along the perimeter and 2.5 km along the median to reach the

11 Adda247 Publications For any detail, mail us at


Publications@adda247.com
Cracker Book for Bank (IBPS | SBI | RRB PO | Clerk) Mains Exams

starting point. He then turn towards right and walked 4km and finally took
a left turn and walked 1 km. Finally, he is 2km away from the shorter side
and 1.5 km away from the longer side.

10. In which direction is he finally walking?


(a) North (b) South (c) Northeast
(d) Southeast (e) None of these

11. What is the total length of the shorter side of the rectangle?
(a) 10km (b) 4km (c) 8km
(d) 5km (e) None of these

12. In which direction is A is in with respect to its starting position?


(a) Southeast (b) South (c) Northeast
(d) Southwest (e) None of these

Direction (13-14): Study the following information carefully and answer


the question given below-
Point F is to the north of E. Point A is to the west of B. Point B is to the south
of point C. Point F is to the north east of point C which is in-line with point
E. Point D is exactly in between point C and E. Point J is to the north of point
D. Point K is 5m away from point J and is exactly horizontally in line with
point J. Point B is 8m in the south of Point K. The distance between point F
and E is 4m.

13. If the distance between point K and F is 8m and both points are inline,
then what will the shortest distance between point J and E?
(a) 6 m (b) 5 m (c) 7 m
(d) 8 m (e) None of these

14. Point F is in which direction from point A?


(a) North East (b) North West
(c) South West (d) South East
(e) None of these
12 Adda247 Publications For any detail, mail us at
Publications@adda247.com
Cracker Book for Bank (IBPS | SBI | RRB PO | Clerk) Mains Exams

Direction (15-17): Study the following information carefully and answer


the question below-
Point A is 5km in north of point B. Point B is 6km in west point C. Point C is
exactly between point E and F. Point E is horizontally inline with point A.
Point F is as far as from point D (horizontally inline) as Point A from point
E. Point D is vertically not inline with point B.

15. If point G is 6km east of point E then point G is how far and in which
direction from point D?
(a) 6km, North (b) 5km, North-east (c) 10km, North
(d) 14km, South (e) None of these

16. Point A is in which direction from point F?


(a) North (b) South (c) northwest
(d) Southeast (e) None of these

17. How far is point E from point B?


(a) 6 km (b) √87 km (c) √61 km
(d) 8 km (e) None of these

Directions (18-20): Read the following information carefully and answer


the questions that follows:
Six persons i.e. A, B, D, E, G and H are standing in a ground at seven different
points.
(i) B is standing 3m to the west of A.
(ii) E is standing 7m west of person D who is standing 3m east of G.
(iii) H is standing 4m south of D.
(iv) A is standing in 5m south east direction of point E.

18. What is the shortest distance between G and H?


(a) 4m (b) 5m (c) 2m
(d) 3m (e) Cannot be determined

19. How many persons are standing in the south west direction of G if E is
perpendicular to B?
(a) Three (b) More than three (c) Two
(d) None of these (e) Cannot be determined
13 Adda247 Publications For any detail, mail us at
Publications@adda247.com
Cracker Book for Bank (IBPS | SBI | RRB PO | Clerk) Mains Exams

20. How many Persons are standing to the west of G?


(a) None (b) Two (c) Four
(d) Three (e) None of these

Directions (21-22): Read the following information carefully to answer the


questions that follow. The questions are based on following coding formats:
P1Q means – P is in North of Q
P2Q – P is in South of Q
P3Q – P is in East of Q
P4Q – P is in West of Q
@ means – Either 2 or 7 m
# – Either 5 or 10 m
& – Either 8 or 13 m
Conditions given are as:
I. A @1 B
II. A &3 D
III. F #3 E
IV. G @4 F
V. E #1 D

21. What could be the shortest distance between E and B considering the
smallest among the given two-possible distances?
(a) √113 (b) √111 (c) √115
(d) √117 (e) none of these

22. In which direction is point G with respect to D?


(a) south west (b) north west (c) north
(d) north east (e) can’t be determined

Directions (23-24): Following questions are based on the information


given below.

Ajay start driving from his house to his friend Jeet’s house. He start driving
in the west direction from his house by a car. After going 15m he takes a
right turn from point N and continue driving for 20m and reaches the point

14 Adda247 Publications For any detail, mail us at


Publications@adda247.com
Cracker Book for Bank (IBPS | SBI | RRB PO | Clerk) Mains Exams

P. From point P he takes a right turn and after driving 30m, reaches the
point Q. from point Q he takes a left turn and continue moving 20 m to
reach Jeet’s house. In the way Ajay stop a Shop at point O which is exactly
between P and Q.

23. Find the shortest distance between jeet’s house and point O?
(a) 35m (b) 30m (c) 25m
(d) 10√13 (e) None of these

24. Jeet’s house is in which direction from Ajay’s house?


(a) North (b) North East
(c) North West (d) South West
(e) South

Directions (25-27): These questions are based on the information given


below:
Eight persons A, B, C, D, E, F, G and H are standing in such a way that C is 20
m apart from B towards West, B is 30 m South with respect to A. A is 40 m
towards West with respect to E. D is 50 m towards South with respect to E.
F is 15 m apart from G towards North. H is 20 m towards East with respect
to G. F is 40 m towards West with respect to D.

25. In which direction is B standing with respect to F?


(a) North-West
(b) North
(c) North-East
(d) Cannot be determined
(e) None of these

26. If one more person I is standing towards South-West with respect to


D, then in which direction is H, standing with respect to I?
(a) South-West
(b) North-East
(c) North-West
(d) Cannot be determined
(e) None of these
15 Adda247 Publications For any detail, mail us at
Publications@adda247.com
Cracker Book for Bank (IBPS | SBI | RRB PO | Clerk) Mains Exams

27. What is the direction of C with respect to H?


(a) North-West (b) North (c) West
(d) Cannot be determined (e) None of these

28. Mohan starts walking in the west direction and walks for 8 km, then
he takes a right turn and walks for 5 km again he takes a right turn
and walks for 2.5 km and at last he takes a right turn walks for 5 km,
then in which direction and at what distance is Mohan from his initial
position?
(a) 5 km west (b) 5.5 km East (c) 5 Km East
(d) None of these (e) 5.5 km west

Direction (29-30): Two friends Silva and Jones started walking from two
different points. Silva, who started from point A moved along a circular
track and after walking 11km he reached point B, from where he turned left
and walked 5km to reach point C after which he stopped. Jones started from
point F and after walking 5km, he reached point E. From point E he turned
45 degree towards his left and walked √8 km to reach point D. After
reaching point D, he turned 45 degrees in clockwise direction. Then, he
moved ahead 5km to reach point C which is towards the west of point D.
Point B, C and D are in straight line.

29. In which direction was Silva facing at instant he travelled three-fourth


of his total journey?
(a) East (b) West (c) North East
(d) South East (e) Cannot be determined

30. If Silva keeps on walking ahead from point C and reached point G after
walking 12 km, Point F is to the north of point G then what is the
shortest distance between point F and G?
(a) 7km (b) 4km (c) 2km
(d) More than 7km (e) None of these

16 Adda247 Publications For any detail, mail us at


Publications@adda247.com
Cracker Book for Bank (IBPS | SBI | RRB PO | Clerk) Mains Exams

Solutions

Solutions (1-2):
As from the given conditions we get that only possible condition is that Tom
start walking in North direction.

1. (a);

2. (d); distance between the points at Jerry stopped from the point he
starts walking= √102 − 62 m = 8m

Solutions (3-4):
3. (d); The distance between point B and C= √32 + 52 = √34 m

17 Adda247 Publications For any detail, mail us at


Publications@adda247.com
Cracker Book for Bank (IBPS | SBI | RRB PO | Clerk) Mains Exams

4. (b); The distance between B and C = √62 + 32 = √45 m

Solutions (5-6):

5. (c); 6. (b);

Solutions (7-9):
Four cars are parked in a rectangular formation such that the car which is
just longer than Q is in north east of it and the car which is just longer than
V is in north west of it. Both Q and V are 8km apart and are horizontally
inline. Only two cars are shorter than R. So, from this we get that----

> > > R> >


Car S is longer than P and car V is longer than Q. So, from this we know that
either S or U is the longest Car. Two cars are in north of car U. Car R and U
are not in line. The longest car is in north-west of P. U is not the second
longest car. Longest car is not in the west of the third shortest car. The
18 Adda247 Publications For any detail, mail us at
Publications@adda247.com
Cracker Book for Bank (IBPS | SBI | RRB PO | Clerk) Mains Exams

distance between the second shortest and second longest car is 6km. So, Q
can be either shortest or second shortest.
Consider case-1, Q is the shortest, so S or U can be longest but we know that
Two cars are in north of car U and the longest car is in north-west of P
which is not possible so it will get eliminated. Further if S is the longest car,
the longest car is in north-west of P and Longest car is not in the west of the
third shortest car, so this will also get eliminated.
In case-2, Q is the second shortest car then R will be just longer than it and
will be in north of car V. And U can’t be the longest car as already seen from
the given conditions, so clearly S will be the longest car and U will be the
shortest car. So, the final arrangement is----------

S>P>V>R>Q>U

7. (c); Distance between car R and Q = √82 + 62 = 10km

8. (d); shortest distance between car U and V is 10km


Then distance between Q and U is = √102 − 82 = 6km
Distance between car P and U = 6 + 6 = 12km

9. (c);

Solutions (10-12):

10. (a); 11. (d); 12. (c);


19 Adda247 Publications For any detail, mail us at
Publications@adda247.com
Cracker Book for Bank (IBPS | SBI | RRB PO | Clerk) Mains Exams

Solutions (13-14):

13. (b); DE = JF = KF – KJ
DE = JF = 8 – 5 = 3m
JD = FE=4m
JE = √32 + 42 =5m

14. (a);

Solutions (15-17):

15. (c); 16. (c); 17. (c);

Solutions (18-20):

18. (b); 19. (c); 20. (e);


20 Adda247 Publications For any detail, mail us at
Publications@adda247.com
Cracker Book for Bank (IBPS | SBI | RRB PO | Clerk) Mains Exams

Solutions (21-22):

21. (a); 22. (e);

Solutions (23-24):

23. (c); 24. (b);

25. (b);

B is standing in North direction with respect to F.

21 Adda247 Publications For any detail, mail us at


Publications@adda247.com
Cracker Book for Bank (IBPS | SBI | RRB PO | Clerk) Mains Exams

26. (d);

As total distance between I and D is not given, so the position of


H with respect of I cannot be determined.
27. (a);

C is in the North-West direction from H.


28. (e);

29. (a);

30. (c);

22 Adda247 Publications For any detail, mail us at


Publications@adda247.com
Cracker Book for Bank (IBPS | SBI | RRB PO | Clerk) Mains Exams

 Key Points to Remember for New Pattern Questions


➢ The direction-based puzzle is also newly introduced in
the exam, in this there are certain symbol used to define
the directions with the distance. Sometimes the distance
and direction is given by combining it with either/or so
start solving it by drawing both the conditions
simultaneously.
➢ Solving questions of direction requires blindly following
the path as you have to draw as per the directions given
in the question.
➢ After examining from the recent exam, you may get to
know that each and every question is different from the
other one. And a separate diagram is required to be made
from it.
➢ A new kind of questions that has been seen in the recent
exam comprises Axes with the direction information. In
this we require to draw the diagram on the axes XY.
➢ A puzzle form of direction question comprising of a linear
row having different elements placed in it as it is also a
new pattern question. In this the type of question first the
elements should be arranged and then their movement in
different direction should be consider.

23 Adda247 Publications For any detail, mail us at


Publications@adda247.com
Cracker Book for Bank (IBPS | SBI | RRB PO | Clerk) Mains Exams

1 Adda247 Publications For any detail, mail us at


Publications@adda247.com
Cracker Book for Bank (IBPS | SBI | RRB PO | Clerk) Mains Exams

Chapter

10 Blood Relation

BEST APPROACH TO SOLVE THE QUESTIONS

Now a days as you can see there has been a constant change in the
pattern of each and every topic. So, you have to be prepared for all those
changes. That is reason why we are providing you questions based on
recent exams, once you go through all these you may be able to understand
the recent changes in the current examination.

Example-1:
In the following questions, the symbols #, &, @, * , $, % and © are used with
the following meanings as illustrated below. Study the following
information and answer the given questions: (IBPS PO Mains 2017)

P#Q - P is the son of Q.


P@Q - Q is the child of P.
P©Q - P is the parent of Q.
P$Q - P is elder than Q.
P*Q- P is the husband of Q.
P&Q- Q is the daughter-in-law of P.
P%Q- P is the wife of Q.

1. If A@B*D&G%E$F#D then how F is related A?


(a) Grandfather (b) Grandson (c) Daughter
(d) Wife (e) None of these

2. If H*M©O$N#M, the age of N is 20 years and of H is 40 years then


what is the probable age of O?
(a)17 years (b)15 years (c)23 years
(d) 45 years (e)12 years

2 Adda247 Publications For any detail, mail us at


Publications@adda247.com
Cracker Book for Bank (IBPS | SBI | RRB PO | Clerk) Mains Exams

Direction (1-2):
1. (b);

2. (c);

Example-2:
Study the information carefully and answer the questions given below.
P is the husband of Q. R is the grandchild of P.P has only one child(son) who
is married to T’s child. T has only two children one son and one daughter. X
is grandson of T. S is brother in law of son of T. U and V are children of T. W
is married to the son of T. X is son of U’s brother. (SBI PO Mains 2017)
1. If T is married to Y than how is T related to R?
(a) Grandfather (b) Grandmother
(c) Maternal Grandfather (d) Maternal Grandmother
(e) Either (c) or (d)

2. How is X related to V?
(a) Son (b) Daughter (c) Son in law
(d) Daughter in law (e) Husband

Directions (1-2):

1. (e); 2. (a);
3 Adda247 Publications For any detail, mail us at
Publications@adda247.com
Cracker Book for Bank (IBPS | SBI | RRB PO | Clerk) Mains Exams

Example-3:
Study the following information carefully and answer the questions given
below.
Six persons J, L, R, O, X, Z of a family born in six different years 1978, 1982,
1995, 1997, 2013 and 2015. (All the ages of the given persons have been
calculated on base year 2018 and all the persons are considered to be born
on same date of same month). There is a difference of 18 years between the
ages of O and J. O is the father of J. L is younger than R. X is not the oldest
person of the family. R is not married to Z. Z was born in an even numbered
year. L is the only granddaughter of the mother of O. Father of O is 4 years
older than O’s mother. J is the son of R. (SBI PO Mains 2018); Z was born in
an even numbered year. L is the only granddaughter of the mother of O.
Father of O is 4 years older than O’s mother. There is a difference of 18
years between the ages of O and J. O is the father of J. J is the son of R.
Case-1: Case-2:
Years Persons Years Persons
1978 1978
1982 1982
1995 O 1995
1997 1997 O
2013 J 2013
2015 2015 J

L is younger than R. X is not the oldest person of the family. R is not married
to Z. Z was born in an even numbered year. Father of O is 4 years older than
O’s mother.

4 Adda247 Publications For any detail, mail us at


Publications@adda247.com
Cracker Book for Bank (IBPS | SBI | RRB PO | Clerk) Mains Exams

Years Persons
1978 Z
1982 X
1995 O
1997 R
2013 J
2015 L

Practice Exercise Based on new Pattern

Direction (1-2): In the following questions, the symbols #, &, @, *, $, % and


© are used with the following meanings as illustrated below. Study the
following information and answer the given questions:
A@B- A is the child of B.
A©B- A is the parent of B
A%B- A is elder to B
A&B- A is younger to B
A$B- A is brother of B
A*B- A is wife of B
A#B- A is sister-in-law of B

1. If X©F$D&Q@X©E%D, the age of E is 25 years and age of Q is 32


years, so what can be the age of D?
(a)27 years (b)23 years (c)29 years
(d)30 years (e)34 years

2. If G*J$K©Y@V&C and V#G then how K is related to Y?


(a) father (b) Brother-in-law
(c) daughter-in-law (d) Sister (e) None of these
5 Adda247 Publications For any detail, mail us at
Publications@adda247.com
Cracker Book for Bank (IBPS | SBI | RRB PO | Clerk) Mains Exams

Directions (3-5): Study the following information carefully and answer the
questions given below.
In a family of ten people C, G, H, Q, R, S, T, K, U, Y. G and H are married
couple. Q is the only sister of R. S is the grandfather of T. U is the son-in-law
of H. C is the only child of G. R is grandchild of G. T is the child of U. Q is
unmarried. H is the sister-in-law of K. H has no sibling and G is the only son
of Y.

3. If J is the son-in-law of C, then how is R related to C?


(a) Grandson (b) Brother-in-law (c) Daughter
(d) Father-in-law (e) None of these

4. How is Y related to C?
(a) Grandfather (b) Grandmother (c) Mother
(d) Can’t be determined (e) Sister

5. How is Q related to S?
(a) Grandson (b) Brother-in-law (c) Granddaughter
(d) Father-in-law (e) None of these

Directions (6-7): Study the following information carefully and answer the
questions given below.

There are some members in the family. L is brother of M, who has only one
son. A is the sister-in-law of R and is the mother of D. R is the child of M. A is
daughter-in-law of K. G and T are siblings. G is the son of L. S is the sister-
in-law of M and has only two sons. D is not the female member of the family.
T and G are the cousins of J. S is not the sister of K, who is the grandmother
of E. J has only one daughter. R is the daughter of K.

6. How is M related to J?
(a) Brother (b) Mother (c) Father
(d) Sister (e) None of these

7. How is M related to ‘T’?


(a) Father (b) Brother-in-law (c) Mother
(d) Grandfather (e) None of these
6 Adda247 Publications For any detail, mail us at
Publications@adda247.com
Cracker Book for Bank (IBPS | SBI | RRB PO | Clerk) Mains Exams

Direction (8-10): In the following questions, the symbols #, &, @, * , $, %


and © are used with the following meanings as illustrated below. Study the
following information and answer the given questions:
A@B- A is the child of B.
A©B- A is the parent of B
A%B- A is elder to B
A&B- A is younger to B
A$B- A is brother of B
A*B- A is wife of B
A#B- A is sister-in-law of B

8. If Y%H@J$U&K%Y, the age of H is 22 years and age of K is 33 years, so


what can be the age of Y?
(a)17 years (b)13 years (c)29 years
(d)40 years (e)36 years

9. If G©A@T#J*O$L@P©G then how is J related to A?


(a) Uncle (b)Brother-in-law (c) daughter-in-law
(d)Aunt (e) None of these

10. If G©A@T#J*O$L@P©G and P is the wife of U then how is P related to


T?
(a) Mother (b)Mother-in-law (c) sister-in-law
(d) daughter-in-law (e) None of these

Directions (11-12): Study the following information carefully and answer


the questions given below:
All the persons mentioned belong to one family. Z is the only sibling of Y,
who has only one daughter. M is married to N. O is grandchild of Y, who is
not in same generation as M. L is aunt of O. N is mother-in-law of V. V and Z
are sister-in-law of each other. S is married to R but is not a female.

11. How many married couples are there in the family?


(a) one (b) two (c) three
(d) four (e) can’t be determined

7 Adda247 Publications For any detail, mail us at


Publications@adda247.com
Cracker Book for Bank (IBPS | SBI | RRB PO | Clerk) Mains Exams

12. How many generations are there in the given family?


(a) one (b) two (c) three
(d) four (e) can’t be determined

Direction (13-14): Study the following information carefully and answer


the question given below-
There are nine members in the family. M is unmarried sibling of F. B is
sister-in-law of D. C is mother-in-law of E, who is parent of F. E has a
daughter and a son. H is the only child of F. B has a sister and a brother. The
maximum number of members are in the oldest generation. G is the
maternal grandmother of H. A is the unmarried.

13. Four of the five are alike in a certain way, who among the following
does not belongs to that group?
(a) A (b) B (c) C
(d) D (e) F

14. How is D related to M?


(a) uncle
(b) aunt
(c) Maternal grandfather
(d) Maternal grandmother
(e) None of these

Directions (15-16): Study the following information carefully and answer


the questions that follow:
Ten persons P, Q, R, S, T, U, V, W, X and Z are there in a family of three
generation. There are three married couple. S is the sister-in-law of R but
not married to P. P is the son of V. W is the brother of T. Z is the father-in-
law of Q. U is the mother of X and married to the brother of R. Q has only
one sister, who is unmarried. X has no sibling. Z has only one grandson and
no girl child. T is the youngest person of the family.

15. What is the ratio of male members to female members of the family
respectively?
(a) 5:4 (b) 1:1 (c) 3:2
(d) 2:3 (e) none of these
8 Adda247 Publications For any detail, mail us at
Publications@adda247.com
Cracker Book for Bank (IBPS | SBI | RRB PO | Clerk) Mains Exams

16. How is S related to T?


(a) mother (b) father (c) uncle
(d) aunt (e) none of these

Direction (17-18): Study the following information carefully and answer


the question below-

Eight persons K, L, M, N, O, P, Q, R are from three generations and four


married couple. R is the mother-in-law of O. P is married to the brother of
M. Q is the father-in-law of L. N is married to Q. K is the father of M. O is the
sister-in-law of L.

17. How is P related to M?


(a) Son-in-law (b) Son (c) sister -in-law
(d) Daughter (e) None of these

18. If J is the only child of L then how is N related to J?


(a) Maternal Grand mother
(b) mother
(c) Aunt
(d) Father
(e) None of these

Directions (19-20): Following questions are based on the information


given below.
‘P × Q’ means ‘P is father of Q’
‘P – Q’ means ‘P is sister of Q’
‘P + Q’ means ‘P is mother of Q’
‘P ÷ Q’ means ‘P is the brother of Q’

19. In the expression A × R –G + D ÷ N, how is D related to A?


(a) Granddaughter
(b) Son
(c) Grandson
(d) Granddaughter or Grandson
(e) None of these
9 Adda247 Publications For any detail, mail us at
Publications@adda247.com
Cracker Book for Bank (IBPS | SBI | RRB PO | Clerk) Mains Exams

20. Which of the following represents ‘Q’ is son of ‘U’?


(a) Q ÷ I – G × U (b) Q + I – G × U (c) Q ÷ M – N × U
(d) Q ÷ J – N + U (e) None of these

Directions (21-22): Following questions are based on the information


given below.
‘P × Q’ means ‘P is Son of Q’
‘P – Q’ means ‘P is sister of Q’
‘P + Q’ means ‘P is mother of Q’
‘P ÷ Q’ means ‘P is the brother of Q’

21. In the expression R ÷ Q + M ÷ N × G, how is N related to R?


(a) Granddaughter
(b) Son
(c) Niece
(d) Granddaughter or Grandson
(e) None of these

22. In which of the following expression Q is wife of Z?


(a) A–M÷Q+N×Z (b) A–Q÷M+N×Z (c) Q–M÷A+N×Z
(d) Q–R÷A÷N×Z (e) None of these

Directions (23-24): Following questions are based on the information


given below.
‘A × B’ means ‘A is Son of B’
‘A – B’ means ‘A is sister of B’
‘A + B’ means ‘A is mother of B’
‘A ÷ B’ means ‘A is the brother of B’

23. In the expression ‘R + L – S × M ÷ N’, how is N related to R?


(a) Brother (b) Brother-in-law (c) Sister
(d) Grandson (e) None of these

24. In which of the following expression N is brother-in-law of C?


(a) N÷D+R÷S×C (b) N÷D+C÷S×R (c) S÷D+R÷N×C
(d) M÷D+N÷S×C (e) None of these

10 Adda247 Publications For any detail, mail us at


Publications@adda247.com
Cracker Book for Bank (IBPS | SBI | RRB PO | Clerk) Mains Exams

25. A is father of X; B is mother of Y. The sister of X and Z is Y. Which of


the following statements is definitely not true?
(a) B is the mother of Z
(b) X is the sister of Z
(c) Y is the son of A
(d) B has one daughter
(e) B is the wife of A

26. Pointing to a woman, Meenu says to Ankit, “she is my mother who has
only two child’s and her only son is your maternal uncle.” In this
relation Meenu is Female. How is the narrator related to Ankit’s
father?
(a) Brother (b) Wife (c) Sister
(d) Mother (e) Father

Directions (27-29): These questions are based on the following


information:
‘A @ B’ means ‘A is father of B’.
‘A $ B’ means ‘A is mother of B’.
‘A * B’ means ‘A is sister of B’.
‘A # B’ means ‘A is daughter of B’.
‘A = B’ means ‘A is son of B’

27. How is D related to H, in the given expression


D@E=T$G*H
(a) Father (b) Mother (c) Sister
(d) Son (e) None of these

28. Which of the following statements is/are true, if the given expression
Q = W # V @ M = P is true?
(a) Q is father of M
(b) V is husband of P
(c) P is grandmother of Q
(d) Only (b) and (c) are true
(e) All are true

11 Adda247 Publications For any detail, mail us at


Publications@adda247.com
Cracker Book for Bank (IBPS | SBI | RRB PO | Clerk) Mains Exams

29. What is the relation between E and W in the given expression?


E@H$N*P=W
(a) E is father of W
(b) E is daughter of W
(c) E is father-in-law of W
(d) E is brother of W
(e) None of these

Directions (30-32): Each of these questions is based on the following


information:
There are seven members in a family Ayan, Brad, Axelsen, Rishi, Roman,
Andy, and Piyush among them only two are married couples. Brad is
daughter in law of Roman. Andy is grandmother of Rishi. Piyush is sister in
law of Roman. Ayan has two children Axelsen and Rishi. Andy and Piyush
are siblings. Brad has no son.

30. Who among the following is son of Piyush’s brother in law?


(a) Rishi (b) Axelsen (c) Ayan
(d) Brad (e) None of these

31. How is Axelsen related to Ayan’s mother?


(a) Grandson (b) Granddaughter (c) Son
(d) None of these (e) Daughter

32. How is Piyush related to Axelsen’s father?


(a) Maternal aunt (b) Paternal aunt (c) Mother
(d) None of these (e) Son

Direction (33-34): Read the following information carefully and answer


the questions given below:
Amit, Browni, Cendy, David, Julie, Feriah, Gautam, Henry, and Iram are
family members among them only six male members. Amit is married to
David. Each female has two sons and one daughter except one female. David
is grandmother of Iram who has no children. Browni is daughter of Amit,
who is married. Iram is daughter of Gautam. Henry and Feriah are brothers
and both are child of Gautam.
12 Adda247 Publications For any detail, mail us at
Publications@adda247.com
Cracker Book for Bank (IBPS | SBI | RRB PO | Clerk) Mains Exams

33. Who among the following is a brother-in-law of Gautam?


(a) Amit (b) Henry (c) Feriah
(d) Cendy (e) None of these

34. Who among the following is a grandson of David?


(a) Amit (b) Henry (c) Cendy
(d) Julie (e) None of these

Directions (35-36): Each of these questions is based on the following


information:
(i) A % B means A is the mother of B.
(ii) A @ B means A is the sister of B.
(iii) A $ B means A is the father of B.
(iv) A * B means A is the son of B.
(v) A + B means B is brother of A
35. If the expression F $ M@ N * G + P is definitely true, then which of the
following is true?
(a) P is sister in law of F
(b) P is sister of G
(c) P is brother of G
(d) None of these
(e) N is niece of P

36. If the expression S + Q $ T+ R * U is definitely true, then which of the


following is not true?
(a) Q is father of R
(b) R is nephew of S
(c) S is aunt of T
(d) None of these
(e) U is sister in law of S

Directions (37-39): These questions are based on the following


information. Study it carefully and answer the questions.
(i) ‘A × B’ means ‘A is father of B’
(ii) ‘A ÷ B’ means ‘A is daughter of B’
(iii) ‘A + B’ means ‘A is sister of B’
(iv) ‘A – B’ means ‘A is husband of B’
13 Adda247 Publications For any detail, mail us at
Publications@adda247.com
Cracker Book for Bank (IBPS | SBI | RRB PO | Clerk) Mains Exams

37. In F÷ R × H – L, how is H related to F ?


(a) Father (b) Brother (c) Sister
(d) cannot be determined (e) None of these

38. Which of the following indicates ‘N is mother of K’?


(a) K + L ÷ N × F (b) K + L ÷ N – M (c) H × K ÷ N
(d) N × F+ K (e) None of these

39. In F – R + H ÷ T, how is F related to T?


(a) Son-in-law (b) Daughter-in-law (c) Son
(d) Daughter (e) None of these

Direction (40): Study the following information to answer the given


question.
P + Q means P is the mother of Q.
P÷Q means P is the father of Q.
P×Q means P is the sister of Q.
P — Q means P is the brother of Q.

40. Which of the following should come in place of question mark in the
given expression to establish that A is the maternal grandfather of D?
‘A÷C×B ? D’
(a) + (b) — (c) ÷
(d) Either + or ÷ (e) None of these

Solutions

Direction (1-2):
1. (b);

And D is younger than both E and Q. So, the probable age of D is


23 yrs.
14 Adda247 Publications For any detail, mail us at
Publications@adda247.com
Cracker Book for Bank (IBPS | SBI | RRB PO | Clerk) Mains Exams

2. (a);

And V is younger than C.

Direction (3-5):

3. (c) 4. (d) 5. (c)

Directions (6-7):

6. (c); 7. (e);

Direction (8-10):
8. (c);

15 Adda247 Publications For any detail, mail us at


Publications@adda247.com
Cracker Book for Bank (IBPS | SBI | RRB PO | Clerk) Mains Exams

9. (d);

10. (b);

Directions (11-12):

11. (c); 12. (d);

Direction (13-14):

13. (e); 14. (c);

16 Adda247 Publications For any detail, mail us at


Publications@adda247.com
Cracker Book for Bank (IBPS | SBI | RRB PO | Clerk) Mains Exams

Direction (15-16):

15. (d); 16. (d);

Direction (17-18):

17. (c); 18. (a);

19. (c);

20. (e);

21. (e); After de-coding the given coded blood relation we get the
relation between N and R in which N is the nephew of R.

17 Adda247 Publications For any detail, mail us at


Publications@adda247.com
Cracker Book for Bank (IBPS | SBI | RRB PO | Clerk) Mains Exams

22. (a); From Expression 1 we get our final answer and deduce the
blood relation given below:

Directions (23-24):
23. (e);

24. (a);

25. (c);

26. (b);

27. (a);

18 Adda247 Publications For any detail, mail us at


Publications@adda247.com
Cracker Book for Bank (IBPS | SBI | RRB PO | Clerk) Mains Exams

28. (d);

29. (c);

Solutions (30-32):

30. (c); 31. (b); 32. (a);

Direction (33-34):

33. (d); 34. (b);


19 Adda247 Publications For any detail, mail us at
Publications@adda247.com
Cracker Book for Bank (IBPS | SBI | RRB PO | Clerk) Mains Exams

Directions (35-36):
35. (c);

36. (c);

Directions (37-39):
37. (b);

38. (c);

39. (a);

40. (a);

 Key Points to Remember for New Pattern Questions


➢ A Blood relation tree is nothing but a tree having its
branches which is considered as the different
generation. And the fruits or leaves of this tree are the
persons or members of that family.
20 Adda247 Publications For any detail, mail us at
Publications@adda247.com
Cracker Book for Bank (IBPS | SBI | RRB PO | Clerk) Mains Exams

➢ In blood relation the new concept that has been seen in


the question is the coded form of blood relation. It
consists of the codes for the different relations and we
have to decode the relations first then draw the tree.
➢ A few changes that has been seen is the blood relation
consist of different years of births of the family member
as it forms a puzzle. In this both the years of birth and
blood relation have to be solved simultaneously.
➢ The blood relation concept has also been comprised in
the seating arrangement in which the blood relation
plays an important role and the arrangement cannot be
solved without solving the blood relation.
➢ In some cases, the relations are already defined in the
puzzle and you have to arrange the persons in that
relation accordingly.

21 Adda247 Publications For any detail, mail us at


Publications@adda247.com
Cracker Book for Bank (IBPS | SBI | RRB PO | Clerk) Mains Exams

1 Adda247 Publications For any detail, mail us at


Publications@adda247.com
Cracker Book for Bank (IBPS | SBI | RRB PO | Clerk) Mains Exams

Chapter

11 Miscellaneous

Practice Exercise Based on new Pattern

Directions (1-5): Study the following alphanumeric series carefully and


answer the questions given below:
7 A 6 P & R $ 4 Y Q % T @ 3 9 S I O 9 9 J L E U *
K # 3

STEP I- The letters which are immediately preceded and immediately


followed by a symbol are arranged in the end of the series in the
alphabetical order. (They are arranged just after 3)

STEP II- The numbers which are immediately preceded by the letter and
immediately followed by the Symbol are arranged between 9 and S in the
increasing order.

STEP III- The numbers which are immediately followed by letter are
interchanged its position with respect to the element just after it.
(STEP II is applied after STEP I and STEP III is applied after STEP II)

1. How many letters are arranged at the end of the series in the step-1?
(a) one (b) Three (c) Four
(d) Five (e) More than five

2. Which among the following are the elements of the series which are
second position from the left end and fifth position from the right end
in step-III?
(a) 63 (b) 7# (c) P#
(d) AK (e) 6#

3. How many symbols are immediately followed by numbers in step-III?


(a) one (b) Three (c) Four
(d) Five (e) Two
2 Adda247 Publications For any detail, mail us at
Publications@adda247.com
Cracker Book for Bank (IBPS | SBI | RRB PO | Clerk) Mains Exams

4. Which of the following element is third to the left of the seventh


element from the right in step III?
(a) E (b) J (c) 9
(d) L (e) None of these

5. Which of the following is the third letter from the right end in step II?
(a) R (b) S (c) T
(d) K (e) None of these

Direction (6-10): In every question two rows are given and to find out the
resultant of a particular row you need to follow the following steps: -

Step 1: If an odd number is followed by a perfect square then the resultant


will be the subtraction of the square number from the odd number.

Step 2: If an even number is followed by an odd (prime) number then the


resultant will be the addition of both the numbers.

Step 3: If an even number is followed by an even number then the resultant


will be the difference of both the numbers.

Step 4: If an odd number is followed by another odd number then the


resultant will be the addition of both the numbers.

Step 5: If an even number is followed by an odd (non-prime) number


except (1) then the resultant will be the subtraction of the odd number
from the even number.

Step 6: If an odd number is followed by an even number then the resultant


comes by multiplying the numbers.

6. 8 7 6
11 9 5
Find the difference of two rows
(a) 35 (b) 83 (c) 31
(d) 63 (e) None of these

3 Adda247 Publications For any detail, mail us at


Publications@adda247.com
Cracker Book for Bank (IBPS | SBI | RRB PO | Clerk) Mains Exams

7. Find the resultant of second row if M is the resultant of first row.


3 7 9
M 12 8
(a) 11 (b) 20 (c) 4
(d) 3 (e) None of these

8. If the sum of the resultants of two rows is 275. Then find the value of
K?
15 20 42
9 K 5
(a) 4 (b) 9 (c) 3
(d) 5 (e) None of these

9. Find the multiple of the resultant of first and second row.


5 4 15
21 16 4
(a) 36 (b) 16 (c) 10
(d) 13 (e) None of these

10. If Q is the resultant of second row, then find the difference of the
resultant of two rows.
9 8 Q
12 9 4
(a) 68 (b) 72 (c) 73
(d) 71 (e) None of these

Direction (11-15): In every question two rows are given and to find out
the resultant of a particular row you need to follow the following steps: -
Note: (All the resultant value consider as a positive integer)

Step 1: If an odd number is followed by an even number then the resultant


will be the subtraction of both the numbers.

Step 2: If an even number is followed by a perfect cube then the resultant


will be the sum of the numbers.

4 Adda247 Publications For any detail, mail us at


Publications@adda247.com
Cracker Book for Bank (IBPS | SBI | RRB PO | Clerk) Mains Exams

Step 3: If an odd number is followed by another odd number then the


resultant will be the subtraction of both the numbers.

Step 4: If an even number is followed by an odd (prime) number then the


resultant will be the subtraction of both the number.

Step 5: If above four conditions are not applied than simply add both the
number.

11. Find the resultant sum of two rows?


8 27 2
15 7 3
(a) 18 (b) 25 (c) 35
(d) 38 (e) None of the above

12. Find the resultant of second row if D is the resultant of first row.
9 7 2
13 D 3
(a) 5 (b) 7 (c) 9
(d) 3 (e) 6

13. If the resultants value of second rows is 7, then find the value of L?
24 27 12
12 3 L
(a) 12 (b) 16 (c) 15
(d) 9 (e) None of the above

14. Find the multiplication of the resultant value of first and second row.
4 8 8
27 6 7
(a) 441 (b) 156 (c) 625
(d) 400 (e) 280

5 Adda247 Publications For any detail, mail us at


Publications@adda247.com
Cracker Book for Bank (IBPS | SBI | RRB PO | Clerk) Mains Exams

15. If Z is the resultant of second row, then find the sum of the resultant of
two rows.
Z 15 36
6 512 100
(a) 633 (b) 518 (c) 618
(d) 597 (e) 1215

Directions (16-20): Study the following alphanumeric series carefully and


answer the questions given below:
12AIFR 42 WBOL 63EOCN
15TVMR 75GYTR

16. How many meaningful words can be formed from the letters of the
words which are attached with the numbers in which At least one
digit is even digit in the given alphanumeric series?
(a) one (b) two (c) three
(d) four (e) More than four

17. The words are arranged according to the descending order of the
numbers which are attached to them from left to right, then how
many alphabets are between the letters of second letter from the left
end and seventh letter from the right end?
(a) one (b) two (c) three
(d) four (e) More than five

18. The letters which are attached with the numbers in which at least one
is odd digit are arranged according to the ascending orders of their
numbers from left to right and then all letters of the words together
are arranged in the alphabetical order, then which of the following is
the 9th letter from the right end?
(a) M (b) O (c) N
(d) R (e) None of these

19. How many meaningful words can be formed from the letters of the
words which are attached with the numbers in which both the digits
are even digit in the given alphanumeric series?
(a) Three (b) Two (c) Four
(d) One (e) None of these
6 Adda247 Publications For any detail, mail us at
Publications@adda247.com
Cracker Book for Bank (IBPS | SBI | RRB PO | Clerk) Mains Exams

20. How many meaningful words can be formed from the letters of the
words which are attached with the numbers in which both the digit
are odd digit in the given alphanumeric series??
(a) Three (b) Two (c) Four
(d) None (e) None of these

Direction (21-25): In every question two rows are given and to find out
the resultant of a particular row you need to follow the following steps: -

Condition I: If an odd number is followed by a perfect square then the


resultant will be the multiplication of both the numbers.

Condition II: If an even number is followed by an odd (prime) number then


the resultant will be the division of both the numbers.

Condition III: If an even number is followed by an even number then the


resultant will be the difference of both the numbers.

Condition IV: If an odd number is followed by another odd number then


the resultant will be the addition of both the numbers.

Condition V: If an even number is followed by an odd (non-prime) then the


resultant will be the subtraction of both the numbers.

Note: Resultant value is always considered as positive integer. And Greater


number is dividing by the smaller number.

21. 12 8 8
7 16 5
Find the sum of two rows?
(a) 135 (b) 133.4 (c) 134.5
(d) 134.4 (e) None of these

22. Find the value of the multiplication of both row’s resultant value than
addition of 1.
12 27 5
1 3 23
(a) 116 (b) 120 (c) 114
(d) 113 (e) None of these
7 Adda247 Publications For any detail, mail us at
Publications@adda247.com
Cracker Book for Bank (IBPS | SBI | RRB PO | Clerk) Mains Exams

23. If the multiplication of the resultants of two rows is 84. Then find the
value of X?
12 9 4
4 X 9
(a) 4 (b) 9 (c) 3
(d) 5 (e) None of these

24. Find the division of the sum of the resultant of first and second row.
30 5 2
17 11 12
(a) 4 (b) 2 (c) 8
(d) 6 (e) None of these

25. If X is the resultant of the first and second digit of the second row,
then find the sum of the resultant of two rows.
24 9 X
7 9 3
(a) 142 (b) 144 (c) 154
(d) 152 (e) None of these

Direction (26-30): There are two rows given and to find out the resultant
of a particular row we need to follow the following steps: -

Step 1: If an even number is followed by an odd number then the resultant


will be the addition of both the numbers.

Step 2: If an odd number is followed by a perfect square then the resultant


will be the difference of that square number and the odd number.

Step 3: If an odd number is followed by another odd number (but not a


perfect square) then the resultant will be the addition of both the numbers.

Step 4: If an odd number is followed by an even number (but not a perfect


square) then the resultant comes by multiplying the numbers.

Step 5: If an even number is followed by another even number then the


resultant will be the division of first number by the second number.
8 Adda247 Publications For any detail, mail us at
Publications@adda247.com
Cracker Book for Bank (IBPS | SBI | RRB PO | Clerk) Mains Exams

26. Find the sum of two rows


8 4 1
11 6 7
(a) 78 (b) 52 (c) 64
(d) 76 (e) None of the above

27. If the sum of the resultants of two rows is 46. Then find the value of X.
9 2 7
24 4 X
(a) 16 (b) 27 (c) 8
(d) 15 (e) None of the above

28. Find the difference between the resultant of first and second row.
13 3 7
4 11 12
(a) 117 (b) 126 (c) 157
(d) 96 (e) None of the above

29. Find the multiplication of the resultant of first and second row.
21 19 8
16 13 9
(a) 110 (b) 85 (c) 100
(d) 120 (e) None of the above

30. If the multiplication of the resultant of first and second row is 39, then
find the value of ‘X’.
7 5 4
6 2 X
(a) 11 (b) 15 (c) 16
(d) 12 (e) None of the above

9 Adda247 Publications For any detail, mail us at


Publications@adda247.com
Cracker Book for Bank (IBPS | SBI | RRB PO | Clerk) Mains Exams

Solutions

Solution (1-5):
Input :
7 A 6 P & R $ 4 Y Q % T @ 3 9 S I O 9 9 J L E U
* K # 3

Step 1:
7 A 6 P & $ 4 Y Q % @ 3 9 S I O 9 9 J L E U * #
3 K T R

Step 2:
7 A 6 P & $ 4 Y Q % @ 3 9 S I O 9 9 J L E U * #
3 K T R

Step 3:
A 7 P 6 & $ Y 4 Q % @ 3 S 9 I O 9 J 9 L E U * # K
3 T R

1. (b); 2. (b); 3. (a);


4. (c); 5. (d);

Solutions (6-10):
6. (b); In row-1
Even number is followed by an odd prime number so=8+7= 15
15 6
Odd number is followed by an even number so=15*6= 90
In row-2
Odd number is followed by a perfect square so=11-9= 2
2 5
Even number is followed by an odd prime number so=2+5= 7
So the difference of both row = 90 - 7 = 83
10 Adda247 Publications For any detail, mail us at
Publications@adda247.com
Cracker Book for Bank (IBPS | SBI | RRB PO | Clerk) Mains Exams

7. (c); In row-1
Odd number is followed by another odd number so=3+7= 10
10 9
Even number is followed by an odd (non-prime) number so=10-
9= 1
From row-1 M value is 1 so,
In row-2
Odd number is followed by an even number so=1*12= 12
12 8
Even number is followed by an even number so=12-8= 4
So the resultant of the second row = 4

8. (c); In row-1
Odd number is followed by an even number so=15*20= 300
300 42
Even number is followed by an even number so=300-42= 258
In row-2
When put K=3 then,
3 5
Odd number is followed by another odd number so=9+3= 12
12 5
Even number is followed by an odd (prime) number so=12+5=
17
So the Sum of the resultant of the both rows = 275
So this condition can satisfy only when K=3.

9. (b); In row-1
Odd number is followed by a perfect square so=5-4= 1
1 15
Odd number is followed by another odd number so=1+15= 16
In row-2
Odd number is followed by a perfect square so=21-16= 5
5 4
Odd number is followed by a perfect square so=5-4 = 1
So the multiple of the resultant of the both rows = 16*1= 16

11 Adda247 Publications For any detail, mail us at


Publications@adda247.com
Cracker Book for Bank (IBPS | SBI | RRB PO | Clerk) Mains Exams

10. (b); In row-2


Even number is followed by an odd (non-prime) number so=12-
9= 3
3 4
Odd number is followed by a perfect square so=3-4= (-1)
So the value of Q= (-1)
In row-1
Odd number is followed by an even number so=9*8= 72
72 (-1)
Even number is followed by an odd (prime) number so=72+ (-1)
= 71
So difference of the resultant of two rows = 71-(-1) = 72

Solution (11-15):
11. (d); Row-I: Even number is followed by a perfect cube= 8+27=35
Now 35 2 odd number is followed by an even number=35-2=33
Row-II: Odd number is followed by another odd=15-7=8
8 3 even number is followed by an odd (prime) number=8-3=5
Sum of both the row= 33+5=38

12. (e); Row-I: odd number is followed by another odd number= 9-7=2
2 2 above four conditions are not applied=2+2=4
Row-II: odd number is followed by another even number=13-
4(D)=9
9 3 odd number is followed by another odd number=9-3=6

13. (b); Row-I: even number is followed by an odd (prime) number=3


3 12 odd number is followed by an even number= 12-3=9
Row-II: even number is followed by an odd (prime)
number=12-3=9
9 L=16 odd number is followed by an even number number 16-
9=7

12 Adda247 Publications For any detail, mail us at


Publications@adda247.com
Cracker Book for Bank (IBPS | SBI | RRB PO | Clerk) Mains Exams

14. (e); Row-I: even number is followed by a perfect cube 4+8=12


12 8 even number is followed by a perfect cube 12+8=20
Row-II: odd number is followed by an even number 27-6=21
21 7 odd number is followed by another odd number=21-7=14
Multiplication= 20*14=280

15. (e); Row-II: even number is followed by a perfect cube= 512+6=518


518 100 above four conditions are not applied=518+100=618
Row-I: Z=618
Above four conditions are not applied = 618 + 15 = 633
Odd number is followed by an even number=633-36=597
Resultant of two row= 618+597=1215

Solutions (16-20):
16. (b); There are Five meaningful words- Fair, Bowl, Blow, Cone, Once

17. (b);

18. (c);

19. (b); There are two words can be made ‘Bowl and Blow’.

20. (d); There are two such words ‘TVMR and GYTR’.
But no meaningful words can be formed.

Solutions (21-25):
21. (e); Row-I: Even number is followed by an even number= 12 – 8 = 4
Even number is followed by an even number = 8 – 4 = 4
Row-II: odd number is followed by a perfect square= 7*16=112
Even number is followed by an odd (prime)
number=112/5=22.4
Sum of both the row= 26.4

22. (a); Row-I: even number is followed by an odd (non-prime) =27-


12=15

13 Adda247 Publications For any detail, mail us at


Publications@adda247.com
Cracker Book for Bank (IBPS | SBI | RRB PO | Clerk) Mains Exams

Odd number is followed by another odd number= 15+5=20


Row-II: odd number is followed by another odd number= 1+3=4
Even number is followed by an odd (prime) number=23/4=5.75
Multiplication of both row’s resultant value than addition of 1=
5.75*20=115+1=116

23. (e); Row-I: even number is followed by an odd (non-prime) = 12-


9=3
Odd number is followed by a perfect square = 3*4=12
Row-II: Even number is followed by an even number = 6-4=2
Even number is followed by an odd (non-prime) = 9-2=7
Multiplication of both the row’s resultant value = 12*7 = 84

24. (a); Row-I: even number is followed by an odd (prime) number =


30/5=6
Even number is followed by an even number = 6-2=4
Row-II: odd number is followed by another odd number =
17+11= 28
Even number is followed by an even number = 28-12 = 16
Division= 16/4 = 4

25. (b); Row-II: odd number is followed by a perfect square = 7*9 = 63


Odd number is followed by another odd number = 63+3=66
Row-I: Even number is followed by an odd (non-prime) = 24-
9=15
Odd number is followed by another odd number = 15+63 = 78
Sum of the resultant of two rows = 78+66= 144

Solutions (26-30):
26. (d); Row-I: even number is followed by an even number = 8/4=2
Even number is followed by an odd number = 2+1=3
Row-II: odd number is followed by even number = 11*6= 66
Even number is followed by an odd number = 66+7 = 73
Sum of two rows= 73+3= 76

14 Adda247 Publications For any detail, mail us at


Publications@adda247.com
Cracker Book for Bank (IBPS | SBI | RRB PO | Clerk) Mains Exams

27. (d); Row-I: odd number is followed by an even number = 9*2= 18


Even number is followed by an odd number = 18+7=25
Row-II: even number is followed by another even number =
24/4= 6
Difference of total and first row is = 46-25= 21
The resultant of second row is 21
Hence, if X= 15 then,
Even number followed by odd number= 6+15= 21.

28. (c); Row-I: odd number is followed by an odd (not perfect square)
number = 13+3= 16
Even number is followed by an odd number = 16+7= 23
Row-II: even number is followed by odd number = 4+11= 15
odd number is followed by an even number = 15*12 = 180
The difference of the resultants= 180-23= 157

29. (c); Row-I: odd number is followed by an odd number = 21+19 = 40


Even number is followed by an even number = 40/8 = 5
Row-II: even number is followed by another odd number =
16+13= 29
odd number is followed by an odd (perfect square) number =
29-9 = 20
Multiplication= 20*5 = 100

30. (c); Row-I: odd number is followed by an odd number = 7+5= 12


Even number is followed by an even number = 12/4=3
Resultant of both rows is 39 i.e. 3*13= 39 (resultant of second
row is 13).
Row-II: even number is followed by another even number =
6/2= 3
Odd number is followed by an even (perfect square) number =
16-3 = 13
So, X= 16

15 Adda247 Publications For any detail, mail us at


Publications@adda247.com
Cracker Book for Bank (IBPS | SBI | RRB PO | Clerk) Mains Exams

1 Adda247 Publications For any detail, mail us at


Publications@adda247.com
Cracker Book for Bank (IBPS | SBI | RRB PO | Clerk) Mains Exams

Chapter

1 Inferences
STUDY TIPS

Questions based on inference can be of two types, either the inference


is given and the aspirant must identify the relevant paragraph or a
paragraph is given followed by few probable inferences. Understanding
the essence of the paragraph helps to deduce the inference effectively.
While reading the paragraph one must observe that it provides the
complete information to arrive at the appropriate inference.

Practice Exercise Based on new Pattern


Direction (1-14): In each of the given questions an inference is given in
bold which is then followed by three paragraphs. You must find the
paragraph(s) from where it is inferred. Choose the option with the best
possible outcome as your choice.

1. Electric vehicles can reduce urban pollution significantly.


[I] Addressing vehicular emissions is within our grasp but requires a
multi-pronged approach. It needs to combine the already-proposed
tighter emission norms (in form of BS VI), with a push for shared
mobility and public transport and adoption of alternate mobility
technologies. While shared mobility can moderate the demand for
individual vehicle ownership and usage, technology solutions today
can allow for a sharp reduction in emissions per vehicle. Government
policy will impact adoption that will affect both the extent and the
future growth of urban pollution.

[II] The policy roadmap should encompass three key elements based on
global learnings. First, incentives for adoption of alternate mobility
technologies. Second, restrictions on elements that contribute
negatively to strategic objectives (such as congestion charges on
polluting technologies), and last provision of enabling infrastructure.

2 Adda247 Publications For any detail, mail us at


Publications@adda247.com
Cracker Book for Bank (IBPS | SBI | RRB PO | Clerk) Mains Exams

[III] There is a need to impose restrictions through supply-side regulations


on OEMs to increase production of zero emission vehicles to curb
urban pollution. Most Western countries adopt enforceable norms
that ensure supply of electric vehicles. China has mandated OEMs to
produce 10 per cent electric vehicles of their total production. There
will, of course, be the need to think about improving the provision of
non-polluting public transport. These include electric buses, metros,
and shared EV fleets to reduce traffic and usage.
(a) only (II) (b) only (III) (c) both (I) and (III)
(d) all (I), (II) and (III) (e) none of these

2. NHPS’s model tender document leaves several questions


unanswered.
[I] Immediately after the announcement of National Health Protection
Scheme (NHPS), questions were raised over the capacity of the
country’s healthcare system to handle a project of such proportions.
There were also apprehensions that private hospitals would milk the
NHPS by prescribing unnecessary investigations. The Model Tender
Document for The Selection of Implementing Agencies For the NHPS,
released by the Union Ministry of Health and Family Welfare, tries to
address some of these concerns.

[II] The model tender document for The Selection of Implementing


Agencies For the NHPS, released by the Union Ministry of Health and
Family Welfare states that several procedures, including emergency
consultation for acute colic, nebulisation for an asthma attack,
hypoglycaemia in a diabetic and treatment of “dengue without
complication”, will be covered by the scheme only if the treatment is
availed in a government hospital. Such ailments can indeed be taken
care of at a primary health centre (PHC)
[III] The model tender document states that nearly 47 per cent of the
packages under the NHPS, including those related to heart ailments
and cancer, require pre-authorisation. In other words, hospitals
empanelled under the scheme cannot perform these procedures until
they have an authorisation letter from the NHPS’s Implementation
Support Agency. Such concerns were raised when the scheme was
announced. With about two months to go for the NHPC’s launch, it is
disquieting that the government has not yet managed to address them
convincingly.
(a) only (II) (b) only (III) (c) both (I) and (III)
(d) all (I), (II) and (III) (e) none of these
3 Adda247 Publications For any detail, mail us at
Publications@adda247.com
Cracker Book for Bank (IBPS | SBI | RRB PO | Clerk) Mains Exams

3. Latest alterations in textbooks short-circuit established


processes, undermine NCERT autonomy
[I] In a major break with practices until 2016, in the current process of
alterations in NCERT books, there was no consultation between the
NCERT’s chief advisors and the TDCs that prepared the books during
2005-9. There was no alert to writer-contributors who had not
waived their rights over their contributions. Nor is there any
indication that a prevailing system of revision, referring to TDCs, has
itself been officially revised. The names that figure on the books as
“textbook development committees” remain the same, though most of
them have not been involved in the insertion/revision process.

[II] The fact that much of the textbook material has been left alone in the
latest NCERT changes indicates that the pedagogic purpose and
outcome of the initiative are still able to argue for themselves; and
that the initiative is still valued in the NCERT establishment. This, in
turn, raises questions about why the current changes have been made
as they have. The long arm of political directive is suggested — in an
autonomous body that has shown its ability to take an imaginative
course while generating a discursive relationship with all those
involved in education, without rendering itself an arm of the state.

[III] In history, this was to be achieved with due attention to the formative
processes in Indian and world history from a plurality of perspectives;
in economics, sociology and political studies, commerce and
geography, it involved a wide invocation of the experiences of India’s
various communities while preserving the interactive domain of the
Social Sciences and the value of the disciplines individually. A series of
NCERT Focus Group Reports of the mid-2000s articulated aspects of
this agenda.
(a) both (I) and (II) (b) only (III) (c) only (II)
(d) all (I), (II) and (III) (e) none of these

4 Adda247 Publications For any detail, mail us at


Publications@adda247.com
Cracker Book for Bank (IBPS | SBI | RRB PO | Clerk) Mains Exams

4. Since policy-making and implementation increasingly need


specialists, lateral entry into senior levels of bureaucracy is a
good idea.
[I] The UPSC system does draw people from diverse educational
backgrounds — doctors, engineers, graduates in the social sciences,
humanities and management studies — into the Indian
Administrative Service (IAS). But the IAS’s scheme of posting and
transfer values general competency more than specialised skills. This
means that by the time a bureaucrat attains seniority, she has served
in so many departments that her original set of skills and expertise
has attenuated considerably.

[II] The Centre’s decision to make possible “lateral entry” of “talented and
motivated Indian nationals” into the senior levels of the bureaucracy
is a much-needed reform. In an advertisement issued on Sunday, the
Department of Personnel and Training invited applications from
outstanding individuals, including those from the private sector, for
appointment to joint secretary-level posts. Although it is an initial
offering of 10 posts in areas such as financial services, agriculture,
environment, renewable energy, transport and revenue, the move
could be a significant step towards fulfilling the longstanding need for
domain specialists in positions crucial to policy-making and
implementation of government schemes.

[III] In the past, governments have occasionally inducted talent from


outside the bureaucracy for administrative purposes. The UPA
government appointed Nandan Nilekani to head the UIDAI. But in
general, governments have tried to meet the need for experts by
appointing consultants. The Second Administrative Reforms
Commission’s (ARC) recommendation of an “institutionalised and
transparent process for lateral entry at both the Central and state
levels” had so far gone unheeded.
(a) both (I) and (II) (b) only (III) (c) only (II)
(d) all (I), (II) and (III) (e) none of these
5 Adda247 Publications For any detail, mail us at
Publications@adda247.com
Cracker Book for Bank (IBPS | SBI | RRB PO | Clerk) Mains Exams

5. SBI to stop handling payments for oil imports from Iran.


[I] Although the government had cut imports from Tehran in 2017/18
due to a dispute over a giant gas field, Iran remained its third-biggest
oil supplier. Iran supplied about 458,000 barrels per day (bpd), or
about a tenth of the country's more than 4.5 million bpd of imports, in
the fiscal year to March 2018.

[II] The new Indian government has promised to put the economy back
on an accelerated growth path with reforms in the energy, financial,
and employment sectors. Energy is the backbone of the Indian
economy, so the right energy policies will spur growth in all other
sectors. With India soon expected to be the world’s third largest
energy consumer, there is an urgent need to get these right: current
demand for imported coal, oil, and natural gas is significantly
outpacing domestic production, and the country is being forced to
spend valuable foreign capital to procure additional energy resources.

[III] Imports was reported at 4,308.30 Barrel/Day th in Dec 2016. This


records an increase from the previous number of 3,935.50 Barrel/Day
th for Dec 2015. India’s Crude Oil: Imports data is updated yearly,
averaging 2,078.15 Barrel/Day th from Dec 1995 to 2016, with 22
observations. The data reached an all-time high of 4,308.30
Barrel/Day th in 2016 and a record low of 602.50 Barrel/Day th in
1995.
(a) both (I) and (II) (b) only (III) (c) only (II)
(d) all (I), (II) and (III) (e) none of these

6. South Korea’s President doesn’t share U.S. goals on North Korea.


[I] South Korean leader Moon claims to be the mediator between the
White House and Pyongyang, but it isn't clear he's representing the
U.S. position. Instead he is pressing the U.S. to give benefits to North
Korea in return for mere steps toward denuclearization. He has
adopted the North's position that the negotiations should agree to
"phased and synchronous measures," meaning the North gets benefits
in exchange for incremental steps such as allowing inspectors to visit
nuclear sites.
6 Adda247 Publications For any detail, mail us at
Publications@adda247.com
Cracker Book for Bank (IBPS | SBI | RRB PO | Clerk) Mains Exams

[II] The matter of denuclearization of North Korea became worse since


Moon was personally invested in a Trump-Kim summit. He and his
administration worked tirelessly to make it happen in hopes that
Washington and Pyongyang could agree on the future of North
Korea’s nuclear program. Of course, part of his motivation to do this
was to protect South Koreans from Pyongyang’s nuclear arsenal.
However, trump calls off the meeting
[III] North Korea’s entire foreign policy and national identity has evolved
around the threat of war with America. As a result, they’ve always
been trying to improve their military capabilities in order to deter the
US from invading.
(a) only (I) (b) only (II) (c) only (III)
(d) Both (I) and (II) (e) All (I), (II) and (III)

7. Banks needs to step up the digital game.


[I] With digital platforms are enjoying increasing customer trust and
reliance, banks need to respond by improving their digital marketing
capabilities — nurturing a digital brand image that places them top of
mind for customers. Customer adoption of digital banking has gone
beyond online transactions to an increased number of digital
purchases of banking products.
[II] Traditional banks must evaluate their place within the payments
ecosystem and be open to partnering with FinTechs and third-party
developers to drive value collaboratively. The Internet of Things (IoT)
is expected to transform the payments market landscape as more and
more devices are connected. In mature markets, a combination of
near field communication (NFC), contactless technology, internet
banking and mobile payments are fueling growth.
[III] In Asia, customers rely quite heavily on the internet to evaluate
banking products such as credit cards and auto loans, and to compare
products, benefits and prices before purchase. For banks, a focused
digital marketing effort will involve building up their online brand and
presence and investing in customer acquisition campaigns and
engagement through social media, internet advertising and other
media.
(a) only (I) (b) only (II) (c) only (III)
(d) Both (I) and (III) (e) All (I), (II) and (III)
7 Adda247 Publications For any detail, mail us at
Publications@adda247.com
Cracker Book for Bank (IBPS | SBI | RRB PO | Clerk) Mains Exams

8. Rapid changes in climate is becoming a threat to agriculture.


[I] Agricultural pollution refers to biotic and abiotic byproducts of
farming practices that result in contamination or degradation of the
environment and surrounding ecosystems, and/or cause injury to
humans and their economic interests. The pollution may come from a
variety of sources, ranging from point source water pollution (from a
single discharge point) to more diffuse, landscape-level causes, also
known as non-point source pollution.
[II] Climate change has perhaps posed the most extreme challenges that
agriculture in India and across the world has to deal with today and in
the future. There is now scientific consensus that the world is getting
warmer due to climate change and such increasing weather variability
and worsening extremes will impact the agriculture sector more and
more adversely.
[III] Climate change is already affecting agriculture, with effects unevenly
distributed across the world. Future climate change will likely
negatively affect crop production in low latitude countries, while
effects in northern latitudes may be positive or negative. Climate
change will probably increase the risk of food insecurity for some
vulnerable groups, such as the poor. Animal agriculture is also
responsible for greenhouse gas production of CO2 and a percentage of
the world's methane, and future land infertility, and the displacement
of local species.
(a) Both (II) and (III) (b) only (II) (c) only (III)
(d) only (I) (e) All (I), (II) and (III)

9. Nanotechnology has benefitted by simplifying the complex


procedures of many fields.
[I] Many benefits of nanotechnology depend on the fact that it is possible
to tailor the structures of materials at extremely small scales to
achieve specific properties, thus greatly extending the materials
science toolkit. Using nanotechnology, materials can effectively be
made stronger, lighter, more durable, more reactive, more sieve-like,
or better electrical conductors, among many other traits. Moreover,
nanoscale additives to or surface treatments of fabrics can provide
lightweight ballistic energy deflection in personal body armor, or can
help them resist wrinkling, staining, and bacterial growth.

8 Adda247 Publications For any detail, mail us at


Publications@adda247.com
Cracker Book for Bank (IBPS | SBI | RRB PO | Clerk) Mains Exams

[II] Authors David Yeo, Ph.D., and Prof. Chenjie Xu, Ph.D., of the School of
Chemical and Biomedical Engineering at Nanyang Technological
University (Singapore) use NanoFlare to enable biopsy-free disease
diagnosis and progression monitoring in response to therapy. This
vision of simplifying disease diagnosis using topically-applied
nanotechnology could change the way skin diseases such as abnormal
scars are diagnosed and managed.
[III] Nanotechnology shows promise here and researchers have already
begun to develop nano-based versions of existing pesticides and
fertilizers. These nanoagrochemicals have several advantages over
conventional formulas – for example, they might be delivered directly
to a pest and/or may be more efficient.
(a) Both (II) and (III) (b) only (II) (c) only (III)
(d) only (I) (e) All (I), (II) and (III)

10. Policy forces schools to sweep learning deficits at the


foundational level under the carpet.
[I] Council monitors the school's financial performance against the
budget in conjunction with the principal. The convenor of the finance
committee, as elected from council members, is preferably a non-
Department parent member or a community member. The business
manager/bursar should not hold this position. The convenor may be
appointed as treasurer by council.
[II] The findings of the National Achievement Survey (NAS) 2018, which
tested learning outcomes in schools, reflect the damage the Right to
Education’s ‘no detention till Class VIII’ policy has caused. The Union
government moved last year to scrap the policy—after 24 states
insisted on this—and it is likely to go soon. But given it has been in
effect for over eight years, it would likely have impaired learning for
millions of Indian students.
[III] The no-detention policy was implemented to emulate education
policy in many developed jurisdictions—the idea was to lessen the
stress of exams on students in the junior classes. It also had an
ancillary benefit for the government; it kept primary level enrolment
numbers high.
(a) Both (II) and (III) (b) only (II) (c) only (III)
(d) only (I) (e) All (I), (II) and (III)
9 Adda247 Publications For any detail, mail us at
Publications@adda247.com
Cracker Book for Bank (IBPS | SBI | RRB PO | Clerk) Mains Exams

11. A change in present political reforms is required to support


farmers.
[I] Farmers are unhappy across India. Politicians are scrambling to waive
loans, raise procurement prices and promise more handouts. These
will change little. Radical, multi-faceted reform is needed, and that
calls for politics, not magical promises to double farmer incomes by
2022.

[II] Today, the government arbitrarily curtails royalty, pushing out


companies that do research and development. But improved
productivity will not enrich farmers by itself. Farmers need marketing
freedom and the ability to capture as high a share as is possible of the
value that their produce acquires along the way to final consumption.

[III] Agriculture easily gets 2% of GDP as subsidy. The result is still misery
and farm unrest. What if farmers can be persuaded to give up
traditional subsidy on inputs? A part of the subsidy savings can still go
as income support, and the rest ploughed into productivity-enhancing
investment, of which there has been a decline under the present
government. This calls for improved political reforms, and political
will.
(a) only (II) (b) both (I) and (II) (c) only (III)
(d) all (I), (II) and (III) (e) none of these

12. The government needs to handle public sector banks with care.
[I] Former Reserve Bank of India (RBI) governor Y.V. Reddy, in a speech
last week, said that confidence in the working of public sector banks is
at a historic low. The reason for this is not very difficult to discern.
PSU banks are grappling with a high level of bad loans, and a number
of them have been put under RBI’s prompt corrective action and are
not in a position to lend. In the March quarter, PSU banks booked
losses in excess of Rs 62,000 crore and the total gross non-performing
assets (NPAs) stood at about Rs 9 trillion.
[II] Although the government is in the process of recapitalising state-run
banks, it is likely that the current Rs 2.11 trillion PSU bank
recapitalization plan will not be sufficient to put the PSU banks back

10 Adda247 Publications For any detail, mail us at


Publications@adda247.com
Cracker Book for Bank (IBPS | SBI | RRB PO | Clerk) Mains Exams

on track. Since PSU banks own about 70% of banking assets, their
inability to lend will have a direct impact on economic growth.
Therefore, it is important that the situation is handled with care.
[III] Apart from capital needs and faster resolution of stress assets, PSU
banks need governance reforms—something that has been largely
missing so far from the picture. It is correct that the present
government has refrained from micromanaging PSU banks, but this in
itself will not solve the problem. The government, perhaps, needs to
put in place a new framework for governance where, for instance,
appointments at higher levels are made in time, and the board is
professional and accountable.
(a) only (II) (b) both (II) and (III) (c) only (III)
(d) all (I), (II) and (III) (e) none of these

13. India needs a new education system.


[I] In this upcoming age of artificial intelligence, we need teachers
equipped with such technological expertise, more importantly
equipped with the mental make-up to learn the ever-changing
intricacies of education. The need of the hour is to make the education
sector alluring for today’s youth entering the market as a precious
human resource. “The future of the country is in what lies in my
classroom today.”
[II] India has made progress in terms of increasing the primary education
attendance rate and expanding literacy to approximately three-
quarters of the population in the 7–10 age group, by 2011. India's
improved education system is often cited as one of the main
contributors to its economic development. Much of the progress,
especially in higher education and scientific research, has been
credited to various public institutions.
[III] Educationalists are of the view that we are currently in the third
phase of the education system. Education sector comprises of three
vertices i.e. teacher, syllabus (subject) and student. The first phase of
education had its focus on the ‘guru’ (teacher). The teacher had
autonomy to decide the time, place and domain of education. Neither
the subject nor the student had any predomination. It was the teacher
who enjoyed prestige and power. Ancient Gurukuls are the examples
of this system of education.
(a) only (I) (b) both (II) and (III) (c) only (III)
(d) all (I), (II) and (III) (e) none of these
11 Adda247 Publications For any detail, mail us at
Publications@adda247.com
Cracker Book for Bank (IBPS | SBI | RRB PO | Clerk) Mains Exams

14. Skill development of the youth should pay heed to the market
[I] Organizations must focus on understanding aspirations, industry
requirements and standardization across the skill-development value
chain. Well-designed interventions will be effective only if the
candidates are willing, receptive and capable of absorbing the
knowledge or skill being imparted by the intervention. Counselling in
skilling programmes is hence essential to align the aspirations of
programme beneficiaries with the expected outcomes of training.
Further, candidates may already possess specific complementary skill
sets that could provide them with a competitive advantage. A
candidate-selection framework would greatly enhance the efficacy of
such interventions.
[II] While designing programmes, it is critical to map skills being
imparted to the specific needs of potential employers so that the
skilling-to-employment loop is closed seamlessly. NSDC plans to move
to a model where training partners will receive funds as per the
outcomes achieved. The movement towards outcome-based funding is
a welcome step towards strengthening future programme design.
[III] India’s demography provides a great opportunity for the country with
regard to economic growth and development milestones.
Concentrated and evidence-backed efforts which can cohesively
develop and strengthen youth aspirations, the skill development
ecosystem and markets where youth can be employed are necessary
for India to realize that opportunity.
(a) Only (I) (b) Both (II) and (III) (c) Only (II)
(d) All (I), (II) and (III) (e) None of these

Direction (15-16): In this question a small paragraph is given followed by


three possible inferences which may or may not be correct. The question is
then followed by five options. You have to choose the option(s) which can
be inferred from the given paragraph.
15. The proposal of digitising at least part of the public examination is
certainly a step in the right direction. It is more beneficial than the
current system of public examination and if successful, it has
tremendous potential for growth in the future. If the idea is
implemented correctly, the digitised public examination system may
12 Adda247 Publications For any detail, mail us at
Publications@adda247.com
Cracker Book for Bank (IBPS | SBI | RRB PO | Clerk) Mains Exams

become the flagship project that would vindicate the legacy of the
current administration and create new frontiers of educational
innovations for administrations to come.
[I] A move towards the digitized public examinations should be
contemplated.

[II] digitized public examinations will be easy to administer.

[III] digitization exams could really stop the question paper leak crisis.
(a) only (II) (b) both (II) and (III) (c) only (I)
(d) all (I), (II) and (III) (e) none of these

16. The best-ranked universities of the world have, on an average, a


student population of 18,000 per university. In comparison, the best-
ranked Indian universities are operating at a much smaller scale,
averaging at around 11,000 students per institution. Besides, the best
universities of the world have a good mix of undergraduate and
postgraduate students, with Masters and Research Degree students
constituting a sizeable section of their student population, thereby
ensuring a critical mass of students which, in turn, promote better
graduation outcome, research, and reputation.
[I] participation of higher educational institutions of India in global
rankings has risen over time.
[II] Population of students of Indian universities are declining in
comparison to the world’s best universities.
[III] Enabling universities in India to attain world-class status.
(a) only (II) (b) both (II) and (III) (c) only (I)
(d) all (I), (II) and (III) (e) none of these
Direction (17-30): In these questions a small paragraph is given followed
by four possible inferences which may or may not be correct. You have to
choose the option which gives the best possible inference of the given
paragraph. If none of the given option successfully infer the given
paragraph, choose option (e) i.e., “none of these” as your answer choice.

17. There is a way to dramatically cut down on the number of people


impacted by disasters, and that is by using data. If we are to save lives
and prevent damage to economies, it is critical to identify the most
vulnerable populations. Data on these communities can be used to
13 Adda247 Publications For any detail, mail us at
Publications@adda247.com
Cracker Book for Bank (IBPS | SBI | RRB PO | Clerk) Mains Exams

pursue ‘risk-informed development’. For instance, road infrastructure


can be built by calculating the intensity of floods and determining the
types of materials needed to construct durable roads.
(a) Data can help reduce the number of people impacted by natural
disasters
(b) Data and statistics are important in understanding the impacts
and costs of disasters.
(c) Data collection, analysis, and management can help both short
and long-term development goals
(d) Data identifies the gap between the availability and allocation of
resources.
(e) none of these

18. While one can debate the economic costs and benefits of GST, the
analogy with elections is logically flawed. Indeed, the concept of
simultaneous elections fundamentally runs against the grain of our
Westminster-style federal political union. “One nation, one election”
would make sense if India were a unitary state. But we are a union of
states, which is philosophically and politically an essentially different
conception of the Indian nation-state.
(a) Simultaneous elections are a "ploy" to keep people away from
forming a government.
(b) GST swept away a raft of state-specific excise taxes and replaced
them with a uniform tax structure.
(c) simultaneous elections would require a slew of constitutional
amendments.
(d) simultaneous elections would compromise the strength of India’s
federal structure.
(e) none of these

19. Capitalism rests on the bedrock of legitimacy. A lot depends on


whether voters see businessmen as robber barons or men of
enterprise. In a fascinating paper, which was first published in 2013
but has come into prominence more recently, economist Nimish Adhia
has shown that the 1991 economic reforms were preceded by a shift
in popular narratives about capitalism in Hindi films.
(a) Heavy regulation promotes corruption.
(b) Movies that celebrated wealth did well at the box office.

14 Adda247 Publications For any detail, mail us at


Publications@adda247.com
Cracker Book for Bank (IBPS | SBI | RRB PO | Clerk) Mains Exams

(c) Economic reforms need to break the grip of stigmatized


capitalism.
(d) Hindi films show us public perception of how capitalism works in
country matters.
(e) none of these

20. The trouble with Internet of Things (IoT) approach is that it runs
contrary to current Indian telecom regulations. Telecom access
services providers are currently obliged to verify each customer
before issuing them a SIM. However, if cars must already have cellular
connectivity to their Network Operating Centers (NOC) when they roll
off the assembly line, it is impossible for telecom service providers to
complete a customer verification of the future owner of the car before
activating the SIM.
(a) Approach of Internet of Things will be achieved through artificial
intelligence (AI).
(b) The burden of telecom regulatory on the internet of things should
be eased.
(c) SIMs are used for Machine-to-machine communications without
customer verification.
(d) The approach of connected world is still ahead of us.
(e) none of these

21. India’s law governing motor vehicles and transport is archaic, lacking
the provisions necessary to manage fast motorisation. The lacunae in
the Motor Vehicles Act, 1988, require to be addressed to improve road
safety, ensure orderly use of vehicles and expand public transport.
(a) Road accidents have increased in the past few years due to the
hiatus in the Motor Vehicles Act, 1988.
(b) Expansion of vehicles have emerged as a real hindrance in the
path of the Motor Vehicles Act,1988.
(c) New innovative reforms are required in the laws governing motor
vehicles as the already existing reforms fail to address the
problem related to the safety of vehicles.
(d) The obsolete laws governing motor vehicles need improvisation
to ensure smooth management of expanding transport and in
alleviating the road safety.
(e) None of these
15 Adda247 Publications For any detail, mail us at
Publications@adda247.com
Cracker Book for Bank (IBPS | SBI | RRB PO | Clerk) Mains Exams

22. The menace of migratory politics highlighted in the politics of citizens’


registers began in the colonial era when the British attempted to
import labour for the plantations. Major displacements like Partition
and the Bangladesh war added to a huge “illegal” population.
(a) The politics of citizens’ registers undermines the problem of
migratory politics, which is reflected in many instances of history.
(b) The politics of citizens’ registers underlines the problem of
migratory politics, refracted through the layered memories of
many historical events.
(c) The politics of citizens’ registers heightened the problem of
migratory politics, which is seen in various events depicted in the
history.
(d) The politics of citizens’ registers sidelined the problem of
migratory politics, which is seen in various forms of historical
events.
(e) None of these

23. For years now, the multilateral system for the settlement of trade
dispute has been under intense scrutiny and constant criticism. The
U.S. has systematically blocked the appointment of new Appellate
Body members (“judges”) and de facto impeded the work of the WTO
appeal mechanism. With only four working members out of seven
normally serving office in July 2018, the institution is under great
stress.
(a) The multilateral system responsible for trade dispute has been
under constant criticism for quiet sometime now due to the
stagnancy in the appointment of new Appellate body members,
ultimately delayed the work of the WTO appeal mechanism.
(b) The direct system responsible for trade dispute has been
appreciated for years now as US blocked the appointment of new
Apellate body members and it eventually staggered the work of
WTO appeal mechanism.
(c) The lineal system for trade dispute has been has been under
constant criticism for years which led US to block the
appointment of Appellate body members, ultimately impacting
the work of WTO appeal mechanism.
16 Adda247 Publications For any detail, mail us at
Publications@adda247.com
Cracker Book for Bank (IBPS | SBI | RRB PO | Clerk) Mains Exams

(d) The institution that deals with the settlement of trade disputes is
facing great stress as the multi-lateral system has not been under
constant criticism.
(e) None of these

24. Indians are indignant that US President Donald Trump seeks to curtail
the number of H-1B visas, and the scope to work of H-1B visa-holders’
spouses. Indians, on the whole, look askance at the Trump
administration’s anti-immigrant policies people.
(a) Curtailing the number of H-1B visas and the scope to work of H-
1B visa-holders’ in US has caused a disapproval among the
Indians and they disapprove the Trump’s decision.
(b) India should make it easy for anyone born in India or has been
resident in India for a reasonable period of time to acquire Indian
citizenship.
(c) In the globalizing world, Indians support unrestricted movement
of natural people across borders.
(d) Elevating the number of H-1B visas and the scope to work of H-1B
visa-holders’ in US has caused a disapproval among the Indians
and they are angry with the Trump’s decision.
(e) None of these

25. The slew of changes that have periodically been brought about show
that the tax system has stabilised neither in terms of the rate
structure nor in categorisation of goods and services. There are too
many rates now and some rates are still way too high. Rates must be
lowered, converged to three to bring down classification disputes, and
boost collections. The final GST structure should become stable to
achieve a sustained increase in collections.
(a) Multiple reforms have not done anything in stabilizing the tax
structure both in term of rate or in categorization of goods and
services. There is a need for a stabilized GST structure that
promises a sustained increase in tax collections.
(b) Considering the vast number of changes in the GST structure, a
need is felt to stabilize the rate structure and categorise the goods
and services, further discouraging the tax collections.

17 Adda247 Publications For any detail, mail us at


Publications@adda247.com
Cracker Book for Bank (IBPS | SBI | RRB PO | Clerk) Mains Exams

(c) The changes brought in by the multiple reforms have stabilized


the tax structure both in rate structure and in categorization of
goods and services, further encouraging the tax collections.
(d) Rates must be increased to bring the stabilization in tax structure.
(e) None of these

26. The aviation market has seen a long spell of unprecedented growth
over the past few decades. However, 2016 was the first year in a
decade that Indian airlines collectively came into the black. Moreover,
The Centre for Asia Pacific Aviation predicts consolidated industry
losses of between $430-460 million in FY19. Jet Airways has never
truly looked healthy after its troubles during the dog days at the end
of the last decade.
(a) The unprecedented investment in aviation industry has resulted
in the losses incurred by the companies.
(b) Due to the unmatched growth of the aviation market, it has
experienced low profits over the decades.
(c) The aviation industry has gained huge profits after the expansion
of its resources.
(d) Despite the market’s breakneck growth over the past few
decades, the losses of Indian air carriers have often been high.
(e) None of these

27. President Donald Trump accuses the media of being “the opposition
party,” implying a failure of objectivity. But it’s not a bad thing for
opinion journalism, including the editorial boards of major
newspapers, to see them as the opposition to Trump. Opposition like
that keeps democracy alive, and constitutes one of the core
responsibilities of a free press.
(a) A free press is that it expresses a variety of opinions, especially
those that differ from the government.
(b) Democracy needs the press as opposition for informing the
public, by presenting another point of view than the president’s.
(c) Opinions are provisional, not objectively provable.
(d) The justification for a free press doesn’t depend on its being
objective.
(e) None of these

18 Adda247 Publications For any detail, mail us at


Publications@adda247.com
Cracker Book for Bank (IBPS | SBI | RRB PO | Clerk) Mains Exams

28. Kerala is home to 53 large dams with a collective capacity of nearly 7


trillion litres. For dams to truly tame floods, experts say dam
reservoirs need to be relatively empty before the onset of rains.
According to Himanshu Thakkar, coordinator of the South Asia
Network for Dams, Rivers and People, this was not the case in Kerala.
The Idukki dam was already near full capacity by July-end even as
rains were relatively weak (below normal levels) during that period.
(a) The damages from the floods could be lowered by relatively
emptying the dam reservoirs ahead of monsoon rains.
(b) Filling up the reservoirs before the end of the monsoon has
limited the disaster.
(c) Unchecked quarrying and construction in ecologically-sensitive
areas caused these floods.
(d) The management is responsible to improve the conditions of
dams in Kerala.
(e) None of these

29. To widen the capital expenditure pie, Indian army must look inward.
Given the changing nature of warfare, which is likely to be short and
decisive, it also makes strategic sense to require more special
operations forces, cyber war capabilities and integrated/joint
operational capabilities.
(a) India army must modify the policies to increase capital
expenditure.
(b) Defence preparedness cannot be compromised.
(c) There is little leeway for the government to drastically hike
defence spending.
(d) Indian Army should introspect their practices and should adapt to
the changing warfare thus resulting enhancement of expenditure
pie.
(e) None of these
19 Adda247 Publications For any detail, mail us at
Publications@adda247.com
Cracker Book for Bank (IBPS | SBI | RRB PO | Clerk) Mains Exams

30. The crisis in rural areas today is actually one of too much agriculture
— in terms of both relative output and employment. What is needed is
more manufacturing units, including those that process and add value
to agricultural produce.
(a) Agriculture sector would gain from the processes of specialisation
and division of labour
(b) Farming ceases to be a difficult occupation.
(c) Crisis of too much agriculture persists in terms of output and
employment which can be improved by value adding processes in
rural areas.
(d) An unleashing rural entrepreneurship is needed.
(e) None of these

Solutions

1. (b); Read the paragraphs carefully. Among the given paragraphs, only
the (III) paragraph concludes the given inference. It is describing
about the production of more electric vehicles for private as well as
for public transport, to reduce the increasing pollution. However,
paragraph (I) has given emphasis on the adoption of alternate
mobility technologies. It has also suggested that shared mobility
would further help to reduce pollution. While paragraph (II) is
suggesting ways that would help in reducing pollution such as
presenting incentives, banning polluting technologies and
improving infrastructure. It is to be noted that in paragraph (I) and
(II) the alternate technology to reduce pollution has not been
mentioned whereas, in paragraph (III) the alternate technology of
electric vehicles is mentioned. Thus, option (b) becomes the most
viable answer choice.

2. (c); After reading the paragraphs carefully, it can be understood that


the paragraphs (I) and (III) depict the given inference. Paragraph
(I) has mentioned the release of model tender document of NHPS
which states its concerns regarding the implementation of the
scheme in such a vast scale. Moreover, it also expresses its
agitation towards the private hospitals which may receive undue

20 Adda247 Publications For any detail, mail us at


Publications@adda247.com
Cracker Book for Bank (IBPS | SBI | RRB PO | Clerk) Mains Exams

benefits from the schemes. Paragraph (III) is describing about the


issue mentioned in the model tender document of NHPS of the
requisite of pre-authorization to perform on heart ailments and
cancer from the NHPS’s implementation support agency. Since, all
three paragraphs mention the questions raised in The Model
Tender Document for The Selection of Implementing Agencies For
the NHPS, option (c) becomes the most suitable answer choice.

3. (a); Among the given paragraphs, paragraph (I) and (II) are deducing
the given inference in bold. The inference is stating that the latest
alterations made in NCERT books didn’t follow the revision process
like it used to earlier. Though being an autonomous body, this
incident shows the weakness of NCERT. Drawing a hint from the
sentence of paragraph (I) “The names that figure on the books as
“textbook development committees” remain the same, though most
of them have not been involved in the insertion/revision process”,
the given inference can be concluded. Moreover, paragraph (II) is
also questioning these alterations in a critical manner. This can be
understood from the sentence “This, in turn, raises questions about
why the current changes have been made as they have”. While, the
sentence “…in an autonomous body that has shown its ability to
take an imaginative course while generating a discursive
relationship with all those involved in education, without
rendering itself an arm of the state” expresses that NCERT has
failed to correctly implement the revision process in spite of being
an autonomous body. However, paragraph (III) fails to depict the
given inference as it is merely describing the alterations made by
NCERT in the textbooks. Thus, option (a) becomes the most viable
answer choice.

4. (c); The given inference in bold states that with the help of lateral entry
at senior levels of bureaucracy better policy making and
implementation can be done. After reading the paragraphs, it is
inferential that only paragraph (II) provides the given inference
since, it is describing about the entry of new and talented
personnel at joint-secretary levels which is vital for policy-making
and implementation of government schemes. However, paragraph
(I) is describing about selecting employees in IAS scheme who
21 Adda247 Publications For any detail, mail us at
Publications@adda247.com
Cracker Book for Bank (IBPS | SBI | RRB PO | Clerk) Mains Exams

becomes versatile while serving different departments which is


irrelevant in the context of the inference. In addition to, paragraph
(III) is describing about the proposal of selecting process for lateral
entry to be clear and appropriate is still unheard. Thus, the given
inference can be precisely deduced by only paragraph (II), hence,
option (c) becomes the most suitable answer choice.

5. (e); The given inference is stating that SBI will not manage the oil
payments of Iran. After reading the paragraphs it can be
understood that this inference is not expounded in any of them.
Paragraph (I) has illustrated the exports data of oil of Iran. While,
paragraph (II) has mentioned India’s plan to spend foreign capital
on energy resources as it might become the third largest energy
consumer. Moreover, paragraph (III) has merely provided the
imports data of oil of India. Therefore, option (e) becomes the most
suitable answer choice.

6. (d); Read the paragraphs carefully. The given inference can be


concluded from paragraph (I) as well as paragraph (II). Both the
paragraphs are clearly depicting the difference in the view of South
Korea’s president and US president regarding the denuclearization of
North Korea. It can be noted from the sentence in the paragraph
(I) “Instead he is…denuclearization”. This sentence expresses that
SK’s expects more benefits from US. Moreover, paragraph (II) is
describing how the matters have gone worse about the
denuclearization of North Korea. We can receive the hint from the
sentence [the matter of denuclearization… Trump-Kim summit].
However, paragraph (III) is describing about North Korea’s foreign
policy to protect itself from the US attacks. Therefore, option (d)
becomes the most viable answer choice.

7. (e); Read the paragraphs carefully. Among the given paragraphs all
three of them provide the given inference. It is to be noted that the
paragraphs are describing about the requirement of technological
innovations in banking industry according to the growing demands
by customers for digitization. Since, all three paragraphs describe
the need to improve and innovate the technology in banking sector,
option (e) becomes the most suitable answer choice.
22 Adda247 Publications For any detail, mail us at
Publications@adda247.com
Cracker Book for Bank (IBPS | SBI | RRB PO | Clerk) Mains Exams

8. (a); Among the given paragraphs, both paragraph (II) and paragraph
(III) can be used to infer the given inference. Paragraph (II) is
stating about the consequences the world is facing due to climate
change, especially in reference to the agriculture. While, paragraph
(III) is also describing how changes in climate is affecting
agriculture which will ultimately affect the entire human race as well
as animals. However, paragraph (I) has provided information on
agriculture pollution and causes leading to it. Hence, option (a)
becomes the most viable answer choice.

9. (e); Read the given paragraph carefully. It should be noted that all three
paragraphs are following a mutual theme by describing about the
applications of nanotechnology in several fields. Paragraph (I) is
describing about the usage of nanotechnology and nanoparticles in
the production of lightweight fabrics for body armors. In addition to,
paragraph (II) is describing about the utilization of
nanotechnology in the field of medical science by simplifying the
complex therapy such as biopsy. Moreover, paragraph (III) is
mentioning about the implementation of nanotechnology in
production of pesticides and fertilizers. Since, the given inference
can be implied from all the given paragraphs, option (e) becomes
the most viable answer choice.

10.(b); After reading the paragraphs carefully, it can be understood that


the given inference is deducted from paragraph (II). It is to be
noted that only paragraph (II) is describing about the negative
implication of the policy on the learning of the students that had
been implemented in the past. However, paragraph (I) is
describing about the financial management of the school my
councils, thus stands incorrect. Moreover, there might some
confusion with paragraph (III) since, it is also describing about the
policy implemented for students. However, it does not state any
negative effect of the policy which is mentioned in the given
inference. This can be understood from the word “deficit” in the
inference. ‘Deficit’ means a deficit is the amount by which a sum

23 Adda247 Publications For any detail, mail us at


Publications@adda247.com
Cracker Book for Bank (IBPS | SBI | RRB PO | Clerk) Mains Exams

falls short of some reference amount. Here, it is used in reference


with ‘learning’. Thus, the inference is stating about the policy that
has become like a veil to hide the drawbacks of learning system.
Therefore, option (b) becomes the most suitable answer choice.

11.(d); The given inference in bold can be concluded from all the given
paragraphs. After reading the paragraphs it can be understood that
all of them are illustrating the change required to neutralize the
growing dissatisfaction among farmers. Paragraph (I) is narrating
that farmers are displeased with the policies present to support
them. The given inference can be understood with the statement
mentioned satirically in the paragraph “radical, multi-
faceted……incomes by 2022”. Moreover, paragraph (II) has
described that the present practices by government will not enrich
farmers. It has suggested to develop a reform to provide marketing
freedom for farmers that will help them to capture a larger share of
market. For the given inference, the hint can be drawn from the
sentence “But improved productivity will not enrich farmers by
itself”. In addition to, paragraph (III) describes about farmers
unrest. It has recommended a method to overcome this unrest [A
part of subsidy...productivity-enhancing investment]. This can be
achieved only through a new and updated political reform. Since,
all three paragraphs successfully conclude the given inference,
option (d) becomes the most suitable answer choice.

12.(b); After carefully analyzing the paragraphs, it can be understood that


only paragraphs (II) and (III) conclude the given inference. It is to
be noted that paragraph (I) is merely describing the difficulties
facing by the PSUs. It does not depict any steps undertaken by the
government to overcome these issues. Therefore, it fails to infer the
statement given in bold. However, Paragraph (II) is describing
about the government’s process of recapitalization so as to
enhance the lending capacity of banks. The given inference can be
deduced from the sentence “Therefore, it is important that the
situation is handled with care”. Moreover, in paragraph (III) the
given inference can be concluded by drawing a hint from the
sentence “The government, perhaps, needs to put in place a new
24 Adda247 Publications For any detail, mail us at
Publications@adda247.com
Cracker Book for Bank (IBPS | SBI | RRB PO | Clerk) Mains Exams

framework for governance”. Since, among the given paragraphs


only paragraph (II) and (III) provides the given conclusion, thus
option (b) becomes the most viable answer choice.

13.(a); The inference given in bold can be deduced with only the first
paragraph, since it is describing the need to revolutionize the
education system of India. In addition to, it has also mentioned the
areas to work in order to build a worthy generation of youths.
However, paragraph (II) is describing about growth and
advancement made by the educational sector of the country. It has
appreciated the improve education system especially in higher
education and scientific research. Thus, it fails to conclude the
given inference. Furthermore, paragraph (III) has mentioned the
components of an education system and their roles building a
sound education system. Therefore, the given inference cannot be
concluded with the information given in paragraph (III). Since, only
paragraph (I) provides the precise inference, option (a) becomes
the most suitable answer choice.

14.(d); The given inference states that the skill development programmes
should consider the skillset required by the market. Among the
given paragraphs, this can be deduced by all the three paragraphs.
Drawing a hint from the sentence in paragraph (I) “Organizations
must focus on understanding aspirations, industry requirements
and standardization across the skill-development value chain” the
given inference can be deduced. In paragraph (II) it mentions the
importance of mapping skills as per the needs of employers. The
sentence “NSDC plans to move to a model where training partners
will receive funds as per the outcomes achieved” helps to conclude
the given inference. Moreover, paragraph (III) is concluding the
given inference from the sentence “Concentrated and evidence-
backed efforts which can cohesively develop and strengthen youth
aspirations, the skill development ecosystem and markets where
youth can be employed are necessary for India to realize that
opportunity.” Hence, option (d) becomes the most viable answer
choice.

25 Adda247 Publications For any detail, mail us at


Publications@adda247.com
Cracker Book for Bank (IBPS | SBI | RRB PO | Clerk) Mains Exams

15.(c); The paragraph is all about the moving towards digitizing the public
examination. It has further explained the benefits associated with
such a move. Thus, among the three given inferences, statements
(II) and (III) can be eliminated on the basis of lack of proper and
central idea of the paragraph as they just form the parts of the facts
given in the paragraph while statement (I) comprehensively
expresses the true meaning of the passage and draws the most
appropriate inference for the same. Hence option (c) is the correct
choice.

16.(e); The paragraph gives a comparative study between the best-ranked


Indian higher education institutions with the best-ranked
universities of the world. It has further mentioned the areas where
the world universities excel over Indian institutions. Thus, among
the given three inferences, all of them are out of context of the
paragraph as the first inference and the second inference are
describing about the rankings of Indian institutions in global level.
While the third inference, has mentioned about some ways that
would enable the Indian institutions to achieve a global status.
Since none of them can be inferred from the given paragraph
option (e) becomes the correct choice.

17.(a); The paragraph is describing how the appropriate data collection


may reduce number of lives impacted due to natural disasters.
Option (a) expresses the inference of the paragraph. All the other
options are irrelevant in context of the paragraph. Hence, option
(a) is the most viable answer choice.

18.(d); Simultaneous elections mean structuring the Indian election cycle


in a manner that elections to Lok Sabha and State Assemblies are
synchronised together. The given paragraph is describing about its
impact on the federal structure of the nation. All the other options
are irrelevant in context to the paragraph. Hence, option (d)
successfully infer the meaning of the given paragraph.

19.(d); The given paragraph describes that the economic reforms have
noticed a shift in the public perceptions which are influenced by
Hindi films. Therefore, option (d) infers the given paragraph in the
26 Adda247 Publications For any detail, mail us at
Publications@adda247.com
Cracker Book for Bank (IBPS | SBI | RRB PO | Clerk) Mains Exams

most appropriate manner. All the other options are irrelevant in


the context of the paragraph. Hence, option (d) is the most suitable
answer choice.

20.(b); The paragraph is describing about the complications in the


approach of Internet of Things due to the difficulty in post
customer verification. Option (b) aptly infer the meaning of the
paragraph. All the other options are irrelevant in context to the
paragraph. Hence, option (b) is the most suitable answer choice.

21.(d); The correct inference that can be drawn from the paragraph is ‘The
obsolete……………………….road safety”. Other choices are either
irrelevant in context to the passage or contain contextual errors in
them. The paragraph is describing about the old law and provisions
necessary to manage the fast growing vehicles. Therefore, the only
option that draws the correct inference is option (d).
22.(b); The correct inference that can be drawn from the paragraph is,
‘The politics of citizens’ registers underlines the problem of
migratory politics, refracted through the layered memories of
many historical events.” The paragraph is describing about the
politics of citizens’ registers that has its roots in the colonial era
and other historical events. Therefore, the most suitable choice that
fits in context to the paragraph is option (b). Other choices are
irrelevant in context to the paragraph.

23.(a); The correct inference that can be drawn from the paragraph is, ‘(a)
The multilateral system responsible for trade dispute has been
under constant criticism for quiet sometime now due to the
stagnancy in the appointment of new Appellate body members,
ultimately delayed the work of the WTO appeal mechanism”. The
paragraph is describing about the multidimensional system
responsible for resolving trade disputes which is under constant
criticism due to the delay in appointment of new appellate body
members that delayed the work of the WTO appeal mechanism.
Therefore, the most suitable choice that fits in context to the
paragraph is option (a). Other choices are irrelevant in context to
the paragraph.
27 Adda247 Publications For any detail, mail us at
Publications@adda247.com
Cracker Book for Bank (IBPS | SBI | RRB PO | Clerk) Mains Exams

24.(a); The correct inference that can be drawn from the paragraph is, (a)
Curtailing the number of H-1B visas and the scope to work of H-1B
visa-holders’ in US has caused a disapproval among the Indians and
they disapprove the Trump’s decision. The paragraph is describing
about the disapproval caused among many Indians due to the anti-
immigrant policies by Trump. Therefore, the most suitable choice
that fits in context to the paragraph is option (a). Other choices are
irrelevant in context to the paragraph.

25.(a); The correct inference that can be drawn from the paragraph is, (a)
Multiple reforms have not done anything in stabilizing the tax
structure both in term of rate or in categorization of goods and
services. There is a need for a stabilized GST structure that
promises a sustained increase in tax collections. The paragraph is
describing about the reforms responsible for resolving trade
disputes which is under constant criticism due to the delay in
appointment of new appellate body members that delayed the
work of the WTO appeal mechanism. Therefore, the most suitable
choice that fits in context to the paragraph is option (a). Other
choices are irrelevant in context to the paragraph.

26.(d); The correct inference that can be drawn from the paragraph is
“Despite the market’s breakneck growth over the past few decades,
the losses of Indian air carriers have often been high”. Other choices
are either irrelevant in context to the paragraph or contain
contextual errors in them. The paragraph is describing about the
fall in the profits of the aviation industry despite its unprecedented
growth. Therefore, the only option that draws the correct inference
is option (d).

27.(b); The correct inference that can be drawn from the paragraph is
“Democracy needs the press as opposition for informing the public,
by presenting another point of view than the president’s”. Other

28 Adda247 Publications For any detail, mail us at


Publications@adda247.com
Cracker Book for Bank (IBPS | SBI | RRB PO | Clerk) Mains Exams

choices are either irrelevant in context to the paragraph or contain


contextual errors in them. The paragraph is describing about the
importance of free journalism in a democracy. Therefore, the only
option that draws the correct inference is option (b).

28.(a); The correct inference that can be drawn from the paragraph is “The
damages from the floods could be lowered by relatively emptying
the dam reservoirs ahead of monsoon rains”. Other choices are
either irrelevant in context to the paragraph or contain contextual
errors in them. The paragraph is describing about the precautions
that could be taken to minimize the damages caused due to the
floods in Kerala. Therefore, the only option that draws the correct
inference is option (a).

29.(d); The correct inference that can be drawn from the paragraph is
“Indian Army should introspect their practices and should adapt to
the changing warfare thus resulting enhancement of expenditure
pie.” Other choices are either irrelevant in context to the paragraph
or contain contextual errors in them. The paragraph is describing
about bringing a change in the inward practices of Indian army to
fulfill their capital expenditure. Therefore, the only option that
draws the correct inference is option (d).

30.(c); The correct inference that can be drawn from the paragraph is
“Crisis of too much agriculture persists in terms of output and
employment which can be improved by value adding processes in
rural areas”. Other choices are either irrelevant in context to the
paragraph or contain contextual errors in them. The paragraph is
describing about the crisis of rural areas where agriculture is
practiced heavily in terms of output and employment. This
situation can be improved by using more manufacturing units and
adding value to the processes. Therefore, the only option that
draws the correct inference is option (c).

29 Adda247 Publications For any detail, mail us at


Publications@adda247.com
Cracker Book for Bank (IBPS | SBI | RRB PO | Clerk) Mains Exams

1 Adda247 Publications For any detail, mail us at


Publications@adda247.com
Cracker Book for Bank (IBPS | SBI | RRB PO | Clerk) Mains Exams

Chapter

2 Conjunctions
STUDY TIPS

In such questions a conjunction or a connector is provided followed by


few sentences. An appropriate combination of sentences using the given
conjunction will mold into a single coherent sentence. Understanding the
meaning and the usage of the given conjunction will help to connect the
sentences logically. Moreover, sentences of similar subject are more likely
to connect with each other. Therefore, understanding the possible usages
of the conjunctions/ connectors such as BECAUSE, NOTWITHSANDING,
DESPITE, ALTHOUGH etc. will help to answer such questions.

Practice Exercise Based on new Pattern

Directions (1-30): In the questions given below few sentences are given
which are grammatically correct and meaningful. Connect them by the
word given above the statements in the best possible way without changing
the intended meaning. Choose your answer accordingly from the options to
form a correct, coherent sentence.

1. ALTHOUGH
(I) The report cites research that shows that gender gaps cause an
average income loss of 15 percent in the OECD economies, 40
percent of which is due to entrepreneurship gaps.
(II) The vast majority of the economies monitored have laws
establishing non-discrimination in employment based on
gender.
(III) Only 76 mandate equal remuneration for work of equal value
and 37 economies have no laws protecting pregnant workers
from dismissal.

2 Adda247 Publications For any detail, mail us at


Publications@adda247.com
Cracker Book for Bank (IBPS | SBI | RRB PO | Clerk) Mains Exams

(a) only (II)-(III)


(b) only (III)-(I)
(c) Both(I)-(III) and (I)-(II)
(d) only (II)-(I)
(e) none of these

2. ASSUMING THAT
(I) Although it is risky but many banks use One Time Password
(OTP) method for authentication of online payments.
(II) It is expected that new vulnerabilities in mobile payments shall
be discovered.
(III) Only a phone’s owner will see a message sent to it.
(a) only (II)-(III) (b) only (I)-(II) (c) only (III)-(II)
(d) only (I)-(III) (e) none of these

3. THAN
(I) Their work will likely contribute to making the next generation
of ultralow-power communications and sensory devices smaller
and with greater detection and tuning ranges.
(II) Researchers are developing atomically thin 'drumheads'-- tens
of trillions of times thinner.
(III) The human eardrum is able to receive and transmit signals across
a radio frequency range far greater than what we can hear with
the human ear.
(a) only (II)-(I) (b) only (II)-(III) (c) only (III)-(II)
(d) only (I)-(III) (e) none of these

4. EVEN THOUGH
(I) You can email photos to yourself from various computers and
download these photos to your iPad.
(II) You can't easily put photos on an iPad from more than one
computer.
(III) It isn't easy to keep the order of photos when transferring them
from PC to iPad Photo.
(a) only (II)-(I) (b) only (II)-(III) (c) only (III)-(II)
(d) only (I)-(III) (e) none of these
3 Adda247 Publications For any detail, mail us at
Publications@adda247.com
Cracker Book for Bank (IBPS | SBI | RRB PO | Clerk) Mains Exams

5. SINCE
(I) 'Avatar', a James Cameroon movie took the world by storm in
2009.
(II) With over $1.2 billion collection worldwide, 'Black Panther' is
the biggest Non-Sequel blockbuster.
(III) 'Black Panther' wins the box-office for the 5th straight weekend.
(a) only (I)-(III) (b) only (II)-(I) (c) only (III)-(I)
(d) both (I)-(II) and (II)-(III) (e) all of the above

6. TILL
(A) The Supreme Court had earlier this month extended the
deadline for linking of various services with Aadhaar.
(B) Aadhaar number holders can view the details of Aadhaar
Authentication they have done.
(C) The new deadline will be announced once the Supreme Court
delivers its judgment on a batch of petitions challenging the
constitutional validity of the biometrical identification scheme.
(D) This will enable service providers and Aadhaar number holders
to verify if the Aadhaar number is valid and is not deactivated.
(a) Only A-D (b) Only A-C (c) Only A-B
(d) Both A-B and C-D (e) None of these

7. PROVIDED THAT
(A) It is your responsibility to ensure that the builder understands
and complies with all the bank’s conditions of the loan.
(B) You can apply for progress payments at any stage during the
construction process.
(C) Progress payments are made only if sufficient work has been
completed and the bank is satisfied with the progress.
(D) A maximum of six progress payments are often allowed, based
on the work completed.
(a) Only A-D (b) Only A-C (c) Only B-C
(d) Both A-B and C-D (e) None of these

4 Adda247 Publications For any detail, mail us at


Publications@adda247.com
Cracker Book for Bank (IBPS | SBI | RRB PO | Clerk) Mains Exams

8. IN ORDER TO
(A) Fluent conversations can be enjoyed with the knowledge of
idioms and phrases.
(B) English sound more eloquent, through idiomatic expressions,
popular phrases and proverbs.
(C) Eloquent designs the technology to let computers participate in
those conversations.
(D) Gaining familiarity with several idioms and phrases helps you to
carry eloquent conversations.
(a) Only A-D (b) Only A-C (c) Only B-C
(d) Both A-B and C-D (e) None of these

9. NOTWITHSTANDING
(A) The proud claim is Manipur gifted the game of modern Polo to
the world,
(B) Manipuri ponies which are central to the game are facing a very
precarious situation of possible extinction sooner rather than
later.
(C) Most of the Manipuri ponies are kept at home but they are let to
roam and graze freely on roads and public places.
(D) There are also many cases of ponies dying from consuming
poisonous waste materials disposed of on road sides.
(a) Only A-D (b) Both A-C and B-C (c) Only B-C
(d) Only A-B (e) None of these

10. WHENEVER
(A) In the name of education, all sorts of slaveries are nourished.
(B) As some Indian officials have indicated, the days of carving out
separate ‘spheres of influence’ are over.
(C) As such, it remains a big question how the policy would be
translated into action and what would be the fate of Manipur.
(D) Every generation passes on, by way of inheritance, its jealousies,
enmities, stupidities to the new generation through the teacher.
(a) Only A-D (b) Only A-C (c) Only B-C
(d) Only A-B (e) None of these

5 Adda247 Publications For any detail, mail us at


Publications@adda247.com
Cracker Book for Bank (IBPS | SBI | RRB PO | Clerk) Mains Exams

11. WHILE
(A) LinkedIn found that 71% of employees now have a “side hustle,”
and among them, 40% have used allotted time off to work on
these extra endeavors.
(B) It should be stressed that real time off from work and vacations
promote productivity and better health in the long-term.
(C) LinkedIn found that at least one-third (36%) of these workers
said they found success by pursuing a passion project.
(D) It is up to employees ultimately as to how they choose to use
their time.
(a) A-B (b) C-D (c) B-D
(d) both B-D and C-B (e) none of these

12. ONLY IF
(A) Families who were caught together at the US-Mexico border and
separated are now being offered the opportunity to reunite.
(B) Agencies involved in family separation were so badly
coordinated that it was nearly impossible for parents and
immigration attorneys to locate children.
(C) Families must agree to deportation, giving up their attempts to
seek asylum and stay in the US.
(D) Either a parent can keep fighting for asylum and accept that he
may not be able to see his children for the months or years his
case might take — or he can give up, waive both his own rights
and the rights of his child.
(a) A-D (b) A-C (c) B-C
(d) both A-C and B-C (e) none of these

13. UNLESS
(A) Adults are supposed to know how to behave when presented
with contrary ideas; to debate and overcome ideas they dislike
with words, facts, and reason rather than violence, censorship,
or government suppression.
(B) But where free speech and independence of ideas are concerned,
the Right is right and the Left is wrong and those on the Left
would be strongly against their own tactics if the roles were
reversed.
6 Adda247 Publications For any detail, mail us at
Publications@adda247.com
Cracker Book for Bank (IBPS | SBI | RRB PO | Clerk) Mains Exams

(C) More violence will result and that is a sad prospect for our
future.
(D) The violent, intolerant wing of the political left abandons the
strong-arm tactics to get their ideas accepted.
(a) only C-B (b) only A-B (c) only B-D
(d) only D-C (e) both B-C and C-D

14. WHEREIN
(A) We can find anything and everything in politics by which we
wish to strengthen our narrative, no matter how absurd it is.
(B) Politics is arguably the best case study in an act of knowledge
and disinformation.
(C) The greatest enemy of knowledge is not ignorance, it is the
illusion of knowledge.
(D) Political education becomes an obligation and you may not have
an opinion or make a decision instead you are allowing someone
else to make it on your behalf.
(a) only B-D (b) only B-A (c) only D-A
(d) both (b) and (c) (e) none of these

15. WHILE
(A) It was true that the state governments were insisting on keeping
the rates high for fear of tax collections falling short.
(B) The GST Council, though, has to be careful about applying zero
rates on more commodities.
(C) It was never clear why the Centre didn’t push for lower rates as
it was guaranteeing that states would get 14% growth in their
tax revenues in the next five years.
(D) The GST Council has also done well to simplify filing for around
95% of assessees since, instead of having to file 37 returns a
year, those with a turnover of less than Rs 5 crore a year will file
one return a quarter plus one master return in a year.
(a) B-D (b) A-D (c) A-C
(d) both A-D and B-D (e) none of these
7 Adda247 Publications For any detail, mail us at
Publications@adda247.com
Cracker Book for Bank (IBPS | SBI | RRB PO | Clerk) Mains Exams

16. ALTHOUGH
(A) They had to diagnose network issues, where the labels were
written in pencil on paper, plastic, metal and after 10 years they
start to fade.
(B) They examined the rest of the garments and each, upon careful
observation, was identified in a like fashion.
(C) Graphite is a compressed powder, it can be worn off, brushed
off, and if it's on something bound like a notebook, it will most
definitely smudge into illegible markings.
(D) Some of the markings were so faded they were no longer legible.
(a) A-D (b) C-D (c) B-C
(d) B-D (e) both A-B and B-C

17. HOWEVER
(A) The Taiwanese government is claiming that the People’s
Republic of China would invade the island state if the U.S.
withdrew its support for Taipei.
(B) It is dubious that Beijing has the amphibious assault capacity or
capability to conduct a successful sea-based invasion of Taiwan
where a forcible entry is required.
(C) One of the factors that has traditionally prevented Beijing from
forcibly retaking the island state by force is geography.
(D) Chinese capabilities have markedly improved over the course of
the past decade and the People’s Liberation Army continues to
make strides in capability.
(a) C-D (b) A-C (c) A-B
(d) A-D (e) None of these

18. AS
(A) The central bank issued a circular three months ago, banning all
financial institutions under its control from providing services
to companies dealing in crypto currencies.
(B) The crypto banking ban by the Reserve Bank of India (RBI) went
into effect at the end of last week.
(C) A growing number of exchanges have stopped fiat support.

8 Adda247 Publications For any detail, mail us at


Publications@adda247.com
Cracker Book for Bank (IBPS | SBI | RRB PO | Clerk) Mains Exams

(D) Indian crypto currency exchanges ramp up peer-to-peer and


crypto-to-crypto trading.
(a) A-B (b) C-D (c) A-C
(d) both A-D and B-C (e) both D-B and D-A

19. ALTHOUGH
(A) The fastest-growing Top 10 US import commodity from the
European Union through May was unsweetened waters.
(B) The Trump administration unveiled tariffs on steel and
aluminum imports in March.
(C) US trans-Atlantic containerized trade growth with the European
Union slowed in the first five months of the year.
(D) Tariffs did not apply to the European Union until the start of
June.
(a) Only (B) – (D)
(b) Only (A) – (C)
(c) Both (A) – (B) and (B) – (C)
(d) Both (C) – (D) and (A) – (C)
(e) None of these

20. NOT ONLY…BUT ALSO


(A) Cars would be freed from their reliance on planet-killing fossil
fuels.
(B) Researchers are working on making solar panels lighter, nicer
looking, and more efficient.
(C) The next generation of solar materials will be applied like paint,
turning a whole vehicle (or just about anything else) into a solar
panel.
(D) Cars will be spared from the need for a charging infrastructure
whose slow growth remains a major pain point.
(a) Only (B) – (C)
(b) Only (A) – (B)
(c) Only (A) – (D)
(d) Both (A) – (D) and (B) – (C)
(e) None of these
9 Adda247 Publications For any detail, mail us at
Publications@adda247.com
Cracker Book for Bank (IBPS | SBI | RRB PO | Clerk) Mains Exams

21. BY VIRTUE OF
(A) Many brilliant, forceful leaders truly believe that there is in the
world one person who is truly indispensable.
(B) His long experience at the United Nations makes him
indispensable to the talks.
(C) In order to cut costs, my boss has asked me to eliminate all
positions that are not indispensable to day-to-day operations.
(D) Leaders become indispensable by making it to the top of
whatever empire they run, they are made for life.
(a) Only (B) – (C)
(b) Only (A) – (B)
(c) Only (A) – (D)
(d) Both (A) – (D) and (B) – (C)
(e) None of these

22. UNLESS
(A) With the nation in the grip of a heatwave, MPs have warned that
heat-related deaths in the UK will treble by the middle of the
century.
(B) the government should intervene urgently to control the prices
of new cars, claimed by a leading motor figure.
(C) the government needs to tackle the new public health
emergency of heat-related deaths.
(D) New car prices could soar by €3,000 under a new emissions
system.
(a) Only (B) – (C)
(b) Only (D) – (B)
(c) Only (A) – (C)
(d) Both (D) – (B) and (A) – (C)
(e) None of these

23. BECAUSE
(A) The fuel economy of an automobile is the relationship between
the distance traveled and the amount of fuel consumed by the
vehicle.
(B) The New European Driving Cycle is a driving cycle, last updated
in 1997, designed to assess the emission levels of car engines.
10 Adda247 Publications For any detail, mail us at
Publications@adda247.com
Cracker Book for Bank (IBPS | SBI | RRB PO | Clerk) Mains Exams

(C) Tests under the more realistic Worldwide harmonized Light


Vehicles Test Procedure system are producing mostly higher
figures for emissions.
(D) Fuel consumption is shown to be higher than it was under the
outdated New European Driving Cycle regime.
(a) Only (B) – (C)
(b) Only (C) – (D)
(c) Only (A) – (C)
(d) Both (D) – (B) and (A) – (C)
(e) None of these

24. WHILE
(A) The bench had earlier slammed the authorities, saying it will be
an embarrassment if the UNESCO withdraws its world heritage
tag accorded to the ivory-white marble mausoleum.
(B) Director General of Archeological Survey of India (ASI) will be
responsible for maintenance of the Taj Mahal.
(C) joint secretary of Ministry of Environment and commissioner of
Agra Division will jointly be in-charge of the Taj Trapezium Zone
(TTZ).
(D) On the last date of hearing the bench of justices Madan B Lokur
and Deepak Gupta had wanted to know who is in charge of the
Mughal era monument.
(a) Only (C) – (D)
(b) Only (A) – (D)
(c) Only (B) – (C)
(d) Both (A) – (B) and (C) – (D)
(e) none of these

25. WHEREAS
(A) The authorities suspended the rail services in the Valley.
(B) The Army fired live ammunitions to quell the mob, injuring, at
least, five people.
(C) The Army conducted searches following the reports about the
presence of separatist militants.
(D) The shutdown called by the separatists closed marketplaces and
public transport services.
11 Adda247 Publications For any detail, mail us at
Publications@adda247.com
Cracker Book for Bank (IBPS | SBI | RRB PO | Clerk) Mains Exams

(a) Only (C) – (D)


(b) Only (A) – (D)
(c) Only (B) – (C)
(d) Both (A) – (B) and (C) – (D)
(e) none of these

26. SINCE
(A) Mining development in India has been hobbled for a variety of
reasons, from unfriendly government policy
(B) Arvind Panagariya argues that, any move that seeks to make
imports costlier as an import-substitution regime should be
avoided.
(C) India would do better to have a viable export-led strategy and
concentrate on removing bottlenecks.
(D) A liberal trade regime results in cheaper imports and more
competitive exports.
(a) Only (D) – (B)
(b) Only (A) – (C)
(c) Only (D) – (A)
(d) Both (A) – (B) and (C) – (D)
(e) none of these

27. ONCE
(A) E-commerce in India has changed the way millions of Indians
shop and simultaneously influenced operations of
manufacturers and service providers.
(B) The salient feature of this policy is the strategic intent which
underpins it.
(C) These advances have simultaneously transformed many areas
such as financial payments.
(D) The e-commerce phenomenon piggybacks on path breaking
advances in information and communications technology.
(a) Only (D) – (B)
(b) Only (A) – (C)
(c) Only (D) – (A)
(d) Both (A) – (B) and (C) – (D)
(e) none of these
12 Adda247 Publications For any detail, mail us at
Publications@adda247.com
Cracker Book for Bank (IBPS | SBI | RRB PO | Clerk) Mains Exams

28. SO THAT
(A) a content creator can have the confidence that an end viewer
can see exactly the images the Netflix content creators slaved so
hard to create.
(B) The new X1 Ultimate processor, is reckoned to be around twice
as powerful as the previous X1 Extreme system.
(C) Netflix Calibrated Mode automatically adjusts picture settings
when you’re watching the built-in Netflix app.
(D) The image in Netflix Calibrated Mode resembles as closely as
possible the way Netflix’s content creators saw things when
mastering their shows and movies.
(a) Only (C) – (A)
(b) Only (B) – (C)
(c) Only (B) – (A)
(d) Both (C) – (D) and (C) – (A)
(e) None of these

29. ONLY IF
(A) The tone of his voice, his manner of speaking, his walk, gestures,
and the lines of his face and the expression of his eyes, were all
carefully observed.
(B) All these features need to be satisfied to admit him as a
probationer.
(C) He became a probationer for the Scottish ministry in 1661 just
before episcopal government was re-established in Scotland.
(D) These features weren't on the site when it was first launched
because the necessary data did not yet exist.
(a) Only B-D
(b) Only C-A
(c) Only A-B
(d) Both B-C and A-D
(e) None of these

13 Adda247 Publications For any detail, mail us at


Publications@adda247.com
Cracker Book for Bank (IBPS | SBI | RRB PO | Clerk) Mains Exams

30. WHILE
(A) Regulator SEBI is planning to broaden the scope of
cybersecurity initiatives for the market infrastructure
institutions.
(B) Asian share markets crept cautiously higher on Monday as
investors awaited developments on proposed Sino-US trade
talks
(C) The broader markets also participated in the rally today with
the Nifty Midcap index rising over a percent.
(D) All sectorial indices ended in the green with Nifty Bank, FMCG,
Metal and Pharma rising 1-2 percent.
(a) Only B-D
(b) Only C-D
(c) Only A-B
(d) Both B-C and A-D
(e) All of these

Solutions

1. (a); Although the vast majority of the economies monitored have laws
establishing non-discrimination in employment based on gender,
only 76 mandate equal remuneration for work of equal value and
37 economies have no laws protecting pregnant workers from
dismissal.

2. (d); Although it is risky but many banks use One Time Password (OTP)
method for authentication of online payments assuming that only a
phone’s owner will see a message sent to it.

3. (b); Researchers are developing atomically thin 'drumheads'-- tens of


trillions of times thinner than the human eardrum which is able to
receive and transmit signals across a radio frequency range far
greater than what we can hear with the human ear.

4. (a); Even though you can't easily put photos on an iPad from more than
one computer you can email them to yourself from various
computers and download these photos to your iPad.
14 Adda247 Publications For any detail, mail us at
Publications@adda247.com
Cracker Book for Bank (IBPS | SBI | RRB PO | Clerk) Mains Exams

5. (b); With over $1.2 billion collection worldwide 'Black Panther' is the
biggest Non-Sequel blockbuster since 'Avatar', a James Cameroon
movie which took the world by storm in 2009.

6. (b); Statements (A) and (C) can be joined together using the phrase
“till”. The two words until and till mean exactly the same. They are
both a preposition and a conjunction. Until is often shorted to “till”
and the choice of one or the other is on formality vs. informality in
writing as well as on harmonics in the speech. Both the sentences
are in the similar context and therefore, the statement thus formed
is “The Supreme Court had earlier this month extended the
deadline for linking of various services with Aadhaar till it
delivers its judgment on a batch of petitions challenging the
constitutional validity of the biometrical identification scheme.”
Hence, option (b) is the most viable answer choice.

7. (c); Statements (B) and (C) can be joined together using the phrase
“Provided that”. ‘Provided that’ is the one that's more often used as
a conjunction. It means that one thing happening is dependent on
another thing happening. Both the sentences are in the similar
context and sentence (B) is dependent on the condition given in
statement (C). Therefore, the statement thus formed is “You can
apply for progress payments at any stage during the
construction process provided that sufficient work has been
completed and the bank is satisfied with the progress.” Hence,
option (c) is the most viable answer choice.

8. (a); Statements (A) and (D) can be joined together using the phrase
“In order to”. ‘In order to’ expresses the purpose and answer the
question why something is done. Both the sentences are in the
similar context therefore, sentence (D) [Gaining familiarity with
several idioms and phrases] should be done to achieve the
objective of sentence (A) [Fluent conversations can be enjoyed].
Hence, the statement thus formed is “In order to become fluent
and enjoy conversations, it is important to get familiar with
idioms and phrases.” Hence, option (a) is the most viable answer
choice.
15 Adda247 Publications For any detail, mail us at
Publications@adda247.com
Cracker Book for Bank (IBPS | SBI | RRB PO | Clerk) Mains Exams

9. (d); Statements (A) and (B) can be joined together using the phrase
“Notwithstanding”. ‘Notwithstanding’ means although; in spite of
the fact that. Both the sentences are in the similar context
therefore, the single coherent sentence thus formed is
“Notwithstanding the proud claim that Manipur gifted the game
of modern Polo to the world, Manipuri ponies which are central
to the game are facing a very precarious situation of possible
extinction sooner rather than later.” Hence, option (d) is the
most viable answer choice.

10. (e); All the statements given are independent and complete themselves.
It should be noted that all the four statements cannot be
contextually linked together using the given phrase or conjunction.
Hence, option (e) becomes the most suitable choice.

11. (c); Option (c) is the correct choice .The sentence can be used with the
conjunction ‘while’ in such a way , “While it should be stressed that
real time off from work and vacations promote productivity and
better health in the long term ,it is up to the employees ultimately as
to how they choose to use their time”. All the other options fail to
satisfy the conjunction ‘while’.

12. (b); Option (b) is the most appropriate choice. "Only if" and "if only" are
idiomatic phrases that are quite different in meaning. "If" is used to
express a condition. When used after only i.e. only if, it expresses a
strong condition or the only situation in which something can
happen. Hence the statement can be joined as “Families who were
caught together at the US-Mexico border and separated are now
being offered the opportunity to reunite only if they agree to
deportation, giving up their attempts to seek asylum and stay in the
US “

13. (d); Option (d) is the most appropriate choice. We use the conjunction
unless to mean 'except if'. The clause which follows unless is a
subordinate clause .It needs a main clause to make a complete
sentence. Unless is a conditional word (like if), so we don't use will
or would in the subordinate clause. For example, Unless I hear from
16 Adda247 Publications For any detail, mail us at
Publications@adda247.com
Cracker Book for Bank (IBPS | SBI | RRB PO | Clerk) Mains Exams

you, I'll see you at two o'clock .Now the above given sentence can be
joined in the following way , “Unless the violent ,intolerant wing of
the political left abandons the strong arm tactics to get their ideas
accepted ,more violence will result and that is a sad prospect for our
future.”

14. (b); Option (b) is the most suitable choice. ‘Wherein’ is a conjunction as
well as an adverb. It means ‘in which or in which part’. For
example, The industry will have a situation wherein many companies
will be unable to afford to stay in business.
The above given sentences in options can be joined as “Politics is
arguably the best case study in an act of knowledge and
disinformation wherein we can find anything and everything by
which we wish to strengthen our narrative, no matter how absurd it
is.”

15. (c); The appropriate choice is option (c).The sentence can be used with
the conjunction while in such a way, “While it was true that the state
governments were insisting on keeping the rates high for fear of tax
collections falling short, it was never clear why the Centre didn’t push
for lower rates as it was guaranteeing that states would get 14%
growth in their tax revenues in the next five years.” All the other
options fail to satisfy the given conjunction while.
16. (d); The most suitable choice here is option (d).Although and though
both mean “in spite of something.” They are subordinating
conjunctions. This means that the clause that they introduce is a
subordinate clause which needs a main clause to make it complete.
The sentences can be combined with the given conjunction in such
a form, “They examined the rest of the garments and each, upon
careful observation, was identified in a like fashion although some of
the markings were so faded they were no longer legible.”

17. (c); The appropriate option here is option (c).However is a conjunction


which is used to introduce a statement that contrasts with or seems
to contradict something that has been said previously. Here
however can be used in the following way to combine the given
sentences (A) and (B) i.e. “The Taiwanese government is claiming
17 Adda247 Publications For any detail, mail us at
Publications@adda247.com
Cracker Book for Bank (IBPS | SBI | RRB PO | Clerk) Mains Exams

that the People’s Republic of China would invade the island state if
the U.S. withdrew its support for Taipei, however, it is dubious that
Beijing has the amphibious assault capacity or capability to conduct
a successful sea-based invasion of Taiwan where a forcible entry is
required.”

18. (b); The most suitable choice is option (b). “As” when used in the form
of conjunction is used to indicate that something happens during
the time when something else is taking place. The two sentences
(C) and (D) can be combined in the given way by using this
conjunction: “A growing number of exchanges have stopped fiat
support as Indian crypto currency exchanges ramp up peer-to-peer
and crypto-to-crypto trading.”

19. (a); Statements (B) and (D) form the precise combination to mold a
coherent sentence using the subordinating conjunction “Although”.
Although is used to express “in spite of the fact that; even though.”
Therefore, the coherent sentence thus formed is “Although the
Trump administration unveiled tariffs on steel and aluminum
imports in March, they did not apply to the European Union until the
start of June.” All the other combinations fail to form a
grammatically correct and contextually meaningful sentence.
Hence, option (a) is the most suitable answer choice.
20. (c); Statements (A) and (D) form the precise combination to mold a
coherent sentence using the correlative conjunction “not only…but
also”. When using 'not only' and 'but also' in a sentence, they must
have parallel structure, or include the same parts of speech for
each piece of information. Therefore, the grammatically correct
sentence thus formed is “Cars would be freed not only from their
reliance on planet-killing fossil fuels, but also from the need for a
charging infrastructure whose slow growth remains a major pain
point”. Hence, option (c) is the most viable answer choice.

21. (c); Statements (A) and (D) can be joined together using the connector
“by virtue of” to form a coherent sentence. “By virtue of” is used to
express “on account of or by reason of”. Therefore, the sentence
thus formed is “Many brilliant, forceful leaders truly believe that
18 Adda247 Publications For any detail, mail us at
Publications@adda247.com
Cracker Book for Bank (IBPS | SBI | RRB PO | Clerk) Mains Exams

there is in the world one person who is truly indispensable and that
by virtue of making it to the top of whatever empire they run, they
are made for life.” Therefore, option (c) becomes the most suitable
answer choice.

22. (d); Both the combinations of sentences (D) – (B) and (A) – (C)
successfully form coherent sentences using the conjunction
“unless”. “Unless” means except if (used to introduce the case in
which a statement being made is not true or valid). Therefore, the
meaningful sentences thus formed is “New car prices could soar by
€3,000 under a new emissions system unless the government
intervenes urgently, a leading motor figure has claimed” and “With
the nation in the grip of a heatwave, MPs have warned that heat-
related deaths in the UK will treble by the middle of the century
unless the government tackles this new public health emergency”
respectively. Hence, option (d) is the most suitable answer choice.

23. (b); Statements (C) and (D) form the precise combination to mold a
coherent sentence using the conjunction “because”. ‘Because’
means ‘for the reason that; since’. Therefore, the grammatically
correct sentence thus formed is “Tests under the more realistic
Worldwide harmonized Light Vehicles Test Procedure system are
producing high figures for emissions because fuel consumption is
shown to be higher than it was under the outdated New European
Driving Cycle regime”. Hence, option (b) is the most viable answer
choice.
24. (c); Statements (B) and (C) can be joined together to form a meaningful
sentence using the conjunction ‘while’. “While” is used as a
conjunction to refer ‘during the time that; at the same time as’.
Thus, the coherent sentence formed using the statements (B) and
(C) is “Director General of Archeological Survey of India (ASI) will be
responsible for maintenance of the Taj Mahal while joint secretary of
Ministry of Environment and commissioner of Agra Division will
jointly be in-charge of the Taj Trapezium Zone (TTZ).” All the other
combinations fail to form a coherent sentence. Hence, option (c) is
the most viable answer choice.
19 Adda247 Publications For any detail, mail us at
Publications@adda247.com
Cracker Book for Bank (IBPS | SBI | RRB PO | Clerk) Mains Exams

25. (b); Statements (A) and (D) can be combined successfully using the
conjunction ‘whereas’. “whereas” is used to refer ‘in contrast or
comparison with the fact that’. Therefore, using statements (A) and
(D), the coherent sentence thus formed is “The authorities
suspended the rail services in the Valley whereas the shutdown called
by the separatists closed marketplaces and public transport services”.
All the other combinations fail to form a grammatically correct and
contextually meaningful sentence. Hence, option (b) is the most
viable answer choice.

26. (a); Statements (D) and (B) can be joined together using the
conjunction ‘since’ to form a coherent sentence. ‘Since’ is used to
express ‘for the reason that; because’. Therefore, using the
statements (D) and (B) the meaningful sentence thus formed is
“Since a liberal trade regime results in cheaper imports and more
competitive exports Arvind Panagariya argues that, any move that
seeks to make imports costlier as an import-substitution regime
should be avoided”. All the other combinations fail to form a
grammatically correct and contextually meaningful sentence.
Hence, option (a) is the most viable answer choice.

27. (e); None of the given combinations can successfully form a coherent
sentence using the conjunction “Once”. “Once” is used to refer ‘as
soon as; when’. As, none of the statements can be interlinked with
each other therefore, option (e) is the most feasible answer choice.

28. (d); Combinations (C) - (D) and (C) – (A) can form grammatically
correct and contextually meaningful sentences using the connector
“so that”. ‘So that’ is used as a subordinate clause to show purpose
or to give an explanation. It is used to show an action producing an
intended result or a cause producing an effect. Therefore, the
meaningful sentences thus formed using the combination of
statements (C) – (D) and (C) – (A) are “Netflix Calibrated Mode
automatically adjusts picture settings when you’re watching the
built-in Netflix app so that the image resembles as closely as possible

20 Adda247 Publications For any detail, mail us at


Publications@adda247.com
Cracker Book for Bank (IBPS | SBI | RRB PO | Clerk) Mains Exams

the way Netflix’s content creators saw things when mastering their
shows and movies” and “Netflix Calibrated Mode automatically
adjusts picture settings when you’re watching the built-in Netflix app
so that a content creator can have the confidence that an end viewer
can see exactly the images the Netflix content creators slaved so hard
to create.” All the other combinations do not form grammatically
correct and meaningful sentences thus, option (d) is the most
feasible answer choice.

29. (c); Statements (A) and (B) form the precise combination to mold a
coherent sentence using the conjunction “only if”. ‘Only if’
expresses a strong condition or the only situation in which
something can happen. Therefore, the grammatically correct
sentence thus formed is “The tone of his voice, his manner of
speaking, his walk, gestures, and the lines of his face and the
expression of his eyes, were all carefully observed, and only if all these
features were satisfied was he admitted as a probationer”. Hence,
option (c) is the most viable answer choice.

30. (b); Statements (C) and (D) form the precise combination to mold a
coherent sentence using the conjunction “while”. ‘While’ means
during the time that; at the same time as. Therefore, the
grammatically correct sentence thus formed is “The broader
markets also participated in the rally today with the Nifty Midcap
index rising over a percent while all sectorial indices ended in the
green with Nifty Bank, FMCG, Metal and Pharma rising 1-2 percent”.
Hence, option (b) is the most viable answer choice.

21 Adda247 Publications For any detail, mail us at


Publications@adda247.com
Cracker Book for Bank (IBPS | SBI | RRB PO | Clerk) Mains Exams

1 Adda247 Publications For any detail, mail us at


Publications@adda247.com
Cracker Book for Bank (IBPS | SBI | RRB PO | Clerk) Mains Exams

Chapter

3 Deciphering the Phrases


STUDY TIPS

In such questions a phrase/ clause/ idiom is highlighted in the


sentence which is followed by few statements that tries to decipher its
meaning. A thorough knowledge of phrases and idioms helps to solve such
questions. Moreover, it is important to understand the gist and tone of the
sentence to appropriately comprehend the sentence. It should be
observed that the correct alternative should not alter the context of the
given sentence. Sometimes, the theme of the sentence assists to interpret
the meaning of the highlighted sentence.

Practice Exercise Based on new Pattern

Direction (1-20): In the following question a part of the sentence is given


in bold, it is then followed by three sentences which try to explain the
meaning of the phrase given in bold. Choose the best set of alternatives
from the five options given below each question which explains the
meaning of the phrase correctly without altering the meaning of the given
sentence.

1. Since January, the U.S.’s Countering America’s Adversaries


through Sanctions Act against those conducting business with Russia
and Iran, as well as its decision to walk out of the Iran nuclear deal
have come right up against India’s interests.
(I) Since January, the efforts of the United States to counter its
difficulties by implementing Sanctions Act against the countries
who are involved in business with Russia and Iran, along with its
decision to leave the Iran nuclear deal have clashed directly with
India’s interests.
(II) The United States, since January, is tackling with the difficulties
of the America with the help of Sanctions Act against those
nations who are having business deals with Russia and Iran,
2 Adda247 Publications For any detail, mail us at
Publications@adda247.com
Cracker Book for Bank (IBPS | SBI | RRB PO | Clerk) Mains Exams

together with its decision to depart from the Iran nuclear deal is
coming right up in India’s interests.
(III) Since January, the endeavors of the United States to minimize its
problems by having sanctions on the Act against the nations
who are involving themselves in business relations with Russia
and Iran, as well as its decision to walk out of the Iran deal have
come right up against India’s interests.
(a) Only (I) is correct
(b) Only (III) is correct
(c) Both (I) and (III) are correct
(d) Only (II) is correct.
(e) All are correct

2. Despite the outrage, police brutality towards children and the


separation of children from their parents is nothing new in the U.S.
Mr. Trump is merely the most vicious and crude manifestation of
it.
(I) Despite the outrage, the cruelty of the police towards children
and dissociation of children from their parents have not
happened in the past in U.S. Mr. Trump is nothing more than the
most ferocious and rude demonstration of the cruelty.
(II) The brutality of the police towards children and their separation
from their parents had always been happening in the past in U.S.
because of the outrage of the people and Mr. Trump is solely a
cruel and vulgar display of the brutality.
(III) In spite of the fury, the savagery of the police towards children
and the disunion of the children from their parents have
happened in past in the U.S. Mr. Trump is purely the most brutal
and rude representation of that savagery.
(a) Only (I) is correct
(b) Only (II) is correct
(c) Only (III) is correct.
(d) Both (II) and (III) are correct
(e) All are correct
3 Adda247 Publications For any detail, mail us at
Publications@adda247.com
Cracker Book for Bank (IBPS | SBI | RRB PO | Clerk) Mains Exams

3. Ms. Ocasio-Cortez’s win will likely reverberate through the


Democratic Party, and possibly put the Republicans on notice as well,
for it was built on an entirely different ethos from the unfettered
rein given to corporate America by the Citizens United decision of the
Supreme Court in 2010.
(I) The win of Ms. Ocasio-Cortez will probably echo through the
Democratic Party, and plausibly send warning among the
Republicans as well, because it was constructed on an absolutely
different ethics in comparison to the wide-open controls that
were given by the Citizens United decision of the Supreme Court
in 2010 to the American corporates.
(II) Ms. Ocasio-Cortez’s win will possibly reverberate through the
Democratic Party, and probably send a notice to the
Republicans, because it was based on a totally different climate
as compared to the confined powers given to the corporate
America by the Citizens United decision of the Supreme Court in
2010.
(III) As Ms. Ocasio-Cortez’s win was built on a completely different
character as opposed to the unrestricted powers that were
granted to corporate America by the Citizens United decision of
the Supreme Court in 2010, it will possibly resonate through the
Democratic Party, and probably make the Republicans attentive
too.
(a) Only (I) I correct.
(b) Both (I) and (II) are correct.
(c) Both (II) and (III) are correct.
(d) Both (I) and (III) are correct.
(e) All are correct

4. President Recep Tayyip Erdoğan, who has been president since 2014
and whose party, the AKP, has been in power since 2002, snuffed out
the hopes of government critics and secured the presidency with
over 52 percent of the vote.
[I] By securing the presidency with more than 52 percent of the
vote, President Recep Tayyip Erdoğa who has been president
since 2014 from the AKP party which is in power since 2002,
has suppressed the hopes of its critics.
4 Adda247 Publications For any detail, mail us at
Publications@adda247.com
Cracker Book for Bank (IBPS | SBI | RRB PO | Clerk) Mains Exams

[II] President Recep Tayyip Erdoğan, who has been president since
2014 and whose party, the AKP, has been in power since 2002,
developed the hopes for government critics by securing the
presidency with over 52 percent of the vote.
[III] Since 2014, Recep Tayyip Erdoğan has been the president from
the party AKP which has been in power since 2002, has
supported the hopes of government critics and secured the
presidency with over 52 percent of the vote.
(a) only (I)
(b) only (III)
(c) Both (I) and (II)
(d) All (I), (II) and (III)
(e) none of these

5. Messi may be more humane, humble and yes, brilliant, but he


couldn’t cut the mustard to take his team past the tape.
[I] Even though Messi is more humane, humble and brilliant, but he
couldn’t help his team to successfully achieve the winnings.
[II] Despite Messi being so humane, humble and brilliant, he failed
to accomplish a winning position for his team.
[III] However, humane, humble and brilliant Messi is, he failed to
recognize the weaknesses in the team and was unable to stop it
from faltering.
(a) only (I)
(b) only (III)
(c) Both (I) and (II)
(d) All (I), (II) and (III)
(e) none of these

6. After frequently switching electoral partners, Chief Minister Nitish


Kumar does not enjoy high political credibility despite his efforts to
couch opportunism in idealistic garb.
[I] Since, Chief Minister Nitish Kumar frequently switches electoral
partners, he is not considered as a trustworthy partner even
after his attempts to redeem favorable circumstances in the
disguise of idealism.

5 Adda247 Publications For any detail, mail us at


Publications@adda247.com
Cracker Book for Bank (IBPS | SBI | RRB PO | Clerk) Mains Exams

[II] Chief Minister Nitish Kumar declines every opportunity that


comes his way that renders him as an ideal politician as he
seldom switches electoral partners.
[III] Chief Minister Nitish Kumar does not enjoy high political
credibility as he switches electoral partners whenever he sees
an advantage, but he does that in the pretext of idealism.
(a) Both (I) and (III)
(b) only (III)
(c) Both (I) and (II)
(d) All (I), (II) and (III)
(e) none of these

7. He hit the nail on the head when he said that most people won’t
change their ways and continue to blame the system for all their ills.
(I) His opinion that most people won’t change their ways and
continue to blame the system for all their ills was agreed by
everyone.
(II) He was very accurate when he said that majority of the people
do not tend to change their ways and never fail to blame the
system for all their difficulties.
(III) He was pretty much right when he uttered that almost all people
fail to change their ways and persevere to blame the system for
all of their troubles.
(a) Only (III) is correct
(b) Only (II) is correct
(c) Both (II) and (III) are correct
(d) Both (I) and (II) are correct
(e) All are correct

8. While Hollywood has produced countless tales of AI-induced woe,


the vast majority of these movies have proven to be poor predictors of
how AI could potentially work, in strictly theoretical terms.
(I) Hollywood has produced innumerable tales of tribulation
effectuated by AI, but most of those movies have not succeeded
in predicting the real working potential of AI, precisely in
theory.
6 Adda247 Publications For any detail, mail us at
Publications@adda247.com
Cracker Book for Bank (IBPS | SBI | RRB PO | Clerk) Mains Exams

(II) Many movies have been produced by Hollywood which


portrayed AI as something which causes lots of agony, but
majority of these movies have been substandard in foretelling
the working capacity of AI, in rigorously conceptual terms.
(III) A number of Hollywood movies have shown AI to be something
that causes lots of serenity to people but none of them have been
able to anticipate the true potential of AI, in exact theoretical
terms.
(a) Only (I) is correct
(b) Only (II) is correct
(c) Both (I) and (III) are correct
(d) Both (I) and (II) are correct
(e) All are correct

9. Carl Robinson sees himself not merely as a comedian, however, but as


a voice crying in the wilderness, revealing the truth behind the
day's events, trying to disabuse comfortable New Yorkers of their
comfortable illusions.
(I) Carl Robinson sees himself not merely as a comedian but also as
a person who says things related to the daily lives of people who
are not socially acceptable, trying to convince the New Yorkers
that their life is full of cozy illusions, thinks Carl Robinson.
(II) Carl Robinson observes himself not only as a mere comedian,
but also as a person who discloses truths about everyday life
that no one wants to accept, in an attempt to persuade the New
Yorkers that the pleasant life they live is nothing more than
comfortable illusions.
(III) Carl Robinson says that he is not just a comedian, but an orthodox
person who practices a conventional method of telling the happy
New Yorkers about the illusions in which they live their
everyday life.
(a) Only (I) is correct
(b) Only (II) is correct
(c) Both (I) and (II) are correct
(d) Only (III) is correct.
(e) All are correct
7 Adda247 Publications For any detail, mail us at
Publications@adda247.com
Cracker Book for Bank (IBPS | SBI | RRB PO | Clerk) Mains Exams

10. Should you be heading to Australia on a full working visa, you’ll need
to wrap your head around their tax laws and regulations so that you
don’t find yourself in violation of any directives.
(I) If you are moving to Australia on a full working visa, you must
understand completely their tax laws and regulations so that
you don’t fail to comply with any of their authoritative
instructions.
(II) The knowledge of Australian’s tax norms and regulations will
help you to attain full time working visa, since you won’t violate
any of their directives.
(III) While relocating to Australia on a full working visa, you must
have the entire understanding of their tax norms and
regulations, so that you don’t breach their official policies.
(a) Only (III) is correct
(b) Only (II) is correct
(c) Both (I) and (III) are correct
(d) Both (I) and (II) are correct
(e) All are correct

11. As tempting as it is, don't burn bridges at work is a good rule,


especially over something relatively minor.
(I) However strong the urge is, you shouldn’t spoil your
relationships at your work place, particularly over a minor issue.
(II) The policy of not ruining your work relationships in your work
place works better, only in minor issues.
(III) The policy of not working efficiently in your workplace serves
good, especially in case of minor projects.
(a) Only (I) is correct
(b) Only (III) is correct
(c) Both (II) and (III) are correct
(d) Both (I) and (II) are correct
(e) All are correct

8 Adda247 Publications For any detail, mail us at


Publications@adda247.com
Cracker Book for Bank (IBPS | SBI | RRB PO | Clerk) Mains Exams

12. Emerging economies do need a tool to stabilize capital flows,


intervention by central banks in foreign exchange markets is a blunt
instrument with many uncontrolled spillover effects.
(I) There is a requirement to introduce a new method to stabilize
capital flows in emerging economies, since the interference by
central banks in foreign markets has severe consequences in
various economic fields.
(II) Emerging economies must develop a new instrument to liquify
capital flows, as the instruments of central banks in foreign
exchange markets have caused several economic effects.
(III) Central bank’s dealings in foreign exchange market is an
instrument to stabilize capital flows in emerging economies,
however, its outcomes are unsatisfactory.
(a) Only (III) is correct
(b) Only (I) is correct
(c) Both (I) and (III) are correct
(d) Both (I) and (II) are correct
(e) All are correct

13. The up-and-down relationship between President Donald Trump and


the Chinese government has hit another rough patch, and experts say
the US president is treading on thin ice.
(I) The problems in the relationship between President Donald
Trump and the Chinese government have increased which led
the US government to enter in risky trades.
(II) The to-and-fro relationship between the US president and
Chinese government has hit another rough patch, while in
expert’s opinion the US president will face major consequences.
(III) According to the experts, the US President Donald Trump is
taking huge risk since, the fluctuating relationship between the
US president and the Chinese government has hit another low.
(a) Only (III) is correct
(b) Only (I) is correct
(c) Both (I) and (III) are correct
(d) Both (I) and (II) are correct
(e) All are correct
9 Adda247 Publications For any detail, mail us at
Publications@adda247.com
Cracker Book for Bank (IBPS | SBI | RRB PO | Clerk) Mains Exams

14. The future growth that is assumed, depends on the sustainability of


natural capital, which is threatened due to the myopic pursuit of
GDP growth.
(I) The expected growth which relies on the optimum utilization of
natural capital, is difficult to achieve as, the focus is merely on
the short-term growth of GDP.
(II) Sustainable development shall help to achieve the assumed
growth along with the short-term growth in GDP.
(III) Due to the short-term plans for GDP growth, the expected future
growth may not be achieved which depends on the viable
utilization of natural resources.
(a) Only (III) is correct
(b) Only (I) is correct
(c) Both (I) and (III) are correct
(d) Both (I) and (II) are correct
(e) All are correct

15. Tesla and SpaceX CEO, Elon Musk, warned in 2016 that, if artificial
intelligence is left unregulated, humans could devolve into the
equivalent of house cats next to increasingly powerful
supercomputers.
(I) In 2016, the CEO of Tesla and SpaceX, Elon Musk said that if
artificial intelligence is not controlled properly, the very
powerful supercomputers will mutate humans into house cats.
(II) Elon Musk, CEO of Tesla and SpaceX, alerted in 2016 that
artificial intelligence will be left unregulated and humans will
transform into house cats before extremely powerful
supercomputers.
(III)In 2016, Elon Musk, CEO of Tesla and SpaceX, cautioned that if
artificial intelligence will not be properly regulated, in future,
the human condition will deteriorate into something similar to
domesticated pets in front of increasingly powerful computers.
(a) Only (II) is correct
(b) Only (III) is correct
(c) Both (II) and (III) are correct
(d) Both (I) and (III) are correct
(e) All are correct
10 Adda247 Publications For any detail, mail us at
Publications@adda247.com
Cracker Book for Bank (IBPS | SBI | RRB PO | Clerk) Mains Exams

16. The complaints, brought to light by the nonpartisan government


watchdog Common Cause, call on federal prosecutors and
regulators to investigate whether the UK-based data analytics group
violated a US law barring foreign nationals from participating in
certain election-related activities through its work for Donald
Trump’s campaign.
(I) The complaints against the UK-based data analytics group, put
forward by the unbiased government security-keeping
organization Common Cause, appealed the federal prosecutors
and regulators to probe whether a US law which blocks
foreigners from involving in some of the election-related
activities, was breached by the data analytics firm during its
assistance to Donald Trump’s campaign.
(II) The complaints that were brought to everybody’s attention by
an evenhanded government organization for the common cause,
urged the federal law makers to check whether there was
transgression of a US law that stops foreign people from
entering into any election-related works by the UK-based data
analytics group while working for Donald Trump’s campaign.
(III) Common Cause, an impartial law-keeping government
organization, highlighted complaints against the UK-based data
analytics group, requesting the federal prosecutors and
regulators to scrutinize whether a US law that prohibits foreign
persons from indulging in few activities related to election, was
infringed by the data analytics firm while contributing to Donald
Trump’s campaign.
(a) Both (I) and (III) are correct
(b) Only (III) is correct
(c) Only (I) is correct
(d) Only (II) is correct
(e) All are correct

17. Venezuela threw a late spanner in the works when Oil Minister
Erwin Arrieta told ministers during preliminary talks that with a new
government taking over in February he could not commit to an
extension of the existing production cuts.

11 Adda247 Publications For any detail, mail us at


Publications@adda247.com
Cracker Book for Bank (IBPS | SBI | RRB PO | Clerk) Mains Exams

(I) Keeping in mind that a new government was taking over in


February in Venezuela, Oil Minister Erwin Arrieta told the
ministers during initial talks that he could not commit to an
extension of the prevailing production cuts, thereby hindering
the process.
(II) Venezuela obeyed the agreement when during introductory
talks, the ministers were told by Oil Minister Erwin Arrieta that
since a new government was taking control in February, he
cannot commit to an addendum of existing production cuts.
(III) Venezuela hurled a wrench on the ministers during the
preparatory talks, when Oil Minister Erwin Arrieta told them
that because a new government was taking over in February and
so, he would not commit to an enhancement of the existing
production cuts.
(a) Both (I) and (II) are correct
(b) Only (I) is correct
(c) Only (II) is correct
(d) Both (II) and (III) are correct.
(e) All are correct

18. Karnataka Chief Minister Siddaramaiah set the cat among the
pigeons when he highlighted Kannada pride by unveiling an official
state flag last month.
(I) The chief minister of Karnataka Siddaramaiah precipitated a
controversy when he accentuated the Kannada pride by
revealing an official state flag last month.
(II) Karnataka Chief minister Siddaramaiah provoked a ceasefire
when he highlighted Kannada pride by introducing an official
state flag last month.
(III) Last month when the chief minister of Karnataka Siddaramaiah
divulged the official state flag as the Kannada pride, he initiated
a fierce argument.
(a) both (I) and (III)
(b) both (II) and (III)
(c) only (II)
(d) All (I) (II) and (III)
(e) none of these
12 Adda247 Publications For any detail, mail us at
Publications@adda247.com
Cracker Book for Bank (IBPS | SBI | RRB PO | Clerk) Mains Exams

19. In the latest football news, PSG forward, Mbappe-aged 19 years,


underlined his rich talent and claimed his place in the French record
books when he poked home in the first half.
(I) The late news of football reported, Mbappe, a 19 years old PSG
forward undermine his talent and recorded his place in French
record books when he saved the goal in the first half.
(II) According to the latest football news, a 19 years old PSG
forward, Mbappe, underscored his talent and registered himself
in French record books when he poked a player in the first half
of the game.
(III) Mbappe- a forward of PSG, aged 19 years, showcased his
immense talent and positioned himself in French record books
when scored a goal in the first half of the game as reported in
the latest football news.
(a) both (I) and (II)
(b) only (III)
(c) only (II)
(d) both (II) and (III)
(e) none of these

20. In Jammu and Kashmir, the post-poll coalition of the two polar
opposites, the Bharatiya Janata Party and the People’s Democratic
Party which ceases to exist now, was never meant to coalesce into a
credible pre-poll alliance.
(I) In Jammu and Kashmir, the joining of the two opposite parties,
the Bharatiya Janata Party and the People’s Democratic Party
after the poll endures now, was in the first place meant to be
broken before the poll.
(II) The post-poll coalition of the two conflicting parties, the
Bharatiya Janata Party and the People’s Democratic Party that
fails to exist now as it was never meant to form a deceptive
alliance before the polls in Jammu and Kashmir.
(III) In Jammu & Kashmir, the post-poll coalition of the two totally
opposite parties, the Bharatiya Janata Party and the People’s
Democratic Party, that does not exist anymore, was almost
impossible to be observed as a trustworthy alliance even before
the polls.
13 Adda247 Publications For any detail, mail us at
Publications@adda247.com
Cracker Book for Bank (IBPS | SBI | RRB PO | Clerk) Mains Exams

(a) both (I) and (II)


(b) only (III)
(c) only (II)
(d) both (II) and (III)
(e) none of these

Directions (21-25): In the following question a part of the sentence is


given in bold, it is then followed by four sentences which try to explain the
meaning of the idiom/phrase given in bold. Choose the alternative from the
four given below each question which explains the meaning of the phrase
correctly without altering the meaning of the sentence given as question. If
none of the sentence explains the meaning of the highlighted phrase, choose
option (e) i.e., “none of these” as your answer choice.

21. ICICI Bank’s troubles are rooted in a 2016 complaint by an investor


alleging a quid pro quo deal between Ms. Kochhar’s immediate family
members and the Videocon group, which got a Rs 3250 crore loan
from it.
(a) ICICI bank got in trouble after an investor accused that the deal
between Ms Kochhar’s immediate family members and the
Videocon group was a result of a undue favour granted that got a
sum amount of RS 3250 crore loan from it.
(b) ICICI bank got in trouble after an investor asserted that the deal
between Ms Kochhar’s immediate family members and the
Videocon group was an act of rivalry that got a sum amount of
RS 3250 crore loan from it.
(c) ICICI bank got in trouble after an investor accused that the deal
between Ms Kochhar’s immediate family members and the
Videocon group was an act of a denunciation that got a sum
amount of RS 3250 crore loan from it.
(d) ICICI bank got in trouble after an investor maintained that the
deal between Ms Kochhar’s immediate family members and the
Videocon group was a result of a continued adulation that got a
sum amount of RS 3250 crore loan from it.
(e) none of these

14 Adda247 Publications For any detail, mail us at


Publications@adda247.com
Cracker Book for Bank (IBPS | SBI | RRB PO | Clerk) Mains Exams

22. Journalism needs to come up with alternative words and terms to


describe such horrendous attacks on people. The time has come for
journalists to drop these cloak-and-dagger words.
(a) Journalism use plain words to describe horrible attacks on
people that cannot be justified by use of words that works to
conceal such actions.
(b) Journalism requires to use alternate words to describe horrible
attacks on people that cannot be justified by use of words that
works to conceal such actions.
(c) Journalism necessitate plain words to describe horrible attacks
on people that cannot be justified by use of words that works to
conceal such actions.
(d) Journalism calls for plain words to describe horrible attacks on
people that cannot be justified by use of words that works to
conceal such actions.
(e) none of these

23. From khap panchayats to various extra- judicial entities, many


present cruel invasions into the lives of young people as corrective
mechanisms and give a cloak of respectability to crimes with the
prefix ‘honour’.
(a) Various extra-judicial entities along with khap panchayats
present cruel invasive measures as part of corrective measures
in the lives of young people and give an honorable garment to
crimes with the prefix ‘honour’.
(b) Various extra-judicial entities along with khap panchayats
present cruel invasive measures as correct measure in the lives
of young people and give it a respectable status to crimes with
the prefix ‘honor’.
(c) Various extra-judicial entities along with khap panchayats
present cruel invasions in young people’s lives to correct them
and give them a respectable garment.
(d) Various extra-judicial entities are engaged in presenting the
cruel intrusions in young people’s lives as part of the corrective
measure and a mark of respectable thing to conceal the wrong
doings with prefixes such as ‘honour.’
(e) none of these
15 Adda247 Publications For any detail, mail us at
Publications@adda247.com
Cracker Book for Bank (IBPS | SBI | RRB PO | Clerk) Mains Exams

24. Experts opine that India has a solar rooftop power potential of up to
400 GW. The way ahead is to have institutional channel partners to
rev up rooftop capacity.
(a) Experts moot that India has a solar rooftop power potential of
upto 400 GW. Institutional channel partners have to increase the
roof top capacity in order to advance.
(b) Experts moot that India has a solar rooftop power potential of
upto 400 GW. Institutional channel partners have to decrease
the roof top capacity in order to advance.
(c) Experts moot that India has a solar rooftop power potential of
upto 400 GW. Institutional channel partners have to decelerate
the roof top capacity in order to advance.
(d) Experts moot that India has a solar rooftop power potential of
upto 400 GW. Institutional channel partners have to de-escalate
the roof top capacity in order to advance.
(e) none of these

25. Modi said nation building required the joint effort of industrialists as
well as farmers, workers, bankers and civil servants. Crooks in all
spheres must be prosecuted, but all industrialists should not be
tarred with the same brush.
(a) Modi said nation building required the cumulative effort of
industrialists, farmers, workers, bankers and civil servants.
Action should be taken against all the crooks but all
industrialists should not be judged under the same category and
punished for the mistakes they have not committed.
(b) Modi stated nation building required the combined effort of
industrialists as well as farmers, workers, bankers and civil
servants. Crooks in all spheres must be prosecuted, but all
industrialists should not be painted with the same brush.
(c) Modi stated nation building required the combined efforts of
industrialists as well as farmers, workers, bankers and civil
servants. Crooks in all spheres must be prosecuted, but all
industrialists should not be painted with the brush.
(d) Modi stated nation building required the combined effort of
industrialists as well as farmers, workers, bankers and civil
servants. Crooks in all spheres must be prosecuted but
industrialists should be spared.
(e) none of these
16 Adda247 Publications For any detail, mail us at
Publications@adda247.com
Cracker Book for Bank (IBPS | SBI | RRB PO | Clerk) Mains Exams

Directions (26-30): In the following questions, a phrase/idiom is given in


bold whose meaning can be inferred from one of the highlighted
words/phrases of four sentences given below each phrase. Choose the most
appropriate meaning of the phrase among the four options that can also be
replaced by the given phrase without altering the meaning of the sentence.
If none of the sentences conveys the correct meaning, choose (e) i.e., “None
of these” as your answer.

26. burn the candle at both ends


(a) Solving customer crises is a delicate balance, and to make
matters more complicated, no two angry customers are alike.
Some take to social media platforms like YouTube and Twitter
while others leave reviews on Yelp and Google.
(b) He added that “our men are working day and night to make
sure that crashes do not happen and eventually if it happens, our
men are expected to be there within 15 minutes to rescue
victims”.
(c) After years of turmoil in Central African Republic thousands
have lost their lives; many more lost their homes, logging
companies were plundered, vehicles stolen, machines and other
timber production equipment ransacked.
(d) ‘During road rage, will the driver-less car fight for us? ', worried
Indian asks Elon Musk.
(e) none of these

27. A hot potato


(a) Saudi Arabia's crown prince said Iran, Turkey and extremist
groups represent a “triangle of evil,” and predicted the
kingdom's confrontation with Gulf Arab neighbor Qatar could
last for years.
(b) The Actor who played Stephen Hawking in Theory of
Everything, said in a statement: "We have lost a truly beautiful
mind, an astonishing scientist and the funniest man I have ever
had the pleasure to meet. "
(c) World Against Toys Causing Harm (WATCH) urged countries to
ban the most dangerous kid’s toys that could lead to injury – or
death.
17 Adda247 Publications For any detail, mail us at
Publications@adda247.com
Cracker Book for Bank (IBPS | SBI | RRB PO | Clerk) Mains Exams

(d) Beijing, which claims more than 80% of the South China Sea, is
ramping up its military presence and accelerating construction
on disputed desert islands.
(e) none of these

28. Back to the drawing board


(a) The star of "Ace Ventura: Pet Detective" Jim Carrey is making
new paintings and raising eyebrows of political class with anti-
Trump and anti-Republicans themes.
(b) Students should always stay motivated and whatever task they
find most odious, they should move to the front of the line and
finish it first.
(c) People should always ask themselves one question time and
again: How do you know if you're on the road to achieve
success or on the road to failure?
(d) After the death of a woman in Florida from accident caused by
self-driving car of Uber, the self-driving technology is starting it
all over again from scratch.
(e) none of these

29. be glad to see the back of


(a) "I'm so happy I'm getting my life back. I'll be glad to see the
dialysis gone, but I will miss the nurses and the other patients
that I've gotten to know in recent times” said the patient
struggling with dialysis.
(b) Former British high jumper Mike Edwards has accused the IAAF
of cowardice with the rules of athletics' governing body set to
bar him from representing Nigeria at the Commonwealth Games.
(c) This has been a terribly sad time for clowns, those purveyors of
happiness whose recorded history dates back to ancient Greece.
(d) China’s runaway space station Tiangong-1 is all set to re-enter
Earth’s atmosphere.
(e) none of these

18 Adda247 Publications For any detail, mail us at


Publications@adda247.com
Cracker Book for Bank (IBPS | SBI | RRB PO | Clerk) Mains Exams

30. Blessing in disguise


(a) Before the 2018 event, no North Carolina team had advanced
past the first round, but the falcons made that old history fade
away.
(b) A charity shop worker was left stunned after someone donated
an old book containing handwritten notes from her own father
50 years after his death.
(c) There has been a greater recognition lately of town hall and
other municipal buildings as symbols of the localities that built
them and the residents who use them.
(d) Gold prices rose on Monday after three consecutive sessions of
falls as the dollar eased amid fresh concerns of trade tensions
between the United States and China.
(e) none of these

Solutions

1. (a); The phrase “the U.S.’s Countering America’s Adversaries through


Sanctions Act” means the efforts of United States to counter its
adversaries by implementing Sanctions Act. Among the given
statements, only the sentence (I) expresses the meaning which
complies with the meaning of the phrase and at the same time it
makes sure that the actual meaning of the sentence remains intact.
Statements (II) and (III) are irrelevant as they alter the meaning of
the sentence. Hence (a) is the correct option.

2. (c); The phrase “the most vicious and crude manifestation of it” means
the most ferocious and rude representation of the brutality of the
US police. Among the given statements, only the sentence (III)
expresses the meaning which complies with the meaning of the
phrase and at the same time it makes sure that the actual meaning
of the sentence remains intact. Statements (I) and (II) are
irrelevant as they alter the meaning of the sentence. Hence (c) is
the correct option.
19 Adda247 Publications For any detail, mail us at
Publications@adda247.com
Cracker Book for Bank (IBPS | SBI | RRB PO | Clerk) Mains Exams

3. (d); The phrase “an entirely different ethos from the unfettered rein”
means a totally different ethics and characters in comparison to the
wide-open controls and powers that were given to corporate
America. Among the given statements, both the sentences (I) and
(III) express the meaning which complies with the meaning of the
phrase and at the same time they make sure that the actual
meaning of the sentence remains intact. Statement (II) is irrelevant
as it alters the meaning of the sentence. Hence (d) is the correct
option.

4. (a); The phrase “snuffed out the hopes of government critics” means
to cause something to end suddenly. Among the given statements,
only sentence (I) expresses the meaning which complies with the
meaning of the phrase and at the same time it makes sure that the
actual meaning of the sentence remains intact. Statement (II) and
(III) are irrelevant as it alters the meaning of the sentence. Hence
(a) is the most suitable answer choice.

5. (c); The idiom “cut the mustard” means come up to expectations;


reach the required standard, while “past the tape” means to
achieve something successfully. Thus, the phrase “couldn’t cut the
mustard to take his team past the tape” means failing in
assisting his team to secure a win. Among the given statements,
both sentences (I) and (II) express the meaning which complies
with the meaning of the phrase and at the same time they make
sure that the actual meaning of the sentence remains intact.
Statement (III) is irrelevant as it alters the meaning of the sentence.
Hence (c) is the most suitable answer choice.

6. (a); The phrase “to couch opportunism in idealistic garb” means to


present opportunities in an ideal manner. Among the given
statements, both sentences (I) and (III) express the meaning which
complies with the meaning of the phrase and at the same time they
make sure that the actual meaning of the sentence remains intact.
Statement (II) is irrelevant as it alters the meaning of the sentence.
Hence (a) is the most suitable answer choice.

20 Adda247 Publications For any detail, mail us at


Publications@adda247.com
Cracker Book for Bank (IBPS | SBI | RRB PO | Clerk) Mains Exams

7. (c); The phrase “hit the nail on the head” means to be accurately right
about something or find the exact answer. Among the given
statements, both sentences (II) and (III) express the meaning
which complies with the meaning of the phrase and at the same
time they make sure that the actual meaning of the sentence
remains intact. Statement (I) is irrelevant as it alters the meaning
of the sentence. Hence (c) is the correct option.

8. (d); In the context of this sentence, the phrase “countless tales of AI-
induced woe” means many movies which displayed the pain and
sorrow that have been caused because of AI. Among the given
statements, both sentences (I) and (II) express the meaning which
complies with the meaning of the phrase and at the same time they
make sure that the actual meaning of the sentence remains intact.
Statement (III) is irrelevant as it alters the meaning of the sentence.
Hence (d) is the correct option.

9. (c); In the context of this sentence, the phrase “a voice crying in the
wilderness” means to express an idea that is not accepted by
others or to say something that is not acceptable by a group or
society. Among the given statements, both sentences (I) and (II)
express the meaning which complies with the meaning of the
phrase and at the same time they make sure that the actual
meaning of the sentence remains intact. Statement (III) is
irrelevant as it alters the meaning of the sentence. Hence (c) is the
correct option

10. (c); The phrase “to wrap your head around” means to comprehend
something that one considers challenging, confusing, or a foreign
concept. Among the given statements, both sentences (I) and (III)
express the meaning which complies to the meaning of the phrase
and at the same time they make sure that the actual meaning of the
sentence remains intact. Statement (II) is irrelevant as it alters the
meaning of the sentence. Hence (c) is the correct option.

21 Adda247 Publications For any detail, mail us at


Publications@adda247.com
Cracker Book for Bank (IBPS | SBI | RRB PO | Clerk) Mains Exams

11. (a); The phrase “don't burn bridges at work” means ‘to not destroy
one's path, connections, reputation, opportunities, etc., particularly
intentionally.”. Thus, among all the given options only sentence (I)
serves the appropriate meaning of the sentence. Other sentences
stand contextually incorrect. Hence (a) is the correct option.

12. (b); The phrase “many uncontrolled spillover effects” means


unexpected consequences or repercussions spreading in different
sectors. Among the given statements, only sentences (I) expresses
the precise meaning which complies to the meaning of the phrase
and at the same time it makes sure that the actual meaning of the
sentence remains intact. Statement (II) and (III) are irrelevant as it
alters the meaning of the sentence. Hence (b) is the most suitable
answer choice.

13. (a); The phrase “treading on thin ice” is used to describe a situation of
possible danger or risk, where the "ice" could break at any time and
the person treading or walking on it could fall in. Among the given
statements only statement (III) seem to provide the same
explanation, but the statements (I) and (II) change the meaning of
the sentence as they do not specify the correct meaning of the
phrase whereas statement (III) describes the exact sense of the
phrase without altering the meaning of the sentence. Hence option
(a) is the correct choice.

14. (c); The phrase “the myopic pursuit of GDP growth” means ‘the
short-term plans for the growth of GDP’. Among the given
statements, both sentences (I) and (III) express the meaning which
complies to the meaning of the phrase and at the same time they
make sure that the actual meaning of the sentence remains intact.
Statement (II) is irrelevant as it alters the meaning of the sentence.
Hence (c) is the correct option.

22 Adda247 Publications For any detail, mail us at


Publications@adda247.com
Cracker Book for Bank (IBPS | SBI | RRB PO | Clerk) Mains Exams

15. (b); In the context of this sentence, the phrase “devolve into the
equivalent of house cats” means the condition of humans will
worsen into a level identical to that of a pet animal. Among the
given statements, only the sentence (III) expresses the meaning
which complies with the meaning of the phrase and at the same
time it makes sure that the actual meaning of the sentence remains
intact. Statements (I) and (II) are irrelevant as they alter the
meaning of the sentence. Hence (b) is the correct option.

16. (a); In the context of this sentence, the phrase “brought to light by the
nonpartisan government watchdog Common Cause, call on
federal prosecutors and regulators” means that the complaint
was brought to everyone’s attention by an unbiased government
monitoring organization named Common Cause and the
organization requested federal officials to probe into a matter
related to violation of a US law . Among the given statements, both
sentences (I) and (III) express the meaning which complies with
the meaning of the phrase and at the same time they make sure
that the actual meaning of the sentence remains intact. Statement
(II) is irrelevant as it alters the meaning of the sentence. Hence (a)
is the correct option.

17. (b); In the context of this sentence, the phrase “spanner in the works”
means a roadblock that prevents an activity from succeeding or to
deliberately sabotage an activity. Among the given statements, only
the sentence (I) expresses the meaning which complies with the
meaning of the phrase and at the same time it makes sure that the
actual meaning of the sentence remains intact. Statements (II) and
(III) are irrelevant as they alter the meaning of the sentence. Hence
(b) is the correct option.

18. (a); The phrase “set the cat among the pigeons” means to do or say
something that is likely to cause alarm, controversy, or unrest
among a lot of people. Among the given statements, both sentences
(I) and (III) express the meaning which complies with the meaning
of the phrase and at the same time they make sure that the actual
meaning of the sentence remains intact. Statement (II) is irrelevant
as it alters the meaning of the sentence. Hence (a) is the most
suitable answer choice.
23 Adda247 Publications For any detail, mail us at
Publications@adda247.com
Cracker Book for Bank (IBPS | SBI | RRB PO | Clerk) Mains Exams

19. (b); The phrase “he poked home in the first half” means to score a
goal in the first half of the game. Among the given statements, only
statement (III) expresses the meaning which complies with the
meaning of the phrase and at the same time they make sure that
the actual meaning of the sentence remains intact. Statements (I)
and (II) are irrelevant as it alters the meaning of the given
sentence. Hence option (b) is the most viable answer choice.

20. (b); The phrase “coalesce into a credible pre-poll alliance” in


reference to the sentence means that the alliance between the two
parties was never meant to form a reliable union before the polls as
well. Among the given statements, only statement (III) expresses
the meaning which complies with the meaning of the phrase and at
the same time they make sure that the actual meaning of the
sentence remains intact. Statements (I) and (II) are irrelevant as it
alters the meaning of the given sentence. Hence option (b) is the
most viable answer choice.

21. (a); The correct option is (a). The correct meaning of the idiomatic
expression quid pro quo is granting undue favor to someone in
return for something. Therefore, the only statement that complies
with the statement in question is statement (a). All other
statements are irrelevant in context to the statement provided in
question.

22. (d); The correct option is (d). The correct meaning of the idiomatic
expression cloak and dagger is using intrigue or confusing words in
order to conceal the wrongdoings. Therefore, the only statement
that complies with the statement in question is statement (d). All
other statements are grammatically incorrect in context to the
statement provided in question.

24 Adda247 Publications For any detail, mail us at


Publications@adda247.com
Cracker Book for Bank (IBPS | SBI | RRB PO | Clerk) Mains Exams

23. (d); The correct option is (d). The correct meaning of the idiomatic
expression cloak of respectability is using respectable remark or
title for something in order to conceal the wrongdoings. Therefore,
the only statement that complies with the statement in question is
statement (d). All other statements are irrelevant in context to the
statement provided in question.

24. (a); The correct option is (a). The correct meaning of the idiomatic
expression rev up is to increase the rate of or speed of something.
Therefore, the only statement that complies with the statement in
question is statement (a). All other statements are irrelevant in
context to the statement provided in question.

25. (a); The correct option is (a). The correct meaning of the idiomatic
expression tarred up with same brush is to unfairly categorize
someone or something as being the same as another person or
thing, usually in a negative manner. Therefore, the only statement
that complies with the statement in question is statement (a). All
other statements are irrelevant in context to the statement
provided in question.

26. (b); burn the candle at both ends: To overwork or exhaust oneself by
doing too many things, especially both late at night and early in the
morning Ransacked: go through (a place) stealing things and
causing damage

27. (d); A hot potato: Speak of an issue (mostly current) which many
people are talking about and which is usually disputed China’s
illegal control of South China sea is a current and disputed topic
hence (e) fits accurately with the meaning of “a hot potato” as
South China sea is a hot potato.

28. (d); Back to the drawing board: When an attempt fails and it's time to
start all over The self-driving car technology by Uber failed when it
couldn’t recognise a woman on road and now it is back to the

25 Adda247 Publications For any detail, mail us at


Publications@adda247.com
Cracker Book for Bank (IBPS | SBI | RRB PO | Clerk) Mains Exams

drawing board to start it all over again and re-innovate the


technology hence (d) fits perfectly to the meaning of back to the
drawing board.

29. (a); Be glad to see the back of: Be happy when a person or something
leaves.
Here, a patient is talking about being happy when dialysis goes
away (cured), which clearly signifies the meaning of the idiom ‘be
glad to see the back of’
Cowardice: lack of bravery

30. (c); Blessing in disguise: Something good that isn't recognized at first
Old townhall buildings or municipal buildings are the heritage of a
city but they were not recognized earlier but now they are being
recognized hence it conveys meaning of the given idiom.

26 Adda247 Publications For any detail, mail us at


Publications@adda247.com
Cracker Book for Bank (IBPS | SBI | RRB PO | Clerk) Mains Exams

1 Adda247 Publications For any detail, mail us at


Publications@adda247.com
Cracker Book for Bank (IBPS | SBI | RRB PO | Clerk) Mains Exams

Chapter

4 Cloze Test (Part – I)


STUDY TIPS

In solving a cloze test the first thing an aspirant must do is to get the
gist of the passage. Understanding the passage along with the tone of the
passage helps eliminate most of the options contextually. Look for the
word before blank because it mostly defines the word that will fill the
blank. Therefore, the understanding of concepts like verbs, adverbs,
nouns etc. and the usage of words accordingly is very necessary.

Practice Exercise Based on new Pattern

Direction (1-5): Given below is a paragraph consisting of blanks. Identify


the correct option among the five alternative pairs that should fill the blank
against the number to make the paragraph contextually meaningful and
grammatically correct.
(This pattern came in IBPS PO MAINS 2017)

The proposal to cut more than 16,500 trees in different parts of Delhi to
make way for government complexes represents both ___(1)_____ and
hypocrisy. It is shocking that felling of such a large number of trees – many
of which are decades old – was even conceived in a place like Delhi which
over recent years has become a _______(2)______gas chamber. Delhiites have
to suffer poor air quality throughout the year with the intensity of air
pollution increasing during the winter months. In this scenario, cutting
down trees is absolutely _____(3)_______-.
These trees are the lungs of Delhi and getting rid of them is a crime against
every resident of the city. While government might need space for its
functioning and employees, solutions can be found without cutting trees
through smart building design. Every tree is precious here and the
government’s ____(4)_______-that it will plant 10 new saplings for every tree

2 Adda247 Publications For any detail, mail us at


Publications@adda247.com
Cracker Book for Bank (IBPS | SBI | RRB PO | Clerk) Mains Exams

felled simply doesn’t cut ice. Do the authorities even have an idea how long
it takes for a sapling to grow into a full tree that can generate large volumes
of oxygen? This is nothing but utter ____(5)______ for the people of Delhi.

1. (a) Callousness/Insensitiveness
(b) Apathy/Indifference
(c) Kindness/Benevolence
(d) Magnanimity/Cowardice
(e) annihilation/Negligence

2. (a) veritable/indubitable
(b) large/humongous
(c) Petite/ Diminutive
(d) poisonous/venomous
(e) huge/colossal

3. (a) Unconscionable/Unethical
(b) Incumbent/Compelling
(c) Indispensable/Obligatory
(d) Irremissible/Profligate
(e) Requisite/Libertine

4. (a) assertion/pronouncement
(b) assumption/negation
(c) Idea/Interpretation
(d) take/denial
(e) suggestion/move

5. (a) Disregard/Contempt
(b) Reverence/Disdain
(c) Approbation/Insult
(d) Tribute/Scorn
(e) Deference/Deterrence

3 Adda247 Publications For any detail, mail us at


Publications@adda247.com
Cracker Book for Bank (IBPS | SBI | RRB PO | Clerk) Mains Exams

Directions (6-15): In the passage given below there are blanks which are
to be filled with the options given below. Find out the appropriate pair of
words in each case which can most suitably complete the sentence without
altering the meaning of the statement. Both the words of the pair must fill
the blank.

Our _____(6)_______ search for the Holy Grail to fix the challenges of
governance always leads us nowhere because the thing doesn’t exist. But
what we find in the process is a ___(7)____, of… well, nothing; it looks like a
solution but it is in fact a problem. Good intentions, unless _____(8)_____ by
thoughtful deliberation and preparation, do not lead to good policy
outcomes.
The move by the Department of Personnel and Training (DoPT) towards
lateral entry in government service falls in this category. It has invited
applications from “talented and motivated Indian nationals ____(9)_____ to
contribute towards nation building” to be appointed as joint secretaries in
10 Departments/ Ministries at the Centre. One cannot question the good
intentions behind the decision to make lateral entry more institutionalised
than the case till now. Nor should one read too much bad faith into this.
Once we unwrap the new policy, however, what we find is a little
_____(10)_______ that can one day grow into a monster. Since the problem
that the new policy seeks to fix remains _______(11)_______, we cannot hope
for whatever improvements promised. It is also a distant cousin to the
‘committed bureaucracy’ bogey of the 1970s. Moreover, the lateral entry
policy goes ___(12)___ in spirit to the governance philosophy ___(13)___ by
the Constituent Assembly, insofar as it concerns the candidates from
private sector, consultancy firms, international/ multinational
organisations (MNCs).
Traditionally, the services of outside experts were ___(14)___ through
consultative processes, a practice quite widespread with the ____(15)_____
Planning Commission and to some extent with its new avatar, the NITI
Aayog. It is not clear why the government determined that the practice was
not effective.

4 Adda247 Publications For any detail, mail us at


Publications@adda247.com
Cracker Book for Bank (IBPS | SBI | RRB PO | Clerk) Mains Exams

6. (a) impulsive/cursory
(b) ceaseless/ unabated
(c) incessant/ careless
(d) assiduous/ offhand
(e) thorough /hasty

7. (a) counterfeit/ sham


(b) panacea/ retrospection
(c) fraudulent/ virtue
(d) remedy/ liability
(e) meander/ bewildered

8. (a) tempered/ modulated


(b) deliberated/ mired
(c) effectuated/ moderated
(d) alleviated/ subsided
(e) restored/ corroborated

9. (a) pleased/bequeathing
(b) rearing/ gathering
(c) obliging/ grudging
(d) willing/eager
(e) planning/gearing

10. (a) Contradiction/accord


(b) Inconsistency/conformity
(c) Remedy/ variance
(d) Improbability / anomalies
(e) Discrepancy/incongruence

11. (a) indistinct/ manifest


(b) obscure/ palpable
(c) precise/ambiguous
(d) vague/indeterminate
(e) anonymous/transparency

5 Adda247 Publications For any detail, mail us at


Publications@adda247.com
Cracker Book for Bank (IBPS | SBI | RRB PO | Clerk) Mains Exams

12. (a) contrary/ withstanding


(b) reliably/ceaseless
(c) eternal/ pervasive
(d) in tandem / alongside
(e) conversely/ counter

13. (a) affirmed/ belied


(b) proclaim/ pronouncing
(c) enunciated/ articulated
(d) availed / unwrapped
(e) disclosed/ attained

14. (a) commissioned/measured


(b) advantaged/ assisted
(c) accounted/ contracted
(d) availed/utilized
(e) appropriated/ aided

15. (a) sagacious/ shrewdly


(b) erstwhile/ former
(c) Previous/ benign
(d) nonchalant/ prudent
(e) Meticulous/ aggaravating

Directions (16-20): In the following passage, certain parts of the


paragraph are highlighted which suggest that the given expression may be
grammatically incorrect. Choose the most appropriate expression among
the four options given against each number which makes the sentence
grammatically correct and contextually meaningful. If the given expression
does not require any correction, choose option (e) i.e., “No correction
required” as your answer.
It was anticipated that the University Grants Commission (UGC) Act would
be replaced by an avant garde legislation (16) degrading the eminence of
the university for the present and future problems of higher education in
the country. The UGC Act had traversed a journey of over 60 years and
6 Adda247 Publications For any detail, mail us at
Publications@adda247.com
Cracker Book for Bank (IBPS | SBI | RRB PO | Clerk) Mains Exams

gathered moss, baggage as well as wisdom on the way, which (17)


criticizes the quality of higher education. The Ministry of Human
Resource Development (MHRD) has now put out the draft Higher Education
Commission of India (Repeal of University Grants Commission) Act, 2018.
What are its implications for higher education in the country? (18) Is it
juvenile and inticate, marking a departure from the past, or more of the
same? The all-powerful regulator syndrome persists, creating a supra
university which inhibits autonomy and innovation in the system. The
default status of a good university is to be autonomous. The spirit of Clause
15 (3) of the draft Act is no different to Section 12 of the UGC Act where
wide powers are given to the Commission. Research, learning outcomes and
academic performance are already (19) within the purview of
universities. The standards laid down by regulators invariably cater to the
institutions which are at the bottom of the pyramid but are applied to all
universities, including the best, inhibiting excellence. That is the reason why
the IITs, IIMs and other good institutions (20) are willing to be regulated
by UGC.

16. (a) competent of elevating


(b) capable of comprehensively handling
(c) for improving the quality of syllabus
(d) handling the unprecedented difficulties
(e) no correction required

17. (a) required urgent sifting and reinvention


(b) is absolute and competent
(c) free the good colleges from the stranglehold of the universities
(d) could change the university system
(e) no correction required

18. (a) Is it uncooperative and bleak


(b) Is it promising and gloomy
(c) Is it complicated and opposed
(d) Is it positive and visionary
(e) no correction required

7 Adda247 Publications For any detail, mail us at


Publications@adda247.com
Cracker Book for Bank (IBPS | SBI | RRB PO | Clerk) Mains Exams

19. (a) with the umbrella of universities


(b) for the scope of universities
(c) in the horizon to universities
(d) under the notion for universities
(e) no correction required

20. (a) protest the revoking of UGC.


(b) pleased to be controlled by UGC
(c) recoil at the thought of UGC control
(d) will get rid of the bad name it had acquired over the years.
(e) no correction required

Directions (21-25): In the passage given below there are blanks which are
to be filled with the options given below. Find out the appropriate pair of
words in each case which can most suitably complete the sentence without
altering the meaning of the statement. Both the words of the pair must fill
the blank.
This month South Africa will have a rare ______________ (21) to break away
from the difficult political legacy of corruption and governance failures that
have __________ (22) its post-apartheid years. At a conference during
December 16-20, the African National Congress will pick its candidate for
party leader, and that person will go on to _________ (23) 10-year incumbent
President Jacob Zuma. Whoever Mr. Zuma hands over the reins of _________
(24) to will be the favourite to become the country’s next President in 2019.
But he or she will also inherit a troubled political system and a nervous,
________ (25) economy.

21. (a) chance/ opportunity


(b) recourse/ possibility
(c) discretion/ caution
(d) preference/ predilection
(e) redundancy/ discharge

8 Adda247 Publications For any detail, mail us at


Publications@adda247.com
Cracker Book for Bank (IBPS | SBI | RRB PO | Clerk) Mains Exams

22. (a) praised/ commended


(b) accursed/ detested
(c) blighted/ ruined
(d) devoured/ improved
(e) desolated/ ravaged

23. (a) dispense/ allocate


(b) temper/ drift
(c) replace/ succeed
(d) resolve/ reconcile
(e) commute/ curtail

24. (a) bustle/ scuttle


(b) process/ approach
(c) eminence/ prestige
(d) haste/ rapid
(e) power/ control

25. (a) fuddled/ perplexed


(b) confused/ demented
(c) distinct/ definite
(d) gloomy/ troubling
(e) oppressive/ harsh

Directions (26-30): In the following paragraph there are blanks, each of


which has been numbered. These numbers are printed below the paragraph
followed by four words/phrases. One or more of those words/phrases may
fit the blank appropriately. Choose the best set of alternatives among the
five given below each question which fill the blank most appropriately. If all
the words/phrases fill the blank, choose option (e) i.e. “All of these” as your
answer.
The Indian law books are (26) _________________ with all sorts of laws that
seem good on paper while being completely ineffective. India would be a
(27) ______________of social justice if all these impressive laws actually
addressed the problems they seek to deal with. Some hasty laws even have
(28) ________________ consequences. The law against parliamentary defections
9 Adda247 Publications For any detail, mail us at
Publications@adda247.com
Cracker Book for Bank (IBPS | SBI | RRB PO | Clerk) Mains Exams

is a good case in point. It was passed at a time when the problem of


lawmakers crossing the (29) ______________________ through defections was a
hot button issue among the (30) ________________. The Rajiv Gandhi
government won brownie points for introducing the law. It has not ended
political wheeling-dealing—but it has killed democracy within parties.

26. (i) bulging (ii) growing


(iii) reckoning (iv) swelling
(a) only (i)
(b) only (iii)
(c) Both (i) and(iv)
(d) Only (i), (ii) and (iv)
(e) All of these

27. (i) torment (ii) paradise


(iii) utopia (iv) purgatory
(a) Both (ii) and (iii)
(b) only (ii)
(c) only (iv)
(d) Only (i), (iii) and (iv)
(e) All of these

28. (i) benign (ii) congenial


(iii) malign (iv) obliging
(a) Both (iii) and (iv)
(b) only (iii)
(c) only (iv)
(d) Only (i), (ii) and (iv)
(e) All of these

29. (i) aisle (ii) passage


(iii) avenue (iv) couloir
(a) Both (iii) and (iv)
(b) only (ii)
(c) only (i)
(d) Only (i), (ii) and (iii)
(e) All of these
10 Adda247 Publications For any detail, mail us at
Publications@adda247.com
Cracker Book for Bank (IBPS | SBI | RRB PO | Clerk) Mains Exams

30. (i) rabble (ii) aristocratic


(iii) proletarian (iv) elite
(a) Both (ii) and (iv)
(b) only (iii)
(c) only (i)
(d) Only (i), (ii) and (iii)
(e) All of these

Solutions

1. (a); Callousness is a noun which means insensibility and


insensitiveness towards a thing, here it is used to show the attitude
of government towards environment . Here the suggestion by
government to cut down so many trees to build government
complexes is being criticized. Also, we can get a cue from the word
hypocrisy that the blank preceding it should take a noun that
should denote a negative trait of the government. Insensitiveness is
another noun that can be used in place of callousness to denote the
indifference of government towards the environment. All other
choices fail to fit grammatically and contextually in the sentence.
Hence, option (a) is the most viable choice.

2. (a); Veritable is an adjective which is used as an intensifier or to


denote something real or actual. Here, it is used to qualify the gas
chamber (noun) to denote the state of Delhi that has emerged as a
gas chamber in real sense due to the presence of smog and other
pollutants. Indubitable is an adjective which means too obvious to
be doubted or unquestionable. Both Veritable and Indubitable can
be used interchangeably to make the sentence contextually
meaningful and grammatically correct. All other words fail to fit
contextually and grammatically in the sentence. Hence, option (a)
is the most viable choice.

11 Adda247 Publications For any detail, mail us at


Publications@adda247.com
Cracker Book for Bank (IBPS | SBI | RRB PO | Clerk) Mains Exams

3. (a); Unconscionable is an adjective which means greatly exceeding


bounds of reason or moderation or lacking a conscience. Here in
the context it is used to criticize the step of cutting down the trees
considering the prevailing condition of Delhi when it is struggling
with pollution and trees needs to be planted to contain the effects
of pollution but instead of taking curative steps the trees are being
felled in order to create space for accommodation. Unethical is the
synonym for Unconscionable and can be used interchangeably to
make the sentence grammatically and contextually meaningful.
However, all other alternatives fail to fit contextually in the
sentence.
Incumbent means obligatory or binding,
Profligate and libertine mean immoral.
Requisite means something that is essential or important.
Hence, option (a) is the most viable choice.

4. (a); Assertion is a noun which is used for declaration or positive


statement and Pronouncement is a noun which means a
declaration or a statement and hence both assertion and
pronouncement can be used interchangeably to make the sentence
grammatically and contextually meaningful. All other choices fail to
fit contextually and grammatically in the sentence. Here in the
context it is used to state the government’s declaration or
statement that for every tree felled 10 new saplings will be planted.
All other choices fail to fit contextually and grammatically in the
sentence.
Assumption means hypothesis or supposition.
Negation means a negative statement or a statement that is a
refusal or denial of some other statement
Interpretation means make sense of something.
Denial means the act of refusing to comply.
Move means the act of deciding to do something.

12 Adda247 Publications For any detail, mail us at


Publications@adda247.com
Cracker Book for Bank (IBPS | SBI | RRB PO | Clerk) Mains Exams

5. (a); Disregard and Contempt are the words that fit grammatically and
contextually in the sentence. Disregard means showing disrespect
and contempt means the disdain or disrespect. Both these words
can be used interchangeably to make the sentence grammatically
as well as contextually fit. All other choices fail to fit in the context
of the sentence.
Reverence means deep respect.
Disdain means derision or disrespect.
Scorn means to show contempt
Deference means respect.
Approbation means approval or praise.
Deterrence means the action of discouraging an action or event
through instilling doubt or fear of the consequences.
Hence, option(a) is the most viable choice.

6. (b); Option (b) is the correct choice for the given question.
We need an adjective because the blank is given before the word
search. Ceaseless and unabated both are adjective and have
similar meaning. The need for these words is attributed to the
presence of “Holy grail” in the sentence which means something
that is extremely difficult to find or get for which we need a
“thorough and continuous search”.
Ceaseless means constant and unending.
Unabated means without any reduction in intensity or strength.

7. (a); Option (a) is the correct choice for the given question.
Counterfeit means a fraudulent imitation of something.
Counterfeit and sham are similar in meaning and both these words
fit here appropriately which can be suggested because of the
meaning of the line afterwards which conveys that all we really get
for our ceaseless search is a problem and not a solution.
Other options are irrelevant to the context of the passage.

13 Adda247 Publications For any detail, mail us at


Publications@adda247.com
Cracker Book for Bank (IBPS | SBI | RRB PO | Clerk) Mains Exams

8. (a); Tempered and modulated fits in the blank most appropriately.


Tempered here means modified by addition of or mixture with
other qualities, ingredients, etc.
Modulated means exert a modifying or controlling influence on.
They both can be fit here and make a sense that good intentions
without the addition or controlling influence of thoughtful
deliberation and preparation do not give a desired result.

9. (d); Option (d) is the correct choice as both the words fit in the blank
appropriately.
Willing means ready, eager, or prepared to do something.
Eager means strongly wanting to do or have something.
Obliging means willing to do a service or kindness; helpful.

10. (e); Option (e) is the correct choice.


Incongruence means not suitable or not fitting well with
something else:
Discrepancy means an illogical or surprising lack of compatibility
or similarity between two or more facts.
These two words are trying to suggest that the policy is not perfect
and there are certain issues which needs to be addressed otherwise
they may create some bigger problems.
Accord means give or grant someone (power, status, or
recognition).
Anomalies means something that deviates from what is standard,
normal, or expected.

11. (d); Vague and indeterminate are the correct choice for the given blank.
Vague means not exactly known, established, or defined.
Indeterminate means not exactly known, established, or defined.
Manifest means clear or obvious to the eye or mind.
Palpable means (of a feeling or atmosphere) so intense as to seem
almost tangible.
14 Adda247 Publications For any detail, mail us at
Publications@adda247.com
Cracker Book for Bank (IBPS | SBI | RRB PO | Clerk) Mains Exams

12. (e); Conversely and counter are the correct choice for the given
question.
Conversely means introducing a statement or idea which reverses
one that has just been made or referred to.
Counter means in the opposite direction or in opposition to.
The usage of these words is justified because author is presenting
an argument on how the lateral entry does not work in tandem
with the constituent assembly’s philosophy and the reason is also
given in the later lines.

13. (c); Option (c) is the correct choice for the given question.
Enunciated means express (a proposition, theory, etc.) in clear or
definite terms.
Articulated means express (an idea or feeling) fluently and
coherently.
Availed means use or take advantage of (an opportunity or
available resource).
Proclaim means announce officially or publicly.
Belied means (of an appearance) fail to give a true impression of
(something).

14. (d); Option (d) is the correct choice.


Availed means use or take advantage of (an opportunity or
available resource).
Availed and utilized are correct words as they conveyed that the
services were used through the given process.

15. (b); Option (b) is the correct choice because we are talking a traditional
practice of the past and therefore former and erstwhile (former)
fits the blank appropriately as they covey the appropriate meaning
to the sentence.
Nonchalant means (of a person or manner) feeling or appearing
casually calm and relaxed; not displaying anxiety, interest, or
enthusiasm.
Prudent means acting with or showing care and thought for the
future.
Sagacious means having or showing keen mental discernment and
good judgement; wise or shrewd.
15 Adda247 Publications For any detail, mail us at
Publications@adda247.com
Cracker Book for Bank (IBPS | SBI | RRB PO | Clerk) Mains Exams

16. (b); The paragraph is describing about the proposition to replace the
University Grants Commission (UGC) Act, with a more suitable act
that should be capable of resolving all the difficulties of higher
education in the country. Therefore, the most suitable phrase that
should replace the incorrect phrase is “capable of comprehensively
handling”, since it is contextually meaningful and fits in the
grammatical syntax of the sentence. The hint for the correct phrase
can also be drawn from the latter part of the sentence […the
present and future problems]. Moreover, as the paragraph has
mentioned about the replacement of the act, it becomes very logical
that a more capable legislation will be implemented that will have
the stature to solve the problems of higher education. Since, all the
other options are either grammatically incorrect or contextually
meaningless, option (b) becomes the most suitable answer choice.

17. (a); The paragraph is describing about the anticipation of the


replacement of UGC Act for better quality of higher education
system. The given phrase is incorrect since the sentence has
mentioned the positive qualities of the Act, which makes the phrase
contextually incorrect in reference to the sentence. Furthermore, it
is to be noted that since in the previous sentence it has been
mentioned about the replacement of the Act, while in the next
sentence is has been mentioned about the achieved qualities of the
education system through UGC Act. Thus, the most appropriate
phrase to replace the given incorrect phrase is “required urgent
sifting and reinvention” which means examining and reinventing
thoroughly so as to isolate the important qualities of the Act.
However, all the other options are either grammatically incorrect
or contextually irrelevant. Option (b) may also sounds correct but
observing that the paragraph has mentioned about the
replacement of the Act, it cannot be complete and competent. Thus,
the rationale makes the phrase of option (b) incorrect. Hence,
option (a) is the most viable answer choice.
16 Adda247 Publications For any detail, mail us at
Publications@adda247.com
Cracker Book for Bank (IBPS | SBI | RRB PO | Clerk) Mains Exams

18. (d); The most appropriate phrase to replace the incorrect highlighted
phrase is “Is it positive and visionary”. It is to be noted that along
with this sentence, previous sentence is also questioning about the
fidelity of the revocation of the UGC Act. Thus, with reference to the
theme of the paragraph, “IS it positive and visionary” fits in the
most precise manner. Moreover, the latter part of the sentence also
drops a clue, as it has described about the moving of the education
system from the past [problematic system]. Therefore, the initial
part of the sentence, must illustrate the positive outcome of the
repeal. Hence, option (d) is the most suitable answer choice.

19. (e); The given highlighted phrase is grammatically correct and


contextually meaningful and does not require any replacement. All
the options provided to replace the phrase contain grammatical
errors and fail to fit in the sentence. Hence, option (e) becomes the
most viable answer choice.

20. (c); The phrase highlighted “are willing to be regulated by UGC” should
be replaced with “recoil at the thought of UGC control” to make the
sentence of the paragraph contextually relevant with the theme of
the paragraph. It is to be noted that in the previous sentence, it has
been mentioned that how the standards laid by the government
will be barriers for the good institutions to perform excellently.
Therefore, this stands as the reason for the good institutions to
hesitate in fear to fall under the control of UGC. All the other
options are grammatically incorrect or contextually meaningless.
Hence, option (c) is the most viable answer choice.

21. (a); Option (a) is the correct choice.

22. (c); ‘blighted, ruined’ best suits the purpose.


Blighted means spoil or harm.
23. (c); ‘replace/ succeed’ is the correct choice.

24. (e); ‘power/ control’ is the correct choice.

25. (d); Option (d) is the correct choice.


17 Adda247 Publications For any detail, mail us at
Publications@adda247.com
Cracker Book for Bank (IBPS | SBI | RRB PO | Clerk) Mains Exams

26. (c); The paragraph is describing about the various laws introduced for
effective jurisdiction, however, sometimes due to poor
implementation they lead to more chaos. Thus, to adhere the theme
of the paragraph the most suitable word to fill the blank is
“bulging” or “swelling”. “bulging” and “swelling” are verbs
describing the state of law books which means ‘be full of and
distended with’. However, other words fail to provide the
contextual meaning to the paragraph, therefore, option (c) is the
most suitable answer choice.

27. (a); The paragraph has mentioned about a hypothetical situation where
all the laws as formulated could resolve the issues of the society.
Such a perfect situation may give an impression of a paradise for
the country. Therefore, the most suitable word to fill the blank is
either “paradise” or “utopia”. ‘Utopia’ means an imagined place or
state of things in which everything is perfect. Hence, option (a) is
the most suitable answer choice.
Torment means severe physical or mental suffering.
Purgatory means mental anguish or suffering.

28. (b); The paragraph is describing about the various laws introduced for
effective jurisdiction, however, sometimes due to poor
implementation they lead to more chaos. Here, the sentence has
mentioned about the hasty formulation of few laws, since they are
hasty the consequences can’t have affirmative outcomes. Therefore,
the most appropriate word to fill the blank should be “malign”.
‘Malign’ is an adjective which means ‘evil in nature or effect’. All the
other words, fail to comply with the theme of the paragraph. Hence,
option (b) is the most accurate answer choice.

29. (c); The most appropriate word to fill the blank is “aisle”. “Cross the
aisle” is an idiom which means ‘To vote, unite, or otherwise co-
operate with members of another political party in order to achieve
governmental or political action’. All the other words do not
provide the suitable context to the paragraph. Therefore, option (c)
is the most precise answer choice.
18 Adda247 Publications For any detail, mail us at
Publications@adda247.com
Cracker Book for Bank (IBPS | SBI | RRB PO | Clerk) Mains Exams

30. (a); The most appropriate word that will fill the blank adhering to the
theme of the paragraph is either “elite” or “aristocratic” since the
paragraph is mentioning about the lawmakers and politicians who
are the exclusive class of the society. “Aristocratic” means ‘a group
regarded as privileged or superior in a particular sphere’. Hence,
option (a) is the most suitable answer choice.
Rabble means ordinary people, especially when regarded as
socially inferior or uncouth.
Proletarian means relating to the proletariat.

19 Adda247 Publications For any detail, mail us at


Publications@adda247.com
Cracker Book for Bank (IBPS | SBI | RRB PO | Clerk) Mains Exams

1 Adda247 Publications For any detail, mail us at


Publications@adda247.com
Cracker Book for Bank (IBPS | SBI | RRB PO | Clerk) Mains Exams

Chapter

5 Cloze Test (Part – II)


STUDY TIPS

In solving a cloze test the first thing an aspirant must do is to get the
gist of the passage. Understanding the passage along with the tone of the
passage helps eliminate most of the options contextually. Look for the
word before blank because it mostly defines the word that will fill the
blank. Therefore, the understanding of concepts like verbs, adverbs,
nouns etc. and the usage of words accordingly is very necessary.

Practice Exercise Based on new Pattern

Directions (1-5): In the following passage, certain parts of the paragraph


are omitted. Choose the most appropriate expression among the five
options given against each number which makes the sentence of the
paragraph grammatically correct and contextually meaningful.

A Parliamentary panel has recommended the earmarking of a defined


portion of proceeds from the divestment of State-owned enterprises for
__________ (1) ___________ proposals of sick public-sector undertakings (PSUs)
that have the potential to turn around.
“In this manner, the government can extend a hand-holding support to the
select ________(2)___________in [the] future,” the panel said in a report.
The government had set a target of raising 80,000 crores in 2018-19 by
selling stakes in the State-owned firms, with strategic divestment of 24
CPSEs (central public-sector undertakings) on the cards and privatisation of
Air India on track.
Besides, NITI Aayog is preparing another list of sick PSUs that can be
privatised, its chief executive officer Amitabh Kant said last month. The
Prime Minister’s Office (PMO) had asked the think-tank to look into the
_____________(3)______________.

2 Adda247 Publications For any detail, mail us at


Publications@adda247.com
Cracker Book for Bank (IBPS | SBI | RRB PO | Clerk) Mains Exams

The Aayog had already recommended strategic divestment of 40 sick


public-sector undertakings.
In its report, the Parliamentary Standing Committee on Industry said it was
of the firm opinion that while making a decision to disinvest PSUs,
especially those that are profit-making, ____________(4)______________ to the
jobs supported by them, the track record of their contribution to the
national economy, their capex (capital expenditure) creation potential and
also their role in balancing the social/regional fabric.
The committee observed that timely approval of revival plans of CPSEs with
accurate cost estimates, availability of funds with the government and the
_______________ (5)_______________ are crucial factors.

1. (a) renovation, decoration and trimming


(b) finalization, closer and annihilation,
(c) restructuring, renovation and decoration
(d) optimization, management and convention-alisation
(e) funding revival, restructuring and moder-nisation

2. (a) sick PSUs that have the potential to sell their assets, recover losses
and then close their operations
(b) sick PSUs that can never turn around
(c) sick PSUs that have the potential to turn around and sustain
themselves
(d) sick PSUs that have the potential to cease to exist
(e) sick PSUs that cannot face competition from their private sector
counterparts

3. (a) viability of sick State-run companies


(b) lay off criteria in these PSUs
(c) selection process in these sick State-run companies
(d) market capitalisation of these PSUs
(e) joblessness as it is India's most underestimated yet lethal problem

3 Adda247 Publications For any detail, mail us at


Publications@adda247.com
Cracker Book for Bank (IBPS | SBI | RRB PO | Clerk) Mains Exams

4. (a) the government must not accord due consideration


(b) the government must keep in mind the requirement of Basel III
norms which will be in place in 2019
(c) the government should focus on the needs of those who provide
jobs
(d) the government must accord due consideration
(e) the government should listen carefully the proposals of Bank
Boards Bureao.

5. (a) delayed disposal of such funds


(b) timely registration of these PSUs in Share market
(c) timely completion of projects by these PSUs
(d) merger of these CPSEs
(e) timely disposal of such funds

Directions (6-10): In the following passage, certain parts of the paragraph


are highlighted which suggest that the given expression may be
grammatically or contextually incorrect. Choose the most appropriate
expression among the four options given against each number which makes
the sentence grammatically correct and contextually meaningful. If the
given expression does not require any correction, choose option (e) i.e., “No
replacement required” as your answer.

India is the largest user of groundwater in the world. Every year, Indian
farmers pump 250 cubic-km of groundwater— (6) more to US and China
together—to irrigate nearly 63 million hectares (mha) of land. India also
has the largest number of irrigation wells in the world. The 5th Minor
Irrigation Census (MI Census) counted 20.52 million wells and tube wells in
2013-14—one for every seven operational holdings. (7) The peeking for
groundwater boom during 1970s to 1990s, Indian farmers were digging
nearly half a million wells every year, mostly dug wells and shallow tube
wells (STWs). The pace has now slowed down to 0.1 million new wells per
year, but almost all of them are deep tube wells, or DTWs (more than 70
metres). ‘

4 Adda247 Publications For any detail, mail us at


Publications@adda247.com
Cracker Book for Bank (IBPS | SBI | RRB PO | Clerk) Mains Exams

Thirty years ago, there were only 0.1 million DTWs in India; today, there
are more than 2.6 million. Since the 4th MI Census in 2006-07, 0.4 million
dug wells have gone into disuse and the number of STWs has barely
changed, but (8) the number of DTWs has nearly doubled. Rather than
digging new dug wells and STWs, (9) farmer are deepening the newer
ones to chase the receding water table. Even in areas where the water table
is not very deep, (10) it is switching from centrifugal to submersible
pumps. The 5th MI Census recorded more submersible pumps than
centrifugal pumps.

6. (a) lowered as compared to US and China combined


(b) more than the US and China combined
(c) including US and China combined
(d) however higher than the US and China combined
(e) no replacement required

7. (a) Peeking in the groundwater


(b) The highest level of groundwater
(c) The groundwater declines
(d) At the peak of groundwater boom
(e) no replacement required

8. (a) the number of STWs and DTWs have crossed


(b) the number of STWs have declined
(c) though the number of STWs has increased even more
(d) DTWs have grown more in number than the dug wells
(e) no replacement required

9. (a) farmers are deepening the existing ones


(b) farmers are constructing new ones
(c) farmers are covering the existing ones
(d) farmers are replenishing the water levels
(e) no replacement required

5 Adda247 Publications For any detail, mail us at


Publications@adda247.com
Cracker Book for Bank (IBPS | SBI | RRB PO | Clerk) Mains Exams

10. (a) they will choose submersible over centrifugal pumps


(b) it had chosen centrifugal to submersible pumps
(c) they are switching from centrifugal to submersible pumps
(d) it is preferring centrifugal to submersible pumps
(e) no replacement required

Directions (11-15): In the following passage against each number four


words are suggested in bold which may or may not fit into the sentence
contextually. These numbers are printed below the passage and against
each, five options are given. Find out the most appropriate alternative
reflecting the word which doesn’t fit into the blank appropriately and thus
fail to give a contextual meaning to the paragraph. If no such error is there
mark (e) i.e. “all are correct” as your answer choice.
(This pattern came in IPPB mains paper)

India’s shipping industry is at a (11) crossroads. The country is aiming to


make big strides in the maritime sector and investing billions of dollars to
modernise and set up new ports as well as related infrastructure.
But its domestic shipping industry is finding it tough to compete with
foreign shipping lines in carrying India specific export-import trade due to
cost (12) disadvantages and an uneven playing field. Indian shipowners
have asked the Centre to consider having a national fleet.
This is in line with other major maritime powers supporting their own
shipping firms, for adjusting control and securing the transportation of
critical cargo.
A national fleet policy (13) initiates that ships engaged in trade must be
flagged, or registered, in India irrespective of whether they are owned by
Indian or foreign shipping lines. Though India has allowed 100% FDI in
shipping since 2001, foreign lines are yet to flag in India.
Insisting that India must have a national fleet, Anil Devli, CEO, Indian
National Shipowner’s Association (INSA), asked, “If foreign shipping lines
control over 90% of India’s cargo, why should they not be asked to flag
some of their vessels in India and pay taxes like us.”
Foreign lines register their vessels in favourable tax regimes such as
Panama and at their local (14) jurisdictions.

6 Adda247 Publications For any detail, mail us at


Publications@adda247.com
Cracker Book for Bank (IBPS | SBI | RRB PO | Clerk) Mains Exams

Japan, China, the U.S., Malaysia, Indonesia and European nations practice
absolute cabotage to protect their shipping lines. The EU practices
cabotage even in ship recycling. China ensures that 600 million tonnes of
coastal cargo is carried by Chinese vessels only. Cabotage means to a legal
restriction that limits the transportation of goods and people within the
country by that country’s own transport services.
Japan ensures that all its imports are carried on by vessels owned, built and
financed by (15) entities registered in the country. The Donald Trump
system in the U.S. has proposed at least 30% of gas exports should be
executed by their national carriers.

11. (a) crossroads (b) strides (c) modernize


(d) related (e) all are correct

12. (a) disadvantages (b) uneven (c) consider


(d) adjusting (e) all are correct

13. (a) initiates (b) irrespective (c) yet


(d) Insisting (e) all are correct

14. (a) jurisdictions (b) cabotage (c) ensures


(d) means (e) all are correct

15. (a) entities (b) system (c) proposed


(d) executed (e) all are correct

Directions (16-20): In the following paragraph, there is a set of four


highlighted words against each number indicated in bold in the beginning
of the sentences. One of the given words in each set may or may not fit into
the statement. Choose the word which is not suitable in the context of the
paragraph. If all the four words are correct and feasible, choose (e) i.e. “No
error” as your answer.
(16) The Russian doping scandal continues by cast a long shadow over
international sport as the 2018 Winter Olympics begin in PyeongChang,
South Korea, on February 9. (17) In December, the International Olympic

7 Adda247 Publications For any detail, mail us at


Publications@adda247.com
Cracker Book for Bank (IBPS | SBI | RRB PO | Clerk) Mains Exams

Committee banned Russia from winning in the Games following


investigation into an alleged state-sponsored doping programme at the
2014 Winter Olympics in Sochi, Russia. (18) The dissension to ban Russia
came after the IOC’s Disciplinary Commission, headed by former president
of the Swiss Confederation Samuel Schmid, confirmed “systemic
manipulation of the anti-doping rules and system in Russia”. (19) The IOC
had stated, along with, that clean Russian athletes would be allowed to
compete as neutrals and last month invited 169 of them — each to be
known as Olympic Athlete from Russia (OAR) — to participate in the
PyeongChang Games. (20) The announcement did not go down well
outside Russia, even though the IOC declared that “more than 80%” of
those athletes had not competed in Sochi and had been carefully vetted.

16. (a) scandal (b) by (c) shadow


(d) begin (e) no error

17. (a) banned (b) wining (c) investigation


(d) doping (e) no error

18. (a) dissension (b) headed (c) confirmed


(d) manipulation (e) no error

19. (a) along with (b) neutrals (c) known


(d) participate (e) no error

20. (a) announcement (b) outside (c) athletes


(d) vetted (e) no error

Directions (21- 25): In the following passage there are blanks, each of
which has been numbered. These numbers are printed below the passage
and against each, five words are suggested, one of which fits the blank
appropriately. Find out the appropriate word in each case.
The ...(21)... of India as an economic superpower is not reflected in the
...(22)... of life enjoyed by its 1.2 billion citizens according to the Human
Development Index which …(23)... India very low among 182 countries.

8 Adda247 Publications For any detail, mail us at


Publications@adda247.com
Cracker Book for Bank (IBPS | SBI | RRB PO | Clerk) Mains Exams

In our performance oriented world, measurement issues have taken on


...(24)... importance as what we measure affects what we do. In fact, the
French President has established an international commission on the
Measurement of Economic Performance and Social Progress owing to his
...(25)... and that of others with the current state of statistical information
about the economy and society.

21. (a) pursuit (b) perception (c) conversion


(d) title (e) tribute

22. (a) quality (b) spirit (c) span


(d) joy (e) loss

23. (a) scored (b) qualified (c) regard


(d) ranked (e) counted

24. (a) great (b) unduly (c) trivial


(d) considerably (e) negligible

25. (a) confidence (b) belief (c)dissatisfaction


(d) compliance (e) obedience

Directions (26-30): In the following passage there are blanks, each of


which has been numbered. These numbers are printed below the passage
followed by four words/phrases. One or more of those words/phrases may
fit the blank appropriately. Choose the best set of alternatives among the
five given below each question which fill the blanks most appropriately. If
all of the words/phrases fill the blanks, choose option (e) i.e. “All of these”
as your answer.
It is that time of the academic year when most (26)________________ prepare to
begin teaching a new (27) ________________of perceptive, young minds joining
9 Adda247 Publications For any detail, mail us at
Publications@adda247.com
Cracker Book for Bank (IBPS | SBI | RRB PO | Clerk) Mains Exams

university campuses. As an instructor of economics, it is (28) ________________


to welcome students, introduce them to higher possibilities of learning, and
teach foundational concepts using (29) ________________cases based on real-
world scenarios.
Economics in the larger field of social sciences, and over the last century or
so, has (30) ________________a vital space in understanding different aspects of
human behaviour.

26. (A) instructors (B) arbiters


(C) educators (D) conciliators
(a) Only (D) (b) Both (A) and (C) (c) Only (C)
(d) Both (A) and (D) (e) All of these

27. (A) adversary (B) foe


(C) cohort (D) associate
(a) Only (D) (b) Both (A) and (C) (c) Only (C)
(d) Both (A) and (D) (e) All of these

28. (A) exhausting (B) tiring


(C) absorbing (D) fascinating
(a) Both (C) and (D) (b) Both (A) and (C) (c) Only (C)
(d) Both (A) and (D) (e) All of these

29. (A) illustrative (B) interpretative


(C) instructive (D) elucidative
(a) Both (C) and (D) (b) Both (A) and (C) (c) Only (C)
(d) Both (A) and (D) (e) All of these

30. (A) occupied (B) vacated


(C) dropped (D) acquired
(a) Only (D) (b) Both (A) and (C) (c) Only (C)
(d) Both (A) and (D) (e) All of these

10 Adda247 Publications For any detail, mail us at


Publications@adda247.com
Cracker Book for Bank (IBPS | SBI | RRB PO | Clerk) Mains Exams

Solutions

1. (e); Part of the proceeds from the funds received from the
disinvestment of PSUs should be used for funding revival,
restructuring and modernization.
The PSUs which are already sick and need government support,
‘decoration’ or ‘renovation’ of their premises from these critical
funds does not make any sense hence option (a) and option (c)
cannot be the answer.
It is clear from this article that it focuses on restructuring and
revival of PSUs so ‘annihilation or closer’ doesn’t fit in the context
hence option (b) cannot be our answer.
Conventionalisation: the act of conventionalizing (conventional
methods of PSUs might be wrong may be that’s why they are in
such a precarious state so going back to conventional methods is
not a solution therefore option (d) cannot be our answer.

2. (c); The main theme of the passage is about government support


needed for PSUs which have the potential to turn around and
which contribute to the economy hence (c) is the answer choice.

3. (a); NITI Aayog should look into the viability, capability and
sustainability of sick PSUs, to see if they can achieve growth in
future hence option (a) is the answer choice.

4. (d); The government must accord due consideration to the capability


and sustainability of PSUs, their contribution to the economy, jobs
provided by them etc. to see if they can achieve growth in future
hence option (d) is the answer choice.
Option (b) and option (d) cannot be our answer as this passage is
not only about Public-Sector Banks instead all of PSUs.

5. (e); timely disposal of funds is very necessary for the turnaround of


sick PSUs as delay in disposal of such funds can alter their revival
plan negatively.
11 Adda247 Publications For any detail, mail us at
Publications@adda247.com
Cracker Book for Bank (IBPS | SBI | RRB PO | Clerk) Mains Exams

Registration in share market does not guarantee success hence


option (b) cannot be our answer.
Merger of CPSEs does not guarantee success hence option (d)
cannot be our answer.
Timely completion of projects by PSUs does not guarantee besides
there are many different PSUs operating in different sectors
performing different types of work and not all of them have to
necessarily work on a project hence option (c) cannot be our
answer. Therefore, option (e) becomes the most suitable answer
choice.

6. (b); The paragraph is describing about the increase in the usage of


groundwater in India for irrigation. Moreover, it has also
mentioned the statistics of comparison between Dug wells and
shallow tube wells.
The given phrase in bold is grammatically incorrect, as while
comparing the quantity of two objects we use “more than” rather
than “more to”. It is further to be noted that the first sentence of the
paragraph has stated that India is the largest user of groundwater.
Therefore, drawing hints from the first sentence, option (a) and (c)
can be eliminated as they are contextually incorrect. Moreover,
option (d) stands incorrect as the conjunction ‘however’ is used to
join two contrasting statements. However, here both the clauses
support the same context of the sentence. Therefore, to make the
sentence of the paragraph correct, replace ‘more to US and China
together’ by ‘more than the US and China combined’. Hence, option
(b) becomes the most suitable answer choice.

7. (d); The paragraph is describing about the increase in the usage of


groundwater in India for irrigation. Moreover, it has also
mentioned the statistics of comparison between Dug wells and
shallow tube wells. The given phrase in bold of the sentence is
grammatically incorrect and contextually meaningless. It is to be
noted that the verb “peeking” means ‘protrude slightly so as to be
just visible’; while the verb required here is “peaking” which means
‘reach a highest point, either of a specified value or at a specified
12 Adda247 Publications For any detail, mail us at
Publications@adda247.com
Cracker Book for Bank (IBPS | SBI | RRB PO | Clerk) Mains Exams

time’. This also eliminates option (a). Option (b) also stands
incorrect as the sentence is describing a particular phase of time,
Also, it lacks the preposition ‘at’ to make it grammatically correct.
Moreover, from the latter part of the paragraph [The pace has now
slowed down] we can receive a hint that at certain period of past,
the groundwater must be at peak as not it has slowed down, which
makes option (c) incorrect. Therefore, by replacing the phrase “The
peeking for groundwater boom” by “At the peak of groundwater
boom” the sentence of the paragraph becomes grammatically
correct and contextually meaningful. Hence, option (d) becomes
the most viable answer choice.

8. (e); Here, the sentence of the paragraph has made a comparison


between the usage of DTWs and STWs. It has clearly mentioned
that the number of STWs has hardly changed. Taking a clue form
this statement option (a), (b) and (c) can be eliminated. Moreover,
the comparison is made between STWs and DTWs therefore, with
this rationale, option (d) can be eliminated. Hence, the given phrase
in bold is correct and does not require any corrections or
replacements. Thereby, option (e) becomes the most viable answer
choice.

9. (a); The given sentence of the paragraph is describing about the


depleting groundwater levels due to excessive digging of dug wells
and STWs. The given phrase in bold stands incorrect as to comply
with the plural verb “are”, the subject [farmer] should also be in its
plural form [farmers]. In addition to, it has been mentioned that the
farmers are not digging new dug wells, thus the phrase in bold
makes a contextual error since the farmers are not digging new dug
wells so how can they deepen them further.
However, it is to be noted that the usage of the phrase “rather than”
indicates a comparison between two alternatives. ‘Rather than’ is
used to give more importance to one thing when two alternatives
or preferences are being compared. Therefore, with this logic
option (b) and (c) can be eliminated. This can be understood from
the phrase “to chase the receding water table”. Hence, option (d)
13 Adda247 Publications For any detail, mail us at
Publications@adda247.com
Cracker Book for Bank (IBPS | SBI | RRB PO | Clerk) Mains Exams

can also be eliminated. Since option (a) fits best to make the
paragraph comprehensive and grammatically correct it becomes
the most viable answer choice.

10. (c); The given phrase in bold of the sentence contains grammatical
error. To make the phrase correct replace “it is” with “they are”
since the subject of the sentence is “farmers”. Option (b) and (d)
are contextually incorrect as the next sentence has described about
that the census has recorded more submersible pumps than
centrifugal pumps. This implies that farmers are preferring
submersible pumps to centrifugal pumps. Option (a) can be
eliminated as the action of switching from centrifugal to
submersible pumps is an ongoing activity, therefore the sentence
should be in present continuous tense rather than future tense.
Since, option (c) fits best in the given context of the paragraph, it
becomes the most suitable answer choice.

11.(e); all are correct

12.(d); Replace ‘adjusting’ with ‘retaining’


Foreign countries are not supporting their shipping firms for
‘adjusting’ control but they are doing so for ‘retaining’ control. It is
clear from the passage that they want to retain control on their
shipping industry.
Adjusting: adapt or become used to a new situation

13. (a); Replace ‘initiates’ with ‘mandates’


initiates: cause (a process or action) to begin
Initiates means starting something new, which does not fit in the
context of the sentence. National fleet policy is already there so
initiates cannot be used here. The policy mandates or ask shipping
companies to register in India therefore ‘mandates’ or any other
word similar in meaning to mandates will replace ‘initiates’.

14 Adda247 Publications For any detail, mail us at


Publications@adda247.com
Cracker Book for Bank (IBPS | SBI | RRB PO | Clerk) Mains Exams

14. (d); replace ‘means’ with ‘refers’. Use of ‘to’ after ‘means’ is wrong but
‘to’ is used after ‘refers’ therefore ‘means’ will be replaced by
‘refers’.

15. (b); Replace ‘system’ with ‘administration’. Use of system after Donald
Trump’s name is not right as system does not belong to one man
but government and administration can belong to one man.
Example: Donald Trump administration, Narendra Modi
government
In India we say, ‘Narendra Modi government’ while in USA they say,
‘Donald Trump administration’ therefore ‘administration’ will
replace ‘system’.

16. (b); Option (b) is the correct choice. Except for option (b) “by” all the
words completely fit into the paragraph to make it grammatically
and meaningfully correct. It is to be noted that the usage of
preposition “by” is incorrect. An infinitive will almost always begin
with “to” followed by the simple form of the verb, like this: To +
Verb = Infinitive. Therefore, before the base form of the verb “cast”
“to” should be added to make the sentence grammatically correct.

17. (b); Option (b) is the correct choice. All the words except for “winning”
provides a coherent meaning to the sentence. Here, the paragraph
is describing about the controversy of doping by Russian players in
the Olympics. Therefore, the correct word required here is
“competing” or “participating” as the International Olympic
Committee banned the country from competing in the games.
Moreover, in the later part of the paragraph it is mentioned that the
“clean Russian athletes would be allowed to compete as neutrals”.
This statement indicates that previously the athletes were banned
to compete and this restriction was later removed. This rationale
helps to identify option (b) as the error in the given paragraph.

18. (a); Option (a) is the correct choice. “dissension” is a noun which means
disagreement that leads to discord. However, the paragraph is
stating about the verdict that the Russian athletes would no longer
be allowed to compete in 2018 winter Olympics. Therefore, the
correct word required has to “decision” or something similar in
meaning. All the other words help to make the sentence
15 Adda247 Publications For any detail, mail us at
Publications@adda247.com
Cracker Book for Bank (IBPS | SBI | RRB PO | Clerk) Mains Exams

grammatically and meaningfully correct. “manipulation” is a noun


which means the action of manipulating something in a skillful
manner.

19. (a); Option (a) is the correct choice as here, the usage of phrasal
conjunction “along with” is incorrect. “along with” is used to give
the reference of a company with or at the same time as;
accompanying; together with. Thus, it doesn’t signify the meaning
of the sentence correctly. The correct conjunction to be used here is
“however”. “However” can be used to introduce a statement that
contrasts with or seems to contradict something that has been said
previously or in whatever way; regardless of how.
Here it is mentioned to provide the meaning of “regardless of how”.
All the other words are apt in the context of the paragraph.
“neutrals” means an impartial or unbiased state or person. Hence,
the correct option which has the error is option (a).

20. (e); Option (e) is the correct choice. The sentence is contextually and
grammatically correct. All the given words are making the
paragraph logical and comprehensible. “vetted” is a verb which
means to make a careful and critical examination of (something).

21. (b); ‘Perception’ is ‘the way something is viewed as’.

22. (a);

23. (d);

24. (a); Out of these adjectives, great, trivial and negligible, the last two
have negative meanings which is not desirable here. Hence, option
(a).

25. (c); Here, a negative meaning work is required.

26. (b); The paragraph is describing about the methods of teaching


economics to the students so that they can understand different
aspects of human behavior. Therefore, the most suitable word that
16 Adda247 Publications For any detail, mail us at
Publications@adda247.com
Cracker Book for Bank (IBPS | SBI | RRB PO | Clerk) Mains Exams

will fill the blank to make the paragraph coherent is “educators” or


“instructors”. Hence, option (b) is the most suitable answer choice.
Arbiters means a person who settles a dispute or has ultimate
authority in a matter.
Conciliators means a person who acts as a mediator between two
disputing people or groups.

27. (c); The most suitable word that will fill the blank to make the
paragraph coherent is “cohort” which is a noun and it means a
group of people with a shared characteristic. However, all the other
words fail to appropriately fit in the context of the paragraph.
Hence, option (c) is the most feasible answer choice.

28. (a); The most suitable word that will fill the blank to make the
paragraph coherent is “fascinating” or “absorbing”. However, all
the other words fail to appropriately fit in the context of the
paragraph. Hence, option (a) is the most feasible answer choice.
Absorbing means intensely interesting; engrossing.
Tiring means causing one to need rest or sleep; fatiguing.
Exhausting means making one feel very tired; very tiring.

29. (e); All the given words suitably fit in the blank providing appropriate
meaning to the paragraph. Therefore, the most suitable answer
choice is option (e).
Illustrative means serving as an example or explanation.
Interpretative means relating to or providing an interpretation.
Instructive means useful and informative.
Elucidative means to make lucid or clear; throw light upon; explain

30. (d); The most suitable word that will fill the blank to make the
paragraph coherent is “occupied” or “acquired”. However, all the
other words fail to appropriately fit in the context of the paragraph.
Hence, option (d) is the most feasible answer choice.
Acquired means buy or obtain (an asset or object) for oneself.
Dropped means fall vertically.
Vacated means leave (a place that one previously occupied).

17 Adda247 Publications For any detail, mail us at


Publications@adda247.com
Cracker Book for Bank (IBPS | SBI | RRB PO | Clerk) Mains Exams

1 Adda247 Publications For any detail, mail us at


Publications@adda247.com
Cracker Book for Bank (IBPS | SBI | RRB PO | Clerk) Mains Exams

Chapter
Column Based Questions
6
STUDY TIPS

A new format of questions has been introduced in which two coloumns


are provided in which coloumn (A) mentions the words/phrases that may
be incorrect and coloumn (B) mentions the words/phrases that may
replace the incorrect words/phrases. The best way to solve such
questions is by understanding the tone and the grammatical syntax of the
sentence. In this way, it becomes easier to identify any contextual or
grammatical error. One should be extra cautious with the words
highlighted in the paragraph as they may be incorrectly used either in
meaning or in their usage.

Practice Exercise Based on new Pattern

Directions (1-10): In each of the following questions given below, a


sentence is given with some bold words which may contain errors. Below
each of the sentence, a table is given with two columns in which column ‘A’
contains the list of bold words, and in column ‘B’ the suggested corrections
are listed. You have to choose the best alternate among the four given
options. If no correction is required against the given bold words, mark (e)
.i.e. “None of the above” as your answer.
1. Financial planning is the skillful synthesizer of art and science. It’s a
profession that requires emotional intelligence more than
intellectual intelligence. It’s all about knowing what makes people
tick, finding out their deepest inspirations and seeing whether they
have a fighting chance of fulfilling those.

Column A Column B
1. synthesizer 5. synthesis
2. profession 6. possession
3. emotional 7. emotions
4. inspirations 8. aspirations
2 Adda247 Publications For any detail, mail us at
Publications@adda247.com
Cracker Book for Bank (IBPS | SBI | RRB PO | Clerk) Mains Exams

(a) Both (1) - (5) and (4) – (8)


(b) (2) – (6)
(c) (1) – (5)
(d) (3-7)
(e) None of the above

2. Halting rampant environmental degradation in the Himalayas is now


emergency, and it is possible only across cooperation among all
members of the Himalayan basin community, from the lower Mekong
River region and China to the countries of southern Asia.
Column A Column B
1. rampant 5. remnant
2. emergency 6. urgent
3. possible 7. probable
4. across 8. through
(a) (4) – (8)
(b) (2) – (6)
(c) (1) – (5), (2) – (6) and (4) – (8)
(d) (2)- (6) and (4) – (8)
(e) None of the above

3. The report of the committee on real sector statistics advertised by


the National Statistical Commission (NSC) realized earlier this month
was met with a swift response by the ministry of statistics, which is
tasked with bringing out statistical publications.
Column A Column B
1. advertised 5. constituted
2. realized 6. released
3. swift 7. delicate
4. tasked 8. transferred

(a) (4) – (8)


(b) (2) – (6)
(c) (1) – (5), (2) – (6)
(d) (2) - (6) and (4) – (8)
(e) None of the above

3 Adda247 Publications For any detail, mail us at


Publications@adda247.com
Cracker Book for Bank (IBPS | SBI | RRB PO | Clerk) Mains Exams

4. Elections in South Asia are messed up but Pakistanis have taken to


electoral politics with gusto and their polity is colourful and vibrant.
The good thing about these elections in Pakistan has been that the
electoral conquest was focused on the political economy of Pakistan.
Column A Column B
1. messed up 5. messy
2. gusto 6. guts
3. vibrant 7. variance
4. conquest 8. discourse

(a) Both (1) - (5) and (4) – (8)


(b) (2) – (6)
(c) (1) – (5)
(d) (3-7)
(e) None of the above

5. India's stupendous run in the Asian Games is evident from the medal
tally. Our sportspersons have made the nation proud despite the
country developing a sporting culture.
Column A Column B
1. stupendous 5. stupendously
2. evident 6. evidencing
3. despite 7. in spite
4. developing 8. lacking
(a) Both (1) - (5) and (4) – (8)
(b) (4) – (8)
(c) (1) – (5)
(d) (3-7)
(e) None of the above

Directions (6-10): In the following questions a sentence is given, some


parts of the sentence are given in bold which may have grammatical or
spelling error or they may be contextually incorrect. Two columns (A) and
(B) are given, column (A) consists of bold parts of the sentence and column
(B) consists of the appropriate replacement for the bold parts. Match the
correct replacements. If the sentence is correct mark no error as your
answer.
4 Adda247 Publications For any detail, mail us at
Publications@adda247.com
Cracker Book for Bank (IBPS | SBI | RRB PO | Clerk) Mains Exams

6. The NCLT is the right body to resolve corporate insolvency and the
court did the right thing to acquire of a resolution process that it had
took the account for of the homebuyers’ interest, now that
Parliament has amended the law to incorporate homebuyers as
financial creditors.
Column A Column B
(A) to resolve corporate (I) resolving the corporate
insolvency insolvency
(B) to acquire (II) to let go
(C) took the account for (III) taken up on account
(D) has amended the law (IV) has been amended the law

(a) D-IV and B-II


(b) B-II and C-III
(c) A-I and B-II
(d) None of the given options are correct
(e) no error

7. A slow and gradual weakness of the rupees is not particularly


worrisome for India, as they are overvalued significantly in real
terms and the Reserve Bank of India (RBI) has sufficient reserves to
smoothen volatility in the currency market. The RBI has done well by
pre-emptively hiking rates to anchor inflationary expectations, which
will also help reduce volatility in the currency market.
Column A Column B
(A) weakness of the rupees (I) weakening of the rupee
(B) as they are overvalued (II) as it is significantly
significantly overvalued
(C) has sufficient reserves (III) had sufficient reserves
(D) which will also help (IV) who would also help

(a) D-IV and B-II


(b) B-II and C-III
(c) A-I and B-II
(d) None of the given options are correct
(e) no error
5 Adda247 Publications For any detail, mail us at
Publications@adda247.com
Cracker Book for Bank (IBPS | SBI | RRB PO | Clerk) Mains Exams

8. Some of a recent structural reform, such as the implementation of


the goods and services tax (GST) and Insolvency and Bankruptcy
Code (IBC), within the liberalization of foreign direct investment
and improvement in the ease of doing business, will help
improve economic activity.
Column A Column B
(A) a recent structural (I) the recent structural reforms
reform
(B) within the liberalization (II) along with liberalization
(C) in the ease of doing (III) in the easy business doing
business
(D) improve economic (IV) to be improved economic
activity activity
(a) D-IV and B-II
(b) B-II and C-III
(c) A-I and B-II
(d) None of the given options are correct
(e) no error
9. India has succeeded to an extent of the US and Japan, but a
lackadaisical attitude has prevailed on both sides when it comes to
an India-Australia relationship.
Column A Column B
(A) India has succeeded (I) India has been
succeeded
(B) of the US (II) with the US
(C) but a lackadaisical (III) besides lackadaisical
attitude has attitude has
(D) when it comes to an (IV) when they come to an
(a) D-IV and B-II
(b) B-II and C-III
(c) A-I and B-II
(d) None of the given options are correct
(e) no error

6 Adda247 Publications For any detail, mail us at


Publications@adda247.com
Cracker Book for Bank (IBPS | SBI | RRB PO | Clerk) Mains Exams

10. Considering that indexation benefits are not provided in the


current tax regime in the computation of capital gains from sale of
equity shares, it would be appropriate for the I-T department to
consider the FMV of shares and the actual cost of acquisition,
whichever is higher, in all circumstances.

Column A Column B
(A) Considering that indexation (I) Considered indexation
benefits benefits
(B) in the current tax regime (II) inside the current tax
regime
(C) it would be appropriate (III) it is appropriated
(D) and the actual cost of (IV) or the actual cost of
acquisition acquisition

(a) D-IV and B-II


(b) B-II and C-III
(c) A-I and B-II
(d) None of the given options are correct
(e) no error

Directions (11-15): In the following questions a sentence is given, some


parts of the sentence are given in bold which may have grammatical or
spelling error or they may be contextually incorrect. Two columns (A) and
(B) are given, column (A) consists of bold parts of the sentence and column
(B) consists of the appropriate replacement for the bold parts. Match the
correct replacements. If the sentence is correct mark no error as your
answer.
11. Historically, UP has threw up most of the prime ministers with the
largest number of MPs being returned to Lok Sabha, and Modi too
appear to has a game-plan to make an impact by putting all
economic and industrial eggs in the UP basket, instead of spreading
them thin all over India.

7 Adda247 Publications For any detail, mail us at


Publications@adda247.com
Cracker Book for Bank (IBPS | SBI | RRB PO | Clerk) Mains Exams

Column A Column B
(A) UP has threw up most (I) UP has thrown up
of the most of the
(B) the largest number of (II) the largest number
with
(C) Modi too appear to has (III) Modi too appears to
a game-plan have a game-plan
(D) instead of spreading (IV) instead to spread them
them thin thin

(a) C-III and B-II


(b) B-II and D-IV
(c) A-I and C-III
(d) None of the given options are correct
(e) no error

12. When this income level are exceeded, most business income can still
qualify as long as the business pays a certain amount of wages or has
certain depreciable property or has a combination of both.

Column A Column B
(A) When this income level (I) When these income
levels
(B) amount of wages (II) amount in wage
(C) certain (III) a certain amount of
(D) has a combination of both (IV) have a combinations of
both

(a) C-III and B-II


(b) A-I and C-III
(c) B-II and D-IV
(d) None of the given options are correct
(e) no error

8 Adda247 Publications For any detail, mail us at


Publications@adda247.com
Cracker Book for Bank (IBPS | SBI | RRB PO | Clerk) Mains Exams

13. Instead of the temporary problem that it created, people in and out
stood by him, appreciating his approach to root out the evil of
black money.

Column A Column B
(A) Instead of the temporary (I) In spite of the temporary
(B) people in and out (II) people by and large
(C) appreciating his approach (III) to be appreciated his
approach
(D) the evil of black money (IV) the sanity of black money

(a) A-I and B-II


(b) A-I and C-III
(c) B-II and D-IV
(d) None of the given options are correct
(e) no error

14. Due to the reason of high wildfire danger in the Washington and
Idaho, Avista personnel will be making changes to the way they re-
energize downed power lines.

Column A Column B
(A) Due to the reason of high (I) Due to high wildfire
wildfire
(B) in the Washington (II) within the Washington
(C) will be making changes (III) will made the changes
(D) they re-energize (IV) it re-energizes

(a) A-I and B-II


(b) A-I and C-III
(c) B-II and D-IV
(d) None of the given options are correct
(e) no error

9 Adda247 Publications For any detail, mail us at


Publications@adda247.com
Cracker Book for Bank (IBPS | SBI | RRB PO | Clerk) Mains Exams

15. Top management level changes have been affected suddenly,


agreements have been signed with private companies for satellite-
related work, the navigation satellite programme has suffered a
setback, and the moon mission has apparently been postponed.
Column A Column B
(A) have been affected (I) has been affected
suddenly suddenly
(B) signed with private (II) signature for private
companies companies
(C) has suffered a setback (III) have been suffering a
setback
(D) has apparently been (IV) has been apparently
postponed postponed
(a) A-I and B-II
(b) A-I and C-III
(c) B-II and D-IV
(d) None of the given options are correct
(e) no error

Solutions

1. (a); The sentence is providing information on the profession of


Financial Planning. However, the highlighted words “synthesizer”
and “inspirations” are creating contextual errors in the sentence.
They should be replaced by “synthesis” and “aspirations”
respectively to form a coherent sentence. Hence, option (a) is the
most suitable answer choice.
Synthesizer means an electronic musical instrument, typically
operated by a keyboard, producing a wide variety of sounds by
generating and combining signals of different frequencies.
Synthesis means the combination of components or elements to
form a connected whole.
Inspirations means the process of being mentally stimulated to do
or feel something, especially to do something creative.
Aspirations means a hope or ambition of achieving something.
10 Adda247 Publications For any detail, mail us at
Publications@adda247.com
Cracker Book for Bank (IBPS | SBI | RRB PO | Clerk) Mains Exams

2. (d); The sentence is describing about the severe degradation of the


environment which can be halted only through the cooperation
among all members. However, the highlighted words “emergency”
and “across” are grammatically and contextually incorrect. It is to
be noted that for using the word “emergency” the sentence
requires an article “an” which is not provided. Moreover, “across” is
used to from one side to the other of (a place, area, etc.) while
“through” by means of (a process or intermediate stage). Therefore,
these should be replaced by “urgent” and “through” respectively.
Hence, option (d) is the most suitable answer choice.
Rampant means (especially of something unwelcome) flourishing
or spreading unchecked.
Remnant means a part or quantity that is left after the greater part
has been used, removed, or destroyed.

3. (c); The highlighted words “advertised” and “realized” are contextually


incorrect in accordance to the theme of the sentence. They should
be replaced by “constituted” and “released” respectively to frame a
contextually meaningful sentence. The other highlighted words are
grammatically correct and contextually relevant. Hence, option (c)
is the most suitable answer choice.
Advertised means describe or draw attention to (a product, service,
or event) in a public medium in order to promote sales or
attendance.
Constituted means give legal or constitutional form to (an
institution); establish by law.
Realized means become fully aware of (something) as a fact;
understand clearly.
Released means allow or enable to escape from confinement; set
free.
Tasked means make great demands on (someone's resources or
abilities).

4. (a); The words/ phrase given in bold “messed up” and “conquest”
create either grammatical or contextual error in the sentence. To
correct the sentence, replace these words by “messy” and
“discourse” respectively. It is to be noted that while stating a
11 Adda247 Publications For any detail, mail us at
Publications@adda247.com
Cracker Book for Bank (IBPS | SBI | RRB PO | Clerk) Mains Exams

general truth or fact, present tense is used, therefore, the phrase


“messed up” stands incorrect in the sentence. Hence, option (a) is
the most suitable answer choice.
Gusto means enjoyment and enthusiasm in doing something.
Guts means personal courage and determination; toughness of
character
Conquest means the subjugation and assumption of control of a
place or people by military force.
Discourse means written or spoken communication or debate.

5. (b); The word given in bold “developing” creates a contextual error in


the sentence. To correct the sentence, replace these words by
“lacking”. All the other highlighted words are grammatically correct
and contextually meaningful. Hence, option (b) is the most suitable
answer choice.

6. (b); The sentence can be corrected by replacing phrases (B) and (C) by
phrases (II) and (III) respectively. Phrase (B) creates a contextual
error in the sentence, since it is describing about the shift of
authority to resolve corporate insolvency from the court to the
NCLT. Therefore, “to acquire” should be replaced by the idiom “to
let go” which means to allow someone or something to escape or go
free. Moreover, phrase (C) should be replaced phrase (III) as
phrase (C) does not fit in the appropriate grammatical syntax. All
the other highlighted phrases are correct. Hence, option (b) is the
most suitable answer choice.

7. (c); Phrases (A) and (B) are either grammatically incorrect or


contextually meaningless. Phrase (A) means that rupees have some
weakness while the appropriate phrase should be phrase (I) which
means that there is a devaluation of rupee. Moreover, phrase (II)
fits in the appropriate grammatical syntax since the pronoun “it”
should represent “rupee” instead of “they”. All the other
highlighted phrases are correct. Hence, option (c) is the most
suitable answer choice.
12 Adda247 Publications For any detail, mail us at
Publications@adda247.com
Cracker Book for Bank (IBPS | SBI | RRB PO | Clerk) Mains Exams

8. (c); Phrases (A) and (B) are either grammatically incorrect or


contextually meaningless. Therefore, they should be replaced by
the phrases (I) and (II) respectively. It is to be noted that the
sentence is beginning with “some of…” which implies that the noun
associated with it [structural reforms] should be in its plural form.
Moreover, phrase (B) creates a contextual error, hence it should be
replaced by phrase (II). All the other highlighted phrases are
correct. Hence, option (c) is the most viable answer choice.

9. (d); Phrase (B) contains a grammatical error as “of” should be replaced


by “with” to make the sentence grammatically and contextually
correct. Since, all the other phrases are grammatically correct,
option (d) becomes the most viable answer choice.

10. (d); Phrase (D) contains a grammatical error as “and” should be


replaced by “or” to make the sentence grammatically and
contextually correct. The hint for the same can be drawn from the
latter part of the sentence stating, “whichever is higher”. This
indicates a choice between the two objects. Thus, “or the actual cost
of acquisition” is the suitable phrase to fit in the sentence. Since, all
the other phrases are grammatically correct, option (d) becomes
the most viable answer choice.

11. (c); Phrases (A) and (C) are grammatically incorrect which should be
replaced by phrases (I) and (III) respectively. Phrase (A) should be
in past participle tense, therefore, “threw up” should be replaced by
“thrown up”. Moreover, in phrase (C), since Modi is singular, the
verb associated to it should also be singular, hence “appear” should
be replaced by “appears”. In addition to, 'have to' is used to mean
that something is necessary. It is used in the following way in
affirmative sentences: “subject + modal (have to / has to) + verb”.
All the other phrases are correct and do not require any
corrections or replacements. Hence, option (c) is the most viable
answer choice.

13 Adda247 Publications For any detail, mail us at


Publications@adda247.com
Cracker Book for Bank (IBPS | SBI | RRB PO | Clerk) Mains Exams

12. (b); Phrase (A) and (C) are grammatically incorrect and therefore, they
should be replaced with phrases (I) and (III). It is to be noted that
the non-highlighted part “…are exceeded” provides a hint that the
subject of the sentence should also be in its plural form, hence, “this
income level” should be replaced by “these income level”.
Moreover, it is to be noted that the phrase “a certain amount of” is
always followed by an uncountable noun while; “certain” is
followed by a plural noun. Since, the non-highlighted part mentions
“…depreciable property”, therefore, the highlighted part should be
“a certain amount of”. All the other highlighted parts are correct.
Therefore, option (b) is the most feasible answer choice.

13. (a); Phrases (A) and (B) contain errors in them. It is to be noted that,
“instead of” means ‘as a substitute for or alternative to’ while; “in
spite of” means ‘without being affected by the particular factor
mentioned’. Thus, to make the phrase contextually relevant to the
sentence, replace phrase (A) by phrase (I). Moreover, there is a
contextual error in phrase (B) as well. The idiom ‘in and out’ means
inconsistent and unreliable, which fails to provide appropriate
meaning to the sentence, thus it should be replaced by phrase (II)
as, idiom “by and large” means on the whole; everything
considered. All the other highlighted phrases are precise; hence,
option (a) is the most suitable answer choice.

14. (d); The error lies in only the phrase (A) of the sentence. It is to be
noted that the phrase “due to the reason of” stands incorrect as
“due to” and “reason” provides similar contextual meaning, hence
one of the term is redundant in the phrase. Therefore, phrase (A)
should be replaced by phrase (I) to make the sentence
grammatically correct and contextually meaningful. Since none of
the given combinations provide the appropriate answer, option (d)
becomes the most feasible answer choice.

15. (e); All the highlighted phrases of the given sentence are grammatically
correct and contextually meaningful. Thus, they do not require any
replacements or corrections. Hence, option (e) is the most viable
answer choice.
14 Adda247 Publications For any detail, mail us at
Publications@adda247.com
Cracker Book for Bank (IBPS | SBI | RRB PO | Clerk) Mains Exams

1 Adda247 Publications For any detail, mail us at


Publications@adda247.com
Cracker Book for Bank (IBPS | SBI | RRB PO | Clerk) Mains Exams

Chapter
Reading Comprehension (Part – I)
7
STUDY TIPS

This pattern of Reading comprehensions was asked in SBI PO MAINS


2017 EXAM. It is similar to old pattern, but the difference is the kind of
question one expects from these reading comprehensions. Inference
questions and paragraph connection questions are something which you
will find usually in these patterns instead of direct questions.

Tricks: Before reading a student must understand what he/she must


look mostly for in the passage. Understanding the theme and authors view
point is important as it helps to eliminate most of the option. For reading
quickly practice “TIME BASED READING”. In this method you practice by
reading an article in the newspaper within the stipulated time which you
set according to yourself.

Practice Exercise Based on new Pattern

Directions (1-5): Read the following passage carefully and answer the
questions given below them.
Paragraph 1: Former Reserve Bank of India (RBI) governor Y.V. Reddy, in a
speech last week, said that confidence in the working of public sector banks
is at a historic low. The reason for this is not very difficult to discern. PSU
banks are grappling with a high level of bad loans, and a number of them
have been put under RBI’s prompt corrective action and are not in a
position to lend. In the March quarter, PSU banks booked losses in excess of
Rs 62,000 crore and the total gross non-performing assets (NPAs) stood at
about Rs 9 trillion. Although the government is in the process of
recapitalising state-run banks, it is likely that the current Rs 2.11 trillion
PSU bank recapitalization plan will not be sufficient to put the PSU banks
back on track. Since PSU banks own about 70% of banking assets, their
inability to lend will have a direct impact on economic growth. Therefore, it
is important that the situation is handled with care.
2 Adda247 Publications For any detail, mail us at
Publications@adda247.com
Cracker Book for Bank (IBPS | SBI | RRB PO | Clerk) Mains Exams

Paragraph 2: As recently reported by Bloomberg, four out of 21 PSU banks


have not appointed replacements for chief executive officers (CEOs) and top
executives in nine more banks are expected to leave in the coming months.
Given this state of affairs, it is possible that new CEOs may not be appointed
in time. It is certainly not a desirable situation, especially at a time when
banks are stressed and need swift decision making. It is important to have a
plan in place for a smooth transition at the top. However, it is also likely
that the government will find it difficult to attract talent due to the fear of
investigative agencies among bankers. A number of present and former
senior executives are under investigation for past transactions. The
government must ensure that investigations don’t become a witch-hunt,
and that the issue is handled with utmost care.

Paragraph 3: The government is now mulling the formation of an asset


reconstruction company(ARC) for faster resolution of bad loans and has
constituted a committee to make recommendations in this regard. The
committee is expected to submit its recommendation in two weeks. While it
will be interesting to see the suggestions, in principle, the idea is unlikely to
go very far. The basic problem will be one of valuation of stressed assets.
For instance, if they are transferred at par and the resolution is left to a
government-owned ARC, it could end up creating more complications in the
system. Also, the ARC will need a significant amount of capital, which the
government is not in a position to provide. Banks should be able to resolve
bad assets under this framework. If the government can actually find
resources to reduce stress in the banking system, it would do well to
reassess the capital requirement of PSU banks and revisit the capital
infusion plan.

Paragraph 4: Apart from capital needs and faster resolution of stress


assets, PSU banks need governance reforms—something that has been
largely missing so far from the picture. It is correct that the present
government has refrained from micromanaging PSU banks, but this in itself
will not solve the problem. The government, perhaps, needs to put in place
a new framework for governance where, for instance, appointments at
higher levels are made in time, and the board is professional and
3 Adda247 Publications For any detail, mail us at
Publications@adda247.com
Cracker Book for Bank (IBPS | SBI | RRB PO | Clerk) Mains Exams

accountable. A situation where banks run without a CEO should never arise.
PSU banks should be in a position to attract talent by offering competitive
compensation at every level to be able to improve their operation and risk
management systems. Only when banks are run by professionals will they
be in a position to fund India’s growth in the long run and create value for
all stakeholders, including the taxpayer.

Paragraph 5: At a broader level, as Reddy noted in his remarks, there


should be clarity on the future of PSU banks. In fact, some of the banking
reforms will only work if a clear road map is defined. For instance, if the
government believes that a few banks should focus on underbanked areas,
some fiscal support may be warranted. Perhaps banks should be allowed to
focus on specific areas of strength so that they become more efficient over
time and are not dependent on budgetary support for growth.
Though the government has entered its last year in office, it still has time to
initiate broad reforms and give a fresh direction to PSBs. It will be difficult
to sustain higher growth in the medium term without a strong banking
system.

1. What is/ are the issue(s) that need attention in order to ameliorate
the condition of banks?
(I) No reform concocted to create better governance structures in
banks
(II) Government policies are acting as a hindrance to raise funds for
recapitalization.
(III) Absence of senior officials to supervise the activities of banks.
(IV) Inability of the banks to resolve the bad assets.
(a) Only (I)
(b) Only (II) and (III)
(c) Only (I), (III) and (IV)
(d) Only (I), (II) and (III)
(e) All are correct

4 Adda247 Publications For any detail, mail us at


Publications@adda247.com
Cracker Book for Bank (IBPS | SBI | RRB PO | Clerk) Mains Exams

2. Is it correct to say that formation of a government owned asset


reconstruction company is detrimental in the current situation?
(a) Yes, as government has inadequate amount of capital to
reconstruct the assets.
(b) No, as banks are incompetent to resolve their bad assets.
(c) Yes, as banks need support of the government to reduce its
stress.
(d) No, asset reconstruction company needs regular supervision by
the government.
(e) None of these

3. Which of the following can be inferred from paragraph 5 which forms


a connection with paragraph 4?
(a) Public sector banks should make appointments of new talents,
thus bringing a transformation.
(b) Banks should focus on strengthening its specific areas of interest
at one time.
(c) The government should implement new reforms to strengthen
the banks.
(d) Banks should directed and controlled by professionals so that
they can influence the growth of the nation.
(e) Strong banking system is needed to boost economic growth.

4. In context of the passage, the government should focus on


(I) Recapitalizing the public- sector banks.
(II) Administering the investigations of banks’ supervisors.
(III) Improving the regulation of banks by making appointments on
time.
(a) Only (I) and (II)
(b) Only (II)
(c) Only (II) and (III) (d) Only (III)
(e) All of the above.

5 Adda247 Publications For any detail, mail us at


Publications@adda247.com
Cracker Book for Bank (IBPS | SBI | RRB PO | Clerk) Mains Exams

5. In which context does the word ‘mulling’ used in the passage?


(a) to investigate the regulation of the banks.
(b) to initiate new reforms thus giving a fresh direction to Public
sector banks.
(c) to ponder on the construction of a committee for the clearance
of bad loans.
(d) to influx an asset reconstruction company in the banking
system.
(e) All of the above.

Directions (6-10): Read the following passage carefully and answer the
questions given below them.

Paragraph 1: Coupled with a continual increase in voluntary


unemployment, the International Labour Organisation expects
unemployment in India to be higher in 2018. In a country where 65% of the
population is below 35 years, unemployment, especially among youth, can
limit the nation’s ability to reap the much-hyped demographic dividend.
Recognizing this challenge, a wide range of stakeholders, including the
government, companies, civil society organizations, and for-profit
enterprises are working either independently or in cohesion to enhance
youth employability.

Paragraph 2: The answer to the unemployability perhaps lies in the


enormity of the challenge in India—more needs to be done to make a
noticeable dent. Additionally, several other challenges give a unique colour
to the problem. First, unemployment is higher among the formally educated
in comparison to the illiterate. Second, there is higher youth unemployment
in rural areas, while most interventions focus on urban areas. Third, there is
a mismatch between the skill sets that industries require and the skill sets
that youth are equipped with. These structural challenges result in a
demand-supply mismatch which can be summarized as a) mismatch
between youth aspirations and the skills training being provided, b)
mismatch in skills training and industry needs, and c) poor industry buy-in
for vocational training courses because of lack of standardization and
universally accepted certification.
6 Adda247 Publications For any detail, mail us at
Publications@adda247.com
Cracker Book for Bank (IBPS | SBI | RRB PO | Clerk) Mains Exams

Paragraph 3: Solutions, therefore, must focus on understanding


aspirations, industry requirements and standardization across the skill-
development value chain. Well-designed interventions will be effective only
if the candidates are willing, receptive and capable of absorbing the
knowledge or skill being imparted by the intervention. Further, candidates
may already possess specific complementary skill sets that could provide
them with a competitive advantage. A candidate-selection framework
would greatly enhance the efficacy of such interventions.

Paragraph 4: Similarly, while designing programmes, it is critical to map


skills being imparted to the specific needs of potential employers so that the
skilling-to-employment loop is closed seamlessly. A recent right to
information request highlighted a failure of placement-led programmes
undertaken by the National Skill Development Corporation (NSDC). Of the
800,145 candidates trained through non-scheme skilling programmes in
2016-17, only 48.4% received placements. As a result, NSDC plans to move
to a model where training partners will receive funds as per the outcomes
achieved which is a welcome move for strengthening the placement led
programmes. Similarly, when it comes to designing programmes that focus
on self-employment or entrepreneurship, it is important to assess demand
for the product or service, and study policies or schemes that can be
leveraged to enhance sales. This is critical as beneficiaries of such
programmes are expected to source their own work after the training.

Paragraph 5: There is also scope for increased public-private partnerships.


Most skilling organizations struggle to access premises and technical
infrastructure/equipment, which constrains their scale. Public-private
partnerships (PPPs) can use existing under-utilized infrastructure available
with educational institutions to facilitate vocational training and skill
development with regular analysis and monitoring. PPPs can also facilitate
finance and market linkages. For instance, partnerships with financial
institutions can provide the seed capital (through government schemes
such as Stand-up India) required by beneficiaries of self-employment
models to set up micro-businesses.

7 Adda247 Publications For any detail, mail us at


Publications@adda247.com
Cracker Book for Bank (IBPS | SBI | RRB PO | Clerk) Mains Exams

6. What is/are the factor(s) responsible for unemployability in India?


(a) Low rate of manufacturing and production companies in India.
(b) Lesser focus in rural areas compared to urban areas.
(c) Subpar job specific training
(d) Both (b) and (c)
(e) All are correct

7. Which of the following step(s) should be taken in order to curb the


rising unemployment?
(I) Accessing premises and technical infrastructure for training.
(II) Providing outcome-based funding to training partners
(III) Accessing the policies for enhancing sales.
(a) Only (II)
(b) Both (I) and (III)
(c) Only (III)
(d) Both (II) and (III)
(e) All are correct

8. Why there is a mismatch between the skill sets that industries require
and the skill sets that youth are equipped with?
(a) Enterprises and stakeholders are working independently
(b) Lack of scope in public-private partnership`
(c) Inability in sourcing one’s own income
(d) Improper assessment of the skills which are needed to be
imparted as per the specific demand of employers.
(e) None of these.

9. Which of the following can be the reason for the increase in the
voluntary unemployment?
(a) Inability of the nation’s ability to reap the much-hyped
demographic dividend.
(b) Absence of candidate selection framework
(c) Lack of outcome based funding
(d) Improper comprehension of the youth’s aspiration
(e) None of these.

8 Adda247 Publications For any detail, mail us at


Publications@adda247.com
Cracker Book for Bank (IBPS | SBI | RRB PO | Clerk) Mains Exams

10. In this question, a word is given in bold which is then followed by four
words. These four words are then followed by five options which give
the combination of the words. Choose the combination of words
which are “most similar to” the word given in bold?
CONSTRAINS
(I) Coerce (II) Contrive
(III) Immure (IV) Stifle
(a) I-III (b) II-IV (c) I-IV
(d) III-IV (e) I-II

Directions (11- 15): Read the following passage carefully and answer the
questions given below them.

Paragraph 1: On 7 June, the Union ministry of finance dispatched a letter to


the Chairman/Managing Directors of sponsor banks. The letter indicated
that the Union government, in consultation with the National Bank for
Agriculture and Rural Development (NABARD), has decided to go ahead
with phase III of the amalgamation of regional rural banks (RRBs). This will
bring down the total number from 56 to 38 and ensure that most of the
smaller states have only one RRB, while the larger might have two. There is
a lot to be said about the merits of this decision (as well as the previous
decisions) of amalgamating RRBs cutting across sponsor banks. First, these
banks, when they were set up, were seen as alternatives to cooperatives,
adding a touch of professionalism to the local feel ,a cooperative society
provided. They were seen as decentralized solutions for the skewed
banking development that was happening across the country, with the
south and west being well banked while the north-east, east, and central
regions suffered. While we do have large nationalized banks for the
purposes of banking services in general, RRBs provide services proximate
to the rural customer. There are merits in the argument that size reduces
overheads, makes regulation easier and optimizes the use of technology.
But what is the additional benefit it creates for customers?

9 Adda247 Publications For any detail, mail us at


Publications@adda247.com
Cracker Book for Bank (IBPS | SBI | RRB PO | Clerk) Mains Exams

Paragraph 2: While the merits of that argument can be debated, let us look
at the blatant travesty when it comes to corporate governance. This
decision is not being taken in unusual times where the banks are in distress
and customer interests are to be protected. Even then, it would be the
Reserve Bank of India (RBI) rather than the government that would take
this call. These are decisions taken in cold blood. If that is the case, what is
the role of the respective boards of the RRBs, and the boards of the sponsor
banks?

Paragraph 3: The government does not seem to be carrying out even a


token consultation with the RRBs’ boards. Of course, they are substantially
owned by the government. The Union government has a 50% stake, the
respective state governments, 15%, and the sponsor banks have a 35%
stake. There is no indication that the Union government consulted the
respective state governments. The letter from the ministry is terse. It just
asks the chairmen/managing directors to send a no-objection before a
certain date. That is it. It does not expect anybody except the people in the
ministry to apply their minds to the proposal. The forced merger of RRBs
without consultation of the owners can only be seen as tyranny. This is a
warning bell for the public-sector banking reforms that might be in the
offing.

Paragraph 4: Even in the case of public sector banks, there is constant talk
about the Union government wanting to merge some banks and have a few
larger banks. We saw that with the associate banks of the State Bank of
India (SBI) getting merged with the SBI and the chairperson (getting an
extension of tenure just to oversee this merger process). There is much
chatter on the possibility of large-scale merger proposals. These, when they
happen, will be legally valid because the government has the power to do so
under the Bank Nationalization Act. But is this the most appropriate way of
going about the task? After all, these are large listed institutions that not
only have minority shareholders but a large number of depositors whose
savings are at stake. Does the bureaucracy have the necessary expertise to
understand the nuances and see the merits of the mergers? What should be
the reform agenda? Should we not first identify the horse and the cart and
then get the sequence right?
10 Adda247 Publications For any detail, mail us at
Publications@adda247.com
Cracker Book for Bank (IBPS | SBI | RRB PO | Clerk) Mains Exams

Paragraph 5: Governance reform should perhaps start with making public


sector institutions more accountable to market discipline. Move them from
the Bank Nationalization Act to the Companies Act, make them accountable
to the Securities and Exchange Board of India, and address the concerns
expressed by the RBI governor about a level playing field in the supervision
and regulation of public sector banks by removing all the exemptions under
the Banking Regulation Act. This would make them more accountable with
regard to their strategy and financials to a larger set of analysts. This would
also help them discover their strategic positioning. The next step would be
to move towards reducing the government stake, both directly and
indirectly. Not only does the government own these entities directly, it also
exerts control through institutions controlled by the government such as
the Life Insurance Corporation of India. Once that is done, the state has to
ensure that there is an interested single promoter-like shareholder.

11. The reason (s) behind merging of RRBs is/ are


(I) to improve the condition of the cooperative sector.
(II) to make the banks recover from bad loans.
(III) to systematize the banking system across the country.
(IV) to make regulation easier in banks.
(a) Only (IV)
(b) Only (I), (III) and (IV)
(c) Only (III)
(d) Only (II) and (III)
(e) All are correct

12. Which of the following sentences can be inferred from paragraph 3


which form(s) a connection with paragraph 2?
(a) Union government is the maximum stake holder of RRB.
(b) Union government should reduce its stake holding either
directly or indirectly.
(c) Many public- sector banks are merging with private sector
institutions.
(d) The government has not consulted Board members before
giving their decision.
(e) All of the above
11 Adda247 Publications For any detail, mail us at
Publications@adda247.com
Cracker Book for Bank (IBPS | SBI | RRB PO | Clerk) Mains Exams

13. In which context is ‘cold blood’ used in the passage?


(a) to display insensitive step taken by government without consent
of boards members.
(b) to make government realize their role towards banks.
(c) to promote RRBs at larger level.
(d) to underscore the urgency of good governance in the banking
industry.
(e) to highlight the advantages of amalgamating the banks.

14. According to the passage, which is the most appropriate way of


regulating good governance in banks?
(I) Investing in government bonds.
(II) Making banks responsible and reliable entity to market
discipline.
(III) Infusing adequate capital to the banks.
(IV) Employing other institutions to control the banks.
(a) Only (IV)
(b) Only (I), (III) and (IV)
(c) Only (III)
(d) Only (II) and (IV)
(e) All are correct

15. According to the passage, what is the main purpose of forming RRBs?
(a) to assist government in raising the capital.
(b) to support commercial banking in bad loans.
(c) to provide basic facilities to low class families.
(d) to connect rural population to banking sector.
(e) All of the above.

Directions (16- 20): Read the following passage carefully and answer the
questions given below them. Certain words/phrases have been printed in
bold to help you locate them while answering some of the questions.

Paragraph 1: Rudyard Kipling once described Shimla as a “centre of power


and pleasure”. The power faded with the Raj. Now, pleasure is at a
premium. Shimla is struggling with a water crisis that is an echo of Cape
Town’s distress earlier this year. It has run out of municipal drinking water

12 Adda247 Publications For any detail, mail us at


Publications@adda247.com
Cracker Book for Bank (IBPS | SBI | RRB PO | Clerk) Mains Exams

supply during peak tourist season. Citizens are being forced to queue up to
collect water from tankers. Schools have been shut down for 10 days. This
crisis is a reflection of a wider problem confronting India.

Paragraph 2: India has adequate freshwater. The problem is inefficient and


wasteful use. According to the Central Water Commission, agriculture
consumed about 85.3% of total freshwater in 2000. This is likely to
decrease only by a meagre 2% by 2025. Water usage for major crops in
India—paddy and maize, for instance—is two to four times that in other
large farming nations thanks to wasteful flood irrigation, mostly in northern
India. This can be traced to the subsidy regime. The Economic Survey,
2015-16 noted that the present subsidy structure “encourages using more
inputs such as fertilizer, water and power, to the detriment of soil quality,
health and the environment”. Most states provide electricity either for free
or at a flat rate. This inevitably leads to wasteful water extraction. Both the
Economic Survey and a 2015 International Monetary Fund (IMF) study
have noted that these subsidies disproportionately benefit rich and large
farmers. A number of economists have recommended tapering off
electricity and water subsidies. The Aadhaar and financial inclusion drives
have laid the foundations for the Centre and states to do just that via
targeted direct benefit transfers.

Paragraph 3: Concurrently, irrigation infrastruc-ture must be upgraded


and research and development efforts focused on improving agricultural
productivity with lower water usage. There have been some promising
developments here. Punjab Agriculture University, for example, has
recently come up with a new water-saving variety of rice that matures one
to five weeks earlier than other varieties without compromising on the
yield. The government is also collaborating with Israel—an established
leader in water-management techniques—to promote drip irrigation; Asia’s
largest such project took off in Karnataka earlier this year.

Paragraph 4: About 80% of drinking water needs are sustained by


groundwater. Look no further than today’s centre of power to see the
problems here. In 2001, India’s groundwater authority banned private
water extraction in Delhi due to the looming fear of groundwater depletion.
However, the black market persists. Here’s how dire the situation is:
According to scientists at the National Geophysical Research Institute, Delhi
could dry up in a few years.
13 Adda247 Publications For any detail, mail us at
Publications@adda247.com
Cracker Book for Bank (IBPS | SBI | RRB PO | Clerk) Mains Exams

Paragraph 5: It’s worth noting here that water is a state subject and states
have kept water-pricing rates stagnant for about three decades now. Pair
this with the subsidy burden—the IMF study reckoned that it “amounted to
0.6% of global gross domestic product in 2012”—and authorities are left
with little financial means to invest in the water- management practices
that would provide sustainable, long-term solutions. These range from the
construction of reservoirs to building water treatment and recycling
infrastructure. Putting in place viable water-pricing policies and ending
subsidies will be tricky given the political optics. But these are essential
changes. Others are needed as well. India has an antiquated legal
framework to regulate groundwater. Since it is considered a part of land
and gives landowners unrestricted entitlement to it, the government is left
with little leeway to act. Legislative change is important.

16. According to the passage, which of the following sentences portray


the crisis(es) in Shimla?
(I) There is irregular Electricity supply in Shimla.
(II) Educational institutes have been shut down.
(III) People are struggling for water accessibility.
(IV) Shortage of municipal drinking water supply in Shimla.
(a) Only (I)
(b) Only (I) and (II)
(c) Only (II), (III) and (IV)
(d) Only (I), (II) and (III)
(e) All are correct

17. According to the passage, what is/ are the component (s) of
unnecessary use of water?
(a) Excess supply of water to other states.
(b) Excess use of groundwater for personal use.
(c) Large consumption of freshwater in agriculture.
(d) Both (b) and (c)
(e) All are correct.
14 Adda247 Publications For any detail, mail us at
Publications@adda247.com
Cracker Book for Bank (IBPS | SBI | RRB PO | Clerk) Mains Exams

18. What efforts should India make to prevent itself from going dry?
(I) Research and development efforts should be focused on
improving agricultural productivity with lower water usage.
(II) India should provide water- management awareness to its
citizens.
(III) India should collaborate with other countries for learning water
management techniques.
(a) Only (I)
(b) Both (I) and (III)
(c) Both (II) and (III)
(d) Only (III)
(e) All are correct

19. How can we justify “water is a state subject” stated by the author?
(a) As water affects every state of economy.
(b) As a particular country can decide the future of water
availability in their country.
(c) As state of a country is responsible for any water crises
occurred.
(d) As different states have different water procurement process.
(e) None of these

20. The appropriate title of the passage is


(a) India and Israel collaboration
(b) The Aadhaar and financial inclusion
(c) Improving irrigation infrastructure
(d) India to the brink of a water crisis
(e) The global gross domestic product: IMF

Directions (21-30): Read the following passage carefully and answer the
questions given below them. Certain words/phrases have been printed in
bold to help you locate them while answering some of the questions.
Indian roads are usually characterized by poor infrastructure and
congestion which affect travel time and road safety. This is a big hindrance
in economic development and leads to inefficiency in the transportation of
goods and services across the country. To address this, the government has
embarked upon a massive overhaul of the country’s road network through
15 Adda247 Publications For any detail, mail us at
Publications@adda247.com
Cracker Book for Bank (IBPS | SBI | RRB PO | Clerk) Mains Exams

Bharatmala Pariyojana—an umbrella highway development programme


involving 34,800 km of road network at an investment of Rs5.35 trillion, to
be completed by 2022. The programme focuses on optimizing the efficiency
of road traffic movement across the country by bridging critical
infrastructure gaps through shorter routes. The end goal is to create
economic corridors (ECs) along the path—new industries, more
employment and new markets.

The programme, however, will have a negative bearing on the existing road
network because it will compete directly with some of the existing build-
operate-transfer (BOT) toll road projects. Out of the 44 ECs, about 21 would
partially or fully affect the existing alignments, while the remaining 23 that
involve upgradation of existing alignment will not result in any deviation.
Among the 21 corridors affected, eight have a totally different route (which
is shorter) while the remaining 13 have some deviations from the existing
alignment. Overall, there are 24 BOT projects and one operate-maintain-
transfer (OMT) project whose traffic could be affected by the proposed ECs.
The Bharatmala programme may result in traffic diversion from the
existing road network to new roads, thereby affecting the toll collection
and, consequently, the debt servicing ability of some of the BOT and OMT
projects. This has raised the risk of default on 25 national highway toll
projects which involve Rs19,435 crore of debt. The risk of such loan
defaults will add to banks’ and financial lenders’ stressed assets and non-
performing assets. In terms of risk, 12% of the projects have a high risk of
leakage in traffic, if a completely alternative route is available, 16% of the
projects have moderate risks, and 72%, low risks. To arrive at the debt at
risk, the debt outstanding for each of these special purpose vehicles (SPVs),
their repayment tenure, concession end date, credit profile of the SPV and
its sponsor credit risk profile, are considered. Out of the total debt at risk
for the 25 affected projects about Rs6,536 crore, which accounts for about
34% of the total debt at risk, is high-risk. Projects with debt at a moderate
risk have an aggregate debt of Rs3,483 crore, while about Rs9,416 crore of
debt is considered to be low-risk.
To ensure that the existing BOT projects that are at risk of
default do not turn bad for the financial institutions, swift and adequate
measures are needed. The Kelkar committee had observed that since
infrastructure projects span over 20-30 years, a private developer may lose
16 Adda247 Publications For any detail, mail us at
Publications@adda247.com
Cracker Book for Bank (IBPS | SBI | RRB PO | Clerk) Mains Exams

bargaining power owing to abrupt changes in the economic or policy


environment. It has thus recommended that the private sector must be
protected against such loss. This could be ensured by allowing
renegotiation of the terms of the concession agreement.
Financial institutions are already reluctant to finance the infrastructure
sector, given the rise in non-performing assets (NPAs). Add to this the
probable difficulties that would arise in the case of 25 BOT projects, which
would put additional stress on the road infrastructure exposure. The need
of the hour is to realign the terms and conditions of the model concession
agreement to ensure that banks do not end up accumulating NPAs.Having
an appropriate remedial mechanism for BOT operators will help retain
interest for investments in new projects; for the lenders, it will help curtail
the number of stressed assets from the risk of default.

21. According to the passage, Bharatmala Project aims to


(I) impact industrial development positively
(II) create employment and develop new markets
(III) improve the political status of the country.
(a) Only (I)
(b) Only (II) and (III)
(c) Only (III)
(d) Only (I) and (II)
(e) All are correct

22. According to the passage, what worries BOT operators?


(a) The risk of loan default.
(b) Disruptions in integrated Infrastructure network.
(c) Inability to maintain the overall structure of highway network in
India.
(d) Both (a) and (b)
(e) All ae correct

23. The total debt at risk is


(a) Rs10,456 crore (b) Rs3,483 crore (c) Rs19,435 crore
(d) Rs9,416 crore (e) Rs18,544 crore
17 Adda247 Publications For any detail, mail us at
Publications@adda247.com
Cracker Book for Bank (IBPS | SBI | RRB PO | Clerk) Mains Exams

24. The appropriate title of the passage is


(a) renegotiating the concession agreement.
(b) Bharatmala and the rising debt issues
(c) Creating economic corridors
(d) The rising issue of stressed assets
(e) The Kelkar Committee

25. Remedial measure(s) available to BOT operators is/ are


(I) Providing them the necessary resources for accomplishment of
the project.
(II) realigning the terms and conditions of the model concession
agreement to ensure that banks do not end up accumulating
NPAs.
(III) Allowing renegotiation of the terms of concession agreement to
private sector.
(a) Only (I)
(b) Both (II) and (III)
(c) Only (III)
(d) Both (I) and (II)
(e) All are correct

Directions (26- 30): Read the following passage carefully and answer the
questions given below them.

Paragraph 1: Sometime last year, the Union government suddenly


announced that India would make the paradigm shift to full electric
mobility by 2030. This was more a statement of intent signalling a
necessary and transformative shift to ease polluting vehicular emissions on
our congested roads. NITI Aayog has since suggested that 40% of personal
vehicles and public transportation in the country should go completely
electric by 2030. This seems a more realistic target. There have been
extensive discussions over the last few months on the way forward to
increase the use of electric vehicles (EVs). A new policy announcement has
been rightly shelved; Faster Adoption and Manufacturing of Hybrid and
Electric Vehicles (FAME) II appears to have been postponed to September,
and Energy Efficiency Services Ltd has also deferred its purchase of 10,000
EVs. These developments essentially underline the difficulties of proceeding
on an uncharted path.
18 Adda247 Publications For any detail, mail us at
Publications@adda247.com
Cracker Book for Bank (IBPS | SBI | RRB PO | Clerk) Mains Exams

Paragraph 2: In our enthusiasm to adopt electric mobility, we should not


rush into a flawed policy and implementation framework. Sober analysis
and a step-by-step approach are required even as the ecosystem for electric
vehicles develops gradually, alternative models are pursued, and learnings
from experience accrue. Issues today relate to public vs private
transportation; battery charging vs swapping models; nature and quantity
of incentives; and development of storage technologies.

Paragraph 3: Our initial focus must be unquestionably on public


transportation—bus, taxi and auto fleets. Public transport in India is
expected to double by 2030. It is the major consumer of diesel and petrol
and responsible for substantial polluting emissions. Primarily, then, we
must electrify vehicles which travel long distances every day. In personal
transport, two-wheelers must be prioritized since India has one of the
largest two-wheeler markets in the world. If we opt for the charging model,
traffic congestion will worsen, with lengthy queues. The battery swapping
model, however, would involve no waiting time. Stations could be at bus
depots or at select petrol pumps. It will take 2-3 minutes to swap, similar to
the time taken to fill up and pay for petrol. This would control congestion at
these points. Swapping stations would supply fully charged new batteries,
allowing users to get higher range. Their storage conditions will also help
enhance battery life.

Paragraph 4: There have been some concerns about previous experiences


in China and Israel. But these models were meant for personal cars and
were costly. Besides, lack of marketing, proper execution and
mismanagement led to the failure of the promising EV start-up in Israel
Better Place. The subsidy structure also became distorted, favoring large
batteries. Currently, the subsidy structure for a car does not incentivize the
purchase of an electric car as the capital cost is much higher. The same
applies to a bus. Both models should get equivalence in subsidy support. A
small group should immediately study this issue. Efficiency in terms of total
cost per kilometre, not capital costs or larger batteries, should be
incentivized.

19 Adda247 Publications For any detail, mail us at


Publications@adda247.com
Cracker Book for Bank (IBPS | SBI | RRB PO | Clerk) Mains Exams

Paragraph 5: Ideally, the best course would be to select five smart cities
with the objective of fully electrifying their public transportation as well as
50% of their two-wheelers by 2025. This means just EVs should be
registered there two years from now. Each year, five more cities should be
added. Both charging and swapping models should be given space. Within
five years, we will have sufficient experience to plan a further accelerated
transition, including perhaps daytime charging by solar, which could prove
more suitable for swapping. An expert group should lay the groundwork for
these plans. We must take small steps to make the big leap. Shenzhen in
China is adopting EVs in phases. From 1 July, all ride-hailing vehicles should
be EVs, and, by 31 December, all the remaining taxis should be replaced by
EVs.

26. Which of the following can be inferred from the passage?


(a) Battery swapping model is more efficient than charging model.
(b) According to NITI Aayog, 40% of transportation system should
go electric by 2030.
(c) All diesel or petrol vehicles should be replaced by electric
vehicles soon.
(d) Improper management is the factor responsible for failure of EV
model in Israel.
(e) All of the above

27. How electric vehicles project can become feasible in India?


(I) by encouraging the use of less personal vehicles.
(II) by adopting battery swapping model to avoid traffic congestion.
(III) by employing long distance electrified vehicles in order to
reduce pollution emission.
(a) Only (I)
(b) Only (III)
(c) Both (I) and (II)
(d) Both (II) and (III)
(e) All are correct

20 Adda247 Publications For any detail, mail us at


Publications@adda247.com
Cracker Book for Bank (IBPS | SBI | RRB PO | Clerk) Mains Exams

28. According to the passage, how must Electric Vehicles be promoted by


government ?
(a) by incentivizing two- wheelers vehicles
(b) by enhancing battery life
(c) by providing personal cars
(d) by subsidizing its batteries
(e) All of the above

29. According to the passage, how the electric mobility policy should be
implemented?
(a) through expert framework.
(b) by selecting five cities at a time to be electrified.
(c) by allowing both charging and swapping models to be there for
battery backup.
(d) Both (b) and (c)
(e) All are correct

30. Choose the appropriate title of the passage.


(a) Step- by- Step approach to electric vehicles
(b) Faster Adoption and Manufacturing of Electric Vehicles
(c) Increasing pollution and traffic congestion
(d) Road map for electric vehicles in India
(e) Challenges in implementing electric vehicles

Solutions

1. (c); Going through the passage, we come across the issues PSU banks
are facing.
In Paragraph 2, it has been mentioned that appointment of top
officials has not been done since long time, which signifies the poor
supervision of the activities of banks.
In paragraph 3, the inability of banks to resolve the bad assets has
been discussed for which asset reconstruction company has been
decided to form.
21 Adda247 Publications For any detail, mail us at
Publications@adda247.com
Cracker Book for Bank (IBPS | SBI | RRB PO | Clerk) Mains Exams

In paragraph 4, The need for governance reforms has been


discussed.
Whereas, Sentence (II) is irrelevant with respect to the passage.
Hence option (c) is the most appropriate choice.

2. (a); We can assert from paragraph 3 that formation of a government


owned asset reconstruction company is detrimental since, that the
government is not in the position to provide significant amount of
capital to resolve the bad assets.
This proves the fidelity of option (a).
Refer to the lines of 3rd paragraph “The basic problem will be one of
valuation of stressed assets. For instance, if they are transferred at
par and the resolution is left to a government-owned ARC, it could
end up creating more complications in the system. Also, the ARC
will need a significant amount of capital, which the government is
not in a position to provide.”

3. (c); Here (paragraph 4 and paragraph 5) the emphasis has been laid on
the governance reforms needed by the banks. By initiating broad
reforms, government can bring transformation in the current
situation of banks. Thus, we can easily point out that sentence (c)
forms a connection with paragraph 4 and hence is the correct
answer choice.

4. (e); All the three objectives of the government have been mentioned in
the passage. For expression (I) refer to the 1st paragraph “Although
the government is in the process of recapitalizing state-run banks,
it is likely that the current Rs 2.11 trillion PSU bank recapitalization
plan will not be sufficient to put the PSU banks back on track.”
For the 2nd objective, refer to the 2nd paragraph, “A number of
present and former senior executives are under investigation for
past transactions. The government must ensure that investigations
don’t become a witch-hunt, and that the issue is handled with
utmost care.”
While the 3rd objective to be focused by banks is mentioned in 2nd
paragraph “However, it is also likely that the government will find
it difficult to attract talent due to the fear of investigative agencies
22 Adda247 Publications For any detail, mail us at
Publications@adda247.com
Cracker Book for Bank (IBPS | SBI | RRB PO | Clerk) Mains Exams

among bankers.” Since, all three areas of government that requires


focus have been illustrated in the passage, option (e) becomes the
right choice.

5. (c); ‘Mulling’ is a verb which means think about (something) carefully,


especially before making a decision or reaching a conclusion. It has
been used in paragraph 3 indicating the deep thought given on the
formation of an asset reconstruction committee for resolving the
bad loans. This suggests that sentence (c) is the most appropriate
choice. Other sentences are not related to the word ‘mulling’.
Refer the lines “The government is now mulling the formation of an
asset reconstruction company(ARC) for faster resolution of bad
loans and has constituted a committee to make recommendations
in this regard.”

6. (d); We can conclude that sentences (b) and (c) are correct. Refer to the
last 2 lines of the 2nd paragraph of the passage “(c) poor industry
buy-in for vocational training courses because of lack of
standardization and universally accepted certification”, it can be
inferred that option (c) is correct as subpar means something that
is below average, or below what is expected. Option (b) is also
correct, refer to second paragraph of the passage, “there is higher
youth unemployment in rural areas, while most interventions focus
on urban areas.”
7. (d); Refer the fourth paragraph of the passage in which it is stated that
the NSDC planned to provide funds to training partners according
to the outcomes achieved, which is the first move by NSDC for
designing programmes and meeting the specific needs of potential
employers.
Hence option (d) is the most appropriate choice.
Refer the lines “…As a result, NSDC plans to move to a model where
training partners will receive funds as per the outcomes achieved.”

8. (d); Option (d) is the correct choice for the given question. Refer to the
4th paragraph of the passage, “Similarly, when it comes to
designing programmes that focus on self-employment or
entrepreneurship, it is important to assess demand for the product
23 Adda247 Publications For any detail, mail us at
Publications@adda247.com
Cracker Book for Bank (IBPS | SBI | RRB PO | Clerk) Mains Exams

or service, and study policies or schemes that can be leveraged to


enhance sales”, the key word is “assessing” and “similarly”. The
author through his other case wants to imply that there is a lack of
assessment by the placement led programmes.

9. (d); Option (d) is the correct choice. The “Voluntary Unemployment”


refers to the situation when the worker deliberately chooses not to
work because of a low wage scale or not able to find out the
suitable employment for him. Refer to the 1st line of the 3rd
paragraph of the passage, “Solutions, therefore, must focus on
understanding aspirations “. This was referring to the solution to
the rising unemployment. As aspiration means a hope or ambition
of achieving something, therefore, we can infer that the rise in the
voluntary unemployment lies in the improper
comprehension(understanding) of their ambitions or aspirations.

10. (d); Option (d) is the correct choice as Immure and Stifle are similar in
meaning to constrains.
Constrain in the context of the passage means severely restrict the
scope, extent, or activity of.
Immure means enclose or confine (someone) against their will.
Stifle means prevent or constrain (an activity or idea).
Coerce means persuade (an unwilling person) to do something by
using force or threats.
Contrive means create or bring about (an object or a situation) by
deliberate use of skill and artifice.

11. (b); Sentence (I),(III) and (IV) are correct as they provide the correct
reason behind merging of RRBs. Paragraph 1 mentions the merits
of amalgamation of RRBs, making regulation easier is one of them.
Refer the lines (i) “First, these banks, when they were set up, were
seen as alternatives to cooperatives.”
(ii) “They were seen as decentralized solutions for the skewed
banking development that was happening across the country, with
the south and west being well banked while the north-east, east,
and central regions suffered.”
24 Adda247 Publications For any detail, mail us at
Publications@adda247.com
Cracker Book for Bank (IBPS | SBI | RRB PO | Clerk) Mains Exams

(iii)” There are merits in the argument that size reduces overheads,
makes regulation easier and optimizes the use of technology.”

12. (d); It has been discussed in Paragraphs 3 the step taken by


government without even a token consultation with the RRBs’
board, forming a connection with paragraph 2.
All the other options are irrelevant.
Hence option (d) is the correct answer choice.

13. (a); Referring to the second paragraph of the passage we can infer that
‘cold blood’ is used in terms of the decision taken by the
government without consultation of boards of RRBs. Hence this
step is regarded as insensitive taken in unusual times.
Hence sentence (a) is the most appropriate choice here. Refer the
lines “Even then, it would be the Reserve Bank of India (RBI) rather
than the government that would take this call. These are decisions
taken in cold blood. If that is the case, what is the role of the
respective boards of the RRBs, and the boards of the sponsor
banks?”

14. (d); Refer the fifth paragraph of the passage in which it is mentioned
that government should instead of merging the banks go for other
options like reducing their stake and giving it to other institutions
for controlling the banks. Government should also work towards
making public sector institutions more accountable to market
discipline.
Hence only sentences (II) and (IV) are correct proving fidelity of
option (d).
Refer the lines “Governance reform should perhaps start with
making public sector institutions more accountable to market
discipline.”
“The next step would be to move towards reducing the government
stake, both directly and indirectly. Not only does the government
own these entities directly, it also exerts control through
institutions controlled by the government such as the Life
Insurance Corporation of India.”
25 Adda247 Publications For any detail, mail us at
Publications@adda247.com
Cracker Book for Bank (IBPS | SBI | RRB PO | Clerk) Mains Exams

15. (d); Refer first paragraph of the passage in which it is mentioned the
purpose of RRB which is to provide banking services to rural
customers.
All the other sentences are not its correct explanation.
Hence option (d) is the most appropriate choice.
Refer the lines “While we do have large nationalized banks for the
purposes of banking services in general, RRBs provide services
proximate to the rural customer.”

16. (c); It has been mentioned in the first paragraph of the passage that
Shimla is struggling with water crises as municipal water supply is
discontinued, schools have been shut down and citizens are being
forced to queue up to collect water from tankers. From these we
can conclude that sentences (II), (III) and (IV) are correct. Sentence
(I) is irrelevant in context of the passage. Hence option (c) is most
appropriate choice.
Refer the lines “It has run out of municipal drinking water supply
during peak tourist season. Citizens are being forced to queue up to
collect water from tankers. Schools have been shut down for 10
days.”

17. (d); The author has highlighted two components of inefficient and
wasteful use of water in the passage, first is in agriculture and
second is in personal consumption.
Hence, we can make option (d) as our choice.
Sentence (a) is irrelevant with respect to the passage.
Refer the lines of second paragraph “According to the Central
Water Commission, agriculture consumed about 85.3% of total
freshwater in 2000.”
Refer the lines of fourth paragraph “About 80% of drinking water
needs are sustained by groundwater”.

18. (b); Refer the third paragraph of the passage where the efforts needed
to be made are mentioned in order to prevent excess use of water.
Sentence (II) is not mentioned anywhere in the passage.
Hence option (b) is the most appropriate choice to be made.

26 Adda247 Publications For any detail, mail us at


Publications@adda247.com
Cracker Book for Bank (IBPS | SBI | RRB PO | Clerk) Mains Exams

19. (b); Refer the fifth paragraph of the passage in which water is defined
as a state subject which have kept the water- pricing rates constant
since long time thus affecting its availability and GDP.
Hence, we can mark option (b) as its correct explanation.
Refer the lines “It’s worth noting here that water is a state subject
and states have kept water-pricing rates stagnant for about three
decades now. Pair this with the subsidy burden—the IMF study
reckoned that it “amounted to 0.6% of global gross domestic
product in 2012”—and authorities are left with little financial
means to invest in the water- management practices that would
provide sustainable, long-term solutions.”

20. (d); The appropriate title for the given passage is “India to the brink of
a water crisis”. This is because the whole passage describes about
the water crisis that are happening in various parts of India.
For example Shimla is struggling with water crisis due to which
schools have been shut down. Many more such examples have been
given in the passage.

21. (d); We can conclude from first paragraph of the passage that sentences
(I) and (II) are in relevance with the passage. As mentioned in the
paragraph that the goal is to create new industries, more
employment and new markets. Hence, we can clearly infer from
this that these developments will bring the socio- economic change
and will uplift the underdeveloped regions of the country.
Sentence (III) is totally irrelevant in context to the passage.
Hence option (d) is the correct choice.
Refer the lines “The programme focuses on optimizing the
efficiency of road traffic movement across the country by bridging
critical infrastructure gaps through shorter routes. The end goal is
to create economic corridors (ECs) along the path—new industries,
more employment and new markets.”

22. (d); Both (a) and (b) options are correct in context to the passage.
Existing road network got affected with implementation of
Bharatmala Pariyojana. This programme affect the toll collection
and thereby debt servicing ability. Hence option (d) is the correct
27 Adda247 Publications For any detail, mail us at
Publications@adda247.com
Cracker Book for Bank (IBPS | SBI | RRB PO | Clerk) Mains Exams

choice. Sentence (c) is incorrect as it is not mentioned anywhere in


the passage.
Refer the lines “The Bharatmala programme may result in traffic
diversion from the existing road network to new roads, thereby
affecting the toll collection and, consequently, the debt servicing
ability of some of the BOT and OMT projects.”
“Out of the 44 ECs, about 21 would partially or fully affect the
existing alignments, while the remaining 23 that involve
upgradation of existing alignment will not result in any deviation.”

23. (c); The total debt at risk is Rs19,435 crore. It has mentioned in the
passage that debt servicing ability has raised the risk of default on
25 national highway toll projects involving Rs19,435 crore of debt.
Hence option (c) is the most appropriate choice.
Refer the line of second paragraph “This has raised the risk of
default on 25 national highway toll projects which involve
Rs19,435 crore of debt.”

24. (b); “Bharatmala and the rising debt issues” is an appropriate title of
the passage.

25. (b); Refer the fourth paragraph of the passage in which it is clearly
mentioned that the need of the hour is to realign the terms and
conditions of the model concession agreement to ensure that banks
do not end up accumulating NPAs .Now in the third paragraph
,refer the lines “This could be ensured by allowing renegotiation of
the terms of the concession agreement.”, “the NHAI can
compensate the affected BOT operators and account for it in the
total outlay of the Bharatmala programme.” Hence option (b) is the
correct choice. Sentence (I) is irrelevant here.

26. (e); All of the given sentences can be inferred from the passage.
Sentence (a): Refer paragraph 3 “If we opt for the charging model,
traffic congestion will worsen, with lengthy queues. The battery
swapping model, however, would involve no waiting time.”
Sentence (b): Refer paragraph 1 “NITI Aayog has since suggested
that 40% of personal vehicles and public transportation in the
country should go completely electric by 2030.”
28 Adda247 Publications For any detail, mail us at
Publications@adda247.com
Cracker Book for Bank (IBPS | SBI | RRB PO | Clerk) Mains Exams

Sentence (c): Refer paragraph 5 “From 1 July, all ride-hailing


vehicles should be EVs, and, by 31 December, all the remaining
taxis should be replaced by EVs.”
Sentence (d): Refer paragraph 4 “Besides, lack of marketing, proper
execution and mismanagement led to the failure of the promising
EV start-up in Israel Better Place.”

27. (d); In reference to paragraph 3, we can conclude that sentences (II)


and (III) are correct.
Various measures have been mentioned that will encourage the use
of electric vehicles using of battery swapping model and long
distance Electric vehicles are some of them.
Sentence (I) is irrelevant to the passage.
Hence option (d) is the most appropriate choice.
Refer the lines “Primarily, then, we must electrify vehicles which
travel long distances every day. In personal transport, two-
wheelers must be prioritized since India has one of the largest two-
wheeler markets in the world. If we opt for the charging model,
traffic congestion will worsen, with lengthy queues.”

28. (d); We can refer to paragraph 4 of the passage that mentions that
electric vehicles should be subsidized through its large batteries.
Hence option (d) is the most appropriate choice to be made.

29. (e); All the given sentences are correct in context of the paragraph 5. All
the sentences, according to the paragraph 5, indicate that the policy
should be implemented in phases.
Refer the lines “Ideally, the best course would be to select five
smart cities with the objective of fully electrifying their public
transportation as well as 50% of their two-wheelers by 2025.”
“Each year, five more cities should be added. Both charging and
swapping models should be given space.”
“An expert group should lay the groundwork for these plans.”
Hence option (e) is the correct choice.

30. (d);“Road map for electric vehicles in India” is an appropriate title of the
passage as the whole passage revolves around this theme.

29 Adda247 Publications For any detail, mail us at


Publications@adda247.com
Cracker Book for Bank (IBPS | SBI | RRB PO | Clerk) Mains Exams

1 Adda247 Publications For any detail, mail us at


Publications@adda247.com
Cracker Book for Bank (IBPS | SBI | RRB PO | Clerk) Mains Exams

Chapter

8 Reading Comprehension Part - II


STUDY TIPS

Before reading a student must understand what he/she must look


mostly for in the passage. Understanding the theme and authors view
point is important as it helps to eliminate most of the option.

For reading quickly practice “TIME BASED READING”. In this method


you practice by reading an article in the newspaper within the stipulated
time which you set according to yourself.

Practice Exercise Based on New Pattern

Directions (1-10): Read the following passage and answer the following
questions accordingly.
In 1859, when Charles Darwin presented his famed theory of evolution
by natural selection, inherent in its essence was the abiding notion of
survival of the fittest. To evolve, compete and do better has been the
cornerstone of the survival of any species. This has also helped ensure that
countries and societies have created an era that is prosperous and (largely)
peaceful than at any other time in history. Innovation, progressive
disruption and the effective use of intellectual property are the new tools
of the economy this century. For examples, one only has to look at Israel or
Singapore. In the 21st century, they are clear leaders on their strengths in
the fields of scientific, industrial and financial innovations. Hence the
lessons for emerging markets are clear — the need to innovate and
compete globally.
India’s case has been a success story of sorts. From a country that was
hardly acknowledged, to becoming a case study in economic reforms, it has
done much to improve the health of the economy and its people. As one of
the top growing economies in the world, it now stands at the cusp of
leapfrogging from an emerging to a near-advanced economy over the next

2 Adda247 Publications For any detail, mail us at


Publications@adda247.com
Cracker Book for Bank (IBPS | SBI | RRB PO | Clerk) Mains Exams

decade. But going forth, many of the paradigms that have held the Indian
economy in good stead may not be efficient in the decades ahead. Equally
so, providing access to health care, medical technologies and
pharmaceutical products for a billion people is still a challenge, with no
clear consensus on the best way forward.
Improving the environment for innovation and enhancing
competitiveness, especially in areas such as science and technology and
health care, will help propel our economic engine of growth and improve
the health indices of citizens. In an emerging environment of protectionism
and the rise of the anti-globalization movement, the need to innovate and
compete will become even more important. Advanced western economies,
with low single-digit growth rates, are now resorting to protectionist
measures. What is troubling is that matching voices are being heard within
emerging economies like India. The more prudent step would be to
enhance capabilities to innovate rather than merely adopt as this changing
global mood becomes more perverse. The answer lies in creating a
knowledge and information technology-based, intellectual property-
focused and entrepreneurially-led economy — or a KITE economy as it
were.

In the field of health, pharmaceuticals and scientific research, the need to


protect the intellectual property of all entities is crucial. We need a multi-
fold increase in scientific capabilities and infrastructure as well as a
regulatory framework to enhance clinical research. Over the past decade,
clinical research, especially clinical trials, has moved away from India to
countries like China owing to policy decisions. Reversing this will not only
help India to innovate and compete with the rest of the world but also
create jobs across the value chain. Similarly, taking meaningful steps to be
in step with international best practices, remove barriers and enhance
predictability in the patents regimes will help grow the innovation
paradigm in health access, pharmaceutical developments and medical
device manufacturing in India. In order to do this, structural reforms and
increased funding of regulatory agencies are a must. India needs to improve
competitiveness through a paradigm shift in innovation and intellectual
3 Adda247 Publications For any detail, mail us at
Publications@adda247.com
Cracker Book for Bank (IBPS | SBI | RRB PO | Clerk) Mains Exams

property rights. Its scientists, technology entrepreneurs and health


evangelists need to have a protective umbrella of a strong patent regime
that helps them innovate without having to look over their shoulders. Steps
to make India a KITE leader in health are the need of the hour. So, while the
government attempts more structural reforms, it would do well to ensure
that the overall competitiveness of the health sector improves as it
embarks on an ambitious mission to provide quality health care to more
than a billion citizens.
1. Why do the emerging markets need to innovate and compete globally?
(a) For providing a multifold increase in scientific capabilities.
(b) To provide reforms and global funds to individual investors.
(c) As some nations are clear leaders in terms of scientific, financial
and industrial innovations.
(d) For the ineffective use of intellectual property.
(e) None of these.

2. What is the emerging environment mentioned that makes


competition even more important?
(a) Entrepreneurially-led economy based environment.
(b) Adoption of structural reforms.
(c) Knowledge and information technology-based environment.
(d) Protectionism and the rise of the anti-globalization movement.
(e) None of these.

3. How can India compete with rest of the world in terms of clinical
research?
(a) By enhancing research and developing on new techniques.
(b) Reversing the research outflow of India to China.
(c) Through shift in the issues that area taken up for mobilization.
(d) Both (a) and (b)
(e) Both (a) and (c).

4 Adda247 Publications For any detail, mail us at


Publications@adda247.com
Cracker Book for Bank (IBPS | SBI | RRB PO | Clerk) Mains Exams

4. What is/are the steps required to grow the innovation paradigm in


health access and medical device manufacturing in India?
(a) Improve competitiveness through a paradigm shift in
innovation.
(b) Structural Reforms
(c) Increased funding of regulatory agencies.
(d) Both (a) and (b)
(e) All (a), (b) and (c)

5. Which of the statements is definitely false according to the passage?


(a) Economic engine of growth will be propelled through
improvement of the environment for innovation.
(b) We need a multi-fold increase in scientific capabilities and
infrastructure as well as a regulatory framework to enhance
clinical research.
(c) Every species wants to evolve, compete and do better for
survival.
(d) Advanced western economies with high growth rates are
resorting to protectionist measures.
(e) None of these.

Directions (6-8): Certain words have been given in bold in the passage.
Find the word or the pair of words from the given options which is/are the
synonym of the given bold word.
6. Disruption
(i) Commotion (ii) Cede
(iii) Turmoil (iv) Diminution
(a) ii-iii (b) ii-iv (c) i-iii
(d) iii-iv (e) only ii

7. Propel
(i) Deter (ii) Impel
(iii) Stifle (iv) Hinder
(a) ii-iv (b) ii-iii (c) i-ii
(d)Only (ii) (e) i-iii

5 Adda247 Publications For any detail, mail us at


Publications@adda247.com
Cracker Book for Bank (IBPS | SBI | RRB PO | Clerk) Mains Exams

8. Prudent
(i) Chary (ii) Didactic
(iii) Sagacious (iv) Bugged
(a) ii-iv (b) i-iii (c) ii-iii
(d) i-ii (e) iii-iv

Directions (9-10): Certain words have been given in bold in the passage.
Find the word or the pair of words from the given options which is the
antonym of the given bold word.
9. Innovate
(I) Stagnate (ii) Commence
(iii) Pioneer (iv) Impede
(a) i-ii (b) i-iv (c) ii-iv
(d) ii-iii (e) iii-iv

10. Embark
(i) Desolate (ii) Cease
(iii) Culminate (iv) Concord
(a) ii-iii (b) i-iv (c) iii-iv
(d) ii-iv (e) Only i

Directions (11-15): Read the following passage carefully and answer the
questions given below it. Certain words have been printed in bold to help
you locate them, while answering some of the questions.
A major problem of Indian industrial and commercial development was the
supply of capital. Until 1850, British capital was shy of Indian adventure.
The risks and unknown factors were too great, and prospects in other
directions too bright. The working capital of the agency house after 1813 at
first consisted mainly of the savings of the Company’s servants. Their cries
of woe when these houses fell as in the crisis of 1831 were loud and
poignant. Indian capital was also shy for different reasons. It needed to
acquire confidence in the new regime, and outside the presidency towns, to
acquire confidence in the new regime, and outside the presidency towns, to
acquire the habit of investment. Investment for large scale production for
‘enabling’ works like railways was an unfamiliar and suspected practice.
6 Adda247 Publications For any detail, mail us at
Publications@adda247.com
Cracker Book for Bank (IBPS | SBI | RRB PO | Clerk) Mains Exams

Thus, the first big development came when European capital was coaxed
into the country by government guarantees or went of its own free will to
develop industries with which it was already familiar as in the case of jute
or coal. Indian capital followed where it was in touch with European
practice as in Bombay (Mumbai) and dealing with familiar products like
cotton. These considerations throw into all the greater relief the
achievement of the Tata’s in developing iron and steel. Thus, the major part
of the capital provided was British which a steadily increasing Indian
proportion from 1900. As late as 1931-32 the capital of companies
registered abroad was nearly four times that of companies registered in
India. But this is not an exact guide because it leaves out of account the
stock in British companies held by Indians, as well as government stocks.
Speaking plainly, it may be said that the capital of the cotton industry
was mainly Indian, that of the iron and steel industry entirely so, that of the
jute industry about half and half, while the coal and plantation industries
were mainly British, together with that used for the building of railways,
irrigation, and other public works. Management in the cotton and steel
industries was mainly Indian though European technicians were freely
employed, that of the jute, coal, and the plantation industries being
European, the jute men in particular being Scotch. Their capital, apart of
course from government enterprise, operated thorough joint-stock
companies and managing agencies. The latter arose through the
convenience found by bodies of capitalists seeking to develop some new
activities and lacking any Indian experience, of operating through local
agents. It arose in the period after 1813 when private merchants took over
the trade formerly monopolized by the Company. The money world be
found in Britain to promote a tea garden, a coal mine, or a jute mill, but the
management would be confided to a firm already on the spot. The
managing agency was the hyphen connecting capital with experience and
local knowledge.
Until 1914 the policy of the government continued in the main to be one of
‘enabling’ private capital and enterprise to develop the country. Direct
promotion was confined to public utilities like canals and railways. The line
between enabling and interfering action became distinctly blurred,

7 Adda247 Publications For any detail, mail us at


Publications@adda247.com
Cracker Book for Bank (IBPS | SBI | RRB PO | Clerk) Mains Exams

however, in the case of the cotton industry and there was a tendency for
enabling action to pass over into the positive promotion of particular
projects. This was most noticeable in the time of Lord Curzon with his
establishment of an imperial department of agriculture with a research
station at Pusa and a department of commerce and industry presided over
by a sixth member of the Viceroy’s Council. The First World War began the
transition to a new period of active promotion and positive support. As the
conflict lengthened there arose a demand for Indian manufactured goods.
India failed to take full advantage of this opportunity, partly because of
uncertainty as to the future and partly because the means for sudden
expansion were lacking. The outcome of this situation was the appointment
of an industrial commission in 1916, under pressure from London. The
commission criticized the unequal development of Indian industry which
had led to the missing of her, war opportunity. A much closer co-operation
with industry was planned through provincial departments of industry.
Increased technical training and technical assistance to industry was
proposed while it was suggested that the central government should set up
a stores department which should aim at making India self-sufficing in this
respect. The commission’s report was only partially implemented, but a
stores department and provincial industrial departments were created and
something was done towards promoting technical assistance. The
importance of the report and its aftermath was that it marked the transition
from the conception of Indian economy in broadly colonial terms with
freedom for private enterprise to the conception of India as an
autonomous economic unit.
11. The following can be inferred from the passage:
A. Industrial development of a country requires supply of external
capital.
B. Investment in uncertain industries is more when government
provides guarantees against failure.
C. Lack of indigenous technical expertise cab be a constraining
factor in a country’s economic development.
D. Enabling infrastructure like railways would have to be provided
necessarily by the government.
E. Marked development for the final products is a important
prerequisite for industrial development.
(a) A and B (b) A, C and D (c) B, C and E
(d) C, D and E (e) A, B, C and E
8 Adda247 Publications For any detail, mail us at
Publications@adda247.com
Cracker Book for Bank (IBPS | SBI | RRB PO | Clerk) Mains Exams

12. The first capitalists investing in Indian economy were


(a) the Indians
(b) predominantly the British
(c) the Europeans except the British
(d) Both (A) and (B)
(e) Both (A) and (C)

13. After the start of the first World War, all of the following could be
likely reasons for the British government adopting a proactive stance
towards Indian industry except.
(a) The major investors in Indian enterprises were British and they
had missed out on an opportunity.
(b) The war had created a huge demand for industrial goods.
(c) The British government wanted economic development of the
country ad India was strategic economically in the war.
(d) The development of Indian economy was required for
contributing towards the war effort.
(e) The desire to see India as self-sufficient in technical expertise.

14. During the early twentieth century, Indians were restricted to making
investment in stocks of companies that were necessarily listed in
India. This was done with the aim of confining Indian capital to India
so that it could not compete with British capital.
(a) Definitely true as inferred from the passage.
(b) It was true on a selective case by case basis.
(c) This was the fact during the early part of the British rule.
(d) This was true in the later part of the British rule.
(e) No evidence to support the same is given in the passage.

15. From the passage it can be inferred that during the early part of
twentieth century, starting a greenfield project was more difficult for
an Indian capitalist than for an European.
(a) Definitely true as inferred from the passage.
(b) It was true on a selective case by case basis.
(c) No trend of discrimination between the two categories of
capitalists can be inferred from the passage.

9 Adda247 Publications For any detail, mail us at


Publications@adda247.com
Cracker Book for Bank (IBPS | SBI | RRB PO | Clerk) Mains Exams

(d) Preference was given to British capitalists, buffeted by the fact


that the country was under British rule.
(e) Preference was given to European capitalists.

Directions (16- 20): Read the following passage carefully and answer the
questions given below them. Certain words/phrases have been printed in
bold to help you locate them while answering some of the questions.
The painter is now free to paint anything he chooses. There are scarcely any
forbidden subjects, and today everybody is prepared to admit that a
painting of some fruit can be as important as a painting of a hero dying. The
Impressionists did as much as anybody to win this previously unheard-of
freedom for the artist. Yet, by the next generation, painters began to
abandon the subject altogether, and began to paint abstract pictures. Today
the majority of pictures painted are abstract.
Is there a connection between these two developments? Has art gone
abstract because the artist is embarrassed by his freedom? Is it that,
because he is free to paint anything, he doesn't know what to paint?
Apologists for abstract art often talk of it as the art of maximum freedom.
But could this be the freedom of the desert island? It would take too long to
answer these questions properly. I believe there is a connection. Many
things have encouraged the development of abstract art. Among them has
been the artists' wish to avoid the difficulties of finding subjects when all
subjects are equally possible.
I raise the matter now because I want to draw attention to the fact that the
painter's choice of a subject is a far more complicated question than it
would at first seem. A subject does not start with what is put in front of the
easel or with something which the painter happens to remember. A subject
starts with the painter deciding he would like to paint such-and-such
because for some reason or other he finds it meaningful. A subject begins
when the artist selects something for special mention. (What makes it
special or meaningful may seem to the artist to be purely visual its colours
or its form.) When the subject has been selected, the function of the
painting itself is to communicate and justify the significance of that
selection.
10 Adda247 Publications For any detail, mail us at
Publications@adda247.com
Cracker Book for Bank (IBPS | SBI | RRB PO | Clerk) Mains Exams

It is often said today that subject matter is unimportant. But this is only a
reaction against the excessively literary and moralistic interpretation of
subject matter in the nineteenth century. In truth the subject is literally the
beginning and end of a painting. The painting begins with a selection (I will
paint this and not everything else in the world); it is finished when that
selection is justified (now you can see all that I saw and felt in this and how
it is more than merely itself).
Thus, for a painting to succeed it is essential that the painter and his public
agree about what is significant. The subject may have a personal meaning
for the painter or individual spectator; but there must also be the possibility
of their agreement on its general meaning. It is at this point that the culture
of the society and period in question precedes the artist and his art.
Renaissance art would have meant nothing to the Aztecs and vice versa. If,
to some extent, a few intellectuals can appreciate them both today it is
because their culture is an historical one: its inspiration is history and
therefore it can include within itself, in principle if not in every particular,
all known developments to date.

When a culture is secure and certain of its values, it presents its artists with
subjects. The general agreement about what is significant is so well
established that the significance of a particular subject accrues and
becomes traditional. This is true, for instance, of reeds and water in China,
of the nude body in Renaissance, of the animal in Africa. Furthermore, in
such cultures the artist is unlikely to be a free agent: he will be employed
for the sake of particular subjects, and the problem, as we have just
described it, will not occur to him.

When a culture is in a state of disintegration or transition the freedom of


the artist increases-but the question of subject matter becomes problematic
for him: he, himself, has to choose for society. This was at the basis of all the
increasing crises in European art during the nineteenth century. It is too
often forgotten how many of the art scandals of that time were provoked by
the choice of subject (Gericault, Courbet, Daumier, Degas, Lautrec, Van
Gogh, etc.).
11 Adda247 Publications For any detail, mail us at
Publications@adda247.com
Cracker Book for Bank (IBPS | SBI | RRB PO | Clerk) Mains Exams

By the end of the nineteenth century there were, roughly speaking, two
ways in which the painter could meet this challenge of deciding what to
paint and so choosing for society. Either he identified himself with the
people and so allowed their lives to dictate his subjects to him; or he had to
find his subjects within himself as painter. By people I mean everybody
except the bourgeoisie. Many painters did of course work for the
bourgeoisie according to their copy-book of approved subjects, but all of
them, filling the Salon and the Royal Academy year after year, are now
forgotten, buried under the hypocrisy of those they served so sincerely.

16. When a culture is insecure, the painter chooses his subject on the
basis of:
(a) The prevalent style in the society of his time.
(b) Its meaningfulness to the painter.
(c) What is put in front of the easel.
(d) Past experience and memory of the painter.
(e) All of the above.

17. In the sentence, "I believe there is a connection" (second paragraph),


what two developments is the author referring to?
(a) Painters using a dying hero and using a fruit as a subject of
painting.
(b) Growing success of painters and an increase in abstract forms.
(c) Artists gaining freedom to choose subjects and abandoning
subjects altogether.
(d) Rise of Impressionists and an increase in abstract forms.
(e) All of the above

18. Which of the following is NOT necessarily among the attributes


needed for a painter to succeed:
(a) The painter and his public agree on what is significant.
(b) The painting is able to communicate and justify the significance
of its subject selection.
(c) The subject has a personal meaning for the painter.
(d) The painting of subjects is inspired by historical developments.
(e) None of these
12 Adda247 Publications For any detail, mail us at
Publications@adda247.com
Cracker Book for Bank (IBPS | SBI | RRB PO | Clerk) Mains Exams

19. In the context of the passage, which of the following statements would
NOT be true?
(a) Painters decided subjects based on what they remembered from
their own lives.
(b) Painters of reeds and water in China faced no serious problem of
choosing a subject.
(c) The choice of subject was a source of scandals in nineteenth
century European art.
(d) Agreement on the general meaning of a painting is influenced by
culture and historical context.
(e) All of the above

20. Which of the following views is taken by the author?


(a) The more insecure a culture, the greater the freedom of the
artist.
(b) The more secure a culture, the greater the freedom of the artist.
(c) The more secure a culture, more difficult the choice of subject.
(d) The more insecure a culture, the less significant the choice of the
subject.
(e) None of the above.

Directions (21- 30): Read the following passage carefully and answer the
questions given below them. Certain words/phrases have been printed in
bold to help you locate them while answering some of the questions.
Last week, US President Donald Trump announced America’s exit from the
Iran nuclear deal—formally known as the Joint Comprehensive Plan of
Action (JCPOA). The other signatories to the deal—the UK, France,
Germany, Russia and China—V of staying in the deal. If the JCPOA
unravels, it will have consequences for the entire world, including India.
Iran is India’s third largest oil supplier and serves as India’s link to
Afghanistan and Central Asia. Trump had three main problems with JCPOA:
a) The deal does not have binding restrictions on Iran’s ballistic missile
programme; b) many of the restrictions have sunset clauses; and c) the deal
does nothing to prevent Iranian sponsorship of terrorism in its region.

13 Adda247 Publications For any detail, mail us at


Publications@adda247.com
Cracker Book for Bank (IBPS | SBI | RRB PO | Clerk) Mains Exams

On 30 April, Israeli Prime Minister Benjamin Netanyahu had brought to


light Iranian nuclear weapons development archives. But, ironically, those
documents could have been used, as Robert Einhorn of the Brookings
Institution has suggested, to plug JCPOA’s gaps. If the archives contain solid
proof of Iranian plans to mount a nuclear warhead on a ballistic missile, it
would have given teeth to Trump’s efforts to contain Iran’s ballistic missile
development effort. But, for that, Trump had to sincerely abide by the
JCPOA until the International Atomic Energy Agency (IAEA) had established
the links between Iran’s extant ballistic missiles programme and its
previous plans to develop nuclear weapons. On the contrary, Trump has
been running a long, prejudiced campaign against the JCPOA and has been
given more than an impression that he just needed an excuse to exit the
deal.
Other participants of the JCPOA believe that the terms of the sunset clauses
could have been extended in negotiations with Iran. But that would have
also been possible only if the deal was adhered to by all parties. Trump and
other JCPOA opponents argue that lifting sanctions incentivised Iran to step
up terrorist and destabilizing activities in West Asia. They may not be
entirely wrong but the quest for a perfect deal can equally be a credible
path to an Iran with a nuclear bomb. What will happen now? It depends on
how effective the reimposed sanctions on Iran will be. If Europe, along with
Russia and China, refuses to play ball with the US, the sanctions regime will
be ineffective. In that case, Iran will not suffer much and the US will not
have much leverage—other than threat of military action—to stop the
Iranian bomb. Alternatively, the US can`t perforce make the sanctions
effective by threatening secondary sanctions against countries doing
business with Iran. European businesses and banks cannot even
contemplate being shut out of the US. China too knows from previous
experience of dealing with both Iran and North Korea that American
sanctions can indeed bite. Already reeling under sanctions imposed by the
West, Russia will not have much to offer Iran.
Even if the sanctions are effective, it does not necessarily mean that Iran
will come back to the table willing to accept harsher restrictions. Under
siege, Iran may instead decide to go down the path of acquiring the nuclear

14 Adda247 Publications For any detail, mail us at


Publications@adda247.com
Cracker Book for Bank (IBPS | SBI | RRB PO | Clerk) Mains Exams

bomb. Seeing how North Korea’s acquisition of the nuclear bomb has
increased its bargaining power may just act as a motivator. The Trump
administration may be secretly hoping that sanctions will lead to a regime
collapse in Iran. This is an idea fraught with danger. A regime collapse risks
a more hardline section in Iran gaining power. The new rulers may choose
to double down on the nuclear weapons programme and expand regional
influence. After studying other examples of nuclear proliferation, including
that of North Korea.
Given India’s dependence on Iran for oil and connectivity, as mentioned
earlier, it is easy to overstate the implications of JCPOA’s death, if that
occurs. India has many other oil suppliers ready to move in if imports from
Iran get disrupted. Reimposition of sanctions may even make Iran offer oil
on favourable terms to India. In any case, the Indian system has gone
through this before—in the run-up to JCPOA negotiations.

21. What was the impact of Israeli Prime Minister Benjamin Netanyahu’s
revelation?
(a) It brought an end to the dispute among the countries of JCPOA.
(b) It triggered the US’ exit from the JCPOA
(c) It bridged the gap between U.S. and Iran
(d) It gave a fresh impetus to Trump’s efforts to contain Iran’s
ballistic missile development effort
(e) All of the above

22. Which of the following words is most similar to “given teeth” as given
in bold in the passage?
(a) Constrain (b) Baffle (c) Succour
(d) Grim (e) Delay

23. What will make the sanctions regime on Iran effective?


(a) Acceptance of the restrictions by Iran.
(b) Consent by the countries like Russia and China.
(c) Prohibition of doing business by other countries in U.S.
(d) Both (b) and (c)
(e) All are correct
15 Adda247 Publications For any detail, mail us at
Publications@adda247.com
Cracker Book for Bank (IBPS | SBI | RRB PO | Clerk) Mains Exams

24. According to the passage, what may be the consequence(s) of


sanctions in Iran?
(I) acquirement of more nuclear weapons.
(II) collapse of Iran’s economy.
(III) expansion of regional influence.
(a) Only (I) (b) Only (II) (c) Both (II) and (III)
(d) Both (I) and (II) (e) All are correct

25. According to the passage, what will be the impact of sanctioning by


U.S. on India?
(I) India’s economy will suffer.
(II) Import of oil will be easily available by Iran.
(III) India will divert its root of import of oil to other countries.
(a) Only (I) (b) Only (II) (c) Both (II) and (III)
(d) Both (I) and (II) (e) All are correct

Direction (26): A phrase “has been given” given in the passage is


highlighted which may or may not contain grammatical error. There are
five alternatives given below, one of which may replace the existing
highlighted part to make the sentence grammatically correct and
contextually meaningful. Choose the most appropriate alternative as your
answer. If the phrase is grammatically correct, as given, and doesn’t require
any correction, choose option (e) i.e., “No correction required” as your
answer.
(a) has been giving (b) has given (c) has being giving
(d) had given (e) No correction required

Direction (27-28): Which of the following alternatives among the five


options provides the most similar meaning(s) of the word given in BOLD as
used in the passage?
27. Unravel
(I) disintegrate (II) slump
(III) terminate (IV) collapse
(a) Only (III)
(b) Only (II) and (IV)
(c) Only (I), (II) and (III)
(d) Only (II), (III) and (IV)
(e) All are correct
16 Adda247 Publications For any detail, mail us at
Publications@adda247.com
Cracker Book for Bank (IBPS | SBI | RRB PO | Clerk) Mains Exams

28. Perforce
(I) sedate (II) predicament
(III) fragility (IV) collude
(a) Only (I)
(b) Only (II) and (IV)
(c) Only (I), (III) and (III)
(d) Only (II), (I) and (IV)
(e) None of them

Direction (29- 30): Which of the following alternatives among the five
options provides the most opposite meaning(s) of the word given in BOLD
as used in the passage?

29. Fraught
(I) gullible (II) repose
(III) pragmatic (IV) meticulous
(a) Only (II)
(b) Only (I) and (IV)
(c) Only (II), (III) and (III)
(d) Only (II), (I) and (IV)
(e) None of them

30. Abide by
(I) spurn (II) vaunt
(III) nefarious (IV) flout
(a) Only (II)
(b) Only (I) and (IV)
(c) Only (II), (III) and (III)
(d) Only (II), (I) and (IV)
(e) None of them

17 Adda247 Publications For any detail, mail us at


Publications@adda247.com
Cracker Book for Bank (IBPS | SBI | RRB PO | Clerk) Mains Exams

Solutions

1. (c); Option (c) is the appropriate choice. Refer to paragraph 1 where it


is given innovation, progressive disruption and the effective use of
intellectual property are the new tools of the economy this century.
For this the author has given examples of Israel and Singapore. In
the 21st century, they are clear leaders on their strengths in the
fields of scientific, industrial and financial innovations. Hence the
lessons for emerging markets are clear — the need to innovate and
compete globally.

2. (d); Option (d) is the suitable choice. Refer to paragraph 3 where the
text is quoted as “In an emerging environment of protectionism
and the rise of the anti-globalization movement, the need to
innovate and compete will become even more important.”

3. (d); Only option (d) is the correct choice. Refer to last paragraph where
it is given that over the past decade, clinical research, especially
clinical trials, has moved away from India to countries like China
owing to policy decisions. Reversing this will not only help India to
innovate and compete with the rest of the world but also create
jobs across the value chain. Furthermore, it is also given that
“taking meaningful steps to be in step with international best
practices, remove barriers and enhance predictability in the
patents regimes will help grow the innovation paradigm in health
access, pharmaceutical developments and medical device
manufacturing in India.”

4. (e); Option (e) is the correct choice. The answer can be referred from
last paragraph where it is given that structural reforms and
increased funding of regulatory agencies are a must in order to
grow the innovation paradigm in health access, pharmaceutical
developments and medical device manufacturing in India.

5. (d); Option (d) is the most suitable choice. It is the only statement
which is false among all of the given options. Instead this option
can be corrected as advanced western economies with low single-
digit growth rates are resorting to protectionist measures.
18 Adda247 Publications For any detail, mail us at
Publications@adda247.com
Cracker Book for Bank (IBPS | SBI | RRB PO | Clerk) Mains Exams

6. (c); Disruption means disturbance or problems which interrupt an


event, activity, or process.
Commotion means disturbance.
Turmoil means a violent disturbance.
Cede means yield, surrender.
Diminution means decrease, reduction.

7. (d); Propel means drive or push something forwards.


Impel means drive, force, or urge (someone) to do something.
Stifle means restrain (a reaction) or stop oneself acting on (an
emotion).
Hinder means make it difficult for (someone) to do something or
for (something) to happen.

8. (b); Prudent means acting with or showing care and thought for the
future.
Chary means characterized by great caution and wariness.
Sagacious means wise or shrewd.
Didactic means instructive.
Bugged means to annoy or irritate.

9. (b); Innovate means make changes in something established, especially


by introducing new methods, ideas, or products.
Commence means begin.
Pioneer means develop or be the first to use or apply
Stagnate means cease developing.
Impede means delay or prevent (someone or something) by
obstructing them; hinder

10. (a); Embark means begin (a course of action).


Cease means to stop.
Culminate means terminate with.
Desolate means to abandon.
Concord
means harmony, agreement.

19 Adda247 Publications For any detail, mail us at


Publications@adda247.com
Cracker Book for Bank (IBPS | SBI | RRB PO | Clerk) Mains Exams

11. (c); Option A is invalid becomes the passage talks about the “supply of
capital” not “external capital” (first sentence of the passage).
Option B can be inferred from the ninth sentence of the first
paragraph.

12. (d); Refer to 1st paragraph of the passage, “Indian capital followed
where it was in touch ……………….. Indian proportion from 1900”,
hence option (d) is the correct choice for the given question.

13. (e); Refer the fifth sentence onwards of the second paragraph.

14. (e); Only option (e) is correct as no concrete evidence is given in the
passage regarding the statement.

15. (c); Refer towards the last part of the first paragraph

16. (b); Refer the first sentence of the second last paragraph “When a
culture is in a state of disintegration or transition the freedom of
the artist increases-but the question of subject matter becomes
problematic for him: he, himself, has to choose for society.”

17. (c); Refer the first paragraph of the passage.

18. (a); Refer the first two sentences of the fifth paragraph “Thus, for a
painting to succeed it is essential that the painter and his public
agree about what is significant. The subject may have a personal
meaning for the painter or individual spectator; but there must also
be the possibility of their agreement on its general meaning.”

19. (a); Refer sixth paragraph for option (b), seventh paragraph for option
(c) and last few lines of the fifth paragraph for option (d).

20. (a); Refer the second last paragraph of the passage “he, himself, has to
choose for society……”.
20 Adda247 Publications For any detail, mail us at
Publications@adda247.com
Cracker Book for Bank (IBPS | SBI | RRB PO | Clerk) Mains Exams

21. (b); The first sentence of the second paragraph is about revelation
made by Israeli Prime Minister Benjamin Netanyahu regarding
Iranian nuclear weapons development which is the excuse Trump
needed to exit from the JCPOA.
Hence option (b) is the most appropriate choice to be made.
All the other options are not the correct explanations.

22. (c); Option (c) is the correct choice for the given question.
Given teeth means to make something powerful; to give something
a real effect.
Succour means assistance and support in times of hardship and
distress.

23. (b); From third paragraph of the passage, we can infer that the sentence
(b) is responsible for making the sanctions on Iran effective. It has
been mentioned that if Europe, along with Russia and China do not
cooperate with U.S. then sanctions regime will be ineffective.
Hence option (b) is the correct choice.
Refer the lines “If Europe, along with Russia and China, refuses to
play ball with the US, the sanctions regime will be ineffective.”

24. (e); All the given consequences are correct.


We can conclude from fourth paragraph that the sanctioning by
U.S. may lead to doubling down the nuclear weapons programme,
expansion of regional influence and regime collapse in Iran.
Hence option (e) is the most correct choice.
Refer the lines “The Trump administration may be secretly hoping
that sanctions will lead to a regime collapse in Iran. This is an idea
fraught with danger. A regime collapse risks a more hardline
section in Iran gaining power. The new rulers may choose to
double down on the nuclear weapons programme and expand
regional influence.”

21 Adda247 Publications For any detail, mail us at


Publications@adda247.com
Cracker Book for Bank (IBPS | SBI | RRB PO | Clerk) Mains Exams

25. (b); Refer the last paragraph of the passage in which it is clearly stated
that the reimposing of the sanctions will make Iran offer oil on
favourable terms to India. Hence only option (b) will be correct.
Refer the line “India has many other oil suppliers ready to move in
if imports from Iran get disrupted. Reimposition of sanctions may
even make Iran offer oil on favourable terms to India.”

26. (b); ‘has given’ is the correct phrase to be replaced that is delivering the
correct meaning to the sentence.
In this sentence, Trump’s efforts to exit the deal has been
mentioned. Hence, the sentence is talking about the incident of
present as used in the first part of the sentence ‘has been running’.
Hence ‘had’ will not be used.
‘Has been’ is used to express something that has begun in the past
and is continuing or progressing till present. In the sentence, a
complete action is mentioned that is Trump has already given his
impression to exit the deal. Therefore, ‘has’ will be used.
‘has been given’ used in the sentence is the passive voice which is
also incorrect.
Hence option (b) will be the most appropriate choice.

27. (e); Unravel means become undone.


Slump means sit, lean, or fall heavily and limply.
Disintegrate means break up into small parts as the result of
impact or decay.
Hence all the words are similar in meaning with unravel.

28. (e); Perforce means used to express necessity or inevitability.


None of the given words is similar in meaning with Perforce.
Fragility means the quality of being easily broken or damaged.
Collude means come to a secret understanding.
Predicament means a difficult, unpleasant situation.

22 Adda247 Publications For any detail, mail us at


Publications@adda247.com
Cracker Book for Bank (IBPS | SBI | RRB PO | Clerk) Mains Exams

Sedate means calm, dignified and unhurried.


29. (a); Fraught means causing or affected by anxiety or stress. Hence it has
opposite meaning as repose.
Meticulous means showing great attention or careful.
Gullible means easily persuaded to believe something.

30. (b); Abide means accept or act in accordance with (a rule, decision, or
recommendation).
Hence spurn and flout are opposite in meaning as ‘abide by’.
Nefarious means wicked or criminal.
Vaunt means boast about or praise.

23 Adda247 Publications For any detail, mail us at


Publications@adda247.com
Cracker Book for Bank (IBPS | SBI | RRB PO | Clerk) Mains Exams

1 Adda247 Publications For any detail, mail us at


Publications@adda247.com
Cracker Book for Bank (IBPS | SBI | RRB PO | Clerk) Mains Exams

Chapter

9 Starters
STUDY TIPS

In such questions gew sentences are provided followed by three


probable starters that may be used to form a single coherent sentence
using both the statements without altering the context of the sentences.
The best way to solve such questions is by understanding the relation
between both the sentences such as whether they are stating cause and
effect or both are effects or causes. Once the relation is derived one must
choose the grammatically correct starter to frame the sentence as their
answer choice. It should be kept in mind that the context of the sentences
is not amended.

Practice Exercise Based on New Pattern

Direction (1-30): Select the phrase/connector (STARTERS) from the given


three options which can be used to form a single sentence from the two
sentences given below, implying the same meaning as expressed in the
statement sentences.

1. (I) The new timed finishing feature will undoubtedly be the biggest
adjustment in playing FIFA.
(II) It gives the option to shoot/head the ball better by utilising a
double tap of circle in order to generate more power, swerve and
accuracy.
(i) The new timed finishing feature gives the…
(ii) The option to shoot/head…
(iii) The biggest adjustment…
(a) Only (II)
(b) Both (I) and (III)
(c) only (III)
(d) All (I), (II) and (III)
(e) None of these

2 Adda247 Publications For any detail, mail us at


Publications@adda247.com
Cracker Book for Bank (IBPS | SBI | RRB PO | Clerk) Mains Exams

2. (I) Medical student Josh Guild realized that infections continue to


remain problematic following surgery in the developing world.
(II) Nurses receive little training to prevent the infections post-surgery.
(i) The reason that infections continue…
(ii) Since nurses receive little training…
(iii) Due to the little training…
(a) Only (II)
(b) Both (I) and (III)
(c) only (III)
(d) All (I), (II) and (III)
(e) None of these

3. (I) While there is scope to improve the FRL’s operational aspects,


(II) More substantive changes to the framework should be approached
as part of a comprehensive plan.
(i) Realizing that while there…
(ii) In the view of the fact that there is…
(iii) In the knowledge of scope…
(a) Both (I) and (II)
(b) Both (I) and (III)
(c) only (III)
(d) All (I), (II) and (III)
(e) None of these

4. (I) CBSE schools have decided to cut short the Onam holidays to
compensate for the working days lost due to heavy rain.
(II) The Kerala State Commission for Protection of Child Rights has
criticized CBSE schools for their decision.
(i) Reviewing the decision of…
(ii) Owing to heavy rain, the decision of…
(iii) The decision of CBSE schools…
(a) Both (I) and (II)
(b) Only (iii)
(c) Both (II) and (III)
(d) All (I), (II) and (III)
(e) None of these
3 Adda247 Publications For any detail, mail us at
Publications@adda247.com
Cracker Book for Bank (IBPS | SBI | RRB PO | Clerk) Mains Exams

5. (I) A lot of contestants in every season, even the most memorable ones
can end up getting buried within the large franchise.
(II) A lot of contestants have done what they can to cement their place
in Bachelor Nation history.
(i) Nonetheless, a lot of contestants have done…
(ii) In contempt of a lot of contestants in every….
(iii) Even though a lot of contestants have…
(a) Both (I) and (III)
(b) Only (iii)
(c) Both (II) and (III)
(d) All (I), (II) and (III)
(e) None of these

6. (I) The surface of our planet appears to be firm and stationary.


(II) There is convincing evidence that the materials that form the solid
rock on the surface of the earth are moving.
(i) Even though the surface of our planet…
(ii) However, the evidence is convincing…
(iii) Notwithstanding that the surface…
(a) only (i)
(b) only (ii)
(c) both (i) and (iii)
(d) both (ii) and (iii)
(e) none of these

7. (I) the implementation rules of the program describe predefined


criteria for evaluation of projects.
(II) the quality of project proposals is evaluated by the independent
evaluators.
(i) In line with the implementation rules of…
(ii) In accordance with the implementation rules…
(iii) Complying the implementation rules…
(a) only (iii)
(b) only (ii)
(c) both (i) and (ii)
(d) both (ii) and (iii)
(e) All of these
4 Adda247 Publications For any detail, mail us at
Publications@adda247.com
Cracker Book for Bank (IBPS | SBI | RRB PO | Clerk) Mains Exams

8. (I) Theresa May won the general election in 2017.


(II) she has been forced to reshuffle her Cabinet a number of times
amid a number of high profile resignations.
(i) Since Theresa May won…
(ii) Owing to the fact that Theresa May…
(iii) While Theresa May won the…
(a) only (iii)
(b) only (ii)
(c) both (i) and (iii)
(d) both (i) and (ii)
(e) All of these

9. (I) It’s a case where there are significant adverse effects and an
offender who continually ignores the council’s requests.
(II) This has been a difficult and significant case that the council has
been working on for a number of years
(i) However, it’s a case where…
(ii) since an offender who…
(iii) Whereas this has been difficult…
(a) only (i)
(b) only (ii)
(c) both (i) and (iii)
(d) both (ii) and (iii)
(e) none of these

10. (I) Most of the players of Eagles received their rings during a
ceremony on Wednesday.
(II) The Eagles made sure Dorenbos was remembered and gave him a
ring on Thursday.
(i) Even though most of…
(ii) As the Eagles made sure…
(iii) While most of the player…
(a) only (i)
(b) only (ii)
(c) both (i) and (iii)
(d) both (ii) and (iii)
(e) none of these
5 Adda247 Publications For any detail, mail us at
Publications@adda247.com
Cracker Book for Bank (IBPS | SBI | RRB PO | Clerk) Mains Exams

11. (I) Trump’s desire to improve relations with Russia is analyzed by the
editorial.
(II) The editorial pointed out that Trump values strength and attaches
importance to military strength especially nuclear strength.
(i) Analysing Trump’s desire to…
(ii) The editorial analyzed…
(iii) Trump values strength and…
(a) Both (I) and (II)
(b) Both (I) and (III)
(c) only (III)
(d) All (I), (II) and (III)
(e) None of these

12.(I) The ministry has proposed to cap royalty payments in the use of
trade mark and brand names.
(II) The ministry has proposed to cap royalty payments at 1 per cent of
sales and 2 per cent of exports of an entity.
(i) With a view for trade mark to…
(ii) Proposed to cap royalty payments in the use of…
(iii) With regard to the use of…
(a) Both (I) and (II)
(b) Both (II) and (III)
(c) only (III)
(d) All (I), (II) and (III)
(e) None of these

13. (I) The key factor that is boosting the level of adoption for Network
Access Control solutions includes rising security issues.
(II) Security issues are rising due to unwanted devices or unauthorized
users that initiates network breaches.
(i) Owing to unwanted…
(ii) Because of unwanted devices…
(iii) Rising security issues owing to…
(a) Both (I) and (II)
(b) Both (II) and (III)
(c) only (III)
(d) All (I), (II) and (III)
(e) None of these
6 Adda247 Publications For any detail, mail us at
Publications@adda247.com
Cracker Book for Bank (IBPS | SBI | RRB PO | Clerk) Mains Exams

14. (I) Flipkart has been around for more than 10 years now and Amazon
for at least five.
(II) the government’s e-commerce policy is almost an afterthought.
(i) Given how Flipkart…
(ii) Owing to the fact that Flipkart…
(iii) Considering Flipkart has…
(a) Both (I) and (II)
(b) Both (II) and (III)
(c) only (III)
(d) All (I), (II) and (III)
(e) None of these

15.(I) the much-reviled education system, introduced by the British has


many shortcomings.
(II) It had been successful in promoting upward mobility on a truly
large scale.
(i) Despite its many shortcomings…
(ii) However, the much-reviled…
(iii) In spite the shortcomings of…
(a) Both (I) and (II)
(b) Both (I) and (III)
(c) only (III)
(d) All (I), (II) and (III)
(e) None of these

16. (1) India had in the past always hesitated to impose its will and
intentions on SAARC.
(2) India did not wish to be seen as the Big Brother of the region that
seeks to dominate smaller nations.
(I) As India did not wish to…
(II) Considering that India did not…
(III) Despite imposing its will and…
(a) Only (II)
(b) Both (I) and (II)
(c) Both (II) and (III)
(d) Only (I)
(e) All (I), (II) and (III)
7 Adda247 Publications For any detail, mail us at
Publications@adda247.com
Cracker Book for Bank (IBPS | SBI | RRB PO | Clerk) Mains Exams

17. (1) Oil prices back up to over US$70 per barrel.


(2) The economic winds are favorable for making the necessary tough
reforms for Malaysia, who is a net energy exporter.
(I) Seeing that oil prices back up…
(II) With oil prices back up to…
(III)Being a net energy exporter, oil prices…
(a) Only (I)
(b) Both (II) and (III)
(c) Both (I) and (II)
(d) Only (III)
(e) All (I), (II) and (III)

18. (1) The CISF has handled the security duties at some important
airports for quite a few years.
(2) The CISF has acquired much experience, and hopefully a degree of
expertise too, in the specialised task.
(I) Despite handling the security…
(II) Having handled the security duties…
(III) In a bid to acquire much experience…
(a) Only (I)
(b) Only (II)
(c) Both (I) and (III)
(d) Both (II) and (III)
(e) All (I), (II) and (III)

19. (1) We have more schools, more classrooms, more playgrounds, better
infrastructure, better facilities and services, more teachers, more
training, more students in the schools in keeping with the
parameters laid down in the Right to Education Act, 2009.
(2) We don’t have quality and class in our education system.
(I) Since we have more schools…
(II) Even though we have more schools…
(III) Despite having more schools….
(a) Only (I)
(b) Both (I) and (II)
(c) Only (III)
(d) Both (II) and (III)
(e) All (I), (II) and (III)
8 Adda247 Publications For any detail, mail us at
Publications@adda247.com
Cracker Book for Bank (IBPS | SBI | RRB PO | Clerk) Mains Exams

20. (1) The subjective interpretive reasoning cannot be formulated in


mathematical ways.
(2) It seems to me that logic should be unique and hence quantifiable.
(I) While the subjective interpretive…
(II) The formulation of mathematical and…
(III) That logic of mathematical ways…
(a) Only (I)
(b) Only (II)
(c) Both (I) and (III)
(d) Both (I) and (II)
(e) All (I), (II) and (III)

21. (I) There has been a legacy of stock market scams through the 1990s.
(II) Capital markets regulator Securities and Exchange Board of India’s
(Sebi’s) impulse has been to protect retail investors at all cost.
(i) Owing to a legacy…
(ii) Taking into account a legacy…
(iii) Considering a legacy of…
(a) Only (III)
(b) Only (I)
(c) Both (II) and (III)
(d) All (I), (II) and (III)
(e) None of these

22. (I) Companies were providing more innovative benefits to attract


employees.
(II) Employees fell back on bouquets or fruit baskets when they went
through a hard time.
(i)Even though companies were…
(ii) Despite the fact that companies…
(iii) Regardless of employees felling back on…
(a) Only (III)
(b) Only (I)
(c) Both (I) and (II)
(d) All (I), (II) and (III)
(e) None of these
9 Adda247 Publications For any detail, mail us at
Publications@adda247.com
Cracker Book for Bank (IBPS | SBI | RRB PO | Clerk) Mains Exams

23. (I) You may prefer any kind of coffee to drink, including the likes of
decaf or instant coffee, coffee-drinkers.
(II) Coffee drinkers who seem addicted to drinking several cups of
coffee a day fare better health-wise in the long run.
(i) Regardless of the kind of coffee you prefer…
(ii) Even though coffee drinkers who seem…
(iii) Irrespective of the kind of…
(a) Both (I) and (III)
(b) Only (III)
(c) Both (I) and (II)
(d) All (I), (II) and (III)
(e) None of these

24. (I) Make in India exposed weak links in what passed off as
manufacturing.
(II) The technology shift from 3G to 4G caught the Indian
manufacturers unprepared.
(i) As long as the technology shift…
(ii) While Make in India…
(iii) Although Make in India exposed…
(a) Both (I) and (III)
(b) Only (III)
(c) Both (II) and (III)
(d) All (I), (II) and (III)
(e) None of these

25. (I) The Insolvency and Bankruptcy Code (IBC) strengthens creditors’
rights, which can go a long way in resolving the current bad loans
crisis in banking.
(II) It lends transparency and predictability to the resolution process
itself.
(i) The Insolvency and Bankruptcy Code (IBC) not only…
(ii) Lending transparency and predictability…
(iii) Along with strengthening creditors’ rights…
(a) Both (I) and (III)
(b) Only (III)
(c) Both (II) and (III)
(d) All (I), (II) and (III)
(e) None of these
10 Adda247 Publications For any detail, mail us at
Publications@adda247.com
Cracker Book for Bank (IBPS | SBI | RRB PO | Clerk) Mains Exams

26. (I) Vigorous governmental intervention into curbing the lynching


menace is sought in an incisive order.
(II) the Supreme Court has struck a powerful blow which serves as a
model for the political class to follow suit.
(i) In an incisive order that sought…
(ii) A model for the political class…
(iii) Curbing and lynching menace…
(a) Only (I) is correct
(b) Only (II) is correct
(c) Both (I) and (III) are correct
(d) Both (II) and (III) are correct
(e) All are correct

27. (I) Police implicate innocents in crimes or are unable to stop crimes
from happening and courts take an inordinately long time to
deliver justice.
(II) This is when the seeds of vigilantism are sown.
(i) The seeds of vigilantism are sown after police implicate…
(ii) While police implicate innocents…
(iii) when police implicate…
(a) Only (I) is correct
(b) Only (III) is correct
(c) Both (I) and (III) are correct
(d) Both (II) and (III) are correct
(e) All are correct

28. (I) Schnatter remains on the board and still owns about 30% of the
company’s shares.
(II) Schnatter resigned as chairman this month and no longer holds a
formal management role.
(i) Ensuring Schnatter resigned as…
(ii) Though Schnatter resigned as chairman…
(iii) Despite the fact Schnatter resigned as chairman…
(a) Only (I) is correct
(b) Only (III) is correct
(c) Both (I) and (III) are correct
(d) Both (II) and (III) are correct
(e) All are correct
11 Adda247 Publications For any detail, mail us at
Publications@adda247.com
Cracker Book for Bank (IBPS | SBI | RRB PO | Clerk) Mains Exams

29. (I) The district consolidation of a number of Manitoba Hydro buildings


occurred a few years ago.
(II) The response time to power outages and other interruptions has
improved by an average of 10 minutes in the region.
(i) Since the district consolidation…
(ii) Owing to the district…
(iii) Due to the district consolidation…
(a) Only (I) is correct
(b) Only (III) is correct
(c) Both (I) and (III) are correct
(d) Both (II) and (III) are correct
(e) All are correct

30. (I) There is a criticism of the changes to the RTI Act that the Centre
wants to bring need nuance.
(II) The underlying concern of those protesting the changes is merited.
(i) Even though the underlying concern…
(ii) Though the criticism of the…
(iii) Although the criticism with the changes…
(a) Only (I) is correct
(b) Only (II) is correct
(c) Both (I) and (III) are correct
(d) Both (II) and (III) are correct
(e) All are correct

Solutions

1. (b); Both the starters (i) and (iii) can be used to frame a meaningful
sentence without altering the exact meaning of the given sentences.
However, it is not possible to construct a contextual sentence using
the second starter as it would alter the intended meaning. Hence
option (b) is the correct choice.
(i) “The new timed finishing feature that gives the option to
shoot/head the ball better by utilising a double tap of circle in
order to generate more power, swerve and accuracy will
undoubtedly be the biggest adjustment in playing FIFA”.
12 Adda247 Publications For any detail, mail us at
Publications@adda247.com
Cracker Book for Bank (IBPS | SBI | RRB PO | Clerk) Mains Exams

(iii) “The biggest adjustment in playing FIFA will undoubtedly be


the new timed finishing feature which basically gives the
option to shoot/head the ball better by utilising a double tap
of circle in order to generate more power, swerve and
accuracy”.

2. (e); All the three starters can be used to frame a meaningful sentence
without altering the intended meaning of the given sentences.
Hence option (e) is the correct choice.
(i) The reason that infections continue to remain problematic
following surgery in the developing world is that nurses
receive little training to prevent them, medical student Josh
Guild realized.
(ii) Since nurses receive little training to prevent infections, the
infections continue to remain problematic following surgery
in the developing world, medical student Josh Guild realized.
(iii) Due to the little training received by the nurses to prevent the
infections post-surgery, the infections continue to remain
problematic following surgery in the developing world,
medical student Josh Guild realized.

3. (a); Both the starters (i) and (ii) can be used to frame a meaningful
sentence without altering the exact meaning of the given sentences.
However, it is not possible to construct a contextual sentence using
the third starter as it would alter the intended meaning. Hence
option (a) is the correct choice.
(i) “Realizing that while there is scope to improve the FRL’s
operational aspects, more substantive changes to the
framework should be approached as part of a comprehensive
plan”.
(ii) “In the view of the fact that there is scope to improve the FRL’s
operational aspects, more substantive changes to the
framework should be approached as part of a comprehensive
plan”.
13 Adda247 Publications For any detail, mail us at
Publications@adda247.com
Cracker Book for Bank (IBPS | SBI | RRB PO | Clerk) Mains Exams

4. (c); Both the starters (ii) and (iii) can be used to frame a meaningful
sentence without altering the exact meaning of the given sentences.
However, it is not possible to construct a contextual sentence using
the first starter as it would alter the intended meaning. Hence
option (c) is the correct choice.
(ii) Owing to heavy rain, the decision of CBSE schools to cut short
the Onam holidays to compensate for the working days lost
due to heavy rains has received criticism from the Kerala State
Commission for Protection of Child Rights.
(iii) The decision of CBSE schools to cut short the Onam holidays
to compensate for the working days lost owing to heavy rain
has drawn criticism from the Kerala State Commission for
Protection of Child Rights.

5. (b); Only (iii) can be used to frame a meaningful sentence without


altering the exact meaning of the given sentences. However, it is
not possible to construct a contextual sentence using the first and
second starter as it would alter the intended meaning. Hence
option (b) is the correct choice.
(iii) “Even though a lot of contestants have done what they can to
cement their place in Bachelor Nation history, a lot of them in
every season, even the most memorable ones can end up
getting buried within the large franchise.”

6. (c); Both the starters (i) and (iii) can be used to frame a meaningful
sentence without altering the exact meaning of the given sentences.
However, it is not possible to construct a contextual sentence using
the second starter as it would alter the intended meaning. Hence
option (c) is the correct choice.
(i) Even though the surface of our planet appears to be firm and
stationary, there is convincing evidence that the materials that
form the solid rock on the surface of the earth are moving.
(iii) Notwithstanding that the surface of our planet appears to be
firm and stationary, there is convincing evidence that the
materials that form the solid rock on the surface of the earth
are moving.

14 Adda247 Publications For any detail, mail us at


Publications@adda247.com
Cracker Book for Bank (IBPS | SBI | RRB PO | Clerk) Mains Exams

7. (e); All the three starters can be used to frame a meaningful sentence
without altering the intended meaning of the given sentences.
Hence option (e) is the correct choice.
(i) In line with the implementation rules of the program, the
quality of project proposals is evaluated by the independent
evaluators based on predefined criteria.
(ii) In accordance with the implementation rules of the program,
the quality of project proposals is evaluated by the
independent evaluators based on predefined criteria.
(iii) Complying with the implementation rules of the program
which describe the predefined criteria for evaluation, the
quality of project proposals is evaluated by the independent
evaluators.

8. (d); Both the starters (i) and (ii) can be used to frame a meaningful
sentence without altering the exact meaning of the given sentences.
However, it is not possible to construct a contextual sentence using
the second starter as it would alter the intended meaning. Hence
option (d) is the correct choice.
(i) Since Theresa May won the general election in 2017 she has
been forced to reshuffle her Cabinet a number of times amid a
number of high profile resignations.
(ii) Owing to the fact that Theresa May won the general election in
2017 she has been forced to reshuffle her Cabinet a number of
times amid a number of high profile resignations.

9. (e); Both the given sentences are individual coherent sentences,


independent of each other. Hence, none of the given starters can be
used to form a single coherent sentence without altering the
meaning of the sentence. Therefore, option (e) is the most suitable
answer choice.

10. (a); Among the three starters, only (I) can give a meaningful sentence
without altering the intended meaning of the given two statements.
The other two options will give an inappropriate meaning to the
sentence. Hence option (a) is the correct choice.
15 Adda247 Publications For any detail, mail us at
Publications@adda247.com
Cracker Book for Bank (IBPS | SBI | RRB PO | Clerk) Mains Exams

(i) Even though most of the players received their rings during a
ceremony on Wednesday, the Eagles made sure Dorenbos was
remembered and gave him a ring on Thursday.

11. (a); Both the starters (i) and (ii) can be used to frame a meaningful
sentence without altering the exact meaning of the given sentences.
However, it is not possible to construct a contextual sentence using
the third starter as it would alter the intended meaning. Hence
option (a) is the correct choice.
(i) Analysing Trump’s desire to improve relations with Russia,
the editorial pointed out that Trump values strength and
attaches importance to military strength especially nuclear
strength.
(ii) The editorial analyzed Trump’s desire to improve relations
with Russia and pointed out that Trump values strength and
attaches importance to military strength especially nuclear
strength.

12. (c); Only the (iii) starter can be used to frame a meaningful sentence
without altering the exact meaning of the given sentences.
However, it is not possible to construct a contextual sentence using
the first and second starters as it would alter the intended
meaning. Hence option (c) is the correct choice.
(iii) With regard to the use of the trade mark and brand names, the
ministry has proposed to cap royalty payments at 1 per cent of
sales and 2 per cent of exports of an entity.
13. (d); All the three starters can be used to frame a meaningful sentence
without altering the intended meaning of the given sentences.
Hence option (d) is the correct choice.
(i) Owing to unwanted devices or unauthorized users that
initiates network breaches, security issues are rising which is
the key factor in boosting the level of adoption for network
access control solutions.
(ii) Because of unwanted devices or unauthorized users that
initiates network breaches, security issues are rising which is
the key factor in boosting the level of adoption for network
access control solutions.
16 Adda247 Publications For any detail, mail us at
Publications@adda247.com
Cracker Book for Bank (IBPS | SBI | RRB PO | Clerk) Mains Exams

(iii) Rising security issues owing to unwanted devices or


unauthorized users that initiate network breaches is the key
factor that is boosting the level of adoption for Network
Access Control solutions.

14. (d); All the three starters can be used to frame a meaningful sentence
without altering the intended meaning of the given sentences.
Hence option (d) is the correct choice.
(i) Given how Flipkart has been around for more than 10 years
now and Amazon for at least five, the government’s e-
commerce policy is almost an afterthought.
(ii) Owing to the fact that Flipkart has been around for more than
10 years now and Amazon for at least five, the government’s
e-commerce policy is almost an afterthought.
(iii) Considering Flipkart has been around for more than 10 years
now and Amazon for at least five, the government’s e-
commerce policy is almost an afterthought.

15. (a); Both the starters (i) and (ii) can be used to frame a meaningful
sentence without altering the exact meaning of the given sentences.
However, it is not possible to construct a grammatically correct
sentence using the third starter as “in spite” should be followed by
the preposition “of”. Hence option (a) is the correct choice.
(i) Despite its many shortcomings, the much reviled-education
system, introduced by the British, had been successful in
promoting upward mobility on a truly large scale.
(ii) However, the much reviled- education system, introduced by
the British has many shortcomings, it had been successful in
promoting upward mobility on a truly large scale.

16. (b); Both the starters (I) and (II) can be used to frame a
meaningful sentence respectively without altering the exact
meaning of the given sentences. However, it is not possible to
construct a contextual sentence using the third starter as it would
alter the intended meaning. Hence option (b) is the correct choice.
(I) As India did not wish to be seen as the Big Brother of the
region that seeks to dominate smaller nations, it had in the
past always hesitated to impose its will and intentions on
SAARC.
17 Adda247 Publications For any detail, mail us at
Publications@adda247.com
Cracker Book for Bank (IBPS | SBI | RRB PO | Clerk) Mains Exams

(II) Considering that India did not wish to be seen as the Big
Brother of the region that seeks to dominate smaller nations,
it had in the past always hesitated to impose its will and
intentions on SAARC.

17. (c); Both the starters (I) and (II) can be used to frame a
meaningful sentence respectively without altering the exact
meaning of the given sentences. However, it is not possible to
construct a contextual sentence using the third starter as it would
alter the intended meaning. Hence option (c) is the correct choice.
(I) Seeing that oil prices back up to over US$70 per barrel, the
economic winds are favorable for making the necessary tough
reforms for Malaysia, who is a net energy exporter.
(II) With oil prices back up to over US$70 per barrel, the economic
winds are favorable for making the necessary tough reforms
for Malaysia, who is a net energy exporter.

18. (b); Among the three starters, only (II) can give a meaningful sentence
without altering the intended meaning of the given two statements.
The other two options will give inappropriate meanings to the
sentence. Hence option (b) is the correct choice.
(II) Having handled the security duties at some important airports
for quite a few years, the CISF has acquired much experience,
and hopefully a degree of expertise too, in the specialised task.

19. (d); Both the starters (II) and (III) can be used to frame a
meaningful sentence respectively without altering the exact
meaning of the given sentences. However, it is not possible to
construct a contextual sentence using the first starter as it would
alter the intended meaning. Hence option (d) is the correct choice.
(II) Even though we have more schools, more classrooms, more
playgrounds, better infrastructure, better facilities and
services, more teachers, more training, more students in the
schools in keeping with the parameters laid down in the Right
to Education Act, 2009, we don’t have quality and class in our
education system.
18 Adda247 Publications For any detail, mail us at
Publications@adda247.com
Cracker Book for Bank (IBPS | SBI | RRB PO | Clerk) Mains Exams

(III) Despite having more schools, more classrooms, more


playgrounds, better infrastructure, better facilities and
services, more teachers, more training, more students in the
schools in keeping with the parameters laid down in the Right
to Education Act, 2009, we don’t have quality and class in our
education system.

20. (a); Among the three starters, only (I) can give a meaningful sentence
without altering the intended meaning of the given two statements.
The other two options will give inappropriate meanings to the
sentence. Hence option (a) is the correct choice.
(I) While the subjective interpretive reasoning cannot be
formulated in mathematical ways, it seems to me that logic
should be unique and hence quantifiable.

21. (d); All the three starters can be used to frame a meaningful sentence
without altering the intended meaning of the given sentences.
Hence option (d) is the correct choice.
(i) “Owing to a legacy of stock market scams through the 1990s,
capital markets regulator Securities and Exchange Board of
India’s (Sebi’s) impulse has been to protect retail investors at
all cost.”
(ii) “Taking into account a legacy of stock market scams through
the 1990s, capital markets regulator Securities and Exchange
Board of India’s (Sebi’s) impulse has been to protect retail
investors at all cost.”
(iii) “Considering a legacy of stock market scams through the
1990s, capital markets regulator Securities and Exchange
Board of India’s (Sebi’s) impulse has been to protect retail
investors at all cost.”

22. (c); Both the starters (i) and (ii) can be used to frame a meaningful
sentence without altering the exact meaning of the given sentences.
However, it is not possible to construct a contextual sentence using
the third starter as it would alter the intended meaning. Hence
option (c) is the correct choice.

19 Adda247 Publications For any detail, mail us at


Publications@adda247.com
Cracker Book for Bank (IBPS | SBI | RRB PO | Clerk) Mains Exams

(i) “Even though companies were providing more innovative


benefits to attract employees, they still fell back on bouquets
or fruit baskets when workers went through a hard time”.
(ii) “Despite the fact that companies were providing more
innovative benefits to attract employees, they still fell back on
bouquets or fruit baskets when workers went through a hard
time”.

23. (a); Both the starters (i) and (iii) can be used to frame a meaningful
sentence without altering the exact meaning of the given sentences.
However, it is not possible to construct a contextual sentence using
the second starter as it would alter the intended meaning. Hence
option (a) is the correct choice.
(i) ”Regardless of the kind of coffee you prefer to drink, including
the likes of decaf or instant coffee, coffee-drinkers who seem
to be addicted to drinking several cups a day fare better
health-wise in the long run.”
(iii) Irrespective of the kind of coffee you prefer to drink, including
the likes of decaf or instant coffee, coffee-drinkers who seem
to be addicted to drinking several cups a day fare better
health-wise in the long run.”

24. (c); Both the starters (ii) and (iii) can be used to frame a meaningful
sentence without altering the exact meaning of the given sentences.
However, it is not possible to construct a contextual sentence using
the first starter as it would alter the intended meaning. Hence
option (c) is the correct choice.
(ii) While Make in India exposed weak links in what passed off as
manufacturing, the technology shift from 3G to 4G caught the
Indian manufacturers unprepared.
(iii) Although Make in India exposed weak links in what passed off
as manufacturing, the technology shift from 3G to 4G caught
the Indian manufacturers unprepared.

25. (a); Both the starters (i) and (iii) can be used to frame a meaningful
sentence without altering the exact meaning of the given sentences.
However, it is not possible to construct a contextual sentence using
the second starter as it would alter the intended meaning. Hence
option (a) is the correct choice.
20 Adda247 Publications For any detail, mail us at
Publications@adda247.com
Cracker Book for Bank (IBPS | SBI | RRB PO | Clerk) Mains Exams

(i) “The Insolvency and Bankruptcy Code (IBC) not only


strengthens creditors’ rights, which can go a long way in
resolving the current bad loans crisis in banking, but also
lends transparency and predictability to the resolution
process itself”.
(iii) “Along with strengthening creditors’ rights which can go a
long way in resolving the current bad loans crisis in banking,
the Insolvency and Bankruptcy Code also lends transparency
and predictability to the resolution process itself”.

26. (a); Among the three starters, only (i) can give a meaningful sentence
without altering the intended meaning of the given two statements.
The other two options will give an inappropriate meaning to the
sentence. Hence option (a) is the correct choice.
(i) In an incisive order that sought vigorous governmental
intervention into curbing the lynching menace, the Supreme
Court has struck a powerful blow which serves as a model for
the political class to follow suit.

27. (c); Both the starters (i) and (iii) can be used to frame a meaningful
sentence without altering the exact meaning of the given sentences.
However, it is not possible to construct a contextual sentence using
the second starter as it would alter the intended meaning. Hence
option (c) is the correct choice.
(i) The seeds of vigilantism are sown after police implicate
innocents in crimes or are unable to stop crimes from
happening and courts take an inordinately long time to deliver
justice.
(iii) When police implicate innocents in crimes or are unable to
stop crimes from happening and courts take an inordinately
long time to deliver justice, the seeds of vigilantism are sown.

28. (d); Both the starters (ii) and (iii) can be used to frame a meaningful
sentence without altering the exact meaning of the given sentences.
However, it is not possible to construct a contextual sentence using
the second starter as it would alter the intended meaning. Hence
option (d) is the correct choice.
21 Adda247 Publications For any detail, mail us at
Publications@adda247.com
Cracker Book for Bank (IBPS | SBI | RRB PO | Clerk) Mains Exams

(ii) Though Schnatter resigned as chairman this month and no


longer holds a formal management role, he remains on the
board and still owns about 30% of the company’s shares.
(iii) Despite the fact Schnatter resigned as chairman this month
and no longer holds a formal management role, he remains on
the board and still owns about 30% of the company’s shares.

29. (e); All the three starters can be used to frame a meaningful sentence
without altering the meanings of the given two sentences. Hence
option (e) is the correct choice.
(i) Since the district consolidation of a number of Manitoba
Hydro buildings occurred a few years ago, the response time
to power outages and other interruptions has improved by an
average of 10 minutes in the region.
(ii) Owing to the district consolidation of a number of Manitoba
Hydro buildings occurred a few years ago, the response time
to power outages and other interruptions has improved by an
average of 10 minutes in the region.
(iii) Due to the district consolidation of a number of Manitoba
Hydro buildings occurred a few years ago, the response time
to power outages and other interruptions has improved by an
average of 10 minutes in the region

30. (b); Among the three starters, only (ii) can give a meaningful sentence
without altering the intended meaning of the given two statements.
The other two options will give an inappropriate meaning to the
sentence. Hence option (b) is the correct choice.
(ii) Though the criticism of the changes to the RTI Act that the
Centre wants to bring need nuance, the underlying concern of
those protesting the changes is merited.

22 Adda247 Publications For any detail, mail us at


Publications@adda247.com
Cracker Book for Bank (IBPS | SBI | RRB PO | Clerk) Mains Exams

1 Adda247 Publications For any detail, mail us at


Publications@adda247.com
Cracker Book for Bank (IBPS | SBI | RRB PO | Clerk) Mains Exams

Chapter

10 Coherent Paragraphs
STUDY TIPS

In coherent paragraphs, a passage is split into sentences along with a


sentence which was not the part of the passage originally. We have to
figure out the sentence which is not conforming to the theme of the
passage. The understanding of the subject and a particular view point
regarding it is must because the same tone is generally used in a passage.
For example, if a passage talks about the adversities of the farmer and one
of the sentence is describing the boom in the agricultural sector then that
sentence would be the odd one out.

Practice Exercise Based on New Pattern

Directions (1-30): In the following questions few sentences are provided.


Identify and mark the sentence which may fail to become the part of the
paragraph coherently. In questions where “none of these” is an option and
all the sentences are meaningful and logical with the context of the
paragraph mark (e) i.e., “none of these” as your answer choice.

1. (a) Man has always pondered on nature and made endeavours to learn
from its principles.
(b) Blockchain has the potential to disrupt nearly every industry,
starting from financial institutions to manufacturing, education and
power.
(c) Aeroplanes were inspired by birds, bullet trains by the kingfisher,
sonar by dolphins, and robots were designed to mimic humans
themselves.
(d) One such example that has been inspired by microbes and is
proving to be a solution for numerous problems is the distributed
ledger technology (DLT), popularly known as blockchain
(e) We often tend to look up to nature to find solutions for simple yet
complex problems.

2 Adda247 Publications For any detail, mail us at


Publications@adda247.com
Cracker Book for Bank (IBPS | SBI | RRB PO | Clerk) Mains Exams

2. (a) Economies with a developed agrarian structure, such as the US and


China, have recently established private extension services for
farmers who produce on a commercially-viable and enduring scale.
(b) Although agriculture contributes only around 17% to the country’s
GDP, nearly half of India’s labour force (47%) works in the agrarian
sector.
(c) And amongst those living in rural areas, nearly 69% of the
workforce operates in the agricultural sector (according to Census
2011).
(d) With a predominantly rural backbone of the agrarian economy,
efforts to improve agricultural labour productivity are needed to
raise the living conditions of the majority of the population.
(e) A working paper by ICRIER demonstrates this specific
requirement, and analyses the importance of agricultural research
and education, along with extension and training.

3. (a) Petrol prices in India have two components, one is due to global
crude oil prices, and one is due to taxes by the Indian government.
(b) Such sharp increases in fuel prices choke economic growth, stoke
inflation, hurt consumers and domestic budgets.
(c) Retail prices of petrol and diesel were increased for the ninth
straight day, pushing them to record levels despite international
crude price being nowhere near the level it was four years ago.
(d) In Delhi, retail petrol and diesel prices were Rs 76.87 a litre and Rs
68.08 a litre respectively.
(e) none of these

4. (a) Traditional social norms have conditioned social conduct for


generations and these are coded into the mundane minutiae of
language and everyday practice.
(b) State patronage of violation of laws and democratic norms is at the
core of impunity.
(c) The promise of democracy is to eradicate such social norms
hierarchies and offer equality of opportunity, if not of outcomes.
(d) Traditional social norms in the country have been hierarchical and
those at the bottom rungs got a raw deal.
(e) None of these
3 Adda247 Publications For any detail, mail us at
Publications@adda247.com
Cracker Book for Bank (IBPS | SBI | RRB PO | Clerk) Mains Exams

5. (a) When the desire for a job is fulfilled, it is replaced by the desire for
a promotion.
(b) If your bank balance is in lakhs, the next desire will be for it to be in
crores.
(c) That which can never be satiated is a desire. When a desire gets
fulfilled – it gives birth to many more desires.
(d) When the desire for a house is fulfilled, you will want a bigger
house, better furniture, even a bigger car.
(e) None of these

6. (a) Rivers have been the lifeline of all civilisations; no wonder they are
considered sacred across cultures.
(b) When a seeker can honour the sanctity of the environment, then he
can experience the same sanctity in his own mind.
(c) Sanskrit has a beautiful word for water, apa and the word aptha
means ‘that which is born out of water’. Aptha also means ‘the
dearest one’. So, it’s the same word for both ‘the dear one’ and
‘water’.
(d) 60-70% of our body is made up of the water element. You need
water vapour in the atmosphere; you need certain amount of
humidity in the atmosphere. And you need water to drink as a fluid.
(e) In India, the Ganga symbolises knowledge, Yamuna was known for
love stories, Narmada stood for bhakti, knowledge and logic,
Saraswati for brilliance and architecture, and India got its name
from the Sindhu.

7. (a) Private hospitals use modern and constantly updated medical


infrastructure with advanced technology that needs upgrades.
(b) While their revenues might have increased, importantly, overall
profitability margins are rather low. There needs to be a fine
balance between world class healthcare and affordability.
(c) What patients should understand is that provision of high-standard
ICU care in India is rather expensive if all international norms are
adhered to, but still is markedly lower than that in developed
countries, for example the US.
(d) The Health Department has issued show cause notice to three
private medical colleges for violating the fee structure stipulated by
the Fee Committee appointed by the government.
(e) None of these
4 Adda247 Publications For any detail, mail us at
Publications@adda247.com
Cracker Book for Bank (IBPS | SBI | RRB PO | Clerk) Mains Exams

8. (a) It is painful to see the present condition of education. In the name


of education, all sorts of slaveries are nourished.
(b) The root causes for the ugliness and crippling of humanity are
hidden in our education.
(c) The present education has cut man off from nature and has not
been able to give birth to any refinement, to any culture.
(d) This education system has given birth to unnatural life. Every
generation goes on imposing this unnaturalness on the next
generation.
(e) None of these

9. (a) Children learn from adult role models. When they see adults
expressing prejudice and bigotry, they think that is the way to be.
Our children are innocent and pure at heart, but they learn from
our example.
(b) If we teach them to lie, they will lie. If we teach them to steal, they
will steal. If we teach them to hate, they will hate.
(c) The teacher has been an instrument for many types of
exploitations. He is instrumental in spreading many diseases.
(d) Instead, if we teach them to be loving, truthful and kind, they will
be loving truthful and kind.
(e) From the moment they are born, they are being influenced by the
environment. They are learning to speak what they hear, and are
learning to act in the manner they see the adults around them
acting.

10. (a) While faith is an essential ingredient of any religion, Easter for
Christians is not just a matter of faith in an extraordinary event that
took place eons ago.
(b) The economic impact of Christmas has grown steadily over the past
few centuries in many regions of the world.
(c) Easter is the celebration of the living presence of Jesus Christ in the
life of the world, the Church and of individuals.
(d) Easter also called Pascha (Greek, Latin) or Resurrection Sunday, is
a festival and holiday celebrating the resurrection of Jesus from the
dead.
(e) Mother Teresa found the suffering Jesus in the poorest of the poor
and drew strength from his resurrection celebrated as Easter to do
all she did.
5 Adda247 Publications For any detail, mail us at
Publications@adda247.com
Cracker Book for Bank (IBPS | SBI | RRB PO | Clerk) Mains Exams

11. (a) Witness Uber’s bowing to the #DeleteUber social media campaign
last year. Part of it is to do with the #MeToo movement.
(b) Social Media can bring out the worst in the public by enabling risk-
free mob mentality and witch hunts.
(c) But by the same token, it can allow the public to directly punish
businesses and business leaders for actions it disagrees with in a
manner that simply wasn’t possible before.
(d) Negative market sentiments claimed Uber co-founder Travis
Kalanick’s scalp last year.
(e) None of these

12. (a) Others maintain that the increase is not enough, that the
government has not delivered on its promise of announcing MSPs
that are 50% over cost, as had been recommended by the National
Commission on Farmers (Swaminathan Commission).
(b) Some analysts believe that the increase has been excessive, that it
will push up inflation, both directly and also indirectly via the fiscal
burden of higher subsidies.
(c) Cost of production is only one of several considerations factored
into the determination of MSPs, such as the estimated demand-
supply balance, global prices, etc.
(d) The recent increase in the minimum support prices (MSP) for
major kharif crops has reignited the debate about food price policy.
(e) None of these

13. (a) Since state governments are increasingly raising resources from
the bond market, higher issuance can complicate fiscal
management.
(b) The share of market borrowing in the financing of fiscal deficit is
expected to top 90% in the current year, compared with about 61%
in 2015-16.
(c) This will enable higher investment and help attain higher
sustainable growth.
(d) The maturity profile of state government bonds shows that
redemption pressure has started increasing since the last fiscal.
(e) none of these
6 Adda247 Publications For any detail, mail us at
Publications@adda247.com
Cracker Book for Bank (IBPS | SBI | RRB PO | Clerk) Mains Exams

14. (a) As the Lok Sabha debates the vote of no confidence,


representatives of farmers from across the country will be
marching outside Parliament under the banner of All India Kisan
Sangharsh Coordination Committee (AIKSCC), an umbrella body of
201 farmer organisations.
(b) Farmers have already passed a vote of no confidence against this
government.
(c) Far from helping the farmers, this government has actually harmed
them in their hour of crisis. This is a strong indictment, backed by
solid evidence.
(d) Further, the government has effectively bypassed this law in the
land acquisitions done by central agencies like the National
Highways Authority of India (NHAI) and has also allowed State
governments to nullify the provisions benefitting the land-owning
farmers.
(e) None of these

15. (a) Already they are present in the form of thermocol, dust and in
some cases, they are now used in making fatty oils.
(b) Government authorities have to understand the importance of the
order of Plastic Ban in a nutshell – UP is the largest and the most
populated state, plastic is in use in every corner of the state, even in
the remotest area. It’s been in use for more than 30 years in
different forms.
(c) In winters, poor burn plastic to get some warmth. Gases which emit
from this burning are poisonous and directly affect our lungs. In a
nutshell, it is the most dangerous man-made material in this world.
(d) This is the most dangerous aspect of it, because this oil goes
directly to the consumer leading to health disaster in many ways.
(e) None of these

16. (a) State-sponsored Jewish immigration, the “law of return” applicable


only to Jews, and the denial of the right to return to Palestinian
refugees and their descendants were all products of this latter urge.
(b) Israel has always struggled to present itself as a typical nation state
that belongs to all Israelis regardless of race or religion but also as
the homeland of the Jewish people no matter where they live. (c)
7 Adda247 Publications For any detail, mail us at
Publications@adda247.com
Cracker Book for Bank (IBPS | SBI | RRB PO | Clerk) Mains Exams

The dilemma was exacerbated because of the continued occupation


of Palestinian territories since 1967 that has led to official
projections that soon there will be as many Arabs as Jews in the
land between the Jordan River and the Mediterranean Sea.
(d) This will likely not merely impact their security as an ethnic group
but also reduce the clout of the pro-Israel lobby in the American
political system.
(e) None of these

17. (a) The only constant is fast-changing consumer preferences, based on


ease of experience, for which companies continuously compete.
(b) The new big companies too may have uncertain and short lives,
until replaced by others.
(c) There is no fear of failure—global experience shows that around
90% of start-ups fail.
(d) The last two or three decades of “disruptive innovations” have seen
famous companies, like Kodak and dozens of other majors going
under to become history, and the gale of new technologies
sweeping aside many well-known giants.
(e) None of these.

18. (a) Banks had the lure of CASA deposits (current account and savings
account), and assumed that since these funds remained constant in
proportion over the longer term, it actually provided the asset
liability management (ALM) congruence.
(b) This led to higher exposures to infra project and other heavy
investment projects, which got associated with non-performing
assets (NPAs).
(c) The proposed rule says that all such companies that have a rating
of AA and above will perforce have to borrow 25% of incremental
requirement from the bond market.
(d) The Indian financial system has evolved over the last two decades
or so, which involved the conversion of development finance
institutions (DFIs) into universals banks.
(e) None of these

8 Adda247 Publications For any detail, mail us at


Publications@adda247.com
Cracker Book for Bank (IBPS | SBI | RRB PO | Clerk) Mains Exams

19. (a) It is amusing to see that in a world where we know so much, where
technology is so advanced that varied information is available to a
person in split seconds, we still have not been able to discover the
mystery of the human brain and what exactly is mind and its full
effects on the body.
(b) The paradigm shift towards treating psychiatry as an important
and valuable field is slow, yet the tides are turning in its favor.
(c) Most medical scientists and pharmaceutical companies are
researching on the underlying pathology of mental diseases and
treating it.
(d) The problem is that this trend of over-specialization has led to an
undetermined amount of doctors consulting on one patient case.
(e) None of these

20. (a) They file an online application, which would be reviewed by the
facilitation councils, and is also visible to the concerned central
ministries, state governments and central public sector enterprises
(CPSEs) for proactive actions.
(b) This facility is gradually gaining popularity, proving to be an asset
for the aggrieved MSMEs.
(c) The data in cases reported on the MSME Samadhan Portal can be
seen in the accompanying table.
(d) The ministry of MSME has launched a specialized portal for
facilitating MSMEs to report cases of the delayed payment of bills
to draw the attention of their buyers, the facilitation councils and
the ministry of MSME.
(e) None of these.

21. (a) The Armed Forces of some countries have become smaller in
number, cost-effective, more lethal and civilianised.
(b) The right to form associations or unions is a fundamental right
under Part III of our Constitution.
However, it is not available to every Indian.
(c) Article 33 of the Constitution gives Parliament the power to modify
the fundamental rights of the members of the Armed Forces,

9 Adda247 Publications For any detail, mail us at


Publications@adda247.com
Cracker Book for Bank (IBPS | SBI | RRB PO | Clerk) Mains Exams

intelligence services, and those employed in the telecommunication


systems of these organisations ‘for ensuring the proper discharge
of their duties’ and ‘for the maintenance of discipline among them’.
(d) The legislations governing the Armed Forces and central police
forces restrict the members’ fundamental right to form a trade
union, to attend or address any political demonstration and to
communicate with the press.
(e) None of these

22. (a) PM Narendra Modi did well to declare that he was not scared to be
seen in the company of corporate chieftains.
(b) The selection process of particular industrialists for vital projects
such as building warplanes and aid-funded power plants in other
countries should be transparent.
(c) This false discourse must be abandoned and those who create
wealth, produce goods, generate jobs, pay taxes and, in these days
of mandated corporate social responsibility, even shoulder many of
the governance tasks that rightfully are the responsibility of the
state, accorded the respect they are due.
(d) This comes not a day too soon. With bad loans and flighty
industrialists dominating the news, a narrative has once again been
building up in which people who run companies are cast as villains.
(e) none of these

23. (a) Monetary policy, however, is about more than just the current
situation: the signaling function of policy is forward looking and is
articulated to anchor inflationary expectations at least a year
ahead.
(b) Our own view is that the uncertainty in the current economic
environment should tip the scale for a hold on the repo rate and a
“wait and watch” neutral stance.
(c) The RBI implements the monetary policy through open market
operations, bank rate policy, reserve system, credit control policy,
moral persuasion and through many other instruments.
(d) The Monetary Policy Committee (MPC) will have persuasive
arguments either for continuing with a second consecutive hike in
repo rate or remaining on hold.
(e) none of these
10 Adda247 Publications For any detail, mail us at
Publications@adda247.com
Cracker Book for Bank (IBPS | SBI | RRB PO | Clerk) Mains Exams

24. (a) The All India Survey of Higher Education 2016-17 showed
continued higher enrolment of women and Muslims in higher
education in the country.
(b) Women’s enrolment has shown a 45% increase, from 12 million in
2010-11 to 17.4 million in 2017-18.
(c) The number of Muslims who are enrolled in higher education
institutes has also increased by 37% over the past five years,
compared to the increase of 18% across all categories of students.
(d) In 2001, as per the Sachar report, while Muslims constituted just
6.3% of all graduates across the country—lower than the 8.2%
share for SCs/STs—amongst the 20-year-plus population of
Muslims, the community’s graduates accounted for just 3.6%.
(e) None of these

25. (a) Therefore, prior to finalising an e-commerce policy government


has to make sure that all laws dealing with data, privacy, and digital
transactions are consistent.
(b) Big business is increasingly supposed to be a policy innovator, in
addition to its more traditional role of innovator in technology and
in business practices.
(c) An important part of the draft overlaps the recommendations of
the BN Srikrishna panel on data protection.
(d) India’s national draft e-commerce policy which was unveiled on
Monday gives the first clear sense of the framework government
has in mind for the rapidly evolving sector.
(e) None of these

26. (a) Mr. Purnama, a Christian, narrowly lost the election after a hard-
fought campaign in which hardline groups accused him of
blasphemy.
(b) The electoral face-off between Mr. Widodo and Mr. Prabowo comes
also against the backdrop of a highly charged gubernatorial race in
Jakarta in 2017.
(c) The incumbent at that time, Basuki Tjahaja Purnama, had
succeeded Mr. Widodo when he became President in 2014 and was
considered his protégé.
(d) He was subsequently imprisoned on the charge.
(e) An economic nationalist, he has denied the accusations against him
of human rights violations.
11 Adda247 Publications For any detail, mail us at
Publications@adda247.com
Cracker Book for Bank (IBPS | SBI | RRB PO | Clerk) Mains Exams

27. (a) Sending out the signal that the government is finally recognising
the linkages between health care and economic development.
(b) Political parties have not yet made the right to health a campaign
issue, and the National Health Policy does not recommend such a
right since it cannot be fulfilled.
(c) Prime Minister Narendra Modi’s announcement on Independence
Day that Ayushman Bharat, or the National Health Protection
Mission, will be launched formally on September 25.
(d) Less ethical institutions have been found ordering unnecessary
treatments to claim insurance compensation.
(e) But there is increasing awareness that Ayushman Bharat is
unsustainable for a country of 1.3 billion people to rely on
household savings to pay for health care.

28. (a) India’s stock markets are booming, with the BSE Sensex touching
new highs.
(b) The regulation of securities markets has evolved over the last two
and a half decades since the setting up of the Securities and
Exchange Board of Indi, but it is still a work in progress.
(c) A panel headed by T.K. Viswanathan, a former Lok Sabha Secretary
General, has now submitted recommendations to curb illegal
practices in the stock market and ensure fair conduct among
investors.
(d) A key recommendation is that the stock market watchdog be
granted the power to act directly against “perpetrators of financial
statements fraud”.
(e) The newly SEBI Regulations, 2018 limits the number of
directorship of a person to eight entities.

29. (a) Given that e-cigarettes are a young technology, they said, it will
take time to uncover their ill-effects.
(b) Writing in the same journal issue, however, another group of
researchers advocated the precautionary principle.
(c) In the American Annual Review of Public Health, in January 2018 a
group of researchers argued that e-cigarettes must be viewed from
a “harm minimisation” perspective.
(d) Either way, conflicting evidence makes it a tough call for
policymakers.
12 Adda247 Publications For any detail, mail us at
Publications@adda247.com
Cracker Book for Bank (IBPS | SBI | RRB PO | Clerk) Mains Exams

(e) Given that combustible cigarettes are more noxious than electronic
ones, switching from the former to the latter can only help addicts,
they argued.

30. (a) Over the years though, the challenge also assumed physical
proportions.
(b) He played all four Majors and raised hopes of winning two of them,
the other being the British Open, where he eventually finished in a
tied sixth place.
(c) In a relative sense, 2018 has been a decent year for Woods.
(d) When personal issues led to his form dropping a decade ago, it
seemed as if what prevented him from returning to his winning
ways lay entirely in the mind.
(e) A recent trend is to produce woods and hybrids that can be
adjusted by the player to provide different lofts and other settings.

Solutions

1. (b); All the sentences except for sentence (b) are providing information
on how man seeks for answers from nature for all his problems.
The sentences form a coherent paragraph in the sequence of aced.
However, sentence (b) is providing information on blockchain.
Since, option (b) fails to relate coherently with the other sentences,
it becomes the most suitable answer choice.

2. (a); After carefully reading the sentences it should be noted that all the
sentences except sentence (a) are describing about the position of
agriculture sector in Indian economy. However, sentence (a) is
mentioning about the impact of agriculture on economies of
different developed countries. The sequence of the coherent
sentences is thus formed as bcde. Hence, option (a) becomes the
most viable answer choice.

3. (a); All the sentences except sentence (a) are describing about the
continuous rise of fuel prices. However, sentence (a) is describing
the components which formulates the fuel prices for the country.
13 Adda247 Publications For any detail, mail us at
Publications@adda247.com
Cracker Book for Bank (IBPS | SBI | RRB PO | Clerk) Mains Exams

Thus, the sentences can be coherently related in the sequence of


cdb. Therefore, option (a) becomes the most suitable answer
choice.

4. (b); Among all the given sentences, only sentence (b) fails to connect
with the other sentences coherently. The sentences can be logically
sequenced as adc, eliminating sentence (b) making it a coherent
paragraph. All the coherent sentences are describing about the role
of traditional social norms in everyday practice. However, sentence
(b) is describing about violation of state and democratic law.
Hence, as option (b) fails to find relevance, it becomes the most
suitable answer choice.

5. (e); All the sentences together form a coherent paragraph in the


sequence cadb and follow the similar theme of the growing desires
of an individual. Therefore, option (e) becomes the most viable
answer choice.

6. (b); All the given sentences except sentence (b) are providing
information on the importance of water in the past and present
scenario of the world. However, only sentence (b) is describing
about the honour and sanctity towards environment as a whole.
Therefore, it fails to adhere with the theme of the paragraph.
Hence, option (b) becomes the most suitable answer choice.

7. (d); After carefully reading the sentences it should be noted that all the
sentences except sentence (d) are describing about the functioning
and financial conditions of private hospitals. However, sentence (d)
is mentioning about the notice issued to three medical colleges by
Health Department. Hence, option (d) becomes the most viable
answer choice.

8. (e); All the sentences together form a coherent paragraph and follow
the similar theme of the downfall of education system and its
effects on the upcoming generations. Therefore, option (e)
becomes the most viable answer choice.

14 Adda247 Publications For any detail, mail us at


Publications@adda247.com
Cracker Book for Bank (IBPS | SBI | RRB PO | Clerk) Mains Exams

9. (c); All the sentences except sentence (c) are describing about the
relationship between the parent and the child. The upbringing
which a child receives through its parents. However, sentence (c)
is describing about the relationship of a child with its teacher.
Therefore, option (c) becomes the most suitable answer choice.

10. (b); After carefully reading the sentences it should be noted that all the
sentences except sentence (b) are describing about the auspicious
occasion of Easter and its influence on several saints inspiring them
to follow the philosophies of Jesus Christ. However, sentence (b) is
mentioning about the festival of Christmas and its impact on
several regions of the world. Hence, option (b) becomes the most
viable answer choice.

11. (d); The sentences can be sequenced as bca to form a coherent


paragraph which is describing the powers of social media platform
to make or break a business house. However, sentence (d) is
specifically mentioning about the case of Uber co-founder, which is
irrelevant in context of the coherent paragraph. Hence, option (d)
is the most suitable answer choice.

12. (c); The correct sequence of the sentences to form a coherent


paragraph is dba, which is describing about the opinions and
reactions of analysts on the increase on Minimum Support Prices
(MSP). However, sentence (c) is merely mentioning the factors
considered while determining of MSPs, thus, it fails to find the
relevance with the other sentences. Hence, option (c) is the most
viable answer choice.

13. (c); The sentences can be sequenced as abd to form a coherent


paragraph, which is describing about the stare government’s
borrowings through bond market which may cause difficulties in
fiscal management. However, sentence (c) mentions that a solution
[This] will enable higher investment, but the solution has not been
mentioned in the paragraph which would have acted as a linking
sentence to form sentence (c) as a part of coherent paragraph.
Hence, option (c) is the most feasible answer choice.

15 Adda247 Publications For any detail, mail us at


Publications@adda247.com
Cracker Book for Bank (IBPS | SBI | RRB PO | Clerk) Mains Exams

14. (d); The sequence of the paragraph after rearranging the coherent
sentences is abc. The paragraph is describing about the protest by
farmers against the government policies that fail to protect the
farmers in the hour of crisis. However, sentence (d) is not giving
any coherence to the paragraph. Therefore, option (d) becomes the
most viable answer choice.

15. (e); All the sentences together form a coherent paragraph in the
sequence of badc. The paragraph is describing about the law of
plastic ban implemented in the state of UP due to its harmful
effects. Therefore, option (e) becomes the most viable answer
choice.

16. (d); The sentences can be sequenced as bac in the form of coherent
paragraph as the paragraph is talking about Israel and its struggle
to present itself as a typical nation state but the new law is a
departure from the territorial principal and is likely to erode
Israel’s legitimacy as a nation state. However sentence (d) refers
about something that will impact their security and reduce the
power of pro –Israel lobby in relation to American political system.
This is not mentioned anywhere in the paragraph. Hence option (d)
is the most appropriate choice.

17. (c); The coherent paragraph can be formed from the given sentences in
the form of dba as the paragraph is based upon the troublesome
innovations at have happened in decades in famous companies and
which have swept many other technology giants and only the
consumer preference based on the ease of access is changing for
which companies continuously compete. On the other hand
sentence (c), refers about the fear of failure of the start-ups about
which nothing has been mentioned in any line.

18. (c); The sentences can be sequenced as dab except sentence (c) .All
other sentences are based on the conversion of Indian finance
institutions into universal banks and how banks had the lure of
CASA deposits and they provided the ALM congruence which led to
higher exposure to infra project which got associated with NPA’s.
The sentence (c) refers about the proposed rule to be applicable on

16 Adda247 Publications For any detail, mail us at


Publications@adda247.com
Cracker Book for Bank (IBPS | SBI | RRB PO | Clerk) Mains Exams

companies which imposes on them a necessity to borrow some


percent of incremental requirement from the bond market while
the reference about bond market has not been given anywhere in
the given passage.

19. (d); The sequence of the paragraph after rearranging the coherent
sentences is cba. The paragraph which is formed coherently is
describing about the medical scientists and pharmaceutical
companies and the underlying pathology of mental diseases and
way of treating them. However sentence (d) refers about ‘this’
trend of over specialization while over specialization has not even
been mentioned in the paragraph anywhere which has led to an
undetermined amount of doctors consulting on one patient case.

20. (e); All of the sentences are in contextual reference and they form a
coherent paragraph i.e. dabc .The coherent paragraph is based on
why timely payments to MSME’s are crucial. MSMEs have long been
facing a problem of delayed realization of their bills and
receivables, particularly from large corporate buyers and
government organizations.

21. (a); These sentences can be arranged in the sequence of bcd to form a
coherent paragraph describing about the rights of citizens
regarding the formations of unions and associations. However,
article 33 modifies this right for armed forces and intelligence
services. Sentence (a) fails to become the part of the coherent
paragraph as it is describing about features of armed forces in
some countries. Since, sentence (a) doesn’t find a logical place in
the coherent paragraph, option (a) becomes the most viable
answer choice.

22. (b); These sentences can be logically ties in the sequence of adc to form
a coherent paragraph. The rearranged coherent paragraph is
describing about the tarnished profile of the industrialists which
needs to be rectified in the eyes of the society. PM Narendra Modi is
helping to achieve this cause. Except for sentence (b) which is
describing about the way to select the industrialists, all the other
sentences find relevance with each other. Hence, option (b) is the
most suitable answer choice.

17 Adda247 Publications For any detail, mail us at


Publications@adda247.com
Cracker Book for Bank (IBPS | SBI | RRB PO | Clerk) Mains Exams

23. (c); A coherent paragraph can be formed by arranging the sentences in


the logical sequence as dba. The paragraph is describing about the
fixation of repo rates by monetary policy committee. It also
mentions about the risks involved in stabilization or hiking of the
repo rates. However, sentence (c) fails to find a logical place to
adhere the theme of the paragraph as, it is merely describing about
the instruments of monetary policy. Hence, option (c) is the most
viable answer choice.

24. (e); All the sentences in the sequence of abcd forms a coherent
paragraph providing a comparative data of higher education
among women, musims and SCs/STs. Since, all the statements find
relevance with each other, option (e) is the most suitable answer
choice.

25. (b); The sentences can be logically arranged in the sequence of dca. The
rearranged coherent paragraph thus provides information
regarding the draft formulated about the e-commerce policy.
However, sentence (b) is describing about the changes required by
big business to flourish which is irrelevant with the context of the
paragraph. Hence, option (b) is the most viable answer choice.

26. (e); The proper logical order of the above given sentences should be
‘bcad’ and the given option (e) doesn’t fit well in the paragraph. In
sentence (e) some person has been called as an ‘economic
nationalist’. So the predecessor of the sentence should have some
personality.

27. (d); the proper logical order of the above given sentences should be
‘cabe’ and the given option (d) doesn’t fit well in the paragraph. In
sentence (d) though they have talked about some institutions
ordering unnecessary treatments but there is no mention or
discussion of the Ayushman Bharat, or the National Health
Protection Mission.
18 Adda247 Publications For any detail, mail us at
Publications@adda247.com
Cracker Book for Bank (IBPS | SBI | RRB PO | Clerk) Mains Exams

28. (e); The proper logical order of the above given sentences should be
‘abcd’ and the given option (e) doesn’t fit well in the paragraph. In
sentence (e) it has been mentioned about SEBI’s regulation
regarding the number of directorship which is irrelevant in the
theme of the sentences.

29. (d); The proper logical order of the above given sentences should be
‘ceba’ and clearly the given option (d) doesn’t fit well in the
paragraph. In sentence (d) there is a discussion of some evidences
that are said to be conflicting but there is nowhere mention of such
evidences or conflicts.

30. (e); The proper logical order of the above given sentences should be
‘cbda’ and clearly the given option (e) doesn’t fits well in the
paragraph. Sentence (e) is describing about the improvements
made in the game for the benefits of the player and hence fails to
find relevance with other sentences.

19 Adda247 Publications For any detail, mail us at


Publications@adda247.com
Cracker Book for Bank (IBPS | SBI | RRB PO | Clerk) Mains Exams

1 Adda247 Publications For any detail, mail us at


Publications@adda247.com
Cracker Book for Bank (IBPS | SBI | RRB PO | Clerk) Mains Exams

Chapter

11 Error Correction Part-I


STUDY TIPS

Error correction is the most important part of banking English. There


are lot of changes in the pattern, but this is not something we should be
concerned about as the good understanding of the key concepts is the
only requirement we need to solve these questions. Aspirants are advised
to get a good grip on the topics like adverbs, articles, noun and numbers,
verbs, tenses etc. Some of the changes you can expect is that you can be
asked to find the correct part instead of an incorrect one.

Practice Exercise Based on new Pattern

Direction (1-5): The following questions consist of a sentence which is


divided into three parts which contain grammatical errors in one or more
than one part of the sentence. If there is an error in any part(s) of the
sentence, find the correct alternative(s) to replace that part(s) from the
three options given below each question to make the sentence
grammatically correct. If the given sentence is grammatically correct or
does not require any correction, choose (e) i.e. no error as your answer.

1. One of the most significant governance reform (I)/ undertaken by the


government in recent years (II)/ had been the Direct Benefits
Transfer (DBT) initiative. (III)
(I) One of the most significant governance reforms
(II) undertaken by the government in recent years
(III) has been the Direct Benefits Transfer (DBT) initiative
(a) only (I)
(b) only (II)
(c) only (III)
(d) Both (I) and (III)
(e) no error

2 Adda247 Publications For any detail, mail us at


Publications@adda247.com
Cracker Book for Bank (IBPS | SBI | RRB PO | Clerk) Mains Exams

2. The money she earned from each of the trips (I)/ across the desert
were devoted entirely to paying off the loan, but the money she
earned working (II)/ at the bank was set aside to purchase supplies
until the ranch started paying for itself. (III)
(I) The money she earned from each of the trips
(II) across the desert was devoted entirely to paying off the loan, but
the money she earned working
(III) at the bank was set aside to purchase supplies until the ranch
started paying for itself
(a) only (I)
(b) only (II)
(c) only (III)
(d) Both (II) and (III)
(e) no error

3. Once this ball gets rolling, it will speed up and, (I)/ because of it, we
will all wake up each morning (II)/ with a few extra spring in our step
and sparkle in our eye. (III)
(I) Once this ball gets rolling, it will speed up and
(II) because of it, we will all wake up each morning
(III) with a little extra spring in our step and sparkle in our eye
(a) only (I)
(b) Both (II) and (III)
(c) only (III)
(d) only (II)
(e) no error

4. They learned that Chilon was the very quiet man, (I)/ that he never
spoke to himself, and that he spent all his time in trying (II)/ to make
his country great, strong and happy. (III)
(I) They learned that Chilon was a very quiet man,
(II) That he never spoke about himself, and that he spent all his time
in trying
(III) To make his country great, strong and happy
(a) only (I)
(b) Both (II) and (III)
(c) only (III)
(d) Both (I) and (II)
(e) no error
3 Adda247 Publications For any detail, mail us at
Publications@adda247.com
Cracker Book for Bank (IBPS | SBI | RRB PO | Clerk) Mains Exams

5. Air India, which was offered (I)/ along with $5 billion of its debt, is
surviving (II)/ on taxpayer bailouts after losing money for years. (III)
(I) Air India, who were offered
(II) Along with $5 billion of the debt, is surviving
(III) On taxpayer’s bailouts after losing money for years
(a) only (I)
(b) Both (II) and (III)
(c) only (III)
(d) Both (I) and (II)
(e) no error

Directions (6-10): There are four sentences given in the following


question. Find the sentence(s) which is/are grammatically correct and
mark your answer choosing the best possible alternative among the five
given below each question. If all the sentences are correct, choose (e) as
your answer.

6. (I) The involvement of the teaching community is both imperative and


effective in limiting the progress of elementary mental health issues
into serious psychological hazards.
(II) The NAM was established and founded at the height of the Cold War
and during the collapse of the colonial system spurred by the
independence struggles of people belonging to Asia, Africa, Latin
America and various other regions of the world.
(III) The great advantage that India has at the time of Independence was
an organised political party and an efficient merit-based
bureaucracy.
(IV) The age-old tussle over scarce resources is nothing new and with
land increasingly becoming a scarce resource the world is
witnessing an increased struggle for it.
(a) Only (I) is correct
(b) Both (II) and (IV) are correct
(c) Both (I) and (III) are correct
(d) Only (I), (II) and (IV) are correct
(e) All are correct
4 Adda247 Publications For any detail, mail us at
Publications@adda247.com
Cracker Book for Bank (IBPS | SBI | RRB PO | Clerk) Mains Exams

7. (I) The long-term dynamics of development and historical experience


of other countries indicate that as an economy grows, the share of
agriculture in GDP and employment goes down.
(II) The recent Gujarat elections revealed the political implications of
the farmers’ movement that put the Union government under
tremendous pressure for taking up the farmers’ issue serious.
(III) It is well known that use of chemical fertilizers leads to a loss of
flavour and taste in food but now it is increasingly realised that in
addition this also causes a loss of nutrition quality and create new
health risks.
(IV) While the bulk of chemical fertilizers seek to provide some major
nutrients for quick growth, many badly needed micro nutrients are
neglected and depleted very rapidly.
(a) Only (I) is correct
(b) Both (I) and (IV) are correct
(c) Only (I), (III) and (IV) are correct
(d) Only (I), (II) and (III) are correct
(e) All are correct

8. (I) Maruti Suzuki and Toyota signed a Memorandum of Understanding


according to which both companies will be working together to
introduce electric cars by 2020.
(II) Japan’s Suzuki Motor will be setting up its lithium-ion battery joint
venture with Toshiba and Denso, investing Rs 1151 crores to set up
production facilities.
(III) The government should focus on developing the charging
infrastructure so that EVs can be introduced in the Indian market
with more ease.
(IV) Success has been difficult to achieve for electric cars, mainly
because of lack of recharge facilities and high acquisition cost.
(a) Only (II) is correct
(b) Only (I), (II) and (III) are correct
(c) Only (II), (III) and (IV) are correct
(d) Only (I), (III) and (IV) are correct
(e) All are correct
5 Adda247 Publications For any detail, mail us at
Publications@adda247.com
Cracker Book for Bank (IBPS | SBI | RRB PO | Clerk) Mains Exams

9. (I) The fear of conspiracies has been a potent force among the political
spectrum, from the colonial era to the present.
(II) The sharing of educational materials has significant potential to
improve the quality, transparency and accessibility of higher
education systems.
(III) The onset of the 4th industrial revolution is anticipated to be
disruptive and will change the dynamics of the workforce between
men or women.
(IV) A Deloitte report on gender imbalance in the workplace reported
that the last two decades have seen a decline in the percentage of
women in technology.
(a) Only (IV) is correct
(b) Both (I) and (II) are correct
(c) Both (II) and (IV) are correct
(d) Only (I), (III) and (IV) are correct
(e) All are correct

10.(I) While eating red meat that is rich in protein is linked with increased
risk of heart disease, protein from nuts and seeds could be
beneficial for the human heart.
(II) Eating cruciferous vegetables including broccoli, cauliflower,
cabbage and sprouts three or more times per day can prevent
hardening of neck arteries in elderly women and can also decrease
the risk of heart diseases.
(III) Schemes like NREGA which provide work near home while creating
livelihood and enhancing assets need significantly higher
allocations.
(IV) Availability of subsidised food and pensions have declined or even
stopped for some vulnerable people due to new Aadhaar linkages or
other changes.
(a) Only (I) is correct
(b) Both (III) and (IV) are correct
(c) Only (I), (II) and (III) are correct
(d) Only (II), (III) and (IV) are correct
(e) All are correct

6 Adda247 Publications For any detail, mail us at


Publications@adda247.com
Cracker Book for Bank (IBPS | SBI | RRB PO | Clerk) Mains Exams

Directions (11-15): The following question consists of a sentence, whose


certain parts are highlighted in bold. The highlighted portions are
grammatically correct. Each sentence is followed by the three expressions
which may or may not contain grammatical errors. Choose the most
appropriate alternative among the five options which states the incorrect
part(s) of the sentence. If the given sentence is grammatically correct and
contextually meaningful, choose option (e) i.e., “No error” as your answer.

11. The entire idea of separate civil aviation and combat air shows in
two cities are bound to damage India’s overall self-interest and the
aviation development programme.
(I) of separate civil aviation and combat air shows
(II) in two cities are bound to damage
(III) and the aviation development programme
(a) Only (I)
(b) Only (II)
(c) Both (I) and (III)
(d) Both (II) and (III)
(e) No error

12. Even though much city dwellers continue to have some links along
their rural homes, there is a growing feeling of urban elites getting
more alienated and aloof from urgent issues concerning the
majority of rural people.
(I) Even though much city dwellers
(II) along their rural homes, there is a growing feeling
(III) of urban elites getting more alienated and aloof from urgent
issues
(a) Only (I)
(b) Only (III)
(c) Both (I) and (II)
(d) All (I), (II) and (III)
(e) No error
7 Adda247 Publications For any detail, mail us at
Publications@adda247.com
Cracker Book for Bank (IBPS | SBI | RRB PO | Clerk) Mains Exams

13. Living Farms has contacted group of women, youth and elderly
citizens not only to work on healthy foods but also to seek their
involvement in wider issues of farming and the food system.
(I) Living Farms has contacted group of women
(II) on healthy foods but also to seek
(III) their involvement in wider issues of
(a) Only (I)
(b) Only (II)
(c) Both (II) and (III)
(d) Both (I) and (II)
(e) No error

14. Considering the volume of pork that is exported to China, higher


tariffs on US exports is bound to harm US producers and undermine
the rural economy.
(I) of pork that is exported to China
(II) higher tariffs on US exports is bound
(III) and undermine the rural economy
(a) Only (I)
(b) Only (II)
(c) Only (III)
(d) Both (I) and (II)
(e) No error

15. MSP gives sufficient remuneration to farmers, provides foodgrains


supply to buffer stocks and support the Food Security Programme
through PDS and other programmes.
(I) MSP gives sufficient remuneration
(II) Supply to buffer stocks and
(III) Support the Food Security Programme
(a) Only (I)
(b) Only (III)
(c) Both (I) and (II)
(d) Both (II) and (III)
(e) No error
8 Adda247 Publications For any detail, mail us at
Publications@adda247.com
Cracker Book for Bank (IBPS | SBI | RRB PO | Clerk) Mains Exams

Directions (16-20): In the questions given below, four sentences are given
which may or may not be grammatically correct and contextually
meaningful. Choose the most suitable alternative reflecting the sentences
which are grammatically correct and meaningful. If all the given sentences
are correct, choose option (e) i.e., ‘All are correct’ as your answer choice.

16.(I) None of Our Business are a new Audible production based on


stories of eavesdropping, hosted by comedians Gwynne Phillips and
Briana Templeton.
(II) This transaction has given rise to many discussion as to its
trustworthiness and the extent of its operation.
(III) A vivid but somewhat chauvinistic history of Bela's reign will be
found in Acsady's History of the Hungarian Realm.
(IV) The typical elements of a foreign culture are changed into the
typical elements of the local culture of translators.
(a) only (I) is correct
(b) only (IV) is correct
(c) Both (III) and (IV) are correct
(d) Both (I) and (II) are correct
(e) All are correct

17.(I) He is one of the soldiers who fights for their country.


(II) The Narendra Modi government has been accused of undermining
various constitutional institutions including the EC.
(III) There are several problems that political parties and counting
agents face while dealing with ballot papers.
(IV) Punjab, with the highest cropping intensity, irrigation coverage and
use of fertilisers is battling with increasing rural indebtedness.
(a) only (I) is correct
(b) only (IV) is correct
(c) Both (III) and (IV) are correct
(d) Only (II), (III) and (IV) are correct
(e) All are correct
9 Adda247 Publications For any detail, mail us at
Publications@adda247.com
Cracker Book for Bank (IBPS | SBI | RRB PO | Clerk) Mains Exams

18.(I) Reduced demand for farm labour, increased expenditure on medical


and education needs and expensive non-institutional credit have
pushed them into debt traps.
(II) Romans park their cars the way I had park if I would have just
spilled a beaker of hydrochloric acid on my lap.
(III) Umesh Yadav looks to perform to the best of his abilities whenever
given a chance instead of worrying over his on and off selection in
the Indian cricket team in the T20s.
(IV) As soon as I opened the front door than I smelled the distinctive
aroma of fresh coffee.
(a) only (I) is correct
(b) Both (I) and (III) are correct
(c) only (IV) is correct
(d) Only (I), (II) and (III) are correct
(e) All are correct

19.(I) Many tax experts argue that sales tax holidays don’t really boost
annual retail shopping overall, but simply shift the spending from
one time of the year to another.
(II) The statue would come up in the elevated land located at the
junction where the Museum-Vellayambalam road branches off to
Nandavanam.
(III) There was no lack of entertainment on the cruise ship, since there
were comedy shows as well as musical performances.
(IV) It is thus difficult to form a judgment as to what has most claim to
acceptance as the general law, and what may be regarded as local or
exceptional.
(a) only (I) is correct
(b) Both (I) and (III) are correct
(c) only (IV) is correct
(d) Only (I), (II) and (III) are correct
(e) All are correct

10 Adda247 Publications For any detail, mail us at


Publications@adda247.com
Cracker Book for Bank (IBPS | SBI | RRB PO | Clerk) Mains Exams

20.(I) A farmer’s dharma is to nurture produce, not to destroy it in anger


and clash of egos.
(II) With deposits in either of this scheme, you can not only increase
your savings but also grow your wealth as your deposits.
(III) an overarching theme is that “inadequate” data protection allows
digital ecosystem entities an advantage in the use of the data, as
compared to the user.
(IV) The close dependency of all mental operations on brain also tempts
them to the conclusion that brain is not only an organ, but the whole
organ of conscious mind.
(a) only (I) is correct
(b) Both (I) and (III) are correct
(c) only (IV) is correct
(d) Only (I), (III) and (IV) are correct
(e) All are correct

Directions (21-25): In the questions given below, a sentence is divided


into five parts, out of which certain parts are highlighted which may or may
not be grammatically correct. Choose the grammatically correct part out of
the highlighted parts of the sentence as your answer. If all the highlighted
parts are grammatically correct then choose (e) as your choice. If all the
given parts of the sentence are grammatically incorrect then choose (d).

21. The National Center for Education Information (a) / says a predicted
teacher (b) / shortage won't plan on (c) / because teachers who
left (d) / the profession are returning back. (e)
(a) B (b) C (c) E
(d) None of these (e) No error

22. Carroll gave an overview of (a) / each of the players the team has (b) /
signed during free agency (c) / and how they may fit in (d) / with
the team in 2018. (e)
(a) C (b) D (c) E
(d) None of these (e) No error
11 Adda247 Publications For any detail, mail us at
Publications@adda247.com
Cracker Book for Bank (IBPS | SBI | RRB PO | Clerk) Mains Exams

23. The ongoing synchronized cyclical upswing (a) / is good news, but
(b) / underneath these impressive growth (c) / is risk that is
perhaps (d) / not being adequately recognized. (e)
(a) A (b) C (c) D
(d) None of these (e) No error

24. Sona ordered (a) / her servant to bring (b) / her some hot water
(c) / as she has to (d) / take some medicine. (e)
(a) A (b) C (c) D
(d) None of these (e) No error

25. There are at least four broad reasons (a) / why policymakers
across (b) / the world should be worried(c) / about rising global
debt (d) / and its consequences. (e)
(a) A (b) D (c) E
(d) None of these (e) No error

Solutions

1. (d); The first and the third part of the sentence contain errors. To make
the first part grammatically correct, replace ‘reform’ with ‘reforms’.
It is to be noted that the general rule for the phrase ‘one of the’ is
“One of the + PLURAL NOUN + that/who etc. + SINGULAR/PLURAL
VERB”. Moreover, in the third part of the sentence, “had been”
should be replaced with “has been” since, “Had been” means
something began in the past, lasted for some time, then ended. This
is entirely in the past while, both “Has been” and “Have been” mean
something began in the past and has lasted into the present time.
Drawing a hint from the phrase “the government in recent years”, it
can be understood that a reform undertaken by the government
has been beneficial for (DBT) from past few years and still
continues to be beneficial. However, part (II) of the sentence is
from all grammatical errors. Hence, option (d) becomes the most
suitable answer choice.

12 Adda247 Publications For any detail, mail us at


Publications@adda247.com
Cracker Book for Bank (IBPS | SBI | RRB PO | Clerk) Mains Exams

2. (b); There is an error in the second part of the sentence. To make the
sentence grammatically correct, replace, the auxiliary verb ‘were’
with ‘was’, as the subject [money] associated with verb is an
uncountable noun. 'Money' is an uncountable noun. This is because
we cannot say, "1 money, 2 money, etc." It's true, that money can be
counted in the sense as, "1 rupee, 2 rupees, etc." But we are
counting 'rupees', not 'money'. Therefore, 'rupees' makes money
countable, so 'rupees' is a countable noun while ‘money’ is
uncountable. Moreover, it is to be noted that with an uncountable
noun the verb associated is always singular. However, part (I) and
(III) are devoid of all errors, hence, option (b) becomes the most
viable answer choice.

3. (c); The error lies in the third part of the sentence. It is to be noted that
(a) little and (a) few are quantifiers meaning ‘some’. Little and few
have negative meanings. They are used to mean ‘not as much as
may be expected or wished for’. ‘a little’ is used with singular
uncountable nouns while ‘a few’ with plural countable nouns. Since
the noun associated with the quantifier is an uncountable noun
[extra spring in our step and sparkle in our eye], the quantifier to
be used should be ‘a little’. However, part (I) and (II) are
grammatically correct therefore, option (c) becomes the most
viable answer choice.

4. (d); The error lies in part (I) and part (II) of the sentence. It is to be
noted that if the noun is modified by an adjective, an article is
placed before the adjective. Here the article which is used is “the”
which should be replaced by “a”. Moreover, to make the second
part of the sentence contextually correct, replace ‘to’ with ‘about’.
As part (III) of the sentence is error free, option (d) becomes the
most suitable answer choice

5. (e); All the parts of the given sentence are grammatically correct and
contextually meaningful hence, do not require any corrections.
Therefore, option (e) becomes the most viable answer choice.

13 Adda247 Publications For any detail, mail us at


Publications@adda247.com
Cracker Book for Bank (IBPS | SBI | RRB PO | Clerk) Mains Exams

6. (d); All the given sentences, except (III), are grammatically correct. In
the case of sentence (III), the verb “has” should be replaced by
“had” as the sentence is referring to past events [at the time of
Independence]. Hence option (d) is the correct choice.

7. (b); There are certain errors in both the second and the third sentences.
In the case of second statement, the adjective “serious” should be
replaced by the adverb “seriously” as the structure of the sentence
requires an adverb instead of an adjective. In the sentence (III), the
verb “create” used in the latter half of the sentence should be
replaced by its singular “creates” as the verbs “causes’ and
“creates” are used for the same singular subject. The other two
statements are both grammatically correct. Hence option (b) is the
correct choice.

8. (e); All the given sentences are both grammatically correct. Hence
option (e) is the correct choice.

9. (c); There are errors in both the statements (I) and (III). In the case of
first sentence, the preposition “among” should be replaced by
‘across” to make the sentence grammatically and contextually
feasible. It is to be noted that the preposition “among” implies
“indicating a division, choice, or differentiation involving three
or more participants”, while the preposition “across” implies
“from one side to the other of (a place, area, etc.)”. Thus, the
expression “across the political spectrum” gives a meaningful
sense to the sentence. In the case of third sentence, the conjunction
“or” should be replaced by “and” as the preposition “between”
preceding the conjunction indicates that the two nouns should be
connected by “and”. The two statements are both grammatically
correct. Hence option (c) is the correct choice.

10. (e); All the given sentences are both grammatically correct. Hence
option (e) is the correct choice.

14 Adda247 Publications For any detail, mail us at


Publications@adda247.com
Cracker Book for Bank (IBPS | SBI | RRB PO | Clerk) Mains Exams

11. (b); There is a grammatical error in only the part (II) of the sentence,
the verb “are” should be replaced by “is” as the subject of the verb
is singular [the entire idea]. The other two parts are both
grammatically correct and contextually meaningful. Hence option
(b) is the correct choice.

12. (c); There are grammatical errors in both the parts (I) and (II). In the
part (I), the determiner “much” should be replaced by “many” as
the noun it signifies is countable [city dwellers]. It is to be noted
that both ‘much’ and ‘many’ are determiners, and have the same or
similar meaning i.e., ‘a lot of’, or ‘in great quantities’, or ‘a great
amount’, but their usage differs. If a noun is an uncountable noun
(which is often in singular form), the ‘much’ determiner should be
used, while the determiner ‘many’ should be used with countable
nouns, or plural nouns. In the part (II) of the sentence, the
preposition “along” should be replaced by “with” as “to have some
links with their rural homes” is the correct expression in the
context of the sentence. The part (III) of the sentence doesn’t
require any correction. Hence option (c) is the correct expression.

13. (a); There is an error in only the part (I) of the sentence, the singular
form of the word “group” should be replaced by its plural “groups”
as the word is used for all three: women, youth and elderly citizens
i.e., group of women, group of youth and group of elderly
citizens. The other parts of the sentence are both grammatically
correct and contextually meaningful. Hence option (a) is the correct
choice.

14. (e); The given sentence is both grammatically correct and contextually
meaningful. Hence option (e) is the correct choice.

15. (b); There is an error in the part (III) of the sentence, the verb
“support” should be replaced by “supports” as the subject of the
verb is singular [MSP]. The other parts of the sentence are both
grammatically and contextually correct. Hence option (b) is the
correct choice.
15 Adda247 Publications For any detail, mail us at
Publications@adda247.com
Cracker Book for Bank (IBPS | SBI | RRB PO | Clerk) Mains Exams

16. (c); Among the given sentences, both (III) and (IV) are grammatically
correct. However, there are errors in (I) and (II) sentences. In
sentence (I) there is an error of subject verb agreement, ‘are’
should be replaced with ‘is’, since the subject “none of our
business” indicates individuality of each business.
Moreover, in sentence (II) “many discussion” should be replaced
with “much discussion” as with many “discussions” should be used.
The quantifier ‘much’ always takes singular noun with it. Hence,
option (c) is the most suitable answer choice,

17. (d); Among the given sentences, except for sentence (I) all are correct.
In sentence (I) it should be noted that the verb fight has to be plural
here. The reason is that the relative clause has to refer to soldiers,
not one. The fact that ‘their’ is used should already be considered
evidence of this. The correct grammatical structure of such
sentences is “One of the + PLURAL NOUN + that/who etc. +
SINGULAR/PLURAL VERB”. Since, sentence (I), (III) and (IV) are
correct, option (d) becomes the most viable answer choice.

18. (b); Among the given sentences, sentences (I) and (III) are
grammatically correct. However, there are errors in sentence (II)
and (IV). In sentence (II) the grammatical syntax of the sentence is
incorrect. Note that when using the third conditional, we use the
past perfect (i.e., had + past participle) in the if-clause. The modal
auxiliary (would, could, shoud, etc.) + have + past participle in the
main clause expresses the theoretical situation that could have
happened. Therefore, the correct sentence should be “Romans park
their cars the way I would park if I had just spilled a beaker of
hydrochloric acid on my lap.” Moreover, to correct sentence (IV),
omit “than” from the it, since words like “than”, “so”, “therefore”
stands superfluous in the sentences beginning with words/
phrases like “as soon as”, “because”, “since”, “so”, “for”, “as”. As,
sentences (I) and (III) are grammatically correct, option (b)
becomes the most viable answer choice.
16 Adda247 Publications For any detail, mail us at
Publications@adda247.com
Cracker Book for Bank (IBPS | SBI | RRB PO | Clerk) Mains Exams

19. (e); All the given sentences are grammatically correct and contextually
meaningful, therefore, option (e) is the most viable answer choice.

20. (d); All the given sentences are correct, except sentence (II). To make
the sentence grammatically correct, replace “this scheme” with
“these schemes”. It is to be noted that phrases like ‘one of’, ‘each of’,
‘either of’, ‘neither of’, is always associated with a plural noun or
pronoun. Since, all the other sentence is precise in their
grammatical context, option (d) becomes the most suitable answer
choice.

21. (a); Only the second part of the sentence or part (b) is grammatically
correct and follows the structure in the context of the sentence.
However, there are grammatical errors in both the parts (c) and
(e). In part (c), the usage of the phrase “plan on” is incorrect as it
means to prepare for something; to be ready for something; to
anticipate something. Thus, the correct phrase that should replace
the highlighted phrase should be [ shortage won't plan out] as the
phrase “plan out” means to think about and decide what you are
going to do or how you are going to do something. In part (e) , the
usage of “back” is superfluous as the word “return” itself means to
“come back” or “give back”. Hence, it should be removed to make
the sentence grammatically and contextually correct. Hence option
(a) is the correct choice.

22. (e); The given sentence is grammatically correct. All the highlighted
parts along with the ones not highlighted are both grammatically
correct and contextually meaningful. Hence option (e) is the correct
choice.

23. (a); Only the part (a) of the highlighted parts is grammatically correct.
There are grammatical errors in part (c) and (d) of the sentence. In
the part (c), the determiner “these” should be replaced with “this”
as a determiner ‘this’ is used to identify a specific person or thing
close at hand or being experienced, while ‘these’ is a plural form of
‘this’. Therefore, for the subject [impressive growth] the singular
17 Adda247 Publications For any detail, mail us at
Publications@adda247.com
Cracker Book for Bank (IBPS | SBI | RRB PO | Clerk) Mains Exams

form of the determiner i.e., ‘this’ is required. In the part (d), the
insertion of the article ‘a’ is required before ‘risk’ to give the
contextual sense to the sentence. Hence, option (a) is the correct
choice.

24. (e); The given sentence is grammatically correct. All the highlighted
parts along with the ones not highlighted are both grammatically
correct and contextually meaningful. Hence option (e) is the correct
choice.

25. (e); The given sentence is grammatically correct. All the highlighted
parts along with the ones not highlighted are both grammatically
correct and contextually meaningful. Hence option (e) is the correct
choice.

18 Adda247 Publications For any detail, mail us at


Publications@adda247.com
Cracker Book for Bank (IBPS | SBI | RRB PO | Clerk) Mains Exams

1 Adda247 Publications For any detail, mail us at


Publications@adda247.com
Cracker Book for Bank (IBPS | SBI | RRB PO | Clerk) Mains Exams

Chapter

12 Error Correction Part-II


STUDY TIPS

Error correction is the most important part of banking English. There


are lot of changes in the pattern, but this is not something we should be
concerned about as the good understanding of the key concepts is the
only requirement we need to solve these questions. Aspirants are advised
to get a good grip on the topics like adverbs, articles, noun and numbers,
verbs, tenses etc. Some of the changes you can expect is that you can be
asked to find the correct part instead of an incorrect one.

Practice Exercise Based on new Pattern

Directions (1-8): In the questions given below, a sentence is divided into


five parts, out of which certain parts are highlighted which may or may not
be grammatically correct. Choose the grammatically correct part out of the
highlighted parts of the sentence as your answer. If all the highlighted parts
are grammatically correct then choose (e) as your choice. If all the given
parts of the sentence are grammatically incorrect then choose (d).

1. The writer complains that(A)/ champions in League deal too much


damaging (B)/ the current meta, and they wish the game (C)/ could
return to seasons three and four, (D)/ when, apparently, people
didn't died as much. (E)
(a) B (b) D (c) E
(d)None of these (e)No error

2. Byron Bay’s photographer and artist Sean O'Shea (A)/ was enough
lucky to (B)/ capture the image (C)/ of a Migaloo the white whale
(D)/ as he passed by the North Coast. (E)
(a) B (b) D (c) E
(d) None of these (e) No error

2 Adda247 Publications For any detail, mail us at


Publications@adda247.com
Cracker Book for Bank (IBPS | SBI | RRB PO | Clerk) Mains Exams

3. Higney has a background (A)/in distribution and (B)/ came


across with a product (C)/ called The Bug Bite Thing when (D)/
she was working overseas at Denmark. (E)
(a) A (b) C (c) E
(d) None of these (e) No error

4. Just over a quarter - 27 per cent - of (A)/ people living in the


country have (B)/ confidence in their local police (C)/ force to
keep them safe, (D)/ according to the National Rural Crime
Survey.(E)
(a) A (b) B (c) D
(d) None of these (e) No error

5. It is important to view the entrants (A)/ in the digital world in


the context of (B)/ communications technology and artificial
intelligence (C)/ and their impact on human society, politics,
(D)/ the growing economic divide and the lifestyles of people. (E)
(a) A (b) C (c) D
(d) None of these (e) No error

6. Times such as ours when bankers (A)/ as well as promoters are


been disparaged (B)/ should not blind us (C)/ to the valuable
role that both play in (D)/ managing the risks involved with new
projects. (E)
(a) B (b) C (c) E
(d) none of these (e) no error

7. Tech giant Google on Wednesday said (A)/ it will offer an


accelerator programme (B)/ to help game developers in India
(C)/ and other South East Asian countries (D)/ scale their gaming
startups. (E)
(a) A (b) B (c) E
(d) none of these (e) no error

8. Even in the case of public sector banks, (A)/ there is constant talk
(B)/ about the Union government (C)/ wanting to merge some
banks (D)/ and have a few larger banks. (E)
(a) B (b) C (c) E
(d) none of these (e) no error
3 Adda247 Publications For any detail, mail us at
Publications@adda247.com
Cracker Book for Bank (IBPS | SBI | RRB PO | Clerk) Mains Exams

Directions (9-13): In the following question, certain parts of the sentence


are highlighted which may or may not contain grammatical errors. There
are five alternatives given below each sentence, one of which may replace
the existing highlighted parts to make the sentence grammatically correct
and contextually meaningful. Choose the most appropriate alternative as
your answer. If the given sentence is grammatically correct, and doesn’t
require any correction, choose option (e) i.e., “No correction required” as
your answer.

9. The central principle which should be guiding the Centre in


improving maternal and child nutrition is that early childhood is
being the foundation for the health and well-being of an individual.
(a) which should guide its, is a foundation of the health
(b) which should be guiding the, is the foundation for their health
(c) that should guide the, is the foundation for the health
(d) which should guide the, is the foundation of their health
(e) No correction required

10. The initiative appears to address the reform that the Ministry hopes
to bring about that will be of critical significance for higher
education in India.
(a) the address to reform that, to bringing about will be
(b) to address the reforms that, for bringing about that will be
(c) to addressing in the reforms with, to bring about should be
(d) to addressing the reforms, to bring about that would be
(e) no correction required

11. When the Centre recently launched POSHAN Abhiyaan, an integrator


that would be building capacity amongst nutrition workers, it
acknowledged that while official data show a reduction in some of the
depressing aspects of women and child health, the ground reality
was far from comfort.
(a) that will build capacity among nutrition workers, the ground
reality is far from comforting.
(b) that would build capacity for nutrition workers, the ground
reality is far comfort.
(c) which has built the capacity between nutrition workers, the
reality of the ground was far comforting.
4 Adda247 Publications For any detail, mail us at
Publications@adda247.com
Cracker Book for Bank (IBPS | SBI | RRB PO | Clerk) Mains Exams

(d) which built the capacity with nutrition workers, the ground
reality is not comfort.
(e) No correction required

12. A major highlight of the financial stability report is the central


bank’s finding that public sector banks (PSBs) are far more prone
to fraud than their private sector counterparts.
(a) are the central banks finding that, are far more than prone to
(b) is the central’s bank finding that, are more far than prone for
(c) is the finding of the central bank, were more fare prone to
(d) was the central bank to find that, are far prone for
(e) no correction required

13. The Urban Development Secretary in each State, who heads the
monitoring committee under the rules, should mandate to produce
a monthly report on how much plastic waste is collected, including
details of the types for chemical involvement, and the disposal
methods.
(a) should mandate to be produced, types of chemical involved in
(b) would be mandated in producing, the involvement of types of
chemicals
(c) should be mandated to produce, the types of chemicals involved
(d) may be mandated to produced, the types of chemicals involving
(e) no correction required

Directions (14-18): In the following question, a sentence is divided into


five different parts including two highlighted ones. The highlighted parts of
the sentence are grammatically correct and do not require any correction.
However, the remaining three parts may contain errors in one or more than
one parts of the sentence. Choose the best alternative among the five
options given below each sentence that determines the portions that
require correction to make the sentence grammatically correct.

14. The sell-off begun early (I)/ in the session after (II)/ Libya's National
Oil Company said it would (III)/ reopen ports which were closed
(IV)/ since late June. (V)
(a) Only (I) (b) Only (IV) (c) Both (I) and (IV)
(d) Both (I) and (II) (e) No error
5 Adda247 Publications For any detail, mail us at
Publications@adda247.com
Cracker Book for Bank (IBPS | SBI | RRB PO | Clerk) Mains Exams

15. Haj pilgrims are advised (I)/ for cutting down on outside activities
such as (II)/ shopping and making visits especially in the day time,
(III)/ apart from the ones organised by Tabung Haji (IV)/ while in
the Holy Land in view of the rather hot weather. (V)
(a) Only (II) (b) Only (V) (c) Both (III) and (V)
(d) Both (II) and (III) (e) No error

16. An employer wishes to procure (I)/ a background check from a


private company (II)/ he must first provide the job applicant (III)/
or employee with a written disclosure (IV)/ stating its intention to
conduct a background check. (V)
(a) Only (III) (b) Only (V) (c) Both (I) and (V)
(d) Both (I) and (III) (e) No error

17. Underweight people in India appears (I)/ to be at greater risk of


premature death (II)/ than that slightly overweight or even obese,
(III)/ India's first study to analyse causes of deaths (IV)/ among
people with and without diabetes has shown. (V)
(a) Only (III) (b) Only (V) (c) Both (III) and (V)
(d) Both (I) and (III) (e) No error

18. Indian cities do not have (I)/ the financial muscle to build (II)/ the
infrastructure needed (III)/ to sustain (IV)/ as engines of job
creation. (V)
(a) Only (III) (b) Only (V) (c) Both (III) and (V)
(d) Both (I) and (III) (e)No error

Directions (19-23): There are three sentences given in each question. Find
the sentence(s) which is/are grammatically incorrect and mark your
answer choosing the best possible alternative among the five options given
below each question. If all the sentences are incorrect, choose (e) as your
answer.

19. (I) The latest official consumer inflation numbers from Statistics South
Africa shows that bread and cereal prices, as well as the price of
fruit, fell by more 3% in the year to June.
(II) Accreditation is an important tool for quality improvement in
learning outcomes.
6 Adda247 Publications For any detail, mail us at
Publications@adda247.com
Cracker Book for Bank (IBPS | SBI | RRB PO | Clerk) Mains Exams

(III) The present system of affiliation has had a restrictive effect on the
quality of higher education.
(a) Only (I)
(b) Only (III)
(c) Both (I) and (III)
(d) Both (II) and (III)
(e) None is correct

20. (I) Existing literature models the spread of fake news in a social
network in one of two ways.
(II) From oral communication that connected men and women without
any help from technology, we have moved over the millennia into
the digital age.
(III)It is a cluster of old-fashioned cottage in a unique position on the
sides of a rocky cleft in the north coast; its main street resembles a
staircase which descends 400 ft.
(a) Only (I)
(b) Only (III)
(c) Both (I) and (III)
(d) Both (II) and (III)
(e) None is correct

21. (I) The public equities asset class has grown by leaps and bounds over
the past decade.
(II) The patient’s doctor would have all the patient’s clinical
information in their own clinical software on the desktop.
(III)Vaccine-related rebound effects are guiding research on other
diseases, particularly malaria.
(a) Only (I)
(b) Only (III)
(c) Both (I) and (III)
(d) All are correct
(e) None is correct
7 Adda247 Publications For any detail, mail us at
Publications@adda247.com
Cracker Book for Bank (IBPS | SBI | RRB PO | Clerk) Mains Exams

22. (I) Its safe to say that both the awareness and appetite of the Indian
retail investor has increased manifold.
(II) Unfortunately, teacher’s unions have shown a blatant disregard for
this duty of care, frequently engaging in union meetings and
unplanned strikes during school hours
(III)Congress belongs to each and every human beings of India
irrespective of caste, creed, language, religion etc.
(a) Only (I)
(b) Only (III)
(c) Both (I) and (III)
(d) All are correct
(e) None is correct

23. (I) Staff were reportedly told the business will make efforts to help
out-of-work employees transition into possible employment with
Coles
(II) The yuan has depreciated by over 5% since the beginning of June
and some people may be tempted to believe that China is doing this
to undo the damage being done by Trump’s tariffs.
(III) It is not simply the genes directly inherited from parents that
influence child behaviour but also the family environment created
by parents with certain genes.
(a) Only (I)
(b) Only (III)
(c) Both (I) and (III)
(d) All are correct
(e) None is correct

24. (I) I was pretty sure that he would support my views for changing the
old and static structure of our organization.
(ii) You would have certainly completed the work in the allotted time
had you planned it properly.
(iii)No variety of mango produced in our country is as tasty as this
variety.
(a) Only (i) is correct
(b) Both (ii) and (iii) are correct
(c) Both (i) and (ii) are correct
(d) None is correct
(e) All are correct
8 Adda247 Publications For any detail, mail us at
Publications@adda247.com
Cracker Book for Bank (IBPS | SBI | RRB PO | Clerk) Mains Exams

Directions (25-29): The following questions consist of a sentence which is


divided into five parts which contain grammatical errors in one or more
than one part of the sentence. The portion specified in bold suggests that it
is grammatically correct and does not require any correction. If there is an
error in rest part of the sentence, find the correct alternative among the five
options which indicates the part(s) of the sentence containing grammatical
error(s). If the given sentence is grammatically correct or does not require
any correction, choose (e), i.e., “No error” as your answer.
25. When victims understand (I)/ that a person's bizarre beliefs (II)/
may actually acquit them (III)/ from the crime it is another (IV)/
reason not to trust the justice system. (V)
(a) Both (I) and (III)
(b) Only (V)
(c) Both (I) and (V)
(d) only (IV)
(e) No error

26. The Board of Directors govern (I)/ the Brainerd Lakes Area
Community Foundation, conduct (II)/ development activities, approve
(III)/grant decisions and works to engage people, (IV)/ connect
resources and build community. (V)
(a) Both (I) and (III)
(b) Only (II)
(c) Only (I), (II) and (III)
(d) All (I), (II), (III) and (IV)
(e) No error

27. The junior is one of hundreds of (I)/ students in the area who have to
find (II)/ somewhere to store his belongings and somewhere to
sleep (III)/ during the short gap within leases (IV)/ ending late July
and starting 1 Aug. (V)
(a) Both (II) and (IV)
(b) Only (V)
(c) Both (I) and (V)
(d) only (I)
(e) No error
9 Adda247 Publications For any detail, mail us at
Publications@adda247.com
Cracker Book for Bank (IBPS | SBI | RRB PO | Clerk) Mains Exams

28. Properties with a view of the sea (I)/ can command prices up to 32
per cent (II)/ higher than a similar (III)/ property with no sea view,
(IV)/ a study has found. (V)
(a) Both (II) and (IV)
(b) Only (V)
(c) Both (I) and (V)
(d) only (I)
(e) No error

29. We have not been (I)/ given a chance to compete (II)/ in spite of we
(III)/ can supply the (IV)/ best cost-effective solution. (V)
(a) Both (I) and (II)
(b) Only (III)
(c) Both (I) and (V)
(d) only (I)
(e) No error

Direction (30): The following question consists of a sentence which is


divided into three parts which contain grammatical errors in one or more
than one part of the sentence. If there is an error in any part(s) of the
sentence, find the correct alternative(s) to replace that part(s) from the
three options given below each question to make the sentence
grammatically correct. If there is an error in any part of the sentence and
none of the alternatives is correct to replace that part, then choose (d), i.e.,
None of the (I), (II) and (III) as your answer. If the given sentence is
grammatically correct or does not require any correction, choose (e), i.e., No
correction required as your answer.

30. One in six Americans is expected to develop dementia, (I)/ and care
can be enough costly to force even(II)/ middle-class families into
poverty and onto the public payroll. (III)
(I) One in six American is expected to develop dementia,
(II) and care can be costly enough to force even
(III) middle-class families into poverty and public payroll.
(a) only (III)
(b) only (II)
(c) Both (I) and (II)
(d) None of (I), (II) and (III)
(e) no correction required
10 Adda247 Publications For any detail, mail us at
Publications@adda247.com
Cracker Book for Bank (IBPS | SBI | RRB PO | Clerk) Mains Exams

Solutions

1. (b); Only the fourth part of the sentence or part (D) is grammatically
correct and follows the structure in the context of the sentence.
However, there are grammatical errors in both the parts (B) and
(E). In part (B) of the sentence, it is to be noted that Too much
modifies a noun and a verb, and much too modifies an adjective or
adverb. Since, “damaging” is an adjective, it should be replaced with
its noun form i.e., “damage in” to make part (B) of the sentence
correct. Moreover, in order to correct part (E) replace “died” with
“die” since the auxiliary verb (didn’t) is marked for past tense, but
the main verb is not. It appears in its base form. As, only part (D)
among the highlighted parts is correct, option (b) becomes the
most suitable answer choice.

2. (c); Among the given highlighted parts, only part (E) is devoid of all
grammatical and contextual errors. However, parts (B) and (D)
contain grammatical errors. In part (B) it is to be noted that
whenever “enough” is used as an adverb of quantity, then the
adverb enough is used to modify an adjective. In this case, it goes
after the adjective it modifies. E.g. She is old enough to earn a living.
Therefore, “enough lucky” should be replaced by “lucky enough”.
Moreover, in part (D), since the photographer has taken the
photographs of specific whale named Migaloo, therefore, article “a”
should be omitted to make the sentence grammatically correct and
contextually meaningful. Hence, option (c) becomes the most viable
answer choice.

3. (a); Among the given highlighted parts, only part (A) is grammatically
and contextually accurate. However, parts (C) and (E) contain
grammatical errors. To correct part (C) of the sentence, omit “with”
from the highlighted phrase, as while using the phrasal verb “came
across”, the usage of preposition “with” is always avoided.
Moreover, in part (E) replace the preposition “at” with “in”.
Generally, at usually conveys a sense of position or location in

11 Adda247 Publications For any detail, mail us at


Publications@adda247.com
Cracker Book for Bank (IBPS | SBI | RRB PO | Clerk) Mains Exams

terms of a single, cohesive point which might be literal ("I'm at the


office") or figurative ("He's at the top of his profession"). ‘In’
usually conveys the sense of position or location in terms of an area
represented by limits or boundaries. These may similarly be literal
("I'm in the garden") or figurative ("His business is in financial
difficulties"). The other parts of the sentence are grammatically
correct, therefore, option (a) is the most viable answer choice.

4. (e); All the highlighted parts of the sentence are grammatically correct
and contextually meaningful and do not require any corrections.
We always have confidence in somebody. Since, all the other parts
of the sentence are correct, option (d) becomes the most viable
answer choice.

5. (e); The given sentence is grammatically correct. All the highlighted


parts along with the ones not highlighted are both grammatically
correct and contextually meaningful. Hence option (e) is the correct
choice.

6. (b); Among the highlighted parts, only part (C) is grammatically correct
and contextually relevant. It is to be noted in part (B), “been”
should be replaced by “being”, since, ‘been’ is the past particle of
‘be’ verb while ‘being’ is its present participle. The context of the
sentence is related with the present time. This can be understood
from part (A) [Times such as ours]. Therefore, ‘being’ should be
used. Moreover, part (E) of the sentence can be corrected by
replacing the preposition “with” by “in”. Hence, option (b) is the
most suitable answer choice

7. (e); The given sentence is grammatically correct. All the highlighted


parts along with the ones not highlighted are both grammatically
correct and contextually meaningful. Hence option (e) is the correct
choice.

8. (e); The given sentence is grammatically correct. All the highlighted


parts along with the ones not highlighted are both grammatically
correct and contextually meaningful. Hence option (e) is the correct
choice.
12 Adda247 Publications For any detail, mail us at
Publications@adda247.com
Cracker Book for Bank (IBPS | SBI | RRB PO | Clerk) Mains Exams

9. (c); ‘that should guide the, is the foundation for the health’ are the
correct phrases to be replaced.
The use of ‘which’ after ‘central principle’ is incorrect as ‘which’ is
used for introducing non- defining clauses whereas ‘that’ is used
for introducing defining clauses.
(A defining clause (also called an essential clause or a restrictive
clause) gives information essential to the meaning of the sentence.
Ex. My bike that has a broken seat is in the garage. It means that
there is at least one other bike.)
In second clause, Early Childhood is regarded as foundation for the
well- being of an individual. Here present tense will be used.
Hence option (c) is the correct choice.

10. (e) The given sentence with the highlighted phrases is grammatically
correct and contextually meaningful. Thus, it doesn’t require any
other alterations, hence, option (e) is the most suitable answer
choice.

11. (a); The most appropriate set of phrases that should replace the
incorrect phrases of the sentence is “that will build capacity among
nutrition workers, the ground reality is far from comforting”. It is
to be noted that “Would” is a past-tense form of will. If you are
writing about past events, you can use it to indicate something that
was in the future at that point in time, but is not necessarily in the
future right now. In other words, you use would to preserve the
future aspect when talking about the past. Therefore, would should
be replaced by “will”. Moreover, the second highlighted phrase
should be in the present tense, therefore, “was” should be replaced
by “is”. In addition to, to describe the quality of the “ground reality”
the adjective form of ‘comfort’ should be used. Thus, ‘comfort’
should be replaced by ‘comforting’, hence, option (a) is the most
viable answer choice.

12. (e); The given sentence with the highlighted phrases is grammatically
correct and contextually meaningful. Thus, it doesn’t require any
other alterations, hence, option (e) is the most suitable answer
choice.
13 Adda247 Publications For any detail, mail us at
Publications@adda247.com
Cracker Book for Bank (IBPS | SBI | RRB PO | Clerk) Mains Exams

13. (c); The most suitable phrases to replace the incorrect highlighted
phrases are “should be mandated to produce; the types of
chemicals involved”. All the other phrases are either
grammatically incorrect or contextually meaningless”. Hence,
option (c) is the most suitable answer choice.

14. (c); The error lies in Part (I) as well as in part (IV) of the sentence. It is
to be noted to make the first part of sentence grammatically
correct, replace “begun” with “began” as the sentence is in the past
tense. This can be understood from the highlighted part (III)
[…said]. Moreover, the usage of “since” in part (V) of the sentence
indicates the intervening period between (the time mentioned) and
the time under consideration, typically the present. Therefore, it
implies that the company would reopen ports which are still closed
from past June, hence “were” should be replaced with “had been” as
“Had been” means something began in the past [June], lasted for
some time, then ended [present]. Hence, option (c) is the most
viable answer choice.

15. (a); The error lies in part (II) of the sentence. It is to be noted that the
verb “advised” given in the first part of the sentence will always be
followed by “to + V1” structure, rather than “for +ing”. Therefore,
“for cutting” should be replaced by “to cut” in order to make the
sentence grammatically correct. All the other parts of the sentence
are in absolute grammatical syntax. Hence, option (a) is the most
viable answer choice.

16. (d); The error lies in parts (I) and (III) of the sentence. In order to make
the first part of the sentence error free, replace “wishes” by
“wishing” as the present participle form is used to describe the
continuous form of a verb. Moreover, in part (III) of the sentence,
‘he’ is superfluously used and thus shall omitted, since the subject
of the sentence has been already mentioned [Employer]. The other
parts of the sentence are grammatically correct, hence, option (d) is
the most suitable answer choice.
14 Adda247 Publications For any detail, mail us at
Publications@adda247.com
Cracker Book for Bank (IBPS | SBI | RRB PO | Clerk) Mains Exams

17. (d); The error lies in parts (I) and (III) of the sentence. It is to be noted
that If the collective noun [Underweight people] refers to the group
as a unit, then it takes a singular verb. If it refers to the individuals
in the group or the parts that make up the group, then the verb
should be plural. Here, the sentence is referring to the individuals
of the group, hence the verb [appears] should be in its plural form
[appear]. Moreover, in part (III) of the sentence “that” should be
replaced by “those”, as the comparison is made between the plural
subjects. However, “that” is used for the comparison between
singular subjects. Hence, option (d) is the most suitable answer
choice.

18. (e); All the parts of the given sentence are grammatically correct and do
not require any corrections and improvements. Therefore, option
(e) becomes the most suitable answer choice.

19. (a); Among the given sentences, Sentence (I) stands to be


grammatically incorrect. To make the sentence correct replace
“shows” by “show”, since the associated subject [official consumer
inflation numbers] of the verb is plural in nature. Thus, to comply
with the subject-verb agreement, the verb should be in its plural
form [show]. Both the other sentences are grammatically correct
and do not require any corrections, therefore, option (a) is the
most suitable answer choice.

20. (b); Among the given sentences, sentence (III) is grammatically


incorrect. To make the sentence correct replace “cottage” by
“cottages”, since the collective noun is formed with several
individual objects. Therefore, since together they are more than
one, “cottage” should be it its plural form. Both the other sentences
are grammatically correct and contextually meaningful. Hence,
option (b) is the most suitable answer choice.

21. (d); All the given sentences are grammatically correct and contextually
meaningful. Therefore, they do not require any corrections. Hence,
option (d) becomes the most viable answer choice.

22. (e); Among the given sentences, none of them is grammatically correct.
In sentence (I) ‘Its’ should be replaced with “It’s”, since It's is
always the abbreviation of "it is" as in "It's (= it is) a nice day, isn't
15 Adda247 Publications For any detail, mail us at
Publications@adda247.com
Cracker Book for Bank (IBPS | SBI | RRB PO | Clerk) Mains Exams

it?" ‘Its’ is the possessive of "it" as in, "That is Morton's puppy, but I
don't know its name." In the second sentence, its means "belonging
to it". In sentence (II), “teacher’s” should be replaced with
“teachers’”, since, the possessive of a plural noun is formed by
adding only an apostrophe when the noun ends in ‘s’, and by
adding both an apostrophe and s when it ends in a letter other than
‘s’. Moreover, in sentence (III) “beings” should be replaced by
“being”, since each one of and every one of are followed by a plural
noun or pronoun. Hence, option (e) is the most viable answer
choice.

23. (a); Among the given sentences, sentence (I) is grammatically incorrect.
To make sentence (I) correct, replace “will” with ‘would’, as the
reporting verb “told” is in the past tense. It is to be noted if
reporting verb is in the past tense, then in reported speech of
indirect narration, instead of “will” ‘would’ is used. Sentences (II)
and (III) are grammatically correct. Therefore, option (a) is the
most viable answer choice.

24. (d); (i) Replace ‘for changing’ by ‘of changing’ as ‘view(s)’ is followed
by the Preposition ‘of’.
(ii) Replace ‘in’ by ‘within’ as ‘within the allotted time’ is the
correct usage.
(iii) Replace ‘No variety’ by ‘No other variety’ as when a particular
class of people or things is compared with the same class of all
people or things in positive degree, then the correct syntax is “No
other + Singular Noun…so/as + Positive Degree + as…”.
e.g. No other leader in the R.J.D. is so/as powerful as Mr. Laloo
Prasad.

25. (d); Among all the given parts of the sentence, the error lies in part (IV)
of it. It is to be noticed that the verb ‘acquit’ which means free
(someone) from a criminal charge by a verdict of not guilty, always
takes the preposition ‘of’ to express “acquit a person of all
charges” while ‘in’ is used to express “acquit a person in a crime”.
Since, all the other parts of the sentence are correct, option (d) is
the most suitable answer choice.
16 Adda247 Publications For any detail, mail us at
Publications@adda247.com
Cracker Book for Bank (IBPS | SBI | RRB PO | Clerk) Mains Exams

26. (c); The error lies in parts (I), (II) and (III) of the sentence. It is to be
noted that the ‘board of directors’ is a group of people which fits
into the category of noun if the people in them is considered as a
unit, or plural if they are considered as individuals. Since, here
‘board of directors’ are considered as a single body, the verb
associated to it should also be in its singular form, therefore the
verbs ‘govern’, ‘conduct’, ‘approve’ should be replaced by
‘governs’, ‘conducts’ and ‘approves’ to make the sentence
grammatically correct. Since, part (IV) is already grammatically
correct, option (c) becomes the most viable answer choice.

27. (a); Among the given parts of the sentence, part (II) and (IV) contain
grammatical errors in them. It is to be noted that the noun or a
pronoun used after the phrase “one of” is always in the plural form
(as we are talking of one person/place/thing out of many).
However, the helping verb will always be in the singular form, as
the helping verb agrees with “one of” and not with the plural noun
in the sentence. Moreover, in part (IV) ‘within’ should be replaced
by ‘between’ as ‘within’ means inside (something) while ‘between’
is used to express ‘in the period separating (two points in time)’.
Since, the other parts are correct, option (a) is the most feasible
answer choice.

28. (e); All the parts of the given sentence are grammatically correct and
contextually meaningful therefore, they do not require any
corrections. Hence, option (e) is the most feasible answer choice.

29.(b); Part (III) of the sentence contains a grammatical error. To make


the sentence grammatically correct, replace the phrase “in spite of”
with “in spite of the fact that”. It is to be noted that “in spite of
something” is always followed by a noun or a noun equivalent,
whereas, “in spite of the fact that” is followed by Clause (subject +
Verb). E.g. In spite of his best efforts, he failed; In spite of the fact
that he worked very hard, he failed. All the other parts of the
sentence are in appropriate grammatical syntax. Hence, option (b)
is the most suitable answer choice.

30. (b); Only the second part of the sentence contains an error in it. It is to
be noted that ‘enough’ should follow the adverb ‘costly’. Since
whenever, ‘enough’ is used as an adverb of quantity, it is always
followed to an associated adjective or an adverb with it. Hence,
option (b) becomes the most suitable answer choice.
17 Adda247 Publications For any detail, mail us at
Publications@adda247.com
Cracker Book for Bank (IBPS | SBI | RRB PO | Clerk) Mains Exams

1 Adda247 Publications For any detail, mail us at


Publications@adda247.com
Cracker Book for Bank (IBPS | SBI | RRB PO | Clerk) Mains Exams

Chapter

13 Sentence formation
STUDY TIP & TRICKS

Try to find sentence which introduces a topic or person or idea so as to


find an opening sentence. Make mandatory pairs. Mandatory pairs are
sentences which will necessarily come together. Try to find the time
sequences: in general, if a given set of statements contains reference to
time based events, make sure maintain a particular order that is past-
present-future or vice-versa. Try to find the transition words such as: as
also, as well as, besides, furthermore, in addition, likewise, moreover,
similarly, consequently, hence, for one thing, above all, aside from,
besides, in other words, instead, likewise, on one hand, on the other hand,
rather, meanwhile, next, then, soon, in conclusion, after all, all in all, and
then try to sum up.

Practice Exercise Based on new Pattern

Direction (1-20): There are sets of statements in question given below


which when connected using the correct sentence structure forms a
complete single sentence without altering the meaning of the sentences
given in the question. There are four options given below the question,
choose the sentence that forms the correct formation of single sentence
which is both grammatically correct and contextually meaningful. If none
follows, choose (e) as your answer.
1. Their strategic relationship, upgraded to a major defense partnership
only recently; failed to make progress on signing foundational
agreements; the two governments have failed; the failure in turn has
held up talks on defense procurement and technology transfers
(a) Because of their strategic relationship, upgraded to a major
defense partnership only recently but failed to make progress on
signing foundational agreements, so, the two governments have
failed but that failure in turn has held up talks on defense
procurement and technology transfers.
2 Adda247 Publications For any detail, mail us at
Publications@adda247.com
Cracker Book for Bank (IBPS | SBI | RRB PO | Clerk) Mains Exams

(b) Despite their strategic relationship, upgraded to a major defense


partnership only recently, the two governments have failed to
make progress on signing foundational agreements, which in
turn have held up talks on defense procurement and technology
transfers.
(c) On their strategic relationship, upgraded to a major defense
partnership only recently, the two governments have failed to
make progress on signing foundational agreements, which in
turn has held up talks on defense procurement and technology
transfers.
(d) With their strategic relationship, upgraded to a major defense
partnership only recently and failed to make progress on signing
foundational agreements, but the failure in turn has held up
talks on defense procurement and technology transfers, hence
the two governments have failed.
(e) None of the above is correct.

2. The rise in global trade tensions; the ongoing trade war could be
another factor; the rout in emerging market currencies; its impact on
the rupee remains unclear as of now
(a) Due to the rise in global trade tensions, the ongoing war trade
war could be another factor because its impact on the rupee
remains unclear as of now against the rout in emerging market
currencies.
(b) As the rise in global trade tensions between the ongoing trade
war could be another factor against the rout in emerges market
currencies, but its impact on the rupee remains unclear as of
now.
(c) For the rise in global trade tensions among the ongoing trade
war could be another factor behind the rout in emerging market
currencies, but its impact on the rupee remain unclear as of
now.
(d) The rise in global trade tensions amidst the ongoing trade war
could be another factor behind the rout in emerging market
currencies, but its impact on the rupee remains unclear as of
now.
(e) None of the above is correct.
3 Adda247 Publications For any detail, mail us at
Publications@adda247.com
Cracker Book for Bank (IBPS | SBI | RRB PO | Clerk) Mains Exams

3. School authorities in several districts routinely summon police


officers; handcuff and arrest children as young as six and seven years
old; assaulting teachers, damaging school property; even for simply
throwing a tantrum
(a) School authorities in several districts routinely summon police
officers by handcuff and arrest children as young as six and
seven years old in spite of assaulting teachers, damaging school
property, as well as, even for simply throwing a tantrum.
(b) School authorities in several districts routinely summon police
officers to handcuff and arrest children as young as six and
seven years old for assaulting teachers, damaging school
property, or even for simply throwing a tantrum.
(c) School authorities in several districts routinely summon police
officers to handcuff and arrest children as young as six and
seven years old despite assaulting teachers, damaging school
property, or even for simply throwing a tantrum.
(d) School authorities in several districts routinely summon police
officers for assaulting teachers, damaging school property, or
even for simply throwing a tantrum and handcuff and arrest
children as young as six and seven years old.
(e) None of the above is correct.

4. A definitive legal framework recognizing; resolve disputes that arise


in their commercial transactions; businesses will be encouraged to
consider mediation in managing; enforce mediated settlement
agreements
(a) As a definitive legal framework recognizing to resolve disputes
that arise in their commercial transactions in order to enforce
mediated settlement agreements though businesses will be
encouraged to consider mediation in managing.
(b) While a definitive legal framework recognizing enforcing
mediated settlement agreements, the businesses will be
encouraged to consider mediation in managing but to resolve
issues that arise in their commercial transactions.
(c) For a definitive legal framework recognizing enforcing mediated
settlement agreements, the businesses will be encouraged to
consider mediation in managing instead of resolving issues that
arise in their commercial transactions.
4 Adda247 Publications For any detail, mail us at
Publications@adda247.com
Cracker Book for Bank (IBPS | SBI | RRB PO | Clerk) Mains Exams

(d) With a definitive legal framework recognizing and enforcing


mediated settlement agreements, businesses will be encouraged
to consider mediation in managing and resolving disputes that
arise in their commercial transactions.
(e) None of the above is correct.

5. Quick and precise adjustments to the export and import rules; divert
the excess supplies to overseas markets; the adjustments could have
arrested the price fall; the changes required were not carried out in
time
(a) Quick and precise adjustments to the export and import rules
will divert the excess supplies to overseas markets and the
adjustments could have arrested the price fall, so, the changes
required were not carried out in time.
(b) Quick and precise adjustments to the export and import rules
and the changes required were not carried out in time, so, it
diverted the excess supplies to overseas market even though the
adjustments could have arrested the price fall.
(c) Quick and precise adjustments to the export and import rules
could have arrested the price fall by diverting the excess
supplies to overseas markets, but the changes required were not
carried out in time.
(d) Quick and precise adjustments to the export and import rules
might divert the excess supplies to overseas markets but the
adjustments could have arrested the price fall, since the changes
required were not carried out in time.
(e) None of the above is correct.

6. The financial stability report released by the Reserve Bank of India on


Tuesday; the rise would be the highest level of bad debt in almost two
decades; the report has warned that the gross non-performing assets
(GNPAs) of scheduled commercial banks in the country could rise;
11.6% in March 2018 to 12.2% in March 2019
(a) The financial stability report released by the Reserve Bank of
India on Tuesday said that the rise would be the highest level of
bad debt in almost two decades but the report has warned that
the gross non-performing assets (GNPAs) of scheduled
5 Adda247 Publications For any detail, mail us at
Publications@adda247.com
Cracker Book for Bank (IBPS | SBI | RRB PO | Clerk) Mains Exams

commercial banks in the country could rise by 11.6% in March


2018 to 12.2% in March 2019.
(b) The financial stability report released by the Reserve Bank of
India on Tuesday has warned that the gross non-performing
assets (GNPAs) of scheduled commercial banks in the country
could risen from 11.6% in March 2018 to 12.2% in March 2019,
which would be the highest level of bad debt in almost two
decades.
(c) The financial stability report released by the Reserve Bank of
India on Tuesday has warned that the gross non-performing
assets (GNPAs) of scheduled commercial banks in the country
could rise from 11.6% in March 2018 to 12.2% in March 2019,
which would be the highest level of bad debt in almost two
decades.
(d) The financial stability report released by the Reserve Bank of
India on Tuesday has warns that the gross non-performing
assets (GNPAs) of scheduled commercial banks in the country
could rise from 11.6% in March 2018 to 12.2% in March 2019,
which would be the highest level of bad debt in almost two
decades.
(e) None of the above is correct.

7. Caroline Lucas pointed to a range of promises; the promises were


made during the course of the referendum campaign by the Leave side
that had been unfulfilled; promises such as £350 million a week extra
for the National Health Service (NHS); the difficulties related to
Northern Ireland and other matters that voters had not been aware of
at the time of the vote.
(a) Caroline Lucas pointed to a range of promises but the promises
were made during the course of the referendum campaign by
the Leave side that had been unfulfilled and the difficulties
related to Northern Ireland and other matters that voters had
not been aware of at the time of the vote because of promises
such as £350 million a week extra for the National Health
Service (NHS).
(b) Caroline Lucas pointed to a range of promises and the promises
were made during the course of the referendum campaign by
6 Adda247 Publications For any detail, mail us at
Publications@adda247.com
Cracker Book for Bank (IBPS | SBI | RRB PO | Clerk) Mains Exams

the Leave side that had been unfulfilled but the difficulties
related to Northern Ireland and other matters that voters had
not been aware of at the time of the vote due to promises such
as £350 million a week extra for the National Health Service
(NHS)
(c) Caroline Lucas pointed to a range of promises made during the
course of the referendum campaign by the Leave side that had
been unfulfilled, such as £350 million a week extra for the
National Health Service (NHS) as well as the difficulties relates
to Northern Ireland and other matters that voters not been
aware of at the time of the vote.
(d) Caroline Lucas pointed to a range of promises made during the
course of the referendum campaign by the Leave side that had
been unfulfilled, such as £350 million a week extra for the
National Health Service (NHS) as well as the difficulties related
to Northern Ireland and other matters that voters had not been
aware of at the time of the vote.
(e) None of the above is correct.

8. At such a tender age, his ability to pinpoint the best possible


continuation; has the chess world fascinated; then justify his choice by
executing a larger plan; among the many that present themselves after
every move
(a) At such a tender age, his ability to pinpoint the best possible
continuation has the chess world fascinated but then justifying
his choice by executing a larger plan among the many that
present themselves after every move.
(b) At such a tender age, his ability to pinpoint the best possible
continuation from among the many that present themselves
after every move, and then justify his choice by executing a
larger plan, has the chess world fascinated.
(c) At such a tender age, his ability to pinpoint the best possible
continuation has the chess world fascinated and among the
many that present themselves after every move but then justify
his choice by executing a larger plan.
(d) At such a tender age, his ability to pinpoint the best possible
continuation from among the many that present themself after
every move, and then justify his choice by execute a larger plan,
has the chess world fascinated.
(e) None of the above is correct.
7 Adda247 Publications For any detail, mail us at
Publications@adda247.com
Cracker Book for Bank (IBPS | SBI | RRB PO | Clerk) Mains Exams

9. Rules of caste difference and deference required; social reformism in


general seemed outraged only; the rules required that the chest of
those deemed to be social inferiors be exposed in the presence of
ostensible social superiors; the nakedness of the female form
(a) As the rules of caste difference and deference required that the
nakedness of the female form and social reformism in general
seemed outraged only by the rules that required that the chest
of those deemed to be social inferiors be exposed in the
presence of ostensible social superiors.
(b) As the rules of caste difference and deference required by the
nakedness of the female form and social reformism in general
seemed outraged only by the rules that required that the chest
of those deemed to be social inferiors be exposed in the
presence of ostensible social superiors.
(c) While rules of caste difference and deference required that the
chest of those deemed to be social inferiors be expose in the
presence of ostensible social superiors, social reformism in
general seemed outraged only by the nakedness of the female
form.
(d) While rules of caste difference and deference required that the
chest of those deemed to be social inferiors be exposed in the
presence of ostensible social superiors, social reformism in
general seemed outraged only by the nakedness of the female
form.
(e) None of the above is correct.

10. Video footage of the children; the anguished cries unnerved even
President Donald Trump’s most ardent supporters; audio tapes of
their anguished cries; the children were imprisoned in cage-like
detention centers
(a) Video footage of the children and the anguished cries unnerved
even President Donald Trump’s most ardent supporters because
in the audio tapes of their anguished cries, the children were
imprisoned in cage-like detention centers.
(b) Video footage of the children and in the audio tapes of their
anguished cries, the children were imprisoned in cage-like
detention centers and the anguished cries unnerved even
President Donald Trump’s most ardent supporters.
8 Adda247 Publications For any detail, mail us at
Publications@adda247.com
Cracker Book for Bank (IBPS | SBI | RRB PO | Clerk) Mains Exams

(c) Video footage of the children imprisoned in cage-like detention


centers, and audio tapes of their anguished cries, have unnerved
even President Donald Trump’s most ardent supporters.
(d) Video footage of the children and the anguished cries unnerved
even President Donald Trump’s most ardent supporters because
the children were imprisoned in cage-like detention centers in
audio tapes of their anguished cries.
(e) None of the above is correct.

11. Arguably, a conversation with persons; permeate divided societies;


might well be strangers in traditional environments contributes a
great deal; the ironing out of senseless tensions.
(a) Arguably, a conversation with persons who permeate divided
societies might well be strangers in traditional environments
contributes a great deal into the ironing out of senseless
tensions.
(b) Arguably, a conversation with persons who might well be
strangers in traditional environments contribute a great deal to
the ironing out of senseless tensions that permeate divided
societies.
(c) Arguably, a conversation with persons that permeate divided
societies and might well be strangers in traditional
environments contributes a great deal by the ironing out of
senseless tensions.
(d) Arguably, a conversation with persons who might well be
strangers in traditional environments contributes a great deal to
the ironing out of senseless tensions that permeate divided
societies.
(e) None of the above is correct.

9 Adda247 Publications For any detail, mail us at


Publications@adda247.com
Cracker Book for Bank (IBPS | SBI | RRB PO | Clerk) Mains Exams

12. In the choice of target and the timing of the murder, the attack sends
out a chilling message; there are forces determined to gut the
emerging consensus; on the eve of Id-ul-Fitr and in the closing days of
the government’s Ramzan ceasefire against militants; extending the
cease-ops and preparing the ground for dialogue.
(a) In the choice of target and the timing of the murder, the attack
sends out a chilling message that there are forces determined to
gut the emerging consensus on the eve of Id-ul-Fitr and in the
closing days of the government’s Ramzan ceasefire against
militants by extending the cease-ops and preparing the ground
for dialogue.
(b) In the choice of target and the timing of the murder, the attack
sends out a chilling message that on the eve of Id-ul-Fitr and in
the closing days of the government’s Ramzan ceasefire against
militants, there are forces determined to gut the emerging
consensus for extending the cease-ops and preparing the ground
for dialogue.
(c) In the choice of target and the timing of the murder, the attack
sends out a chilling message that on the eve of Id-ul-Fitr and in
the closing days of the government’s Ramzan ceasefire against
militants, there is forces determined to gut the emerging
consensus for extending the cease-ops and preparing the ground
for dialogue.
(d) In the choice of target and the timing of the murder, the attack
sends out a chilling message that due to extending the cease-ops
and preparing the ground for dialogue on the eve of Id-ul-Fitr
and in the closing days of the government’s Ramzan ceasefire
against militants, there are forces determined to gut the
emerging consensus.
(e) None of the above is correct.

10 Adda247 Publications For any detail, mail us at


Publications@adda247.com
Cracker Book for Bank (IBPS | SBI | RRB PO | Clerk) Mains Exams

13. The colonial government was directed; Indian religions were


considered heathen and false; disentangle itself from “superstitious”
Indian religious institutions.
(a) As soon as the colonial government was directed that Indian
religions were considered heathen and false, it did not
disentangle itself from “superstitious” Indian religious
institutions.
(b) Although the colonial government was directed that Indian
religions were considered heathen and false, it disentangle itself
from “superstitious” Indian religious institutions.
(c) The colonial government was directed to disentangle itself from
“superstitious” Indian religious institutions, because Indian
religions were considered heathen and false.
(d) Unless the colonial government was not directed that Indian
religions were considered heathen and false, it did not
disentangle itself from “superstitious” Indian religious
institutions.
(e) None of the above is correct.

14. The wall of separation between temple and colonial state in India was
achieved in 1863; take over the superintendence of any land or other
property; belongs to a Mosque, Temple, or other religious
establishment; a law was enacted which said that it would no longer
be lawful for any Government in India.
(a) The wall of separation between temple and colonial state in
India was achieved in 1863 taking over the superintendence of
any land or other property that belongs to a Mosque, Temple, or
other religious establishment due to a law was enacted which
said that it would no longer be lawful for any Government in
India.
(b) The wall of separation between temple and colonial state in
India was achieved in 1863, when a law was enact which said
that it would no longer be lawful for any Government in India to
take over the superintendence of any land or other property
belonging to a Mosque, Temple, or other religious establishment.

11 Adda247 Publications For any detail, mail us at


Publications@adda247.com
Cracker Book for Bank (IBPS | SBI | RRB PO | Clerk) Mains Exams

(c) The wall of separation between temple and colonial state in


India was achieved in 1863, when a law was enacted which said
that it would no longer be lawful for any Government in India to
take over the superintendence of any land or other property
belonging to a Mosque, Temple, or other religious establishment.
(d) The wall of separation between temple and colonial state in
India was achieved in 1863, when a law was enacted which said
that it would no longer be lawful for any Government in India to
take over the superintendence of any land or other property
belonged to a Mosque, Temple, or other religious establishment.
(e) None of the above is correct.

15. Shimla was in the news recently; the last week of May this summer;
the hill town experienced acute water shortage; water almost dried up
in the Nauti-Khad stream.
(a) Shimla was in the news recently when the hill town experienced
acute water shortage since the last week of May this summer
after water almost dried up in the Nauti-Khad stream.
(b) Shimla was in the news recently when the hill town experienced
acute water shortage for the last week of May this summer after
water almost dried up in the Nauti-Khad stream.
(c) Shimla was in the news recently when the hill town experienced
acute water shortage from the last week of May this summer
after water almost dries up in the Nauti-Khad stream.
(d) Shimla was in the news recently when water almost dried up in
the Nauti-Khad stream since the last week of May this summer
but the hill town experienced acute water shortage.
(e) None of the above is correct.

12 Adda247 Publications For any detail, mail us at


Publications@adda247.com
Cracker Book for Bank (IBPS | SBI | RRB PO | Clerk) Mains Exams

16. The United States withdrew from The Paris Agreement; grass root
organizations, the rising generation and public-private-partnerships;
there is cause for hope in a tumultuous climate change landscape; step
up to the plate of American environmental leadership.
(a) If the United States withdrew from The Paris Agreement, there
is cause for hope in a tumultuous climate change landscape for
grass root organizations, the rising generation, and public-
private-partnerships step up to the plate of American
environmental leadership.
(b) Hardly had the United States withdrew from The Paris
Agreement than there is cause for hope in a tumultuous climate
change landscape as grass root organizations, the rising
generation, and public-private-partnerships step up to the plate
of American environmental leadership.
(c) Because the United States withdrew from The Paris Agreement,
so, there is cause for hope in a tumultuous climate change
landscape for grass root organizations, the rising generation but
public-private-partnerships step up to the plate of American
environmental leadership.
(d) While the United States withdrew from The Paris Agreement,
there is cause for hope in a tumultuous climate change
landscape as grass root organizations, the rising generation, and
public-private-partnerships step up to the plate of American
environmental leadership.
(e) None of the above is correct.

17. The Pentagon changing the U.S. Navy’s biggest and oldest fleet
command name; Indo-Pacific command from Pacific Command
(PACOM) might be symbolic; their commitments towards better
relations with India; it is the least the United States could do to show
seriousness.
(a) The Pentagon changing the U.S. Navy’s biggest and oldest fleet
command name by Indo-Pacific command from Pacific
Command (PACOM) might be symbolic and their commitments
towards better relations with India is the least the United States
could do to show seriousness.
13 Adda247 Publications For any detail, mail us at
Publications@adda247.com
Cracker Book for Bank (IBPS | SBI | RRB PO | Clerk) Mains Exams

(b) The Pentagon changing the U.S. Navy’s biggest and oldest fleet
command name to Indo-Pacific command from Pacific Command
(PACOM) might be symbolic, it is the least the United States
could do to show seriousness about their commitments towards
better relations with India.
(c) The Pentagon changing the U.S. Navy’s biggest and oldest fleet
command name to Indo-Pacific command from Pacific Command
(PACOM) might be symbolic, it is the least the United States
could do to show seriousness from their commitments towards
better relations with India.
(d) The Pentagon changing the U.S. Navy’s biggest and oldest fleet
command name for Indo-Pacific command from Pacific
Command (PACOM) might be symbolic, but their commitments
towards better relations with India is the least the United States
could do to show seriousness.
(e) None of the above is correct.

18. The latest rankings of Global Illicit Trade Environment Index; their
lack of automation in their systems; Austria ranked highly in three of
the four main categories; failed to score as high on the customs
environment.
(a) Although in the latest rankings of Global Illicit Trade
Environment Index, Austria ranked highly in three of the four
main categories and their lack of automation in their systems
failed to score as high on the customs environment.
(b) Regardless of the latest rankings of Global Illicit Trade
Environment Index, Austria ranked highly in three of the four
main categories and failed to score as high on the customs
environment due to their lack of automation in their systems.
(c) In the latest rankings of Global Illicit Trade Environment Index,
Austria ranked highly in three of the four main categories but
failed to score as high on the customs environment due to their
lack of automation in their systems.
(d) With regards to the latest rankings of Global Illicit Trade
Environment Index, Austria ranked highly in three of the four
main categories but fails to score as high on the customs
environment owing to their lack of automation in their systems.
(e) None of the above is correct.
14 Adda247 Publications For any detail, mail us at
Publications@adda247.com
Cracker Book for Bank (IBPS | SBI | RRB PO | Clerk) Mains Exams

19. A powerful and sustained implementation of inquiry-based science


education teamed; the issues of critical importance, such as climate
change, the health of the world’s oceans, and clean energy; civic
learning and social emotional learning can help students focus;
examining each issue from multiple perspectives.
(a) A powerful and sustained implementation of inquiry-based
science education teamed after the issues of critical importance,
such as climate change, the health of the world’s oceans, and
clean energy due to civic learning and social emotional learning
can help students focus before examining each issue from
multiple perspectives.
(b) A powerful and sustained implementation of inquiry-based
science education teamed on the issues of critical importance,
such as climate change, the health of the world’s oceans, and
clean energy by civic learning and social emotional learning can
help students focus against examining each issue from multiple
perspectives.
(c) A powerful and sustained implementation of inquiry-based
science education teamed for civic learning and social emotional
learning can help students focus with the issued of critical
importance, such as climate change, the health of the world’s
oceans, and clean energy, and examining each issue from
multiple perspectives.
(d) A powerful and sustained implementation of inquiry-based
science education teamed with civic learning and social
emotional learning can help students focus on the issues of
critical importance, such as climate change, the health of the
world’s oceans, and clean energy, while examining each issue
from multiple perspectives.
(e) None of the above is correct.

20. China and India routinely get compared to each other; based on
technological, social and economic trends; they are wildly divergent;
their population sizes and geographical proximity.
(a) China and India routinely get compared to each other based on
technological, social and economic trends because they are
15 Adda247 Publications For any detail, mail us at
Publications@adda247.com
Cracker Book for Bank (IBPS | SBI | RRB PO | Clerk) Mains Exams

wildly divergent ahead of their population sizes and


geographical proximity.
(b) China and India routinely get compared to each other because of
their population sizes and geographical proximity but based on
technological, social and economic trends, they are wildly
divergent.
(c) China and India routinely get compared to each other because of
their population sizes and geographical proximity and base on
technological, social and economic trends, they are wildly
divergent.
(d) China and India routinely get compared to each other based on
technological, social and economic trends since they are wildly
divergent ahead of their population sizes and geographical
proximity.
(e) None of the above is correct.

Solutions

1. (c); Among the given options, sentence (b) is grammatically incorrect.


Sentences (a) and (d) are contextually different and structurally
incorrect. They are not inferring the same meaning as per the
demand of the question. Hence only option (c) forms the correct
sentence which follows the sentences given in the question both
grammatically and contextually.

2. (d); Among the given options, sentences (b) and (c) are grammatically
incorrect. Sentence (a) is contextually different and structurally
incorrect. It is not inferring the same meaning as per the demand of
the question. Hence only option (d) forms the correct sentence
which follows the sentences given in the question both
grammatically and contextually.

3. (b); Among the given options, sentences (a) and (c) are grammatically
incorrect. Sentence (d) is contextually different and structurally
incorrect. It is not inferring the same meaning as per the demand of
16 Adda247 Publications For any detail, mail us at
Publications@adda247.com
Cracker Book for Bank (IBPS | SBI | RRB PO | Clerk) Mains Exams

the question. Hence only option (b) forms the correct sentence
which follows the sentences given in the question both
grammatically and contextually.

4. (a); The phrase “the U.S.’s Countering America’s Adversaries


through Sanctions Act” means the efforts of United States to
counter its adversaries by implementing Sanctions Act. Among the
given statements, only the sentence (I) expresses the meaning
which complies with the meaning of the phrase and at the same
time it makes sure that the actual meaning of the sentence remains
intact. Statements (II) and (III) are irrelevant as they alter the
meaning of the sentence. Hence (a) is the correct option.

5. (c); Among the given options, other than sentence (c), all the other
sentences, i.e. Sentences (a), (b) and (d) are contextually different
and structurally incorrect. They are not inferring the same meaning
as per the demand of the question. Hence only option (c) forms the
correct sentence which follows the sentences given in the question
both grammatically and contextually.

6. (c); Among the given options, sentences (b) and (d) are grammatically
incorrect. In sentence (b) “rise” should be used in place of “risen”.
In sentence (d) “warned” should be used in place of “warns” as the
sentence is in past tense. Sentence (a) is contextually different and
structurally incorrect. It is not inferring the same meaning as per
the demand of the question. Hence only option (c) forms the
correct sentence which follows the sentences given in the question
both grammatically and contextually.

7. (d); Among the given options, sentence (c) is grammatically incorrect


since the sentence is in past tense and therefore related should be
used in place of relates. Sentences (a) and (b) are contextually
different and structurally incorrect. They are not inferring the same
meaning as per the demand of the question. Hence only option (d)
forms the correct sentence which follows the sentences given in the
question both grammatically and contextually.

17 Adda247 Publications For any detail, mail us at


Publications@adda247.com
Cracker Book for Bank (IBPS | SBI | RRB PO | Clerk) Mains Exams

8. (b); Among the given options, sentence (d) is grammatically incorrect.


Sentences (a) and (c) are contextually different and structurally
incorrect. They are not inferring the same meaning as per the
demand of the question. Hence only option (b) forms the correct
sentence which follows the sentences given in the question both
grammatically and contextually.

9. (d); Among the given options, sentence (c) is grammatically incorrect.


In sentence (c) “exposed” should be used in place of “expose”.
Sentences (a) and (b) are contextually different and structurally
incorrect. They are not inferring the same meaning as per the
demand of the question. Hence only option (b) forms the correct
sentence which follows the sentences given in the question both
grammatically and contextually.

10. (c); Among the given options, other than sentence (c), all the other
sentences, i.e. Sentences (a), (b) and (d) are contextually different
and structurally incorrect. They are not inferring the same meaning
as per the demand of the question. Hence only option (c) forms the
correct sentence which follows the sentences given in the question
both grammatically and contextually.

11. (d); Among the given options, sentence (b) is grammatically incorrect
as “contributes” should be used in place of “contribute”. Sentences
(a) and (c) are contextually different and structurally incorrect.
They are not inferring the same meaning as per the demand of the
question. Hence only option (d) forms the correct sentence which
follows the sentences given in the question both grammatically and
contextually

12. (b); Among the given options, sentence (c) is grammatically incorrect
as ‘are’ should be used in place of ‘is’ because of the use of ‘forces’.
Sentences (a) and (d) are contextually different and structurally
incorrect. They are not inferring the same meaning as per the
demand of the question. Hence only option (b) forms the correct
sentence which follows the sentences given in the question both
grammatically and contextually.
18 Adda247 Publications For any detail, mail us at
Publications@adda247.com
Cracker Book for Bank (IBPS | SBI | RRB PO | Clerk) Mains Exams

13. (c); Among the given options, sentence (d) is grammatically incorrect.
Sentences (a) and (b) are contextually different and structurally
incorrect. Option (b) is grammatically incorrect also as
‘disentangled’ should be used in place of disentangles. They are not
inferring the same meaning as per the demand of the question.
Hence only option (c) forms the correct sentence which follows the
sentences given in the question both grammatically and
contextually.

14. (c); Among the given options, sentences (b) and (d) are grammatically
incorrect. In sentence (b) “enacted” should be used in place of
“enact”. In sentence (d) “belonging” should be used in place of
“belonged”. Sentence (a) is contextually different and structurally
incorrect. It is not inferring the same meaning as per the demand of
the question. Hence only option (c) forms the correct sentence
which follows the sentences given in the question both
grammatically and contextually.

15. (a); Among the given options, sentences (b) and (c) are grammatically
incorrect. Sentence (d) is contextually different and structurally
incorrect. “For” is used to show the duration and last week is a
“point of time” therefore “since” should be used. It is not inferring
the same meaning as per the demand of the question. Hence only
option (a) forms the correct sentence which follows the sentences
given in the question both grammatically and contextually.

16. (d); Among the given options, sentence (b) is grammatically incorrect
as in sentences with this kind of structure we use “when” and not
than with “hardly”. Sentences (a) and (c) are contextually different
and structurally incorrect. They are not inferring the same meaning
as per the demand of the question. Hence only option (d) forms the
correct sentence which follows the sentences given in the question
both grammatically and contextually.

19 Adda247 Publications For any detail, mail us at


Publications@adda247.com
Cracker Book for Bank (IBPS | SBI | RRB PO | Clerk) Mains Exams

17. (b); Among the given options, sentence (c) is grammatically incorrect.
Sentences (a) and (d) are contextually different and structurally
incorrect. They are not inferring the same meaning as per the
demand of the question. Hence only option (b) forms the correct
sentence which follows the sentences given in the question both
grammatically and contextually.

18. (c); Among the given options, sentence (d) is grammatically incorrect.
Sentences (a) and (b) are contextually different and structurally
incorrect. They are not inferring the same meaning as per the
demand of the question. Hence only option (c) forms the correct
sentence which follows the sentences given in the question both
grammatically and contextually.

19. (d); Among the given options, sentence (c) is grammatically incorrect.
Sentences (a) and (b) are contextually different and structurally
incorrect. They are not inferring the same meaning as per the
demand of the question. Hence only option (d) forms the correct
sentence which follows the sentences given in the question both
grammatically and contextually.

20. (b); Among the given options, sentence (c) is grammatically incorrect.
Sentences (a) and (d) are contextually different and structurally
incorrect. They are not inferring the same meaning as per the
demand of the question. Hence only option (b) forms the correct
sentence which follows the sentences given in the question both
grammatically and contextually.

20 Adda247 Publications For any detail, mail us at


Publications@adda247.com
Cracker Book for Bank (IBPS | SBI | RRB PO | Clerk) Mains Exams

1 Adda247 Publications For any detail, mail us at


Publications@adda247.com
Cracker Book for Bank (IBPS | SBI | RRB PO | Clerk) Mains Exams

Chapter

14 Paragraph Completion

STUDY TIPS

Scope of the correct answer choice should be the same as that of


the passage. Scope loosely refers to the subject of the passage and the
issues discussed in it. Try to find the tone of the paragraph, If the
passage is laudatory, it is not possible for the correct answer choice to be
critical. Continuity, It is the last thought or issue that has to be taken
forward and not something that is dead and buried.

Practice Exercise Based on New Pattern

Directions (1-25): Each of the following questions has a paragraph from


which one sentence has been deleted. From the given options, choose the
one that completes the paragraph in the most appropriate/coherent way.

1. NASA’s Hubble Space Telescope has discovered the farthest individual


star ever seen — an enormous blue stellar body nicknamed Icarus
located over halfway across the universe. The star, harboured in a
very distant spiral galaxy, is so far away that its light has taken nine
billion years to reach Earth. It appears to us as it did when the
universe was about 30% of its current age. Normally, the star would
be too faint to view, even with the world’s largest telescopes.
_________________________________ “This is the first time we’re seeing a
magnified, individual star,” said Patrick Kelly, who led the study as a
postdoctoral fellow in University of California at Berkeley.
(a) However, through a phenomenon called gravitational lensing that
tremendously amplifies the star’s feeble glow, astronomers were
able to pinpoint this faraway star and set a new distance record.
(b) The discovery of Icarus through gravitational lensing has initiated
a new way for astronomers to study individual stars in distant
galaxies.
2 Adda247 Publications For any detail, mail us at
Publications@adda247.com
Cracker Book for Bank (IBPS | SBI | RRB PO | Clerk) Mains Exams

(c) Located about 5 billion light-years from Earth, this massive


cluster of galaxies sits between the Earth and the galaxy that
contains the distant star.
(d) Setting a new distance record, astronomers using NASA's Hubble
Space Telescope have discovered an enormous blue star
nicknamed "Icarus", the farthest individual star ever seen.
(e) “You can see individual galaxies out there, but this star is at least
100 times farther away than the next individual star we can
study, except for supernova explosions,” said Kelly, who is now at
the University of Minnesota in the US.

2. Mahatma Gandhi came from a poor family, and he had dropped out of
the cheapest college he could afford. Mavji Dave Joshiji, a Brahmin
priest and family friend, advised Gandhi and his family that he should
consider law studies in London. In July 1888, his wife Kasturba gave
birth to their first surviving son, Harilal. His mother was not
comfortable about Gandhi leaving his wife and family, and going so far
from home. Gandhi's uncle Tulsidas also tried to dissuade his nephew.
Gandhi wanted to go. To persuade his mother Putlibai, Gandhi made a
vow in front of his mother that he would abstain from meat, alcohol
and women. Gandhi's brother Laxmidas, who was already a lawyer,
cheered Gandhi's London studies plan and offered to support him.
____________________________________________________.
(a) They encouraged Gandhi to join them in reading the Bhagavad
Gita both in translation as well as in the original.
(b) In London, Gandhi studied law and jurisprudence and enrolled at
the Inner Temple with the intention of becoming a barrister.
(c) Gandhi, at age 22, was called to the bar in June 1891 and then left
London for India, where he learned that his mother had died
while he was in London and that his family had kept the news
from him.
(d) Putlibai gave Gandhi her permission and blessing.
(e) None of these

3. _______________________________________________________ In some countries,


bananas used for cooking may be called plantains, in contrast to
dessert bananas. The fruit is variable in size, color, and firmness, but
is usually elongated and curved, with soft flesh rich in starch covered
with a rind, which may be green, yellow, red, purple, or brown when
3 Adda247 Publications For any detail, mail us at
Publications@adda247.com
Cracker Book for Bank (IBPS | SBI | RRB PO | Clerk) Mains Exams

ripe. The fruits grow in clusters hanging from the top of the plant.
Almost all modern edible parthenocarpic (seedless) bananas come
from two wild species – Musa acuminata and Musa balbisiana. The
scientific names of most cultivated bananas are Musa acuminata,
Musa balbisiana, and Musa × paradisiaca for the hybrid Musa
acuminata × M. balbisiana, depending on their genomic constitution.
The old scientific name Musa sapientum is no longer used.
(a) The banana fruits develop from the banana heart, in a large
hanging cluster, made up of tiers.
(b) A banana is an edible fruit – botanically a berry – produced by
several kinds of large herbaceous flowering plants in the genus
Musa.
(c) There are fuzzy bananas whose skins are bubblegum pink; green-
and-white striped bananas with pulp the color of orange sherbet;
bananas that, when cooked, taste like strawberries.
(d) All the above-ground parts of a banana plant grow from a
structure usually called a "corm".
(e) Farmers in Southeast Asia and Papua New Guinea first
domesticated bananas.

4. _____________________, protein from nuts and seeds could be beneficial for


the human heart, suggests a study including more than 80,000
participants. The study, appearing in the International Journal of
Epidemiology, found that people who consumed large amounts of
meat protein experienced a 60 per cent increase in cardiovascular
disease (CVD), while people who consumed large amounts of protein
from nuts and seeds experienced a 40-per cent reduction in CVD.
(a) It can help you lose weight (especially belly fat), and increase
your muscle mass and strength, to name a few
(b) many health and fitness experts believe that we need much more
protein than what we get
(c) While eating red meat that is rich in protein is linked with
increased risk of heart disease
(d) Almonds are loaded with high protein including other important
nutrients, fiber, vitamin E manganese and magnesium
(e) none of these
4 Adda247 Publications For any detail, mail us at
Publications@adda247.com
Cracker Book for Bank (IBPS | SBI | RRB PO | Clerk) Mains Exams

5. LED lamps are often promoted as energy-efficient lighting for both


inside the home and outside.
However, experts say that LED lighting -- if not installed in
moderation -- could affect our health. Blue light exposure disrupts our
body’s ability to produce melatonin, a hormone that helps us fall
asleep. _________________________________________________________________ Many
studies have suggested that blue light emitted by our mobile phones
and laptops is disruptive because its short wavelength tells our brain
to stay alert.
(a) The United States Department of Energy has claimed that LED
lights are no more harmful than any other lighting.
(b) We should ensure that lighting products meet photobiological
safety standards, says Lighting Designer and Educator Anil Valia.
(c) When this process is disrupted, our body's clock goes off rhythm.
(d) Modern LED street lights which are cheaper to run and have
lower emissions 'damage people's eyesight and dazzle motorists'.
(e) none of these

6. Over the last few days, Delhi residents have been protesting against
the government’s approval for felling over 14,000 trees in south Delhi.
Faced with severe criticism, the National Buildings Construction
Corporation, tasked with redeveloping half a dozen south Delhi
colonies, on Monday assured the Delhi High Court that no trees would
be cut for the project till July 4, which is temporary relief. Many of the
trees proposed to be felled are mature, local, fruit-bearing ones that
provide clean air, shade and water recharge to humans and are homes
to many birds. These areas of Delhi have served as the “lungs” of the
city. ________________________
(a) However, the project reports overlook these qualities.
(b) Large constructions have been difficult to manage in India.
(c) The Minister for Urban Development has stated that this public
campaign is “misinformed”.
(d) The governments could join hands by committing to review these
projects.
(e) none of these

5 Adda247 Publications For any detail, mail us at


Publications@adda247.com
Cracker Book for Bank (IBPS | SBI | RRB PO | Clerk) Mains Exams

7. The inefficient agriculture supply chain, which is plagued by the


presence of inefficient intermediaries capturing a large share of
profits, poor infrastructure and outdated farming techniques,
_________________________________. Entrepreneurs could also develop low-
cost tools, smart farming techniques and warehousing facilities,
besides improving the quality of inputs and extending the reach of
irrigation, to transform agriculture in India.
(a) will do well to infuse the entrepreneurial energy.
(b) and for the country’s long-term economic success.
(c) which in turn is negatively impacting health.
(d) also provides significant opportunities for entrepreneurial
innovation.
(e) none of these

8. The future of Indian universities (public and private) will significantly


depend upon our ability to harness the possibility of individual,
institutional and corporate philanthropy for the purposes of higher
education. A major legal and policy reform to promote some form of
mandatory corporate social responsibility (CSR) was initiated through
the Companies Act, 2013.
___________________________________________________________. Unfortunately,
the results so far have not been encouraging.
(a) However, these recommendations were not implemented.
(b) Path-breaking, it had the potential to transform the relationship
between business and society.
(c) A range of reforms are being promoted in higher education.
(d) leadership in philanthropy is central to enabling an institutional
vision
(e) none of these
6 Adda247 Publications For any detail, mail us at
Publications@adda247.com
Cracker Book for Bank (IBPS | SBI | RRB PO | Clerk) Mains Exams

9. The government has finally put healthcare front and centre, with a
bold set of policy and regulatory activities. It has realised that a strong
intervention on Universal Health Coverage (UHC) is the only solution
to tackle poverty and improve the quality of life of our citizens.
_________________________________________. The Ayushman Bharat,
announced in this year’s Union Budget, promises an insurance benefit
cover of Rs 5 lakh per family, per year, for 40% of our population,
apart from other benefits. It is perhaps the greatest example in recent
times to prove that the government is serious with healthcare, and its
subsequent uplifting. The National Health Policy 2017 (NHP)
envisages the private sector as a strategic partner in meeting these
goals.
(a) It includes calls for substantially increased health expenditure as
a percentage of GDP, UHC and quality healthcare available to all.
(b) To advance the benefits of medical technology, we must have the
right public policies to support investment, innovation and
patient access.
(c) This is the complex but far more rewarding path of looking at
various aspects of healthcare costs in the context of innovations
to improve longevity and quality of life.
(d) Building a sustainable business can be done neither by solely
focusing on profitability, nor charity.
(e) none of these

10. A temple or any place of worship cannot claim a right to the deity who
is being worshipped in that place, since the deity by definition is
present to all at all other places. God is not an entity who can be
privatized and put under the control of certain individuals or
communities or some dominant males.
(__________________).Even rituals cannot be privatized and copyrighted
since they are also done for god, a public being.
(a) And if god should not be privatized, education, water and air
should also not be privatized.

7 Adda247 Publications For any detail, mail us at


Publications@adda247.com
Cracker Book for Bank (IBPS | SBI | RRB PO | Clerk) Mains Exams

(b) Temple is not a private space because of its dependence on the


idea of god as the supreme ‘public entity’ has an important
corollary.
(c) Equivalently, god really has no privacy, nor can any human claim
‘copyright’ over their gods.
(d) Thus, all the arguments about celibacy invoked in this context are
attempts to impose concepts of the private on a public being.
(e) It will be useful to understand the implications of the claim that
temples cannot be private places.

11. The government raised social expenditure through its employment


guarantee scheme, which, together with a construction boom, raised
wages in the informal sector. (______________________). The result: the
growth in average wages for rural workers in 2013-14 was a huge
28% year-on-year. It refused to curtail subsidies, because it was
scared of a backlash from the middle classes.
(a) Globalization proved to be the final nail in the coffin for the
western working class.
(b) The contending classes are different, because organized labor
forms a minuscule proportion of the working class in India.
(c) In terms of the conflict theory, the competing claims of various
sections of the population were all accommodated by the
government in a please-all policy.
(d) It increased minimum support prices for farmers.
(e) None of these.

12. By making citizens liable for offering a bribe to a public servant, the
anti-corruption law has been brought in line with the UN Convention
Against Corruption. (________________). This exception kicks in only
when the fact that one was forced to pay a bribe is reported to a law
enforcement authority within seven days. The penal provision can
empower people by allowing them to cite it to refuse to pay a bribe.
(a) Further, it may render them vulnerable to threats from
unscrupulous public servants.
(b) The only exception to this rule is when one is forced to give a
bribe.
8 Adda247 Publications For any detail, mail us at
Publications@adda247.com
Cracker Book for Bank (IBPS | SBI | RRB PO | Clerk) Mains Exams

(c) The most unacceptable change is the introduction of a prior


approval norm to start an investigation.
(d) These should contain penal provisions as well as assure citizens
of time bound services.
(e) Public servants need to be protected against unfair prosecution.

13. (____________). The Great Depression discredited the idea that


economies were basically self-correcting, and the following decades
saw the development of Keynesian theory and the use of fiscal
stimulus. The stagflation of the 1970s led to the development of real
business cycle models, which saw recessions as the efficient working
of the economy, and central bank meddling as likely only to cause
inflation.
(a) The notion that economic booms cause busts, instead of being
random unrelated events seems to have much more currency
beyond the ivory tower than within it.
(b) There was immediately a flurry of activity, as economists
hastened to shoehorn finance into their standard models.
(c) Macroeconomics tends to advance—or, at least, to change—one
crisis at a time.
(d) When extrapolative expectations are combined with an
inherently fragile financial system, a predictable cycle of booms
and busts is the result.
(e) none of these.

14. Duties are those actions which take one to one’s goal. Thus having a
goal is crucial. The ideal action has a goal or destination.
(_______________________). The entire focus is on the action, not the fruit.
Like Arjuna , all one can see is the eye of the bird. Only the action
exists. When all energy is thus directed to the action, the goal is
achieved faster. And the journey is enjoyable!
(a) But when this goal becomes a craving, a longing, when it becomes
a hook on which the happiness of life hinges, the fruit starts
disturbing the action and life itself.
(b) To work with fruit makes one unhappy.
9 Adda247 Publications For any detail, mail us at
Publications@adda247.com
Cracker Book for Bank (IBPS | SBI | RRB PO | Clerk) Mains Exams

(c) To want something, to believe one will get it and then one will be
happy is pure ignorance.
(d) The journey is anxious; the loss is heartbreaking.
(e) But once the action starts, the goal should not be allowed to
disturb one’s concentration.

15. There is in fact no need to panic on account of the rupee. Barring the
gradual decline in its value this year, the Indian currency has been
fairly stable over 2016 and 2017; ______________________ its purchasing
power at home has been falling.
(a) with the Sensex rising by half a percentage point.
(b) With both consumer and wholesale price inflation easing in July.
(c) with inflation being higher than in developed countries.
(d) with the U.S. The Trump administration has just sanctioned
Turkey’s Justice
(e) which has only intervened sparingly in the forex market so far

16. State governments, which will administer it through their own agency,
___________________________________, at pre-determined rates. Reaching a
consensus on treatment costs through a transparent consultative
process is vital for a smooth and steady rollout.
(a) Will be launched formally on September 25 sends out the signal
that the government is finally recognising
(b) and to raise skill levels. Reducing the cost of universal health
coverage is imperative
(c) The Centre should extend the scheme to all children and senior
citizens, and cover out-patient consultation
(d) will have to purchase care from a variety of players, including in
the private sector
(e) have been found ordering unnecessary treatments to claim
insurance compensation

17. The decision to seek the Lok Sabha’s approval to withdraw the
legislation this week is a clear acknowledgement by the government
that it had underestimated the extent and intensity of public
opposition to the proposed law. One provision in the Bill had,
10 Adda247 Publications For any detail, mail us at
Publications@adda247.com
Cracker Book for Bank (IBPS | SBI | RRB PO | Clerk) Mains Exams

____________________________________ the “bail-in” clause. That banks, by the


very nature of their business, are essentially dependent on the funds
lent to them by depositors to serve as the pool of lendable resources.
(a) in particular, generated the greatest debate and attracted the
fiercest criticism and ultimately proved to be its very undoing
(b) explaining the rationale for the Bill as well as the built-in
“safeguards” relating to the bail-in provision
(c) resolution of these issues would require a comprehensive
examination and reconsideration
(d) such a review ought to include an evaluation of the progress made
by the Insolvency and Bankruptcy Code in addressing the crucial
issue of
(e) anomaly must be addressed, especially at a time when several
state-run public sector banks have been roiled by a series of
frauds and high levels of bad loans.

18. Michel Barnier, the European Union’s chief negotiator, suggested in a


newspaper article a softening of the EU’s position on
_______________________________________________ until a permanent solution is
found. Both the EU and the U.K. are against a hard border between
Ireland and Northern Ireland, key to the Good Friday Agreement that
has ensured peace on the island since 1998.
(a) a post-Brexit free trade area between Britain and the EU for goods
alone, leaving trade in services for a separate agreement.
(b) the Irish “backstop” — a temporary customs arrangement to
avoid a hard border between the Republic of Ireland and
Northern Ireland, until
(c) just as Ms. May was trying to win support on the continent, will
throw a spanner in the works for her
(d) a lot to gain from parts of the financial sector leaving the U.K. after
Brexit.
(e) how Britain proposes to achieve this while exiting the EU
Customs Union and Single Market.
11 Adda247 Publications For any detail, mail us at
Publications@adda247.com
Cracker Book for Bank (IBPS | SBI | RRB PO | Clerk) Mains Exams

19. The People’s Democratic Party, ________________________ when it won 32


of the 47 seats. At his party’s first rally on August 4, Mr. Tobgay touted
the 8% GDP growth in favour of his party, which has been fuelled by a
construction and tourist boom in Bhutan.
(a) this election comes after the 73-day India-China stand-off in 2017
in the Bhutan-claimed area of Doklam.
(b) religious figure in eastern Bhutan, has a similarly worded
campaign manifesto title: “For a self-reliant Bhutan: our concern,
our responsibility”.
(c) led by incumbent Prime Minister Tshering Tobgay, enters the
elections with a visible edge, even if it may not be able to better
its landslide victory in 2013,
(d) led to the country’s burgeoning national debt. Although the
government agreed to raise tariffs for the original hydropower
plant in Chukha in February this year, other tariffs will need to be
renegotiated too
(e) can also take credit for stabilising the rupee-ngultrum crisis that
he had inherited, as well as for economic reforms including lifting
the import ban on cars.

20. The newly appointed Governor of Jammu and Kashmir, Satya Pal
Malik is the first politician to be appointed to the position in over half
a century. ______________________________. Speaking exclusively to The
Hindu after meeting with President Ram Nath Kovind and Prime
Minister Narendra Modi, Mr. Malik said his brief was to “reach out to
the people of the State, to listen to them and after listening, do
whatever is in their interests.”
(a) He, however, did not comment on whether these “political”
leaders would include leaders of the Hurriyat Conference.
(b) Mr. Malik, in one of his first orders of business after taking over,
issued instructions that district collectors should earmark at least
one day in the week for public hearings.
(c) He, however, has said, “I may be a politician appointed as
governor, but I’m not in Jammu and Kashmir for politicking.”
(d) “I will be inviting leaders of political parties to meet with me and
if anyone feels uncomfortable to come to me, I will go to them,”
Mr. Malik said.
(e) None of the above
12 Adda247 Publications For any detail, mail us at
Publications@adda247.com
Cracker Book for Bank (IBPS | SBI | RRB PO | Clerk) Mains Exams

21. “We shall overcome, some day. Oh, deep in my heart, I do believe, We
shall overcome, some day,” sang Supreme Court judge, Justice Kurian
Joseph, for the distressed people of his native State Kerala.
______________________________. Midway through his rendition, the judge
urged the audience, comprising Chief Justice of India Dipak Misra and
his fellow judges, to sing along.
(a) However, it was the latest entrant to the Supreme Court, Justice
K.M. Joseph, who stole the show when judges, journalists, and
lawyers got together for the noble cause.
(b) Justice Kurian belted out the number in the company of
Bollywood singer Mohit Chauhan at a flood-relief fundraiser
organised by a group of journalists at the Indian Society of
International Law, a stone’s throw from the Supreme Court on
Monday.
(c) “They came in with their boats and saved many lives. It showed
their innate goodwill.”
(d) He credited Justice Kurian with bringing him on stage.
(e) None of the above

22. A special SIT court on Monday sentenced two more persons to life
imprisonment, while acquitting three others in the 2002 Godhra train
burning case in which 59 Hindu pilgrims returning from Ayodhya
were burnt alive.
______________________________. The court acquitted Hussain Suleman
Mohan, Kasam Bhamedi and Faruk Dhantiya for want of evidence. The
five were apprehended in 2015-2016 were tried in a special court set
up inside the Sabarmati central jail in Ahmedabad.
(a) As many as eight accused of the case are still absconding.
(b) Of the 31 convicts, 11 were awarded capital punishment, while 20
others were handed the life term in jail.
(c) In February 2002, a mob attacked Sabarmati Express in which
pilgrims were returning from Ayodhya.
(d) Special SIT court judge H.C. Vora awarded life sentence to Farooq
Bhana and Imran Sheru after the prosecution established their
role as conspirators in the burning of two coaches of the
Sabarmati Express.
(e) None of the above
13 Adda247 Publications For any detail, mail us at
Publications@adda247.com
Cracker Book for Bank (IBPS | SBI | RRB PO | Clerk) Mains Exams

23. A Delhi court has acquitted two members of the erstwhile Dal Khalsa
in a case of hijacking of a Delhi-Srinagar Indian Airlines flight to
Lahore in 1981. Five persons were accused in the case; three have
been declared absconders. ______________________________. Additional
Sessions Judge Ajay Pandey acquitted Tejinder Pal Singh and Satnam
Singh as the prosecution failed to prove the allegations of waging war
against the Government of India against them.
(a) According to statements by four of the six crew members which
became the basis for initiating prosecution, while hijacking the
plane with 111 passengers on board, the accused raised pro-
Khalistan slogans, such as “Khalistan Zindabad; Bhindranwale
Amar Rahe; Khalistan Lekar Rahenge”.
(b) Orders have been issued to initiate disciplinary action.
(c) The Army ordered a CoI and earlier this month its report reached
the Srinagar-based 15 Corps headquarters.
(d) Now the summary of evidence will be recorded after which the
framing of charges will be undertaken
(e) None of the above

24. Meghalaya CM and National People’s Party president Conrad K.


Sangma on Monday won the South Tura bypoll, defeating his nearest
Congress rival Charlotte W. Momin by over 8,421 votes. But the NPP’s
Martin K. Danggo lost the Ranikor Assembly seat to Pius Marwein of
the United Democratic Party by 2,896 votes. The NPP and the
Congress are now tied at 20 seats each in the 60-member House. Mr.
Sangma secured 13,656 votes while Ms. Momin polled 5,235 votes.
__________________________________. Two independents – John Leslee K.
Sangma and Chris Kabul A. Sangma – managed 3,108 votes between
them while NOTA, or none of the above, received 218 votes.
(a) Mr. Danggo resigned from the Congress in June to join the NPP.
(b) while Congress’ Jackiush A. Sangma finished fourth with 938
votes
(c) Incidentally, Mr. Danggo had won the Ranikor seat in February for
the fifth time on Congress ticket.
(d) The margin of victory was the highest for South Tura.
(e) None of the above
14 Adda247 Publications For any detail, mail us at
Publications@adda247.com
Cracker Book for Bank (IBPS | SBI | RRB PO | Clerk) Mains Exams

25. When Congress president Rahul Gandhi recently met about a hundred
women journalists, including those writing on gender issues, women’s
equality was to be a major point of discussion. Fortunately, we live in
a country ___________________________________________, ranging from war and
conflicts to politics and economics. And when a political leader meets
journalists, women or men, all kinds of current topics will be
discussed. So indeed, a myriad of them were discussed. Some of these
have also been reported in the media.
(a) where women journalists write on everything
(b) scholars speak about two kinds of ’empowerment’
(c) Satisfaction of the above needs is a necessary
(d) talking about women’s equality with
(e) feminine aspects of men and women

Solutions

1. (a); The statement just before the blank says that the star could not be
identified even with the best of the telescopes so it is obvious that
the next sentence would be how come they found the star which
has been explained in (a).
(b) cannot be the answer as it talks about newer discoveries
through gravitational lensing but not anything about ‘gravitational
lensing’ itself.
(d) cannot be the answer as the statement just before the blank
says that the star could not be identified even with the best of the
telescopes but (d) does not explain that if the best telescopes
cannot identify the star then how come Hubble telescope identified
it but (a) explains it.

2. (d); The paragraph talks about Mahatma Gandhi urging his mother and
wife to allow him to go to London. Therefore, the blank must be
filled with something related to that hence (d) fills the blank
appropriately as he was asking permission to leave from his wife
and mother and his wife allowed him in (d).
15 Adda247 Publications For any detail, mail us at
Publications@adda247.com
Cracker Book for Bank (IBPS | SBI | RRB PO | Clerk) Mains Exams

3. (b); Here choosing the correct answer between (b) and (c) can get very
difficult. We have to first read the passage very carefully and here
one can see that it is describing us what a banana is. It tells us that
it is an edible fruit, its colour, size, shape, starch content, where the
fruit grows in the plant, main species etc. therefore some sentence
which gives a proper starter for an introduction would fit in the
blank which is (b) as it gives the proper starter “what a banana is”.
(c) cannot be our answer as it does not represent a proper starter
instead it describes few types of bananas.

4. (c); Upon reading the passage it can be concluded that it is comparing


between 2 protein diets i.e. red meat and nuts and seeds and their
benefits of risks to CVD therefore the blank should be filled with a
sentence which compares the two diets (‘while’ is used for
comparing). The sentence after blank talks about good effects of
protein from nuts and seeds. Therefore, the sentence which talks
bad effects of red meat will fit the blank appropriately.

5. (c); The sentence before blank says that blue light exposure disrupts
our body’s ability to produce melatonin, a hormone that helps us
fall asleep, (c) is just the continuity of that statement that this
disruption affects our body’s clock which is the reason for affecting
our sleep hence (c) will fill the blank appropriately.

6. (a); The given paragraph is describing about the government’s


approval for cutting down 14000 trees for construction purpose.
Furthermore, it has mentioned the qualities of these trees and their
importance in the local area. Therefore, the most suitable sentence
that would complete the paragraph should be “However, the
project reports overlook these qualities” as, it is describing how the
project report failed to notice these important qualities. Moreover,
all the other sentences don’t comply with the theme of the
paragraph. Hence, option (a) is the most suitable answer choice.

16 Adda247 Publications For any detail, mail us at


Publications@adda247.com
Cracker Book for Bank (IBPS | SBI | RRB PO | Clerk) Mains Exams

7. (d); The paragraph has described about the problems associated with
the agricultural sector and how these problems can be turned into
new entrepreneurial opportunities. The sentence after the blank
describes the various entrepreneurial options derived from the
agricultural sector. Therefore, only option (d) suitably fits in the
paragraph. Hence, it becomes the most suitable answer choice.

8. (b); The paragraph is describing about the importance of corporate


social responsibility to transform the society especially the
education sector. The sentence before the blank mentions about
the policy of corporate social responsibility. Therefore, the most
coherent sentence that would fill the blank to complete the
paragraph is option (b) which is describing about the relationship
between the business and society. Hence, option (b) is the most
viable answer choice.

9. (a); The paragraph is describing about the government efforts towards


Universal Health coverage (UHC) in order to eradicate poverty and
gain healthcare benefits. Thus, the most suitable sentence that
would complete the paragraph is option (a) as it is describing
about the requirements to successfully implement UHC. All the
other sentences are irrelevant in context of the paragraph. Hence,
option (a) is the most suitable answer choice.

10. (c); Option (c) is the most appropriate choice. The paragraph talks
about the privatization of God, the supreme entity. As the second
line refers about how God cannot be controlled by certain
individuals or communities or some dominant males , only option
(c) fits as it mentions about how equivalently no human can claim
copyright over their ‘gods’.

11. (d); The sentence that must be filled in the given blank is option
(d).This is so because the paragraph is referring about the social
expenditure that was raised by the government which raised wages
in the informal sector. To portray its aftermaths the only option
that suits is option (d) , that is it increased minimum support
prices(MSP) for farmers .All other options do not satisfy the given
blank .Hence option (d) is the answer.
17 Adda247 Publications For any detail, mail us at
Publications@adda247.com
Cracker Book for Bank (IBPS | SBI | RRB PO | Clerk) Mains Exams

12. (b); Only option (b) fills the blank in the most appropriate way. As the
next line after the blank mentions about the specific exception, we
have to fill the blank with the exception that is referred. In option
(b), the only exception to the rule of anti –corruption law is given
that is, when one is forced to give a bribe. Hence option (b) is the
answer.

13. (c); The most suitable option here is option (c). As the paragraph talks
about macroeconomics, the branch of economics concerned with
large-scale or general economic factors, such as interest rates and
national productivity, which tend to advance, this is the only
statement among the given options which acts as an opener for the
paragraph. The next line also supports the first line which talks
about the economies which are basically self-correcting. Hence
option (c) is the appropriate answer.

14. (e); The most appropriate answer here that fills the blank is option
(e).This is so because the line before the blank talks about the
action which has a goal or destination. So the next line must
describe about how that action must be led to or in what way it
must work and that is described by the statement (e) .That is , one’s
concentration should not be diverted as it affects a goal ,once the
action starts. Hence option (e) is the answer.

15. (c); Option (d) and (e) can be easily omitted as they have no
information in context of any countries purchasing power option
(a) has no relevance here as Sensex can’t define some countries
purchasing power sensex is related to share markets. In option (b)
they are discussing the consumer and wholesale price inflation
easing in July.

18 Adda247 Publications For any detail, mail us at


Publications@adda247.com
Cracker Book for Bank (IBPS | SBI | RRB PO | Clerk) Mains Exams

16. (d); Option (c) and (e) can be easily omitted as there is no mention
either of any scheme or of any insurance for treatments. There is
no mention of any launch so option (a) omits itself. Option (d) is
the best answer for the fill in the blanks as ‘treatment’ and ‘care’
are quite co-related.

17. (a); Option (c) is talking about some issues but there are no such issues
discussed in paragraph. Similarly there is no review of anything as
stated in option (d). There is no such anomaly that has been
discussed in the paragraph so the best suitable option to fill with is
option (a).

18. (b); Option (b) is the best choice as it is explicitly, mentioned in the
paragraph as well as option, about the hard border between
Republic of Ireland and Northern Ireland.

19. (c); It is clear from the paragraph that The People’s Democratic Party,
led by Tobgay, as it is stated that he touted the 8% GDP growth so
the best answer to fill the paragraph with is (c).

20. (c); The sentence after the blank informs us that Mr. Malik started
sharing his opinions and making comments after the blank. So,
options (a) and (d), which highlights the statements made by Mr.
Malik, are wrong.
Now, the first sentence of the paragraph informs us that Mr. Malik
is the first politician to be chosen as the governor of the Jammu and
Kashmir. Option (c) adds further to the aspect (as conveyed by the
first sentence).
Hence, option (c) is the correct answer.

21. (b); The first sentence introduces a song which was sung by Justice
Kurian Joseph. ‘His’ in the third sentence after the blank is likely to
refer ‘Justice Kurian’. The third sentence, which is after the blank,
provides information about the circumstances/situations when the
song was sung. It is likely that the blank which precedes the third
sentence would also be talking about the circumstances when the
song was being sung.
19 Adda247 Publications For any detail, mail us at
Publications@adda247.com
Cracker Book for Bank (IBPS | SBI | RRB PO | Clerk) Mains Exams

Option (c) provides the quotation of the sentence made by


someone but doesn’t provide information on who made the
statement. It would be incoherent if the option (c) fills the blank.
Option (a) introduces another justice K.M. Joseph who is not
mentioned neither in the first sentence nor in the second sentence.
It would be incoherent if option (a) fills the blank.
Similarly, option (d) introduces a second person, with a pronoun
‘he’ when neither the sentence 1 nor the sentence 3rd gives any
suggestion if the 2nd sentence would abruptly and incoherently
introduce second person. So, option (d) is also wrong.
Option (b) which provides information on when, where, in whose
presence, was the song sung satisfy the contextual requirement of
the blank enforced due to the nature of the third sentence which
provides information on the circumstance or situation when the
song was sung.
Hence, Option (b) is the correct answer.

22. (d); The first sentence introduces the theme of the paragraph. The
theme of the paragraph is: sentencing of two more persons and
acquittal of three other persons.
The third sentence provide information about the names of the
persons who were acquitted. The fourth sentence provides
information about the place where were the five were tried. No
given sentences among the given paragraph provide information
about the names of the persons who were sentenced.
So, it would be more coherent, and is likely that the information
about the names of the person who were sentenced was provided
in the blank.
Among the options, the option (d) satisfy the contextual
requirement of the blank and is the correct answer.
20 Adda247 Publications For any detail, mail us at
Publications@adda247.com
Cracker Book for Bank (IBPS | SBI | RRB PO | Clerk) Mains Exams

23. (a); The first sentence provides information about the theme of the
paragraph which is acquittal of two members of the erstwhile Dal
Khalsa who were accused to be involved in the hijacking of an Indian
Airplane in 1981. The second sentence provides information about
the number of accused in the case and what happened with the
other three accused.
The fourth sentence provides information about the reason for the
acquittal of the two persons.
Options (b), (c) and (d) are irrelevant in the context of the
paragraph.
Only option (a) satisfies the contextual requirement of the blank
and is the correct answer.

24. (d); The first sentence of the paragraph introduces the theme of the
paragraph. The theme of the paragraph is wining of the by-poll
election at the South Tura by the Meghalaya’s CM and NPP
President Mr. Conrad K. Sangma. The second sentence of the
paragraph provides a contrasting information about another
member of the NPP, Martin K. Danggo, who lost the election at the
Ranikor Assembly seat. The third sentence of the paragraph
provides information about the status of seats at the Assembly. The
fourth sentence provides information about the number of votes
secured by Mr. Sangma and his opponent Mr. Momin at the South
Tura bypoll. The last sentence, which follows the blank, provides
information about the number of votes secured by the
independents at the South Tura bypoll.
The blank is not likely to provide information about who finished at
what position. So, the options (a), (b) and (c), which provide
information about the Ranikor assembly seats or the candidates
who contested election at the Ranikor assembly seat, are wrong.
The blank is likely to provide information about the bypoll election
at the South Tura assembly seat.
Among the options, option (d) satisfies the contextual requirement
of the paragraph and is the correct answer.

25. (a); The paragraph is describing about a press conference held by


Rahul Gandhi with the women journalists to discuss on current
topics. Thus, the most suitable phrase that will complete the
paragraph adhering to the theme of the paragraph is “where
women journalists write on everything”. Hence, option (a) is the
most suitable answer choice.
21 Adda247 Publications For any detail, mail us at
Publications@adda247.com
Cracker Book for Bank (IBPS | SBI | RRB PO | Clerk) Mains Exams

1 Adda247 Publications For any detail, mail us at


Publications@adda247.com
Cracker Book for Bank (IBPS | SBI | RRB PO | Clerk) Mains Exams

Chapter

15 Sentence Rearrangement
STUDY TIPS

In such questions, a paragraph with jumbled sentences is provided. The


best way to solve this question is to understand the tone and the context
of the sentences. In addition to that the knowledge of pronouns,
determiners and relative pronouns also helps to pair the coherent
sentences. Words like also, moreover, meanwhile, in addition to, however,
indicates that these sentences cannot introduce a paragraph. Pronouns
like he, she, it they should be follow the statements that include their
antecedents. It is comparatively easier to pair sentences and then arrange
them in the logical sequence.

Practice Exercise Based on new Pattern

Direction (1-6): Rearrange the following sentences to form a meaningful


paragraph and then answer the questions that follow.

1. If sentence (B) “Grain prices in India are dominated by the state, being
set by CACP reports, and with a major share of public purchase at
least in Northwest India and UP, the private trade dare not speculate.”
is the first sentence of the paragraph, then what is the sequence of
other sentences after rearrangement?
(A) This is also true for vegetables and fruits and for fish and forest
products. Marketing here is a big constraint.
(B) Grain prices in India are dominated by the state, being set by
CACP reports, and with a major share of public purchase at least
in Northwest India and UP, the private trade dare not speculate.
(C) The income elasticity of demand for animal husbandry products
like milk, cheese as also meat, chicken and eggs being in the
range of 1.5-2, demand rises by 10% or more with per-capita
income rising by 6%.

2 Adda247 Publications For any detail, mail us at


Publications@adda247.com
Cracker Book for Bank (IBPS | SBI | RRB PO | Clerk) Mains Exams

(D) But within grain, the problem was in pulses, and to an extent in
some inferior cereals like corn and bajra, also used as fodder. It
is the non-grain prices which are the problem.
(E) More than 40 lakh farmers moved to census towns, chasing the
demand for such products in the last decade, and one can be
sure that the number is more in this decade.
(F) APMCs are in any case strangled by the brokers as a number of
studies show. The supply chain is ruptured at a crucial point.
(G) These products are sold outside Agricultural Produce Market
Committees (APMC) and the infrastructure support to the kisan
is abysmal, as all field reports show.
(a) DCAEGF (b) ACDEFG (c) DCFGEA
(d) DEGAFC (e) DGECAF

2. If sentence (B), “Maharashtra also wants 5% of the seats in coaching


centres reserved for BPL students. Class-sizes are also to be capped,
and each centre will be subject to grading based on performance-
based audits every three years.” is the last sentence of the paragraph,
then which of the following sentences does not fit into the paragraph
formed after rearranging other sentences?
(A) It requires coaching centres to get a government licence and
seek a renewal every three year, while home tutors must get
their licences renewed every five years.
(B) Maharashtra also wants 5% of the seats in coaching centres
reserved for BPL students. Class-sizes are also to be capped, and
each centre will be subject to grading based on performance-
based audits every three years.
(C) The Maharashtra government’s proposal to regulate private
coaching centres and home tuition providers not only is
unwarranted, but also sets a dangerous precedent.
(D) Only the schools that have more seats in standard are compared
to their pre-primary section, will have to fill the remaining quota
seats.
(E) The state had also proposed making government approval of
fees charged mandatory, with the government having the
authority to reduce these.
3 Adda247 Publications For any detail, mail us at
Publications@adda247.com
Cracker Book for Bank (IBPS | SBI | RRB PO | Clerk) Mains Exams

(F) Surprisingly, even though the right to legal redress is a


fundamental right, the proposed regulation prescribes a ban on
coaching centres from approaching the high court and the
Supreme Court, reposing the final say with an appeals
committee formed by the government.
(a) A (b) C (c) E
(d) D (e) F

3. If Sentence (C), “Petrol and diesel prices have crept up slowly over the
last three months, but without inviting the kind of anger and criticism
usually directed at such fuel price hikes.” is the first sentence, what is
the order of other sentences after rearrangement?
(A) Daily pricing is now being seen by many as a ploy to increase
prices while allowing the government to escape any political
backlash.
(B) A comparison of crude oil prices with domestic petrol and diesel
prices, however, suggests that this argument is far from
convincing.
(C) Petrol and diesel prices have crept up slowly over the last three
months, but without inviting the kind of anger and criticism
usually directed at such fuel price hikes.
(D) The government, for now, has ruled out any change to the
current pricing policy arguing that it, in fact, ensures that the
benefit of lower international crude oil prices is passed on to
domestic consumers.
(E) In 2012, when India purchased a barrel of crude for around
$120, a litre of petrol was sold at around Rs.65 in retail fuel
stations.
(F) The price of petrol in Delhi, for instance, has cumulatively
increased by almost Rs.5 since the introduction of the daily
pricing policy on June 16 this year.
(a) FBAED (b) BFAED (c) EDAFB
(d) FADBE (e) DAEBF
4 Adda247 Publications For any detail, mail us at
Publications@adda247.com
Cracker Book for Bank (IBPS | SBI | RRB PO | Clerk) Mains Exams

4. If sentence (E), “This initial trend will need to be corroborated by


inflows for subsequent months, but with many more taxpayers
registering in August, the GST appears to have begun well as far as the
exchequer is concerned” is the last sentence of the paragraph, then
which of the following sentences does not fit into the paragraph
formed after rearranging other sentences?
(A) Revenue collections from the first month appear robust, with just
70% of eligible taxpayers bringing in Rs. 95,000 crore.
(B) The GST Council has already changed the announced tax rates on
over 100 products and services within about 75 days of the roll-
out.
(C) This would be significantly higher than the Rs.91,000 crore
indirect tax target for the Centre and the States on an overall
basis.
(D) At this rate, the total tally could well surge close to Rs.1.2 lakh
crore.
(E) This initial trend will need to be corroborated by inflows for
subsequent months, but with many more taxpayers registering in
August, the GST appears to have begun well as far as the
exchequer is concerned.
(F) India’s goods and services tax regime is nearing the end of its first
full quarter since roll-out this July.
(a) B (b) A (c) D
(d) E (e) C

5. If sentence (F), “To paraphrase a popular idiom, one may take


manufacturing out of China, but one cannot take China out of
manufacturing” is the LAST sentence of the paragraph, then which of
the following sentences does not fit into the paragraph formed after
rearranging other sentences?
(A) It would be folly to view the Harare meeting in isolation. It is a
result of years of hard work and politicking by the Chinese
government to present the yuan as a credible alternative to the US
dollar.
(B) After becoming the preferred trade partner for the African
continent, China’s ambitions have expanded to operate the
preferred reserve currency for nations in the region.
(C) Since then, Ghana, South Africa, Zimbabwe and, most recently,
Nigeria have entered into currency swap agreements with China
to reduce reliance on the US dollar.

5 Adda247 Publications For any detail, mail us at


Publications@adda247.com
Cracker Book for Bank (IBPS | SBI | RRB PO | Clerk) Mains Exams

(D) This strategy could have significant consequences at a time when


Africa is being touted as the “next factory of the world” after
China, being developed through Chinese loans which are likely to
be repaid in Chinese currency.
(E) Not too long ago, in December 2015, the yuan was inducted as
one of the currencies in which special drawing rights could be
exercised—making it one of the most reliable currencies in the
world.
(F) To paraphrase a popular idiom, one may take manufacturing out
of China, but one cannot take China out of manufacturing.
(G) India is likely to use a gradual approach in pursuit of
internationalization of the rupee, despite increasing competition
from the yuan.
(a) A (b) E (c) D
(d) B (e) G

6. If sentence (A), “The RBI governor Urjit Patel has sought more
legislative powers to effectively regulate state-owned banks.” is the
first sentence of the paragraph, then what is the sequence of other
sentences after rearrangement?
(A) The RBI governor Urjit Patel has sought more legislative powers
to effectively regulate state-owned banks.
(B) The reform will enable the regulator to take legal action against
any errant PSB, level the playing field between private and public-
sector banks and raise corporate governance.
(C) The regulator is in talks with the government on this issue, and
that’s welcome. The RBI’s remit should be in charge of bank
supervision, and not operations.
(D) His plea to the Parliamentary Standing Committee on Finance to
make banking regulatory powers ownership-neutral makes sense.
(E) The RBI should be able to exercise the same supervisory powers
over state-owned banks as over private sector banks.
(F) The RBI should also stop the practice of appointing its nominees
on the boards of PSBs, given that there is a conflict of interest
with its supervisory role.
(a) FEDCB (b) EDBCF (c) DEFCB
(d) DEBFC (e) EBDFC
6 Adda247 Publications For any detail, mail us at
Publications@adda247.com
Cracker Book for Bank (IBPS | SBI | RRB PO | Clerk) Mains Exams

Directions (7-11): Answer the following questions after rearranging the


following sentences into a coherent paragraph and identify the sentence
that doesn’t fit in the context of the paragraph.
(A) Through a series of complex deals thereafter, the company that
extended the loan got wholly acquired by a trust controlled by Deepak
Kochhar in April 2013.
(B) There’s no rule preventing a top banker’s or bureaucrat’s relative
from doing business.
(C) The case of ICICI Bank — involving its managing director and CEO
Chanda Kochhar’s husband, Deepak, and the Videocon Group’s
chairman Venugopal Dhoot — is a textbook example of this.
(D) But a potential conflict of interest situation arises when that relative’s
dealings are with someone having significant commercial transactions
with the bank or government department concerned.
(E) Well before that, Dhoot had also sold his 50 per cent stake in the
renewables joint venture for a paltry Rs 2.5 lakh.
(F) What’s not in doubt is a clear conflict of interest and the bank’s
seeming reluctance to get to the bottom of it
(G) As an investigation by The Indian Express has revealed, a Dhoot-
owned company, in March 2010, loaned Rs 64 crore to a renewables
firm that he and Deepak Kochhar had jointly promoted only in late-
2008.

7. Considering the statement (C), “The case of ICICI Bank — involving


its managing director and CEO Chanda Kochhar’s husband,
Deepak, and the Videocon Group’s chairman Venugopal Dhoot —
is a textbook example of this.” as the third statement of the
paragraph after the rearrangement. Which one among the following
statements does not belong to the coherent paragraph?
(a) D (b) C (c) G
(d) A (e) F
7 Adda247 Publications For any detail, mail us at
Publications@adda247.com
Cracker Book for Bank (IBPS | SBI | RRB PO | Clerk) Mains Exams

8. Considering statement (C), “The case of ICICI Bank — involving its


managing director and CEO Chanda Kochhar’s husband, Deepak,
and the Videocon Group’s chairman Venugopal Dhoot — is a
textbook example of this.” the third statement of the paragraph after
the rearrangement (excluding the incoherent statement), which one
of the following should follow the last statement of the paragraph?
(a) The Videocon loan became a non-performing asset (NPA), but
that, by itself, cannot have been ascribed to any nepotism or quid
pro quo.
(b) What’s more, even the transfer of shares in the lending company
to Deepak Kochhar’s trust was made at just Rs 9 lakh.
(c) It is easy to blame the government of the day, the boards that
conduct the examinations, the unscrupulous exam mafias that
provide imposters and technological cheating tools for a hefty
price.
(d) A knee-jerk press release in defence of Chanda Kochhar and
talking assurances by the bank’s board do not make for a credible
institutional response.
(e) None of these

9. If statement (C), “The case of ICICI Bank — involving its


managing director and CEO Chanda Kochhar’s husband,
Deepak, and the Videocon Group’s chairman Venugopal Dhoot
— is a textbook example of this.” is the third statement of the
paragraph after the rearrangement (excluding the incoherent
statement), which one among the following pairs is consecutively
related with each other?
(a) G-A (b) B-G (c) E-F
(d) C-A (e) A-F

10. If statement (C), “The case of ICICI Bank — involving its


managing director and CEO Chanda Kochhar’s husband,
Deepak, and the Videocon Group’s chairman Venugopal Dhoot
— is a textbook example of this.” is the third statement of the
paragraph after the rearrangement (excluding the incoherent
statement). Which one should be the first statement of the
paragraph after the rearrangement?
(a) E (b) F (c) D
(d) B (e) A
8 Adda247 Publications For any detail, mail us at
Publications@adda247.com
Cracker Book for Bank (IBPS | SBI | RRB PO | Clerk) Mains Exams

11. Considering statement (C) “The case of ICICI Bank — involving its
managing director and CEO Chanda Kochhar’s husband, Deepak,
and the Videocon Group’s chairman Venugopal Dhoot — is a
textbook example of this.” as the third sentence of the paragraph
after the rearrangement (excluding the incoherent statement), what
should be the logical and meaningful sequence of the rearranged
paragraph after eliminating the incoherent statement?
(a) BECABD (b) EGCABD (c) BDCGAE
(d) AFCEBD (e) DBCGAF

Directions (12-16): Answer the following questions after rearranging the


following sentences into a coherent paragraph and identify the sentence
that doesn’t fit in the context of the paragraph.
(A) Overall, during the pendency of the case, political interests have over-
ridden the principle of equity and justice.
(B) Since it is apparent that there is no meeting point between the
warring States and whoever is at the Centre prefers to play vote bank
politics, the Supreme Court should constitute a panel comprising
retired judges and experts on the subject to monitor the
implementation of its order and ensure equitable distribution of
water till a consensus is arrived at by all the parties concerned.
(C) The long-drawn dispute in the case of sharing Cauvery waters among
the respective southern States will go down in the annals of Indian
judicial history as one where court directives have been flouted with
impunity from time to time.
(D) The river's basin covers three states and a Union Territory as follows:
Tamil Nadu, 43,856 square kilometres (16,933 sq mi); Karnataka,
34,273 square kilometres (13,233 sq mi); Kerala, 2,866 square
kilometres (1,107 sq mi), and Puducherry, 160 square kilometres (62
sq mi).
(E) After having been accused of being lax in its approach, the Tamil Nadu
government is now left with no other option than to file a contempt
plea.
(F) It is no surprise that with both the BJP and the Congress having high
stakes as major players in Karnataka’s Assembly election and with
nothing to lose in Tamil Nadu, the State is deprived of their firm
support.
9 Adda247 Publications For any detail, mail us at
Publications@adda247.com
Cracker Book for Bank (IBPS | SBI | RRB PO | Clerk) Mains Exams

12. Considering the statement (C), “The long-drawn dispute in the case
of sharing Cauvery waters among the respective southern States
will go down in the annals of Indian judicial history as one where
court directives have been flouted with impunity from time to
time.” as the first statement of the paragraph after the rearrangement
of sentences. Which one among the following doesn’t relate
coherently with the theme of the paragraph?
(a) B (b) A (c) E
(d) D (e) F

13. If the statement (C), “The long-drawn dispute in the case of sharing
Cauvery waters among the respective southern States will go
down in the annals of Indian judicial history as one where court
directives have been flouted with impunity from time to time.” is
the first statement of the paragraph after the rearrangement
(excluding the incoherent statement), which one among the following
pairs is consecutively related with each other?
(a) B-D (b) A-B (c) D-A
(d) A-F (e) E-B

14. Considering the statement (C), “The long-drawn dispute in the case
of sharing Cauvery waters among the respective southern States
will go down in the annals of Indian judicial history as one where
court directives have been flouted with impunity from time to
time.” as the first statement of the paragraph which one among the
following may fit in between the first and second statements of the
rearranged paragraph without altering the context of the paragraph?
(a) Dams, such as the Krishna Raja Sagara Dam, Mettur Dam, and
those on its tributaries such as Gorur dam, Harangi dam, Kabini
dam, Amaravati dam and Banasura Sagar Dam store water from
monsoon periods and release the water during the dry months.
(b) Rising in southwestern Karnataka, it flows southeast some 800
kilometres (500 mi) to enter the Bay of Bengal.
(c) The centre had told the court that the Cauvery water sharing is a
sensitive subject and would have an impact on the Karnataka
election.
(d) Chief Justice of India Dipak Misra said: "We understand the
problems of Tamil Nadu. We will solve the issue."
(e) None of these

10 Adda247 Publications For any detail, mail us at


Publications@adda247.com
Cracker Book for Bank (IBPS | SBI | RRB PO | Clerk) Mains Exams

15. Considering the statement (C), “The long-drawn dispute in the case
of sharing Cauvery waters among the respective southern States
will go down in the annals of Indian judicial history as one where
court directives have been flouted with impunity from time to
time.” as the first statement of the paragraph which one among the
following may replace statement (B) of the paragraph?
(a) A dam called the Grand Anicut was built in the 2nd century at the
point where the river divides.
(b) Agreeing to take up Tamil Nadu's petition for contempt against
the central government over the Cauvery water dispute, the
Supreme Court today said: "We will see that Tamil Nadu gets
water".
(c) The verdict, ahead of the Karnataka elections, was claimed by the
ruling Congress as a big win.
(d) It rises on Brahmagiri Hill of the Western Ghats in southwestern
Karnataka state, flows in a southeasterly direction for 475 miles
(765 km) through the states of Karnataka and Tamil Nadu, and
descends the Eastern Ghats in a series of great falls.
(e) None of these

16. Considering the statement (C), “The long-drawn dispute in the case
of sharing Cauvery waters among the respective southern States
will go down in the annals of Indian judicial history as one where
court directives have been flouted with impunity from time to
time.” as the first sentence of the paragraph after the rearrangement
(excluding the incoherent statement), what should be the logical and
meaningful sequence of the rearranged paragraph after eliminating
the incoherent statement?
(a) CFEBA (b) CDABF (c) CEABD
(d) CFEAD (e) CEAFB
11 Adda247 Publications For any detail, mail us at
Publications@adda247.com
Cracker Book for Bank (IBPS | SBI | RRB PO | Clerk) Mains Exams

Directions (17-21): Answer the following questions after rearranging the


following sentences into a coherent paragraph and identify the sentence
that doesn’t fit into the context of the paragraph.
(A) Atalji irreversibly changed India’s place in the world. He overcame the
hesitation of our nation, the resistance of the world and threat of
isolation to make India a nuclear weapons power.
(B) Equally important, he then brought to bear his extraordinary
understanding of world affairs and formidable diplomatic skills to
gain global acceptance of new realities.
(C) It was not a decision he took lightly, but one he knew was of
paramount importance in the face of mounting challenges to India’s
security.
(D) No longer would India’s security be vulnerable. At that moment of
surge in national pride, his was a voice of restraint and responsibility.
And, the world listened to the wisdom of the man of peace.
(E) Born into a family of modest means and high ideals, he hailed from a
small town in MP.
(F) Indeed, it is the combination of his legacies of creating strategic
capabilities, promoting stronger economic growth, undertaking multi-
directional diplomacy and harnessing of diaspora energies that is
today the basis for the respect we command across the world.

17. Considering statement (A) “Atalji irreversibly changed India’s place in


the world. He overcame the hesitation of our nation, the resistance of
the world and threat of isolation to make India a nuclear weapons
power” as the first sentence of the rearranged paragraph, then which
among the following fails to become the part of the coherent
paragraph?
(a) C (b) D (c) B
(d) F (e) E

18. Among the following pairs which one of them is formed with two
consecutive statements after the rearrangement?
(a) D – F (b) C – D (c) A - B
(d) C – B (e) A – F

12 Adda247 Publications For any detail, mail us at


Publications@adda247.com
Cracker Book for Bank (IBPS | SBI | RRB PO | Clerk) Mains Exams

19. Considering statement (A) “Atalji irreversibly changed India’s place in


the world. He overcame the hesitation of our nation, the resistance of
the world and threat of isolation to make India a nuclear weapons
power” as the first sentence of the rearranged paragraph, identify the
correct sequence of the sentences to form a coherent paragraph
(excluding the incoherent one).
(a) ABCDF (b) AFBCD (c) ADCFB
(d) ACDBF (e) ABDCF

20. Considering statement (A) “Atalji irreversibly changed India’s place in


the world. He overcame the hesitation of our nation, the resistance of
the world and threat of isolation to make India a nuclear weapons
power” as the first sentence of the rearranged paragraph, which of the
following statement should FOURTH sentence after the
rearrangement? (Excluding the incoherent sentence)
(a) B (b) H (c) E
(d) G (e) F

21. Considering statement (A) “Atalji irreversibly changed India’s place in


the world. He overcame the hesitation of our nation, the resistance of
the world and threat of isolation to make India a nuclear weapons
power” as the first sentence of the rearranged paragraph, which of the
following statement should SECOND sentence after the
rearrangement? (Excluding the incoherent sentence)
(a) B (b) H (c) E
(d) C (e) F

Directions (22-24): Rearrange the following sentences to form a


meaningful paragraph and then answer the questions that follow.
22. (A) The nature of threats is such that they continue to evolve all the
time.
(B) Both the 2001 terror attack in New York and the November 2008
attack in Mumbai were one of a kind with few parallels at the time.
(C) Anticipating an attack of this nature remains in the area of an
“intelligence gap” rather than an “intelligence failure”.
(D) The real problem is that when dealing with terrorism and terror
networks, no two situations in the actual world are identical.
13 Adda247 Publications For any detail, mail us at
Publications@adda247.com
Cracker Book for Bank (IBPS | SBI | RRB PO | Clerk) Mains Exams

(E) Most experts explain an intelligence gap as one denoting an


absence of intelligence output while an intelligence failure is one
where, based on available evidence, no warning was issued.
(F) Even while the IS has gained a great deal of prominence due to its
brand of violence, other terror networks have continued to be no
less active.
If the sentence (D), “The real problem is that when dealing with
terrorism and terror networks, no two situations in the actual
world are identical.” is the first sentence of the paragraph, then
which of the following sentences does not fit into the paragraph
formed after rearranging other sentences?
(a) A (b) E (c) C
(d) B (e) F

23. (A) A number of viruses, malware and cryptoworms are also being
developed in the JavaScript, which gives the attackers cross-
platform options.
(B) The attacks aren’t limited to mobile phones and e-Pads.
(C) While Windows operating systems were the most vulnerable to
cyberattacks, a number of Android threats have been reported in
the last couple of years, including potent crypto-ransomware
attacks on Android devices.
(D) In 2016, the first known Ransomware, named KeRanger, targeting
Mac users was also reported.
(E) All devices, including televisions that use Android, are also
potentially vulnerable.
(F) The Mirai botnet malware affected 2.5 million home router users
and other Internet of Things devices.
If the sentence (C), “While Windows operating systems were the
most vulnerable to cyberattacks, a number of Android threats have
been reported in the last couple of years, including potent crypto-
ransomware attacks on Android devices.” is the first sentence of
the paragraph, then what is the sequence of other sentences after
rearrangement?
(a) DEFAB (b) BEDFA (c) BADFE
(d) AFBED (e) ADBEF
14 Adda247 Publications For any detail, mail us at
Publications@adda247.com
Cracker Book for Bank (IBPS | SBI | RRB PO | Clerk) Mains Exams

24. (A) Funding varies with the political climate: there will be money to
buy equipment but no certainty that resources will flow for all the
years needed to ensure significant results.
(B) But, Mr. Bhattacharjee says, “Working with hands is not
encouraged among scientists. The words used in Indian labs are:
one needs hands to do experiments, not brains.”
(C) To succeed, experiments require at least two conditions:
guarantees of long-term funding and scientists’ collaboration with
each other.
(D) Lab assistants are the hands, while scientists avoid what they
regard as mere manual labour.
(E) Experimental science “is very poor in India”.
(F) And collaboration is a social process, not an intellectual one.
(G) It involves, among other things, physical labour together with
others.
If the sentence (F), “And collaboration is a social process, not an
intellectual one.” is the fourth sentence of the paragraph, then
which of the following options indicates the follow-up sentence i.e
the FIFTH sentence of the coherent paragraph so formed?
(a) A (b) C (c) G
(d) B (e) D

Direction (25): The sentences given in each of the following questions,


when properly sequenced, form a coherent paragraph. Each sentence is
labeled with a letter. From among the five choices given below each
question, choose the most logical order of sentences that construct a
coherent paragraph.

25. (A) However, "motivation" which drives our voluntary behaviours can
be said to be the strongest driving factor of all.
(B) Motivation, previous academic record, self-financing, quality of
teaching and nature of class lectures are factors that influence
attendance.
(C) The control that students have over their action, their
environment, determines their choice.
(D) The level of motivation and the class context determines whether
one wishes to attend a class or not.
(E) Another critical factor that drives attendance is quality of teaching,
which varies from brilliant to sadly, pathetic.
(a) ABDCE (b) BADCE (c) CBDAE
(d) DBCEA (e) EBDCA
15 Adda247 Publications For any detail, mail us at
Publications@adda247.com
Cracker Book for Bank (IBPS | SBI | RRB PO | Clerk) Mains Exams

Directions (26-30): Rearrange the following six sentences (A), (B), (C), (D),
(E) and (F) in the proper sequence to form a meaningful paragraph and
then answer the questions given below.
A. In 2011, he presented the Leonard Hastings Schoff Memorial Lectures
at Columbia, and these form the basis of this posthumous book, an
analysis of plot in drama and fiction.
B. Foundational largesse on such a scale certainly reinforces the
impression that the life of a tenured professor at a major university
resembles one long holiday, some of it spent with a beautiful view of
Lake Como.
C. Not to sound philistine, but this seems an inordinate amount of money
and time off, especially given the relatively small scholarly return.
D. According to the introduction by Russian scholar Robin Feuer Miller,
“Plots,” though brief, reflects many years of thought and research.
E. A prefatory note by Belknap duly thanks “the Woodrow Wilson
Fellowship Program, the Bellagio Center, the Kennan Institute at the
Smithsonian Institute, the John Simon Guggenheim Foundation, the
National Endowment for the Humanities, and the International
Research and Exchanges Board.”
F. Robert L. Belknap was a longtime Columbia University professor and
the author of two important studies of Dostoevsky’s “Brothers Kara-
mazov.”

26. Which of the following should be the FIRST sentence after


rearrangement?
(a) A (b) B (c) C
(d) D (e) F

27. Which of the following should be the FIFTH sentence after


rearrangement?
(a)A (b) B (c) C
(d) D (e) F

28. Which of the following should be the THIRD sentence after


rearrangement?
(a) A (b) B (c) C
(d) D (e) F
16 Adda247 Publications For any detail, mail us at
Publications@adda247.com
Cracker Book for Bank (IBPS | SBI | RRB PO | Clerk) Mains Exams

29. Which of the following should be the FOURTH sentence after


rearrangement?
(a) A (b) B (c) C
(d) D (e) E

30. Which of the following should be the SIXTH (last) sentence after
rearrangement?
(a) A (b) B (c) C
(d) D (e) E

Solutions

1.(a); As mentioned in the question, sentence (B) stands as the first


statement of the coherent paragraph. It is describing that the
pricing of grains is largely influenced by the states and the public
purchase. Therefore, the next statement that should follow
sentence (B) must further describe about the pricing of grains. Only
sentence (D) is describing about the problem associated with the
pricing of pulses and cereals. Moreover, it also provides a hint for
the next statement that should follow it by mentioning the problem
associated with the non-grain prices. So, the next statement that
should consecutively follow statement (D) must contain
information on the non-grain products. It is to be noted that
statement (C) asserts information on the non-grain i.e., animal
husbandry products, therefore, it logically connects with the
previous statement i.e., statement (D). Moreover, the starter of the
sentence (A) “This is also true for vegetables…” provides a hint that
it forms a pair with statement (C) as the term ‘also’ indicates that
the fact associated with the animal husbandry products is similar
with vegetables and other products. Sentence (A) also mentions
about the limitation of marketing in these products. Sentence (E)
elongates the information on chasing the demand by farmers, thus,
it forms coherent pair with sentence (A). Moreover, sentence (G)

17 Adda247 Publications For any detail, mail us at


Publications@adda247.com
Cracker Book for Bank (IBPS | SBI | RRB PO | Clerk) Mains Exams

illustrates about the poor infrastructure facility that acts as a


hindrance for the farmers to sell their products. Lastly, statement
(F) has further provided information on the disrupted supply chain
available to farmers for their products. Therefore, the correct
sequence of the statements to form a coherent paragraph is
BDCAEGF. All the other sequences fail to logically connect the
sentences, hence, option (a) becomes the most viable answer
choice.

2. (d); The paragraph after the rearrangement is describing about the new
proposal of Maharashtra government regarding the policies to
regulate and control private coaching centers. All the sentences
together are forming a coherent paragraph except for statement
(D). Sentence (D) has mentioned about admissions in pre-primary
section through quotas which is irrelevant in the context of the
coherent paragraph. Therefore, the correct sequence of the
rearranged paragraph is CAEFB. Hence option (d) is the most
viable answer choice.

3. (d); As mentioned in the question, if sentence (C) is the first sentence


then the correct order of other sentences after rearrangement is
FADBE.

4. (a); As mentioned in the question, sentence (E) “This initial trend will
need to be corroborated by inflows for subsequent months, but
with many more taxpayers registering in August, the GST appears
to have begun well as far as the exchequer is concerned” is the last
sentence of the paragraph. Thus FADCE forms a coherent
paragraph. The paragraph talks about the revenue collected after
GST tax regime which was higher than the target for the Centre and
the States on an overall basis while sentence (B) talks about
already changed announced tax rates by the GST Council which
seems to be unrelated with other sentences. Hence sentence (B) is
not a part of the paragraph so formed.
18 Adda247 Publications For any detail, mail us at
Publications@adda247.com
Cracker Book for Bank (IBPS | SBI | RRB PO | Clerk) Mains Exams

5. (e); The paragraph after the rearrangement is describing about the


position of china’s currency i.e., yuan in the global level in a
comparison with dollars. It further describes about china’s
intention to become the preferred currency of trade among many
countries. However, sentence (G) is describing about the India’s
approach to pursue internationalization of its currency in a
comparison with yuan. Therefore, the logical sequence of the
coherent sentences is AECBDF. Thus, with the elimination of
statement (G), option (e) becomes the most viable answer choice.

6. (d); The first given sentence provides a clue for the theme of the
paragraph which is about the new legislative powers desired by
Urjit patel to regulate public sector banks. Following the idea,
statement (D) and (E) forms a logical pair, which should be
followed by the pair of statements (B) and (F). Moreover,
statements (C) expresses the conclusion for issue, which
completely satisfies the criterion for the last statement. Thus,
considering statement (A) “The RBI governor Urjit Patel has sought
more legislative powers to effectively regulate state-owned banks”
is the first sentence of the coherent paragraph the correct sequence
of other sentences after rearrangement is DEBFC. Hence, option (d)
is the most suitable answer choice.

7. (e); The third statement i.e., statement (C) of the paragraph clearly
indicates the theme of the paragraph which is about a case stating
the issue of continuing the business by a top banker’s or
bureaucrat’s relative. However, statement (F) “What’s not in
doubt is a clear conflict of interest and the bank’s seeming
reluctance to get to the bottom of it” describes about the conflict of
interest of the banks which finds no relevance with the facts
discussed in the paragraph. Thus, among all the given options only
statement (F) cannot fit contextually with the theme of the
paragraph. Therefore, the option (e) becomes the most suitable
answer choice.

19 Adda247 Publications For any detail, mail us at


Publications@adda247.com
Cracker Book for Bank (IBPS | SBI | RRB PO | Clerk) Mains Exams

8. (b); The third statement i.e., statement (C) of the paragraph clearly
indicates the theme of the paragraph which is about a case stating
the issue of continuing the business by a top banker’s or
bureaucrat’s relative. Taking a hint from the theme of the
paragraph and eliminating the incoherent statement i.e., statement
(F) the sentences can be arranged in the sequence of BDCGAE to
form a coherent paragraph. As, the last statement of the paragraph
is stating a fact regarding the case of ICICI Bank — involving its
managing director and CEO Chanda Kochhar’s husband, Deepak,
and the Videocon Group’s chairman Venugopal Dhoot where the
50% of the stakes have been transefered for a very small amount
Rs. 2.5 L. Therefore, only option (b) continues to state the fact
describing that transfer of shares were also made at just Rs. 9 L. All
the other options are either irrelevant or doesn’t fit into the
coherence of the paragraph. Hence, option (b) becomes the most
suitable choice.

9. (a); The third statement i.e., statement (C) of the paragraph clearly
indicates the theme of the paragraph which is about a case stating
the issue of continuing the business by a top banker’s or
bureaucrat’s relative. Taking a hint from the theme of the
paragraph and eliminating the incoherent statement i.e., statement
(F) the sentences can be arranged in the sequence of BDCGAE to
form a coherent paragraph. Statements (G) and (A) are describing
about the facts associated with the case of ICICI bank. As, statement
(G) is introducing the case, statement (A) should follow it.
Therefore, option (a) becomes the most suitable answer choice.

10. (d); The third statement i.e., statement (C) of the paragraph clearly
indicates the theme of the paragraph which is about a case stating
the issue of continuing the business by a top banker’s or
bureaucrat’s relative. Taking a hint from the theme of the
paragraph and eliminating the incoherent statement i.e.,
statement (F) the sentences can be arranged in the sequence of
BDCGAE to form a coherent paragraph. Hence, as the first
statement of the paragraph is statement (B), option (d) becomes
the most viable answer choice.
20 Adda247 Publications For any detail, mail us at
Publications@adda247.com
Cracker Book for Bank (IBPS | SBI | RRB PO | Clerk) Mains Exams

11. (c); The third statement i.e., statement (C) of the paragraph clearly
indicates the theme of the paragraph which is about a case stating
the issue of continuing the business by a top banker’s or
bureaucrat’s relative. Taking a hint from the theme of the
paragraph and eliminating the incoherent statement i.e., statement
(F) the sentences can be arranged in the sequence of BDCGAE to
form a coherent paragraph.

12. (d); As statement (C) is the first sentence of the rearranged paragraph,
it clearly indicates the theme of the paragraph which is about the
dispute regarding the distribution of Cauvery water among
Southern states. However, after carefully reading the sentences it
can be noted that sentence (D) “The river's basin covers three
states and a Union Territory as follows: Tamil Nadu, 43,856 square
kilometres (16,933 sq mi); Karnataka, 34,273 square kilometres
(13,233 sq mi); Kerala, 2,866 square kilometres (1,107 sq mi), and
Puducherry, 160 square kilometres (62 sq mi)” is describing about
the coverage of Cauvery river flowing through different states.
Therefore, the correct sequence after rearranging the sentences is
CEAFB after eliminating the sentence (D) as the incoherent one.
Therefore, option (d) becomes the most suitable answer choice.

13. (d); The first statement i.e., statement (C) of the paragraph clearly
indicates the theme of the paragraph which is about the dispute
regarding the distribution of Cauvery water among Southern
states. Taking a hint from the theme of the paragraph and
eliminating the incoherent statement i.e., statement (D) the
sentences can be arranged in the sequence of CEAFB to form a
coherent paragraph. Statements (A) and (F) are describing about
the political interests of the parties of the states regarding the
distribution of the Cauvery river water among the states.
Therefore, as statement (F) consecutively follows statement (A),
option (d) becomes the most suitable answer choice.

21 Adda247 Publications For any detail, mail us at


Publications@adda247.com
Cracker Book for Bank (IBPS | SBI | RRB PO | Clerk) Mains Exams

14. (e); The first statement i.e., statement (C) of the paragraph clearly
indicates the theme of the paragraph which is about the dispute
regarding the distribution of Cauvery water among Southern
states. Taking a hint from the theme of the paragraph and
eliminating the incoherent statement i.e., statement (D) the
sentences can be arranged in the sequence of CEAFB to form a
coherent paragraph. Therefore, statements (C) and (E) are
describing about the plea to file on the controversy regarding the
distribution of Cauvery river’s water. Thus, the statement that may
come in between them should also be in similar context. None of
the given options is satisfying the ongoing context of the paragraph.
Therefore, option (e) becomes the most suitable answer choice.

15. (b); The first statement i.e., statement (C) of the paragraph clearly
indicates the theme of the paragraph which is about the dispute
regarding the distribution of Cauvery water among Southern
states. Taking a hint from the theme of the paragraph and
eliminating the incoherent statement i.e., statement (D) the
sentences can be arranged in the sequence of CEAFB to form a
coherent paragraph. Sentence (B) can be replaced with option (b)
“Agreeing to take up Tamil Nadu's petition for contempt against the
central government over the Cauvery water dispute, the Supreme
Court today said: "We will see that Tamil Nadu gets water".” as it is
expressing the response of the court on the petition filed against
the central government over Cauvery dispute. Hence, option (b)
becomes the most viable answer choice.

16. (e); The first statement i.e., statement (C) of the paragraph clearly
indicates the theme of the paragraph which is about the dispute
regarding the distribution of Cauvery water among Southern
states. Taking a hint from the theme of the paragraph and
eliminating the incoherent statement i.e., statement (D) the
sentences can be arranged in the sequence of CEAFB to form a
coherent paragraph. Hence, option (e) becomes the most suitable
answer choice.
22 Adda247 Publications For any detail, mail us at
Publications@adda247.com
Cracker Book for Bank (IBPS | SBI | RRB PO | Clerk) Mains Exams

17. (e); Drawing a hint from the first sentence, the theme of the paragraph
after rearrangement can be articulated that it is describing about
the reforms brought by Atalji in the field of nation’s security by
wise decisions he made during the course of his leadership.
Sentence (C) logically connects with sentence (A) as it is further
mentioning about his decision mentioned in sentence (A).
Moreover, sentences (C) and (D) forms a coherent pair as they are
illustrating about the importance of the decision made by him
regarding the nation’s security. Furthermore, the next sentence
that connects logically is sentence (B). This can be understood from
the beginning of the sentence “Equally important…” which
indicates that it is the second part of the paragraph that describes
the policies implemented by him using his wisdom. The last
sentence in the sequence should be sentence (F) as it sums up the
qualities of Atalji that helped the nation to prosper and stay strong
across the world. However, sentence (E) fails to become the part of
the paragraph as it is providing information of his personal life
while the paragraph is illustrating about his qualities that helped
him to formulate and implement wise policies to make India
stronger across the world. Therefore, the logical and meaningful
sequence of the sentences thus formed to mold them into a
coherent paragraph is ACDBF. Since, sentence (E) is incoherent;
option (e) becomes the most viable answer choice.

18. (b); Sentences (C) and (D) form a coherent pair as they are illustrating
about the importance of the decision made by Atalji regarding the
nation’s security. The logical and meaningful sequence of the
sentences formed to mold them into a coherent paragraph is
ACDBF. Hence, option (b) is the most suitable answer choice.

19. (d); Drawing a hint from the first sentence, the theme of the paragraph
after rearrangement can be articulated that it is describing about
the reforms brought by Atalji in the field of nation’s security by
wise decisions he made during the course of his leadership.
Sentence (C) logically connects with sentence (A) as it is further
mentioning about his decision mentioned in sentence (A).
23 Adda247 Publications For any detail, mail us at
Publications@adda247.com
Cracker Book for Bank (IBPS | SBI | RRB PO | Clerk) Mains Exams

Moreover, sentences (C) and (D) forms a coherent pair as they are
illustrating about the importance of the decision made by him
regarding the nation’s security. Furthermore, the next sentence
that connects logically is sentence (B). This can be understood from
the beginning of the sentence “Equally important…” which
indicates that it is the second part of the paragraph that describes
the policies implemented by him using his wisdom. The last
sentence in the sequence should be sentence (F) as it sums up the
qualities of Atalji that helped the nation to prosper and stay strong
across the world. However, sentence (E) fails to become the part of
the paragraph as it is providing information of his personal life
while the paragraph is illustrating about his qualities that helped
him to formulate and implement wise policies to make India
stronger across the world. Therefore, the logical and meaningful
sequence of the sentences thus formed to mold them into a
coherent paragraph is ACDBF. Hence, option (d) is the most
suitable answer choice.

20. (a); The logical and meaningful sequence of the sentences thus formed
to mold them into a coherent paragraph is ACDBF. Hence, option
(a) is the most suitable answer choice.

21. (d); The logical and meaningful sequence of the sentences thus formed
to mold them into a coherent paragraph is ACDBF. Hence, option
(d) is the most suitable answer choice.

22. (e); If the sentence (D) is the first sentence of the paragraph, the
sentences in the sequence of DABCE form a coherent paragraph
which is about terrorism and the intelligence gap in anticipating
such attacks. However, the sentence (A) finds no alternative to be
matched with as it talks about the organization of IS and its terror
networks which has no relevance from any other sentences among
the given options. Hence (e) is the correct option as the sentence
(F) does not belong to the paragraph so formed.

24 Adda247 Publications For any detail, mail us at


Publications@adda247.com
Cracker Book for Bank (IBPS | SBI | RRB PO | Clerk) Mains Exams

23. (b); If (C) is the first sentence, the correct sequence of other sentences
after rearrangement should be BEDFA. The mention of “crypto-
ransomware attacks” in the first sentence indicates that the
following sentence should be (B) [The attacks…]. Sentences (B) and
(E) make a clear connection. Similarly sentences (D) and (F) make
another combination. Thus, sentences in the sequence of CBEDFA
form a coherent paragraph which is about the cyber security
against certain cyber threats these days. Hence option (b) is the
correct choice.

24. (c); If (F) is the fourth sentence of the paragraph, the sentences in the
sequence of ECAFGBD form a coherent paragraph which is about
the experimental science and the role of collaboration in this field.
There is a clear evidence that sentence (E) would be the starting
sentence of the paragraph and the sentence (C) should follow (E) to
form a logical chain. As the sentence (F) is the fourth sentence, it
can be viewed that the sentence in the option (G) makes a
connection with it. The subject “It” in this case refers to the process
of “collaboration” as mentioned in the sentence (F). Thus, it should
be the follow-up or fifth sentence of the paragraph. Hence (c) is the
correct choice.
25. (b); Among the given statements, the sentence (B) specifies certain
factors that influence attendance in the education sector, which
indicates that it should be the first sentence of the paragraph. The
sentence (A) follows the sentence (B) perfectly, furthering the
derivation of the factors. The other three sentences (D), (C) and (E)
form a chain following one after the another. Thus, the sentences in
the sequence of BADCE form a coherent paragraph which is about
the factors that lead a student to the classroom. Hence option (b) is
the correct choice.
The correct sequence to form a meaningful paragraph is FADECB.
26. (e); 27. (c); 28. (d);
29. (e); 30. (b);

25 Adda247 Publications For any detail, mail us at


Publications@adda247.com
Cracker Book for Bank (IBPS | SBI | RRB PO | Clerk) Mains Exams

1 Adda247 Publications For any detail, mail us at


Publications@adda247.com
Cracker Book for Bank (IBPS | SBI | RRB PO | Clerk) Mains Exams

Chapter

16 Sentence Fillers
STUDY TIPS

For solving fillers an aspirant must start building his/her vocabulary.


Also, a sentence will give an idea or hint about the word that must be
filled. Sometimes grammar helps in eliminating few options. Therefore,
look for the tense, singular and plural nouns etc. In fillers also, the word
before the blank helps in finding the correct word. Therefore,
understanding of the usage of the word in different aspects is very
important.

Practice Exercise Based on new Pattern

Direction (1-10): In question given below there are two statements, each
statement consists of two blanks. You have to choose the option which
provides the correct set of words that fits both the blanks in both the
statements appropriately and in the same order making them meaningful
and grammatically correct.

1. (1) Confidence was increasing that men, through __________ and


effective action, could __________ their existence and even prolong their
lives.
(2) Others, perhaps with greater __________, wanted the struggle to
continue in order to __________ the conditions of the peasants in the
countryside.
(a) prodigal, fortify
(b) discrimination, vitiate
(c) improvidence, remediate
(d) foresight, ameliorate
(e) shortsightedness, blemish

2 Adda247 Publications For any detail, mail us at


Publications@adda247.com
Cracker Book for Bank (IBPS | SBI | RRB PO | Clerk) Mains Exams

2. (1) A shopkeeper with red pimples on his cheeks near the nose, and a
calm, __________, calculating expression on his plump face, hurriedly
and __________ approached the officer, swinging his arms.
(II) The party has been __________ with its representatives and today
some of them have become __________ rich and some even preach a
gospel of wealth.
(a) relentless, modestly
(b) persistent, ostentatiously
(c) faltering, calmly
(d) vacillating, plainly
(e) hesitant, moderately

3. (I) The legal system currently punishes the


most __________ forms of child abuse and neglect, but such crimes are
__________ to prove.
(II) Demonetization was the most __________ step
taken by Modi government and it made the life of common people
__________ for next few months.
(a) modest, brutal
(b) concealed, facile
(c) unobtrusive, comfortable
(d) inconspicuous, superficial
(e) egregious, arduous

4. (I) His plans, probably not very definite, were disturbed by


an __________ message from the queen, ordering him not to return to
England without her __________.
(II) The events which were taking place in Europe made it __________ to
send home a part of the army of Africa, and Medea gave his __________
for the same.
(a) dispensable, revocation
(b) unimportant, injunction
(c) imperative, consent
(d) discretionary, deterrence
(e) inconsequential, embargo

3 Adda247 Publications For any detail, mail us at


Publications@adda247.com
Cracker Book for Bank (IBPS | SBI | RRB PO | Clerk) Mains Exams

5. (I) The __________ modern being is a bit like Howard Roark, who in Ayn
Rand’s The Fountainhead sits atop a metaphorical mountain __________
to view life from a distance.
(II) For the subordinates, the __________ managerial role was to create a
supportive environment within which they could prosper
professionally without being __________ by anyone.
(a) quintessential, condemned
(b) substandard, endorsed
(c) atrocious, eulogized
(d) ordinary, consecrated
(e) mediocre, venerated

6. [I] The question that had___________________ Pierre on the Mozhaysk hill


and all that day now seemed to him quite clear and completely
___________________.
[II] For the ancients these ___________________ questions were
___________________ by a belief in the direct participation of the Deity in
human affairs.
(a) flustered, vague
(b) answered, tangled
(c) baffled, unsettled
(d) perturbed, solved
(e) procrastinated, mystery

7. [I] Land left ___________________was occupied by colonial farmers, and


over 2000 German ___________________ were introduced by Sir George
and settled along the frontier (1858-1859).
[II] The building was ___________________ for decades after that,
___________________who didn’t pass the required intelligence tests were
moved to these rooms until their eventual deportations.
(a) engaged, refugees
(b) populated, fighters
(c) reliable, revolutionaries
(d) occupied, soldiers
(e) derelict, immigrants

4 Adda247 Publications For any detail, mail us at


Publications@adda247.com
Cracker Book for Bank (IBPS | SBI | RRB PO | Clerk) Mains Exams

8. [I] Some of the employees, in their ___________________to win the


business of wealthy Americans with a ___________________ for fibbing to
the taxman, seem to have confused discretion with spycraft.
[II] There will always be some employees with the ___________________ to
pursue profits with minimal regard for larger social consequences;
however, this ___________________ need not be fashionable.
(a) zeal, predilection
(b) enthusiasm, animosity
(c) competition, antipathy
(d) sloth, propensity
(e) lethargy, druthers

9. [I] Mr. Bonar Law was whole-heartedly in favour of the


___________________, and frequently ___________________ his Conservative
friends to remain true to it
[II] As a single person, I find it very difficult to fight the
___________________ of my neighbors who are against the installation of a
neighborhood charter, so I solemnly ___________________the higher
authorities to stand in my support.
(a) alliance, impoverished
(b) union, trespassed
(c) combination, appealed
(d) affiliation, declined
(e) coalition, adjured

10. [I] After a poor display in a tour match, in which Pathan was hit for 70
runs in 12 overs and appeared ___________________, he was dropped from
the Test team as V. R. V. Singh became the third pace bowler,
___________________ a news channel.
[II] Most citizens of Juarez are somewhat ___________________with the
routinely ___________________ accounts of killings by rival drug lords or
dueling gang members.
(a) bored, created
(b) jaded, reported
(c) energized, declared
(d) fatigued, translated
(e) excited, recorded
5 Adda247 Publications For any detail, mail us at
Publications@adda247.com
Cracker Book for Bank (IBPS | SBI | RRB PO | Clerk) Mains Exams

Directions (11-15): In each of the questions given below, a paragraph is


given which has some blanks and those blanks must be filed with the same
word out of five words given below it. You must choose that same as your
answer and fill up the blanks with that appropriate answer.

11. After a muted Onam, Kerala woke up on Sunday to the reality of


providing __________ to the humungous number of stranded people and
to the enormity of rehabilitation and restoration of normal life.
According to Chief Minister Pinarayi Vijayan, who held a review of the
__________ and rehabilitation work on Sunday, 462456 persons are still
housed in 1435 __________ camps.
(a) Nonchalance (b) Hindrance (c) Succor
(d) Deluge (e) Torrent

12. Four fresh recruits planning to enter Pakistan-occupied Kashmir


(PoK) for arms training were __________ in the frontier district of
Kupwara on Sunday. The four were __________ after a brief gunfight in
the Kupwara district. Extreme restraint was exercised, and
opportunity was given to the militants to surrender. The four
surrendered and subsequently ___________.
(a) zapped (b) killed (c) spawned
(d) arrested (e) eliminated

13. Former PM Manmohan Singh has written to Prime Minister Narendra


Modi saying that the historic Teen Murti complex, ___________ to the
memory of Jawaharlal Nehru, should be left undisturbed.
It comes in the wake of reports that the Modi government wants to
create a museum ___________ to all Prime Ministers in the Teen Murti
complex. Even his political rivals ___________ their reverence to his
distinctiveness and greatness, said Dr. Singh.
(a) posited (b) dedicated (c) belonged
(d) imprinted (e) altered

6 Adda247 Publications For any detail, mail us at


Publications@adda247.com
Cracker Book for Bank (IBPS | SBI | RRB PO | Clerk) Mains Exams

14. Chief Justice of India (CJI) Justice Dipak Misra on Sunday called upon
law students to engage in the practice of ‘cause lawyering’ and
develop the idea of serviceability to law by taking up pro bono cases
to protect human ______________, ____________ of individuals and
_______________ of under-privileged.
(a) succor (b) travesty (c) equanimity
(d) deluge (e) rights

15. Mr. Bhagat, a loco pilot of the Bamanhat-Siliguri Jn. passenger train
applied the brakes after spotting a __________ of elephants close to the
track between Sivok and Gumla stations in northern West Bengal on
August 24 evening. My assistant [Mr. Kumar] spotted the __________ at
about 5.35 p.m. The __________ was not visible from my side of the
locomotive because of a bend in the track.
(a) herd (b) pronouncement (c) army
(d) fleet (e) swarm

Directions (16-20): In the following question, there are three statements


with a missing word in each case. There are four words given below each
question which can be used to fill up the blanks with the same word in each
sentence to make it meaningful. Choose the correct alternative among the
five options given below each question which satisfies the meaning of the
sentences with the proper usage of word(s) in each case. If all four words
are feasible, choose option (e) i.e., “All (I), (II), (III) and (IV)” as your
answer.
16. (1) Never mind that the Turkish supremo did his best to ___________ the
Kurdish-led defence of Kobani against an IS assault.
(2) To do what the superior power requires of him, he must ____________
his inclinations.
(3) Turkish security forces have been involved in a long-running
campaign to ____________ Daesh attacks.
(I) impede (II) thwart
(III) inhibit (IV) facilitate
(a) Only (I)
(b) Both (I) and (II)
(c) Both (II) and (IV)
(d) Only (I), (II) and (III)
(e) All (I), (II), (III) and (IV)
7 Adda247 Publications For any detail, mail us at
Publications@adda247.com
Cracker Book for Bank (IBPS | SBI | RRB PO | Clerk) Mains Exams

17. (1) Mathletics is described as an engaging, supportive online learning


resource ____________ at primary school level maths curriculum,
allowing children to play live mental arithmetic games against
other children from all over the world.
(2) Military force would be applied in a purer form and _____________
primarily against the military capabilities of an opponent.
(3) These definitions were specifically ____________ at Indigenous
Australians, and were prevalent on the majority of the word's
entries up until recently.
(I) targeted (II) aimed
(III) directed (IV) intended
(a) Only (I)
(b) Only (II)
(c) Both (III) and (IV)
(d) Only (I), (III) and (IV)
(e) All (I), (II), (III) and (IV)

18. (1) The Museum of Geographical Heritage is among recent initiatives


that promise to expand the scope of the museum beyond a
_______________.
(2) As may be imagined he was a ________________ of much valuable and
varied information.
(3) Brush with Fame is a chance for readers of The Canton
_______________ to share photos of their chance encounters with
celebrities and the famous folks who walk among us.
(I) shade (II) repository
(III) wealth (IV) dungeon
(a) Only (II)
(b) Both (I) and (III)
(c) Both (II) and (III)
(d) Both (I) and (IV)
(e) All (I), (II), (III) and (IV)
8 Adda247 Publications For any detail, mail us at
Publications@adda247.com
Cracker Book for Bank (IBPS | SBI | RRB PO | Clerk) Mains Exams

19. (1) Contrary to the popular belief that people tend to be more satisfied
with an increase in their wage, a new study suggests that it may be
temporary and not have a ______________ effect on job satisfaction.
(2) Hermione knew something of the _______________ infidelities of
Neapolitan men.
(3) Inside the house she heard the ________________, energetic sound of
hammering.
(I) persistent (II) lingering
(III) intermittent (IV) yielding
(a) Only (I)
(b) Both (I) and (II)
(c) Only (I), (II) and (IV)
(d) Only (II), (III) and (IV)
(e) All (I), (II), (III) and (IV)

20. (1) It has been a victory for democracy in Hong Kong, and the signal
emitted by the island nation’s highest court must be particularly
_______________ for China.
(2) At that instant a dull but _________________ thump was heard on the
roof overhead.
(3) Asserting that the inaugural edition of the Khelo India School
Games has been a _________________ success, Sports minister
Rajyavardhan Singh Rathore hoped on Tuesday that it will help to
improve India's performance in the international arena.
(I) emphatic (II) phenomenal
(III) resounding (IV) massive
(a) Only (IV)
(b) Both (II) and (III)
(c) Only (I), (II) and (III)
(d) Only (II), (III) and (IV)
(e) All (I), (II), (III) and (IV)

9 Adda247 Publications For any detail, mail us at


Publications@adda247.com
Cracker Book for Bank (IBPS | SBI | RRB PO | Clerk) Mains Exams

Direction (21-25): In question given below there are two statements, each
statement consists of two blanks. You have to choose the option which
provides the correct set of words that fits both the blanks in both the
statements appropriately and in the same order making them meaningful
and grammatically correct.

21. (1) The alliance between the PDP and the BJP in Jammu and Kashmir
began as an act of necessity, persisted due to sunk-costs and
political ____________, and has finally ended as a result of political
___________.
(2) Burke carried into the world of theory those politics of ____________
as well as ____________ of which Walpole had been the practical
originator.
(a) infeasibility, disadvantage
(b) impracticality, appropriateness
(c) expediency, opportunism
(d) unwisdom, timelessness
(e) dilemma, losses

22. (1) One can understand the banks’ ____________ to switch to the lower
MCLR-based rates, given the multiple pressures they face, including
record levels of NPAs, and ____________ treasury losses.
(2) But as time progressed, his obvious ____________ to arranged
marriage presented a ____________ problem in front of his parents.
(a) willingness, astonishing
(b) assurance, ordinary
(c) certainty, prosaic
(d) suspicion, forgettable
(e) reluctance, noteworthy
10 Adda247 Publications For any detail, mail us at
Publications@adda247.com
Cracker Book for Bank (IBPS | SBI | RRB PO | Clerk) Mains Exams

23. (1) Traditionally, the services of outside experts were ____________


through consultative processes, a practice quite ____________ with the
erstwhile Planning Commission and to some extent with its new
avatar, the NITI Aayog.
(2) Alexander ____________ himself of the defeat of the French to break
the power of the Orsini, following the general tendency of all the
princes of the day to crush the ____________ feudatories and establish
a centralized despotism.
(a) satisfied, restricted
(b) availed, widespread
(c) missed, compelled
(d) hindered, restrained
(e) bestowed, circumscribed

24. (1) Democrats and Republicans alike have expressed deep concern
about the ethics of using children, facing ____________ from
separation from their parents, to ____________ further undocumented
border crossings.
(2) Not every employee is prepared for such ____________, even though
studies show that low price stocks ____________ investments, in the
long run.
(a) equanimity, animate
(b) complacence, distress
(c) contentment, galvanize
(d) trauma, dissuade
(e) alleviation, invigorate

25. (1) Farmer’s collectives such as Farmer Producer Organizations need


to be ____________ and these would be critical to ____________ the ZBNF
(Zero Budget Natural Farming) programme.
(2) Only a well ____________ military dictatorship would be capable
of ____________ an ordered society in the aftermath of a severe
conflict.
(a) confuted, bypassing
(b) embedded, nurturing
(c) established, sustaining
(d) furnished, repudiating
(e) authenticated, resisting
11 Adda247 Publications For any detail, mail us at
Publications@adda247.com
Cracker Book for Bank (IBPS | SBI | RRB PO | Clerk) Mains Exams

Directions (26-30): In each of the following questions, a sentence is given


with a blank. There are five phrases given below each sentence, one out of
which can be used in the blank to form a meaningful sentence. Choose the
most appropriate phrase among the five options that makes the sentence
contextually meaningful.

26. More children growing up in a world so increasingly diverse that


stock racial identities no longer ________________________.
(a) call off (b) hold up (c) leave out
(d) blow up (e) bring up

27. A pleasant spoken young constable arrived at Riverview with a


warrant and asked if he might ________________________ the boat.
(a) turn up (b) take down (c) make out
(d) make up (e) look over

28. We came into the theatre at about two for a ________________________,


Matt came in as well and he was struggling with his voice.
(a) put out (b) put up (c) run away
(d) run through (e) run into

29. Jet needs to urgently ________________________its cost strategy as it does


not have enough funds to continue operations for more than two
months.
(a) work out (b) warm up (c) wake up
(d) turn down (e) take out

30. Cameron Borthwick-Jackson hopes to make up for lost time and


________________________his best as the Manchester United left-back
begins his season-long loan at Scunthorpe.
(a) get up (b) get back at (c) get back to
(d) get along with (e) get around

12 Adda247 Publications For any detail, mail us at


Publications@adda247.com
Cracker Book for Bank (IBPS | SBI | RRB PO | Clerk) Mains Exams

Solutions

1. (d) ‘foresight, ameliorate’ is the pair of words that fits in the two
sentences to make both the sentences grammatically and
contextually complete. Hence, option (d) is the most appropriate
choice.
Foresight means the ability to predict what will happen or be
needed in the future.
Ameliorate means make (something bad or unsatisfactory) better.
Prodigal means spending money or using resources freely and
recklessly; wastefully extravagant
Vitiate means spoil or impair the quality or efficiency of.
Improvidence means the quality or state of not foreseeing and
providing for the future
Remediate means to settle (disputes, strikes, etc.) as an
intermediary between parties; reconcile.
Blemish means a small mark or flaw which spoils the appearance
of something.

2. (b); ‘persistent, ostentatiously’ is the pair of words that fits in the two
sentences to make both the sentences grammatically and
contextually complete. Hence, option (b) is the most appropriate
choice.
Persistent means continuing firmly or obstinately in an opinion or
course of action in spite of difficulty or opposition.
Ostentatiously means in a pretentious or showy way designed to
impress.
Relentless means unceasingly intense.
Modestly means in an unassuming manner; without vanity or
arrogance.
Faltering means losing strength or momentum.
Vacillating means wavering between different opinions or actions;
irresolute.

13 Adda247 Publications For any detail, mail us at


Publications@adda247.com
Cracker Book for Bank (IBPS | SBI | RRB PO | Clerk) Mains Exams

3. (e); ‘egregious, arduous’ is the pair of words that fits in the two
sentences to make both the sentences grammatically and
contextually complete. Hence, option (e) is the most appropriate
choice.
Egregious means shocking.
Arduous means difficult and tiring.
Brutal means savagely violent.
Facile means ignoring the true complexities of an issue; superficial
Unobtrusive means not conspicuous or attracting attention.
Inconspicuous means not clearly visible or attracting attention.
Superficial means appearing to be true or real only until examined
more closely.

4. (c); ‘imperative, consent’ is the pair of words that fits in the two
sentences to make both the sentences grammatically and
contextually complete. Hence, option (c) is the most appropriate
choice.
Imperative means of vital importance; crucial
Consent means permission for something to happen or agreement
to do something.
Dispensable means able to be replaced or done without;
superfluous.
Revocation means the official cancellation of a decree, decision, or
promise.
Injunction means an authoritative warning or order.
Discretionary means available for use at the discretion of the user.
Deterrence means the action of discouraging an action or event
through instilling doubt or fear of the consequences.
Inconsequential means not important or significant.
Embargo means an official ban on any activity.

14 Adda247 Publications For any detail, mail us at


Publications@adda247.com
Cracker Book for Bank (IBPS | SBI | RRB PO | Clerk) Mains Exams

5. (a); ‘quintessential, condemned’ is the pair of words that fits in the two
sentences to make both the sentences grammatically and
contextually complete. Hence, option (a) is the most appropriate
choice.
Quintessential means representing the most perfect or typical
example of a quality or class.
Condemned means express complete disapproval of
Substandard means below the usual or required standard.
Endorsed means declare one's public approval or support of.
Atrocious means horrifyingly wicked.
Eulogized means praise highly in speech or writing.
Consecrated means make or declare (something, typically a
church) sacred
Venerated means regard with great respect.

6. (d); The most appropriate set of words that appropriately fit in the
context of the paragraph is ‘perturbed, solved’. ‘perturbed’ is an
adjective which means anxious or unsettled; upset. Moreover,
‘solved’ is a verb which means to find an answer to, explanation
for, or means of effectively dealing with (a problem or mystery).
Since, all the other sets of words fail to form a comprehensive
sentence, option (d) becomes the most suitable answer choice.
Flustered means make (someone) agitated or confused.
Tangled means twisted together untidily; matted.
Baffled means totally bewilder or perplex.
Procrastinated means delay or postpone action; put off doing
something.

7. (e); The most appropriate set of words that appropriately fit in the
context of the paragraph is ‘derelict, immigrants’. ‘Derelict’ is an
adjective which means in a very poor condition as a result of
disuse and neglect. Moreover, ‘immigrants’ is a noun which means
a person who comes to live permanently in a foreign country. Since,
all the other sets of words fail to form a comprehensive sentence,
option (e) becomes the most suitable answer choice.

15 Adda247 Publications For any detail, mail us at


Publications@adda247.com
Cracker Book for Bank (IBPS | SBI | RRB PO | Clerk) Mains Exams

8. (a); The most appropriate set of words that appropriately fit in the
context of the paragraph is ‘zeal, predilection’. ‘Zeal’ is a noun
which means great energy or enthusiasm in pursuit of a cause or an
objective. Moreover, ‘predilection’ is a noun which means a
preference or special liking for something; a bias in favour of
something. Since, all the other sets of words fail to form a
comprehensive sentence, option (a) becomes the most suitable
answer choice.
Animosity means strong hostility.
Antipathy means a deep-seated feeling of aversion.
Sloth means reluctance to work or make an effort; laziness.
Propensity means an inclination or natural tendency to behave in
a particular way.

9. (e); The most appropriate set of words that appropriately fit in the
context of the paragraph is ‘coalition, adjured’. ‘Coalition’ is a
noun which means a temporary alliance for combined action,
especially of political parties forming a government. Moreover,
‘adjured’ is a verb which means urge or request (someone)
solemnly or earnestly to do something. Since, all the other sets of
words fail to form a comprehensive sentence, option (e) becomes
the most suitable answer choice.

10. (b); The most appropriate set of words that appropriately fit in the
context of the paragraph is ‘jaded, reported’. ‘Jaded’ is an adjective
which means bored or lacking enthusiasm, typically after having
had too much of something. Moreover, ‘reported’ is a verb which
means give a spoken or written account of something that one has
observed, heard, done, or investigated. Since, all the other sets of
words fail to form a comprehensive sentence, option (b) becomes
the most suitable answer choice.
Fatigued means cause (someone) to feel exhausted.
Energized means give vitality and enthusiasm to.
16 Adda247 Publications For any detail, mail us at
Publications@adda247.com
Cracker Book for Bank (IBPS | SBI | RRB PO | Clerk) Mains Exams

11. (c); Stranded people requires ‘Relief’. People should be housed in


‘Relief’ camps. CM would have held a review of the ‘Relief’ and
rehabilitation work.
Nonchalance [noun] means ‘The state of being nonchalant; calm
behavior that suggests you are not interested or do not care;
calmness; equanimity; coolness;’.
Succor [noun] means ‘assistance and support in times of hardships
and distress’.
Deluge [noun] means ‘a severe flood’;
Torrent [noun] means ‘a severe flood’;
Among the given options, ‘succor’ has a meaning very close to the
meaning of ‘relief’.
Hence, ‘Succor’ is the correct answer and option (c) is the correct
answer.

12. (d); The last two sentences give the hint for the word which would fill
the blank. ‘Extreme restraint was exercised, and opportunity was
given to the militants to surrender. The four surrendered’.
The highlighted sentences suggest that the militants weren’t killed
or eliminated because extreme restraint was exercised, and the
militants surrendered.
Zap [verb] means ‘go quickly’;
Spawn [verb] ‘(of a fish, frog, mollusk, crustacean, etc) release or
deposit eggs; produce (offspring)’;
The words ‘zapped’ and ‘spawned’ are irrelevant to the context of
the sentences.
It makes more sense that the surrendered militants were arrested.
Hence, option (d) is the correct answer.

13. (b); Posit [verb] means ‘to put forward as a fact or as a basis for
argument’. The meaning of the word is irrelevant in the context of
the given blanks.
Similarly, the meaning of the words ‘belonged’, ’imprinted’ and
‘altered’ is irrelevant in the context of the given blanks.
The Teen Murti Complex should be ‘attributed’ to Nehru Ji.
The meaning of the word ‘dedicated’ is closer to the meaning of
the word ‘attributed’.
Hence, option (b) is the correct answer.
17 Adda247 Publications For any detail, mail us at
Publications@adda247.com
Cracker Book for Bank (IBPS | SBI | RRB PO | Clerk) Mains Exams

14. (e); Succor [noun] means ‘assistance and support in times of hardships
and distress’;
Travesty [noun] means ‘something that fails to represent the values
and qualities that it is intended to represent, in a way that is
shocking or offensive’;
Equanimity [noun] means ‘calmness and composure, especially in a
difficult situation’;
Deluge [noun] means ‘a severe flood’;
The words succor, travesty, equanimity and deluge are irrelevant
in the context of the blanks.
There is something called ‘rights of humans, rights of individuals,
and rights of under-privileged;
The word ‘rights’ correctly fits the blank.
Hence, option (e) is the correct answer.

15. (a); Only the option (a) ‘herd’, upon filling the blanks, satisfy the
contextual and grammatical requirements of the blanks.
Hence, Option (a) is the correct answer.
herd [noun] means ‘a large group of animals, especially hoofed
mammals, that live together or are kept together as livestock’;
Pronouncement [noun] means ‘a formal or authoritative
announcement or declaration’;
Army [noun] means ‘a group of soldiers’;
Fleet [noun] means ‘a group of ships’;
Swarm [noun] means ‘a large group of flying insects’;

16. (d); All three words (I), (II) and (III) are befitting to the given blanks as
they give the similar meaning in all three cases to make the
sentences meaningful. All three words “impede”, “thwart” and
“inhibit” describe the same meaning “prevent”. Hence option (d)
is the correct choice.
Impede means delay or prevent (someone or something) by
obstructing them; hinder.
Thwart means prevent (someone) from accomplishing something.
Inhibit means hinder, restrain, or prevent (an action or process).
Facilitate means make (an action or process) easy or easier.
18 Adda247 Publications For any detail, mail us at
Publications@adda247.com
Cracker Book for Bank (IBPS | SBI | RRB PO | Clerk) Mains Exams

17. (e); All the given words can be used in the provided blanks as they all
give the similar meaning to the three sentences making them
meaningful. Hence option (e) is the correct choice.
Targeted means aimed or directed (something).
Intended means designed or destined something for a particular
purpose.

18. (a); Among the given choices, only the second word gives a contextual
meaning to all the three sentences. Other words are not suitable in
the context of adding meaning to the sentence. Hence option (a) is
the correct choice.
Repository means a person or thing regarded as a store of
information or in which a particular quality may be found.
Shade means a position of relative inferiority or obscurity.
Dungeon means a strong underground prison cell, especially in a
castle.

19. (b); Both the first and the second words can be used in the provided
blanks to make them meaningful. Both the words give the similar
meaning to the sentences. Hence option (b) is the correct choice.
Persistent means continuing to exist or occur over a prolonged
period.
Lingering means lasting for a long time or slow to end.
Intermittent means occurring at irregular intervals; not
continuous or steady.
Yielding means (of a person) complying with the requests or
desires of others.

20. (e); All the four words are correct enough to fit into the provided
blanks and thus adding the similar meaning in all the cases. All the
four words mean the same. Hence option (e) is the correct choice.
Emphatic means expressing something forcibly and clearly.
Phenomenal means remarkable or exceptional, especially
exceptionally good.
Resounding means unmistakable; emphatic.
Massive means large and heavy or solid.
19 Adda247 Publications For any detail, mail us at
Publications@adda247.com
Cracker Book for Bank (IBPS | SBI | RRB PO | Clerk) Mains Exams

21. (c); ‘expediency, opportunism’ is the pair of words that fits in the two
sentences to make both the sentences grammatically and
contextually complete.
Expediency means the quality of being convenient and practical
despite possibly being improper or immoral; convenience.
Opportunism means the taking of opportunities as and when they
arise, regardless of planning or principle.
Here, the first sentence talks about persistence of alliance between
PDP and BJP, such alliances persists because of political
expediency. Hence, option (c) is the most appropriate choice.
Infeasibility means not feasible; impracticable.
Impracticality means not practical; not wise to put into or keep in
practice or effect.
Unwisdom means lack of wisdom.
Dilemma means a situation in which a difficult choice has to be
made between two or more alternatives, especially ones that are
equally undesirable.

22. (e); ‘reluctance, noteworthy’ is the pair of words that fits in the two
sentences to make both the sentences grammatically and
contextually complete.
Reluctance means unwillingness or disinclination to do something.
Noteworthy means worth paying attention to; significant.
Hence, option (e) is the most appropriate choice.
Willingness means the quality or state of being prepared to do
something; readiness.
Astonishing means extremely surprising or impressive; amazing.
Prosaic means having or using the style or diction of prose as
opposed to poetry; lacking imaginativeness or originality.
Suspicion means a feeling or thought that something is possible,
likely, or true.
20 Adda247 Publications For any detail, mail us at
Publications@adda247.com
Cracker Book for Bank (IBPS | SBI | RRB PO | Clerk) Mains Exams

23. (b); ‘availed, widespread’ is the pair of words that fits in the two
sentences to make both the sentences grammatically and
contextually complete.
Availed means use or take advantage of (an opportunity or
available resource); help or benefit
Widespread means found or distributed over a large area or
number of people.
In the first sentence ‘availed’ means taken or obtained and in the
second sentence it means benefited. Hence, option (b) is the most
appropriate choice.
Compelled means forced or obliged (someone) to do something.
Hindered means make it difficult for (someone) to do something
or for (something) to happen.
Restrained means kept under control.
Bestowed means confer or present (an honour, right, or gift).
Circumscribed means restrict (something) within limits.

24. (d); ‘trauma, discourage’ is the pair of words that fits in the two
sentences to make both the sentences grammatically and
contextually complete.
Trauma means a deeply distressing or disturbing experience.
Dissuade means persuade (someone) not to take a particular
course of action.
Hence, option (d) is the most appropriate choice.
Equanimity means calmness and composure, especially in a
difficult situation.
Animate means give inspiration, encouragement, or renewed
vigour to.
Complacence means a feeling of smug or uncritical satisfaction
with oneself or one's achievements.
Distress means extreme anxiety, sorrow, or pain.
Contentment means a state of happiness and satisfaction.
Galvanize means shock or excite (someone) into taking action.
Alleviation means the action or process of making suffering,
deficiency, or a problem less severe.
Invigorate means give strength or energy to.
21 Adda247 Publications For any detail, mail us at
Publications@adda247.com
Cracker Book for Bank (IBPS | SBI | RRB PO | Clerk) Mains Exams

25. (c); ‘established, sustaining’ is the pair of words that fits in the two
sentences to make both the sentences grammatically and
contextually complete.
Established means having existed or done something for a long
time and therefore recognized and generally accepted.
Sustaining means strengthening or supporting physically or
mentally
Hence, option (c) is the most appropriate choice.
Confuted means prove (a person or an assertion or accusation) to
be wrong.
Embedded means fixed (an object) firmly and deeply in a
surrounding mass
Repudiating means refusing to accept; rejecting.
Authenticated means proved or showed (something) to be true,
genuine, or valid

26. (b); The most suitable phrase to fill the blank is ‘hold up’ which means
to remain strong or vigorous. All the other phrases fail to provide
the appropriate context to the sentence. Hence, option (b) becomes
the most viable answer choice.
Call off means to decide that a planned event will not happen
Leave out means to not include or mention (someone or
something)
Blow up means explode
Bring up means (chiefly of a ship) come to a stop.

27. (e); The most suitable phrase to fill the blank is ‘look over’ which
means a quick inspection of something; a survey. All the other
phrases fail to provide the appropriate context to the sentence.
Hence, option (e) becomes the most viable answer choice.
Turn up means be found, especially by chance, after being lost.
Take down means a police raid or arrest.
Make out means make progress; fare.
Make up means invent a story, lie, or plan.
22 Adda247 Publications For any detail, mail us at
Publications@adda247.com
Cracker Book for Bank (IBPS | SBI | RRB PO | Clerk) Mains Exams

28. (d); The most suitable phrase to fill the blank is ‘run through’ which
means to practise something so that it is correct for a performance
or test. All the other phrases fail to provide the appropriate context
to the sentence. Hence, option (d) becomes the most viable answer
choice.
put out means cause someone trouble or inconvenience.
put up means stay temporarily in accommodation other than one's
own home.
Run-away means escape from a place, person, or situation.
run into means collide with.

29. (a); The most suitable phrase to fill the blank is ‘work out’ which means
to find the solution by thinking or talking about it. All the other
phrases fail to provide the appropriate context to the sentence.
Hence, option (a) becomes the most viable answer choice.
warm up means prepare for physical exertion or a performance by
exercising or practising gently beforehand.
wake up means an instance of a person waking up or being woken
up
turn down means a rejection or refusal.
take out means a bid (in a different suit) made in response to a bid
or double by one's partner.

30. (c); The most suitable phrase to fill the blank is ‘get back to’ which
means to return to someone or something. This phrase is often
used to say that you will return with an answer to a question or a
request at a later time. All the other phrases fail to provide the
appropriate context to the sentence. Hence, option (c) becomes the
most viable answer choice.
get up means (of wind or the sea) become strong or agitated.
get back at means take revenge on (someone).
get along with means another way of saying get on.
get around means deal successfully with
(a problem).

23 Adda247 Publications For any detail, mail us at


Publications@adda247.com
Cracker Book for Bank (IBPS | SBI | RRB PO | Clerk) Mains Exams

1 Adda247 Publications For any detail, mail us at


Publications@adda247.com
Cracker Book for Bank (IBPS | SBI | RRB PO | Clerk) Mains Exams

Chapter

17 Vocabulary

Practice Exercise Based on new Pattern

Directions (1-4): In each of the following questions five options are given,
of which one word is most nearly the same or opposite in meaning to the
given word in the question. Find the correct option having either same or
opposite meaning.

1. REPERCUSSION
(a) Reliant (b) Consequence (c) Impertinence
(d) Prompting (e) Callous

2. MASQUERADE
(a) Contentious (b) Pretense (c) Belligerent
(d) Arrogant (e) Reminiscent

3. Wither
(a) Ingenious (b) Gnawed (c) Persistent
(d) Thrive (e) Reverie

4. REPROBATE
(a) Repudiate (b) Strident (c) Virtuous
(d) Flagrant (e) Stoic

Directions (5 – 9): In the following questions, four words are given, choose
the most suitable alternative reflecting the combination of the words which
are either synonyms or antonyms of each other. If no such pair is formed,
mark option (e) i.e., “none of these” as your answer choice.

5. (i) conclusion (ii) paradox


(iii) discrepancy (iv) pragmatic
(a) only (i) – (iv) (b) only (i) – (iii) (c) only (ii) – (iii)
(d) only (i) – (ii) (e) none of these
2 Adda247 Publications For any detail, mail us at
Publications@adda247.com
Cracker Book for Bank (IBPS | SBI | RRB PO | Clerk) Mains Exams

6. (i) declaration (ii) modicum


(iii) laxity (iv) surveillance
(a) only (iii) – (iv) (b) only (i) – (iii) (c) only (ii) – (iii)
(d) only (i) – (ii) (e) none of these

7. (i) historical (ii) tussle


(iii) cumbrous (iv) combat
(a) only (iii) – (iv) (b) only (i) – (iii) (c) only (ii) – (iv)
(d) only (i) – (ii) (e) none of these

8. (i) prudent (ii) crucial


(iii) fluid (iv) reckless
(a) only (ii) – (iii) (b) only (i) – (iii) (c) only (ii) – (iv)
(d) only (i) – (iv) (e) none of these

9. (i) itinerant (ii) nomads


(iii) eradication (iv) penchant
(a) only (i) – (ii) (b) only (ii) – (iii) (c) only (ii) – (iv)
(d) only (i) – (iv) (e) none of these

Directions (10-11): In the following questions, five words are given,


choose the most suitable alternative reflecting the combination of word(s)
similar in meaning with each other.

10. (I) Guise (II) Staple


(III) Wean (IV) Affliction
(V) Torment
(a) I and II (b) II and III (c) III and IV
(d) IV and V (e) I, III and IV

11. (I) Abstention (II) Reverence


(III) Travesty (IV) Veneration
(V) homage
(a) I and II (b) II and III (c) II, IV and V
(d) I, II and V (e) I, III and IV

3 Adda247 Publications For any detail, mail us at


Publications@adda247.com
Cracker Book for Bank (IBPS | SBI | RRB PO | Clerk) Mains Exams

Directions (12-16): Choose the word/group of words which is most


SIMILAR in meaning to the word/group of words printed in bold as used in
passage.

12. Impromptu
(a) extempore (b) enticing (c) rapturous
(d)entralling (e) enamouring

13. Peculiar
(a) intuitive (b) winsome (c) queer
(d) imperious (e) inciting

14. Dire
(a) devour (b) revoke (c) unprecedent
(d) appalling (e) extraneous

15. Tapering off


(a) tenable (b) Abate (c) ameliorate
(d) repudiate (e) inundate

16. Inevitable
(a) testimony (b) allude (c) inexorable
(d) fester (e) eerie

Directions (17-19): Choose the word/group of words which is most


OPPOSITE in meaning to the word GIVEN WORD.

17. Provocative
(a) valiant (b) repressive (c) chivalrous
(d) vigorous (e) ingenious

18. Looming
(a) punitive (b) humongous (c) ambiguous
(d) inimitable (e) receding

19. Detriment
(a) exacerbate (b) farthing (c) privilege
(d) covet (e) nuance
4 Adda247 Publications For any detail, mail us at
Publications@adda247.com
Cracker Book for Bank (IBPS | SBI | RRB PO | Clerk) Mains Exams

Directions (20-22): In the following questions, a word is given in bold,


choose the most suitable alternative reflecting the combination of word (s)
which is similar in meaning of the highlighted word.

20. Aberration
(I) Digression (II) Diversion
(III) Eccentricity (IV) Transgression
(a) Only (II)
(b)Only (I), (II) and (III)
(c) Both (III) and (IV)
(d) Both (I) and (II)
(e) All of these

21. Entrenching
(I) Establishing (II) Ensconcing
(III) Dislodging (IV) Defenestrating
(a) only (I)
(b) both (I) and (II)
(c) both (II) and (III)
(d) Both (III) and (IV)
(e) only (IV)

22. Pared
(I) Sheared (II) Pruned
(III) Whittled (IV) Expatiated
(a) both (II) and (III)
(b) only (I), (III) and (IV)
(c) only (I), (II) and (III)
(d) only (II), (III) and (IV)
(e) both (I) and (IV)
5 Adda247 Publications For any detail, mail us at
Publications@adda247.com
Cracker Book for Bank (IBPS | SBI | RRB PO | Clerk) Mains Exams

Directions (23-25): In the following questions, a word is given in bold,


choose the most suitable alternative reflecting the combination of word (s)
which is opposite in meaning of the highlighted word.

23. Threshold
(I) brink (II) verge
(III) portal (IV) nadir
(a) both (II) and (III)
(b) only (IV)
(c) only (III)
(d) only (I)
(e) both (I) and (IV)

24. Pegged
(I) Noted (II) Overlooked
(III) Discovered (IV) Unearthed
(a) only (I)
(b) both (I) and (III)
(c) both (III) and (IV)
(d) only (II)
(e) all of these

25. Hefty
(I) gaunt (II) covet
(III) humongous (IV) concoct
(a) Only (I)
(b) Only (II) and (III)
(c) Only (I), (II) and (IV)
(d) Only (II), (III) and (IV)
(e) All of the above

6 Adda247 Publications For any detail, mail us at


Publications@adda247.com
Cracker Book for Bank (IBPS | SBI | RRB PO | Clerk) Mains Exams

Solutions

1. (b); Option (b) is the correct choice.


Repercussion means an unintended consequence of an event or
action, especially an unwelcome one hence consequence is the
word which is most similar in meaning to this.
Callous means showing or having an insensitive and cruel
disregard for others.
Prompting means the action of saying something to persuade,
encourage, or remind someone to do or say something.
Impertinence means lack of respect; rudeness.

2. (b); Masquerade means a false show or pretence. Hence masquerade


and pretense are similar in meaning.
Pretense means an attempt to make something that is not the case
appear true.
Belligerent means a nation or person engaged in war or conflict,
as recognized by international law.
Reminiscent means suggesting something by resemblance.

3. (d); Wither means (to cause) to become weak and dry and decay hence
thrive which means (of a child, animal, or plant) grow or develop
well or vigorously is the word most opposite in meaning to the
given word.
Reverie means dreamy
Gnawed means bite at or nibble something persistently.
Persistent means continuing firmly or obstinately in an opinion or
course of action in spite of difficulty or opposition.

4. (c); Reprobate and virtuous are opposite in meaning hence option (c) is
the correct choice for the given question.
Reprobate means an unprincipled person.
Virtuous means having or showing high moral standards.
Stoic means indifferent to pleasure or pain
Strident means harsh
Repudiate means to disown.
Flagrant means glaringly wrong
7 Adda247 Publications For any detail, mail us at
Publications@adda247.com
Cracker Book for Bank (IBPS | SBI | RRB PO | Clerk) Mains Exams

5. (c); The most suitable words that are synonym of each other are
‘paradox’ and ‘discrepancy’. ‘paradox’ means a seemingly absurd
or contradictory statement or proposition which when investigated
may prove to be well founded or true while ‘discrepancy’ means
an illogical or surprising lack of compatibility or similarity between
two or more facts. All the other words are irrelevant to each other.
Hence, option (c) is the most viable answer choice.
Pragmatic means dealing with things sensibly and realistically in a
way that is based on practical rather than theoretical
considerations.

6. (a); The most suitable words that are opposite of each other are
‘surveillance’ and ‘combat’. ‘surveillance’ means close observation,
especially of a suspected spy or criminal while ‘laxity’ means lack
of strictness or care. All the other words are not related to each
other. Hence, option (c) is the most viable answer choice.
Declaration means a formal or explicit statement or
announcement.
Modicum means a small quantity of a particular thing, especially
something desirable or valuable.

7. (c); The most suitable words that are synonym of each other are ‘tussle’
and ‘combat’. ‘tussle’ means a vigorous struggle or scuffle, typically
in order to obtain or achieve something; while, ‘combat’ means
fighting between armed forces. Since, they both are synonyms of
each other, option (c) becomes the most viable answer choice.
Historical means of or concerning history or past events
Cumbrous means slow or complicated and therefore inefficient.

8. (d); The most suitable words that are opposite of each other are
‘prudent’ and ‘reckless’. ‘prudent’ means acting with or showing
care and thought for the future, while; ‘reckless’ means heedless of
danger or the consequences of one's actions; rash or impetuous.
Hence, option (d) is the most viable answer choice.
Crucial means decisive or critical, especially in the success or
failure of something.
Fluid means smoothly elegant or graceful.
8 Adda247 Publications For any detail, mail us at
Publications@adda247.com
Cracker Book for Bank (IBPS | SBI | RRB PO | Clerk) Mains Exams

9. (a); The most suitable words that are synonym of each other are
‘itinerant’ and ‘nomads’. ‘itinerant’ means travelling from place to
place while; ‘nomads’ means a person who does not stay long in
the same place; a wanderer. All the other words are irrelevant to
each other. Hence, option (a) is the most viable answer choice.
Eradication means the complete destruction of something.
penchant means a strong or habitual liking for something or
tendency to do something.

10. (d); Here, Affliction and Torment are similar in meaning to each other.
Affliction means the state of being in pain, which is similar in
meaning to ‘torment’.
Wean means be strongly influenced by (something).
Staple means a main or important element of something.
Guise means an external form, appearance, or manner of
presentation, typically concealing the true nature of something.

11. (c); Here, Reverence, Veneration and Homage are synonym to one
another. Reverence means deep respect for someone or
something, which is similar to veneration and exaltation.
Abstention means restraint in one's consumption; abstinence.
Travesty means a false, absurd, or distorted representation of
something.

12. (a); Impromptu means done without being planned or rehearsed.


Hence it has same meaning as extempore.
Enamouring means be filled with love for.

13. (c); Peculiar means different to what is normal or expected, strange.


Hence it has same meaning as queer.
Winsome means attractive or appealing in a fresh, innocent way.
Imperious means arrogant and domineering.
Intuitive means easy to use and understand.

9 Adda247 Publications For any detail, mail us at


Publications@adda247.com
Cracker Book for Bank (IBPS | SBI | RRB PO | Clerk) Mains Exams

14. (d); Dire means extremely serious or urgent. Hence it has same
meaning as appalling.
Extraneous means irrelevant or unrelated to the subject being
dealt with.
Unprecedent means never done or known before.
Revoke means cancel officially.
Devour means destroy completely.

15. (b); Tapering off means to become gradually smaller or weaker. Hence
it has same meaning as abate.
Ameliorate means become or make greater in size, amount.
Inundate means overwhelm (someone) with things or people to be
dealt with.
Repudiate means refuse to accept, reject, deny.
Tenable means able to be maintained or defended against attack
or objection.

16. (c); Inevitable means certain to happen; unavoidable. Hence it has


same meaning as inexorable.
Eerie means strange and frightening.
Fester means become septic; suppurate.
Allude means suggest or call attention to indirectly; hint at.
Testimony means evidence or proof of something.

17. (b); Provocative means causing anger or another strong reaction,


especially deliberately. Hence it has opposite meaning as
repressive.
Ingenious means clever, original, and inventive.
Chivalrous means courteous and gallant, especially towards
women.
Valiant means possessing or showing courage or determination.

10 Adda247 Publications For any detail, mail us at


Publications@adda247.com
Cracker Book for Bank (IBPS | SBI | RRB PO | Clerk) Mains Exams

18. (e); Looming means appear as a vague form, especially one that is
large or threatening. Hence it has opposite meaning as receding
which means to retreat.
Inimitable means so good or unusual as to be impossible to copy;
unique.
Ambiguous means not clear or decided.
Humongous means huge or enormous.
Punitive means inflicting or intended as punishment.

19. (c); Detriment means the state of being harmed or damaged. Hence it
has opposite meaning as privilege which means advantage.
Covet means desire.
Farthing means least possible amount.
Exacerbate means make worse.
Nuance means variation, degree.
20. (b); Option (b) is the correct choice. Aberration means divergence
which is a synonym of digression and eccentricity.

21. (b); Entrenching means establish (an attitude, habit, or belief) so firmly
that change is very difficult or unlikely. Establishing and
ensconcing both mean the same.
Ensconcing means settling.
Dislodging means knocking or forcing out of position.
Defenestrating means removing or dismissing (someone) from a
position of power or authority.

22. (c); Pared means reduce (something) in size, extent, or quantity in a


number of small successive stages.
Sheared means have something cut off.
Pruned means detached.
Whittled means reduce something in size, amount, or extent by a
gradual series of steps.
Expatiated means amplified
11 Adda247 Publications For any detail, mail us at
Publications@adda247.com
Cracker Book for Bank (IBPS | SBI | RRB PO | Clerk) Mains Exams

23. (b); Threshold means a point of entry or beginning. The word that is in
exact opposite to the given word is nadir. Nadir means the lowest
or most unsuccessful point in a situation.
Brink means edge.
Verge means an extreme limit beyond which something specified
will happen.
Portal means a doorway, gate, or other entrance, especially a large
and imposing one.

24. (d); Pegged means to find out. Noted, discovered and unearthed all
mean the same. Overlooked is the only word which is opposite in
meaning and it means failed to notice. Hence only option (d) is the
answer.

25. (a); Hefty means large and heavy. Hence it has opposite meaning as
gaunt.
Covet means desire.
Humongous means huge or enormous.
Concoct means create or devise (a story or plan).

12 Adda247 Publications For any detail, mail us at


Publications@adda247.com
Cracker Book for Bank (IBPS | SBI | RRB PO | Clerk) Mains Exams

1 Adda247 Publications For any detail, mail us at


Publications@adda247.com
Cracker Book for Bank (IBPS | SBI | RRB PO | Clerk) Mains Exams

Chapter

18 Miscellaneous

Practice Exercise Based on new Pattern

Direction (1-5): There are parts of statements in question given below


which when connected using the correct sentence structure forms a
complete single sentence without altering the meaning of the sentence
given in the question. There are four options given below the question,
choose the sentence that forms the correct formation of single sentence
which is both grammatically correct and contextually meaningful. If none
follows, choose (e) as your answer.

1. The conception of the world and of human life is controlled by natural


law; is alien to Hebrew thought in general; is alien to the prophetic
and liturgical Hebrew literature.
(a) Because the conception of the world and of human life as
controlled by natural law has been alien to Hebrew thought in
general but is alien to the prophetic and liturgical Hebrew
literature.
(b) The conception of the world and of human life as controls by
natural law is alien not only to the prophetic and liturgical
Hebrew literature but also to Hebrew thought in general.
(c) Although the conception of the world and of human life is
controlled by natural law is alien to the prophetic and liturgical
Hebrew literature, so, it is also alien to Hebrew thought in
general.
(d) The conception of the world and of human life as controlled by
natural law is alien not only to the prophetic and liturgical
Hebrew literature but also to Hebrew thought in general.
(e) None of the above is correct.

2 Adda247 Publications For any detail, mail us at


Publications@adda247.com
Cracker Book for Bank (IBPS | SBI | RRB PO | Clerk) Mains Exams

2. She is the mother of Ur, the personified fire of hell; mastered by Hibil
and thrown in chains down to the black water; in anger and pride
made a violent onset on the world of light; imprisoned within seven
iron and seven golden walls.
(a) She is the mother of Ur, the personified fire of hell, who in anger
and pride made a violent onset on the world of light, but was
master by Hibil and thrown in chains down to the black water,
and imprisoned within seven iron and seven golden walls.
(b) She is the mother of Ur, the personified fire of hells, who in anger
and pride make a violent onset on the world of light, but was
mastered by Hibil and thrown in chains down to the black water,
and imprisoned within seven iron and seven golden walls.
(c) She is the mother of Ur, the personified fire of hell, who in anger
and pride made a violent onset on the world of light, but was
mastered by Hibil and thrown in chains down to the black water,
and imprisoned within seven iron and seven golden walls.
(d) She is the mother of Ur, the personified fire of hell, who was
mastered by Hibil and thrown in chains down to the black water
because in anger and pride he made a violent onset on the world
of light and imprisoned within seven iron and seven golden walls.
(e) None of the above is correct.

3. The movement gradually assumed the form of terrorism; the natural


result was that the reactionary tendencies of the government were
strengthened; the movement was aimed at the assassination of
prominent officials; the movement was aimed at the assassination of
even the emperor himself.
(a) The movement gradually assumed the form of terrorism because
it aimed at the assassination of prominent officials and even of the
emperor himself as the natural result was that the reactionary
tendencies of the government were strengthened.
(b) The movement gradually assumed the form of terrorism, and
aimed at the assassination of prominent officials, and even of the
emperor himself, and the natural result was that the reactionary
tendencies of the government were strengthened.
3 Adda247 Publications For any detail, mail us at
Publications@adda247.com
Cracker Book for Bank (IBPS | SBI | RRB PO | Clerk) Mains Exams

(c) The movement gradually assumed the form of terrorism and


aimed at the assassination of prominent officials, and even of the
emperor himself, because the natural result was that the
reactionary tendencies of the government were strengthened.
(d) The movement gradually assumed the form of terrorism, and
aimed at the assassination of prominent officials, and even of the
emperor himself, but the natural result was that the reactionary
tendencies of the government was strengthen.
(e) None of the above is correct.

4. Gagern attempted to influence the Prussian Northern Union in the


direction of the national policy; soon realizing the hopelessness of any
good results from the vacillating policy of Prussia, he retired from the
contest; he took part in the sessions of the Erfurt parliament; as a
major in the service of the Schleswig-Holstein government, took part
in the Danish War of 1850.
(a) Gagern attempted to influence the Prussian Northern Union in the
direction of the national policy, but he took part in the sessions of
the Erfurt parliament, and, soon realizing the hopelessness of any
good results from the vacillating policy of Prussia, he retired from
the contest because as a major in the service of the Schleswig-
Holstein government, took part in the Danish War of 1850.
(b) Gagern attempted to influence the Prussian Northern Union in the
direction of the national policy and soon realizing the
hopelessness of any good results from the vacillating policy of
Prussia, he retired from the contest, so he took part in the
sessions of the Erfurt parliament and as a major in the service of
the Schleswig-Holstein government, took part in the Danish War
of 1850.
(c) Gagern attempted to influence the Prussian Northern Union in the
direction of the national policy, and he took part in the sessions of
4 Adda247 Publications For any detail, mail us at
Publications@adda247.com
Cracker Book for Bank (IBPS | SBI | RRB PO | Clerk) Mains Exams

the Erfurt parliament but, soon realizing the hopelessness of any


good results from the vacillating policy of Prussia, he retires for
the contest, and, as a major in the service of the Schleswig-
Holstein government, took part in the Danish War of 1850.
(d) Gagern attempted to influence the Prussian Northern Union in the
direction of the national policy and took part in the sessions of the
Erfurt parliament but, soon realizing the hopelessness of any
good results from the vacillating policy of Prussia, he retired from
the contest, and, as a major in the service of the Schleswig-
Holstein government, took part in the Danish War of 1850.
(e) None of the above is correct.

5. It was close enough to catch broadcasts of Phillies baseball and Eagles


football; the city of Brotherly Love; it was far enough away to
be isolated; most of the brutality associated;
(a) While it was close enough to catch broadcasts of Phillies baseball
and Eagles football, it was far enough away to be isolated from
most of the brutality associated with the city of Brotherly Love.
(b) Since it was close enough to catch broadcasts of Phillies baseball
and Eagles football for the city of Brotherly Love, it was far
enough away to be isolated with most of the brutality associated.
(c) As it was close enough to catch broadcasts of Phillies baseball and
Eagles football, it was far enough away to been isolated from most
of the brutality associated with the city of Brotherly Love.
(d) Because it was close enough to catch broadcasts of Phillies
baseball and Eagles football, it was far enough away to
been isolated from most of the brutality associated with the city
of Brotherly Love.
(e) None of the above is correct.

5 Adda247 Publications For any detail, mail us at


Publications@adda247.com
Cracker Book for Bank (IBPS | SBI | RRB PO | Clerk) Mains Exams

Directions (6-10): In each of the following questions, various


sentences are given, and you have to choose the one which has some
or any grammatical error in it.
6. (a) Immediately after boarding the bus Mahesh asked the conductor
that if he knew where the museum was.
(b) Then I knelt down and opened my heart to her and poured out all
the affection that was suffocating me.
(c) She was decidedly pretty, but it is surprising, monsieur, how much
prettier women seem to us when the day is fine at the beginning of
the spring.
(d) One morning on waking I saw from my window the blue sky
glowing in the sun above the neighbouring houses.
(e) My neighbour raised her eyes again, and this time, as I was still
looking at her, she smiled decidedly.

7. (a) No longer able to bear with her anguish she surrendered her life to
the ocean.
(b) She experienced the summer grass beneath her feet and wore
shoes that carried her over rocky terrain.
(c) Since then many a seafarer has told stories of his encounters with
the mermaid.
(d) The success of our efforts depend upon the number of people who
actually use the new methods.
(e) She lost her love to as much greed as any man could acquire, greed
for luxury , greed for temporary pleasure and greed through self
absorbed advancement.

8. (a) By the Time he reaches his ship, time has crept ten fold and he soon
finds himself a crippled old man, unable to board his ship and
escape.
(b) For Time is a treasure we rarely give credit to.
(c) Hardly had he threw the ball when it fell on the ground.
(d) Love no longer had value to him, her time meant little and her
heart was no longer enough to offer him happiness.
(e) And the journey from the cavern back to his ship would prove to be
futile.
6 Adda247 Publications For any detail, mail us at
Publications@adda247.com
Cracker Book for Bank (IBPS | SBI | RRB PO | Clerk) Mains Exams

9. (a) We can earn our gold and silver, we can lose it all and earn it back.
(b) Shyam is working in our organization for the last few years in the
publication department.
(c) In a state of anguish she travelled alone across miles of ocean until
she reached the Island of Alchemy.
(d) The morning starts with a bang or one bang after another, a series,
a chorus - the same sound that lulled me to sleep the night before.
(e) A vital strong man would soon find the treasure he carries become
a little harder to hold, and each step he makes him age.

10. (a) They talked for a while and then they will continue to play the
game till tomorrow morning.
(b) In any case, it would take me very little persuasion to accept a
Chungwah though I often refuse others, and perhaps make secret
enemies.
(c) Anyway, I walk out and out the compound that houses the teachers
and oficials' residences, looking for a place to get some chow.
(d) I walk down the four flights of the hotel building where I and
others of my ilk are housed.
(e) I sit and spend the morning thus, making friends and losing some
and finally decide lunch can wait no longer.

Directions (11-12): In the following question, certain parts of the sentence


are highlighted which may or may not contain grammatical errors. There
are five alternatives given below each sentence, one of which may replace
the existing highlighted parts to make the sentence grammatically correct
and contextually meaningful. Choose the most appropriate alternative as
your answer. If the sentence is grammatically correct, as given, and doesn’t
require any correction, choose option (e) i.e., “No replacement required” as
your answer.

11. The Divine Mother is bounded to respond to the call of her children.
Renunciation of worldly matters may depends on circumstances,
but inward renunciation is an essential part of the process.
(a) is bound to respond, may depend on circumstances
(b) are bound for responding, might depends on circumstances
(c) was bounded to respond, must be dependent in circumstances
(d) were bounded to respond, can depends on circumstance
(e) no replacement required
7 Adda247 Publications For any detail, mail us at
Publications@adda247.com
Cracker Book for Bank (IBPS | SBI | RRB PO | Clerk) Mains Exams

12. The Indian rupee has actually appreciated against the US dollar in
real terms in recent quarters, which is calculated after taking into
account the rise in domestic prices.
(a) have actually got appreciation, that are calculated after
(b) has been appreciated actually, that calculates after
(c) had been actually appreciated, which calculated after
(d) would gain actual appreciation, that has been calculated after
(e) No replacement required

Directions (13-17): In the following passage against each number four


words are suggested in bold which may or may not fit into the sentence
contextually. These numbers are printed below the passage and against
each, five options are given. Find out the most appropriate alternative
reflecting the word which doesn’t fit into the blank appropriately and thus
fail to give a contextual meaning to the paragraph. If no such error is there
mark (e) i.e. “all are correct” as your answer choice.
(13) The decision by the Reserve Bank of India’s Monetary Policy
Committee to raise benchmark interest rates again by 25 basis points is a
reckless one. This is the second successive rate increase in as many
months, a response to mounting uncertainties on the inflation front. (14)
Continuing volatility in crude oil prices, the recent softening regarding,
and its vulnerability to geopolitical tensions and supply disruptions is one
of the main risks to the inflation outlook. (15) Among the RBI’s other
concerns are volatile global financial markets, possibilities of fiscal
stabilization at the Central and State levels, the likely impact of the
increase in the minimum support price for kharif crops, and the staggered
impact of upward revisions to house rent allowance paid by State
governments. (16) Rainfall has so far been 6% below the long-period
average and deficient over a wider area than last year — more than a fifth
of the country’s 36 sub-divisions have reported shortfalls. This has
resulted in a drop in the total sown area under kharif. (17) The monetary
authority has generated the need to keep a close watch on rain over the
remainder of the season, given the risks regional imbalances may pose to
paddy output and CPI inflation. The June round of the RBI’s own survey of
household inflation expectations reveals that families see prices hardening
even further over both the three- and 12-month horizons.

8 Adda247 Publications For any detail, mail us at


Publications@adda247.com
Cracker Book for Bank (IBPS | SBI | RRB PO | Clerk) Mains Exams

13. (a) benchmark (b) reckless (c) mounting


(d) front (e) no error

14. (a) volatility (b) regarding (c) vulnerability


(d) disruptions (e) no error

15. (a) volatile (b) stabilization (c) staggered


(d) revisions (e) no error

16. (a) deficient (b) shortfalls (c) resulted


(d) sown (e) no error

17. (a) generated (b) remainder (c) imbalances


(d) hardening (e) no error

Directions (18-20): In the questions given below few sentences are given
which are grammatically correct and meaningful. Connect them by the
word given above the statements in the best possible way without changing
the intended meaning. Choose your answer accordingly from the options to
form a correct, coherent sentence.

18. AS
(A) Some residents in less affected cities such as Kochi have begun
returning to their homes to assess the damage.
(B) Water levels had fallen in the southern Indian state of Kerala
and rain is predicted to ease in the coming days.
(C) The severest flood warnings were withdrawn from districts
across the state on Sunday.
(D) Rescue operations continue to free thousands of people still
marooned by the worst flooding in a century.
(a) Only B-D
(b) Only C-A
(c) Only A-D
(d) Only B-C
(e) All of these

9 Adda247 Publications For any detail, mail us at


Publications@adda247.com
Cracker Book for Bank (IBPS | SBI | RRB PO | Clerk) Mains Exams

19. NOTWITHSTANDING
(A) The fact is reckless lending would eventually put the banks at a
risk.
(B) Fiscal discipline was compromised and the banking system was
advised to go in for reckless lending.
(C) The result was that global growth picked up.
(D) Most economies were doing well and all emerging economies
started showing a high growth rate.
(a) Only B-A (b) Only C-D (c) Only A-B
(d) Both B-C and A-D (e) None of these

20. HOWEVER
(A) The planning phase of the first manned mission will see ISRO
deploy its Geosynchronous Satellite Launch Vehicle Mark III
(GSLV MK III) with a crew of three into a low earth orbit for five
to seven days.
(B) The conservative prime minister said that India would be only the
fourth country after Russia, the United States and China.
(C) There is no other democracy that is so large and so diverse.
(D) Prime Minister Narendra Modi’s Independence Day
announcement that the country will send a manned mission to
space by 2022 is truly remarkable.
(a) Only B-A (b) Only C-D (c) Only A-B
(d) Both B-C and A-D (e) None of these

Directions (21-23): In the following question a part of the sentence is


given in bold, it is then followed by four sentences which try to explain the
meaning of the idiom/phrase given in bold. Choose the alternative from the
four given below each question which explains the meaning of the phrase
correctly without altering the meaning of the sentence given as question. If
none of the sentence explains the meaning of the highlighted phrase, choose
option (e) i.e., “none of these” as your answer choice.
10 Adda247 Publications For any detail, mail us at
Publications@adda247.com
Cracker Book for Bank (IBPS | SBI | RRB PO | Clerk) Mains Exams

21. In December 1999, like a bolt from the blue came the hijacking of the
Indian airliner from Kathmandu to Kandahar which lasted for a week.
(a) In December 1999, the fatal incident of hijacking of the Indian
airliner from Kathmandu to Kandahar happened which lasted for
a week
(b) In December1999, the formidable episode of hijacking of the
Indian airliner from Kathmandu to Kandahar occurred which
lasted for a week.
(c) In December 1999, a frightening event occurred, of hijacking the
Indian airliner from Kathmandu to Kandahar which lasted for a
week.
(d) In December 1999, a sudden shocking event occurred of hijacking
the Indian airliner from Kathmandu to Kandahar which lasted for
a week.
(e) None of these

22. It's been a long journey for this dyed-in-the-wool entrepreneur who
holds her mother in high regard for being a pillar of support.
(a) It’s been a long journey for this pretentious entrepreneur who
holds her mother in high regard for being a pillar of support.
(b) It’s been a long journey for this conspicuous entrepreneur who
holds her mother in high regard for being a pillar of support.
(c) It’s been a long journey for this inveterate entrepreneur who
holds her mother in high regard for being a pillar of support.
(d) It’s been a long journey for this revolutionary entrepreneur who
holds her mother in high regard for being a pillar of support.
(e) None of these

23. Some pundits are advising European politicians to keep a stiff upper
lip and take on Trump’s challenge to take more responsibility for
their own defense.
(a) Some pundits are advising European politicians to be aghast and
take on Trump’s challenge to take more responsibility for their
own defense.

11 Adda247 Publications For any detail, mail us at


Publications@adda247.com
Cracker Book for Bank (IBPS | SBI | RRB PO | Clerk) Mains Exams

(b) Some pundits are advising European politicians to be valiant and


take on Trump’s challenge to take more responsibility for their
own defense.
(c) Some pundits are advising European politicians to be restrained
and take on Trump’s challenge to take more responsibility for
their own defense.
(d) Some pundits are advising European politicians to remain hushed
and take on Trump’s challenge to take more responsibility for
their own defense.
(e) None of these

Direction (24-26): In each of the given questions an inference is given in


bold which is then followed by three paragraphs. You have to find the
paragraph(s) from where it is inferred. Choose the option with the best
possible outcome as your choice.

24. Education in India remains a vital challenge


[I] Through the ages, education has been a crucial component for
growth and prosperity of nations. It is even more important in
today’s knowledge economy. Yet, the Indian education system
has not been able to fully align itself to this reality. Examination
scores that rate students’ ability to recall information from
memory rather than their comprehension and ability to apply
their knowledge continue to be the most important marker of
performance.

[II] Education at the school level must focus on students’ ability to


understand and apply concepts. Improved teacher training,
curricula reform and more freedom to educators to experiment
are crucial if India is to address the crisis in its education
system. Improved learning at the school level will lead to more
innovation and research in higher education institutions.

12 Adda247 Publications For any detail, mail us at


Publications@adda247.com
Cracker Book for Bank (IBPS | SBI | RRB PO | Clerk) Mains Exams

[III] Three Indian higher education institutes are among the top 250
of the Quacquarelli Symonds (QS) World University Rankings
while another five make it to the list of top 500. Predictably it is
the IITs and the IISc, Bangalore that have made the cut. At the
same time, industry estimates some 94% of engineering school
graduates are unemployable.
(a) only (I)
(b) Both (II) and (III)
(c) only (II)
(d) All (I), (II) and (III)
(e) none of these

25. The ladder of economic mobility has several broken rungs in


countries such as India.
[I] Economic liberals are usually more concerned about equality of
opportunity than equality of outcomes. Inequality is more
acceptable in countries where mobility across generations is
high, so that where you are born does not completely determine
your path in life. A mobile society is far better than one frozen
for the benefit of a plutocracy.

[II] Economic mobility is not just a matter of higher public spending


on education or labour market reforms. Other factors also
matter. For example, underdeveloped capital markets ensure
that the poor are credit-constrained. They are also denied
insurance that could help them smoothen cash flows. The
enterprises of the poor suffer as a result. Along with education,
reforms in factor markets—one of the failures of Indian
economic policy—deserve more public attention in the context
of higher economic mobility.
[III] It is worth pointing out that India continues to have the worst
record on income mobility among the six developing countries.
A recent article by the Plain Facts team of a newspaper showed
that parental education influenced the income of an Indian to a
far greater extent than in the case of comparable countries. The

13 Adda247 Publications For any detail, mail us at


Publications@adda247.com
Cracker Book for Bank (IBPS | SBI | RRB PO | Clerk) Mains Exams

result is that occupational mobility is also very low in India. For


example, only 3% of the sons of farmers end up with white-
collar jobs. Most remain trapped on the farm. On the other hand,
38% of the sons of white-collar workers eventually get similar
jobs.
(a) only (I)
(b) Both (II) and (III)
(c) only (II)
(d) All (I), (II) and (III)
(e) none of these

26. With international crude prices rising and India’s trade deficit
widening, the economy is badly impacted.
[I] Brent crude prices have increased from $70.30 to over $80 per
barrel since the beginning of the new financial year in April. This
is mainly due to concerns over supply disruptions after the rise
in US tensions with Iran, which contributes 11-12% of OPEC
production. As oil prices rise, India’s trade deficit — excess of
imports over exports — will worsen, which can in turn impact
the current account deficit.

[II] Importers will be hit as the cost of getting goods or equipment


into India will increase. When the rupee weakens, importers,
especially oil companies and other import-intensive companies,
have to shell out more rupees to buy an equivalent amount of
dollars. In this sense, a weak rupee can act as a kind of import
tax. For the oil sector, it is a double whammy, as the rise in crude
prices and the decline in rupee value add to retail fuel prices.
Margins of oil companies will come under pressure.
[III] The fiscal and current account deficits are interlinked. When
fiscal deficit is high, government borrowing rises, leading to
higher interest rates. However, when foreign funds start flowing
in, the rupee strengthens and exports become more expensive.
Crude prices are expected to rise further this year, and imports
are expected to grow by at least 14%, says a note from SBI
14 Adda247 Publications For any detail, mail us at
Publications@adda247.com
Cracker Book for Bank (IBPS | SBI | RRB PO | Clerk) Mains Exams

Research. This is bound to enlarge the import bill and push up


the trade deficit, which will in turn add to the CAD and push the
FY19 figure to 2.5% of GDP.
(a) Both (I) and (II)
(b) Both (I) and (III)
(c) only (I)
(d) All (I), (II) and (III)
(e) none of these

27. Which one of the following, if it were to happen, weakens the


conclusion drawn from the passage given below the most?
According to the law, domestic work is not hazardous and allows the
employment of adolescents. Since most adolescent domestic workers
are girls, and as there is lack of public scrutiny, they are especially
vulnerable to sexual abuse and exploitations. Additional hazards
include fatigue caused by long working hours, inadequate rest, denial
of access to health services, education and recreation. All of these can
cause long-term damage to her overall development and future life
prospects.
(a) The new law needs to get more nuanced about what hazardous
professions are, by adopting child-sensitive benchmarks.
(b) Rag-picking and scavenging is another area which is not deemed
hazardous as per the law.
(c) The law protects the children and adolescents (within the age
bracket of 15-18 years) by banning their employment in
hazardous occupations and seeks to regulate their engagement in
the non-hazardous work.
(d) There are enough instances where the employment of children
and adolescents goes unregulated due to existing loopholes in the
law.
(e) None of these
15 Adda247 Publications For any detail, mail us at
Publications@adda247.com
Cracker Book for Bank (IBPS | SBI | RRB PO | Clerk) Mains Exams

28. Which one of the following statements if true, can be said to best
provide supporting evidence for the surmise mentioned in the
passage?
An engineering degree is valuable, but the sense of empathy that
comes from music, arts, literature, and psychology provides a big
advantage in design. A history major who has studied the
Enlightenment or the rise and fall of the Roman Empire gains an
insight into the human elements of technology and the importance of
its usability. A psychologist is more likely to know how to motivate
people and to understand what users want than is an engineer who
has worked only in the technology trenches. A musician or artist is
king in a world in which you can 3D-print anything that you can
imagine.
(a) Tackling today’s biggest social and technological challenges
requires the ability to think critically.
(b) 37% have degrees in engineering or computer technology, and
two percent in mathematics.
(c) Indian parents dread the thought of their children becoming
musicians or artists
(d) To create the amazing future that technology is enabling, India
needs it musicians and artists working hand in hand with its
engineers.
(e) none of these

Directions (29-30): The following questions are based on the passage


given below:
The shortage of faculties in IIMs is rooted in the fact that both existing
faculty members and young PhDs who could have been potential recruits
are now increasingly preferring high-paying corporate jobs over teaching
jobs at the IIMs. As per the ET report, many IIM officials seem to think that
the IIM Act that was implemented this January would help solve the
problem because it empowers the IIMs to recruit freely and decide on
teachers’ salaries; they believe the provision will let IIMs offer competitive
salaries. But with the government planning to regulate IIM fees, and
ironically in the present context, push IIMs to increase intake,

16 Adda247 Publications For any detail, mail us at


Publications@adda247.com
Cracker Book for Bank (IBPS | SBI | RRB PO | Clerk) Mains Exams

_____________________________. To deal with its faculty crunch, the IIMs must


look Westwards—apart from competitive salaries, top US research
universities offer many incentives for research that keeps ace teaching
talent from looking for greener pastures. Even at the public-school level,
teachers are rewarded generously for performance, which in turn, causes
lesser attrition.
29. Select the appropriate statement to fill the above blank in the passage.
(a) where all the 20 IIMs can get top quality faculty.
(b) the hope that the IIM Act will be empowering could be misplaced.
(c) to select their faculty as well as curriculum.
(d) will be subject to overt control by the Ministry of Human
Resource Development.
(e) none of these

30. Which among the following options would most strongly support the
author’s concern about IIMs?
(a) Attracting good students to attract high caliber faculties is a
chicken-egg problem.
(b) High-quality faculty, in turn, insists on and attracts high-quality
staff and students.
(c) IIM officials believe the new IIM Act will offer greater freedom to
B-schools to attract and retain faculty.
(d) because of IIMs increasing seats and classes, IIMs may need a
Lesson in Managing Faculty Crunch
(e) none of these

Solutions

1. (d); Among the given options, sentence (b) is grammatically (due to the
usage of controls) and contextually incorrect. Sentences (a) and (c)
are contextually different and structurally incorrect. They are not
inferring the same meaning as per the demand of the question.
Hence only option (d) forms the correct sentence which follows the
sentences given in the question both grammatically and
contextually.
17 Adda247 Publications For any detail, mail us at
Publications@adda247.com
Cracker Book for Bank (IBPS | SBI | RRB PO | Clerk) Mains Exams

2. (c); Among the given options, sentences (a) and (b) are grammatically
and contextually incorrect.
In (a) mastered should be used but it is not the only error as the
sentence structure is incorrect.
In (b) ‘made’ should be used in place of ‘make’
Sentence (d) is contextually different and structurally incorrect. It
is not inferring the same meaning as per the demand of the
question. Hence only option (c) forms the correct sentence which
follows the sentences given in the question both grammatically and
contextually.

3. (b); Among the given options, sentence (d) is grammatically and


contextually incorrect. Sentences (a) and (c) are contextually
different and structurally incorrect. They are not inferring the same
meaning as per the demand of the question. Hence only option (b)
forms the correct sentence which follows the sentences given in the
question both grammatically and contextually.

4. (d); Among the given options, sentence (c) is grammatically and


contextually incorrect. Sentences (a) and (b) are contextually
different and structurally incorrect. They are not inferring the same
meaning as per the demand of the question. Option (c) is incorrect
as ‘retired’ should be used in place of ‘retires’ Hence only option (d)
forms the correct sentence which follows the sentences given in the
question both grammatically and contextually.

5. (a); Among the given options, sentence (c) is grammatically and


contextually incorrect. Sentences (b) and (d) are contextually
different and structurally incorrect. They are not inferring the same
meaning as per the demand of the question. Hence only option (a)
forms the correct sentence which follows the sentences given in the
question both grammatically and contextually.

18 Adda247 Publications For any detail, mail us at


Publications@adda247.com
Cracker Book for Bank (IBPS | SBI | RRB PO | Clerk) Mains Exams

6. (a); ‘that’ will not be used before ‘if’ as this is an interrogative sentence
of indirect speech in which that is replaced by ‘if/ whether’.
Ex. He asked me if/ whether I was ready.

7. (d); ‘depends’ will be used in place of ‘depend’ as the subject of the


sentence ‘success’ is singular.

8. (c); ‘thrown’ will be used in place of ‘threw’ as ‘have/ has/ had/ having
+ V3’ is used.
Ex. Having taken breakfast, he went out.

9. (b); ‘has been working’ will be used in place of ‘is working’ as the
sentence is in present perfect continuous tense ‘for+ time’.

10. (a); ‘will talk’ will be used in place of ‘talked’ as two actions, that will
happen in future are described here. Ex. He will go to his sister and
then he will go to his friend.

11. (a); The given phrases in bold contains grammatical error and thus
making the sentence incorrect. “is bound to respond, may depend
on circumstances” is the correct set of phrases in context of the
sentence structure and subsequent grammar usage. respectively.
Hence, option (a) is the most suitable answer choice.

12. (e); The given phrases in bold do not require any further corrections or
replacements to make the sentence grammatically correct and
contextually meaningful. Therefore, option (e) is the most suitable
answer choice.

13. (b); The paragraph is describing about the issues undertaken by the
RBI and formulation of policies to benefit the society in whole.
Among the given highlighted words, ‘reckless’ is incorrectly used as
the later part of the sentence has mentioned about the policies to
fight inflation. Thus, the most suitable word in place of reckless
should be “prudent” as it means acting with or showing care and

19 Adda247 Publications For any detail, mail us at


Publications@adda247.com
Cracker Book for Bank (IBPS | SBI | RRB PO | Clerk) Mains Exams

thought for the future. All the other words are completely adhering
to the theme of the paragraph. Hence, option (b) is the most
suitable answer choice.
Benchmark means a standard or point of reference against which
things may be compared.
Mounting means grow larger or more numerous.

14. (b); Among the highlighted words, ‘regarding’ does not fit into the
theme of the paragraph. Also, the sentence mentions about the
“volatility” as well as “recent softening” of crude oil prices. Thus,
the most appropriate that would justify replacing “regarding” is
“notwithstanding” as it means “in spite of”. All the other words are
grammatically as well as contextually correct. Therefore, option (b)
becomes the most suitable answer choice.
Volatility means liability to change rapidly and unpredictably,
especially for the worse.
Vulnerability means the quality or state of being exposed to the
possibility of being attacked or harmed, either physically or
emotionally.
Disruptions means disturbance or problems which interrupt an
event, activity, or process.

15. (b); The inaccurate word among the highlighted words is


“stabilization”. It is to be noted that the sentence of the paragraph
is illustrating about the concerns or issues to be considered by RBI.
Therefore, a word similar meaning of “slippage” should be used in
place of “stabilization” as “Slippage” means ‘the action or process of
slipping or subsiding’. All the other words are grammatically and
contextually correct. Hence, option (b) is the most suitable answer
choice.
Staggered means walk or move unsteadily, as if about to fall.
Revisions means the action of revising.

20 Adda247 Publications For any detail, mail us at


Publications@adda247.com
Cracker Book for Bank (IBPS | SBI | RRB PO | Clerk) Mains Exams

16. (e); The paragraph is describing about the issues undertaken by the
RBI and formulation of policies to benefit the society in whole. All
the given words are correct with reference to the theme of the
paragraph. Hence, option (e) becomes the most suitable answer
choice.
Deficient means not having enough of a specified quality or
ingredient.
Sown means plant (seed) by scattering it on or in the earth.

17. (a); The previous sentence has mentioned about the shortfalls of rain in
many parts of the country. Due to shortfalls, the authority has
marked to pay attention towards the rain for the remaining season.
Thus, to make the next sentence coherent to the paragraph, replace
the word “generated” by “flagged” or a word similar in meaning.
“flagged” means mark (an item) for attention or treatment in a
specified way. However, all the other words are precisely in context
of the paragraph. Hence, option (a) is the most suitable answer
choice.
Imbalances means lack of proportion or relation between
corresponding things.
Hardening means make or become more severe and less
sympathetic.

18. (e); All the given combinations of statements successfully form


grammatically correct and contextually meaningful sentences using
the conjunction “As”. “As” as a conjunction is used to indicate that
something happens during the time when something else is taking
place. Hence, option (e) is the most suitable answer choice.
Statement B-D: Water levels had fallen in the southern Indian
state of Kerala and rain is predicted to ease in the coming days as
rescue operations continue to free thousands of people still
marooned by the worst flooding in a century.
Statement C-A: As the severest flood warnings were withdrawn
from districts across the state on Sunday, some residents in less

21 Adda247 Publications For any detail, mail us at


Publications@adda247.com
Cracker Book for Bank (IBPS | SBI | RRB PO | Clerk) Mains Exams

affected cities such as Kochi have begun returning to their homes


to assess the damage.
Statement A-D: As some residents in less affected cities such as
Kochi have begun returning to their homes to assess the damage,
rescue operations continue to free thousands of people still
marooned by the worst flooding in a century.
Statement B-C: As water levels had fallen in the southern Indian
state of Kerala and rain is predicted to ease in the coming days, the
severest flood warnings were withdrawn from districts across the
state on Sunday.

19. (a); Statements (B) and (A) can be combined using the conjunction
“notwithstanding” to form a grammatically correct and
contextually meaningful sentence. “Notwithstanding” means
although; in spite of the fact that. Therefore the coherent sentence
thus formed is “Fiscal discipline was compromised and the
banking system was advised to go in for reckless lending,
notwithstanding the fact that it would eventually put the
banks at a risk”. Hence, option (a) is the most suitable answer
choice.

20. (e); None of the given combinations of the statements aptly connect
with each other using the connector “however” to form a coherent
sentence. ‘However’ is used to introduce a statement that contrasts
with or seems to contradict something that has been said
previously. Hence, the most suitable answer choice is option (e).

21. (d); The most appropriate option that reflects the meaning of the given
sentence is option (d) as the idiom “a bolt from the blue” means
something important or unusual that happens suddenly or
unexpectedly. However, the words “fatal”, “frightening” and
“formidable” do not indicate the suddenness of the event. Hence,
option (d) is the most suitable answer choice.

22 Adda247 Publications For any detail, mail us at


Publications@adda247.com
Cracker Book for Bank (IBPS | SBI | RRB PO | Clerk) Mains Exams

22. (c); The most appropriate option that reflects the meaning of the given
sentence is option (c) as the idiom “dyed-in-the-wool” means
unchanging in a particular belief or opinion; inveterate. However,
the words “pretentious”, “conspicuous” and “revolutionary” do not
express the similar meaning. Hence, option (c) is the most suitable
answer choice.
Pretentious means attempting to impress by affecting greater
importance or merit than is actually possessed.
Conspicuous means attracting notice or attention.
Inveterate means having a particular habit, activity, or interest that
is long-established and unlikely to change.
Revolutionary means involving or causing a complete or dramatic
change.

23. (b); The most appropriate option that reflects the meaning of the given
sentence is option(b) as the idiom “keep a stiff upper lip” means
to be courageous or stoic in the face of adversity. However, the
words “aghast”, “restrained” and “hushed” do not express the
similar meaning. Hence, option (b) is the most suitable answer
choice.
Aghast means filled with horror or shock.
Valiant means possessing or showing courage or determination.
Restrained means characterized by reserve or moderation;
unemotional or dispassionate.
Hushed means (of a voice or conversation) quiet and serious.

24. (a); Among the given paragraphs, only paragraph (I) provides the
conclusion as given in the question. Paragraph (I) has mentioned
about the challenges that India is facing in its education system.
This can be articulated form the sentence “Examination scores that
rate students’ ability to recall information from memory rather
than their comprehension and ability to apply their knowledge
continue to be the most important marker of performance”.
However, paragraph (II) is merely providing the solutions to
improve the education system of the country. While paragraph (III)

23 Adda247 Publications For any detail, mail us at


Publications@adda247.com
Cracker Book for Bank (IBPS | SBI | RRB PO | Clerk) Mains Exams

has provided data regarding the position of Indian institutes on


global basis. Thus, the given inference can be drawn from the first
paragraph only, therefore, option (a) becomes the most suitable
answer choice.

25. (b); Read the paragraphs carefully. It is to be noted that the inference
means the economic mobility (ability of an individual, family or
some other group to improve (or lower) their economic status—
usually measured in income) of India has several hurdles while
moving towards it. Both the paragraphs (II) and (III) are providing
the given inference. Paragraph (II) has mentioned those hurdles in
the sentence “Along with education…economic mobility”. This
sentence helps to conclude the given inference. Moreover, the
inference from paragraph (III) can be drawn from the sentence
“…parental education influenced the income of an Indian to a far
greater extent than in the case of comparable countries”. However,
paragraph (I) is merely providing additional information on
economic mobility. Thus, option (b) is the most feasible answer
choice.

26. (d); All the given paragraphs successfully infer the given inference. The
conclusion can be deduced from the first paragraph of the sentence
“As oil prices rise, India’s trade deficit — excess of imports over
exports — will worsen, which can in turn impact the current
account deficit.” Moreover, paragraph (II) also mentions about the
increase in crude prices which affects economy of the country. This
can be understood from the sentence “For the oil sector, it is a
double whammy, as the rise in crude prices and the decline in
rupee value add to retail fuel prices. Margins of oil companies will
come under pressure.” while, paragraph (III) mentions “Crude
prices are expected to rise further this year, and imports are
expected to grow by at least 14%, says a note from SBI Research.
This is bound to enlarge the import bill and push up the trade
deficit, which will in turn add to the CAD and push the FY19 figure
to 2.5% of GDP.” Since, all three paragraphs infer the given
conclusion, option (d) becomes the most suitable answer choice.

24 Adda247 Publications For any detail, mail us at


Publications@adda247.com
Cracker Book for Bank (IBPS | SBI | RRB PO | Clerk) Mains Exams

27. (c); The paragraph has mentioned about the amendments made in the
labour laws. However, these amendments are not enough to
protect the children and adolescents, since it has a very narrow
scope of hazardous work. Only sentence (c) weakens the
conclusion drawn from the passage, as it has mentioned about the
assured protection provided by the law to the children and
adolescents. All the other sentences are mere additional
information, supporting the idea of the paragraph. Hence, option
(c) is the most suitable answer choice.

28. (d); The given paragraph has described about the importance of art and
humanities along with engineering to build a new innovated era.
Statement (d) completely supports the theme of the paragraph,
while all the other sentences are irrelevant. Hence, option (d) is the
most suitable answer choice.

29. (b); The paragraph has mentioned about the imbalance in the ratio of
faculty and students in IIMs. Moreover, the IIM act has decided to
regulate the fees, so as to increase the intake of the students. Thus,
the most suitable sentence that will fill the blank is “the hope that
the IIM Act will be empowering could be misplaced”, since this
amendment in the act will further disturb the proportionality
between the teachers and the students. All the other given
statements are irrelevant and do not form a coherent paragraph.
Hence, option (b) is the most suitable answer choice.

30. (c); The author has mentioned his concern about the shortage of faculty
members for the students which will soon rise due to the
amendment in IIM bill [But with the government planning to
regulate IIM fees, and ironically in the present context, push IIMs to
increase intake]. However, option (c) mentions about the retention
of the faculties. Therefore, option (c) is the most suitable answer
choice.

25 Adda247 Publications For any detail, mail us at


Publications@adda247.com

You might also like